Download as pdf or txt
Download as pdf or txt
You are on page 1of 994

GATE-2023

Electronics Engineering

Network Theory
Chapterwise & Topicwise

Contents
S.No. Topic Page No.

1. Basics of Network Analysis .............................................................................................................. 1-41

2. Sinusoidal Steady State .................................................................................................................. 42-68

3. Network Theorems ....................................................................................................................... 69-104

4. Transient Analysis ........................................................................................................................ 105-139

5. Two Port Networks ..................................................................................................................... 140-161

6. Network Functions ..................................................................................................................... 162-169


1 Basics of Network Analysis

ELECTRO NICS EN GINEERIN G 1 2H

(GATE Previous Years Solved Papers)

Q.1 A square waveform as shown in figure is


N4
applied across 1 mH ideal inductor. The current 1F
through the inductor is a ______ wave of ______
peak amplitude. (a) N1 and N2 (b) N2 and N4
V (c) N1 and N3 (d) N1 and N4
[EC-1992 : 2 Marks]
1
Q.3 A network contains linear resistors and ideal
voltage sources. If values of all the resistors are
0 0.5 1 t(m-sec) doubled, then the voltage across each resistor is
(a) halved
–1
(b) doubled
[EC-1987 : 2 Marks] (c) increased by four times
(d) not changed
Q.2 Of the networks, N1, N2, N3 and N4 of figure, the
[EC-1993 : 2 Marks]
networks having identical driving point
function are Q.4 The two electrical sub-networks N1 and N2 are
connected through three resistors as shown in
2H 1
figure. The voltage across 5 resistor and 1
resistor are given to be 10 V and 5 V, respectively.
1 1F
Then voltage across 15 resistor is
N1
1F + 10 V –
5

15
2H 1 N1 N2
1

+ 5V –
N2
2 1F (a) –105 V (b) +105 V
(c) –15 V (d) +15 V
[EC-1993 : 2 Marks]
1
1 Q.5 A dc circuit shown in figure has a voltage source
V, a current source I and several resistors. A
1H particular resistor R dissipates a power of
4 Watts when V alone is active. The same resistor
N3
1F R dissipates a power of 9 Watts when I alone is
active. The power dissipated by R when both
sources are active will be
2 Electronics Engineering Network Theory

Q.9 The voltage V in figure is always equal to


+
Resistive 2A
V R 2
network

+
I
V 5V


(a) 1 W (b) 5 W
(c) 13 W (d) 25 W
(a) 9 V (b) 5 V
[EC-1993 : 1 Mark]
(c) 1 V (d) None of these
Q.6 Two 2 H inductance coils are connected in series [EC-1997 : 1 Mark]
and are also magnetically coupled to each other
the coefficient of coupling being 0.1. The total Q.10 The voltage V in figure is
inductance of the combination can be 3
(a) 0.4 H (b) 3.2 H +
(c) 4.0 H (d) 4.4 H
V 10 V 5V
[EC-1995 : 1 Mark]

Q.7 The current i4 in the circuit of figure is equal to –

(a) 10 V (b) 15 V
(c) 5 V (d) None of these
[EC-1997 : 1 Mark]

Q.11 In the circuit of figure the equivalent impedance


seen across terminals A, B is

2 4
j3
(a) 12 A (b) –12 A
(c) 4 A (d) None of these
Zeq
[EC-1997 : 1 Mark] –j 4
2 4
Q.8 The voltage V in figure is equal to
B

4V
16 8
(a) (b)
3 3
5V 2 4V
8
(c) + 12 j (d) None of the above
3
+ V – [EC-1997 : 2 Marks]

(a) 3 V (b) –3 V Q12 In the circuit shown in the figure the current iD
(c) 5 V (d) None of these through the ideal diode (zero cut in voltage and
[EC-1997 : 1 Mark] zero forward resistance) equals
GATE Previous Years Solved Paper 3

iD Q.16 In the circuit of the figure, the voltage v(t) is


4
1 1 1

+
10 V 4 1 2A at bt
e V(t) 1H e

(a) 0 A (b) 4 A
(a) eat – ebt (b) eat + ebt
(c) 1 A (d) None of these
(c) aeat – bebt (d) aeat + bebt
[EC-1997 : 3 Marks]
[EC-2000 : 1 Mark]
Q.13 The nodal method of circuit analysis is based
Q.17 In the circuit of the figure, the value of the voltage
on
source E is
(a) KVL and Ohm’s law
V2 0V
(b) KCL and Ohm’s law
+ +
(c) KCL and KVL
1V – – 1V
(d) KCL, KVL and Ohm’s law
[EC-1998 : 1 Mark] +
E=?

Q.14 The voltage across the terminals ‘a’ and ‘b’ in
figure is 4V +
+ 5V
2 1 – –
a
V1 10 V

+ (a) –16 V (b) 4 V


1V 2 3A (c) –6 V (d) 16 V

[EC-2000 : 1 Mark]

b Q.18 For the circuit in the figure, the voltage Vo is


(a) 0.5 V (b) 3.0 V 2 2
(c) 3.5 V (d) 4.0 V
[EC-1998 : 1 Mark]
+ –
4V 2V
Q.15 A Delta-connected network with its Wye- – – +
2 Vo
equivalent is shown in figure. The resistance +
R1, R2 and R3 (in ) are respectively
a (a) 2 V (b) 1 V
a (c) –1 V (d) None of these
R1 [EC-2000 : 2 Marks]
5 30
Q.19 The voltage eo in the figure is
R2 R3
b c 4 2
15
b c
+
(a) 1, 5, 3 and 9 (b) 3, 9 and 1.5 12 V 4 eo 2
(c) 9, 3 and 1.5 (d) 3, 1.5 and 9 –
[EC-1999 : 2 Marks]
4 Electronics Engineering Network Theory

4 ZL ZL
(a) 2 V (b) V
3 3-phase
400 V ZL
(c) 4 V (d) 8 V balanced
source
[EC-2000 : 1 Mark]

Q.20 If each branch of a delta circuit has impedance

3 Z , then each branch of the equivalent Wye (a) 90 32.44° (b) 80 32.44°
(c) 80 –32.44° (d) 90 –32.44°
circuit has impedance.
[EC-2002 : 2 Marks]
Z
(a) (b) 3Z
3 Q.24 The minimum number of equations required to
analyze the circuit shown in the figure is
Z
(c) 3 3 Z (d) C C
3
[EC-2000 : 1 Mark]

Q.21 The voltage eo in the figure is R R

V R C R
2

16 V +
(a) 3 (b) 4
8A 10 12 eo
(c) 6 (d) 7
6 – [EC-2003 : 1 Mark]

Q.25 Twelve 1 resistances are used as edges to form


(a) 48 V (b) 24 V a cube. The resistance between two diagonally
opposite corners of the cube is
(c) 36 V (d) 28 V
[EC-2001 : 2 Marks] 5
(a) (b) 1
6
Q.22 The dependent current source shown in the
6 3
figure, (c) (d)
5 2
5 [EC-2003 : 2 Marks]

+ Q.26 The current flowing through the resistance R in


V1
V1 = 20 V I 5 A the circuit in the figure has the form P cos4t,
5
– where P is
M = 0.75 H
(a) delivers 80 W 1/10.24 F

(b) absorbs 80 W
R = 3.92
(c) delivers 40 W
3
(d) absorbs 40 W
[EC-2002 : 1 Mark] V = 2 cos4t
20 V

Q.23 If the three-phase balanced source in the figure


(a) (0.18 + j0.72) (b) (0.46 + j1.90)
delivers 1500 W at a leading power factor 0.844,
then the value of ZL (in ohm) is approximately (c) –(0.18 + j1.90) (d) –(0.192 + j0.144)
[EC-2003 : 2 Marks]
GATE Previous Years Solved Paper 5

Q.27 An ideal sawtooth voltage waveform of Q.30 Impedance Z as shown in the given figure is
frequency 500 Hz and amplitude 3 V is
j5 j2
generated by charging a capacitor of 2 µF in
every cycle.
The charging requires j10
j2
(a) constant voltage source of 3 V for 1 ms. j10
(b) constant voltage source of 3 V for 2 ms.
(c) constant current source of 3 mA for 1 ms.
(d) constant current source of 3 mA for 2 ms. Z

[EC-2003 : 2 Marks] (a) j29 (b) j9

Q.28 The equivalent inductance measured between (c) j19 (d) j39
the terminals 1 and 2 for the circuit shown in [EC-2005 : 2 Marks]
the figure is Q.31 If R1 = R2 = R4 = R and R3 = 1.1 R in the bridge
M circuit shown in the figure, then the reading in
1
the ideal voltmeter connected between
L1 L2 ‘a’ and ‘b’ is

R1 R4
2
+
(a) L1 + L2 + M (b) L1 + L2 – M a b
10 V –
+
(c) L1 + L2 + 2M (d) L1 + L2 – 2M –
[EC-2004 : 1 Mark] R2 R3

Q.29 For the circuit in the figure, the initial conditions

V (s)
are zero. Its transfer function H (s ) = c is,
Vi (s ) (a) 0.238 V (b) 0.138 V

10 mH
(c) –0.238 V (d) 1 V
10 k
[EC-2005 : 2 Marks]

vi(t ) 100 µF vc(t) Q.32 In the interconnection of ideal sources shown


in the figure, it is known that the 60 V source is
absorbing power.
1 20 V
(a)
s + 10 s + 106
2 6

106
(b)
s 2 + 10 3 s + 106 I 60 V

10 3
(c)
s 2 + 10 3 s + 106
12 A
106
(d) Which of the following can be the value of the
s 2 + 10 6 s + 106
current source I ?
[EC-2004 : 2 Marks]
6 Electronics Engineering Network Theory

(a) 10 A (b) 13 A
(c) 15 A (d) 18 A I
j4 –j 4
[EC-2009 : 1 Mark] + 6
14 0° A
Q.33 A fully charged mobile phone with a 12 V battery –
is good for a 10 minute talk-time. Assume that, 6 6
during the talk-time the battery delivers a
constant current of 2 A and its voltage drops
(a) 1.4 0° A (b) 2.0 0° A
linearly from 12 V to 10 V as shown in the figure.
(c) 2.8 0° A (d) 3.2 0° A
How much energy does the battery deliver
[EC-2011 : 2 Marks]
during this talk-time?
Q.36 In the circuit shown below, the current through
v (t ) the inductor is
12 V
10 V
1 j1

1 0A
t
0 10 min 1 0V 1 0V

– + + –
(a) 220 J (b) 12 kJ
(c) 13.2 kJ (d) 14.4 J

1 0A
[EC-2009 : 1 Mark] –j1 1

Q.34 In the circuit shown, the power supplied by the


voltage source is
2 1
(a) A (b) A
1 1+ j 1+ j
1
(c) A (d) 0 A
1 1 1+ j
[EC-2012 : 1 Mark]
10 V
Q.37 The average power delivered to an impedance
1A
(4 – j3) by a current 5 cos(100 t + 100) A is
1 2A
(a) 44.2 W (b) 50 W
(c) 62.5 W (d) 125 W
1
[EC-2012 : 1 Mark]

(a) 0 W (b) 5 W Q.38 If VA – VB = 6 V, then VC – VD is


(c) 10 W (d) 100 W R VA 2 VB R
[EC-2010 : 2 Marks]

Q.35 In the circuit shown below, the current I is R
R
R R 10 V
1 R
equal to +

+ –
VC VD R
5V 2A
GATE Previous Years Solved Paper 7

(a) –5 V (b) 2 V Q.41 Three capacitors C1, C2 and C3 whose values


(c) 3 V (d) 6 V are 10 µF, 5 µF and 2 µF respectively, have
[EC-2012 : 2 Marks] breakdown voltages of 10 V, 5 V and 2 V
respectively. For the interconnection shown
Q.39 Consider a delta-connection of resistors and its below, the maximum safe voltage in volts that
equivalent star-connection as shown below. If can be applied across the combination, and the
all elements of the delta-connection are scaled corresponding total charge in µC stored in the
by a factor k, k > 0, the elements of the effective capacitance across the terminals are,
corresponding star equivalent will be scaled by respectively
a factor of
C2 C3
Ra

Rb Rc

C1

RC RB (a) 2.8 and 36 (b) 7 and 119


(c) 2.8 and 32 (d) 7 and 80
[EC-2013 : 2 Marks]
RA
Common Data for Questions (42 and 43):
Consider the following figure:
(a) k 2 (b) k
(c) 1/k (d) k
5
[EC-2013 : 1 Mark] Is

10 V 2 1
Q.40 The following arrangement consists of an ideal +
transformer and an attenuator which attenuates Vs 2A
by a factor of 0.8. An ac voltage VWX1 = 100 V is –
applied across WX to get an open-circuit voltage
VYZ across YZ. Next, an ac voltage VYZ2 = 100 V
1 Q.42 The current Is in amperes in the voltage source,
is applied across YZ to get an open-circuit
voltage VWX2 across WX. Then, VYZ1/VWX1, and voltage Vs in volts across the current source
VWX2/VYZ2 are respectively, respectively, are
(a) 13, –20 (b) 8, –10
W
(c) –8, 20 (d) –13, 20
1 : 1.25
[EC-2013 : 2 Marks]
Y
Q.43 The current in the 1 resistor in amperes is
(a) 2 (b) 3.33
X Z
(c) 10 (d) 12
125 80 100 80 [EC-2013 : 2 Marks]
(a) and (b) and
100 100 100 100
Q.44 Consider the configuration shown in the figure
100 100 80 80
(c) and (b) and which is a portion of a larger electrical network.
100 100 100 100
[EC-2013 : 2 Marks]
8 Electronics Engineering Network Theory

i5
(a) 22 + j2 V (b) 2 + j22 V
i2 (c) 22 – j2 V (d) 2 – j22 V
R R [EC-2014 : 2 Marks]
i3
Q.48 The circuit shown in the figure represents a
i4
i1 R i6

For R = 1 and currents i1 = 2 A, i4 = –1 A,


Ii AIIi
i5 = –4 A, which one of the following is true? R
(a) i6 = 5 A
(b) i3 = –4 A
(c) data is sufficient to conclude that the
supposed currents are impossible (a) voltage controlled voltage source

(d) data is insufficient to identify the currents (b) voltage controlled current source
i2, i3 and i6 (c) current controlled current source
[EC-2014 : 1 Mark] (d) current controlled voltage source
[EC-2014 : 1 Mark]
Q.45 A Y-network has resistance of 10 each in two
of its arms, while the third arms has a resistance Q.49 The magnitude of current (in mA) through the
of 11 in the equivalent -network, the lowest resistor R2 in the figure shown is _____ .
value (in ) among the three resistances is __ .
R2
[EC-2014 : 2 Marks]
1k
Q.46 For the Y-network shown in the figure, the value
of R1 (in ) in the equivalent -network is ___ . 10 mA R1 2k R3 4k 2 mA
R1
R4
5
3

3k

[EC-2014 : 1 Mark]

Q.50 The equivalent resistance in the infinite ladder


7.5

network shown in the figure, is Re .

2R R R R
[EC-2014 : 2 Marks]

Q.47 In the circuit shown in the figure, the value of


Re R R R R .....
node voltage V2 is
10 0°

The value Re/R is ______ .


V1 4 V2
[EC-2014 : 2 Marks]

Q.51 In the network shown in the figure, all resistors


4 0° A –j 3 6 j6
are identical with R = 300 . The resistance Rab
(in ) of the network is _____ .
GATE Previous Years Solved Paper 9

a R3
e h

R R R R R
R R2 R2 R3

R3
R R R g
f R2
Rab R c
d
R R R R R R1
R2

R1 R2
b R2

[EC-2015 : 1 Mark]
a b
R1
Irms
Q.52 In the given circuit, the values of V1 and V2
respectively are V = 10 sin(t)

4 Rms current Irms (in mA) through the diode is


______ .
I
+
[EC-2016 : 2 Marks]
+
V2 5A 4 4 2I V1 Q.55 In the given circuit, each resistor has a value
– –
equal to 1 .

(a) 5 V, 25 V (b) 10 V, 30 V
(c) 15 V, 35 V (d) 0 V, 20 V
[EC-2015 : 1 Mark]

Q.53 In the circuit shown, the voltage Vx (in Volts) is


_______ .

0.5Vx
10 b

What is the equivalent resistance across the


+
5A Vx 20 8 + 0.25Vx terminals ‘a’ and ‘b’?

– 1 1
(a) (b)
6 3
[EC-2015 : 1 Mark] 9 8
(c) (d)
20 15
Q.54 An AC voltage source V = 10 sin(t) Volts is
[EC-2016 : 2 Marks]
applied to the following network. Assume that,
R1 = 3 k , R2 = 6 k and R3 = 9 k , and that the Q.56 In the circuit shown in the figure, the magnitude
diode is ideal. of the current (in Amperes) through R2 is ____ .
10 Electronics Engineering Network Theory

R1 R2
R1 R1
5 3
R3 +
60 V 0.04Vx 5 Vx
– R2

R1 R1 11 V
[EC-2016 : 2 Marks] R3 R3

Q.57 In the figure shown, the current ‘i’ (in Amperes) R2


is ______ .

5 R1 R1

1A
1 8V The magnitude of the current (in amperes,
accurate to two decimal places) through the
1 1
source is ______ .

8V
[EC-2018 : 2 Marks]
i
Q.60 Consider the circuit shown in the figure.
1

3
5A
[EC-2016 : 2 Marks] 2
7 I
Q.58 A connection is made consisting of resistance A P Q

in series with a parallel combination of


resistances B and C. Three resistors of value 6 2
10 , 5 , 2 are provided. Consider all
possible permutations of the given resistors into
The current ‘I’ flowing through the 7 resistor
the positions A, B, C and identify the
between P and Q (Rounded off to 1 decimal place)
configurations with maximum possible overall
is _______ A.
resistance. The ratio of maximum to minimum
values of the resistances (up to two decimal [EC-2021 : 1 Mark]
place) is _____ . Q.61 Consider the circuit shown in the figure.
[EC-2017 : 1 Mark]
6 mA
Q.59 Consider the network shown below with
R1 = 1 , R2 = 2 and R3 = 3 . The network is
connected to a constant voltage source of 11 V.
1k
4V
+
1k vo 2 mA

1k
GATE Previous Years Solved Paper 11

The value of Vo (Rounded off to one decimal 1 2


(a) A (b) A
place) is _______ Volt. 15 15
[EC-2021 : 1 Mark] 4 8
(c) A (d) A
15 15
Q.62 The current ‘I’ in the circuit shown is _____ .
[EE-1992 : 1 Mark]
I 2k 2k
Q.2 All resistance in the circuit in figure are of (R )
each. The switch is initially open. What
5V
–3 happens to the lamp’s intensity when the switch
2k 10 A
is closed?
200 V
Lamp
+ –

(a) 1.25 × 10–3 A (b) 0.75 × 10–3 A


(c) –0.5 × 10–3 A (d) 1.16 × 10–3 A
[EC-2022]

Q.63 Consider the circuit shown in the figure. The


current ‘I’ flowing through the 10 resistor is
________ .
(a) Increases
1 I 10 2
(b) Decreases
(c) Remains same
+ + (d) Answer depends on the value of R
3V 2 1 3V
– – [EE-1992 : 1 Mark]

Q.3 In the circuit shown in figure, X is an element


which always absorbs power. During a
particular operation, it sets up a current of
(a) 1 A (b) 0 A 1 ampere in the possible that X can be absorb
(c) 0.1 A (d) –0.1 A the same power Px for another current i. Then
[EC-2022] the value of this current is

ELECTRICAL EN GINEERIN G X

(GATE Previous Years Solved Papers) i

Q.1 All resistance in figure are 1 each. The value


– +
of current ‘I’ is 1
6V

(a) (3 14) A (b) (3 + 14 ) A


(c) 5 A (d) None of these
I
[EE-1996 : 1 Mark]
+ Q.4 A practical current source is usually represented
1V
– by
(a) a resistance in series with an ideal current
source.
12 Electronics Engineering Network Theory

(b) a resistance in parallel with an ideal current Cs Cc


source. (a) Cc + (b) Cc +
2 2
(c) a resistance in parallel with an ideal voltage
source. (Cs + 3Cc )
(c) (d) 3 Cc + 2 Cs
(d) none of these [EE-1997 : 1 Mark] 2
[EE-1999 : 1 Mark]
Q.5 A 10 Volt battery with an internal resistance of
1 is connected across a non-linear load whose Q.9 When a resistor R is connected to a current
V-I characteristic is given by 7I = V 2 + 2V. The source, it consumes a power of 18 W. When the
current delivered by the battery is _______ A. same R is connected to a voltage source having
[EE-1997 : 1 Mark] the same magnitude as the current source, the
power absorbed by R is 4.5 W. The magnitude
Q.6 The value of E and I for the circuit shown in of the current source and the value of R are
figure, are _______ V and _______ A.
(a) 18 A and 1 (b) 3 A and 2
I 1 1
(c) 1 A and 18 (d) 6 A and 0.5
2A
[EE-1999 : 2 Marks]
E 6 4 1
Q.10 When a periodic triangular voltage of peak
amplitude 1 V and frequency 0.5 Hz is applied
to a parallel combination of 1 resistor and 1 F
capacitance, the current through the voltage
[EE-1997 : 2 Marks]
source has waveform.
Q.7 The voltage and current waveforms for an
element are shown in figure. The circuit element (a)
is ______ and its value is _______ .

i(t) V(t) (b)

2A 2V

(c)
t t
0 2s 0 2s

[EE-1997 : 2 Marks]
(d)
Q.8 For the circuit shown in figure, the capacitance
measured between terminals B and Y will be
[EE-1999 : 2 Marks]
R
Q.11 The circuit shown in the figure is equivalent to
Cc
a load of
Cc

I 2
B Y
Cc
Cs Cs
4 + 2I

GATE Previous Years Solved Paper 13

4 8 (a) 3 – 8 cos2t (b) 32 sin2t


(a) (b) (c) 16 sin2t (d) 16 cos2t
3 3
(c) 4 (d) 2 [EE-2003 : 1 Mark]
[EE-2000 : 2 Marks] Q.15 Figure shows the waveform of the current
passing through an inductor of resistance 1
Q.12 Two incandescent light bulbs of 40 W and 60 W
and inductance 2 H. The energy absorbed by
ratings are connected in series across the mains.
the inductor in the first four second is
Then,
(a) the bulbs together consume 100 W.
(b) the bulbs together consume 50 W. 6A

(c) the 60 W bulb glows brighter.


(d) the 40 W bulb glows brighter.
t
[EE-2001 : 1 Mark] 0 2s 4s

Q.13 Consider the star network shown in figure. The (a) 144 J (b) 98 J
resistance between terminals A and B with (c) 132 J (d) 168 J
terminal C open is 6 , between terminal B and [EE-2003 : 1 Mark]
C with terminal A open is 11 , and between
Q.16 In figure, the potential difference between points
terminals C and A with terminal B open is 9 .
P and Q is
Then,
A
2A
RA
2 R 4
P Q
RB RC +
10 V

B

C 8 6
(a) RA = 4 , RB = 2 , RC = 5
(b) RA = 2 , RB = 4 , RC = 7 (a) 12 V (b) 10 V

(c) RA = 3 , RB = 3 , RC = 4 (c) –6 V (d) 8 V


(d) RA = 5 , RB = 1 , RC = 10 [EE-2003 : 2 Marks]
[EE-2001 : 2 Marks] Q.17 In figure, the value of R is
Q.14 A segment of a circuit shown in figure VR = 5 V, R
VC = 4 sin2t. The voltage VL is given by
Q 14 1
1A
+ 10 A 5A
5 VR
– 2
1F
P R 100 V 40 V
+ –
Vc
2A +
2H VL
(a) 10 (b) 18

S (c) 24 (d) 12
[EE-2003 : 2 Marks]
14 Electronics Engineering Network Theory

Q.18 In figure, the value of the source voltage is Q.22 In the figure given below the value of R is
10 Va 1 2A 8A R
P

100 V 10 10
1A 6 E

Q
(a) 2.5 (b) 5.0
(a) 12 V (b) 24 V
(c) 7.5 (d) 10.0
(c) 30 V (d) 44 V
[EE-2005 : 1 Mark]
[EE-2004 : 2 Marks]
Q.23 A 3 V d.c. supply with an internal resistance of
Q.19 In figure, the value of resistance R in is 2 supplies a passive non-linear resistance
10 2A 2
characterized by the relation VNL = I NL . The
power dissipated in the non-linear resistance is
100 V 10 R (a) 1.0 W (b) 1.5 W
(c) 2.5 W (d) 3.0 W
[EE-2007 : 2 Marks]
(a) 10 (b) 20
Q.24 Assuming ideal elements in the circuit shown
(c) 30 (d) 40 below, the voltage Vab will be
[EE-2004 : 2 Marks]
a R
Q.20 In figure, Ra, Rb and Rc are 20 , 10 and 10 +
respectively. The resistance R1, R2 and R3 in
of an equivalent star-connection are 1A Vab i 5V

a –
b
a R1
(a) –3 V (b) 0 V
Rb Rc
R3 R2
(c) 3 V (d) 5 V
[EE-2008 : 2 Marks]
Ra c
c c b Q.25 In the circuit shown in the figure, the value of
(a) 2.5, 5, 5 (b) 5, 2.5, 5 the current i will be given by
(c) 5, 5, 2.5 (d) 2.5, 5, 2.5 1 a b 3
+ Vab –
[EE-2004 : 2 Marks]

Q.21 The rms value of the current in the wire which 5V 1 1 i + 4V


– ab

carries a dc current of 10 A and a sinusoidal


alternating current of peak value of 20 A is
(a) 10 A (b) 14.14 A (a) 0.31 A (b) 1.25 A
(c) 15 A (d) 17.32 A (c) 1.75 A (d) 2.5 A
[EE-2004 : 2 Marks] [EE-2008 : 2 Marks]
GATE Previous Years Solved Paper 15

Q.26 The current through the 2 k resistance in the (a) (5 A, Put Vs = 20 V)


circuit shown is (b) (2 A, Put Vs = 8 V)
1k C 1k (c) (5 A, Put Is = 10 V)
(d) (7 A, Put Is = 12 V)
A B [EE-2009 : 2 Marks]
2k
Q.30 As shown in the figure, a 1 resistance is
connected across a source that has a load line
1k D 1k v + i = 100. The current through the resistance is
i
+
6V
Source V 1
(a) 0 mA (b) 1 mA
(c) 2 mA (d) 6 mA –

[EE-2009 : 1 Mark] (a) 25 A (b) 50 A

Q.27 How many 200 W/220 V incandescent lamps (c) 100 A (d) 200 A
connected in series would consume the same [EE-2010 : 1 Mark]
total power as a single 100 W/220 V
Q.31 If the electrical circuit of Fig. (b) is an equivalent
incandescent lamp?
of the coupled tank system of Fig. (a), then
(a) non possible (b) 4
(c) 3 (d) 2
[EE-2009 : 1 Mark]

Q.28 The equivalent capacitance of the input loop of


the circuit shown is
i1 1k 1k
h1
h2
1k 49 i1
Input 100 µF
loop

(a) Coupled tank


100 µF

(a) 2 µF (b) 100 µF B D

(c) 200 µF (d) 4 µF


[EE-2009 : 2 Marks]
A C
Q.29 For the circuit shown, find out the current
flowing through the 2 resistance. Also identify
the changes to be made of double the current
through the 2 resistance. ( b) Electrical equivalent

(a) A, B are resistance and C, D capacitances.


(b) A, C are resistance and B, D capacitances.
Vs = 4 V Is = 5 A 2 (c) A, B are capacitances and C, D resistances.
(d) A, C are capacitances and B, D resistances.
[EE-2010 : 1 Mark]
16 Electronics Engineering Network Theory

Q.32 If the 12 resistor draws a current of 1 A as Q.35 Three capacitors C1, C2 and C3 whose values are
shown in the figure, the value of resistance R is 10 µF, 5 µF and 2 µF respectively have breakdown
voltages of 10 V, 5 V and 2 V respectively. For the
1 R
interconnection shown below, the maximum safe
voltage in volts that can be applied across the
combination, and the corresponding total charge
2A 1A 12 6V
in µC stored in the effective capacitance across
the terminals are, respectively
C2 C3

(a) 4 (b) 6
(c) 8 (d) 18
[EE-2010 : 2 Marks]
C1
Q.33 If VA – VB = 6 V, then VC – VD is
(a) 2.8 and 36 (b) 7 and 119
R VA 2 VB R
(c) 2.8 and 32 (d) 7 and 80
[EE-2013 : 2 Marks]

R R – Q.36 Consider a delta-connection of resistors and its


R R R 10 V
1
+ equivalent star-connection as shown below. If
all elements of the delta-connection are scaled
by a factor k, k > 0, the elements of the
+ –
VC VD R corresponding star equivalent will be scaled by
5V 2A
a factor of
(a) –5 V (b) 2 V
Ra RC RB
(c) 3 V (d) 6 V
[EE-2012 : 2 Marks]
Rb Rc RA
Q.34 In the circuit shown below, the current through
the inductor is
(a) k 2 (b) 5
I2
1
j1 (c) (d) k
1

1 k
0A

[EE-2013 : 1 Mark]
1 0V 1 0V
– + + –
Q.37 The three circuit elements shown in the figure
are part of an electric circuit. The total power
1
0A

absorbed by the three circuit elements in watts


–j1 1
is _______ .
10 A 8A

2 1
(a) A (b) A 100 V 80 V
1+ j 1+ j
1 15 V
(c) A (d) 0 A
1+ j
[EE-2012 : 1 Mark]
[EE-2014 : 1 Mark]
GATE Previous Years Solved Paper 17

Q.38 An incandescent lamp is marked 40 W, 240 V. If (a) 5 (b) 7


resistance at room temperature (26°C) is 120 , (c) 10 (d) 14
and temperature coefficient of resistance is [EE-2015 : 1 Mark]
4.5 × 10 –3/°C, then its ‘ON’ state filament
temperature in °C is approximately _______ . Q.42 In the given circuit, the parameter ‘k’ is positive,
[EE-2014 : 2 Marks] and the power dissipated in the 2 resistor is
12.5 W. The value of ‘k’ is _______ .
Q.39 In the figure, the value of resistor R is
2 5
I + –
25 + , where I is the current in amperes. Vo
2
10
The current I is _______ . 4V 5A

kVo
I

300 V R [EE-2015 : 2 Marks]

Q.43 The current i (in Ampere) in the 2 resistor of


the given network is _______ .
[EE-2014 : 2 Marks]
1
Q.40 The power delivered by the current source, in
the figure, is _______ . i
1 1

5V 2
1V
1 1

1
1V 2A 1
[EE-2015 : 1 Mark]

Q.44 RA and RB are the input resistance of circuits as


[EE-2014 : 2 Marks] shown below. The circuits extend infinitely in
the direction shown. Which one of the following
Q.41 The voltages developed across the 3 and 2
statements is true?
resistors shown in the figure are 6 V and 2 V
respectively, with the polarity as marked. What 2 2 2
is the power (in Watt) delivered by the 5 V
voltage source? RA
1 1 1
3
– +
6V
2 2 2
Network 2 Network
N-1 + – N-2
2V RB
1 1 1 1

– +
5V
18 Electronics Engineering Network Theory

(a) RA = RB (b) RA = RB = 0 (a) 0 (b) 5


RA (c) 10 (d) 20
(c) RA < RB (d) RB =
(1 + RA ) [EE-2016 : 1 Mark]
[EE-2016 : 1 Mark]
Q.49 The equivalent resistance between the terminals
Q.45 In the portion of a circuit shown, if the heat A and B is _______ .
generated in 5 resistance is 10 calories/sec, 1 2 1
then heat generated by the 4 resistance, in A
6
calories per second, is _______ .
4 6 3 6 1

3
B
0.8
5
[EE-2017 : 1 Mark]
[EE-2016 : 1 Mark]
Q.50 The power supplied by the 25 V source in the
Q.46 In the given circuit, the current supplied by the
figure shown below is ______ W.
battery, in ampere, is ________ .
R1
I1 1 1 I2 I
+ 17 V –
+

1V I2 1 25 V 14 A R2 0.4I

[EE-2016 : 1 Mark]
[EE-2017 : 1 Mark]
Q.47 In the circuit shown below, the node voltage VA
Q.51 The equivalent impedance Zeq for the infinite
is _______ V.
ladder circuit shown in the figure is
A I1 5
j9 j9

5 5
5 5A
j5 j5
– Zeq
+ 10I1 10 V ......

–j1 j1
[EE-2016 : 2 Marks]

Q.48 In the circuit shown below, the voltage and


current sources are ideal. The voltage (Vout) (a) j12 (b) –j12
across the current source (in Volts), is ______ . (c) j13 (d) 13
2 [EE-2018 : 2 Marks]

Q.52 The current I flowing in the circuit shown below


in amperes (Round off to one decimal place) is
10 V 5A Vo
_________ .
GATE Previous Years Solved Paper 19

I 2 3 1 10°
I2
A2
I1
+ 5I A1
20 V 2A –
I3
A3
1 70°
[EE-2019 : 1 Mark]
[EE-2020 : 1 Mark]
Q.53 Currents through ammeters A2 and A3 in the
figure are 1 10° and 1 70° respectively. The
reading of the ammeter A1
(Rounded off to 3 decimal places) is ______ A.

Electronics & Electrical Engineering


GATE Previous Years Solved Paper

A n swe rs & Expl a n a t i o n s

Answers
EC Basics of Network Analysis

1. (0.5) 2. (c) 3. (d) 4. (a) 5. (d) 6. (d) 7. (b) 8. (a)

9. (d) 10. (a) 11. (b) 12. (c) 13. (b) 14. (c) 15. (d) 16. (d)

17. (a) 18. (d) 19. (c) 20. (a) 21. (d) 22. (a) 23. (d) 24. (a)

25. (a) 26. (*) 27. (d) 28. (d) 29. (d) 30. (b) 31. (c) 32. (a)

33. (c) 34. (a) 35. (b) 36. (c) 37. (b) 38. (a) 39. (b) 40. (b)

41. (c) 42. (d) 43. (c) 44. (a) 45. (29.09) 46. (10) 47. (d) 48. (c)

49. (2.8) 50. (2.62) 51. (100) 52. (a) 53. (8) 54. (1) 55. (d) 56. (5)

57. (–1) 58. (2.143) 59. (8) 60. (0.5) 61. (1) 62. (b) 63. (b)
20 Electronics Engineering Network Theory

Solutions Basics of Network Analysis


EC

1. Sol. 3. (d)
Triangular wave, 0.5 ampere peak, If all resistors are doubled then the current get
1 halved,
Vdt
iL =
L I
So, the current through inductor is the I =
2
integration of the applied voltage across the
inductor. R = 2R
Integration of square wave is a triangular wave. I
V = 2 R = IR = V
So, the current through the inductor is a 2
triangular wave.
Now, v(t) = u(t) – 2u(t – 0.5) + 2u(t – 1) +... 4. (a)
iL(t) = r(t) – 2r(t – 0.5) + 2r(t – 1) +... Current through 5 resistor,
iL(t )
10
i5 = = 2 Amp.
0.5 5
Current through 1 resistor,
0.5 1 1.5 2 2.5 3 t(m-sec) 5
i1 = = 5 Amp.
Ip = 0.5 A 1
So, the current through 15 resistor,
5. (c) i 15 = –(i1 + i5)
N1 : = –(5 + 2) = –7 Amp.
1 1 Voltage across 15 resistor,
Y1(s) = s + +
2s + 1 1 + 2 V15 = 15(i15)
s = 15(–7) = –105 V
2 s2 + 2s + 1
Y1(s) = 5. (d)
2s + 1
N2 : P1 = 4 W, P2 = 9 W
1 1 1+s From superposition theorem,
Y2(s) = + =
2s + 1 2 + 1 2 s + 1 P = ( P1 + P2 )2
s
N3 : = ( 4 + 9)2
1 1+ s
Y3(s) = s + 1
= s+ P = (2 + 3)2 = 25 W
1+ s+1+s
1
1+ 6. (d)
s
L = L1 + L2 ± 2 M
2 s2 + 2s + 1
Y3(s) = = L1 + L2 ± 2 k L1 L2
2s + 1
N4 :
L = 2 + 2 ± 2(0.1) 2 × 2
1 2s 2 + 2s + 1 = 4 ± 0.4
Y4(s) = s + =
2s + 1 2s + 1 L = 3.6 H and 4.4 H
So, N1 and N3 networks having identical driving
point function.
GATE Previous Years Solved Paper 21

7. (b) 1

iD
A
2.5 A 2 2V

i6
i5
Using source transformation,
2 1

iD
Apply KCL at node A, 5V 2V
i0 + i1 + i4 = 0
7 + 5 + i4 = 0
i 4 = –12 A
5 2 3
iD = = = 1 Amp.
8. (a) 2+1 3
Apply KVL, 13. (b)
V+5–4–4 = 0
The nodal or mesh method is based on KCL and
V = 3V
Ohm’s law.
9. (d)
14. (c)
V = V2 A + 2 × 2 + 5
Apply superposition theorem:
= V2 A + 9
For 1 Volt source,
Since, the voltage of 2 A current source is not
2
known. So, it is not possible to find the value of Vab1 = 1 ×
2+2
voltage V.
= 0.5 V
10. (a) For 3 A source,
Voltage in parallel is always equal. 2
Vab3 = 3 × ×2 = 3 V
2+2
11. (b)
The bridge is balanced, Vab = Vab1 + Vab3

Zeq = (2 4) + (2 4) = 0.5 + 3 = 3.5 V

2×4 2×4 4 4 8 15. (d)


Zeq = + = + =
2+4 2+4 3 3 3 a

12. (c)
R1
iD 5 30

1 R2 R3
2.5 A 4 4 b c
15
2V
5 × 30
R1 = =3
5 + 30 + 15
22 Electronics Engineering Network Theory

15 × 5 20. (a)
R2 = = 1.5
50 Z = 3Z Y
15 × 30 3Z = 3Z Y
R3 = =9
50
Z
ZY =
16. (d) 3
Applying KCL at the node (1), 21. (d)
eat + ebt = iL(t)
Applying source conversion,
d
v(t) = L [ e at + ebt ] 10 2 eo
dt
v(t) = aeat + bebt 16 V

17. (a) 80 V 12
6
V2 0V

1V 1V eo 80 eo eo 16
+ + =0
12 12 6
E 4eo = 112
112
eo = = 28 V
4V 5V 4

22. (a)
V1 10 V
Applying KVL,
0 – 1 – E – 5 – 10 = 0 V1
E = –16 V 20 5I 5 I + =0
5
18. (d) 20 – 10I – 20 = 0
Since diode is forward bias it is taken as short- I=0
circuit. Only dependent source acts,
Applying KCL, V1
= 4A
V 4 V V +2 5
+ + =0 Power delivered = I2R
2 2 2
3V = 2 = 16 × 5 = 80 W
2 23. (d)
V=
3
3Vp Ip cos = 1500
2
Vo = V= VL VL
3 3 cos = 1500
3 3ZL
19. (c)
VL2 cos
Applying KCL, ZL =
1500
eo 12 eo eo
+ + =0 400 2 × 0.844
4 4 4 = = 90
1500
3eo = 12
= cos–1(0.844) = 32.44
eo = 4 V
GATE Previous Years Solved Paper 23

As power factor is leading, load is capacitive so 28. (d)


angle will be negative,
MdI/dt MdI/dt
= –32.44° L1 L2
–+ –+
24. (a)
As voltage at 1 node is known.
using nodal analysis only 3 equations V
required.

25. (a) If current enters the dotted terminals of coil 1


i i/3 then a voltage is developed across coil 2 whose
a
higher potential is at dotted terminals,
i/3
MdI L1 dI MdI dI
i/3 V= + + L2
i/6
dt dt dt dt
dI
= (L1 + L2 2M)
dt
i
b dI
i/3 V = Leq
dt
i i i
Vab = ×1+ ×1+ ×1
3 6 3 29. (d)
Vab 5
Req = = 1/ sC 1
i 6 H(s) = =
1 2
s LC + sCR + 1
R + sL +
sC
26. (*)
Question is incomplete. 1 10 6
H(s) = =
10 6 s 2 + s + 1 s 2 + 106 s + 10 6
27. (d)
vc(t) 30. (b)
X = X1 + X2 + X3 + 2Xm – 2Xm
3V
= (j5 + j2 + j2 + j20 – j20)
= j9 (one additive and other subtractive)
t 31. (c)
T
Va = 5 (R1 = R2)
T
1
vc(t) = i dt R3 1.1
C Vb = × 10 = × 10
0 R3 + R4 2.1
1 1 V = Va – Vb = –0.238 V
Here, T= = = 2 m-sec
t 500
C = 2 µF 32. (a)
i Since, the power is absorbed by 60 V source,
3 = T
2 µF I = 12 – I
3 × 2 µF 3 × 2 µF I >0
i= = = 3 mA 12 – I > 0
T 2 m-sec
I < 12 A
Hence, the charging requires constant current
source of 3 mA for 2 m-sec.
24 Electronics Engineering Network Theory

20 V
I 2 –j4

j4 2
I 60 V 14 0°

12 A
Equivalent impedance of the circuit,
33. (c) Z = (2 + j 4) (2 j 4) + 2
P = VI 4 + 16
Z= +2 =7
Energy = P t 4
I = 2 A (Given) 14 0°
Current, I= = 2 0° A
V t = Area under V – t curve 7
1 36. (c)
V t= × 2 × 600 + (10 + 600)
2
A
= 600 + 6000
V t = 6600 i (i + 1)
E = (6600) × 2 = 13200 = 13.2 1 j1
kJ 1A

1 0V 1 0V
34. (a) D
B – – C
+ +
1 3+i i

1A
1 –j1 1
1
i (i + 1)
3A
10 V E
2+i
1A
According to KCL at node D there will be no
2A
1 current in voltage sources.
According to KCL at node A current through
1 inductor will be
i1 = i + 1 ...(1)
Applying KVL in outer loop,
Applying KVL in loop ACDBA we have
(3 + i) 2 + (2 + i) 2 = 10
1 × i + (i + 1) j1 + 1 0 – 1 0 = 0
6 + 2i + 4 + 2i = 10
i + (i + 1) j = 0
4i = 0
(1 + j) i = –j
i=0
j
Power supplied by the voltage across, i= ...(2)
1+ j
P = Vi
Therefore from (1) and (2) we have,
= 10 × 0 = 0 W
j
i1 = i + 1 = +1
35. (b) j+1
Converting delta into star, the circuit can be 1
redrawn as below: i1 =
1+ j
GATE Previous Years Solved Paper 25

37. (b) 40. (b)


Average power is same as rms power, VYZ1 = 100 × 1.25 × 0.8
2
2 5 = 100 V
P = Irms R = ×4
2 In second case, when 100 V is applied at YZ
25 terminals, this whole 100 V will appear across
× 4 = 50 W
=
2 the secondary winding.
Note : Power is consumed only by resistance 100
i.e. by real part of impedance. Hence, VWX2 = = 80 V
1.25
38. (a) YYZ1 100 VWX2 80
= , =
R VA 2 VB R i1 YWX1 100 VYZ2 100
E
i i2 i3 i4
41. (c)
R –
R R R 10 V Q = CV
1 F R +
Q1 = C1V1 = 10 × 10–6 × 10 = 100 µC
i5

Q2 = C2V2 = 5 × 10–6 × 5 = 25 µC
+ VC VD
5V 2A R Q3 = C3V3 = 2 × 10–6 × 2 = 4 µC
VA VB 6 Capacitors C2 and C3 are in series.
i= =
2 2 In series charge in same.
= 3A ...(i) So, the maximum charge on C2 and C3 will be
i5 = 3 A minimum of (Q2, Q3) = min(25 µC , 4 µC ) = 4 µC
KCL at node F, we have, = Q23.
i6 + 2 + i5 = 0 In series the equivalent capacitance of C2 and
i 6 = –2 – i5 C3 is
i 6 = –5 A C 2C 3 5 × 2 10
C 23 = = = µF
So, VC – VD = 1 × i6 = –5 V C2 + C3 5 + 2 7
39. (b) So, the equivalent voltage,

Rb Rc Q23 4 × 10 6
RA = V23 = =
Ra + Rb + Rc C 23 10 × 10 6
7
Ra = kRa
28
Rb = kRb = = 2.8 V
10
Rc = kRc In parallel, the voltage is same,
V1 = V23 = 2.8 V
kRb kRc
RA = Charge in capacitor C1,
kRa + kRb + kRc
Q1 = C1V1
k 2 Rb Rc = 10 × 10–6 × 2.8
= = 28 µC
k( RA + Rb + Rc )
In parallel, the total charge,
Rb Rc
= k× Q = Q1 + Q23
Ra + Rb + Rc
Q = 4 + 28
R A = kRA Q = 32 µC
26 Electronics Engineering Network Theory

42. (d) Using KVL at all the three nodes we get


At node A,
i5 – i3 + i2 = 0 ...(i)
5 At node B,
Is
i4 + i1 – i2 = 0 ...(ii)
10 V 2 1
+ At node C,
Vs 2A i6 + i3 – i1 = 0 ...(iii)
– By putting the value of i3 and i2 from equation
(i) and (ii) in equation (iii) we get,
Voltage across 1 resistance, i6 + (i2 + i5) – i1 = 0
V1 = 10 V i6 + (i1 + i4 + i5) – i1 = 0
Current through 1 resistance, i6 + (2 – 1 – 4) – 2 = 0
10 i6 = 5 A
= 10 Amp.
I1 =
1
Voltage across 2 resistance, 45. Sol.
V2 = 10 V According to the question,
Current through 2 resistance,
10
I2 == 5 Amp. 11
2
Apply KCL at node A,
–2 + Is + I2 + I1 = 0
10 10
Is = 2 – I1 – I2 = 2 – 10 – 5
Is = –13 Ampere
Voltage at node A,
The equivalent -network of the above
VA = 10 V
Y-network is,
Vs – 10 = 10 V
Vs = 10 + 10 = 20 V

43. (c)
11

The current in the 1 resistor, Ra Rb


10
I1 = = 10 Ampere
1 10
10
44. (a)
Rc
Given data: i 1 = 2 A, i4 = –1 A, i5 = –4 A
R=1 10 × 11
Here, Ra = 10 + 11 + = 32
To calculate, i6 = ? 10
10 × 11
i5 Rb = 10 + 11 + = 32
A 10
10 × 10
R R Rc = 10 + 10 + = 29.09
11
i2 i3 Hence, the lowest value among the three
i4
B C i6 resistances is 29.09 .
R i1
GATE Previous Years Solved Paper 27

46. Sol. 50. Sol.

R1 For an infinite ladder network, if all the


resistance are comprises of same value R, then,
5
R R R

R2 7.53 R3 R R R ....

Req
R R
Using star-delta conversions:
The value of R1 is given by
R R ....
5× 3
= 5+3+ = 10
7.5

47. (d) Req

Using the concepts of super node, we get or,


V1 – V2 = 10 0° ...(i) R
V V V
= 1 + 2 + 2 = 4 0° ...(ii)
3j 6j 6
R Req
2V1 + V2 + jV2
= = 4 0° ...(iii)
6j
From equation (i) and (iii), Req

20 + j 24 31.241 50.194 R Req


V2 = =
( 1 + j) 2 135° Req = R + ...(i)
R + Req
= 22.091 –84.806
After solving equation (i) we get,
or, V2 = 2 – 22j
1+ 5
49. Sol. Req = R ...(ii)
2
Using source transformation, we get,
From the given question, the circuit can be
1k redraw as,

2k 4k R

Re Req
I

20 V 8V
R e = R + Req ...(iii)
3k From equation (ii) and (iii) we get,
Applying KVL in above circuit, we get, 1+ 5
20 – 2I – I – 4I + 8 – 3I = 0 Re = R + R = 2.618 R ...(iv)
2
or, 28 = 10I
Re
or, I = 2.8 mA or, = 2.618 = 2.62
R
28 Electronics Engineering Network Theory

51. Sol. So, V2 = 4(I + I + 2I) + 4I (By KVL)


Modifying the given circuit, I + I + 2I = 5 (By KCL)
5
a I= A
4
R R R 4× 5
R
R
R V2 = 4 × 5 + = 25 V
4
4×5
V1 = 4 I = =5V
Rab 4
R
R R R R R
53. Sol.
b

a 0.5Vx

Vx 10 Vy
R R R R R R

+
5A Vx 20 8 + 0.25Vx

Rab R R R R R R
Vx Vx Vy
b + + 0.5Vx = 5 A
20 10
a 1 1 Vy
Vx + + 0.5 = 5 +
20 10 10
13Vx = 100 + 2Vy ...(i)
2R R R 2R and also, Vy = 0.25 Vx ...(ii)
Solving equations (i) and (ii), we have
Rab
52Vy = 100 + 2Vy
b
50Vy = 100 Vy = 2 V
1 Vx = 4Vy = 8 V
1 1 1 1
Rab = + + +
2R R R 2R 54. Sol.
R 300 R3
= = = 100 e h
3 3

52. (a) R2 R2 R3

4 R3 I/6 I/3
g
f R2
I I c
d
R1 I/6
+ + R2
I/3
V2 5A 4 4 2I V1
R1 R2
– I/3 R2

I/6

a
R1 I/3 b
Current flowing through both the parallel 4
will be I. I
10 sin(t)
GATE Previous Years Solved Paper 29

The equivalent resistance across terminal ah


(outer loop) is, a

R1 R2 R3

4/
I/3 I/6 I/3 1

3
4/3
1

10 sint

3
4/
1 I I
V= ×3 k + ×6 k + ×9 k = 5I 1
3 6 3
V b
or, = 5k
I
For half wave rectifier, Again by star to delta conversion,
Im 10 sin t a
Irms = = = 2 sin t mA
(2) 5k
Im 1
Irms = = 1 mA
2
4
55. (d) 4
1
Req 4

1 1
1
b

1 1 Rab = {(4 1) + (4 1)} {(1 4)}


1 1
1 4 4 4 8
= + =
5 5 5 15

1 56. Sol.
1
0.16Vx R1 Vx/5 R2
b

By using delta to star conversion, 5 3

a
+
60 V 0.4Vx 5 Vx

1
1

1/3 Using KVL in the outer loop,


1 Vx
1 60 5(0.16 Vx ) × 3 Vx = 0
1/3 5
1/3
or, Vx = 25 V
The current flowing through,
1 1
Vx 25
R2 = = = 5A
b
5 5
30 Electronics Engineering Network Theory

57. Sol. 59. Sol.

5
R1 R1
1A
1 8V I
A B
R2
1 V1 1

8V R1 R1 11 V
i R3 R3

1 R2
C D

Using KCL at V1, R1 R1


V1 V1 8 V1 8 V1
+ + + = 0 or V1 = 4 V
1 1 1 1
Considering KVL, we get,
1A 5A R1 R1

4A I
A B

4A
i = –1 A
R1 R1 11 V
R3 R3
58. Sol.
The connection of resistors is as shown below, C D
RB

R1 R1
RA

RC

Given resistor values are: 10 , 5 , 2


The maximum resistance possible is,
R1 R1
RT(max) = 10 + (5 2 )
I
10 80
= 10 + =
7 7
The minimum resistance possible is,
R1 R1 R3 R3 11 V
RT(min) = 2 + (10 5 )

10 16
= 2+ =
3 3
RT (max) 80 /7 15 R1 R1
= = = 2.143
RT (min) 16 /3 7
GATE Previous Years Solved Paper 31

As the network is symmetric, Given that,


VA = VB and VC = VD R1 = 1 and R3 = 3
So, current through R2 resistors is zero and as 1 3 3 /2 11
VA = VB and VC = VD, electrically the circuit can So, RT = 1 + = 1+ =
2 2 4 8
be reduced as,
11 V 11
Total resistance, I= = =8A
RT (11/8)
R T = 2 ( R1 R1 ) + ( R1 R1 R3 R3 )

R1 R3
= R1 +
2 2

Answers
EE Basics of Network Analysis

1. (d) 2. (c) 3. (c) 4. (b) 5. (5) 6. (31) 7. (2) 8. (c)


9. (b) 10. (d) 11. (b) 12. (d) 13. (b) 14. (b) 15. (c) 16. (c)
17. (d) 18. (c) 19. (b) 20. (a) 21. (d) 22. (c) 23. (a) 24. (a)
25. (b) 26. (a) 27. (d) 28. (a) 29. (b) 30. (b) 31. (d) 32. (b)
33. (a) 34. (c) 35. (c) 36. (b) 37. (330) 38. (2470.44°) 39. (10) 40. (3)
41. (a) 42. (0.5) 43. (0) 44. (d) 45. (2) 46. (0.5) 47. (11.42) 48. (d)
49. (3) 50. (250) 51. (a) 52. (1.4) 53. (1.732)

Solutions
EE Basics of Network Analysis

1. (d) By putting the options, it can be concluded that


for i = 5 A,
R = 1 + [(1 1 + 1) (1 1 + 1) + 1]
Px = (6 × 5) – (52 × 1) = 5 W
[(1 (1 1 + 1) (1 1 + 1) + 1] Option (c) is correct.
15
= 5. Sol.
8
1 I
V 1 8
I= = = A +
R 15 /8 15
Non-linear
2. (c) 10 V V
load
As the given bridge is balanced Wheatstone
bridge, current flowing through the lamp will –

remain same irrespective of the state of switch. Using KVL,


Hence intensity of lamp will remain same. V + I = 10 ...(i)
Given, 2
7I = V + 2 V ...(ii)
3. (c)
On solving equation (i) and equation (ii)
P x = P6 V – P1 we get, V = 5 V, I = 5 A
= 6 × 1 – 12 × 1 = 5 W
32 Electronics Engineering Network Theory

6. Sol. 9. (b)

1 I2 1 When resistor R is connected to a current source,


I V1 8V

I1 2A I3
I R
E 6 4 1

P = I2 R = 18 W
When resistor R is connected to a voltage source,
Voltage across 4 resistor = 4 × 2 = 8 V
Current through 1 resistor,
8 V R
=8A
I3 =
1
I2 = I3 + 2 = 10 A
V2
V1 = 8 + 1 × 10 P= = 4.5 W
R
= 18 V
Given, V = I (in magnitude)
Current through 6 resistor,
I2R = 18 ...(i)
18
I1 = =3A I2
6 = 4.5 ...(ii)
R
Current through 1 resistor,
On solving these two equations, we get,
I= I1 + I2
I = 3A; R=2
= 3 + 10 = 13 A
E= V1 + I 1 10. (d)
= 18 + 13 × 1 = 31 V
i
iR ic
7. Sol.
For the given waveforms, v(t) C 1F
R 1
di(t )
v(t) = 2
dt
Comparing it with, i = iR + iC
di(t ) iR = v(t)
v(t) = L
dt 1

we get, L = 2H
t
8. (c) ic =
dv(t )
dt
Given circuit can be written as, 1

B Y t
Cc

Cs Cc Cs Cc
i = iR + ic

2
R
1

0 t
C + Cc C + 3Cc
C BY = s + Cc + s –1
2 2
v(t ) dv(t )
i = iR + iC = +1
1 dt
GATE Previous Years Solved Paper 33

11. (b) 14. (b)

I I2 2 By KCL,
+ IP + IQ + IC + IL = 0
I1
2 + 1 + IC + IL = 0
V + 2I dv

4 But, IC = C ×
dt

d
= 1× (4 sin 2t ) = (8 cos 2t )
Current through 4 resistor, dt
IL = –(2 + 1 + 8 cos2t)
V
I1 = = –3 – 8 cos2t
4
Current through 2 resistor, di
VL = L = 2 × 2 × 8sin 2t
dt
V 2I
I2 = = 32 sin2t
2
Note: KCL is based on the law of conservation
V V 2I
Total current, I = I1 + I 2 = + of charges.
4 2
V V 15. (c)
I= + I
4 2 For 0 < t < 2s current varies linearly with time
3 and given as, i(t) = 3t and for 2s < t < 4s current
2I = V
4 is constant, i(t) = 6 A.
V 8 The energy absorbed by the inductor (Resistance
Load = = neglected) in the first 2 sec,
I 3
T di
12. (d) EL = Li dt = EL1 + EL 2
0 dt
1 T di
Q P EL1 = Li dt
R 0 dt
Therefore resitance of 40 W bulb > resistance of 2
60 W bulb. = 2 × 3t × 3 dt
0
For series connection, current through both the 2
bulbs will be seme P = I2R (for series connection). 2 t2
= 18 t dt = 8 ×
Power consumed by 40 W bulb > power
0 2
0
consumed by 60 W bulb. 4
Hence, the 40 W bulb brighter. = 18 × 0 = 36 J
2
13. (b) The energy absorbed by the inductor in (2 4)
second,
When C is open,
R AB = RA + RB = 6 4 di
EL2 = Li dt
When B is copen,
2 dt
RAC = RA + RC = 9 4
= 2 6 0 dt = 0 J
2
When A is open,
A pure inductor does not dissipate energy but
RBC = RB + RC = 11
only stores it. Due to resistance, some energy is
On solving above equations,
dissipated in the resistor. Therefore, total energy
R A = 2 , RB = 4
absorbed by the inductor is the sum of energy
and RC = 7
stored in the inductor and the energy dissipated
in the resistor.
34 Electronics Engineering Network Theory

The energy dissipated by the resistance in 4 sec, 17. (d)


T 2 By KCL:
ER = i R dt
0
VP 40 VP 100 VP
T 4 2 + + =0
2 1 14 2
= (3t ) × 1 dt + 6 × 1 dt
0 2
22VP = 660
2 4
= (9t 2 ) dt + 36 1 dt R I
0 2

2
t3 4 8 14 P 1
= 9× + 36 t 2 = 9 × + 36 × 2 x y
3 3
0
10 A 5A
= 24 + 72 = 96 J
+ 2 +
The total energy absorbed by the inductor in
100 V V V 40 V
4 sec, – –
= 96 J + 36 J = 132 J

16. (c)
VP = 30 V
Given, VR = 10 V
Potential difference between node x and y = 60 V.
By KCL,
By taking KCL at node y,
VP 10 VP
+2+ =0 ...(i) 40 30
2 8 I 5+ =0
1
VQ 10 VQ I = 5A
2+ =0 ...(ii)
4 6 60
I= = 12
5
2A
18. (c)
10 Va 6 P 2A
R
P Q
2 4 6
+
10 V 1A E

I

8 6 a
Method-1:
4(VP – 10) + 2 × 8 + VP = 0 Using KCL,
4 VP – 40 + 16 + VP = 0 Va E Va
+ 1 =0
5 VP – 24 = 0 6 6
VP = 4.8 2Va – E = 6 ...(i)
From equation (ii), E Va
6(VQ – 10) – 2 × 4 × 6 + 4VQ = 0 where, =2
6
10 VQ – 108 = 0
E – Va = 12 ...(ii)
VQ = 10.8
Solving equaton (i) and (ii), we get
VP – VQ = –6 V
Va = 18 V
and E = 30 V
GATE Previous Years Solved Paper 35

Method-2 : Rc Ra
I = 2+1=3A R2 =
Ra + Rb + Rc
Apply KVL in second loop,
10 × 20
E = 2 × 6 + 3 × 6 = 30 V = =5
20 + 10 + 10
19. (b) Ra Rb
R3 =
Ra + Rb + Rc
10 P 2A
20 × 10
= =5
20 + 10 + 10
Remember : If all the branches of -connection
100 V 10 R
has same impedance Z, then the impedance of
branch of Y-connection be Z/3.

21. (d)
VP 100 VP
+ +2 = 0 Rms value of dc current = 10 A = Idc
10 10
20
2VP – 100 + 20 = 0 Rms value of sinusoidal current = A = Iac
2
80 Rms value of resultant,
VP = = 40 V
2
IR = 2 2
VP 40 Idc + Iac
R= = = 20
2 2 2
20
= 10 2 +
20. (a) 2
= 17.32 A
a
22. (c)
Rb Rc -Y The resultant (R) when viewed from voltage
Transformation
100
source = = 12.5
Ra 8
c c
a R + 10 10 = 12.5

R1 R = 12.5 10 10
= 12.5 – 5 = 7.5
R3 R2
23. (a)

c INL
b
+
Given, Ra = 20 , Rb = 10
DC R
and Rc = 10 VNL
Non-linear
supply resistor
Rb Rc E
R1 =
Ra + Rb + Rc

10 × 10
= = 2.5 2
20 + 10 + 10 VNL = I NL ...(i)
36 Electronics Engineering Network Theory

VNL = E – INLR Va 2.5


Vb = = = 1.25 V
where, E = 3V 2 2
and R=2 i 2 = Vab = Vb
2 i 2 = 1.25 A
VNL = 3 2 I NL = I NL
2 26. (a)
I NL + 2 I NL 3 =0
Bridge is balanced i.e. node C and node D are at
INL = –3 A or 1 A
same parallel. Therefore, no current flows
–3 A is rejected, because the non-linear resistor
through 2 k resistor.
is passive and the only active element in the
circuit is 3 V dc supply. Which is supplying the 27. (d)
power to the resistor. Let resistance of single incandescent lamp = R.
So, INL = 1 A Power consumed by a single lamp,
Power dissipated in the non-linear resistor P = 200 W
2 When connected across voltage,
= VNL I NL I NL I NL
V = 220 V
3
= I NL = 13 = 1 W
V2 220 2
So, P= 200 =
24. (a) R R
R = 242
i = 1A
Let, ‘n’ number of lamps are connected is series
Applying KVL,
across voltage,
Vab – 2i + 5 = 0
V = 200 V
Vab = –5 + 2i
So total resistance of lamps,
= –5 + 2 × 1 = –3 V
Req. = nR = 242 n
Note: KVL is based on the conservation of
Total power consumed,
energy.
V2
25. (b) P=
Req.
1 Va Vb 3
+ Vab – 220 2
100 = n=2
242 n
5V i1 1 1 i2 + 4V
– ab
28. (a)

i1 1k 1k

By KVL in Loop-1,
5 – i1 – i1 = 0 1k 49 i1

5 Vin 100 µF
i1 = = 2.5 A
2 100 µF
Va = 2.5 V
50 i1
By KVL in Loop-2,
4Vab = 3i2 + i2 Applying KVL,
4Vab Vin – i1(1 + 1) – 50 i1(–jXC) = 0
i2 = = Vab
4 Vin = i1[2 – j50XC]]
Vb = 1 × i2 = Vab Vin
Input impedance = =2 j 50XC
Vb = Va – Vb i1
GATE Previous Years Solved Paper 37

As imaginary part is negative, input impedance 32. (b)


has equivalent capacitive reactance XCeq. Assuming voltage of the node Va,
XCeq. = 50 XC
1 Va R
1 50 50 1
= = =
C eq. C × 100 2 I
Ceq. = 2 µF 1A
2A 12 6V
29. (b)
Voltage across 2 resistance
= Vs = 4 V
Va = 1 × 12
Current through 2 resistance
= 12 V
Vs 4 Applying KCL,
= = =2A
R 2 –2 + 1 + I = 0
Current source has no effect, when connected
I = 1A
across voltage source.
Va 6 12 6 6
So, to double current though 2 resistance, I= = =
R R R
voltage source is doubled i.e.,
Vs = 8 V 6
I= R=6
R
30. (b)
A resistor has linear characteristics. 33. (a)
i.e., V = Ri
V=i R VA 2 VB R
Load line, V + i = 100
i + i = 100

Current through resistance, R
R
R R
R
10 V
1
100 +
i= = 50 A
2
+ –
31. (d) 5V
VC VD R
2A
In such system, volumetric flow rate C is
analogous to current and pressure is analogous
VA 2 VB
to voltage. The hydraulic capacitance due to
storage in gravity field is defined as, I
Network Network
A A B
C= 2V
g I
where, A = Area of the tank VC 1 VD

r = Density of the fluid


g = Acceleration due to gravity VA – VB = 2I
The hydraulic capacitance is represented by 2I = 6
A and C. Liquid trying to flow out of a container, I = 3A
can meet with resistance in several ways. If the VC + 2 + 1 × 1 = VD
outlet is a pipe, the friction between the liquid VC – VD = –2 – 3
and the pipe walls produces resistance to flow. = –5 V
Such resistance is represented by B and D.
38 Electronics Engineering Network Theory

34. (c) In parallel, the voltage is same,


V1 = V23 = 2.8 V
A
Change in capacitor C1,
(1 – I2)
Q1 = C1V1
I2
= 10 × 10–6 × 2.8
1A J1
= 28 µC
1
In parallel, the total charge,
C Q = Q1 + Q23
B D
– + + – Q = 4 + 28 = 32 µC
1 0 1 0

36. (b)
1A
–j1 1 Rb Rc
RA =
Ra + Rb + Rc
Ra = kRa
Apply KCL node at ‘A’:
Rb = kRb
So, current flowing through 1 is (1 – I2)
Applying KVL in ABCD loop, Rc = kRc
1 0 – 1 0 + 1(1 – I2) – jI2 = 0 kRb kRc
RA =
1 kRa + kRb + kRc
I2 =
1+ j
k 2 Rb Rc
=
35. (c) k( Ra + Rb + Rc )
Q = CV Rb Rc
= k× = kRA
Q1 = C1V1 Ra + Rb + Rc
= 10 × 10–6 × 10
37. Sol.
= 10 µC
Q2 = C2V2 Given, electrical circuit is shown below:
= 5 × 10–6 × 5 Node
= 25 µC 10 A 8A
Q3 = C3V3
= 2 × 10–6 × 2 = 4 µC 100 V 80 V

Capacitors C2 and C3 are in series. 2A 15 V


In series charge is same.
So, the maximum charge on C2 and C3 will be
minimum of (Q2, Q3) = min(25 µC, 4 µC) = Applying KCL at node, current through 15 V
4 µC = Q23. voltage source = 2 A.
In series the equivalent capacitance of C2 and Power absorbed by 100 V voltage source
C3 is, = 10 × 100 = 1000 Watt
C 2C 3 5 × 2 10 Power absorbed by 80 V voltage source
C 23 = = = µF
C2 + C3 5 + 2 7 = –(80 × 8) = –640 Watts
So, the equivalent voltage, and power absorbed by 15 V voltage source
= –(15 × 2) = –30 Watt
Q23 4 × 10 6 28
V23 = = = = 2.8 V Total power absorbed by the three circuit
C 23 10
× 10 6 10
7 elements = (100 – 640 – 30) Watts = 33 Watts
GATE Previous Years Solved Paper 39

38. Sol. Applying nodal analysis at node P, we have,


Let the resistance of incandescent lamp VI V1 VI V2
+ 2 =0
1 1
V 2 (240)2
= RT = = or, 2VI – (V1 + V2) = 2
P 40
2 + (V1 + V2 )
= 1440 or, VI = ...(i)
2
Given, R 0 = 120 , = 4.5 × 10–3/°C
Let, RT be the resistance of the filament in ON Also, V1 – V2 = 1 Volt
state at temperature T. and V1 = 1 Volt
Then, R T = R0[1 + T] V2 = V1 – 1
= 1 – 1 = 0 Volt
RT 1440
or, [1 + T] = = = 12 Putting values of V1 and V2 in equation (i), we
R0 20
get,
or, T = 11
2 + (1 + 0) 3
or, T = 2444.44°C VI = = Volt
2 2
or, T = 2444.44° + 26°
Power delivered by the current source
= 2470.44°C
3
Therefore, ON state temperature of filament = VI I = × 2 [Q I = 2 A(given]
2
= 2470.44°C
= 3 Watts
39. Sol.
41. (a)
I
Given, R= 25 + – 6V +
2
3
or, I = (2R – 50) I
2A
Applying KVL in given loop, 2
300 V R 1A
we have, + –
2V
300 = IR
1A
or 300 = (2R – 50) × R
we get, R = 30 or –5 – +

Since resistance can’t be negative. Therefore, 5V

R = 30 Power = 5 × 1 = 5 Watt
Hence, I = (2R – 50)
42. Sol.
= (2 × 30 – 50) A = 10 A
current, I = 10 A 2 A 5
+ –
Vo
40. Sol.
10
1V 4V 5A

kVo

1 VI 1
V1 V2
V02
= 12.5
2
1V 2A 1
V02 = 12.5 × 2
V0 = 5
5
I0 = = 2.5 A
2
40 Electronics Engineering Network Theory

KCL at A : 46. Sol.


–2.5 – k(5) + 5 = 0
E I1 1 A 1 I2 B
k(5) = 2.5
2.5 1
k= =
5 2
1V I2 1

43. Sol.
Redrawing the circuit, D C

Applying KCL at node A,


1 1 –I1 + I2 + I2 = 0
2I2 = I1 ...(i)
2
and applying KVL in loop ABCD,
1 – I1 – I2 – I2 = 0
1 1
I1 + 2I2 = 1 ...(ii)
From equation (i) and (ii):
2I2 + 2I2 = 1
5V
4I2 = 1
Bridge is balance, so current flowing through
1
2 resistor is 0 A. I2 = A
4
44. (d) 1 1
and I1 = 2 × = A
If the equivalent resistance of first figure is RA 4 2
then from the second figure, we can see that
47. Sol.
RB = RA 1 .
A I1 5
RA
RB =
RA + 1 5 5
5 5A
45. Sol.

+ 10I1 10 V
4 6

Applying KCL at node A, we get,


VA VA 10 VA + 10 I 1
+ + =5
5 5 10 5
So, 2VA + VA – 10 + 2VA + 20I1 = 5
I 4 6
5VA + 20I1 = 60
VA 10
Since, I1 =
10
2I 5 So, 5VA + 2VA – 20 = 60
7VA = 80
and (2I)2 × 5 = 10
10 2.5 80
= = 0.5 VA =
I2 = 7
5× 4 5
So, I2 × 4 = 0.5 × 4 = 2 cal/sec. = 11.42 Volt
GATE Previous Years Solved Paper 41

48. (d) Z1
A C
2
+
5A Z2 Zeq

10 V 5A Vo
B D
– Zeq

So, Vout = (5 × 2) + 10 = 20 V Z 1 = j9
Z 2 = j5 – j1 = j4
49. Sol.
Z2 Zeq
Consider the following circuit diagram, Zeq = Z1 +
Z2 + Zeq
1 2
A By solving above equation,
Zeq = j12
3 6 3 6 1
52. Sol.
B
I 2 Vx 3
0.8
After rearrangement we get,
A
20 V 2A + 5I
1 –

2 3

0.8
B
Applying nodal at node x,
6 Vx 5I
Now, R AB = 1 + + 0.8 = 3 I 2+ =0
5 3
–3I – 6 + Vx – 5I = 0
50. Sol.
8I = Vx – 6 ...(i)
Using KCL at node, we get,
20 Vx
I + 0.4I = 14 As, I=
2
or, I = 10 A
Vx = 20 – 2I ...(ii)
Now, power supplied,
Substituting (ii) in (i),
P = 25 × 10
8I = 20 – 2I – 6
= 250 W
10I = 14
51. (a) I = 1.4 A
Z1 C Z2 53. Sol.
A
I = 1 10° + 1 70°
Z2 Z2 I = 1.732 40°
The ready of ammeter is 1.732 A.
B
D
Zeq Zeq
2 Sinusoidal Steady State

ELECTRO NICS EN GINEERIN G Q.4 In a series RLC high Q circuit, the current peaks
at a frequency
(GATE Previous Years Solved Papers)
(a) equal to the resonant frequency.
Q.1 The value of current through the 1 Farad (b) greater than the resonant frequency.
capacitor of figure is
(c) less than the resonant frequency.
0.5 F (d) none of the above is true.
1 1 2 [EC-1991 : 2 Marks]
1F
2 sin100t Q.5 For the series RLC circuit of Fig. (1), the partial
1 1 1H phasor diagram at a certain frequency is a
0.5 F
shown in Fig. (2). The operating frequency of
the circuit is
(a) zero (b) once
(c) two (d) three VR VL
+ – + –
[EC-1987 : 2 Marks] +
+
Q.2 The half power bandwidth of the resonant V VC

circuit of figure can be increased by

Fig. (1)

R1 VR

C R2 V

(a) increasing R1 (b) decreasing R1


VC
(c) increasing R2 (b) decreasing R2
Fig. (2)
[EC-1989 : 2 Marks]
(a) equal to the resonance frequency
Q.3 The resonant frequency of the series circuit
(b) less than the resonance frequency
shown in figure is
(c) greater than the resonance frequency
M=1H
(d) not zero
[EC-1992 : 2 Marks]

2H 2H 2F Q.6 In the series circuit shown in figure, for series


resonance, the value of the coupling coefficient
1 1
(a) Hz (b) Hz ‘k’ will be
4 3 4
k
1 1
(c) Hz (d) Hz 18
2 10 4 2
–j12 j2 j8
[EC-1990 : 2 Marks]
GATE Previous Years Solved Paper 43

(a) 0.25 (b) 0.5 Q.11 The current i(t) , through a 10 resistor in series
(c) 0.999 (d) 1.0 with an inductance, is given by
[EC-1993 : 2 Mark] i(t) = 3 + 4 sin(100t + 45°
+ 4 sin(300t + 60°) Amperes
Q.7 In figure, A1, A2 and A3 are ideal ammeters. If A1
The rms value of the current and the power
reads 5 A, A2 reads 12 A, then A3 should read
dissipated in the circuit are:
R (a) 41 A, 410 W respectively
A1

A3 (b) 35 A, 350 W respectively


C
A2 (c) 5 A, 250 W respectively
(d) 11 A, 1210 W respectively
[EC-1995 : 1 Mark]
100 sin t
Q.12 A series RLC circuit has a Q of 100 and an
(a) 7 A (b) 12 A impedance of (100 + j0) at its resonant
7
angular frequency of 10 radians/sec. The
(c) 13 A (d) 17 A
[EC-1993 : 2 Marks] values of R and L are:
R = _______ , L = ________ Henries.
Q.8 A series LCR circuit consisting of R = 10 ,
[EC-1995 : 1 Mark]
XL = 20 and XC = 20 is connected
Q.13 The rms value of a rectangular wave of period
across an a.c. supply of 200 V rms. The rms T, having a value of +V for a duration, T1 (<T)
voltage across the capacitor is and –V for the duration, T – T1 = T2, equals
(a) 200 –90° V (b) 200 +90° V T1 T2
(c) 400 +90° V (d) 400 –90° V (a) V (b) V
T
[EC-1994 : 1 Mark] T1
V
(c) (d) V
Q.9 A DC voltage source is connected across a series 2 T2
RLC circuit. Under steady-state conditions, the [EC-1995 : 1 Mark]
applied DC voltage drops entirely across the
Q.14 In figure A1, A2 and A3 are ideal ammeters. If A2
(a) R only
and A3 read 3 A and 4 A respectively, the A1
(b) L only
should read
(c) C only
L
(d) R and L combination A2
[EC-1995 : 1 Mark]
A1
R
Q.10 Consider a DC voltage source connected to a A3
series R-C circuit. When the steady-state reaches,
the ratio of the energy stored in the capacitor to
the total energy supplied by the voltage source,
Sinusoidal
is equal to
voltage source
(a) 0.362 (b) 0.500
(a) 1 A (b) 5 A
(c) 0.632 (d) 1.000
(c) 7 A (d) None of the above
[EC-1995 : 1 Mark]
[EC-1996 : 1 Mark]
44 Electronics Engineering Network Theory

Q.15 In the circuit of figure, assume that the diodes


are ideal and the meter is an average indicating
ammeter. The ammeter will read =0 E = Em 0°
Em
2 R2
A (b)
+ D1
D2 Em
4 sin( t) 2 R2 I2
10 k
Volts
10 k =

(a) 0.4 2 mA (b) 0.4 mA I2

0.8 0.4
(c) mA (d) mA (c) =0 =

[EC-1996 : 1 Mark] Em Em Em
E = Em 0°

2 R2 2 R2 2 R2
Q.16 The parallel RLC circuit shown in figure is in
resonance. In this circuit,

IR IL IC E = Em 0°
Em =0
1 mA R L C 2 R2
(rms)
Em
(d) 2 R2
(a) I R < 1 mA (b) I R + I L > 1 mA
Em
(c) I R + IC < 1 mA (d) I L + IC > 1 mA 2 R2 I2
[EC-1998 : 1 Mark]
=
Q.17 When the angular frequency in the figure is
varied from 0 to , the locus of the current phasor
I2 is given by [EC-2001 : 2 Marks]

i (t ) Q.18 A series RLC circuit has a resonance frequency


i1(t ) i2(t )
of 1 kHz and a quality factor Q = 100. If each of
R, L and C is doubled from its original value,
Em cos t R2 the new Q of the circuit is
R1
(a) 25 (b) 50
C
(c) 100 (d) 200
[EC-2003 : 1 Mark]

=0
I2
Q.19 An input voltage v(t ) = 10 5 cos(t + 10°)

(a) =
+ 10 5 cos(2t + 10°) V is applied to a series
combination of resistance R = 1 and an
E = Em 0° inductance L = 1 H. The resulting steady-state
Em Em
2 R2 2 R2 current i(t) in ampere is
GATE Previous Years Solved Paper 45

(a) 10 cos(t + 55°) + 10 cos(2t + 10° + tan–1 2) The quality factor (Q-factor) of this circuit is
3 (a) 25 (b) 50
(b) 10 cos(t + 55°) + 10 cos(2t + 55)
2 (c) 10 (d) 5000
(c) 10 cos(t – 35°) + 10 cos(2t + 10° – tan–1 2) [EC-2004 : 2 Marks]

3 Q.23 Consider the following statements S1 and S2:


(d) 10 cos(t 35°) + 10 cos(2t 35°)
2 S1 : At the resonant frequency the impedance
[EC-2003 : 2 Marks] of a series R-L-C circuit is zero.
S2 : In a parallel G-L-C circuit, increasing the
1 conductance G results in increase in its
Q.20 The circuit shown in the figure, with R = ,
3 Q-factor.
Which one of the following is correct?
1
L= H, C = 3 F has input voltage V(t) = sin2t. (a) S1 is false and S2 is true.
4
(b) Both S1 and S2 are true.
The resulting current i(t) is
(c) S1 is true and S2 is false.
i (t )
(d) Both S1 and S2 are false.
[EC-2004 : 2 Marks]
v(t ) R L C
Q.24 The condition on R, L and C such that the step
response y(t) in the figure has no oscillations, is

(a) 5 sin(2t + 53.1°) (b) 5 sin(2t – 53.1°) L R

(c) 25 sin(2t + 53.1°) (d) 25 sin(2t – 53.1°)


[EC-2004 : 1 Mark] +
u(t ) C y(t )
Q.21 For the circuit shown in the figure, the time –
constant RC = 1 ms. The input voltage is

vi (t ) = 2 sin 10 3 t. The output voltage vo(t) is


euqal to 1 L L
(a) R (b) R
2 C C
R

L 1
(c) R 2 (d) R =
C LC
vi(t ) C v o (t )
[EC-2005 : 1 Mark]

Q.25 In a series RLC circuit, R = 2 k , L = 1 H and


(a) sin(103t – 45°) (b) sin(103t + 45°)
1
(c) sin(103t – 53°) (d) sin(103t + 53°) C= µF.
400
[EC-2004 : 1 Mark]
The resonant frequency is
V (s) 1
Q.22 The transfer function H (s ) = o of an RLC (a) 2 × 104 Hz (b) × 10 4 Hz
Vi (s )
circuit is given by (c) 104 Hz (d) 2 × 104 Hz
10 6 [EC-2005 : 1 Mark]
H (s) = 2
s + 20s + 106
46 Electronics Engineering Network Theory

Q.26 For the circuit shown in the figure, the (b) The bandwidth of the circuit remains same
instantaneous current i1(t) is, if L is increased.
j2 –j2 (c) At resonance, input impedance is a real
quantity.
+ (d) At resonance, the magnitude of input
5 0° A 3
impedance attains its minimum value.
i1 10 60° A
– [EC-2010 : 1 Mark]

Q.30 The current ‘I’ in the circuit shown is


20 mH
10 3 10 3
(a) 90° A (b) 90° A
2 2
(c) 5 60° A (d) 5 –60° A
20 0° V I 1 50 µF
[EC-2005 : 2 Marks] 3
= 10 rad/s

Q.27 In the AC network shown in the figure, the


phasor voltage VAB (in Volts) is (a) –j1 A (b) j1 A
A (c) 0 A (d) 20 A
[EC-2010 : 2 Marks]
5 5 Q.31 The circuit shown below is driven by a
5 30° A sinusoidal input vi = Vp cos(t/RC). The steady
output vo is,
–j3 j3
R C

B +
+
(a) 0 (b) 5 30°
vi R C vo
(c) 12.5 30° (d) 17 30° –
[EC-2007 : 2 Marks] –

Q.28 An AC source of rms voltage 20 V with internal


Vp t
impedance Zs = (1 + 2j) feeds a load of (a) cos
3 RC
impedance ZL = (7 + 4j) in the figure below.
The reactive power consumed by the load is Vp t
(b) sin
3 RC
Zs = (1 + 2j)
ZL = (7 + 4j)

Vp t
20 0° V (c) cos
2 RC

Vp t
(a) 8 VAR (b) 16 VAR (d) sin
2 RC
(c) 28 VAR (d) 32 VAR
[EC-2011 : 1 Mark]
[EC-2009 : 2 Marks]
Q.32 Two magnetically uncoupled inductive coils
Q.29 For a parallel RLC circuit, which one of the have Q factors q1 and q2 at the chosen operating
following statements is not correct? frequency. Their respective resistance are R1 and
(a) The bandwidth of the circuit decreases if R R2. When connected in series, their effective Q
is increased. factor at the same operating frequency is
GATE Previous Years Solved Paper 47

1 1 i(t)
(a) q1 + q2 (b) +
q1 q2

(q1 R1 + q2 R2 ) (q1 R2 + q2 R1 ) (b)


(c) (d)
( R1 + R2 ) ( R1 + R2 )
[EC-2013 : 2 Marks] t

Q.33 A 230 V rms source supplies power to two loads i(t)


connected in parallel. The first load draws
10 kW at 0.8 leading power factor and the
second one draws 10 kVA at 0.8 lagging power (c)
factor. The complex power delivered by the
source is
t
(a) (18 + j1.5) kVA (b) (18 – j1.5) kVA
(c) (20 + j1.5) kVA (d) (20 – j1.5) kVA i(t)

[EC-2014 : 2 Marks]

Q.34 A periodic variable x is shown in the figure as a (d)


function of time. The root-mean-square (rms)
value of x is ______ .
t
x
[EC-2014 : 1 Mark]
1 Q.36 The steady-state output of the circuit shown in
the figure is given by
y(t) = A( ) sin( t + ( ))
t If the amplitude A( ) = 0.25, then the
0
T/2 T/2
frequency is
[EC-2014 : 2 Marks]
R
Q.35 A series RC circuit is connected to DC voltage
source at time t = 0. The relation between the C y (t )
source voltage V s , the resistance R, the sin( t) C
capacitance C, and the current i(t) is given below,
C
t
1
Vs = Ri (t ) + i(t ) dt
C
0 1 2
(a) (b)
Which one of the following represents the 3RC 3RC
current i(t)? 1 2
(c) (d)
i(t) RC RC
[EC-2014 : 2 Marks]

Q.37 In the circuit shown, at resonance, the amplitude


(a) of the sinusoidal voltage (in Volts) across the
capacitor is ________ .
t
48 Electronics Engineering Network Theory

4 0.1 mH 1
(a)
2 LC

1 C
10 cos t (V ) 1 µF (b) 1 R2
2 LC L

1 L
(c) 1
2 LC R 2C
[EC-2015 : 1 Mark]
1 C
Q.38 The damping ratio of a series RLC circuit can be (d) 1 R2
2 LC L
expressed as
[EC-2015 : 2 Marks]
2 2L
R C
(a) (b) Q.42 At very high frequencies, the peak output
2L R 2C
voltage Vo (in Volts) is _______ .
R C 2 L 100 µF
(c) (d)
2 L R C
[EC-2015 : 2 Marks]
1k 1k Vo
Q.39 In the circuit shown, the average value of the 100 µF
1.0 sin( t) V
voltage Vab (in Volts) in steady-state condition
is ______ . 1k 1k

1k 1 µF 1 mH 2k
b a
100 µF

Vab +
[EC-2015 : 1 Mark]
5 sin(5000t) 5V
Q.43 In the circuit shown, the current/flowing
through the 50 resistor will be zero if the value
of capacitor C (in µF) is _______ .
[EC-2015 : 1 Mark]
50 I 1 mH 1 mH
Q.40 The voltage (Vc) across the capacitor (in Volts)
in the network shown in ______ .

80 V 40 V Vc 1 mH
5 sin(5000t) C

[EC-2015 : 2 Marks]

100 V, 50 Hz Q.44 The figure shows at RLC circuit with a


sinusoidal current source.
[EC-2015 : 1 Mark]
IR IL IC
Q.41 An LC tank circuit consists of an ideal capacitor
C connected in parallel with a coil of inductance Im sin t R 10 L 10 mH C 10 µF
L having an internal resistance R. The resonant
frequency of the tank circuit is
GATE Previous Years Solved Paper 49

IL V1
At resonance, the ratio , i.e., the ratio of the
IR 4 1H

magnitude of the inductor current phasor and


the resistor current phasor, is _____ . 5
[EC-2016 : 1 Mark] 100 cos3t V2

Q.45 In the RLC circuit shown in the figure, the input 1/36 F

voltage is given by
Vi(t) = 2 cos(200t) + 4 sin(500t) [EC-2017 : 2 Marks]
The output voltage Vo(t) is
Q.48 In the circuit shown, V is a sinusoidal voltage
0.25 H 100 µF source. The current I is in phase with voltage V.
amplitude of voltage across the capacitor
+ + The ratio
amplitude of voltage across the resistor
2
0.4 mH 10 µF is ______ .
Vi(t) Vo(t )
5 5H I

2
– –
V 5F
(a) cos(200t) + 2 sin(500t)
(b) 2 cos(200t) + 4 sin(500t)
(c) sin(200t) + 2 cos(500t) [EC-2017 : 1 Mark]
(d) 2 sin(200t) + 4 cos(500t)
Q.49 For the circuit given in the figure, the voltage VC
[EC-2016 : 1 Mark]
(in Volts) across the capacitor is
Q.46 In the circuit shown, the positive angular 100 k
frequency (in radians/second) at which the
magnitude of the phase difference between the +
+
voltages V1 and V2 equals /4 radians, is ____ . 5 sin(5t) V 1 µF VC
– –
V2

1 1 mH
100 k

(a) 1.25 2 sin(5t 0.25 )


100 cos t 1 V1
(b) 1.25 2 sin(5t 0.125 )

(c) 2.5 2 sin(5t 0.25 )


[EC-2017 : 1 Mark] (d) 2.5 2 sin(5t 0.125 )
Q.47 The figure shows an RLC circuit excited by the [EC-2018 : 2 Marks]
sinusoidal voltage 100 cos(3t) Volts, where ‘t’ is
Q.50 In the circuit shown, if v(t) = 2 sin(1000t) Volts.
amplitude of V2 R = 1 k and C = 1 µF, then the steady-state
in seconds. The ratio is ___ .
amplitude of V1 current i(t), (in mA), is
50 Electronics Engineering Network Theory

R1
i (t )

+ Z(j )
C R2 C
v (t ) R R

C C

R Im(Z) k

(a) 3 sin(1000t) + cos(1000t)


1.5
(b) sin(1000t) + cos(1000t)
=0
(c) sin(1000t) + 3 cos(1000t)
(d) 2 sin(1000t) + 2 cos(1000t) 0 1 2 3 4 5 Re (Z) k
[EC-2019 : 2 Marks]
(a) R1 = 2 k , R2 = 3 k
Q.51 The current in the RL circuit shown below is (b) R1 = 5 k , R2 = 2 k
i(t) = 10 cos(5t – /4) A. The value of the inductor
(c) R1 = 5 k , R2 = 2.5 k
(Rounded off to two decimal places) is ____ H.
(d) R1 = 2 k , R2 = 5 k
R i(t) [EC-2022]

+
Q.54 Consider the circuit shown in the figure with
input V(t) in volts. The sinusoidal steady-state
200 cos(5t) V L
– current I(t) flowing through the circuit is shown
graphically (where ‘t’ is in seconds). The circuit
element ‘Z’ can be _______ .
[EC-2020 : 1 Mark] I(t) 1

Q.52 The current ‘I’ in the given network is

V(t) = sin(t) Z

120 –90° V Z
Z = (80 – j35)

I
t = /4
1
A
120 –30° V Z 2
I(t)
0A t
1
A
2
(a) 2.38 –23.63° A (b) 0 A
(c) 2.38 –96.37° A (d) 2.38 143.63° A (a) a capacitor of 1 F
[EC-2020 : 2 Marks] (b) an inductor of 1 H
(c) a capacitor of 3F
Q.53 For the circuit shown, the locus of the
impedance Z(j ) is plotted as increases from (d) an inductor of 3F
zero to infinity. The values of R1 and R2 are: [EC-2022]
GATE Previous Years Solved Paper 51

ELECTRICAL EN GINEERIN G (a) 10 cos( t + 36.87°) mA

(GATE Previous Years Solved Papers) (b) 14 cos( t + 36.87°) mA


(c) –14 cos( t + 36.87°) mA
Q.1 In the given circuit, the voltage VL has a phase
(d) –10 cos( t + 36.87°) mA
angle of ______ with respect to Vs .
[EE-1999 : 2 Marks]
17.32 j10
+ V – Q.6 In figure, the admittance values of the elements
L
in Siemens are YR = 0.5 + j0, YL = 0 – j1.5,
YC = 0 + j0.3 respectively. The value of ‘I’ as a
phasor when the voltage E across the elements
Vs is 10 0° V is

[EE-1994 : 2 Marks]

Q.2 In the circuit shown in figure, ammeter A2 reads


I YR YL YC E = 10 0° V
12 A and A3 reads 9 A. A1 will read ______ A.

A1

(a) 1.5 + j0.5 (b) 5 – j18


(c) 0.5 + j1.8 (d) 5 – j12
A2 A3 [EE-2004 : 2 Marks]

Q.7 The rms value of the voltage u(t) = 3 + 4 cos(3t) is


[EE-1995 : 1 Mark]
(a) 17 V (b) 5 V
Q.3 Energy stored in capacitor over a cycle, when
excited by an ac source is (c) 7 V (d) (3 + 2 2 ) V
(a) the same as that due to a dc source of [EE-2005 : 1 Mark]
equivalent magnitude.
Q.8 The RL circuit of the figure is fed from a constant
(b) half of the due to a dc source of equivalent
magnitude, variable frequency sinusoidal
magnitude.
voltage source VIN. At 100 Hz, the R and L
(c) zero.
elements each have a voltage drop urms. If the
(d) none of the above frequency of the source is changed to 50 Hz,
[EE-1997 : 1 Mark] then new voltage drop across R is
Q.4 The rms value of half wave rectified R
symmetrical square wave current of 2 A is
(a) 2A (b) 1 A
VIN L
1
(c) (d) 3A
2A
[EE-1997 : 1 Mark] 5 2
(a) urms (b) urms
Q.5 Current I1, I2 and I3 meet at a junction (node) in 8 3
a circuit. All currents are marked as entering 8 3
(c) urms (d) urms
the node. If I 1 = –6 sin( t) mA and 5 2
I2 = 8 cos( t) mA, then I3 will be [EE-2005 : 2 Marks]
52 Electronics Engineering Network Theory

Q.9 An energy meter connected to an immersion Q.12 The power dissipated in the resistor R is
heater (resistive) operating on an AC 230 V, (a) 0.5 W (b) 1 W
50 Hz, AC single phase source reads 2.3 units
(kWh) in 1 hour. The heater is removed from the (c) 2W (d) 2 W
supply and now connected to a 400 V peak to
[EE-2011 : 2 Marks]
peak square wave source of 150 Hz. The power
in kW dissipated by the heater will be Q.13 The current IC in the figure above is
(a) 3.478 (b) 1.739 1
(a) –j2 A (b) j A
(c) 1.540 (d) 0.870 2
[EE-2006 : 2 Marks] 1
(c) + j A (d) +j2 A
Q.10 The rms value of the current i(t) in the circuit 2
shown below is [EE-2011 : 2 Marks]

1F 1H Q.14 The average power delivered to an impedance


(4 – j3) by a current 5 cos(100 t + 100) A is
1 (a) 44.2 W (b) 50 W
i(t)
(c) 62.5 W (d) 125 W
+ –
1
(1.0 sint) V [EE-2012 : 1 Mark]

1 1 Statement for Linked Answer Questions (15 and 16):


(a) A (b) A
2 2 In the circuit shown, the three voltmeter reading are:
(c) 1 A (d) 2A V1 = 220 V, V2 = 122 V, V3 = 136 V.
[EE-2011 : 1 Mark] R I
Q.11 The voltage applied to a circuit is
V2
100 2 cos(100 t ) Volts and the circuit draws a RL
V1 V3 Load
current of 10 2 sin(100 t + /4) amperes.
X
Taking the voltage as the reference phasor, the
phasor representation of the current in amperes
is
Q.15 The power factor of the load is
(a) 10 2 (b) 10
4 4 (a) 0.45 (b) 0.50
(c) 10 + (d) 10 2 + (c) 0.55 (d) 0.60
4 4
[EE-2012 : 2 Marks]
[EE-2011 : 1 Mark]
Q.16 If RL = 5 , the approximate power consumption
Common Data for Questions (12 and 13):
in the load is
An RLC circuit with relevant data is given below.
(a) 700 W (b) 750 W
Is
IRL IC (c) 800 W (d) 850 W
[EE-2012 : 2 Marks]
R
Vs C Vs = 1 0 V
Q.17 The total power dissipated in the circuit, shown
Is = 2 /4 A
L in the figure, is 1 kW.
I RL = 2 /4 A
GATE Previous Years Solved Paper 53

Q.21 In the figure, the voltages are v1(t) = 100 cos( t),
10 A 2A 1 XC1 XL R

v2 (t ) = 100 cos t+ and v3 (t ) = 100 cos t+ .


18 36
Load
AC XC2 V 200 V The circuit is in sinusoidal steady-state, and
source
R << L. P1, P2 and P3 are the average power
outputs. Which one of the following statements
is true?
The voltmeter, across the load, reads 200 V. The
value of XL is ______ . R L L R

[EE-2014 : 2 Marks]
P1 P2 P3
Q.18 The voltage (V) and current (I) across a load are
as follows:
+ + +
V(t) = 100 sin( t)
V1(t) V2(t) V3(t)
i(t) = 10 sin( t – 60°) + 2 sin(3 t) + 5 sin(5 t)
– – –
The average power consumed by the load,
(in Watt), is _______ .
(a) P1 = P2 = P3 = 0
[EE-2016 : 1 Mark]
(b) P1 < 0, P2 > 0, P3 > 0
Q.19 A resistance and a coil are connected in series (c) P1 < 0, P2 > 0, P3 < 0
and supplied from a single phase, 100 V, 50 Hz
(d) P1 > 0, P2 < 0, P3 > 0 [EE-2018 : 1 Mark]
ac source as shown in the figure below. The rms
values of plausible voltages across the Q.22 The voltage across the circuit in the figure and
resistance (V R) and coil (VC) respectively, the current through it, are given by the following
(in Volts) are expressions:
v(t) = 5 – 10 cos( t + 60°) V
VR
i(t) = 5 + X cos( t) A
VS VC where, = 100 radians/s. If the average power
delivered to the circuit is zero, then the value of
X(in Amperes) is _____ (upto 2 decimal places).

(a) 65, 35 (b) 50, 50 i(t)

(c) 60, 90 (d) 60, 80 +


[EE-2016 : 1 Mark]
Electrical
V(t )
Q.20 In the circuit shown below, the supply voltage circuit
is 10 sin(1000t) Volts. The peak value of the
steady-state current through the 1 resistor, in –
amperes, is ______ .
4 [EE-2018 : 2 Mark]
2 µF
Q.23 A 0.1 µF capacitor charged to 100 V is
250 µF 1 500 mH
discharged through a 1 k resistor. The time in
rms (round off of 2 decimal) required for the
5
voltage across the capacitor to drop to 1 V is
4 mH
________.
[EE-2019 : 2 Marks]
10 sin(1000t )
[EE-2016 : 2 Marks]
54 Electronics Engineering Network Theory

Electronics & Electrical Engineering


GATE Previous Years Solved Paper

A n swe rs & Expl a n a t i o n s

Answers
EC Sinusoidal Steady State

1. (a) 2. (a, d) 3. (b) 4. (a) 5. (b) 6. (a) 7. (c) 8. (d)

9. (c) 10. (b) 11. (c) 12. (1) 13. (a) 14. (b) 15. (d) 16. (b)

17. (a) 18. (b) 19. (c) 20. (a) 21. (a) 22. (d) 23. (b) 24. (c)

25. (b) 26. (a) 27. (d) 28. (b) 29. (d) 30. (a) 31. (a) 32. (c)

33. (b) 34. (0.408) 35. (a) 36. (b) 37. (17.68) 38. (c) 39. (5) 40. (100)

41. (b) 42. (0.5) 43. (20) 44. (0.316) 45. (b) 46. (1) 47. (2.6) 48. (0.2)

49. (c) 50. (a) 51. (2.828) 52. (d) 53. (a) 54. (b)

Solutions
EC Sinusoidal Steady State

1. (a) 1 1
B.W. ; B.W.
The given circuit is a bridge, Q selectivity
If, R1 0
2
1 1+ and R2
s
then the circuit will have only L and C elements
2 sin100t 2+s and has high selectivity.
1F
2
So, the half power bandwidth can be increased
1 1+
s by reducing the selectivity.
So, by increasing the series resistance R1 and
Product of opposite arms are equal,
decreasing the parallel resistance R2, the half
2 2 power bandwidth can be increased.
1 1+ = 1 1+
s s
So, the current through the diagonal element 3. (b)
(1 F capacitor) is zero. Leq = L1 + L2 – 2 M
Leq = 2 + 2 – 2(1) = 2 H
2. (a, d)
At resonance,
Selectivity Q XL = XC
fr 1 1
Q= ; Q
B.W. B.W. Leq =
C
GATE Previous Years Solved Paper 55

2
1 XL = j 2 + j 8 + 2 k j 2 j 8 = j 12
=
LeqC
2 k j4 = j2
1 1 k = 0.25
= = rad/sec.
2×2 2
7. (c)
1
2 f=
2 A32 = A12 + A22
1
f= Hz A32 = (5)2 + (12)2
4

4. (a) A32 = 169

At resonance frequency, A3 = 13 Ampere


Zmin = R 8. (d)
V Q XL = XC
Imax =
Zmin So, the circuit is at resonance,

5. (b) V 200
I= = = 20 Ampere
R 10
Given network is the series R-L-C circuit in
Voltage across the capacitor,
resistor R, voltage VR and current IR is in phase
Vc = I(–jXc) = 20(–j20) = –j400
and in series circuit current is same in all the
Vc = 400 –90° V
elements,
I = IR 9. (c)
So, the current is leading the voltage in the For DC supply:
circuit. Inductor behave as short-circuit capacitor
So, the given circuit will behave as capacitive behave as open-circuit.
circuit, So, under steady-state conditions, the applied
VC > VL dc voltage drops entirely across the capacitor
IXC > IXL (C) only.
XC > XL
1 10. (b)
> L
C
Ws = CVs2
1
2 < 1
LC Wc = CVs2
2
2 < 2
r Wc
= 0.5
< r Ws

6. (a) 11. (c)


At resonance,
2 2
XL – XC = 0 4 4
Irms = 32 + + =5A
XL = XC 2 2
Power is dissipated only in the 10 resistor,
XL = j12
2 2
XL = j12 P = Irms R = (5) × 10
= 250 W
XL = XL1 + XL2 + 2 k XL1 XL2
56 Electronics Engineering Network Theory

12. Sol. 17. (a)


Z = R + j(XL – XC) Em cos t j C
= 100 + j0 i2(t) = = Em 0
1 1 + j CR2
R2 +
Compare the real part, j C
R = 100
Em 0 C 90°
L i2(t) =
2 2 2 1
Q= 1+ C R tan CR2
R
QR 100 × 100 Em C
L= = = 1 mH i2(t) = 90° tan 1 CR
7 2 2 2
10 1+ C R

13. (a) E
At = 0, i2(t) = 0, = , i2 ( t ) = m
R2
x(t )
Option (a) satisfies both conditions.
T1
+V
18. (b)
t
T1 T1 + T2 f0
Q=
–V B.W.
T2
1
Rms value of x(t) f0 =
2 LC
T T1 T1 + T2 R
1 2 1 B.W. =
= x (t ) dt = (V )2 dt + ( V )2 dt
T T1 + T2 L
0 0 T1

2 Rs 1
1 (Characteristic equation = s + + )
= [V 2T1 + V 2T2 ] = V 2 = V L LC
T1 + T2
1 L
or, Q=
14. (b) R C
when R, L, C are doubled,
A12 = A22 + A32 = 32 + 4 2
1
Q = Q = 50
A1 = 5 Ampere 2

15. (d) 19. (c)


Diode D1 will conduct for the positive half cycle v(t ) 10 2 cos(t + 10°) 10 5 cos(2t + 10°)
of the input. i(t ) = = +
R+ j L 1 + 1j 1+2j
The ammeter will read the average value,
Vm 1 4 1 10 2 cos(t + 10°) 10 5 cos(2t + 10°)
× = × i(t ) = +
Iavg. = 2 45° 5 tan 1 2
R 10 × 10 3
0.4 i(t) = 10 cos(t – 35°) + 10 cos(2t + 10 – tan–1 2)
Iavg. = mA
20. (a)
16. (b) i(t) = V(t) Y
At resonance, 1 1
I = IR = 1 mA Y = V (t ) + +j C
R1 j L
I R + IL = I R2 + I L2 = 12 + IL2 > 1 mA
4
I R + IL > 1 mA = sin 2t 3 + + j×2×3
2j
GATE Previous Years Solved Paper 57

= sin 2t[3 – 2j + 6j] = sin 2t[3 + 4j] R


2 =
1 4 n L
= 5 sin 2t tan = 5 sin (2t + 53.1°)
3 1
n =
21. (a) LC

1 R R C
= LC =
j C 1 2L 2 L
vo(t) = vi (t ) = 2 sin 10 3 t
1 1 + j CR For no oscillations, 1
R+
j C
R C L
1 3 1; R 2
= 2 sin 10 t 2 L C
1 + j × 10 3 × 10 3
vo(t) = sin(103t – 45°) 25. (b)

22. (b) 1 1
f0 = =
2 LC 1 6
Characteristic equation = s2 + 20s + 106 2 1× × 10
400
Q=
o
, o = 10 6
B.W. 103 × 20 10 4
= = Hz
2
103 1000
Q= = = 50
20 20 26. (a)

23. (d) When 5 0° is acting along,


i1(t) = –5 0°
S1 : Impedance of series RLC circuit at resonant
(as 10 60° is kept open)
frequency is minimum,
When 10 60° is acting along,
1 i1(t) = 10 60°
Z = R+ j L
C
(as 5 0° is kept open)
1 i1(t) = 10 60° – 5 0°
L =0
C = 5 + 8.66j – 5
Z = R (Purely resistive) i1(t) = 8.66j
C i1(t) = 5 3 90°
S2 : Q= R
L 10
= 3 90°
1 1 C 2
G= Q=
R G L
27. (d)
G then Q if C and L are same.
VAB = Current × Impedance
24. (c)
= 5 30°× (5 3 j ) (5 + 3 j )
Transfer function
(5 3 j ) × (5 + 3 j )
1 = 5 30°×
1 5 3j + 5+ 3j
sC =
=
1 2
s LC + sCR + 1
R + sL + 25 + 9
sC = 5 30°×
10
1 = 5 30° × 3.4
Y (s) LC
= = 17 30°
U (s) 2 R 1
s + s+
L LC
58 Electronics Engineering Network Theory

28. (b) 1 20
VA + 1 + j 10 3 × 50 × 10 6
=
j 103 × 20 × 10 3
j 10 3 × 20 × 10 3
1 + 2j
j j
VA +1+ = –j1
20 20
20 0° V I 7 + 4j
VA = –j1 V
VA
I= = j1 A
1

20 10 31. (a)
I= = =2 36.87°
8+ 6j 4 + 3j Redrawing the circuit s-domain,
Reactive power,
Q = I2 XL = 4 × 4 = 16 VAR R+
1
sC
29. (d)
Characteristic equation for a parallel RLC +
circuit is 1
V(s ) I(s) R Vo(s )
sC
1 1 –
s2 + s+ =0
RC LC
1
where, Bandwidth =
RC
1
(i) It is clear that the bandwidth of a parallel R
1 sC I (s )
Vi(s) = R+ I (s) +
RLC circuit is independent of L and sC 1
R+
decreases if R is increased. sC
(ii) At resonance, imaginary part of input
1 + sCR R
impedance is zero. Hence, at resonance Vi(s) = I (s) + I (s) ...(i)
sC 1 + sCR
input impedance is a real quantity.
Q v i = Vp cos(t/RC)
(iii) In parallel RLC circuit, the admittance is
minimum at resonance. Hence magnitude 1
So here, =
RC
of input impedance attains its maximum
value at resonance. Now,
(1 + j CR) R
30. (a) Vi(s) = I(s) + I (s)
j C (1 + j CR )
j L VA 1
Put, =
RC

I
1 (1 + j ) R R
20 0° V 1
j C So, Vi(s) = + I (s)
3
= 10 rad/s j 1+ j

Vi (s) 3R
=
(L = 20 mH, C = 50 µH) I (s ) (1 + j )
Nodal analysis at node A, Vi (s )
I(s) = × (1 + j )
VA 20 VA VA 3R
+ + =0
j L 1 1
1
j C Now, Vo(s) = R I (s)
sC
GATE Previous Years Solved Paper 59

1 Total power (kVA),


R P = V I2
sC I (s )
Vo(s) = 1 I2 = 43.47 –36.86 ...(iii)
R+
sC
Therefore, I = I1 + I 2 = 78.25 + j6.25 ...(iv)
R Vi (s )
Vo(s) = (1 + j ) So, the complex power delivered by source,
1 + sCR 3R
S = VI = 230(78.25 – j6.25)
R Vi (s)
Vo(s) = (1 + j ) or, S = (18 – j1.5) kVA
1 + j 3R
Vi (s ) 34. Sol.
Vo(s) =
3 x
In time domain,
1 1
Vo(t) = Vi (t )
3
Vp t
Vo(t) = cos
3 RC t
0
T/2 T/2
32. (c)
T
L1 L2 1 2
q1 = and q2 = Rms value = x (t ) dt
R1 R2 T
0
L1 = q1R1 and L2 = q2R2 where, T = time period
Coils are connected in series, For the given signal,
So, q R = L1 + L2 = q1R1 + q2R2
T /2 2 T
q R +q R 1 2
q= 1 1 2 2 Rms value = t dt + (0)2 dt
R T T
0 T /2
R = R1 + R2
T /2 T /2
q R +q R 1 4 1 4 t3
q= 1 1 2 2 = t 2 dt = × 2×
R1 + R2 T
0 T2 4 T 3
0

33. (b) 4 T3 1
= 3 24
= = 0.408
T 6
I1 10 kW, 0.8 lead
Z1
35. (a)
I2 10 kVA, 0.8 lag t
Z2 1
Given that, Vs = Ri (t ) + i(t ) dt ...(i)
C
0
I Using Laplace transform,
1
V(s) = RI (s ) + I (s) ...(ii)
Real power (kW), Cs
P = VI cos ...(i) V (s)
or, I(s) = ...(iii)
P1 = V × I1 cos 1
R+
Cs
10 × 10 3
Thus, I1 = = 54.34 36.86 ...(ii)
230 × 0.8 1
For, V(s) =
s
60 Electronics Engineering Network Theory

From equation (iii) and (iv), 1 1


XC = = = 10
C 1 C 10 5 × 1 × 10 6
I(s) = = ...(v)
( RCs + 1) 1 10 / 2 25
R s+
RC VC = IXC = 10 × = = 17.68 V
4 2
Using inverse Laplace transform in equation (v),
we get, 38. (c)
1 t / RC 1 1 L
e
i(t) = = ; Q=
R 2Q R C
Thus, option (a) is correct.
R C
Damping ratio = =
36. (b) 2 L
R VA 39. Sol.

y (t ) 1k 1 µF 1 mH 2k
C b a

sin( t) C V ab +

C 5 sin(5000t) 5V

Applying KCL at node A, we get


Applying superposition:
VA sin t VA V
= + + A =0 ...(i) Vab =5 V (open circuited in steady-state)
R 1 2
j C j C 1k 1 µF 1 mH 2k

V ab +
1 j C sin t = 1 0°
= VA + j C+ = 5V
R 2 R
2
= VA = ...(ii)
2 + 3 RC j
1k 1 µF 1 mH 2k
VA 1
Also, Y= = ...(iii) –
2 2 + 3 j RC V ab +

1 5 sin(5000t)
Q A( ) =
4
1 1
=
4 4+9 2
( RC )2
Vab will be sinusoid with average value zero.
2 Average, Vab = 5 V
or, =
3RC
40. Sol.
37. Sol. Vc
80 V 40 V
At resonance,
10 / 2
I=
4
1
= = 10 5 rad/sec
3 6
0.1 × 10 × 10
100 V, 50 Hz
GATE Previous Years Solved Paper 61

(80)2 + (40 – VC)2 = 100 2 Vo = 1 sin t × 1 k


(40 – VC)2 = 1002 – 802 = 3600 = 0.5 sin t
40 VC = 60 Peak output = 0.5 V

VC = 100 V 43. Sol.

41. j5 j5
50

L
1
× ( j L + R) C 5 sin(5000t) j5 C
j C
Zeq = 1 R
× j L+R
j C

R L 1 Z Zeq
+ R j L
j C C C
Zeq = ×
1 1
R+ j L R j L 1
C C j5 × + j5
j C
Equating imaginary part to zero, Zeq = 1
j5 + j5 +
j C
R2 L 1
Img = L =0
C C C I = 0 if Z is
Z = Zeq + 50 + j5
R2 L 1
+ L =0 For Z to be ,
C C C
Zeq =
2 2 2
CR + LC L 1
2
=0 j5 + j5 + =0
C j C

1 R 2C 1
2 = 1 10 =
LC L 5000 × C
1
1 R 2C C= = 20 µF
f = 1 5 × 10 3 × 10
2 LC L
44. Sol.
42. Sol.
At resonance (for parallel RLC circuit),
Circuit contains balanced Wheatstone bridge. IR = I
Also at high frequencies capacitor can be IL = QI –90°
considered as short-circuits. IC = QI –90°
Redrawing the circuit, For parallel RLC circuit,
Vo
IL IQ C
= =Q= R
1k 1k IR I L

1.0 sin( t) 10 × 10 6
= 10 = 0.316
10 × 10 3
1k 1k
62 Electronics Engineering Network Theory

45. (b) 46. Sol.

0.25 H 100 µF V2

1 1 mH
+ +
100 cos t i (t ) 1 V1
2
0.4 mH 10 µF
Vi(t) Vo(t)

2 Let, i(t) = Im i and Z2 = 1 + j


– –
= 2
1+ 2
where, Vi(t) = 2 cos(200t) + 4 sin(500t)
= tan 1
As different frequencies are operating, using 2
1
superposition theorem, we get for V1 = i(t) ( 1 ) = Im i
= 200 rad/sec V2 = i(t) Z2
XL = L = (200) (0.25) = 50
= Im 1 + 2 2 + i
1 1
XC = =
C 200 × 100 × 10 6 From the given data,
= 50
( i+ 2) – ( i) =
Short-circuit
4
j50 –j50
2 =
4
+ +
1
2
tan =
1 4
Vi(t) Vo(t) = 1 rad/sec

47. Sol.
– – V1

Q Vo(t) = Vi(t) 4 1H

For, = 500 rad/sec.


5
i (t ) V2
XL = 0.4 × 500 = 200
1 1/36 H
XC = 6
= 200
10 × 10 × 500

Open-circuit
= 3 rad/sec,
Z 1 = (4 + j3)
+ + and Z 2 = (5 – j12)
j200 –j 200 V2 =
Vi(t) Vo(t)
i Z2 = i 52 + 12 2 = 13 i
2
– –
V1 = i Z1 = i 4 2 + 32 = 5 i

V2 13 i 13
Vo(t) = Vi(t) = = = 2.6
V1 5i 5
Therefore, Vo(t) = 2 cos(200t) + 4 sin(500t)
GATE Previous Years Solved Paper 63

48. Sol. 1 1 1
Here, XC = = 3 6
= 3
5H
C 10 × 10 10
5 I
XC = 103
+ V –
R
R = 103 (Given)
+
V VC 5F v(t) = 2 sin 1000t V
– = 2 0° V
Redrawing the given network, we get,

Given that, V and I have same phase. So, the


circuit is resonance.
R R
At resonance, VC = QVR
Amplitude of VC 1 L –jXC
So, = Q=
Amplitude of VR R C
–jXC –jXC
1 5
= = 0.2
5 5
R
49. (c)
i(t)
1 1
= 6
= 200 k
C 5 × 10 v(t )
200 k
As the bridge is balanced, it can be redrawn as,
+ +
5 sin(5t) V –j200 k VC
– R R

5 0°
VC = × ( j 200) V
200 j 200
5 0°× 1 90° –jXC –jXC
=
2 45°
R
5
= 45° V = 2.5 2 sin 5t V
2 4

= 2.5 2 sin(5t 0.25 ) V


v(t)

50. (a)
2R
i (t )
2/3R
–2jXC
–2jXC
C
v (t ) R R R
i(t)

i(t)
C C v (t )

R v (t )
64 Electronics Engineering Network Theory

Yeq = Y1 + Y2 Impedance, Z(0) = R1 + R2


3 1 With the help of graph at = 0,
= +
2R 2 jXC Z(0) = 5 k
3 1 R1 + R2 = 5 k
= × 10 3 + j × 10 3 At = rad/sec,
2 2
3 1 1
i(t) = v(t) × Yeq = 2 0° +j mA XC =
2 2 C
= (3 + j1) mA XC = 0
= 3 sin(1000t) + cos(1000t) mA R1

51. Sol.
V 200 0° XC = 0
Z= = = 20 45° R2
I 10 45°
Z = 10 2 + j 10 2 Z(0)

XL = 10 2
Impedance, Z( ) = R1 = 2 k
L = 10 2 R1 + R2 = 5 k
10 2 R2 = 3 k
L= = 2.828 H
5
54. (b)
52. (d) As the current i(t) is lagging, element Z is
inductor,
I1
V sin t
120 –90° V Z I= ; I=
Z0 Z0
Z = (80 – j35)

I Maximum value of current,


1
i(t) =
120 –30° V Z 2
Z0 = 2
I2
Z 0 = R + jXL = 1 + j L
I = –[I1 + I2]
1+ 2 2
L = 2
120 90° 120 30°
I= + 2L2
80 j 35 80 j 35 1+ = 2
Given, = 1 rad/sec.
I = 2.38 143.7°
2
L = 1
53. (a) L= 1H
At = 0 rad/sec,
1
XC = =
C
R1

Z(0) R2
GATE Previous Years Solved Paper 65

Answers
EE Sinusoidal Steady State

1. (60) 2. (15) 3. (c) 4. (a) 5. (d) 6. (d) 7. (a) 8. (c)

9. (b) 10. (b) 11. (b) 12. (b) 13. (d) 14. (b) 15. (a) 16. (b)

17. (17.332) 18. (250) 19. (*) 20. (1) 21. (c) 22. (10) 23. (0.46)

Solutions
EE Sinusoidal Steady State

1. Sol. 5. (d)
j 10 I1 + I2 + I3 = 0
VL = Vs ×
17.32 + j 10 I3 = –I1 – I2
= –[–6 sin( t)] – 8 cos t
(Using voltage division rule)
= Vs × 0.5 60° Volts 6 8
= 10 sin t cos t
Hence, VL has phase angle of 60° with respect 10 10
to Vs. = –10[cos(36.87°) cos t
– sin(36.87°) sin t]
2. Sol. = –10 cos( t + 36.87°) mA
Q Currents in resistor and inductor well be in
6. (d)
quadrature for same voltage across them.
2 2
IA1 = IA 2
+ IA 8
= 12 2 + 92 = 15 A IR IL IC

E = 10 0° V
I
3. (c) YC
YR YL
When excited by an ac source, capacitor stores
the energy in one half cycle and delivers that
energy in another half cycle. Hence total energy IR = YRE = (0.5 + j0) × 10 0° = 5 A
stored in a capacitor over a complete cycle, when IY = YLE = (0.5 – j1.5) × 10 0°
excited by an ac source is zero. = –j15 A
IC = YCE = (0 + j0.3) × 10 0° = j3 A
4. (a)
I = IR + IY + IC
To find: Rms value of i(t) = 5 + (–j15) + j3 = 5 – j12 A
We have,
i (t ) 7. (a)
2A (rms)
Rms value of dc voltage = Vdc =3V

t (rms)
Rms value of ac voltage = Vac
T/2 T 3T/2 2T

T 4
1 2 = V
Irms = i (t ) dt 2
T
0 Rms value of the voltage

= 1 T 4
2
× 4× = 2 A 32 + = 9+8 =
T 2 = 17 V
2
66 Electronics Engineering Network Theory

8. (c) Rms value of the input voltage


At, f = 100 Hz = Vrms = 230 V
2
VR = VL Vrms
P1 =
As R and L are series connected, current through R
R and L is same. 230 2
230 × 103 = R = 23
So, IR = IXL = I L R
R = XL = L (ii) When heater connected to 400 V (peak to
Vin Vin Vin peak) square wave source of 150 Hz.
I= = =
R 2
+ XL2 2
R +R 2 2R
V
VR = urms = IR 200 V
Vin V
VR = × R = in
2R 2

Vin = 2 urms ...(i) –200 V


T/2
At, f = 50 Hz,
1 1
XL f T= = sec.
f 150
50 XL R
XL = X L × = =
100 2 2 Vrms value of the input voltage,

Vin 1/2
1 T
So, I = Vrms = V 2 dt
R 2 + ( XL ) T 0

{ }
1/2
1 T /2 T
Vin 2V = 200 2 dt + ( 200)2 dt
= = in T 0 T /2
R 2 5R
R2 + Vrms = 200 V
2
2
Vrms 200 2 3
P2 = = × 10 = 1.739 kW
2 Vin 2 R 23
VR = I R = R= Vin
5R 5
10. (b)
From equation (i),
Vs = 1 sint Vm sin t
2
VR = × ( 2 urms ) Vm = 1 V and = 1 rad/sec.
5
Impedance of the branch containing inductor
2 2 8 and capacitor,
= urms = urms
5 5 Z = j(XL – XC)

9. (b) 1
= j L
C
Assuming resistance of the heater = R
(i) When heater connected to 230 V, 50 Hz 1
= j 1× 1 =0
source, energy consumed by the heater = 1× 1
2.3 units of 2.3 kWh in 1 hour. So, this branch is short-circuit and the whole
current flow through it,
Power consumed by the heater
1.0 sin t
energy 2.3 kWh i(t) = = 1.0 sin t
= = 1
time period 1 hour
1
P1 = 2.3 kW Rms value of the current = A
2
GATE Previous Years Solved Paper 67

11. (b) 1
= × 25 × 5 × cos(36.87°)
2
V(t) = 100 2 cos(100 t )
1 4
Voltage represented in phasor form, = × 25 × 5 × = 50 W
2 5
Vph = Vrms
Alternate method:
100 2
Vph = 0° P = Irms 2 R
2
2
5
i(t) = 10 2 sin 100 t + P= × 4 = 50
4 2

i(t) = 10 2 cos 100 t + 15. (a)


4 2
V3 V1
10 2
Iph = = 10 A
2 4 2 4

12. (b)
Power supplied by the source = Vs Is cos
V2
where, = angle between Vs and Is = I
4
Inductor and capacitor do not consume power. V12 = V22 + V32 + 2V2V3 cos
Therefore, power dissipated in (220)2 = (122)2 + (136)2 + 2 × 122 × 136 × cos
R = Power supplied by the source cos = 0.45
PR = Vs Is cos
16. (b)
= 1 × 2 × cos Given, RL = 5
4
1 RL
= 2× =1W cos =
2 Z
5
13. (d) Z =
= 11.11
0.45
Using KCL, Power consumed by load,
–Is + IRL + IC = 0
2
IC = Is – IRL V3
PL = RL
Z
= 2 2
4 4 2
136
= 2 90° = +j2 A = × 5 =749.1 750 W
11.11
14. (b)
17. Sol.
i = 5 cos(100 t + 100° A)
= 5 100° A 10 A 2A 1 XC1 10 A XL R 10 A
Z = (4 – j3)
= 5 –36.87° Load

v = iZ = 25 63.13° V AC XC2 V 200 V


source
The average power is,
1
P= Vm Im cos Given, total power dissipated in the circuit
2
= 1 kW = 1000 Watt
68 Electronics Engineering Network Theory

22 × 1 + 102 × R = 100 21. (c)


998
or, R= = 9.98 V2
100
Also, voltage drop across R,
VR = IR = 10 × 9.98 V3

= 99.8 Volt
Voltage drop across load, 10°

V = 200 Volt = VR2 + VXL
2 V1

Voltage drop across inductor, 180°


V2 : = = 10°
18 18
VXL = V 2 VR2
180°
V3 : = = 5°
= (200) 2
(99.8) 2 36 36
V2 leads V1 and V3.
= 173.32 Volt
So, V2 is a source, V1 and V3 are absorbing.
VXL
Now, VXL = IXL or Hence, P2 > 0, P1, P3 < 0
I
173.32 22. Sol.
= = 17.332
10 Given that,
XL = 17.332 v(t) = 5 – 10 cos( t + 60°)
i(t) = 5 + X cos( t – 0°)
18. Sol.
Preq = 0
The average power consumed by the load =
1
P = V1I1 cos 1 0 = 5 × 5 + [( 10) ( X )cos(60°)]
2
100 10 1
= cos 60° = 250 W
2 2 –25 = [( 10) ( X ) cos(60°)]
2
X = 10
19. (*)
All answer are wrong, answer given by IISc is (c). 23. Sol.
vc(t) = Vo e–t/
20. Sol. +
Vo = 100 V 0.1 µF 100 V
If we observe the parallel LC combination we 1k
= RC –
get that at = 1000 rad/sec the parallel LC is at 3 –7
= (10 ) (10 )
resonance thus it is open-circuited. The circuit
= 10–4 sec
given in question can be redrawn as,
4
vc(t) = 100 e 10 t V
4 Let the time required by the voltage across the
1
capacitor to drop to 1 V is t1.
5 4
vc(t1) = 100 e 10 t1 , vc (t1 ) = 1 V
4
1 = 100 e 10 t1
10 sin(1000t) 4
e 10 t1 = 0.01
10 sin 1000t t1 = 0.46 msec
So, I= = sin 100t
10
So peak value is 1 Amp.
3 Network Theorems

ELECTRO NICS EN GINEERIN G Q.4 If the secondary winding of the ideal transformer
shown in the circuit of figure has 40 turns, the
(GATE Previous Years Solved Papers)
number of turns in the primary winding for
Q.1 If an impedance ZL is connected across voltage maximum power transfer to the 2 resistor
source V with source impedance Zs, then for will be
maximum power transfer the load impedance
Ideal
must be equal to 8 transformer
(a) source impedance Zs
(b) complex conjugate of Zs
Vg 40 turns 2
(c) real part of Zs
(d) imaginary part of Zs
[EC-1988 : 2 Marks]

Q.2 In the circuit of figure, the power dissipated in (a) 20 (b) 40


the resistor R is a 1 W when only source ‘1’ is (c) 80 (d) 160
present and ‘2’ is replaced by a short. The power [EC-1993 : 1 Mark]
dissipated in the same resistor R is 4 W when
Q.5 In the circuit of figure , when switch S1 is closed,
only source ‘2’ is present and ‘1’ is replaced by
the ideal ammeter M1 reads 5 A. What will the
a short. When both the sources ‘1’ and ‘2’ are
ideal voltmeter M2 read when S1 is kept open?
present, the power dissipated in R will be
(The value of E is not specified).
1 2
M2
Volts

Volts

V
Source R=1 Source
j t

j t

‘1’ ‘2’ 4 3
V1 e

V2 e

S1

(a) 1 W (b) 3 W 6 A M1

(c) 4 W (d) 5 W – +
E 10
[EC-1989 : 2 Marks]
8 5
Q.3 A load, ZL = RL + jXL is to be matched, using an
ideal transformer, to a generator of internal
impedance, Zs = Rs + jXs. The turns ratio of the 2 3
transformer required is
[EC-1993 : 2 Marks]
(a) ZL / Zs (b) RL / Rs
Q.6 A generator of internal impedance, ZG delivers
maximum power to a load impedance, ZL only
(c) RL / Zs (d) RL / Zs
if ZL = ______ .
[EC-1989 : 2 Marks] [EC-1994 : 1 Mark]
70 Electronics Engineering Network Theory

Q.7 The value of the resistance, R connected across (a) 2 (b) 4


the terminals, A and B (ref. figure), which will (c) 8 (d) 16
absorb the maximum power, is [EC-1999 : 2 Marks]

Q.11 Use the data of the Fig. (a). The current ‘i’ in the
3k 4k circuit of the Fig. (b).
R2
A B
R
R1 R3
6k 4k
+
10 V R4 2A

(a) 4.00 k (b) 4.11 k


(c) 8.00 k (d) 9.00 k Fig. (a)

[EC-1995 : 1 Mark] R2

Q.8 Superposition theorem is not applicable to


R1 R3
networks containing
(a) non-linear elements
i=? R4 20 V
(b) dependent voltage sources +
(c) dependent current sources
Fig. (b)
(d) transformers
[EC-1998 : 1 Mark] (a) –2 A (b) 2 A
(c) –4 A (d) +4 A
Q.9 The Thevenin equivalent voltage VTh appearing
[EC-2000 : 2 Marks]
between the terminals A and B of the network
shown in the figure is given by Q.12 In the figure, the value of the load resistor R
3 which maximizes the power delivered to it is
+ 10 1H

100 0° V j2 –j6 +j4 VTh

Em cos10t RL

(a) j16(3 – j4) (b) j16(3 + j4)


(c) 16(3 + j4) (d) 16(3 – j4) (a) 14.14 (b) 10
[EC-1999 : 2 Marks] (c) 200 (d) 28.28
[EC-2001 : 2 Marks]
Q.10 The value of R (in ) required for maximum
power transfer in the network shown in the Q.13 In the network of the figure, the maximum power
figure is is delivered to RL if its value is
5 4
I1

40
+ 0.5I1 20 RL
25 V 20 3A R
– +
50 V

GATE Previous Years Solved Paper 71

40 Q.17 An independent voltage source in series with


(a) 16 (b) an impedance Zs = Rs + jXs delivers a maximum
3
average power to a load impedance ZL when
(c) 60 (d) 20
(a) ZL = Rs + jXs (b) ZL = Rs
[EC-2002 : 2 Marks]
(c) ZL = jXs (d) ZL = Rs – jXs
Q.14 A source of angular frequency 1 rad/sec has a [EC-2007 : 1 Mark]
source impedance consisting of 1 resistance
in series with 1 H inductance. The load that Q.18 For the circuit shown in the figure, the Thevenin
will obtain the maximum power transfer is voltage and resistance looking into X-Y are

(a) 1 resistance. 1
X
(b) 1 resistance in parallel with 1 H
i
inductance.
(c) 1 resistance in series with 1 F capacitor. 2i + 1 2A 2

(d) 1 resistance in parallel with 1 F capacitor.
[EC-2003 : 1 Mark]
Y
Q.15 The maximum power that can be transferred to
the load resistor RL from the voltage source in
4 2
the figure is (a) V, 2 (b) 4 V,
3 3
100 4 2
(c) V, (d) 4 V, 2
3 3
+ [EC-2007 : 2 Marks]
10 V RL

Q.19 The Thevenin equivalent impedance Z Th
between the nodes P and Q in the following
(a) 1 W (b) 10 W circuit is

(c) 0.25 W (d) 0.5 W 1H 1F


[EC-2005 : 1 Mark]

Q.16 For the circuit shown in the figure, Thevenin’s 1 P

voltage and Thevenin’s equivalent resistance


1A 1
at terminals a-b is
10 V Q
5

1A I1

a + 1
(a) 1 (b) 1 + s +
0.5I1 +
– 5 10 V s
b –

1 s2 + s + 1
(c) 2 + s + (d)
s s2 + 2s + 1
(a) 5 V and 2 (b) 7.5 V and 2.5 [EC-2008 : 2 Marks]
(c) 4 V and 2 (d) 3 V and 2.5
Q.20 In the circuit shown, what value of R L
[EC-2005 : 2 Marks]
maximizes the power delivered to RL?
72 Electronics Engineering Network Theory

Vx Q.23 The impedance looking into nodes 1 and 2 in


4
–+ the given circuit is

ib
4 4

Vx + 1k 99ib

Vt 100 V RL
9k
1
100
2
8
(a) 2.4 (b)
3
(a) 50 (b) 100
(c) 4 (d) 6
(c) 5 k (d) 10.1 k
[EC-2009 : 2 Marks]
[EC-2012 : 1 Mark]
Q.21 In the circuit shown below, the Norton
equivalent current in amperes with respect to Q.24 Assuming both the voltage sources are in phase,
the terminals P and Q is the value of R for which maximum power is
transferred from circuit A to circuit B is,
j30
P
2 R

16 0° A 25 –j 50 + +
10 V –j 1 3V
– –
Q
15
Circuit A Circuit B
(a) 6.4 – j4.8 (b) 6.56 – j7.87
(c) 10 + j0 (d) 16 + j0 (a) 0.8 (b) 1.4
[EC-2011 : 1 Mark] (c) 2 (d) 2.8
[EC-2012 : 2 Marks]
Q.22 In the circuit shown below, the value of RL such
that the power transferred to RL is maximum is Q.25 A source vs(t) = V cos100 t has an internal
impedance of (4 + j3) . If a purely resistive
10 10 load connected to this source has to extract the
maximum power out of the source, its value
10 (in ) should be
(a) 3 (b) 4
5V 1A RL
(c) 5 (d) 7
2V
[EC-2013 : 1 Mark]

Q.26 In the circuit shown below, if the source voltage


(a) 5 (b) 10 V s = 100 53.13 V, then the Thevenin’s
equivalent voltage (in Volts) as seen by the load
(c) 15 (d) 20
resistance RL is
[EC-2011 : 1 Mark]
GATE Previous Years Solved Paper 73

3 j4 j6 5 2
+ –
VL1
Vs I1 + + I2 RL = 10
– – 4 0 Vrms j2 RL
j40I2 10 VL1

(a) 100 90° (b) 800 0°


[EC-2015 : 2 Marks]
(c) 800 90° (d) 100 60°
[EC-2013 : 2 Marks] Q.31 In the circuit shown, the Norton equivalent
resistance (in ) across terminals a-b is ____ .
Q.27 Norton’s theorem states that a complex network
2
connected to a load can be replaced with an a
equivalent impedance
(a) in series with a current source +
4I – 2 4
(b) in parallel with a voltage source
I
(c) in series with a voltage source
b
(d) in parallel with a current source
[EC-2015 : 2 Marks]
[EC-2014 : 1 Mark]
Q.32 For the current shown in the figure, the
Q.28 In the figure shown, the value of the current I
Thevenin equivalent voltage (in Volts) across
(in Amperes) is _____ .
terminals a-b is _____ .
5 5
3
a
I

5V 1A 10 12 V 1A 6

[EC-2014 : 1 Mark] [EC-2015 : 1 Mark]

Q.29 In the circuit shown in the figure, the angular Q.33 In the circuit shown in the figure, the maximum
frequency (in rad/sec), at which the Norton power (in Watt) delivered to the resistor R is
equivalent impedance as seen from terminals _______ .
b-b is purely resistive, is ________ . 3k 10 k

1 1F
b
+
+ 5V 2k Vo 100Vo +
– 40 k R
10 cos t (Volts) 0.5 H –

b
[EC-2016 : 2 Marks]
[EC-2014 : 2 Marks]
Q.34 In the circuit shown below, V s is constant
Q.30 In the given circuit, the maximum power voltage source and IL is a constant current load.
(in Watts) that can be transferred to the load The value of IL that maximizes the power
RL is _____ . absorbed by the constant current load is
74 Electronics Engineering Network Theory

R 1 2

+
5V R 1A
Vs IL

(a)

Vs Vs 1 2
(a) (b)
4R 2R
Vs
(c) (d) ? R 5V
R
[EC-2016 : 1 Mark]
(b)
Q.35 Consider the circuit shown in the figure.
(a) 0.5 A (b) 2.5 A
–+ (c) 1 A (d) 2 A
3io
[EC-2019 : 1 Mark]
P
1 io Q.37 In the circuit shown below, the Thevenin voltage
VTh is
10 V 1 1 2V 2 4
+

Q
1 1A 1 2A 2 VTh

The Thevenin equivalent resistance (in )


across P-Q is ______ . –

[EC-2017 : 2 Marks]
(a) 2.8 V (b) 3.6 V
Q.36 Consider the two-port resistive network shown (c) 2.4 V (d) 4.5 V
in the figure. When an excitation of 5 V is [EC-2020 : 1 Mark]
applied across port-1 and port-2 is shorted, the
current through the short-circuit at port-2 is ELECTRICAL EN GINEERIN G
measured to be 1 A [see (a) in the figure]. (GATE Previous Years Solved Papers)
Now, if an excitation of 5 V is applied across
port-2, and port-1 is shorted [see (b) in the SECTIO N - A
figure], what is the through the short-circuit at Q.1 The following circuit shown in figure resonates
port-1? at
1 2 4H 1F
10

Port-1 R Port-2
1F
GATE Previous Years Solved Paper 75

(a) all frequencies (b) 0.5 rad/sec (a) a semicircle with a diameter of V/X.
(c) 5 rad/sec (d) 1 rad/sec (b) a straight line with a slope of R/X.
[EE-1993 : 1 Mark] (c) an ellipse with V/R as major axis.
(d) a circle of radius R/X and origin at (0, V/2).
Q.2 At resonance, the given parallel circuit
[EE-1998 : 1 Mark]
constituted by an iron-cored coil and a capacitor
behaves like Q.6 A circuit with a resistor, inductor and capacitor
in series is resonant at f0 Hz. If all the component
a values are now doubled, the new resonant
frequency is
R (a) 2 f0 (b) still f0
C f0 f0
L (c) (d)
4 2
b
[EE-1998 : 1 Mark]

Q.7 A fixed capacitor of reactance –j0.02 is


(a) an open-circuit connected in parallel across a series
(b) a short-circuit combination of a fixed inductor of reactance
(c) a pure resistor of value R j0.01 and a variable resistance R. As R is varied
(d) a pure resistor of value much higher than R from zero to infinity, the locus diagram of the
admittance of this RLC circuit will be
[EE-1994 : 1 Mark]
(a) a semi-circle of diameter j100 and center at
Q.3 A series RLC circuit has the following parameter zero.
values: R = 10 , L = 0.01 H, C = 100 mF. The (b) a semi-circle of diameter j50 and center at
Q-factor of the circuit at resonance is _____ . zero.
[EE-1995 : 1 Mark] (c) a straight line inclined at an angle.
(d) a straight line parallel to the x-axis.
Q.4 A coil (which can be modeled as a series RL
[EE-1999 : 2 Marks]
circuit) has been designed for high
Q-performance at a rated voltage and a specified Q.8 A series RLC circuit when excited by a 10 V
frequency. If the frequency of operation is sinusoidal voltage source of variable frequency,
doubled and the coil is operated at the same exhibits resonance at 100 Hz and has a 3 dB
rated voltage then the Q-factor and the active bandwidth of 5 Hz. The voltage across the
power P consumed by the coil will be affected inductor L at resonance is
as follows: (a) 10 V (b) 10 2 V
(a) P is doubled, Q is halved.
10
(b) P is halved, Q is doubled. (c) (d) 200 V
2V
(c) P remains constant, Q is doubled.
[EE-1999 : 1 Mark]
(d) P decreased 4 times, Q is doubled.
[EE-1996 : 2 Marks] Q.9 The current in the circuit shown in figure is
20 200 V
I + –
Q.5 A sinusoidal source of voltage V and frequency
f is connected to a series circuit of variable
+
resistance R and a fixed reactance X. The locus 100 V 200 V
50 Hz –
of the tip of the current phasor I as R is, varied
from 0 to is
76 Electronics Engineering Network Theory

(a) 5 A (b) 10 A (a) 2.14 mH (b) 5.30 mH


(c) 15 A (d) 25 A (c) 31.8 mH (d) 1.32 mH
[EE-1999 : 1 Mark] [EE-2003 : 2 Marks]

Q.10 In a series RLC circuit at resonance, the Q.14 The value of Z in figure which is most
magnitude of the voltage developed across the appropriate to cause parallel resonance at
capacitor 500 Hz is
(a) is always zero.
(b) can never be greater than the input voltage. 5

(c) can be greater than the input voltage, 2H Z


however it is 90° out of phase with the input
voltage.
(d) can be greater than the input voltage, and
(a) 125.00 mH (b) 304.20 µF
is in phase with the input voltage.
(c) 2.0 µF (d) 0.05 µF
[EE-2000 : 1 Mark]
[EE-2004 : 1 Mark]
Q.11 In the circuit shown in figure, what value of C
Q.15 The circuit shown in the figure is energized by
will cause a unity power factor at the ac source.
a sinusoidal voltage source V1 at a frequency
which causes resonance with a current of I.

230 V C ZL = 30 40°
50 Hz I

V2
(a) 68.1 µF (b) 165 µF
(c) 0.681 µF (d) 6.81 µF
V1
[EE-2002 : 2 Marks] Vc

Q.12 A series RLC circuit has R = 50 , L = 100 µH


and C = 1 µF. The lower half power frequency of
The phasor diagram which is applicable to this
the circuit is
circuit is
(a) 30.55 kHz (b) 3.055 kHz
I V2
(c) 51.92 kHz (d) 1.92 kHz V1
[EE-2002 : 2 Marks]
(a)
Q.13 In the circuit of figure, the magnitudes of VL and
Vc
VC are twice that of VR. Given that, f = 50 Hz, the
inductance of the coil is
5 I
V2
VR
C VC (b) V1

5 0°
L VL
Vc
GATE Previous Years Solved Paper 77

Vc Im

(c) V1
(c)
V2 I
Re

V1
Im

V2

(d)
(d)
I

Re

Vc

[EE-2006 : 2 Marks] [EE-2007 : 2 Marks]

Q.16 The RLC series circuit shown is supplied from Q.17 In the figure given below all phasors are with
a variable frequency voltage source. The reference to the potential at point ‘O’. The locus
admittance locus of the RLC network at of voltage phasor VYX as R is varied from zero to
terminals AB for increasing frequency is infinity is shown by

A R

V 0° R

L VYX

X Y

V 0° C
B C

Im
O

O 2V Locus of VYX

(a) Re VYX
(a) V YX (b)

Locus of VYX 2V O

Locus of VYX
O 2V
Im
VYX VYX
(c) (d)
O 2V
Locus of VYX
(b) Re
[EE-2007 : 2 Marks]

Q.18 The resonant frequency for the given circuit will


be
78 Electronics Engineering Network Theory

0.1 H z

Inductive

1F 1
(a) f

(a) 1 rad/sec (b) 2 rad/sec Capactive

(c) 3 rad/sec (d) 4 rad/sec


[EE-2008 : 2 Marks] z
Inductive
Q.19 Two magnetically uncoupled inductive coils
have Q-factors q1 and q2 at the chosen operating
(b) f
frequency. Their respectively resistances are R1
and R2. When connected in series, their effective Capactive
Q-factor at the same operating frequency is

1 1
(a) q1 + q2 (b) + z
q1 q2
Inductive
q1 R1 + q2 R2 q1 R2 + q2 R1
(c) (d) Capacitive
R1 + R2 R1 + R2 (c)
[EE-2013 : 2 Marks]
f
Q.20 A series RLC circuit is observed at two
frequencies. At 1 = 1 k-rad/s, we note that
source voltage V1 = 100 0° V results in a current z

I1 = 0.03 31° A.
At 2 = 2 k-rad/s the source voltage Inductive
(d) f
V2 = 100 0° V results i a current I2 = 2 0° A. Capacitive
The closest values for R, L, C out of the following
options are:
(a) R = 50 , L = 25 mH, C = 10 µF
[EE-2015 : 1 Mark]
(b) R = 50 , L = 10 mH, C = 25 µF
(c) R = 50 , L = 50 mH, C = 5 µF Q.22 The circuit below is excited by a sinusoidal
(d) R = 50 , L = 5 mH, C = 50 µF source. The value of R in , for which the
admittance of the circuit becomes a pure
[EE-2014 : 2 Marks]
conductance at all frequencies is _______ .
Q.21 An inductor is connected in parallel with a
100 µF R
capacitor as shown in the figure.

L
i
Z 0.02 H R
C

As the frequency of current i is increased, the


impedance (z) of the network varies as, [EE-2016 : 2 Marks]
GATE Previous Years Solved Paper 79

Q.23 In the balanced 3-phase, 50 Hz circuit shown i (t )


below, the value of inductance (L) is 10 mH.
The value of the capacitance (C) for which all v(t)

the line currents are zero, in milli-farads, is (a) (0, –5)


________ .

L C L
i(t)

C C

L
(b) v(t)
(5, 0)
[EE-2016 : 2 Marks]

Q.24 The voltage v(t) across the terminals a and b


shown in the figure, is a sinusoidal voltage i (t )
having a frequency = 100 rad/sec. When the
inductor current i(t) is in phase with the voltage
(c)
v(t), the magnitude of the impedance Z (in ) (0, 5)

seen between the terminals a and b is ______


v(t)
(upto 2 decimal places).

i (t ) L i (t )
a
+

V(t )
100 µF 100 (d) v(t)
(–5, 0)
Z

b

[EE-2018 : 2 Marks] [EE-2018 : 2 Marks]

Q.25 A dc voltage source is connected to a series L-C SECTIO N -B


circuit by turning on the switch S at time t = 0 as
Q.1 In the following circuit, i(t) under steady-state
shown in the figure. Assume i(0) = 0, v(0) = 0.
is
Which one of the following circular loci
i( t) 1 2H
represents the plot of i(t) versus v(t)?

S i(t) 5V

+ 1F
L=1H
t=0 10 sint
+ –
5V C=1F v (t )
– (a) zero (b) 5
(c) 7.07 sint (d) 7.07 sin(t – 45°)
[EE-1993 : 1 Mark]
80 Electronics Engineering Network Theory

Q.2 Superposition principle is not applicable to a (a) 56.66 45° (b) 60 30°
network containing time-varying resistors. (c) 70 30° (d) 34.4 65°
(True/False) [EE-2003 : 1 Mark]
[EE-1994 : 1 Mark]
Q.6 Two a.c. sources feed a common variable
Q.3 For the circuit shown in figure. The Norton resistive loads as shown in figure. Under the
equivalent source current values is ________ . maximum power transfer condition, the power
A and its resistance is ______ . absorbed by the load resistance RL is
3
A 6 j8 6 j8

3
6V 3 110 0° V i1 RL i2 90 0° V

[EE-1997 : 2 Marks] (a) 2200 W (b) 1250 W


(c) 1000 W (d) 625 W
Q.4 Viewed from the terminals A and B, the
[EE-2003 : 2 Marks]
following circuit shown in figure can be reduced
to an equivalent circuit of a single voltage source Q.7 In the given figure, the Thevenin’s equivalent
in series with a single resistor with the following pair (voltage, impedance), as seen at the
parameters. terminals P-Q, is given by
A
10
P
10 V 4
Unknown
20 4V 10
network
6 5V

B Q

(a) 10 Volt source in series with 10 resistor.


(a) (2 V, 5 ) (b) (2 V, 7.5 )
(b) 7 Volt source in series with 2.4 resistor.
(c) (4 V, 5 ) (d) (4 V, 7.5 )
(c) 15 Volt source in series with 2.4 resistor.
[EE-2005 : 2 Marks]
(d) 1 Volt source in series with 10 resistor.
[EE-1998 : 2 Marks] Q.8 In the figure the current source is 1 0 A, R = 1 ,
the impedance are ZC = –j and ZL = 2 j . The
Q.5 In the figure, Z 1 = 10 –60°, Z 2 = 10 60°,
Thevenin equivalent looking into the cicuit
Z3 = 50 53.13°. Thevenin impedance seen from
across XY is,
X-Y is
X
Z1 Z3
X

+
100 0° Z2

Y
Y
GATE Previous Years Solved Paper 81

(a) 5 2 0° V, (1 + 2 j ) (a) Zero (b) 3


(c) 6 (d) Infinity
(b) 2 45° V, (1 – 2j)
[EE-2011 : 1 Mark]
(c) 2 45° V, (1 + j)
Q.13 The impedance looking into nodes 1 and 2 in
(d) 2 45° V, (1 + j ) [EE-2006 : 1 Mark]
the given circuit is
Q.9 The Thevenin’s equivalent of a circuit operating ib
at = 5 rad/sec has VOC = 3.71 –15.9° V and
Z0 = 2.38 – j0.667 . At this frequency, the 1k 99ib
minimal realization of the Thevenin’s
impedance will have a 9k
1
(a) resistor and a capacitor and an inductor.
100 k
(b) resistor and a capacitor. 2
(c) resistor and an inductor.
(d) capacitor and an inductor. (a) 50 (b) 100
[EE-2008 : 1 Mark] (c) 5 k (d) 10.1 k
[EE-2012 : 1 Mark]
Statement for Linked Answer Questions (10 and 11):
2k
3VAB Q.14 Assuming both the voltage sources are in phase,
+– A the value of R for which maximum power is
transferred from circuit A to circuit B is
5V 2k 1k
2 R

B
+ +
Q.10 For the circuit given above, the Thevenin’s 10 V –j1 3V

resistance across the terminals A and B is – –

(a) 0.5 k (b) 0.2 k


Circuit A Circuit B
(c) 1 k (d) 0.11 k
[EE-2009 : 2 Marks] (a) 0.8 (b) 1.4

Q.11 For the circuit given above, the Thevenin’s (c) 2 (d) 2.8
voltage across the terminal A and B is [EE-2012 : 2 Marks]
(a) 1.25 V (b) 0.25 V Q.15 A source vs(t) = V cos100 t has an internal
(c) 1 V (d) 0.5 V impedance of (4 + j3) . If a purely resistive
[EE-2009 : 2 Marks] load connected to this source has to extract the
maximum power out of the source, its value
Q.12 In the circuit given below, the value of ‘R’
required for the transfer of maximum power to (in ) should be
the load having a resistance of 3 is (a) 3 (b) 4
(c) 5 (d) 7
[EE-2013 : 1 Mark]

Q.16 In the circuit shown below, if the source voltage


+ 6
10 V 3 Load V s = 100 53.13° V, then the Thevenin’s

equivalent voltage (in Volts) as seen by the load
resistance RL is
82 Electronics Engineering Network Theory

3 j4 j6 3 Q.20 The voltage across the capacitor, as shown in


+ – the figure, is expressed as:
VL1
+ 10V Vc(t) = A1 sin( 1t – 1) + A2 sin( 2t – 2)
Vs
I1 40I2 +
– – L1 I2 RL = 10

1 1H

(a) 100 90° V (b) 800 0° V


(c) 800 90° V (b) 100 60° V 20 sin10t V c(t) 1F 10 sin5t

[EE-2013 : 2 Marks]

Q.17 Assuming an ideal transformer, the Thevenin’s


equivalent voltage and impedance as seen from The values of A1 and A2 respectively, are
the terminals x and y for the circuit in figure are (a) 2.0 and 1.98 (b) 2.0 and 4.20
1 (c) 2.5 and 3.50 (d) 5.0 and 6.40
x
[EE-2014 : 2 Marks]

Q.21 For the given circuit, the Thevenin equivalent is


sin( t)
to be determined. The Thevenin voltage, VTh
(in Volt), seen from terminal AB is ______ .
y
1:2 20 i
1
(a) 2 sin( t), 4 (b) 1 sin( t), 1 –+ A
i
(c) 1 sin( t), 2 (d) 2 sin( t), 0.5
+
[EE-2014 : 1 Mark] 2V 1 2

Q.18 A non-ideal voltage source Vs has an internal
impedance of Zs. If a purely resistive load is to B

be chosen that maximizes the power transferred [EE-2015 : 1 Mark]


to the load, its values must be
Q.22 The circuit shown in the figure has two sources
(a) 0
connected in series. The instantaneous voltage
(b) real part of Zs of the AC source (in Volt) is given by
(c) magnitude of Zs V(t) = 12 sint. If the circuit is in steady-state.
(d) complex conjugate of Zs Then the rms value of the current (in Ampere)
[EE-2014 : 1 Mark] flowing in the circuit is ______ .
Q.19 The Norton’s equivalent source in amperes as
seen into the terminals X and Y is ______ . V(t ) 1
2.5 V
X +
1H
8V

2.5
[EE-2015 : 2 Marks]
5 Q.23 In a linear two-port network, when 10 V is
5 5 applied to port-1, a current of 4 A flows through
port-2 when it is short circuited. When 5 V is
5V applied to port-1, a current of 1.25 A flows
Y through a 1 resistance connected across
[EE-2014 : 2 Marks] port-2.
GATE Previous Years Solved Paper 83

When 3 V is applied to port-1, the current Q.27 For the given two-port network, the value of
(in Ampere) through a 2 resistance connected transfer impedance Z21 (in ) is ______ .
across port-2 is ______ . 2
[EE-2015 : 2 Marks]

Q.24 In the circuit shown below, the maximum power 1 2


4 2
transferred to the resistor R is ______ W.
2
3
1 2

5 6V [EE-2017 : 1 Mark]

Q.28 The current I flowing in the circuit shown below


(in Ampere), is ______ .
5V R 5 2A

I
50 40 25 20
[EE-2017 : 2 Marks]
20
Q.25 In the circuit shown below, the value of capacitor
200 V 160 V 100 V 80 V
C required for maximum power to be transferred
to the load is
[EE-2019 : 2 Marks]
Rs = 0.5
Q.29 The Thevenin equivalent voltage, VTh (in Volt)
(Rounded of to 2 decimal places) of the network
5 mH
shown below, is ______ .
V(t) = 10 sin(100t) 2 3
1 C
+

4V 3 5A VTh
Load

(a) 1 nF (b) 1 µF

(c) 1 mF (d) 10 mF
[EE-2017 : 2 Marks] [EE-2020 : 1 Mark]

Q.26 For the network given in figure below, the Q.30 A benchtop dc power supply acts as an ideal
Thevenin’s voltage Vab is 4 A current source as long as its terminal voltage
is below 10 V. Beyond this point, it begins to
10 10 behave as an ideal 10 V voltage source for all
load currents going down to 0 A. When
a connected to an ideal rheostat, find the load
6A 5 10 16 V resistance value at which maximum power is
b transferred, and the corresponding load voltage
and current.
(a) 2.5 , 4 A, 10 V (b) 2.5 , 4 A, 5 V
(a) –1.5 V (b) –0.5 V
(c) Open, 4 A, 0 V (d) Short, A, 10 V
(c) 0.5 V (d) 1.5 V
[EE-2020 : 2 Marks]
[EE-2017 : 2 Marks]
84 Electronics Engineering Network Theory

Electronics & Electrical Engineering


GATE Previous Years Solved Paper

A n swe rs & Expl a n a t i o n s

Answers
EC Network Theorem

1. (b) 2. (a) 3. (a) 4. (c) 5. (5) 6. (Sol.) 7. (a) 8. (a)

9. (a) 10. (c) 11. (c) 12. (a) 13. (a) 14. (c) 15. (c) 16. (b)

17. (d) 18. (d) 19. (a) 20. (c) 21. (a) 22. (c) 23. (a) 24. (a)

25. (c) 26. (c) 27. (d) 28. (0.5) 29. (2) 30. (1.649) 31. (1.33) 32. (10)

33. (0.8) 34. (b) 35. (–1) 36. (c) 37. (b)

Solutions
EC Network Theorem

1. (b) 2
n2 2 1
According to maximum power transfer theorem, = =
n1 8 4
ZL = ZS
n2 1
=
2. (a) n1 2
P1 = 1 W; P2 = 4 W n1 = 2n2
Since the polarity of both the sources are = 2 × 40 = 80
different,
5. Sol.
P = ( P1 P2 )2 Across switch S1,
2 2 ISC = 5 A
P= ( 1 4) = (1 2)
P = 1W RTh = [(4 6 + 2 8) + 3 + 3] 10 + 5
3. (a) RTh = (2.4 + 1.6 + 3 + 3) 10 + 5
2
ZL n2 = 10 10 + 5 = 5 + 5
=
ZS n1 RTh = 10
VOC = VAB = ISC RTh = 5 × 10 = 50 V
n2 ZL
=
n1 ZS 6. Sol.

4. (c) ZL = ZG
2 Z G = RG + jXG
n2 ZL
= ZL = RG – jXG
n1 ZS
GATE Previous Years Solved Paper 85

7. (a) current I in loop B, then interchanging positions


Maximum power will be absorbed by R when an identical source in loop B produces the same
R = RTh. current in loop A”. Since network is linear,
principle of homogeneity holds and so when
R AB = RTh = (3 6) + (4 4) volt source is doubled, current also doubles with
RTh = R = 2 + 2 = 4 k opposite direction.

8. (a) 12. (a)


Superposition theorem is applicable for linear XS = L = 10
network. ZS = 10 + j10

9. (a) R for max power transfer = Zs = 10 2

100 0° 3 = 10 + 10 j = 10 2 45°
A
= 14.14

100 0° V j2 VTh 13. (a)


–j6 +j4
For maximum power delivered to RL, open
circuit RL,
B
A I1
V = 100 0°
4j
VTh = 100 0° 40
3+ 4j 0.5I1 20
100 × 4 j(3 4 j )
= 50 V
25
VTh = 16j(3 – 4j) B
RTh across AB,
10. (c)
1 I1
For MPT, R should be equal to Req of the circuit
seen from the terminal after removing R. I1
Deactivating voltage and current sources.
0.5I1 20 40 V
5 4

20
KCL at node 1,
V V
0.5I1 + I = +
20 40
V V V
0.5 +I = +
R = (5 20) + 4 = 4 + 4 40 20 40
=8
1 1 1
I= V +
11. (c) 20 40 80
This is a reciprocal and linear network. V
= RTh = 16
According to reciprocity theorem which states I
“Two loops A and B of a network N and if an RL = RTh = 16
ideal voltage source E in loop E produces a
86 Electronics Engineering Network Theory

14. (c) 1 1 1 2
So, 2 = VTh + +
ZL = Rs – jXs 2 1 1 1
ZL = 1 – 1j VTh = 4 Volt
From the figure Isc = 2 Ampere
15. (c)
For maximum power transfer, VTh 4
So, RTh = = =2
RL = Rs = 100 I sc 2

V2 5× 5 19. (a)
P= = = 0.25 W
R 100
P 1/s
(V across RL = 5 V) s

16. (b)
For VTh : 1 1
Vab Vab 10
+ =1
5 5
(when current source is in series with voltage
Q
source effect of current source is taken),
2Vab = 15 Vab = 7.5
1
(s + 1) 1 +
0.5I 1 5 1 s
= R = 10 ZTh = (s + 1) 1+ =
I1 R s 1
(s + 1) + 1 +
(Impedance connected to 0.5I1 current source) s
For RTh, make independent sources dead,
(s + 1)2 / s
5 = =1
a (s + 1) (s + 1)/ s

20. (c)
10 5
Vx
4 I1
–+

b
4 4 I
RTh = 5 5 = 2.5 –
Vx +

17. (d) 1V
ZL = Rs – jXs 1
Req =
I
For maximum power transfer,
ZL = Zs For Pmax : RL = Req
ZL = Rs – jXs KVL, 1 = 4I1 + Vx ...(i)
Vx = 4(I – I1) ...(ii)
18. (d) 1
Simplifying, Req = =4
For VTh, applying KCL at node X, I
VTh VTh VTh 2i
2 = + + 21. (a)
2 1 1
For Norton equivalent current short circuiting
VTh
where, i= the terminal PQ.
1
GATE Previous Years Solved Paper 87

j30 A iAB B
Isc Isc 1
P
ib iA
I=0

(9 + 1) k 100 99ib V=1V


16 0° A 25 –j 50

2
Q
15 To find Thevenin impedance across node
Short-circuit current, 1 and 2. Connect a 1 V source and find the
current through voltage source.
25
ISC = × 16 0° 1
15 + j 30 + 25 Then, ZTh =
I
25 (25 × 16) 0° By applying KCL at node B and A,
= × 16 0° =
40 + j 30 50 36.86 iAB + 99ib = I
= 8 –36.86° ib = iA + iAB
Hence Norton current is, ib – iA + 99ib = I
IN = ISC = 8 –36.86° 100ib – iA = I ...(i)
IN = (6.4 – j4.8) A By applying KVL in outer loop,
10 × 103 ib = 1
22. (c)
ib = 10–4 A
For maximum power transfer, and 10 × 103 ib = –100 iA
RL = RTh i A = –100 ib
To calculate R Th deactivate all the energy From equation (i),
sources. 100ib + 100ib = I
10 10 I = 200ib
= 200 × 10–4 = 0.02
10 1 1
ZTh = = = 50
I 0.02
O.C.

S.C. S.C. RTh 24. (a)


Redrawing the diagram,

i 2 R i1
RTh = 10 + 10 10 = 15

23. (a) + +
10 V –j 1 3V
ib – –

1k 99ib Circuit A Circuit B

Current through R will be


9k
1 10 3 7
i = = A
100 2+R 2+R
2
Current through 3 V source is,
3
i1 = i = i 3j
j1
88 Electronics Engineering Network Theory

So power delivered to circuit B by circuit A is, 27. (d)


P = i2R + i1 × 3 Norton’s theorem states that a complex network
7 2
7 connected to a load can be replaced with an
P= R+ 3j 3 equivalent impedance in parallel with a current
2+R 2+R
source.
P
For P to be maximum will be zero,
R
P
=0 Complex
RN RL
R network
RL IN

2
7 98 R 21
3 =0
2+R (2 + R ) (2 + R )2
49(2 + R) – 98R – 21(2 + R) = 0
28. Sol.
98 – 42 = 49R + 21R
5 5
56
R= = 0.8
70 I

25. (c) 5V 1A 10
For pure resistive load to extract the maximum
power,
2 2
RL = Zs = Rs + Xs Using superposition theorem:
When 5 V source acting alone, we get
= 4 2 + 32 = 5
5 5
26. (c)
I1
To find VTh, open-circuit the load voltage RL
then, 5V 10

3 j4 j6 5
+ –
I2 = 0
VL1
Vs I1 j40I2 +

+
– 10 VL1 VTh V 5 1
I1 = = = A ...(i)
Req 10 + 5 + 5 4
When 1 A source acting alone, we get
I2 = 0 5 5
j40I2 = 0
Vs ( j 4) I2
VL1 =
3 + j4
1A 10
100 53.13°
= × 4 90°
5 53.13°
VL1 = 80 90°
1× 5 5 1
I2 = = = A ...(ii)
VTh = 10 VL1 + I 2 j6 + I 2 3 5 + 10 + 5 20 4
VTh = 10 × 80 90° + 0 × j6 + 0 × 3 1
Therefore, I = I1 + I 2 = A = 0.5 A
VTh = 800 90° V 2
GATE Previous Years Solved Paper 89

29. Sol.
1.414 45°
R 1/j
b 2.828 45° 1.414
I

j 0.5

2.828 45°
I= = 1.08 22.5°
b Zbb 1.414 45° + 1.414

Finding ZN : 2
Power = I2R = (1.08) × 2 = 1.649 W
1 × j 0.5 1 j 1
Zbb = + = + 31. Sol.
1 + j 0.5 j 2+ j j
2 2 Io
2 +j a
or, Zbb = 2 ...(i)
2j
Retionalizing equation (i), we get, 4I + 2 4 Vo

2 2
(2 )+ j j2 I1 I
Zbb = 2
× 2 b
2j j2
Vo
2 2 + 4 +2 2 ( 3 4 ) I=
+j 4 4
= 4
+2 2 +2 2 Vo
I1 =
In order to have a purely resistive impedance 2
Zbb the imaginary part of equation (ii) will be Applying KCL,
Vo
4 I Vo Vo
equaled to zero. + + = Io
2 2 4
4 + 3 From there,
4 =0
+2 2 3
or, 3 = 4 Vo = Io
4
or, = 4 = 2 rad/sec. Vo 4
RN = = = 1.33
Io 3
30. Sol.
32. Sol.
2
3 VA
a

4 0 Vrms j2 RL
12 V 1A 6

For maximum power transfer, b

RL = ZTh = 2 j2 VTh = V6
VA 12 VA
2 × j2 + =1
= = 1.414 3 6
2 + j2
1 1
8 90° VA + = 1+4
VTh = = 2.828 45° 3 6
2 + j2
90 Electronics Engineering Network Theory

3
VA =5 –+
6
3io
VTh = V6 = VA
= 10 V P
1 io
33. Sol.
1 1
3k 10 k

+ + Q
5V 2k Vo 100Vo +
– 40 k VTh 1 RTh
– –

• It can be further reduced as follows:

For maximum power transfer,


–+
R = RTh
3io
2k P
Vo = 5 × =2V
5k 1 +
io
From output loop,
Vx 1A
40 k 1
VTh = 100 × 2 ×
50 k

VTh = 160 V Q
1 Vx
RTh =
and RTh = 10 k 40 k 1A

10 × 40
= =8k 3io
50
–+
2
VTh
Max. power = (1 – 4io)
4RTh 1 P

160 × 160 3io + io
1A +
= = 0.8 W
4 × 8000 +
L Vx 1A
Vx 1
34. (b) –
1 –
In maximum power transformation, half of the
+ – (1 – i ) 1A Q
voltage drops across source resistance, (1 – io) o

remaining half across the load.


• By applying KVL in the Loop L,
Voltage across source (R),
Vx = 3io + (1 – io)
V
ILR = s = 2io + 1
2
Also, Vx = io (1 )
Vs
IL = • So, 2io + 1 = io
2R
io = –1 A
35. Sol.
and Vx = –1 V
• The equivalent circuit to calculate the
Vx
Thevenin equivalent resistance (RTh) is as So, RTh = = 1
1A
follows:
GATE Previous Years Solved Paper 91

36. (c) 4

According to reciprocity theorem:


In a linear bilateral single source network the
ratio of response to excitation remains the same 1A 3 2A 2
even after their positions get interchanged.
I 1
= I =1A
5 5
4
37. (b)
By applying source transformation, 6/5

2V VTh
1 2 4
3.6 V

1V 2A 2 VTh = 3.6 V

Answers
EE Network Theorem (Section-A)

1. (b) 2. (d) 3. (0.032) 4. (d) 5. (a) 6. (d) 7. (a) 8. (d)

9. (a) 10. (c) 11. (a) 12. (b) 13. (c) 14. (d) 15. (a) 16. (d)

17. (a) 18. (c) 19. (c) 20. (b) 21. (b) 22. (14.14) 23. (3.03) 24. (50)

25. (b)

Solutions
EE Network Theorem (Section-A)

1. (b) 2. (d)

j j 1 R j L 1
Z = 10 + j 4 Y= × +
R+ j L R j L jXc

R j L j
1 Y= +
4 2
2
R + ( L) 2 Xc
= 10 j
2 Imaginary parts are equal to zero for resonance,
4
L
= C
For circuit to be in resonance imaginary part of R + ( L )2
2

Z must be equal to zero. From this we get ‘ o’


1 At resonance,
Hence, 4 2 =0
res R 1 R 2 + 2o L2
Y= = =
res = 0.5 rad/sec. R2 + 2 2
oL Y R
Z>>R
92 Electronics Engineering Network Theory

3. Sol. V
For R = 0, I = 90°
Q0 = 0.032 X
For series RLC circuit V
For R = X, I = 45°
L 2X
Q-factor at resonance = o
R For R = , I = 0 0°
1 1 On plotting these three points we get,
o =
=
LC (0.01) × (100 × 10 3 )
Im
= 10 10 rad/sec.
R=
Re
L 10 0 × 0.01
Q= o = = 0.032
R 10

4. (d)
V/X R=X
L
Q=
R
When frequency of operation is doubled,
= 2 f, also get doubled
R=0
Consequently, Q also get doubled
2
2 V Hence locus of I is a semi-circle having
P= I R R
2 2 diameter of V/X.
R + ( L)

6. (d)
V2 V2
= =
L
2 R (1 + Q 2 ) 1
R2 1 + f0 =
2 LC
R
(for series RLC resonance)
Q It is given that Q is high.
1
Q2 >> 1 fnew =
2 2 L × 2C
V2 (when all the components values are doubled)
P
RQ 2 f0
Hence, fnew =
Q Q is doubled. 2
P decreased 4 times. 7. (a)
5. (a)
–j 0.02
I R jX
C
R j0.01

V, f
YAB

A B
V
I= 1 1
R + jX Y AB = +
j 0.02 R + j 0.01
V 1 X
= tan For R = 0, Y AB = –j50 = 50 –90°
R2 + X 2 R
GATE Previous Years Solved Paper 93

For R = 0.01, Y AB = 50 11. (a)


For R = , Y AB = j50 = 50 90°
1
On plotting these three points, Y = j C+
30 40°
Im (YAB) = j C + 0.0255 – j0.0214
R= = 0.0255 + j( C – 0.0214)
j50
= Real(Y) + jImg(Y)
To have a unity power factor at ac source i.e.
resonance condition,
50
j100 0 YAB Img(Y) = 0
R = 0.01
C – 0.0214 = 0
= 2 × 50
0.0214
–j50 C= = 68.1 µF
R=0 100

12. (b)
Hence, locus of YAB is a semicircle of diameter
1 1
= = = 10 5 r/s
j100 and center at zero. o LC 100 × 10 6
× 10 6

8. (d) R 50
= = 6
= 50 × 10 4 r/s
L 100 × 10
Resonance frequency
Q=
Bandwidth 2
2
f 0 100 lower = 0 +
= = = 20 2 2
f 5
At resonance, 2
5 × 105 5 × 105
VL = VC = Q Vsource = (105 )2 +
2 2
VL = 20 × 10 = 200 V
5
= 10 1 + 6.25 2.5
9. (a)
= 0.193 × 105 rad/sec
Q VL = –VC (Given)
Hence,
So, this is a case of RLC series resonance.
V lower 0.193 × 10 5
Hence, I= (at resonance) flower = =
R 2 2
100 = 3065 Hz 3.055 kHz
= =5A
20 13. (c)
10. (c) V = VR + i(VL – VC)
In a series RLC circuit, at resonance Since, VL = VC and VL
VL = jQVsource
= 2 VR
and VC = –jQVsource
Therefore, the circuit is at resonance and
Also for Q > 1,
VR = V
VC = Vsource
VL VL 2VR
Hence option (c) is correct. Quality factor = = = =2
V VR VR
94 Electronics Engineering Network Theory

As we know, Voltage across the capacitor,


oL 1
Q= VC = I × XC = I ×
R j C
2 f ×L I
2 = L = 31.8 mH VC = 90°
5 C
So, VC lags the current by 90°.
14. (d)
The phasor diagram on the basis of above
At resonance, the circuit should be in unity
analysis.
power factor.
I V2
Hence ‘Z’ should be capacitive. V1
Admittance of the parallel circuit,
1 1 Vc
Y= + =0
jL 1/ jC
16. (d)
1
+C =0 Admittance of the series connected in RLC,
L
1 1
C= Y=
L× 2 1
R+ j L
C
1
= = 0.05 µF
2 × (2 × 500)2 R j L
1
C
Y=
15. (a) 1 2
R2 + L
C

I I [By rationalization]
Separating, real and imaginary part of
RA RB
admittance,
V2 R
XL Re[Y] = 2
1
V1
R2 + L
L
XC VC
For any value of , the real part of always
positive.
Z = RA + RB + j(XL – XC) 1
At resonance, XL = XC When, L=
C
So, Z = RA + RB 1
Therefore, input impedance is purely resistive, At, o = (Resonance)
LC
is minimum, and the input voltage and output 1
current are in phase. Re[Y] = (Maximum value)
R
So, V1 and I are in phase.
1
V1 L
V2 = × [ RB + j( XL XC )] C
RA + RB + j( XL XC ) Im(Y) =
1 2
R2 + L
But, XL = XC C
V1 1
V2 = × RB L
RA + RB C
= 2
Therefore, V2 is in phase with V1 and V2 < V1. 1
R2 + L
C
GATE Previous Years Solved Paper 95

1 R + jXC
At, o = (Resonance) VYX = V
LC R jXC
Imaginary part of zero
When, R=0
Im(Y) = 0
For, 0 < < o 0 + jXC
VYX = V =V
0 jXC
1
> L
C XC
1+ j
Therefore, Im[Y] > 0 R
VYX = V×
For, XC
o< < 1 j
1 R
< L When, R
C
Therefore, Im[Y] < 0 VYX = –V
On the basis of above analysis, the admittance
18. (c)
locus is,
Input impedance,
Im
1
z = j L+R
j C
=0
=
Re R
z = j L+
1
1 + j RC
o =
LC 0.1 H

17. (a)
Z 1F 1

+ I +
V 0° R 1 1 j
z = j 0.1 + ×

V yx
– 1+ j 1 j
1 j
x y = j 0.1 + 2
+ 1+
V 0° C
1
– = + j × 0.1
1+ 2 1+ 2
O At resonance, imaginary part must be zero,

Let capacitive reactance = XC 0.1 2 =0


1+ 1
V 0° + V 0° 2V 0.1 =
I= = 1+ 2
R jXC R jXC 2 + 1 = 10
Using KVL, 2 = 9

VYX + IR – V = 0 = 3 rad/sec
VYX = V – IR
19. (c)
2V
VYX = V R L1
R jXC q1 =
R1
V ( R + jXC ) q 1 R1
= L1 =
( R jXC )
96 Electronics Engineering Network Theory

q 2 R2 XC XL
Similarly, L2 = or, tan(–31°) =
L1 + L2 R
or, XL – XC = R tan(–31°)
(L1 + L2 )
Q= R1 + R2 = 50 X – 0.6 = –30
R1 + R2
XL XC = –30 ...(iii)
1 1
q1 R1 + q2 R2
= (at 1 = 1 k-rad/sec)
R1 + R2
Also at, w2 = 2 k-rad/sec
20. (b) 1
XL = XC or 2L =
2
2C
2
Given:
At, 1 = 1 k-rad/sec 1
or, L= 2 ...(iv)
V1 = 100 0° V, 2C
I1 = 0.03 31° A From equation (iii),
At, 2 = 2 k-rad/sec 1
V2 = 100 0° V 1L = –30
1C
I2 = 2 0° A
1 1
R L C or, 1 2 = –30 (Using (iv)
2C 1C

1 × 10 3 1
I or, 6
= –30
4 × 10 C 10 3 C
10 3 10 3
or, = –30
V 4C C
At 2 = 2 k-rad/sec, voltage and current are in 3 3
phase. or, × 10 = –30
4C
Thus, it is case of series resonance,
3 × 10 3
XL or, C=
= XC 4 × 30
2 2
or, C = 25 × 10–6 F = 25 µF
V 100 0°
Z = R= 2 = = 50 Substituting the value of C in equation (iv), we
I2 2 0°
get,
Resistance of circuit,
1 1
R = 50 L= 2
= 3 2 6
2C (2 × 10 ) × 25 × 10
Now at, 1 = 1 k-rad/sec
1
V1 100 0° = = 10 mH
Z= = 100
I 1 0.03 31°
Therefore, values are
100 R = 50 , L = 10 mH, C = 25 µF
= 31° ...(i)
0.03
21. (b)
1 XL XC
Also, Z = Z tan ...(ii) L
R
(at 1 = 1 k-rad/sec)
Comparing equations (i) and (ii), we have

1 XL XC
–31° = tan
R C
GATE Previous Years Solved Paper 97

1
=
L /C j L R1 Xc2
Z= j L = Rea of Zeq =
j C j L+ 1 1 2
LC R12 + Xc2
j C
100 × 100 × 100
For, 1 > 2LC, Z = +ve = = 50
2LC, 100 2 + 100 2
For, 1 < Z = –ve
25. (b)
Z
Inductive s I (s )

f +
5/s V(s ) 1/s

Capactive

5 /s 5
I(s) = =
22. Sol. 1 s2 + 1
s+
s
The resonance frequency for the circuit is
i(t) = 5 sint
1 RL2 L /C t
= 1 t
0 LC RC2 L /C v(t) = i dt = 5sin t dt
C 0
Since, (RL = RC = R) 0

So the circuit will have zero real part of v(t) = 5[ cos t ]t0 = 5[ cos t + 1]
admittance.
v(t) = 5 – 5 cost
L 0.02
When, R= = = 14.14 i
C 100 µF
t = T/4
5
23. Sol.
t = T/2
Using star to delta conversion, t=0
v
t=T 5 10

C
L L L C/3 L –5
3T/4
C/3
C C C/3

L L
t i(t ) v(t )
Line current will be zero when the parallel pair 0 0 0
of induction-capacitor is resonant at f = 50 Hz. T
5 5
1 4
So, 50 × 2 = T
LC /3 0 10
2
1 3T
100 = 5 5
LC /3 4
C will be 3.03 mF. T 0 0

24. Sol.
At resonance imaginary part of Zeq = 0
98 Electronics Engineering Network Theory

Answers
EE Network Theorem (Section-B)

1. (d) 2. (False) 3. (2, 4.5) 4. (b) 5. (a) 6. (d) 7. (a) 8. (d)

9. (b) 10. (b) 11. (d) 12. (a) 13. (a) 14. (a) 15. (c) 16. (c)

17. (a) 18. (c) 19. (1) 20. (a) 21. (3.36) 22. (10) 23. (0.545) 24. (3.025)

25. (d) 26. (a) 27. (3) 28. (0) 29. (14) 30. (a)

Solutions
EE Network Theorem (Section-B)

1. (d) Using source transformation,


1.5 6V 3
For d.c. supply of 5 V, the capacitor acts as open-
A
circuit at the steady-state, consequently there
will not be any current flowing in the circuit ISC
2 × 1.5 = 3 V
due to d.c. supply.
For a.c. supply of V = 10 sin(t), = 1 rad/sec.
B
j
Z = R+ j L = 1 + j2 j 3+6
C ISC = =2A
1.5 + 3
= (1 + j ) = 2 45° RN = 1.5 + 3 = 4.5
V 10 sin(t ) 4. (b)
Hence, current, I = =
Z 2 45°
Using Millman’s theorem,
= 7.07 sin(t – 45°) A
10 5
+
V1Y1 + V2Y2
2. Sol. V= = 6 4 =7 V
Y1 + Y2 1 1
False, superposition principle is applicable on +
6 4
both time variant and time invariant resistors. 1 1
R= + = 2.4
Y1 + Y2 1 + 1
3. Sol.
6 4
2A
3
A 5. (a)
By Thevenin’s theorem,
Z1 Z3
3
6V 3 X

B 100 0° Z2

Using source transformation,


6V Y
3
ZTh = ZX – Y = Z1 Z2 + Z3
A
Z1 × Z2
6 = + Z3
3
=2A 3 3 (Z1 + Z2 )
10 60 × 10 60
B = + (50 53.13)
(10 60 + 10 60)
3 3 = 1.5 = 56.66 45°
GATE Previous Years Solved Paper 99

6. (d) 10

For obtaining power absorbed by RL under


maximum power transfer condition. We find
20 10 Rth
Thevenin’s equivalent circuit across RL.
6 j8 6 j8

Rth = 10 10 = 5
Zth Vth = open-circuit voltage at the terminals P-Q.
10
P
+
Zth is calculated by short-circuiting the voltage
20 4V 10 Vth
sources,

Zth = (6 + j8) (6 + j8) = 3 + j 4 –


Q

6 j8 6 j8 4
Vth =× 10 = 2 V
10 + 10
Thevenin’s equivalent circuit,
110 0° Vth 90 0°
Vth = 5
P

Vth = 2 V
Vth 110 0° Vth 90 0°
+ =0
6 + j8 6 + j8
Q
Vth = 100 0° V

3 j4 8. (d)
To calculate Thevenin’s impedance, current-
Zth source is open-circuited,
Vth = 110 0° I RL x

R=1

Zth
For the maximum power transfer, ZL = 2j

2 2
RL = Rth + X th = 32 + 4 2 = 5 Z C = –j

Vth 100 y
I= =
(3 + j 4) + RL 8 + j 4 Zth = R + ZL + ZC
= 11.18 –26.56° A = 1 + 2j – j
Power absorbed by RL(max) = 1+j
= I2RL = 11.182 × 5 = 625 W Open-circuited voltage at terminals X-Y
= I × Zth
7. (a) = 1 0 × (1 + j)
To calculate Rth, (seen at terminals P-Q), voltage = 2 45° Volt
source is short-circuit.
100 Electronics Engineering Network Theory

9. (b) Thevenin equivalent circuit,


Thevenin’s impedance:
Z 0 = 2.38 – j0.667 2 I R

as real part is non-zero, so Z0 has resistor


Img [Z0] = –j0.667
10 V V 3V
Case-I:
Z0 has capacitor (as Img[Z0] is negative)
Case-II: Circuit A Circuit B
Z 0 has both capacitor and inductor, but
inductive reactance < capacitive reactance. 7
I=
At, = 5 rad/sec. R+2
For minimal realization case-I is considered. and V = 10 – 2I
Therefore, Z0 will have a resistor and a capacitor 14 10 R + 6
= = 10
R+2 R+2
10. (b)
Power transferred from circuit A to circuit B,
To calculate Thevenin’s resistance 5 V source is
P = VI
short-circuited and Vdc source is connected at
10 R + 6 7
terminals A and B. = ×
R+2 R+2
1
Then, Zth = dP
I th For P to be maximum =0
dR
By applying KCL at node B and A,
(R + 2)2 (10) – (10R + 6) × 2(R + 2) = 0
iAB + 99ib = Ith
5R2 + 20R + 20 – 10R2 + 26R + 12 = 0
ib = iA + iAB
5R2 + 6R = 8
ib – iA + 99ib = Ith
R = 0.8
100ib – iA = Ith ...(i)
By applying KVL in outer loop, 15. (c)
10 × 103 ib = 1 Using maximum power transfer theorem,
ib = 10–4 A
RL = Z = 4 j 3
and 10 × 103 ib = –100 iA
i A = –100 ib = 4 2 + 32 = 5
From equation (i),
100ib + 100ib = Ith 16. (c)
Ith = 200ib 3 j4 j6 3
= 200 × 10–4 = 0.02 + – I2 = 0
1 1 VL1
Zth = = = 50 Vs +
– 10VL1 Vth
I th 0.02
100 53.13° V

14. (a)

j4
2 R VL1 = 100 53.13°
3 + j4
+ + = 80 90° V
10 V –j1 3V
– – Vth = 10 VL1 = 800 90° V

Circuit A Circuit B
GATE Previous Years Solved Paper 101

17. (a) or we can simply the network,


Thevenin’s equivalent voltage = voltage referred
to secondary. 2.5 V

sin t 1 2.5
We have, =
Vth 2 5 IN
2.5 V
1
x
2.5

sin( t)
Now from the circuit, we get
5
y IN = =1A
1:2 5
or, Vth = 2 sin( t) ...(Thevenin voltage)
20. (a)
Also, Thevenin’s impedance seen from the
Let us apply superposition theorem.
x and y terminals = voltage referred to secondary
Considering the voltage source 20 sin10t alone:
side.
Then, 10 sin5t remain open-circuited.
Zth = Rth = (2)2 × 1
=4 1
...(Thevenin’s impedance)
So, Vth = 2 sin( t)
and Zth = Rth = 4 1
20 sin10t VC1 (t ) = j 0.1
j 10 × 1
18. (c)
The situation of problem is shown in figure:
Zs = Rs ± jX
Let, VC1 (t ) be the voltage across capacitor.

j 0.1
VC1 (t ) = × 20 sin 10t
V RL 1 j 0.1
= (1.99 –84.28°) sin10t
VC1 (t ) = 2 sin(10t – 84.28°) ...(i)
For the transfer of maximum power from source
to load, Considering the current source 10 sin5t alone:
Then, 20 sin10t voltage source remain short-
RL = Rs2 + X 2 = Z
circuited.
Hence, option (c) is correct.
VC2(t) 1H
19. Sol.
Using source transformation theorem,
1 –j0.2 10 sin5t

2.5 V

2.5
5 IN

Let voltage across capacitor = VC (t )


1A 5 5 2
102 Electronics Engineering Network Theory

Applying KCL at the node, we have 22. Sol.


VC2 VC2 8 12 sin t
+ 10 sin 5t = 0 i(t) = + = 8 + 6 2 sin t
1 ( j 0.2) 1 1+ 1
10 sin 5t 1
or, VC2 (t ) = irms(t) = 82 + (6 2 )2 = 10 A
(1 + j 5) 2
or, VC2 (t ) = 1.97 sin(5t – 78.69°) ...(ii)
23. Sol.
Using superposition theorem, voltage across
capacitor is,
VC(t) = VC1 (t ) + VC2 (t )
= 2 sin(10t – 84.28°)
V1 = 10 V, I2 = 4 A, V2 = 0 Cond. ...(i)
+ 1.97 sin(5t – 78.69°)
V1 = 5 V, I2 = 1.25 A,
VC(t) = 2 sin(10t – 84.28°)
V2 = 1.25 × 1 = 1.25 Cond. ...(ii)
+ 1.97 sin(5t – 78.69°) ...(iii)
V1 = 3 V, I2 = ?, R = 2 Cond. ...(iii)
Given,
As we know from ABCD parameter,
VC(t) = A1 sin( 1t – 1)
V1 = AV2 – BI2; I1 = CV2 – DI2
+ A2 sin( 2t – 2) ...(iv)
From condition (i),
Comparing equations (iii) and (iv), we have
10 = A(0) – B(4)
A = 2 and B = 1.97 1.98
10
(closest answer) B=
4
21. Sol. From condition (ii),

20 i 10
1 V i1 5 = A (1.25) × (1.25)
–+ A 4
i
12.5
+ 5
2V 1 2 4
– A= = 1.5
1.25
i2
From condition (iii),
B
2 – 1(i1 + i) – i = 0 10
3 = 1.5(2 I ) ×I
2 – i1 – 2i = 0 4
2i + i1 = 2 ...(i) = 3I + 2.5I = 5.5 I
I = 0.545 A
V 2 V V ( 20i )
+ + =0
1 1 2 24. Sol.
2(V – 2) + 2V + V + 20i =0
To get Rth and Vth, consider the following steps.
4V + 4 + V + 20i =0
Case-1: For Rth
5V + 20i =4
3
V
where, =i
1
25i = 2
4
i= A 5 5
25
4
i1 = 2 2 × = 1.68 A
25 5× 5
VAB = 1.68 × 2 = 3.36 V Rth = = 2.5
5+5
GATE Previous Years Solved Paper 103

Case-2: For Vth Real part of the,


1
Zload =
Vth
6V 1 + C2 2

1
Zload = = 0.5
5 10 V 1 + 2C 2
Vth Putting, = 100 rad/sec.
we get, C = 10 mF
5V 5

26. (a)
Consider the following circuit,
Applying KCL at node, 10 10
Vth 5 Vth + 16
+ =0
5 5 a
6A 5 10 16 V
2Vth = –11
b
Vth = –5.5 V
Maximum power transferred,
2
After rearrangement we get,
Vth
Pmax = = 3.025 W 15 5
4RL

25. (d) +
30 V V ab 8V
Rs = 0.5 –

5 mH From circuit using KCL,


V(t) = 10 sin100t Voltage,
Vab + 30 Vab 8
1 C + =0
15 5
Vab + 30 + 3Vab – 24 = 0
The frequency at which the load is resistive and Vab = –1.5 V
it is equal to 0.5 i.e. The load is resistive means,
the imaginary part of the is equal to zero and 27. Sol.
real part is equal to 0.5 . 2

1

Cs + Ls = 1 + Ls RA RB
Zload = 1
1+ 1 + Cs
Cs RC

(1 Cs )
= + Ls 4 2
1 C 2 s2
2
Put s = j ;
1 j C
Zload = +j L
1 + C2 2
where, RA = 1
1 C RB = 1
= +j L
1 + C2 2
(1 + C 2 2
) 1
RC =
2
104 Electronics Engineering Network Theory

After rearrangement consider the following 29. Sol.


circuit,
4V 2 Vth 3
1 1/2
+

4V 3 5A Vth
3


From the circuit diagram we get,
Vth 4
V =5
Z 21 = 2 =3 2
I1
Vth = 14 V

28. Sol. 30. (a)

I
R

20
VI = K
0V
10 V

By Millman’s theorem,
0 4A
200 160 100 80
+
50 40 25 20 = 0 V Maximum power transistor of VI product is
E= 1 1 1 1
+ + + maximum. If draw the curve, it intersect (10, 4)
50 40 25 20 that will give maximum power. The terminal
1 1 1 1 1 voltage is 10 V (Load voltage) and current is 4 A
= + + +
R 50 40 25 20 (Load current).
Simplified circuit,
10
I = 0A Load resistance is = 2.5 .
4
4 Transient Analysis

ELECTRO NICS EN GINEERIN G R2


(c) u(t )
(GATE Previous Years Solved Papers) R1 + R2

R2
Q.1 A 10 resistor, a 1 H inductor and 1 µF capacitor (d) et / R1C1 u(t )
R1 + R2
are connected in parallel. The combination is
driven by a unit step current. Under the steady- [EC-1992 : 2 Marks]
state condition, the source current flows
Q.4 A ramp voltage, v(t) = 100t Volts, is applied to
through
an RC differentiating circuit with R = 5 k and
(a) the resistor C = 4 µF. The maximum output voltage is
(b) the inductor (a) 0.2 Volt (b) 2.0 Volts
(c) the capacitor only (c) 10.0 Volts (d) 50.0 Volts
(d) all the three elements [EC-1994 : 1 Mark]
[EC-1989 : 2 Marks]
Q.5 The rms value of a rectangular wave of period
Q.2 If the Laplace transform of the voltage across a T, having a value of +V for duration, T1 (<T)
capacitor of value of 1/2 F is and –V for the duration, T – T1 = T2 equals
s+1 T1 T2
Vc ( s ) = 3 2
(a) V (b) V
s +s +s+1 T
The value of the current through the capacitor V T1
(c) (d) V
at t = 0+ is, 2 T2
(a) 0 A (b) 2 A [EC-1995 : 1 Mark]
1
(c) A (d) 1 A Q.6 The voltage VC , VC and VC3 across the
2 1 2
[EC-1989 : 2 Marks] capacitors in the circuit in figure, under steady-
state, are respectively
Q.3 For the compensated attenuator of figure, the
impulse response under the condition 10 k 1H 2F 2H 25 k
R1C1 = R2C2 is + –
VC 2

R1 + + +
100 V VC 1 1F 40 k VC3 3F
– – –
+ +
C1
v1(t) C2 R2 v2(t)

(a) 80 V, 32 V, 48 V (b) 80 V, 48 V, 32 V
– –
(c) 20 V, 8 V, 12 V (d) 20 V, 12 V, 8 V
R2 [EC-1996 : 2 Marks]
(a) [1 e1/ R1C1 ] u(t )
R1 + R2 Q.7 In the circuit of figure the energy absorbed by
R2 the 4 resistor in the time interval (0, ) is
(b) (t )
R1 + R2
106 Electronics Engineering Network Theory

4 Q.10 I1(s) and I2(s) are the Laplace transforms of i1(t)


VC(0) = 6 V and i2(t) respectively. The equations for the loop
+ currents I1(s) and I2(s) for the circuit shown in
10 V 2F VC
– the figure, after the switch is brought from
position 1 to position 2 at t = 0, are

(a) 36 Joules (b) 16 Joules 1


R + Ls + Ls
Cs I 1 (s) V /s
(c) 256 Joules (d) None of the above (a) =
1 I 2 (s) 0
[EC-1997 : 2 Marks] Ls R+
Cs
Q.8 In the figure, the switch was closed for a long 1
time before opening at t = 0. The voltage Vx at R + Ls + Ls
Cs I 1 (s) V /s
(b) =
t = 0+ is, 1 I 2 (s ) 0
Ls R+
Cs
t=0
1
R + Ls + Ls
Cs I 1 (s ) V /s
(c) =
1 I 2 (s ) 0
2 2.5 A Ls R + Ls +
Cs
5H
1
R + Ls + Ls
Cs I 1 (s ) V /s
(d) =
1 I 2 (s ) 0
Ls R + Ls +
20 Cs
– Vx + [EC-2003 : 2 Marks]
(a) 25 V (b) 50 V
Q.11 For the R-L circuit shown in the figure, the input
(c) –50 V (d) 0 V voltage vi(t) = u(t). The current i(t) is
[EC-2002 : 1 Mark]
1H
The circuit for (Q. 9 and Q.10) is given. Assume that the
i (t )
switch S is in position 1 for a long time and thrown to
vi(t) 2
position 2 at t = 0.

Q.9 At t = 0+, the current i1 is

1
S C i(t)

0.5
2
i1(t) i2(t) R
V (a) 0.31
L
R
C 2 t(sec)

i(t)
V V
(a) (b) 1
2R R
V
(c) (d) zero (b) 0.63
4R
[EC-2003 : 2 Marks]
1/2 t(sec)
GATE Previous Years Solved Paper 107

i(t) Q.14 A 2 mH inductor with some initial current can


be represented as shown below, where ‘s’ is the
0.5
Laplace transform variable. The value of initial
(c) 0.31 current is
I (s )
1/2 t(sec)
0.002 s
i(t)

1

(d) 1 mV
0.63
+

2 t(sec)
(a) 0.5 A (b) 2.0 A
[EC-2004 : 1 Mark] (c) 1.0 A (d) 0.0 A

Q.12 The circuit shown in the figure has initial [EC-2006 : 1 Mark]
current iL(0–) = 1 A through the inductor and an Q.15 In the figure shown below, assume that all the
initial voltage vC(0–) = –1 V across the capacitor. capacitors are initially uncharged. If
For input v(t) = u(t), the Laplace transform of the vi(t) = 10 u(t) Volts, vo(t) is given by
current i(t) for t 0 is
1k
1 1H

+ + +
i (t )
+
v(t ) 1F 4 µF
– vi(t ) 4k 1 µF v o (t )


– –
s s+2
(a) (b) (a) 8e–t/0.004 Volts (b) 8(1 – e–t/0.004) Volts
s2 + s + 1 s2 + s + 1
s 2 s 2 (c) 8 u(t) Volts (d) 8 Volts
(c) (d) [EC-2006 : 1 Mark]
s2 + s 1 s2 + s + 1
[EC-2004 : 2 Marks] Q.16 In the circuit shown, Vc is 0 Volts at t = 0 sec. For
Q.13 A square pulse of 3 Volts amplitude is applied t < 0, the capacitor ic(t), where ‘t’ is (in seconds),
to C-R circuit shown in the figure. The capacitor is given by
is initially uncharged. The output voltage V2 at 20 k ic
time t = 2 sec is
Vi 0.1 µF +
10 V 20 k VC 4 µF
3V + + –

V1 1k V2
(a) 0.50 exp(–25t) mA
– –
t (b) 0.25 exp(–25t) mA
2 sec
(c) 0.50 exp(–12.5t) mA
(a) 3 V (b) –3 V
(d) 0.25 exp(–6.25t) mA
(c) 4 V (d) –4 V
[EC-2007 : 2 Marks]
[EC-2005 : 2 Marks]
108 Electronics Engineering Network Theory

Q.17 In the following circuit, the switch S is closed at


(c) tu(t ) + 2 ( 1)n (t nT ) u(t nT )
di + n=1
t = 0. The rate of change of current (0 ) is
dt
(t 2 nT ) (t 2 nT T )
given by (d) [0.5 e + 0.5e ]
n=0
S
R
[EC-2008 : 2 Marks]

Common Data for Questions (19 and 20):


Is Rs i (t ) L The following series RLC circuit with zero initial
conditions is excited by a unit impulse function (t).
1H 1

Rs I s
(a) 0 (b)
L
(t ) 1F Vc(t)
( R + Rs )I s
(c) (d)
L
[EC-2008 : 1 Mark]

Q.18 The circuit shown in the figure is used to charge Q.19 For t > 0, the output voltage Vc(t) is
the capacitor C alternately from two current
2 1 /2 t 3 /2 t
sources as indicated. The switches S1 and S2 (a) (e e )
3
are mechanically coupled and connected as
follows: 2 1 /2 t
(b) te
For 2nT t < (2n + 1) T, (n = 0, 1, 2,....) S1 to P1 and 3
S2 to P2. 2 3
1/2 t
For (2n + 1) T t < (2n + 2) T, (n = 0, 1, 2,....) (c) e cos t
3 2
S1 to Q1 and S2 to Q2.
2 1/2 t 3
Q1 P1 Q2 P2 (d) e sin t
3 2
+
S1 S2
[EC-2008 : 2 Marks]
+
1 0.5 1F C Vc(t) 1 1 Q.20 For t > 0, the voltage across the resistor is

1 3 /2 t 1 /2 t
– (a) (e e )
1A 1A 3

1/2 t 3t 1 3t
Assume that the capacitor has zero initial (b) e cos sin
2 3 2
charge. Given that u(t) is a unit step function,
the voltage Vc(t) across the capacitor is given by
2 1/2 t 3t
(c) e sin
3 2
(a) ( 1)n tu (t nT )
n=0
2 1/2 t 3t
(d) e cos
3 2
(b) u(t ) + 2 ( 1)n u (t nT )
n=1 [EC-2008 : 2 Marks]
GATE Previous Years Solved Paper 109

Q.21 The switch in the circuit shown was on a Q.24 In the circuit shown below, the initial charge on
position a for a long time, and is moved to the capacitor is 2.5 mC, with the voltage polarity
position ‘b’ at time t = 0. The current i(t) for t > 0 as indicated. The switch is closed at time t = 0.
is given by The current i(t) at a time ‘t’ after the switch is
10 k closed is
a b
i (t )
i (t )

10
100 V 0.2 µF 5k
100 V

0.5 µF 0.3 µF 50 µF
+

(a) 0.2e–125t u(t) mA (b) 20e–1250t u(t) mA


(c) 0.2e–1250t u(t) mA (d) 20e–1000t u(t) mA
(a) i(t) = 15 exp(–2 × 103t) A
[EC-2009 : 2 Marks]
(b) i(t) = 5 exp(–2 × 103t) A
Q.22 The time domain behaviour of an RL circuit is (c) i(t) = 10 exp(–2 × 103t) A
represented by (d) i(t) = –5 exp(–2 × 103t) A
di Rt / L [EC-2011 : 2 Marks]
L + Ri = Vo (1 + Be sin t ) u(t )
dt
Q.25 In the following figure, C1 and C2 are ideal
V capacitors. C1 had been charged to 12 V before
For an initial current of i(0) = o , the steady-
R the ideal switch S is closed at t = 0.
state value of the current is given by The current i(t) for all ‘t’ is
Vo 2Vo S
(a) i(t ) (b) i(t )
R R
t=0
Vo 2Vo
(c) i(t ) (1 + B) (d) i(t ) (1 + B)
R R C1 i(t) C2
[EC-2009 : 2 Marks]

Q.23 In the circuit shown, the switch S is open for a


long time and is closed at t = 0. The current i(t) (a) zero
for t 0+ is (b) a step function
10 (c) an exponentially decaying function
t=0 (d) an impulse function
S [EC-2012 : 1 Mark]
1.5 A 10 15 mH
Q.26 For maximum power transfer between two
i(t) 10
cascaded sections of an electrical network, the
relationship between the output impedance Z1
(a) i(t) = 0.5 – 0.125 e–1000t A of the first section to the input impedance Z2 of
the second section is
(b) i(t) = 1.5 – 0.125 e–1000t A
(a) Z2 = Z1 (b) Z2 = –Z1
(c) i(t) = 0.5 – 05 e–1000t A
(d) i(t) = 0.375 e–1000t A (c) Z2 = Z1 (b) Z2 = Z1
[EC-2010 : 2 Marks] [EC-2014 : 1 Mark]
110 Electronics Engineering Network Theory

Q.27 In the circuit shown in the figure, the value of ix 2H


capacitor C (in mF) needed to have critically
damped response i(t) is ______ . 2ix +

+
40 4H C 10 u(t) A 5 vo(t)
+V – –
o
5

i( t )
[EC-2014 : 2 Marks]

Q.31 In the circuit shown, the switch SW is thrown


[EC-2014 : 2 Marks]
from position A to position B at time t = 0. The
Q.28 In the figure shown, the idea switch has been energy (in µJ) taken from the 3 V source to charge
open for a long time. If it is closed at t = 0, then the 0.1 µF capacitor form 0 V to 3 V is
the magnitude of the current (in mA) through SW
120
the 4 k resistor at t = 0+ is ______ . 3V
B A

5k 4k 1k t=0

i 0.1 µF

10 V 10 µF t=0 1 mH
(a) 0.3 (b) 0.45
(c) 0.9 (d) 3
[EC-2015 : 1 Mark]
[EC-2014 : 1 Mark]
Q.32 In the circuit shown, switch SW is closed at t = 0.
Q.29 In the figure shown, the capacitor is initially
Assuming zero initial conditions, the value of
uncharged. Which one of the following
vc(t) (in Volts) at t = 1 sec is ______ .
expressions describes the current I(t) (in mA)
t=0
for t > 0? 3
R1 S
5 +
1k I 10 V 2 F vc(t)
6 –
5V R2 2k C 1 µF

[EC-2015 : 2 Marks]

Q.33 In the circuit shown, the initial voltages across


5 t/ 2
(a) I (t ) = (1 e ), = msec the capacitors C1 and C2 and 1 V and 3 V,
3 3
respectively. The switch is closed at time t = 0.
5 t/ 2 The total energy dissipated (in Joules) in the
(b) I (t ) = (1 e ), = msec
2 3 resistor R until steady-state is reached, is ____ .
5
(c) I (t ) = (1 e t / ), = 3 msec t=0
R = 10
3
5
(d) I (t ) = (1 e t / ), = 3 msec + +
2
C1 = 3 F C2 = 1 F
[EC-2014 : 2 Marks] – –

Q.30 In the circuit shown in the figure, the value of


vo(t) (in Volts) for t is ______ . [EC-2015 : 2 Marks]
GATE Previous Years Solved Paper 111

Q.34 The switch has been in position 1 for a long


time and abruptly changes to position 2 at t = 0. 3F 2
3 4 2 1
1 2
2
t=0 i (t )

10 V 2 + 2 5A
12 V
2F 3
0.1 F VC

[EC-2016 : 2 Marks]
If time ‘t’ is in seconds, the capacitor voltage VC
Q.37 In the circuit shown, the voltage VIN(t) is
(in Volts) for t > 0 is given by
described by
t
(a) 4 1 exp 0, for t < 0
0.5 VIN (t ) =
15 Volts, for t 0
t
(b) 10 6 exp where ‘t’ is in seconds. The time (in seconds) at
0.5
which the current I in the circuit will reach the
t value 2 Ampere is _____ .
(c) 4 1 exp
0.6 1 I
t +
(d) 10 6 exp
0.6
VIN(t ) 1H 2H
[EC-2016 : 1 Mark]

Q.35 The switch S in the circuit shown has been –


closed for a long time. It is opened at time t = 0
and remains open after that. Assume that the [EC-2017 : 2 Marks]
diode has zero reverse current and zero forward
Q.38 The switch in the circuit, shown in the figure,
voltage drop.
was open for a long time and is closed at t = 0.
t=0
1
S i(t) 5

– t=0
10 A 5
10 V 1 mH 10 µF VC
+
2.5 H

The steady-state magnitude of the capacitor The current i(t) (in Ampere) at t = 0.5 seconds is
voltage VC (in Volts), is ______ . ______ .
[EC-2016 : 2 Marks] [EC-2017 : 2 Marks]
Q.36 Assume that the circuit in the figure has reached Q.39 For the circuit given in the figure, the magnitude
the steady-state before time t = 0 when the 3 of the loop current (in amperes, correct to three
resistor suddenly burns out, resulting in an decimal places) 0.5 seconds after closing the
open-circuit. The current i(t) (in Amperes) at t = 0+ switch is _____ .
is _____ .
112 Electronics Engineering Network Theory

1V 1 P t=0
– +

1 1H 20 V 20 k

[EC-2018 : 2 Marks]
dv(t )
The value of at t = 0+ is
Q.40 The RC circuit shown below has a variable dt
resistance R(t) given by the following expression: (a) –5 V/s (b) 3 V/s
t (c) –3 V/s (d) 0 V/s
R(t ) = R0 t for 0 t < T
T [EC-2021 : 2 Marks]
where R0 = 1 , and C = 1 F. We are also given
Q.43 The circuit in the figure contains a current
that T = 3 R0C and the source voltage is Vs = 1 V.
source driving a load having an inductor and a
If the current at time t = 0 is 1 A. Then the current
resistor in series, with a shunt capacitor across
I(t), in amperes, at time t = T/2 is ____ .
the load. The ammeter is assumed to have zero
(Rounded off to 2 decimal places).
resistance. The switch is closed at time, t = 0.
I( t) R(t )
Ammeter
S
A Load

Vs Current 10 mH
C
source
t=0 1A Shunt 100 pF
capacitor
5k

[EC-2019 : 2 Marks]

Q.41 In the circuit shown in the figure, the switch is


closed at time t = 0, while the capacitor is Initially, when the switch is open, the capacitor
initially charged to –5 V (i.e., VC(0) = –5 V). is discharged and the ammeter reads zero
t=0 ampere. After the switch is closed, the ammeter
250
reading keeps fluctuating for some time till it
+ V – settles to a final steady value. The maximum
R

5V 250 Vc(t) 0.6 µF ammeter reading that one will observe after the
VR/500 switch is closed (Rounded off to two decimal
places) is _______ A.
The time after which the voltage across the [EC-2021 : 2 Marks]
capacitor becomes zero (Rounded off to three
decimal places) is ______ ms. ELECTRICAL EN GINEERIN G
[EC-2021 : 2 Marks] (GATE Previous Years Solved Papers)

Q.42 The switch in the circuit in the figure is in Q.1 The time constant of the network shown in figure
position ‘P’ for a long time and then moved to is
position ‘Q’ at time t = 0.
GATE Previous Years Solved Paper 113

R (a) 3 µ-sec
(b) 12 µ-sec
(c) 32 µ-sec
10 V 2R C
(d) unknown, unless the actual network is
specified
[EE-1996 : 1 Mark]
(a) 2 RC (b) 3 RC
2 RC Q.4 An ideal voltage source will charge an ideal
RC
(c) (d) capacitor
2 3
[EE-1992 : 1 Mark] (a) in infinite time (b) exponentially
(c) instantaneously (d) none of these
Q.2 In the series RC circuit shown in figure the
[EE-1997 : 1 Mark]
voltage across C starts increasing when the dc
source is switched ON. The rate of increase of Q.5 In the circuit shown in figure, it is desired to
voltage across C at the instant just after the have a constant direct current i(t) through the
switch is closed (i.e. at t = 0+) will be ideal inductor L. The nature of the voltage source
C R
v(t) must be
+ –
Vc i(t)

I V(t ) L

1V

t=0 V (a) constant voltage


(a) Zero (b) Infinity (b) linearly increasing voltage
1 (c) an ideal impulse
(c) RC (d)
RC (d) exponentially increasing voltage
[EE-1996 : 1 Mark] [EE-1998 : 1 Mark]

Q.3 The v-i characteristic as seen from the terminal Q.6 A rectangular voltage pulse of magnitude V and
pair (A, B) of the network of Fig. (1) is shown in duration T is applied to a series combination of
Fig. (2). If an inductance of value 6 mH is resistance R and capacitance C. The maximum
connected across the terminal - pair (A, B), the voltage developed across the capacitor is
time constant of the system will be
T
i A (a) V 1 exp
Network of + RC
linear resistors V VT
and independent (b)
sources – RC
B (c) V
Fig. (1)
T
i (d) V exp [EE-1999 : 2 Marks]
RC
4 mA
Q.7 A voltage waveform v(t) = 12t2 is applied across
a 1 H inductor for t 0, with initial current
through it being zero. The current through the
(0, 0)
v inductor for t 0 is given by
Fig. (2) 8V
114 Electronics Engineering Network Theory

(a) 12 t (b) 24 t
S
3
(c) 12 t3 (d) 4 t3
[EE-2000 : 1 Mark] t=0 4F
Q.8 A unit step voltage is applied at t = 0 to a series 10 V 4 4H
RL circuit with zero initial conditions. 4
(a) It is possible for the current to be oscillatory.
(b) The voltage across the resistor at t = 0+ is
zero. (a) 2 V (b) 4 V
(c) The energy stored in inductor in the steady- (c) –6 V (d) 8 V
state is zero. [EE-2003 : 2 Marks]
(d) The resistor current eventually falls to zero.
Q.12 In figure, the capacitor initially has a charge of
[EE-2000 : 1 Mark]
10 Coulomb. The current in the circuit one
Q.9 Consider the circuit shown in figure. If the second after the switch ‘S’ is closed will be
frequency of the source is 50 Hz, then the value
t=0
of t0 which results in a transient free response is 2

5 0.01 H S
+
t = t0 100 V 0.5 F

sin( t)

(a) 14.7 A (b) 18.5 A


(a) 0 ms (b) 1.78 ms (c) 40.0 A (d) 50.0 A
(c) 2.71 ms (d) 2.91 ms [EE-2004 : 2 Marks]
[EE-2002 : 2 Marks] Q.13 In the figure given, for the initial capacitor
Q.10 An 11 V pulse of 10 µs duration is applied to the voltage is zero. The switch is closed at t = 0. The
circuit shown in figure. Assuming that the final steady-state voltage across the capacitor
capacitor is completely discharged prior to is
applying the pulse, the peak value of the S 10
capacitor voltage is
1k t=0

20 V 10 µF 10
11 V
10 k 11 nF
0
10 µs
(a) 20 V (b) 10 V
(a) 11 V (b) 5.5 V (c) 5 V (d) 0 V
(c) 6.32 V (d) 0.96 V [EE-2005 : 1 Mark]
[EE-2002 : 2 Marks] Q.14 The circuit shown in the figure is steady-state,
Q.11 In the circuit shown in figure, the switch ‘S’ is when the switch is closed at t = 0. Assuming
closed at time (t = 0). The voltage across the that the inductance is ideal, the current through
inductor at t = 0+, is the inductor at t = 0+ equals.
GATE Previous Years Solved Paper 115

10 Q.18 In the circuit shown in the figure, the current


source I = 1 A, the voltage source V = 5 V,
R1 = R2 = R3 = 1 , L1 = L2 = L3 = 1 H, C1 = C2 = 1 F.
t=0
10 V 10 mH The currents (in A) through R3 and the voltage
source V respectively will be
R1 R2

(a) 0 A (b) 0.5 A


(c) 1 A (d) 2 A L1 C1 L3

[EE-2005 : 2 Marks] C2 V

I L2 R3
Statement for Linked Answer Questions (15 and 16):
A coil of inductance 10 H and resistance 40 is
connected as shown in the figure. After the switch ‘S’ (a) 1 and 4 (b) 5 and 1
has been in contact with point 1 for a very long time, it (c) 5 and 2 (d) 5 and 4
is moved to point 2 at, t = 0. [EE-2006 : 2 Marks]
Q.15 If at t = 0+, the voltage across the coil is 120 V, Q.19 In the figure, transformer T1 has two secondaries,
the value of resistance R is all three windings having the same number of
1 20 turns and with polarities as indicated. One
secondary is shorted by a 10 resistor R, and
S 2
10 H the other by a 15 mF capacitor. The switch SW is
120 V opened (t = 0) when the capacitor is charged to
R
40 5 V with the left plate as positive. At (t = 0+) the
voltage VP and current IR are
(a) 0 (b) 20 S IR
(c) 40 (d) 60 T1 R
[EE-2005 : 2 Marks]
+
Q.16 For the value of resistance obtained in (a), the 25 V VP C
– + –
like taken for 95% of the stored energy to be
dissipated is close to
(a) 0.10 sec (b) 0.15 sec
(a) –25 V, 0.0 A
(c) 0.50 sec (d) 1.0 sec
(b) very large voltage, very large current
[EE-2005 : 2 Marks]
(c) 5.0 V, 0.5 A
Q.17 An ideal capacitor is charged to a voltage Vo (d) –5.0 V, –0.5 A
and connected at t = 0 across an ideal inductor [EE-2007 : 2 Marks]
L. (The circuit now consists of a capacitor and
Q.20 In the circuit shown in figure. Switch SW1 is
1 initially closed and SW2 is open. The inductor
inductor alone). If we let o = , the voltage
LC L carries a current of 10 A and the capacitor
across the capacitor at time t > 0 is given by charged to 10 V with polarities as indicated.
(a) Vo (b) Vo cos( ot) SW2 is closed at t = 0 and SW1 is opened at t = 0.
The current through C and the voltage across L
(c) Vo sin( ot) (d) Vo e ot cos( ot ) at (t = 0+) is
[EE-2006 : 2 Marks]
116 Electronics Engineering Network Theory

SW2 R2 = 10 (a) 18.8 V (b) 23.5 V


(c) –23.5 V (d) –30.6 V
+ [EE-2008 : 2 Marks]
R1 = 10 SW1 L 10 A 10 V C

Q.24 In the figure shown, all elements used are ideal.
For time t < 0, S1 remained closed and S2 open.
(a) 55 A, 4.5 V (b) 5.5 A, 45 V At t = 0, S1 is opened and S2 is closed. If the
(c) 45 A, 5.5 A (d) 4.5 A, 55 V voltage Vo across the capacitor C2 at t = 0– is
2
[EE-2007 : 2 Marks] zero, the voltage across the capacitor
combination at t = 0+ will be
Q.21 The time constant for the given circuit will be
1F S1 S2
3

3A 3V C1 1F C2 2F
1F 1F 3

1 1 (a) 1 V (b) 2 V
(a) sec (b) sec
9 4 (c) 1.5 V (d) 3 V
(c) 4 sec (d) 9 sec [EE-2009 : 2 Marks]
[EE-2008 : 2 Marks]
Q.25 The switch in the circuit has been closed for a
Statement for Linked Answer Questions (22 and 23): long time. It is opened at t = 0. At t = 0+, the
The current i(t) sketched in the figure flows through a current through the 1 µF capacitor is
initially uncharged 0.3 nF capacitor.
1 S
Q.22 The charge stored in the capacitor at t = 5 µs,
t=0
will be
5V 1 µF 4
6
5
4
i(t) mA

(a) 0 A (b) 1 A
3
(c) 1.25 A (d) 5 A
2
[EE-2010 : 1 Mark]
1

0 1 2 3 4 5 6 7 8 9 Q.26 The L-C circuit shown in the figure has an


1 (µs) inductance L = 1 mH and a capacitance
C = 10 µF.
(a) 8 nC (b) 10 nC
(c) 13 nC (d) 16 nC L

[EE-2008 : 2 Marks] i

Q.23 The capacitor charged upto 5 µs, as per the C 100 V
t=0
current profile given in the figure, is connected +
across an inductor of 0.6 mH. Then the value of
voltage across the capacitor after 1 µs will
approximately be
GATE Previous Years Solved Paper 117

The initial current through the inductor is zero, Vs


while the initial capacitor voltage is 100 V. The
switch is closed at t = 0. The current ‘i’ through
(a)
the circuit is t
(a) 5 cos (5 × 103t) A –2

(b) 5 sin (104t) A Vs


(c) 10 cos (5 × 103t) A
(b)
(d) 10 sin (104t) A
t
[EE-2010 : 2 Marks] –2

Q.27 In the following figure C1 and C2 are ideal Vs


capacitors. C1 has been charged to 12 V before
the ideal switch ‘S’ is closed at t = 0. The current (c)
i(t) for all ‘t’ is t

S
Vs

t=0
(d)
C1 i(t) C2 t

[EE-2014 : 1 Mark]
(a) zero Q.29 The switch SW shown in the circuit is kept at
(b) a step function position ‘1’ for a long duration. At t = 0+, the
(c) an exponentially decaying function switch is moved to position ‘2’. Assuming
(d) an impulse function
V02 > V01 , the voltage V c (t) across the
[EE-2012 : 1 Mark]
capacitor is
Q.28 A combination of 1 µF capacitor with an initial
R ‘2’
voltage Vc(0) = –2 V in series with a 100 SW
resistor is connected to a 20 mA ideal dc current
‘1’ R
source by operating both switches at t = 0 is as
V02
shown in the figure. Which of the following V01
graphs shown in the options approximates the C Vc

voltage Vs across the current source over the next


few seconds? (a) Vc(t) = V02(1 – e–t/2RC) – V01
(b) Vc(t) = V02(1 – e–t/2RC) + V01
Vc
+ – (c) Vc(t) = –(V02 + V01) (1 – e–t/2RC) – V01
(d) Vc(t) = (V02 – V01) (1 – e–t/2RC) + V01
t=0 + [EE-2014 : 1 Mark]
Vs 100
Q.30 A series RL circuit is excited at t = 0 by closing a

switch as shown in the figure. Assuming zero

d2 I
t=0 initial conditions, the value of at t = 0+ is
dt 2
118 Electronics Engineering Network Theory

R 6 8

V 50 V 8 32 2H

32
L

V V (a) 2.5 e–4t (b) 5 e–4t


(a) (b)
L R (c) 2.5 e–0.25t (d) 5 e–0.25t
RV [EE-2017 : 2 Marks]
(c) 0 (d)
L2
Q.34 The initial charge in the 1 F capacitor present in
[EE-2015 : 1 Mark]
the circuit shown is zero. The energy in Joules
Q.31 In the circuit shown, switch S2 has been closed transferred from the d.c. source until steady-
for a long time. A time t = 0 switch S1 is closed. state condition is reached equals ______ .
At t = 0+, the rate of change of current through (Give the answer upto one decimal place)
the inductor, in amperes per second, is _____ .

S1 1F
1 S2
5 5
5
2 10 V
3V 1H
5 5
3V

[EE-2016 : 2 Marks]
[EE-2017 : 1 Mark]
Q.32 In the circuit shown below, the initial capacitor
voltage is 4 V. Switch S1 is closed at t = 0. The Q.35 A resistor and a capacitor are connected in series
charge (in µC) lost by the capacitor from t = 25 µs to a 10 V d.c. supply through a switch. The
to t = 100 µs is _______ . switch is closed at t = 0, and the capacitor
voltage is found to cross 0 V at t = 0.4 , where
S1 is the circuit time constant. The absolute value
of percentage change required in the initial
capacitor voltage if the zero crossing has to
4V 5 µF 5
happen at t = 0.2 is _______ .
(Rounded off 2 decimal places).
[EE-2020 : 2 Marks]
[EE-2016 : 2 Marks]

Q.33 The switch in the figure below was closed for a


long time. It is opened at t = 0. The current in the
inductor of 2 H for t 0, is
GATE Previous Years Solved Paper 119

Electronics & Electrical Engineering


GATE Previous Years Solved Paper

A n swe rs & Expl a n a t i o n s

Answers
EC Transient Analysis

1. (b) 2. (c) 3. (b) 4. (b) 5. (a) 6. (b) 7. (b) 8. (c)

9. (a) 10. (c) 11. (c) 12. (b) 13. (b) 14. (a) 15. (c) 16. (a)

17. (b) 18. (c) 19. (d) 20. (b) 21. (b) 22. (a) 23. (a) 24. (a)

25. (d) 26. (c) 27. (10) 28. (1.25) 29. (a) 30. (31.25) 31. (c) 32. (2.528)

33. (1.5) 34. (d) 35. (100) 36. (–1) 37. (0.3405) 38. (8.16) 39. (0.316) 40. (0.25)
41. (0.1386) 42. (c) 43. (1.44)

Solutions
EC Transient Analysis

1. (b) 3. (b)
At steady state:
1
Inductor behave as short-circuit. R2 ×
C2 s R2
So, under steady state condition the source Z2(s) = =
1 R2 2s + 1
C
current flows through the inductor. R2 ×
C1s
2. (c)
1
1 2 R1 ×
ZC(s) = = C1s R1
Cs s Z1(s) = =
1 R1 1s + 1
C
R1 ×
VC (s ) s(s + 1) C1s
IC(s) = =
ZC (s ) 2(s + s 2 + s + 1)
3
V2 (s) Z2 (s )
s(s + 1) =
= V1 (s) Z1 (s) + Z2 (s )
2(s 2 + 1) ( s + 1)
R1C 1 = R2C 2
s
IC(s) = R2
2(s 2 + 1) V2 (s) R2C 2 s + 1 R2
= =
V1 (s) R1 R2 R1 + R2
s2 +
i(0+) = lim sIC (s) = lim R2 C 2 s + 1 R2 C 2 s + 1
s s 2(s 2 + 1)
R2
1 1 V2(s) = V1 (s)
= = Ampere R1 + R2
2+0 2
120 Electronics Engineering Network Theory

For impulse response, 40


VC1 = 100 × = 80 V
V1(s) = 1 10 + 40
v1(t) = (t) 3
VC 2 = 80 × = 48 V
R2 2+3
v2(t) = (t )
R1 + R2 2
VC3 = 80 × = 32 V
2+3
4. (b)
dVi 7. (b)
Vo = RC
dt
V = V + =6V
3 6 d (0 ) (0 )
Vo = (5 × 10 ) (4 × 10 ) (100t ) = 2 V
dt So at t = 0+
VR = 10 – 6 = 4 V
5. (a)
4
x(t) I = = 1 Ampere
T1 R(0+ ) 4
V IR( ) = 0 Ampere
= RC = 8 sec.
T
t
T1
i(t) = i + (i i ) e t/
0+
–V
T2 = 0 + (1 – 0) e–t/8
i(t) = e–t/8
T = T1 + T2
Energy absorbed by 4 resistor in (0, )
T
1 2
Rms = x (t ) dt
T
0 E= i 2 R dt = 4 e t /4
dt
0 0
T1 T
1
= V 2 dt + ( V )2 dt
T = 4 e t /4 dt
0 T1
0
1 2
Rms = [V [T1 0] + V 2 [T T1 ]] t /4
T e t /4
E= 4 = 16[ e ]0
1/4
1 2 0
= [V ][T1 + T T1 ]
T E = –16[0 – 1] = 16 J
1 2
Rms = V T = V2 = V 8. (c)
T
When switch was closed circuit was in steady
6. (b) state,
At steady-state:
Inductor behave as short-circuit.
Capacitor behave as open-circuit, 2 2.5 A
10 k 2F 25 k
+ VC –
2

+ 20
+ +

100 V VC1 40 k V C3 3F Vx +
– – –
iL(0–) = 2.5 A
GATE Previous Years Solved Paper 121

At t = 0+ ; KVL in loop (1),


V 1
I 1 (s ) R + + I 1 (s ) + [ I 1 ( s) I 2 (s )] sL = 0
s sC
2 2.5 A 1 V
I1 (s) R + + sL I 2 (s) sL =
2.5 A sC s
KVL in loop (2),
20 1
[ I 2 (s) I1 (s)] sL + I 2 (s) R + I 2 (s ) =0
+ V – sC
1
V = IR I 1 (s ) sL + I 2 (s) R + sL + =0
sC
= 2.5 × 20 = 50 V
Vx = –50 V 1
R + sL + sL
(Polarity of Vx is given reverse of V) sC I1 (s) V /s
=
1 I 2 (s) 0
sL R + sL +
9. (a) sC
At t = 0– in steady state,
11. (c)
VC
V (s ) 1
R I(s) = =
s + 2 s(s + 2)
V i1(t) i2(t)
1 1 1 1
I(s) = =
s(s + 2) 2 s s+2
1
i1(t) = i2(t) = 0 i(t) = (1 e 2t )
2
VC(0–) = V At t = 0, i(t) = 0
At t = 0+ ; t= , i(t) = 0.5
1
V
t= , i(t) = 0.31
2
R i1 R i(t)

0.5
0.31
–i1R – V – i1R = 0
V t
i1 = 0 1/2
2R
Graph (c) satisfies all conditions.
10. (c)
When switch is in position 2, 12. (b)
KVL :
1/sC
Ldi(t ) 1
v(t) = Ri(t ) + + i(t ) dt
V/s I1(s) I2(s ) + dt C
R 0

– Taking Laplace transform on both sides,
R sL
I ( s ) Vc (0 + )
+ V(s) = RI(s) + LsI (s) LI(0 + ) + +
1 2 1/sC sC s
1 I (s) 1
= I (s ) + sI (s ) 1 +
s s s
122 Electronics Engineering Network Theory

2 I (s) 2 16. (a)


+1 = [s + s + 1]
s s At t = 0+, capacitor is short-circuit and at
s+2 t = , capacitor is open-circuit.
I(s) = 2
s +s+1 10 V
So, Ic(0+) = = 0.5 mA
20 k
13. (b)
Ic( ) = 0
RC = 0.1 × 10–6 × 103 = 10–4
Time constant of the circuit,
= 100 µs
= Req C
As RC is very small, so steady state will be
= 4 µF × 20 k 20 k
reached in 2 second,
= 40 m-sec
Vc = 3 V
Using direct formula,
V2 = –Vc = –3 V
Ic(t) = Ic( ) – [Ic( ) – Ic(0)] e–t/
14. (a) Ic(t) = 0 – (0 – 0.5) e–t/40 m-sec
LdI Ic(t) = 0.5 e–25t mA
V=
dt 17. (b)
V(s) = sLI(s) – LI(0+) At t = 0, the inductor behaves as an open-circuit.
–LI(0+) = –1 mV (Given in So, VL = Is Rs
question)
di +
1 mV VL = L (0 )
I(0+) = = 0.5 A dt
2 mH di + V I R
(0 ) = L = s s
dt L L
15. (c)
18. (c)
1
R1 × The waveform of voltage Vc(t) is shown below.
C1s R1
Z1 = =
1 R1C 1 s + 1 Vc( t)
R1 ×
C1s
1

1k 2T
= t
3 T
4 × 10 s + 1
–1
1
R2 ×
C2 s In mathematical form,
Z2 = 1
R2 × 0 < t < T,
C2 s
C = 1 F, I = 1 A
R2 4k t
= = Vc = dt = t
R2C 2 s + 1 4 × 10 3 s + 1
0

Z2 At t = T, Vc = T
Vo(t) = Vi (t )
Z1 + Z2 t
T < t < 2T, Vc = T dt = 2T t
Z 2 = 4Z 1
0
Z2 4Z1 4Z At t = 2T, Vc = 0
= = 1 = 0.8
Z1 + Z2 Z1 + 4Z1 5Z1
t
vo(t) = 0.8 vi(t) = 0.8 × 10 u(t) 2T < t < 3T, Vc = dt = t 2 T
vo(t) = 8 u(t) 2T
GATE Previous Years Solved Paper 123

At t = 3T, Vc = T t>0
3T < t < 4T,
I(s)
t
Vc = T dt = 4T t 1/sC
R=5k
3T +
+
v(0 ) 100
Vc(t) = tu(t) – 2(t – T) u(t – 2T) =
s s

+ 2(t – 2T) u(t – 2T) ...
C = 0.16 µF
= t u(t ) + 2 ( 1)n (t nT ) u(t nT )
n=1 v(0 + ) t / RC
i(t) = e u(t )
R
19. (d)
v(0+) = 100 V
1 1 1 1 1
Vc(s) = = 2 =
1 s s +s+1 RC 3 6
s+1+ 5 × 10 × 0.16 × 10
s
R = 5k
1 i(t) = 20 e–1250t u(t) mA
= 2
2
1 3
s+ + 23. (a)
2 2
0.75
2 t /2 3 = 0.375 A
i(0+) =
Vc(t) = e sin t 2
3 2
i( ) = 0.5 A
i(t) = i( ) – {i( ) – i(0+)} e–Rt/L
20. (b)
where, R = equivalent resistance seen across L
1 s with current source opened,
VR(s) = 1= 2
1 s +s+1
s+1+ 10
s
1 1 3 2
s+
2 2 2 3
= 2 2 2 2 10 10 R
1 3 1 3
s+ + s+ +
2 2 2 2

t /2 3 1 t /2 3
VR(t) = e cos t e sin t R = 10 + (10 10) = 15
2 3 2
3
i(t) = 0.5 {0.5 0.375} e 15t /15 × 10
t /2 3 1 3
VR(t) = e cos t sin t
= 0.5 – 0.125 e–1000t A
2 3 2

24. (a)
21. (b)
Q
V(0–) = V (0+ ) =

t=0
+
C

+ 0.2 µF 2.5 × 10 3
– = = 50 V
= 0.16 µF

100 V = v(0 )
100 V
+ 50 × 10 6
– = v(0 )
0.3 µF

0.5 µF (Q direction given is opposite)


– V( ) = 100 V
0.8 µF
124 Electronics Engineering Network Theory

2
2
10 or, C=
R
100 V
2
+ 2
V( ) = 100 V L= × 4 = 10 mF
– 40

V(t) = V( ) + [V(0+) – V( )] e–t/ 28. Sol.


= 100 + (–50 – 100) e–t/ At steady state t = 0–,
V(t) = 100 – 150 e–t/ 5k 4k
dV 150 t/
i(t) = C =C e
dt
and = 10 × 50 × 10–6
10 V VOC 1k
= 5 × 10–4
6 150 2 × 103 t
i(t) = 50 × 10 × 6
e
10 × 50 × 10
3 Vc(0) = Vc(0+) = 5 V
= 15 e 2 × 10 t Ampere IL(0–) = IL(0+) = 1 mA
At t = 0, switch get closed,
25. (d)
5k 4k 1k
Since there is no resistance so time constant will
be zero. That means as soon as the switch will
10 V 5V I 1 mA
be closed voltages at C1 and C2 will become
equal and capacitor allows sudden change of
voltage only if impulse of current will pass
through it. Thus, the current through 4 resistance is,
5
27. Sol. I= = 1.25 mA
4 × 10 3
40 4H C
29. (a)
+V –
o
Converting the given circuit into frequency
i( t ) domain and applying KCL at V(s),
R1 V(s)

For critically damped system,


1
= 1= ...(i) 5/s R2 1/Cs
2Q
where, = Damping factor
Q = Quality factor
For series RLC circuit, we get,
1 L 5
Q= ...(ii) V (s )
R C s + V (s) + V (s) = 0 ...(i)
From equation (i) and (ii), R1 R2 1/Cs
1 Q R 1 = 1 k , R2 = 2 k
=1
2 L and C = 1 µF
R C
GATE Previous Years Solved Paper 125

Using the components value we get,


120 B
1 1 5 3V
V (s) + + s =
1 2 s
5 0.1 µF
or, V(s) = ...(ii)
3
s s+
2
Using partial fraction on equation (i), Time constant,
10 10 t = RC = 120 × 0.1 × 10–6
V(s) = ...(iii) vc(t) = 3 + (0 – 3) e–t/
3s 3
3 s+
2 = 3 (1 – e–t/ )
6 t/
Using inverse Laplace transform, C dvc (t ) 0.1 × 10 × 3× e
Ic(t) = =
10 3 /2 t dt
v(t) = [1 e ] ...(iv)
3s 6 t/
0.1 × 10 ×3e 1 t/
V (t ) 5 3 /2 t = 6
= e
Current, I= = [1 e ] mA 120 × 0.1 × 10 40
R2 3

30. Sol. Energy = VI dt


0
At steady state the inductor act as a short-circuit,
Vx ix 1 t/ 3 t/
= 3 ×e = × e
40 40 0
0
2ix +

3 6
10 u(t) A 5 = × 12 × 10 = 0.9 µJ
40
5
32. Sol.
vc(0–) = 0 V
Vx = 5 i x
vc(0+) = 0 V
By KCL,
At t = ,
Vx 2 ix
10 + + ix = 0
5 3

5ix 2 ix
10 + + ix = 0
5 +
8ix 10 V 2 vc( )
or, = 10 –
5
50
ix = A
8
2
vo(t) = 5ix(t) vc( ) = × 10 = 4 V
2+3
5 × 50
= = 31.25 V [By voltage divider]
8
vc(t) = 4[1 – e–t/ ]
31. (c) 3× 2 5
= ReqC = × = 1 sec.
vc (0–)
= 0V 3+2 6
vc(0+) = 0 V vc(1) = 4[1 – e–1/1] = 2.528 Volts
vc( ) = 3 V
126 Electronics Engineering Network Theory

33. Sol. = Req Ceq


= (4 + 2) × 0.1 = 0.6 sec.
1
Initial energy = (C1V12 + C 2V22 ) Vc(t) = Vc( ) + [Vc(0+) – Vc( )] e–t/
2
= 10 + (4 – 10) e–t/0.6
1
(3 × 12 + 1 × 32 ) = 6 J
= Vc(t) = (10 – 6 e–t/0.6) V
2
Final energy stored in capacitor 35. Sol.
1 At t = 0– ;
= (C1 + C 2 ) V 2
2 1
C1V1 + C2V2 = (C1 + C2) V
1×3+3×1 = (1 + 3) V
10
V= 1.5 V 10 V iL (0 ) = = 10 A
1
1
Final energy = (1 + 3) × (1.5)2 = 4.5 J
2
Energy dissipated = 6 – 4.5 = 1.5 J 10
iL(0–) = = 10 A
1
34. (d)
For t > 0 (using Laplace transform)
At t = 0–, switch is at position-1.

3 –3
10 × 10

+ I (s) 106
Vc(s)
10s +
–3
– 10 s
10 V 2 Vc(0 )

3
– 10 × 10
I(s) =
106
10 3 s +
10 × 2 10s
where, Vc(0–) = =4V ...(i)
2+3 106
Vc(s) = I (s ) ×
Vc(0–) = Vc (0+) = 4 V 10 s
At t = ,
10 6
Vc(s) =
4 2 s 2 + 10 8
Taking inverse Laplace, we get,
+ Vc(t) = 100 sin104 t V
Vc( ) 2 5A Steady state magnitude voltage across

capacitor is 100 V.

36. Sol.
Vc( ) = 5 × 2 = 10 V ...(ii) At t = 0–;
The time constant of the circuit is,
+
4 2 V 3F 2
1 –
+
2
i(0 )

2 +
12 V V 2F –
3
GATE Previous Years Solved Paper 127

12
I= =2A
6 5
2 10 A 5
V3F = 10 × =4V
5 –
iL(0 ) = 5 A
3
V2F = 10 × =6V
5
iL(0–) = iL(0+) = 5 A
At t = 0+;
• The Laplace transform model of the circuit
for t > 0 is as follows:
4V 2
1
2
+
i(0 ) 5

10 I(s)
12 V 6V A 5 2.5 s
O.C. s

LiL(0 ) = 12.5 A

4
i(0+) = = 1A
2+2
10 12.5 10 5
Note: As the current direction is not mentioned I(s) = =
s 5 + 2.5s s s+2
in the question, thus by reversing the current
• By taking inverse Laplace transform,
direction 1 A can also be the answer.
i(t) = (10 – 5e–2t) u(t) A
37. Sol. • At t = 0.5 seconds,
Rt
V 5
is(t) = 1 e L i(t) = 10 A = 8.16 A
R e

3t 39. Sol.
15 2
is(t) = 1 e A Loop current,
1
1
i(t) = (1 e t / ) A ; t > 0
Current through 2 H, 1+1
1 L 1 1
i(t) = is (t ) = = = sec.
1+ 2 Req 1 + 1 2
3t
1
i(t) = 5 1 e 2 A i(t) = (1 e 2t ) A ; t > 0
2
At t = 0.5 sec,
At i(t) = 2 A,
1
3t i(t) = (1 e 1 ) A = 0.316 A
2
2 = 5 1 e 2

40. Sol.
By solving, t = 0.3405 sec. T = 3R0C = 3 sec.

38. Sol. t
R(t) = 1 ; 0 t 3 sec
3
• The equivalent circuit at t = 0– is as follows:
128 Electronics Engineering Network Theory

i(t) R(t)
250 Vc( )
+ V – +
R
1V C 5V 250 Vc( )
VR/500

1 By KCL at Vc( ),
R(t ) i(t ) + i(t ) dt = 1
C Vc ( ) 5 VR Vc ( )
+ + =0
t 250 500 250
1 i(t ) + i(t ) dt = 1
3 Vc ( ) 5 5 Vc ( ) Vc ( )
+ + =0
Differentiating both sides, we get, 250 500 250
(VR = 5 – Vc( ))
t di i
1 +i = 0
3 dt 3 1 1 1 5 5
Vc ( ) + =
250 500 250 250 500
di
(3 t ) + 2i = 0 Vc( ) [2 – 1 + 2] = 5(2 – 1)
dt
di 2 5
dt Vc( ) = Volts
= 3
i (3 t )
For time constant, = ReqC
Integrating on both sides, we get,
For Req(V S.C)
ln(i) = 2 ln(3 – t) + ln(c)
i(t) = c(3 – t)2 ; t 0 V-I method:
Given that, i(0) = 1 A 250 I
So, c(3 – 0)2 = 1 A + V –
R
+
1 250 V
c= A –
9 VR/500

1
i(t) = (3 t )2 A
9 By KCL at (V),
T VR VR V
At, t= = 1.5 sec I+ = +
2 250 500 250

1 VR VR V
(1.5)2 = 0.25 A I= +
i(t) = 500 250 250
9
V V V
41. (0.1386) I= + R + (V = –VR)
500 250 250
t=0 2 1 3
250 I= V =V
+ V –
500 500 500
R
S
V 500
5V 250 Vc(t) 0.6 µF = = Req
VR/500 I 3
500
= ReqC = × 0.6 µF
Vc(0) = –5 V = Vo 3
For t > 0, S is closed. 50 × 6
= H = 10 4 sec
For final value at t = (S.S), C O.C. 3
Vc(t) = VC( ) + (VC(0) – VC( )) e–t/
GATE Previous Years Solved Paper 129

5 5 10 4 t 5k
= + 5 e 10 V
3 3
5 4 10 k
10 4 t +
Vc(t) = 5 e ic(0 )
3 3
4
5 20 e 10 t 10 V 1 mA
Vc(t) = Volts t 0
3
By KCL at (10 V)
If, Vc(t) = 0
10
4 1 10 4 t + iC (0 + ) + 1 mA = 0
5 = 20 e 10 t =e 5k
4
iC(0+) = –3 mA
1
ln = –104 t
4 dvC (0 + )
C = –3 mA
+1.386 = +104t dt
t = 1.386 × 10–4
dvC (0 + ) 3m 3m
t = 0.1386 × 10–3 = =
dt C 1m
t = 0.1386 msec
dvC (0 + )
42. (c) = –3 V/sec
dt
Given:
43. (1.44)
P t=0
In steady state the circuit is,
Q S

20 V 20 k iL ( ) = 1 A
+
1A V C( )
– 5k

For t < 0, ‘S’ is in position (P).


At t = 0 – (S.S), L S.C, C O.C iL( ) = 1 A
5k 5k = Ammeter current in steady state
VC( ) = 5 k 1 A = 5 kV
During the transient period the Laplace domain
20 V 20 k Vc(0 )

10 k transformed circuit with zero initial conditions,

– VC(s)
iL(0 )

sL
20
iL(0–) = = 1 mA = iL (0+ ) = I o 1/s 1/sC
20 k
iL(s ) R
By VDR,
20 × 10 k
VC(0–) =
(5 + 5 + 10) k
10 V = VC(0+) = Vo Nodal equation in s-domain
At t = 0+, ‘S’ is in position (Q) 1 VC ( s ) VC ( s )
+ + =0
L O.C. with Io, C S.C. with Vo s 1/ sC R + sL
130 Electronics Engineering Network Theory

1 R + sL Peak overshoot (or) First maxima


VC(s) =
s s 2 LC + RCs + 1 / 1 2
= e
VC (s) 1/ LC
IL(s) = = 0.25 × 3.14
R + sL R 1
s s2 + s + 1 (0.25)2
L LC = e = 0.4443

2
1 iL(t) A
n =
LC 1.444

R R C
and 2 n = =
L 2 L 1.0

5 × 10 3 100 × 10 12
=
2 10 × 10 3 0 TP t(sec)

= 2.5 × 10 +3 10 8 So, the maximum ammeter reading just after the


= 2.5 × 103 × 10–4 switch closed is,
= 0.25 iL (t ) max = 1 + 0.444 = 1.444 A

Answers
EE Transient and Steady-State Response
1. (d) 2. (d) 3. (a) 4. (c) 5. (c) 6. (a) 7. (d) 8. (b)

9. (b) 10. (c) 11. (b) 12. (a) 13. (b) 14. (c) 15. (c) 16. (b)

17. (b) 18. (d) 19. (d) 20. (d) 21. (c) 22. (c) 23. (d) 24. (a)

25. (b) 26. (d) 27. (d) 28. (c) 29. (*) 30. (d) 31. (2)

32. (6.99 × 10–6) 33. (a) 34. (100) 35. (54.99)

Solutions
EE Transient and Steady-State Response
1. (d) 2
Rnet = R 2R = R
Time constant, = Rnet C 3
Rnet = Net resistance across capacitor when all Hence time constant,
the independent voltage sources are short- 2
= RC sec.
circuited and all independent current sources 3
are open-circuited.
2. (d)
When switch is closed, current through
R
capacitor,
S.C. 2R dV (t )
I= C c
Rnet dt
V = RI + Vc(t)
dV (t )
1 = RC c + Vc (t )
dt
GATE Previous Years Solved Paper 131

Q At t = 0+, Vc(t)
Vc(0+) = 0
t/ RC
dV (t ) V(1 – e )
1 = RC c + 0
dt
dVc (0 + ) 1
Hence, = t
dt RC T
i.e., capacitor charges till t = T and then
3. (a)
discharges.
VOC 8 Hence, Vc(max) = V(1 – e– /RC)
R= = 3
=2k
I SC 4 × 10
7. (d)
L = 6 mH
Current through inductor,
L 6 × 10 3
Time constant, = = = 3 µ sec. 1
t
R 2 × 10 3 i(t) = v(t ) dt
L
4. (c)
t
Q Ideal voltage has zero internal resistance, 1
= 12 t 2 dt for t 0 = 4t3 A
Time constant, 1
0
= RC = 0
Hence capacitor will charge instantaneously. 8. (b)
At t = 0+ inductor works as open-circuit, hence
5. (c) complete source voltage drops across it and
di(t ) i( t) consequently, current through the resistor R is
V= L
dt zero. Hence, voltage across the resistor at t = 0+
Constant direct current
For i(t) = I u(t) I is zero, and further with time it rises according
d to VR(t) = (1 – e–Rt/L) u(t).
V= L I u(t ) t
dt V(t)
= LI (t) viz an ideal impulse function
1
6. (a)

R
+
0 t
+
Vs(t) C Vc(t)
– 9. (b)
– For transient free response,
Given, L
tan( t0) =
R
Vs(t )
2 × 50 × 0.01
tan(2 × 50 × t0) =
V 5
1
t 2 × 50 × t0 = tan
0 T 5
i.e., Vs = V[u(t) – u(t – T)] = 32.14° = 0.561 rad
Q Vc(t) = Vs(t) × (1 – e–t/RC) 0.561
t0 = = 1.786 ms
= V(1 – e–t/RC) × [u(t) – u(t – n)] 100
132 Electronics Engineering Network Theory

10. (c) dq (100 C q0 ) 1/ RC


i= = e
Vc(t) = Vc( ) – [Vc( ) – Vc (0)] e–t/RC dt RC
e–t/RC = 40e–1 = 14.7 A
10×10 6
10
×10 3 ×11×10 9 13. (b)
Vc (peak) = 10 (10 0) e 11
At (t 0+), the capacitor act as short-circuit. At
= 10 – (1 – e–1) = 6.32 V
(t ), the capacitor will become open-circuit.
10
[where, Rnet = 10 1 = k ] 10
11
10
Vc( ) = 11 × = 10 V +
10 + 1 VC( )
20 V 10
and –
Q pulse of duration 10 µs is applied.
Hence capacitor charges till 10 µs and then starts
Voltage across capacitor
discharging, so Vc will be maximum at t = 10 µs.
20
= × 10 = 10 V
11. (b) 10 + 10
Before closing the switch, the circuit was not
14. (c)
energized, therefore, current through inductor
Before closing the switch, at t = 0–, the circuit is
and voltage across capacitor are zero.
in steady-state. So, inductor behaves as short-
After closing the switch, at t = 0+ inductor acts
circuit.
as open-circuit and capacitor acts as short-
10
circuit.
Equivalent circuit at t = 0+.
3 10 V iL S.C.

10 V 4 4 VL(0 )
+
10
=1A
iL(0–) =
10
After closing the switch, at t = 0+
Current through inductor can not change
10
I= =2A abruptly.
3+4 4
iL(0+) = iL(0–) = 1 A
VL(0+) = I × (4 4) = 2 × 2 = 4 V
15. (c)

12. (a) Before moving the switch, at t = 0–


The circuit is in steady-state and inductor
Using KVL,
behaves as short-circuit.
dq q
100 = R + The circuit at t = 0–,
dt C
20
dq
100 C = RC +q
dt
q dq 1 t 120 V iL(0 )

= dt
q0 100 C q RC 0 40
100 C – q = (100C – q0) e–t/RC
GATE Previous Years Solved Paper 133

120 1 2
iL(0–) = =2A Remaining Li1 energy in inductor
20 + 40 2
After moving the switch, W1 = 0.05 W0
At t = 0+ = 0.05 × 20 = 1 Joule
Current through inductor can not change 1 2
abruptly. Li1 = 1
2
So, iL(0+) = iL(0–) = 2 A
1
20
× 10 × i12 = 1
2

1
2A i1 = = 0.4472 A
5
R VL
Let at t = T, current decrease to i1,
40 0.4472 = 2e –10T
+ T 0.15 sec.

VL = iL(0+) × {20 + R} 17. (b)


120 = 2 × (20 + R) Voltage across capacitor will discharge through
R = 40 inductor upto voltage across the capacitor
16. (b) becomes zero. During this period, electrostatic
energy stored in capacitor is transferred into
The circuit (in s-domain)
magnetic energy which is stored in inductor.
20
Now inductor will start charging capacitor,
+
magnetic energy in inductor is converted into
LiL(0 ) = 10 × 2 = 20 V
electrostatic energy in capacitor.
I(s) Expression for Vc(t) can be obtained in s-domain.
40 Ls = 10s
As capacitor is charged initially to voltage V0,
then representation of capacitor in s-domain.
40

+
VC(0 ) Vo/s
20
I(s) =
(20 + 40 + 40) + 10s
1
20 2 Zc =
= C sC
=
10s + 100 s + 10
1 1 2 10t
i(t) = L [ I (s)] = L = 2e As current though the inductor is zero at t = 0,
s + 10
then
R eff
+ t
or, i(t) = iL (0 ) e L

(20 + 40 + 40)
t
10t
= 2e 10 = 2e L sL
Initial stored energy in inductor
1 2 +
W0 = LiL (0 )
2
1
= × 10 × 2 2 = 20 Joules
2
134 Electronics Engineering Network Theory

The circuit at t > 0, Voltage across,


R3 = V = 5 V
Vo/s Current through,
V 5
V(s ) I (s) sL R 3 = I1 = = = 5A
R3 1
1/sC Apply KCL,
–I + I1 – IV = 0
Current through voltage source
V0 / s V0
I(s) = = = IV = I1 – I
1 1
+ sL s 2 L + = 5–1=4A
sC C
19. (d)
V0 1
I(s) = R IR
L s2 + 1
LC T1 S1
Vs
Voltage across capacitor = Voltage across
S2
inductor = V(s), + P1
25 V VP C
V(s) = I(s) × (sL) – + –
P2
T1
V0 1 s VT
= × (sL ) = V0 T2
L s2 + 1 1
s2 +
LC LC
All the three windings has same number of
1 turns, so magnitude of induced emf’s in all the
As, 0 =
LC three windings will be same i.e.
s VP = VS = VT
V(s) = V0 2 2
s + 0 Polarity of the windings is decided on the basis
Voltage across the capacitor of dot-convention. As capacitor is charged to
5 V with left plates as positive.
1 V0 s
= V(t) = L–1[(V(s)] = L So, T1 is positive w.r.t T2,
2 2
s + 0
VT = VT1 VT2 = 5 V
V(t) = V0 cos 0t
As T2 has negative polarity. So, P1 has negative
where,
polarity.
1
0 =
LC Therefore, VP = VP1 VP2 = 5 V
Similarly, S1 has negative polarity.
18. (d)
So, VS = VS1 VS2 = 5 V
In steady-state, inductor behaves as short-
Vs 5
circuit and capacitor behaves as open-circuit. IR = = = 0.5 A
R 10
R1 R2

Iv 20. (d)
SW2 R2 = 10
S.C. O.C. I1
O.C. V
+
I S.C. R3
R1 = 10 SW1 L 10 A 10 V C

GATE Previous Years Solved Paper 135

At (t = 0+), the circuit becomes, Charged stored in the capacitor = Area under
VL 10 i - t curve,
Q = A1 + A2
1
+ = (2 × 10 6 ) × (4 × 10 3 )
10 10 A V 10 V
2
– 1
+ (4 + 2) × 10 3 × (5 2) × 10 6
2
6× 3 9
= 4+ × 10 = 13 nC
2
By KCL,
23. (d)
VL VL 10
10 + =0
10 10 Capacitor charged upto 5 µs, so total charge
2VL = 110 stored in capacitor = Q = 13 nC.
VL = 55 V Voltage across the capacitor before connecting
to inductor,
55 10
IC = = 4.5 A
10 Q 13 × 10 9
V0 = = = 43.33 V
C 0.3 × 10 9
21. (c) Voltage across the capacitor at time t,
For finding time constant, we neglect current Vc(t) at t = 1 µs
source as a open-circuit.
Vc (t ) t = 1 µs = [V0 cos 0t]t = 1 µs
Circuit becomes,
1F 3 1 6
1F 3 × 1 × 10
0t = 3 9
0.6 × 10 × 0.3 × 10

1F 1F 3 2F 3
= 2.357 rad = 135°

Vc (t ) t = 1 µs = 43.33 × cos135°

–30.6 V

Ceq = 2/3 F
24. (a)
6 = Req
At t = 0–, S1 is closed, S2 is open.

Time constant = Req Ceq


2
= 6× = 4 sec. 3V C1 C2
3

22. (c)
C1 gets charged upto 3 V
i(t) mA
Charge stored in C1,
4 Q0 = C1V = 1 × 3 = 3C
Voltage across C2 is zero at t = 0–, so no charge
2 is stored in C2.
A1 A2 At t > 0, S1 is open and S2 is closed.
µs Charge stored (Q 0 ) initially in C 1 gets
2 µs 5 µs
redistributed between C1 and C2.
136 Electronics Engineering Network Theory

IC
Q1 Q2 +
5V 4V 4
C1 V C2 –

Let charge stored in C1 = Q1 Current through capacitor at t = 0+


Charge stored in C2 = Q2 4
According to conservation of charge IC(0+) = = 1A
4
Q1 + Q2 = Q0 = 3 ...(i)
Voltage across C1 = Voltage across C2, 26. (d)
Q1 Q2 Q1 Q2 Initial current through the inductor is zero and
= =
C1 C2 1 2 capacitor voltage is charged upto voltage,
Q2 = 2Q1 VC(0–) = 100 V
Solving equation (i) and (ii) we get, As current through inductor and voltage across
Q1 = 1 C and Q2 = 2C capacitor can not change abruptly.
Voltage across combination So, after closing the switch,
iL(0+) = iL(0–) = 0
Q1 1
= = =1V and VC(0+) = VC(0–) = 100 V
C1 1
The circuit is s-domain,
Alternate method:
sL
VC1 C 1 + VC 2 C 2
VC1 = VC 2 = Vc (0 + ) 100
C1 + C 2 =
s s
I (s )
25. (b) 1/sC
As the switch has been closed for a long time,
the circuit is in steady-state. At steady-state,
capacitor is open-circuit, 100 / s 100 1
I(s) = =
1 L s2 + 1
I 1 sL +
sC LC


5V VC(0 ) 4
C 1/ LC
= 100
L 2
1
– s2 +
Circuit at t = 0 LC
Using KVL, Taking inverse Laplace transform,
5 – I – 4I = 0 i(t) = L –1[I(s)]
I = 1A
C 1
VC(0–) = 4 × 1 = 4 V = 100 sin t
L LC
As the voltage across capacitor can not change
10 × 10 3 1
abruptly. = 100 × 3
× sin t
1 × 10 1 × 10 3
× 10 × 10 6
So, VC(0+) = VC(0–) = 4 V
Circuit at t = 0+ i(t) = 10 sin(104 t) A
GATE Previous Years Solved Paper 137

27. (d) Putting values of R, C and I we get,


3
1 3 20 × 10 1
S Vs(s) = s (20 × 10 × 200 2) + 6
×
s
10

t=0 1 3 1 20 × 10 3
= (2 2) + 20 × 10 × =
C1 C2
s s s2
i(t)
20 × 10 3
Vs(s) =
s2
Circuit is s-domain, or, Vs(t) = 20000t u(t)
Vs(s) = (20000)t u(t)...
i(s)
which is the equation of a straight line passing
through origin.
12/s
Hence, option (c) is correct.
1/sC2
29. (*)
1/sC1
Circuit for t < 0,

By applying KVL,
R
12 I (s ) 1 1
+ + =0 V01
s s C1 C 2 + – +
Vc(0 ) = Vc(0 ) = V01
12C1C2 –
I(s) = = k (constant)
C1 + C 2
Circuit for t = :
i(t) = k (t) In steady-state capacitor becomes open-circuit.
Current i(t) is an impulse function. Vc( ) = –V02
28. (c)
R
Given: C = 1 µF, vc(0) = –2 V R
R = 100 , I = 20 mA V02

Circuit for the given condition at time t > 0 is Vc ( ) = V02
shown below. +

We know that,
VC2 (0) 2
= Vc(t) = Vc( ) – [Vc( ) – Vc(0+)] e–t/
s s I/Cs
= Time constant of given circuit
= 2 RC
+ Vc(t) = –V02 – (V02 – V01) e–t/2RC
I/s Vs(s) I/s R
= (V02 – V01) – (V02 – V01) e–t/2RC – V01

or, Vc(t) = (V02 + V01) (e–t/2RC – 1) + V01

30. (d)
Applying KVL we have, R

2 1 1 I
Vs(s) = + R+
s s Cs
V L
1 I 1 I
= 2 + IR + = ( IR 2) +
s Cs s Cs
138 Electronics Engineering Network Theory

Initially (t = 0 – ) the inductor would be KCL at node A,


uncharged. VA 3 3 VA 3
+ + =0
So, I(0+) = 0 1 2 2
The KVL in the loop will be 2(VA – 3) + 3 + (VA – 3) = 0
dI 3 VA = 6, VA = 2
V = RI + L
dt
di(0 + )
dI + VA = L =2
At t = 0+, V = RI (0+ ) + L (0 ) dt
dt
Since, I(0+) = 0 di(0+ ) 2 2
= = = 2 A/sec
dI + dt L 1
V
So, (0 ) =
dt L 32. Sol.
Now, lets differentiate the above equation,
S1
dV dI d2 I
So, = R +L 2
dt dt dt
4V 5 µF 5
dI d2 I
0 = R +L 2
dt dt
dI + dI 2
At t = 0+, 0 = R (0 ) + L 2 (0 + ) 4 t/
dt dt i(t) = e
5
d2 I R t = RC = 25 × 10–6 sec.
So, 2
(0 + ) = 2
V
dt L Change lost by capacitor from t = 25 µs to 100 µs
is
31. Sol.
100 µ sec
S1
1 i(t ) dt = 6.99 × 10–6 C
S2
25 µ sec

2 33.. (a)
3V 1H
From the given circuit, consider the following
3V circuit diagram,
8
At t = 0–,
6
8 32
2 32
3/2 A 50 V

3V
After rearrangement,
5A 8
iL(0+) = iL(0–) = 1.5 A
At t = 0+, 6

1 8 i(0 ) = 2.5 A

50 V
2
3V 1H
For t 0, I0 = i(0–) = 2.5 A
3V we can write, i(t) = I0 e–Rt/L
i(t) = 2.5 e–4t A
GATE Previous Years Solved Paper 139

34. Sol. 35. Sol.


Consider the following circuit diagram, If initial charge polarities on the capacitor is
opposite to the supply voltage then only the
1F capacitor voltage crosses the zero line.
5 5
R
5
10 V
t=0
5 5 10 V C

After minimizing circuit elements we can have


the following circuit, Vc(t) Final value + (Initial value – Final value)
e–t/ ,
1F
0 = 10 + (–V0 – 10) e–0.4
10 = (V0 + 10) e–0.4
10 V i (t ) 5 V0 = 4.918 V
Now, t = 0.2
0 = 10 + ( V0 10) e 0.2
Here, = RC = 5 sec.
V0 = 2.214
Now current,
% change in voltage
V t/ 10 t /5 0.2 t
i(t) = e = e = 2e 4.918 2.214
R 5 = × 100%
Energy supplied by the source, 4.918
= 54.99%
0.2 t
E= 10 × 2 e dt
0

= 100 J
5 Two Port Networks

ELECTRO NICS EN GINEERIN G 2 1 2 8


(a) 8 3 (b) 8 3
(GATE Previous Years Solved Papers)

Q.1 Two 2-port networks are connected in parallel. 0 1 2 1


(c) 1 0 (d) 1 3
The combination is to be represented as a single
two-port network. The parameters of this [EC-1990 : 2 Marks]
network are obtained by addition of the
individual Q.5 Two 2-port networks are connected in cascade.
(a) z-parameters The combination is to the represented as a single
two-port network. The parameters of the
(b) h-parameters
network are obtained by multiplying the
(c) y-parameters
individual
(d) ABCD parameters
(a) z-parameter matrics
[EC-1988 : 2 Marks]
(b) h-parameter matrics
Q.2 For the transfer function of a physical two-port (c) y-parameter matrics
network:
(d) ABCD parameter matrics
(a) all the zeros must lie only in the left half of
[EC-1991 : 2 Marks]
the s-plane.
(b) the poles may lie anywhere in the s-plane. Q.6 For a two-port network to be reciprocal
(c) the poles lying on the imaginary axis must (a) z11 = z22 (b) y21 = y12
be simple. (c) h21 = –h12 (d) AD – BC = 0
(d) a pole may lie at origin. [EC-1992 : 2 Marks]
[EC-1989 : 2 Marks]
Q.7 The condition, that a two-port network is
Q.3 The condition AD – BC = 1 for a two-port reciprocal, can be expressed in terms of its ABCD
network implies that the network is a parameters as _______ .
(a) reciprocal network [EC-1994 : 1 Mark]
(b) lumped element network
Q.8 The short-circuit admittance matrix of a two-
(c) lossless network
port network is
(d) unilateral element network
[EC-1989 : 2 Marks] 0 1/2
1/2 0
Q.4 The open-circuit impedance matrix of the two-
port network shown in figure is The two-port network is
(a) non-reciprocal and passive
I1 2 I2
(b) non-reciprocal and active
+ +
(c) reciprocal and passive
V1 1 3I1 V2 (c) reciprocal and active
[EC-1998 : 1 Mark]
– –
GATE Previous Years Solved Paper 141

Q.9 A two-port network is shown in the figure. The Q.12 The impedance parameters Z11 and Z12 of the
parameters h21 for this network can be given by two-port network in the figure are
2 2 2
I1 R R I2
1 2
+ +
1 1
V1 R V2

– – 1 2
(a) Z11 = 2.75 and Z12 = 0.25
1 1 (b) Z11 = 3 and Z12 = 0.5
(a) (b) +
2 2
(c) Z11 = 3 and Z12 = 0.25
3 3
(c) (d) + (d) Z11 = 2.25 and Z12 = 0.5
2 2
[EC-2003 : 2 Marks]
[EC-1999 : 1 Mark]
Q.13 For the lattice circuit shown in the figure,
Q.10 The admittance parameter y12 in the two-port
Za = j2 and Zb = 2 . The value of the open-
network in figure is
Z11 Z12
I1 20 I2 circuit impedance parameters, are
Z21 Z22

Zb
E1 5 10 E2
1 3

Za

(a) –0.2 (b) 0.1 Za

(c) –0.05 (d) 0.05


2 4
[EC-2001 : 1 Mark] Zb

Q.11 The Z-parameters Z11 and Z21 for the two-port 1 j 1+ j 1 j 1+ j


network in the figure are (a) 1+ j 1+ j (b) 1+ j 1 j
I1 2 I2 1+ j 1+ j 1+ j 1+ j
(c) 1 j 1 j (d) 1+ j 1+ j
4
E1 E2 [EC-2004 : 2 Marks]

10 E1 Q.14 The ABCD parameters of an ideal n : 1

n 0
6 16 transformer shown in the figure are .
(a) Z11 = ; Z21 = 0 X
11 11
6 4 The value of X will be
(b) Z11 = ; Z21 =
11 11 I1 I2
6 16
(c) Z11 = ; Z21 =
11 11
V1 V2
4 4
(d) Z11 = ; Z21 =
11 11
[EC-2001 : 2 Marks] n:1
142 Electronics Engineering Network Theory

(a) n (b) 1/n Linked Answer Questions (18 and 19):


(c) n2 (d) 1/n2 A two-port network shown below is excited by external
[EC-2005 : 1 Mark] dc sources. The voltages and currents are measured
with voltmeters V1, V2 and ammeters A1, A2 (all assumed
Q.15 The h-parameters of the circuit shown in the
to be ideal) as indicated. Under following switch
figure are
conditions, the readings obtained are:
I1 10 I2
S1 S2
+ + + A –
1 2
– A +
1 2

V1 20 V2
+ + +
Two-port +
6V V1 V2 1.5 V
– – Network
– – – –

0.1 0.1 10 1
(a) (b) 1 2
0.1 0.3 1 0.05
(i) S1 – open, S2 – closed A1 = 0 A, V1 = 4.5 V,
30 20 10 1
(c) (d) V2 = 1.5 V, A2 = 1 A
20 20 1 0.05
(ii) S1 – closed, S2 – open A1 = 4 A, V1 = 6 V,
[EC-2005 : 2 Marks]
V2 = 6 V, A2 = 0 A
Q.16 A two-port network is represented by ABCD
Q.18 The Z-parameter matrix for this network is
parameters given by
1.5 1.5 1.5 4.5
V1 A B V2 (a) (b)
= 4.5 1.5 1.5 4.5
I1 C D I2
1.5 4.5 4.5 1.5
If port-2 terminated by RL, the input impedance
(c) 1.5 1.5 (d) 1.5 4.5
seen at port-1 is given by
A + BRL ARL + C [EC-2008 : 2 Marks]
(a) (b)
C + DRL BRL + D Q.19 The h-parameter matrix for this network is

DRL + A B + ARL 3 3 3 1
(c) (d) (a) (b)
BRL + C D + CRL 1 0.67 3 0.67
[EC-2006 : 1 Mark]
3 3 3 1
(c) 1 0.67 (d) 3 0.67
Q.17 In the two-port network shown in the figure
below, Z12 and Z21 are, respectively [EC-2008 : 2 Marks]
I1 I2
Q.20 For the two-port network shown below, the
short-circuit admittance parameter matrix is
re I1 ro 0.5
1 2

0.5 0.5
(a) re and ro (b) 0 and – ro
(c) 0 and ro (d) re and – ro 1 2
[EC-2006 : 1 Mark]
GATE Previous Years Solved Paper 143

4 2 1 0.5 Q.23 With 10 V dc connected at port A, the current


(a) S (b) S drawn by 7 connected at port B is
2 4 0.5 1
3 5
1 0.5 4 2 (a) A (b) A
(c) S (d) S 7 7
0.5 1 2 4
9
[EC-2010 : 1 Mark] (c) 1 A (d) A
7
Q.21 In the circuit shown below, the network N is [EC-2012 : 2 Marks]
described by the following Y matrix:
Q.24 In the h-parameter model of the two-port
0.1 S 0.01 S network given in the figure shown, the value of
Y=
0.01 S 0.1 S h22 (in S) is ______ .
The voltage gain V2/V1 is
3

25 I1 I2
+ + 1 3 3 2

100 V V1 N V2 100
2
– – 1 2 2 2

1 1
(a) (b) [EC-2014 : 2 Marks]
90 90
1 1 Q.25 Consider the building block called ‘Network N’
(c) (d)
99 11 shown in the figure.
[EC-2011 : 2 Marks] Let, C = 100 µF and R = 10 k .

Common Data Questions (22 and 23): Network (N)


With 10 V dc connected at port A in the linear non-
reciprocal two-port network shown below, the following C
were observed.
V1(s) R V2(s)
(i) 1 connected at port B draws a current of 3 A.
(ii) 2.5 connected at port B draws a current of 2 A.

+ +
A B
Two such blocks are connected in cascade, as
– –
shown in the figure.
Q.22 For the same network, with 6 V dc connected at
port A, 1 connected at port B draws 7/3 A. If +
8 V dc is connected to port A, the open-circuit
Network Network
voltage at port B is V1(s)
(N) (N)
V3(s)

(a) 6 V (b) 7 V

(c) 8 V (d) 9 V
[EC-2012 : 2 Marks]
144 Electronics Engineering Network Theory

The transfer function V3(s)/V1(s) of the cascaded 3.5 + j 2 20.5


network is (a)
20.5 3.5 j2
2
s s
(a) (b) 3.5 + j 2 0.5
1+ s 1 + 3s + s 2 (b)
0.5 3.5 j 2
2
s s
(c) (d) 10 2 + j0
1+s 2+s
(c)
[EC-2014 : 2 Marks] 2 + j0 10

Q.26 For the two-port network shown in the figure, 7 + j4 0.5


(d)
the impedance (Z) matrix (in ) is 30.5 7 j4
30 [EC-2015 : 2 Marks]
1 2
+
Q.29 Consider a two-port network with the
10 60 transmission matrix:
A B
– T=
1 2 C D
6 24 9 8 If the network is reciprocal, then
(a) 42 9 (b) 8 24 (a) T–1 = T
9 6 42 6 (b) T2 = T
(c) 6 24 (d) 6 60 (c) Determinant (T) = 0
[EC-2014 : 2 Marks] (d) Determinant (T) = 1
[EC-2016 : 1 Mark]
Q.27 The two-port admittance matrix of the circuit
shown is given by Q.30 The Z-parameter matrix for the two-port
5 network shown is
2j j
j 3+ 2j
10 10
where the entries are in .
Suppose, Zb(j ) = Rb + j
0.3 0.2 15 5 Za Zb
(a) (b) 1 2
0.2 0.3 5 15
3.33 5 0.3 0.4 Zc
(c) 5 3.33 (d) 0.4 0.3
[EC-2015 : 1 Mark] 1 2
Then the value of Rb (in ) equals _____ .
Q.28 The ABCD parameters of the following two-port
network are [EC-2016 : 1 Mark]

(5 + j4) (5 – j4)
Z11 Z12
Q.31 The Z-parameter matrix for the
Z21 Z22
(2 + j0)
two-port network shown is
GATE Previous Years Solved Paper 145

3 Two-port network

Input port Output port

2:1
6
Port-1 Port-2
2 2 2 2 [EC-2020 : 2 Marks]
(a) 2 2 (b) 2 2
Q.35 Consider the two-port network shown in the
9 3 9 3
figure.
(c) 6 9 (d) 6 9
I1 1 I2
[EC-2016 : 2 Marks]
+ +
Q.32 The ABCD matrix for a two-port network is
V1 1 3V2 1 V2
defined by
– –
V1 A B V2
=
I1 C D I2 The admittance parameters, in Siemens are
I1 2 2 I2 (a) y11 = 1, y12 = –2, y21 = –1, y22 = 3
+ + (b) y11 = 2, y12 = –4, y21 = –4, y22 = 2
(c) y11 = 2, y12 = –4, y21 = –4, y22 = 3
V1 5 V2
(d) y11 = 2, y12 = –4, y21 = –1, y22 = 2
– – [EC-2021 : 2 Marks]

The parameter B for the given two-port network Q.36 A linear two-port network is shown in Fig. (a).
(in , correct to two decimal places) is ____ . An ideal DC voltage source of 10 V is connected
[EC-2018 : 1 Mark] across Port-1. A variable resistance R is
connected across Port-2. As R is varied, the
Q.33 In the given circuit, the two-port network has
measured voltage and current at Port-2 is shown
40 60 in Fig. (b) as a V2 versus –I2 plot. Note that for
the impedance matrix [ Z ] = . The V2 = 5 V, I2 = 0 mA and for V2 = 4 V, I2 = –4 mA.
60 120
When the variable resistance R at Port-2 is
value of ZL for which maximum power is replaced by the load shown in Fig. (c), the
transferred to the load is ______ . current I2 is ______ mA (Rounded off to one
10 I1 I2 decimal places).
+ +
I1 I2 A
120 V V1 [Z] V2 ZL + +

– –
10 V V1 Networks V2 R

[EC-2020 : 1 Mark]
– –
Q.34 For a two-port network consisting of an ideal B
lossless transformer, the parameter S21 (rounded Fig. (a)

off to two decimal places) for a reference


impedance of 10 , is ______ .
146 Electronics Engineering Network Theory

(a) higher voltage (b) lower impedance


V2
(c) greater power (d) better regulation
5V
1k [EE-2001 : 1 Mark]
4V
Q.5 A two-port network, shown in figure, is
10 V described by the following equations:
–I2 I1 = Y11E1 + Y12E2
Fig. (b) 4 mA Fig. (c)
I2 = Y21E1 + Y22E2
[EC-2022] 2 2
I1 I2

ELECTRICAL EN GINEERIN G
(GATE Previous Years Solved Papers) E1 2 E2

Q.1 If a two-port network is reciprocal, then we have,


with the usual notation, the following
The admittance parameters, Y11, Y12, Y21 and
relationship
Y22 for the network shown are
(a) h12 = h21 (b) h12 = –h21
(a) 0.5 , 1 , 2 and 1 respectively.
(c) h11 = h22 (d) h11h22 – h12h21 = 1
[EE-1994 : 1 Mark] 1 1 1 1
(b) , , and respectively.
3 6 3 3
Q.2 For the two-port network shown in figure, the
(c) 0.5 , 0.5 , 1.5 and 2 respectively.
admittance matrix is _______ .
2 3 3 2
I1 10 I2 (d) , , and respectively.
5 7 7 5
+ +
[EE-2000 : 2 Marks]
V1 10 10 V2
Q.6 The h-parameters for a two-port network are
– –
defined by
[EE-1997 : 2 Marks]
E1 h11 h12 I1
=
Q.3 A two-port device is defined by the following I2 h21 h22 E2
pair of equations: For the two-port networks shown in figure, the
i1 = 2V1 + V2 and i2 = V1 + V2 value of h12 is given by

Z11 Z12 I1 4 2 2 I2
Its impedance parameters are
Z21 Z22

given by E1 2 4 E2
2 1 1 1
(a) 1 1 (b) 1 2
1 1 2 1 (a) 0.125 (b) 0.167
(c) 1 2 (d) 1 1 (c) 0.625 (d) 0.25
[EE-2003 : 2 Marks]
[EE-2000 : 2 Marks]
Q.7 The Z-matrix of a two-port network is given by
Q.4 A passive two-port network is in steady-state.
Compared to its input, the steady-state output 0.9 0.2
Z=
can never offer 0.2 0.6 .
GATE Previous Years Solved Paper 147

The element Y22 of the corresponding Y matrix Ri Ro


of the same network is given by
+
(a) 1.2 (b) 0.4
(c) –0.4 (d) 1.8 Vi + AVi

[EE-2004 : 2 Marks]

Q.8 For the two-port network shown in the figure
the Z-matrix is given by (a) 1 V, , 10 (b) 1 V, 0, 10

Z2
(c) 1 V, 0, (d) 10 V, , 10
i1 i2
[EE-2006 : 2 Marks]

V1 Z1 V2 Q.11 The parameter type and the matrix


representation of the relevant two-port
parameters that describe the circuit shown are:
Z1 Z1 + Z2 I1 I2
(a)
Z1 + Z2 Z2 + +

Z1 Z1
(b) V1 V2
Z1 + Z2 Z2
– –
Z1 Z2
(c)
Z1 Z1 + Z2 0 0
(a) z-parameters,
Z1 Z1 0 0
(d)
Z1 Z1 + Z2 1 0
(b) h-parameters,
[EE-2005 : 1 Mark] 0 1

Q.9 Two networks are connected in cascade as 0 0


(c) g-parameters,
shown in the figure. With the usual notations 0 0
the equivalent A, B, C and D constants are
1 0
obtained. Given that, C = 0.025 45°, the value (d) z-parameters,
0 1
of Z2 is
[EE-2006 : 2 Marks]
Z1 = 10 30°

Q.12 The two-port network ‘P’ shown in the figure


Z2 has port 1 and 2 denoted by terminals (a, b) and
(c, d), respectively. It has an impedance matrix
Z with parameters denoted by Zij, A 1 resistor
(a) 10 30° (b) 40 –450° is connected in series with the network at port 1
(c) 1 (d) 0 as shown in the figure. The impedance matrix
of the modified two-port network (shown as a
[EE-2005 : 2 Marks]
dashed box) is
Q.10 The parameters of the circuit shown in the figure
1 a
are: e c

Ri = 1 M , Ro = 10 , A = 106 V/V P
If Vi = 1 µV, the output voltage, input impedance
and output impedance respectively are f b d
148 Electronics Engineering Network Theory

Z11 + 1 Z12 + 1 s 4 + 3s 2 + 1 s 4 + 2s 2 + 4
(a) (a) (b)
Z21 Z22 + 1 s3 + 2s s2 + 2
Z11 + 1 Z12
(b) s2 + 1 s3 + 1
Z21 Z22 + 1 (c) (d)
s 4 + s2 + 1 s 4 + s2 + 1
Z11 + 1 Z12 [EE-2014 : 1 Mark]
(c)
Z21 Z22
Q.16 The Z-parameter of the two-port network shown
Z11 + 1 Z12 in the figure are:
(d) [EE-2010 : 2 Marks]
Z21 + 1 Z22 Z11 = 40 , Z12 = 60 , Z21 = 80 and Z22 = 100
Common Data for Questions (13 and 14): The average power delivered to RL = 20 , in
Watts, is _______ .
With 10 V dc connected at port ‘A’ in the linear non-
reciprocal two-port network shown below, the following 10 I1 I2
were observed: + +
(i) 1 connected at port ‘B’ draws a current of
3 A. 20 V V1 [Z] V2 RL

(ii) 2.5 connected at port ‘B’ draws a current


of 2 A. – –

+ + [EE-2016 : 2 Marks]
A B
– – Q.17 The driving point input impedances seen from
the source Vs of the circuit shown below (in ),
Q.13 For the same network with 6 V dc connected at is _______ .
port ‘A’, 1 connected at port ‘B’ draws 7/3 A.
Is 2 2
If 8 V dc is connected to port ‘A’, the open-circuit
+ V –
voltage at port ‘B’ is 1

(a) 6 V (b) 7 V
Vs 3 4V1 4
(c) 8 V (d) 9 V
[EE-2012 : 2 Marks]

Q.14 With 10 V dc connected at port ‘A’, the current [EE-2016 : 2 Marks]


drawn by 7 connected at port ‘B’ is
Q.18 Two passive two-port networks are connected
3 5
(a) A (b) A in cascade as shown in figure. A voltage source
7 7
is connected at port 1.
9
(c) 1 A (d) A I1 I2 I3
7
+ + +
[EE-2012 : 2 Marks] Two-port Two-port
V1 V2 V3
Network-1 Network-2
Q.15 The driving point impedance Z(s) for the circuit –
– –
shown below is Port-1 Port-2 Port-3
1H 1H
Given, V1 = A1V2 + B1I2
I1 = C1V2 + D1I2
Z(s) 1F 1F
V2 = A2V3 + B2I3
I2 = C2V3 + D2I3
GATE Previous Years Solved Paper 149

A1, B1, C1, D1, A2, B2, C2 and D2 are in generalized Q.19 In the two-port network shown, the h 11
circuit constants. If the Thevenin equivalent parameter (where, h11 = V1/Ii , when V2 = 0)
circuit at port 3 consists of a voltage source VT (in ) is _______ (upto 2 decimal places).
and an impedance ZT, connected in series, then 2I1
V1 A B + B1D2
(a) VT = , ZT = 1 2
A1 A2 A1 A2 + B1C 2
1 1

V1 A B + B1D2 + +
(b) VT = , ZT = 1 2 I1
A1 A2 + B1C 2 A1 A2
V1 1 V2
V1 A B + B1D2
(c) VT = , ZT = 1 2 – –
A1 + A2 A1 + A2

V1 A B + B1D2 [EE-2018 : 1 Mark]


(d) VT = , ZT = 1 2
A1 A2 + B1C 2 A1 A2 + B1C 2
[EE-2017 : 2 Marks]

Electronics & Electrical Engineering


GATE Previous Years Solved Paper

A n swe rs & Expl a n a t i o n s

Answers
EC Two-Port Network

1. (c) 2. (c, d) 3. (a) 4. (a) 5. (d) 6. (b, c) 7. (1) 8. (b)


9. (a) 10. (c) 11. (c) 12. (a) 13. (d) 14. (b) 15. (d) 16. (d)
17. (b) 18. (c) 19. (a) 20. (a) 21. (d) 22. (c) 23. (c) 24. (1.25)
25. (b) 26. (c) 27. (a) 28. (b) 29. (d) 30. (3) 31. (a) 32. (4.80)
33. (48) 34. (0.8) 35. (d) 36. (4)

Answers
EC Two-Port Network

1. (c) 3. (a)
[Y] = [Y]A + [Y]B For reciprocal network,
AD – BC = 1
2. (c, d)
The poles lying on the imaginary axis must be 4. (a)
simple. A pole may lie at origin. V1 1 × I2
Z 12 = = =1
I2 I1 = 0 I2
150 Electronics Engineering Network Theory

V1 I1 (y3) I2
Z 22 =
I2 I1 = 0
20
2 I 2 × 1I 2
Z 22 = =3 E1 (y1) 5 10 (y2) E2
I2
2 I1 × 1
Z 11 = = 2
I1 1
y 12 = = 0.05
20
V2
Z 21 =
I1 I2 = 0 11. (c)

6 I1 + V1 6 I1 2 I1 V1 = Z11I1 + Z12I2
Z 21 = = V2 = Z21I1 + Z22I2
I1 I1
Z 21 = –8 V1
Z 11 =
I1 I2 = 0
5. (d)
ABCD parameter matrices, Applying KVL in LHS loop,
E1 – 2I1 – 4I1 + 10E1 = 0
A B A1 B1 A2 B2
= 11E1 = 6I1
C D C1 D1 C2 D2
E1 6
=
6. (b, c) I1 11

y 21 = y 12 V2
Z 21 =
h 21 = –h 12 I1 I2 = 0

7. Sol. KVL in RHS loop,


AD – BC = 1 E2 – 4I1 + 10E1 = 0
6 b
8. (b) E2 4 I 1 + 10 ×I1 = 0 E1 = I1
11 11
Y 12 Y 21
11E2 – 44I1 + 60I1 = 0
So, the given two-port network is non-reciprocal
E2 16
and active. =
I1 11
9. (a)
12. (a)
I2 = h21I1 + h22V2
Using -Y conversion,
I
h 21 = 2 2 2 2
I1 V2 = 0 1 2

when, V2 = 0 = I2R(I1 + I2) R


1 3
I1 I2 1
I2 = = 1 1
2 I1 2
2

10. (c)
1 2
y1 + y 3 y3 y11 y12 2×1 2
= R1 = = = 0.5
y3 y2 + y3 y 21 y 22 4 4
y 12 = –y 3
GATE Previous Years Solved Paper 151

1× 1 1 V1 V1
R2 = = = 0.25 h 11 = , h12 =
4 4 I1 V2
V2 = 0 I1 = 0
2×1
R3 = = 0.5
4 I2 I2
h 21 = , h22 =
2 0.5 0.5 2 I1 V2
1 2 V2 = 0 I1 = 0
(Z1) (Z2) When, V2 = 0
(Z3) 0.25 I1 10 I2
+

1 2 V1
Z1 + Z3 Z3
Z3 Z2 + Z3 –

Z 11 = Z1 + Z3 I1 = –I2
= 2.5 + 0.25 = 2.75 I2
= –1 = h21
Z 12 = Z3 = 0.25 I1
V1 = 10I1
13. (d)
V1
For Lattice network, Z-parameter is given as, = 10
I1
Za + Zb Za Zb
When, I1 = 0
2 2
Za Zb Za + Zb V1 = V2
2 2 V1
= h12 = 1
Za = 2j, Zb = 2 V2

1+ j 1+ j (As no drop in 10 resistance)


1+ j 1+ j V2 = 20I2
I2 1
14. (b) = h22 = = 0.05
V2 20
V1 A B V2
= 10 1
I1 C D I2
1 0.05
I2 V n
= 1 =
I1 V2 1 16. (d)
V1 = AV2 – BI2 I1 I2

V1
A= =n V1 RL V2
V2 I2 = 0

I1 V2 1 V1 = AV2 – BI2
D= = =
I2 V1 n I1 = CV2 – DI2
V2 = 0
V2 = –I2RL
15. (d) V1 AV2 BI 2 A I 2 RL BI 2
= =
V1 = h11I1 + h12V2 I1 CV2 DI 2 C I 2 RL DI 2
I2 = h21I1 + h22V2 ARL + B
Input impedance =
CRL + D
152 Electronics Engineering Network Theory

17. (b) 19. (a)


V1 = Z11I1 + Z12I2 V1 = h11I1 + h12V2
V2 = Z21I1 + Z22I2 I2 = h21I1 + h22V2

V1 V1 4.5
Z 12 = h 12 = = =3
I2 I1 = 0
V2 I1 = 0
1.5

V2 I2 1
Z 12 = h 22 = = = 0.67
I1 I2 = 0
V2 I1 = 0
1.5

When, I1 = 0
V1
V1 = 0 h 11 =
I1 V2 = 0
V1
= 0 = Z12 When, V2 = 0, Z21I1 + Z22I2 = 0
I2
When, I2 = 0, V2 = – I1ro Z21 I1
I2 =
Z22
V2
= – ro = Z21
I1 Z21 I 1
V1 = Z11 I1 + Z12
Z22
18. (c)
When, I= 0 V1 Z12 Z21
= Z11 = h11
then, V1 = 4.5 V I1 Z22
V2 = 1.5 V
4.5 × 1.5
I2 = 1A h 11 = 1.5 = 3
1.5
V1 = Z11I1 + Z12I2
V2 = Z21I1 + Z22I2 I2 Z21
h 21 = =
I1 Z22
V1 4.5 V2 = 0
Z 12 = = = 4.5
I2 1
I1 = 0 1.5
= = 1
1.5
V2 1.5
Z 22 = = = 1.5 3 3
I2 I1 = 0
1 So, h-parameter matrix is .
1 0.67
When, I2 = 0
then, I1 = 4A 20. (a)
V1 = 6V Short-circuit admittance parameters for a 2-port
V2 = 6V -network are,
V1 6 Y 11 = Ya + Yb
Z 11 = = = 1.5 Y 12 = Y21 = –Yb
I1 I2 = 0
4
Y 22 = Yb + Yc
V2 6 Yb
Z 21 = = = 1.5
I1 I2 = 0
4 1 2

1.5 4.5 Ya Yc
So, Z-parameter matrix is .
1.5 1.5
1 2
GATE Previous Years Solved Paper 153

For the given network, For VDC = 6 V, RL = 1 , I = 7/3


1 VTh = I(RTh + RL)
Ya = Yb = Yc = =2
0.5 7
VTh = (2 + 1) = 7 V ...(iv)
So, Y 11 = 2 + 2 = 4 3
Y 12 = Y21 = –2 Q The network is linear and non-reciprocal, it
Y 22 = 2 + 2 = 4 may contain dependent voltage source.
VTh = aV + b ...(v)
21. (d)
9 = a10 + b [From equation (iii)]
I2 = Y21V1 + Y22V2
and 7 = a6 + b [From equation (iv)]
I2 = 0.01V1 + 0.1V2 ...(i)
From given figure, 1
Solving, we get, a = and b = 4
V2 = –I2RL = –100I2 2

V2 V 8
I2 = VTh = +4= +4=8V
100 2 2
Putting value of I2 in equation (i), For 8 V source.
V2 23. (c)
= 0.01V1 + 0.1V2
100
From the above solution:
–0.01V2 – 0.1V2 = 0.01V1
When, VDC = 10 V
V2 0.01 VTh = 9 V
=
V1 0.11 and RTh = 2
V2 1 When, RL = 7 , I = ?
or, =
V1 11 VTh = I(RTh + RL)
VTh
22. (c) I=
RTh + RL
+ + 9
A B = = 1A
2+7
– –
24. Sol.
Case-I:
VDC = 10 V, RL = 1 , I = 3 A When two, 2-port networks are connected in
VTh = I(RTh + RL) = 3(RTh + 1) parallel then individual Y-parameters are
VTh = 3RTh + 3 ...(i) added. Therefore, from the given network,

Case-II: 1
3 =
RL = 2.5 , 3
I = 2 A,
VDC = 10 V 1 1
3 = 3 =
VTh = 2(RTh + 2.5) 1 3 3 2
VTh = 2RTh + 5 ...(ii)
From equation (i) and (ii), 1
2 =
2
VTh = 9 V and RTh = 2 ...(iii)
Now, VTh depends on independent voltage
1 1 1 2
source and varies with applied voltage. RTh does 2 = 2 =
2 2
not depend on voltage source and is same for
any applied voltage source, since voltage source
is short-circuited while calculating RTh.
154 Electronics Engineering Network Theory

For network (1) Y-parameter is, We get,

1 1 1 1
+ R+ I 1 (s) = RI2(s) = V1(s) ...(i)
3 3 3 Cs
Y1 =
1 1 1
+ 1
3 3 3 R+ I 2 (s) RI 2 (s) = 0 ...(ii)
Cs
Similarly for network (2), Also, V3(s) = I2(s) × R ...(iii)
1 From equation (i) and (ii), we get
1
Y2 = 2 1 1
1 = R+ × 1+ I 2 (s ) RI 2 ( s ) = V1 (s )
Cs RCs
1
2 RCs + 1 RCs + 1
= I 2 (s ) RI 2 ( s) = V1 ( s)
Cs RCs
5 5
3 6 RC 2 s 2V1 (s )
Thus, Y = Y1 + Y2 = or, I2(s) = ...(iv)
5 5 (1 + RCs )2 R 2C 2 s 2
6 3
Using equation (iii) and (iv), we get
I1 = Y11V1 + Y12V2
V3 (s) s 2 R2C 2
5 5 = ...(v)
= Y1 V2 ...(i) V1 (s) 1 + 3 RCs + s 2 R 2C 2
3 6
Q R = 10 k , C = 100 µF and RC = 1
I2 = Y21V1 + Y22V2
5 5 V3 (s) s2
= V1 + V2 ...(ii) =
6 3 V1 (s) 1 + 3s + s 2

I2 26. (c)
Also, h 22 =
V2 Converting -network to Y-network, we get,
I1 = 0

From equation (i), we get, 30


1 2
1 +
V1 = V2 ...(iii)
2
10 60
and from equation (ii) and (iii), we get

I 2 15 1 2
h 22 = = = 1.25
V2 12
1 2
3 18 +
25. (b)
6
The cascaded network is,

1/Cs 1/Cs 1 2

+ +
Z-parameter,
V1(s) I1(s) I2(s) V3(s)
Z 11 = 3 + 6 = 9
R R
Z 12 = Z21 = 6
– – Z 22 = 18 + 6 = 24
9 6
Applying mesh analysis to determine the [Z] =
6 24
current I2(s).
GATE Previous Years Solved Paper 155

27. (a) 29. (d)

5
For reciprocal network, AD – BC = 1,
I1 I2
+ + T =1

V1 10 10 V2 30. Sol.

– –
For T-network,
Za Zb
I1 2 2 I2 1 2

+ +
Zc
V1 4 V2

– – 1 2
Z 11 = Za + Zc
V1 = 6I1 + 4I2 Z 22 = Zb + Zc
V2 = 4I1 + 6I2 and Z 12 = Z21 = Zc
6 4 2j j
[Z] =
4 6 Given, [Z] = j 3+ 2j
1 6 4 0.3 0.2 Therefore, Z 12 = j2
Y= =
20 4 6 0.2 0.3 and Z 22 = 3 + 2j = 3 + j + j
Ignoring negative sign: = Zb + Zc = Rb + j + Zc
Rb = 3
0.3 0.2
[Y] =
0.2 0.3 31. (a)
Redrawing the circuit,
28. (b)
2
I1 (5 + j4) (5 – j4) I2
3 –
+ +
+
1 1
V2 (2 + j0) V2

6
– – I1
2
V1 A B V2
=
I1 C D I2 + V1 –

When, I2 = 0 V1
Z 11 = =3 6=2
V1 = (5 + j4 + 2) I1 I1
V2 = 2I1
V2 = –3 × I3
V1 7 + j4 6
A = = = 3.5 + j 2 = 3 × I1 × = 2 I1
V2 I2 = 0
2 9
V2
Z 21 = = 2
I1 1 I1
C = = = 0.5
V2 I2 = 0
2
V2
Now, Z 22 = =3 6=2
From here only (b) option matches. I1 I1 = 0
156 Electronics Engineering Network Theory

3 34. Sol.
V1 = –6 × I6 = 6 × I 2 × = 2
9 For ideal transformer on n : 1, the scattering
2 2 matrix is,
[Z] =
2 2
n2 1 2n
2 2
S11 S12 n +1 n +1
=
3 I2
S21 S22 2n 1 n2

n2 + 1 1 + n2
+ +
V2 2n 2(2) 4
– S21 = 2
= 2
= = 0.8
n +1 2 +1 5
6
I1 = 0 35. (d)
Consider the two-port network,
+ V1 – V1 V2
I1 1 I2
+ +
32. Sol.
V1 1 3V 2 1 V2
V1
B= – –
I2 V2 = 0
For Y-parameters:
When, V2 = 0 (i.e., when port-2 is short-circuited)
I1 = y11V1 + y12V2
I1 2 2 I2 I2 = y21V1 + y22V2
+ + By KCL at (V1)
V1 V1 V2
V1 5 V2 = 0 I1 + 3V2 = +
1 1
– – I1 = 2V1 – 4V2 ...(1)
V2 V2 V1
V1 7 V1 By KCL at (V2), I2 = +
I1 = = 1 1
2 + (5 2 ) 24 I2 = –V1 + 2V2 ...(2)
5 5V1 From equation (1) and (2),
I2 = I1 × =
5 +2 24 I1 = 2V1 – 4V2
I2 = –V1 + 2V2
V1 24
So, B= = = 4.80 y11 y12 2 4
I2 5
y 21 y 22 =
1 2
33. Sol.
36. (4)
From maximum power transfer theorem,
ZL = ZTh For I2 = 0 : V2 = VOC = 5 V
For Thevenin’s resistance RTh,
Z12 × Z21
ZTh = Z22 V2
Rs + Z11
For given data, 5V

60 × 60
ZTh = 120 = 48
10 + 40 4V

ZL = 48
–I2
0 4 mA 20 mA
GATE Previous Years Solved Paper 157

For –I2 = 20 mA, 10 5


I2 =
V2 = 0 1.25 × 10 3
ISC = –I2 5 3
= × 10 A = 4 mA
ISC = 20 mA 1.25
VOC 5
RTh = = × 10 3 = 250
I SC 20
Network is replaced by Thevenin’s equivalent,
250 A I2

1k
5V

10 V

Answers
EE Two Port Networks

1. (b) 3. (b) 4. (c) 5. (b) 6. (d) 7. (d) 8. (d) 9. (b)

10. (a) 11. (c) 12. (c) 13. (b) 14. (c) 15. (a) 16. (35.55) 17. (20)

18. (d) 19. (0.5)

Solutions
EE Two Port Networks

1. (b) 3. (b)
For reciprocity, 2 1
h 12 = –h 21 [y] =
1 1
For symmetry, [z] = [y]–1
h11 h12 1 1 1 1
=1 1
h21 h22 = =
2 1 1 2 1 2

2. Sol. 4. (c)
Using KCL, For a passive two-port network, output power
V1 V1 V2 1 1 can never be greater than input power.
I1 = + = V1 V2
10 10 5 10
Again using KCL, 5. (b)
Using KVL,
V2 V2 V1 1 1
I2 = + = V1 + V2 E1 = 2I1 + 2(I1 + I2)
10 10 10 5
Again using KVL,
0.2 0.1
Hence, [y] = E2 = 2I2 + 2(I1 + I2)
0.1 0.2
4 2
[z] =
2 4
158 Electronics Engineering Network Theory

[y] = [z]–1 8. (d)


1 4 2 i1 Z2 i2
=
(4 × 4) (2 × 2) 2 4

Y11 Y12 v1 Z1 v2
1/3 1/6
=
Y21 Y22 1/6 1 /3

6. (d) v 1 = (i1 + i2) z1


= z1i1 + z1i2 ...(i)
E1 v 2 = z2i1 + z1(i1 + i2)
h 12 =
E2 I1 = 0 = z1i1 + (z1 + z2) i2 ...(ii)
or h12 is ratio of E1 to E2 for the input open- From equation (i) and (ii),
circuited condition. z1 z1
z-matrix =
Two method are provided to solve the problem. z1 z1 + z2
Assuming, I1 = 0
9. (b)
I1 = 0 4 2 Ix 2 I2
Z1 = 10 30°

I1 I1 + I2

E1 2 4 E2 V1 Z2 V2

E2 E V1 = AV2 + BI2
I2 = = 2
2 + (2 + 4) 4 4 I1 = CV2 + DI2
V2 = Z2(I1 + I2) ...(i)
I2
Ix = ×4 I1
(2 + 2) + 4 C=
V2 I2 = 0
I 2 1 E2 E
= = = 2 Putting, I2 = 0 in equation (i),
2 2 4 8
V2 = Z2I1
E2 E
E1 = 2 I x = 2 = 2 V2 1 1
8 4 Z2 = = =
I1 I2 = 0
C 0.025 45°
E1
= 0.25 Z 2 = 40 –45°
E2
10. (a)
7. (d)
Output voltage = V0 = AVi
0.9 0.2 V0 = 106 × 1 × 10–6 = 1 V
[z] = 0.2 0.6
[Given: A = 106 V/V, Vi = 1 µV)
0.6 0.2 To calculate input impedance, Vdc source is
connected at input port,
0.2 0.9
[y] = [ z] 1 = Ii Ri Ro
[0.9 × 0.6 0.04]
+ +
1.2 0.4
= 0.4 1.8 Vdc Vi + AVi
– Vo

y 22 = 1.8
– –
GATE Previous Years Solved Paper 159

Input impedance, V2 0
g 22 = = =0
V I1 I2
Z i = dc V1 = 0
Ii
as loop is not closed, Ii = 0 g11 g12 0 0
So, g-parameters = =
g21 g22 0 0
Vdc
So, Zi =
0
12. (c)
To calculate output impedance, Vdc source is
I1 1 a I2
connected at output port, e c
Ii Ri Ro Io ef ab
V1 V1 P V2
+ +
f d
b
Vi + AVi Vo

V1ab = Z11I1 + Z12I2 ...(i)
– – V2 = Z21I1 + Z22I2 ...(ii)
Output impedance, As 1 resistor is connection in series with the
V0 I0 R0 + AVi network at port-1.
Z0 = =
I0 I0 V2 does not get affected,
ef
As, Vi = 0 V1 = V1ab + I 1 × 1

I 0 R0 + A × 0 = Z11I1 + Z12I2 + I1
Z0 = = R0 = 10
I0 = (Z11 + 1) I1 + Z12I2
Modified Z-parameter
11. (c) Z11 + 1 Z12
=
I1 = g11V1 + g12I2 Z21 Z22
V2 = g21V1 + g22I2
13. (b)
I1 I2
(i) V1 = 10 V; V2 = 3 V
+ +
I2 = –3 A; V1 = AV2 – BI2
V1 V2 10 = 3A + 3B ...(i)
(ii) V2 = 5V
– – I2 = –2 A
Since port-1 is open-circuit, 10 = 5A+2B ...(ii)
I1 = 0 10
A= ...(iii)
Port-2 is short-circuit, 9
V2 = 0 20
B= ...(iv)
9
I 0
g 11 = 1 = =0 Given, V1 = 8V
V1 I2 = 0
V1
(V2)OC = ?
I1 0 I2 = 0
g 12 = = =0 V1 = AV2 – BI2
I2 V1 = 0
I2
8 = A(V2)OC – 0
V2 0 8 8
g 21 = = =0 (V2)OC = = = 7.2 V
V1 I2 = 0
V1 A 10 /9
160 Electronics Engineering Network Theory

14. (c) Using equation (ii) and (iv), we get


Given, V1 = 10 V, V2 = (–7I2) so, V1 = 40I1 + 60I2
V1 = AV2 – BI2 2
= 40 I1 + 60 I1
70 20 3
10 = 7 I 2 A BI 2 = I2 I2
9 9 V1 =0
I2 = –1 A From the figure,
(–ve) sign signifies that current is drawn. 20 = 10I1 + V1
Since, V1 =0
15. (a) So, I1 = 2A
s s 4
So, I2 = A
3
Power dissipated in
Z(s ) 1/s 1/s
2
4
PRL = I 22 RL = × 20
3
Driving point impedance, Z(s) is 16
= × 20 = 35.55 W
1 1
9
s+ ×
s s s2 + 1 s
Z(s) = s + 1 1
=s+ 2
× 2 17. Sol.
s+ + s ( s + 2)
s s To find impedance seen by Vs,
Vs
s2 + 1 s2 (s 2 + 2) + s 2 + 1 Zs =
= s+ = Is
s(s2 + 2) s3 + 2s
V1 = 2Is
4 2
s + 3s + 1
or, Z(s) = Is 2 2
s3 + 2s A
+ V –
1
16. Sol.
Vs 3 4V1 4
10 I1 I2
+ +

20 V V1 [Z] V2 20 Applying KCL at node A,


VA VA
– – Is + 4V1 = +
3 6
Given, Z 11 = 40 , Z12 = 60 VA = Vs – V1
Z 21 = 80 , Z22 = 100 and V1 = 2Is
From the figure, Vs 2 Is Vs 2 Is
So, Is + 8Is = +
V2 = –20I2 ...(i) 3 6
and V1 = 40I1 + 60I2 ...(ii) 54Is = 2Vs – 4Is + Vs – 2Is
V2 = 80I1 + 100I2 ...(iii) 3Vs = 60Is
From equation (i) and (iu), we get
Vs
so, –20I2 = 80I1 + 100I2 = 20
Is
2
I2 = I1 ...(iv)
3
GATE Previous Years Solved Paper 161

18. (d) 19. Sol.


For two port networks we can write, 2I1
1 Vs 1
–+
A B A1 B1 A2 B2
= I1
C D C1 D1 C2 D2
V1 = 1 1 V2 = 0
or, A = A1A2 + B1C2 ...(i)
B = A 1B 2 + B 1D 2 ...(ii)
or, V1 = AV2 – BI2 ...(iii)
To get, VT (I2 = 0), from equation (iii),
By KCL,
V V1 Va 1 Va Va + 2 I 1
V2 = VT = 1 = + + =0
A A1 A2 + B1C 2 1 1 1
To get, ZT (VT = 0), from equation (iii), 3Va + 2I1 = 1 ...(i)
V1 = AV2 – BI2 1 Va
I1 = ...(ii)
0 = AV2 – BI2 1
Substitute equation (ii) in equation (i),
V2 B A1 B2 + B1D2
ZT = = = Va = –1
I 2 A A1 A2 + B1C 2
1 Va 1 ( 1)
I1 = = =2
1 1
V 1
h 11 = 1 = = 0.5
I1 2
6 Network Functions

ELECTRO NICS EN GINEERIN G C2 = 4C1 L2 = L1/4

(GATE Previous Years Solved Papers) + +

Q.1 The circuit of the figure represents a Vi Vo


R
L1
Rs – –
C1 Vo Filter-2

L1 (a) 4 (b) 1
Vs RL
1 1
(c) (d)
2 4
C2
[EC-2007 : 2 Marks]

(a) low pass filter (b) high pass filter Q.4 The driving point impedance of the following
(c) band pass filter (d) band reject filter network, is given by
[EC-2001 : 1 Mark] 0.2 s
Z(s ) = 2
s + 0.1 s + 2
Q.2 The RC circuit shown in the figure is
R
+ C + Z(s)
L C R

Vi R C Vo

– – The component values are


(a) L = 5 H, R = 0.5 , C = 0.1 F
(a) a low-pass filter
(b) L = 0.1 H, R = 0.5 ,C=5F
(b) a high-pass filter
(c) L = 5 H, R = 2 , C = 0.1 F
(c) a band-pass filter
(d) L = 0.1 H, R = 2 ,C=5F
(d) a band-reject filter
[EC-2008 : 2 Marks]
[EC-2007 : 1 Mark]
Q.5 If the transfer function of the following network
Q.3 Two series resonant filters are as shown in the
is,
figure. Let the 3 dB bandwidth of filter 1 be B1
Vo (s) 1
and that of filter 2 be B2. The value of B1/B2 is =
Vi (s) 2 + sCR
C1 L1
R
+ +
+ +
Vi R Vo
Vi C RL Vo
– –
Filter-1 – –
GATE Previous Years Solved Paper 163

The value of the load resistance RL is (a) ab, bc, ad (b) ab, bc, ca
(a) R/4 (b) R/2 (c) ab, bd, cd (d) ac, bd, ad
(c) R (d) 2R [EE-1994 : 1 Mark]
[EC-2009 : 1 Mark]
Q.3 Two identical coils of negligible resistance when
Q.6 The transfer function V2(s)/V1(s) of the circuit connected in series across a 200 V, 50 Hz source
shown below is draws a current of 10 A. When the terminals of
100 µF
one of the coils are reversed, then current drawn
+ + is 8 A. The coefficient of coupling between the
10 k two coils is _______ .
V1(s) V2 ( s )
[EE-1997 : 2 Marks]
100 µF
– – Q.4 A major advantage of active filter is that they
0.5s + 1 3s + 6 can be realized without using
(a) (b)
s+1 s+2 (a) op-amps (b) inductors
s+2 s+1 (c) resistors (d) capacitors
(c) (d)
s+1 s+2 [EE-1997 : 1 Mark]
[EC-2013 : 1 Mark]
Q.5 The effective inductance of the circuit across the
terminals A, B in the figure shown below is
ELECTRICAL EN GINEERIN G
(GATE Previous Years Solved Papers) 4H
A
Q.1 The equivalent inductances seen at terminals
A-B in figure is ______ H. 1H
3H 5H
4H
A
2H
2H
B
1H 4H 6H

1H
(a) 9 H (b) 21 H
B (c) 11 H (d) 6 H
4H
[EE-1998 : 2 Marks]
[EE-1992 : 2 Marks]
Q.6 The impedance seen by the source in the circuit
Q.2 Figure shows a dc resistive network and its
in figure is given by
graph is drawn a side. A ‘proper tree’ chosen
for analysis the network will not contain the 4 –j2
1:4
edges:

ZL = 10 30°
b b
a c a c

(a) (0.54 + j0.313) (b) (4 – j2)


(c) (4.54 – j1.693) (d) (4 + j2)
[EE-2000 : 2 Marks]
d d
164 Electronics Engineering Network Theory

Q.7 Given two coupled inductors L1 and L2, their


W
mutual inductance ‘M’ satisfies 1 : 1.25
(L + L2 )
(a) M = L21 + L22 (b) M > 1 Y
2
(c) M > L1 L2 (d) M L1L2 X Z

[EE-2000 : 1 Mark]
125 80 100 80
(a) and (b) and
Q.8 In the circuit shown in figure it is found that the 100 100 100 100
input ac voltage (Vi) and current ‘i’ are in phase.
100 100 80 80
(c) and (b) and
M 100 100 100 100
The coupling coefficient is K = , where
L1 L2 [EE-2013 : 2 Marks]
M is the mutual inductance between the two
coils. The value of ‘K’ and the dot polarity of the Q.11 Two identical coupled inductors are connected
coil P-Q are in series. The measured inductances for the two
possible series connections are 380 µH and
K
240 µH. Their mutual inductance in µH is
–j2 10 P Q ________ .
j8 j8 [EE-2014 : 1 Mark]
L1 L2
vi i Q.12 Find the transformer ratios a and b such that the
impedance (Zin) is resistive and equals 2.5
when the network is excited with a since wave
voltage of angular frequency of 5000 rad/sec.
(a) K = 0.25 and dot at P
C = 10 µF L = 1 mH
(b) K = 0.5 and dot at P 1:a
(c) K = 0.25 and dot at Q
(b) K = 0.5 and dot at Q Zin R = 2.5
[EE-2002 : 2 Marks]
1:b
Q.9 A first order, low pass filter is given with
R = 50 and C = 5 µF. What is the frequency at (a) a = 0.5, b = 2.0 (b) a = 2.0, b = 0.5
which the gain of the voltage transfer function (c) a = 1.0, b = 1.0 (d) a = 4.0, b = 0.5
of the filter is 0.25? [EE-2015 : 1 Mark]
(a) 4.92 kHz (b) 0.49 kHz
(c) 2.46 kHz (d) 24.6 kHz Q.13 Two identical coils each having inductance L
are placed together on the same core. If an overall
[EE-2002 : 2 Marks]
inductance of aL is obtained by interconnecting
Q.10 The following arrangement consists of an ideal these two coils, the minimum value of a is
transformer and an attenuator which attenuates ________ .
by a factor of 0.8. An a.c. voltage VWX1 = 100 V is [EE-2015 : 2 Marks]
applied across WX to get an open-circuit voltage
V YZ1 across YZ. Next, an a.c. voltage Q.14 If an ideal transformer has an inductive load
VYZ2 = 100 V is applied across YZ to get an open- element at port 2 as shown in the figure below,
circuit voltage VWX2 across WX. Then VYZ1/ the equivalent inductance at port 1 is
VWX1, VWX2/VYZ2 are respectively.
GATE Previous Years Solved Paper 165

Q.15 The following figure shows the connection of


n:1
an ideal transformer with primary to secondary
L turns ratio of 1 : 100. The applied primary
voltage is 100 V(rms), 50 Hz, AC. The rms value
of the current I, in ampere, is ______ .
XL = 10 R = 80 k
Port-1 Port-2 1 : 100
(a) nL (b) n2L I
100 V XC = 40 k
n n2
(c) (d)
L L
[EE-2016 : 1 Mark] [EE-2016 : 1 Mark]

Electronics & Electrical Engineering


GATE Previous Years Solved Paper

A n swe rs & Expl a n a t i o n s

Answers
EC Network Functions

1. (d) 2. (c) 3. (d) 4. (d) 5. (c) 6. (d)

Solutions
EC Network Functions

1. (d) At = ;
Analyzing the circuit for = 0 and = . Rs

At = 0 ;
Rs
Vs RL Vo

Vs RL Vo
Vo RL
= (finite value)
Vs RL + Rs
= 0, Ind = L = 0 (SC)
1
cap = = (OC)
C
Vo RL
= (finite value)
Vs RL + Rs
=0 =
166 Electronics Engineering Network Theory

1 Bandwidth of filter 2;
At = ;
LC R R 4R
B2 = = =
Rs L2 L1 /4 L1
B1 1
So, =
B2 4
RL
4. (d)
0.2 s
Z(s) = 2
vo = 0 s + 0.1s + 2

2. (c) s 2 + 0.1s + 2 s 1 2
Y(s) = = + +
0.2s 0.2 2 0.2 s
At , capacitor short-circuited
Circuit looks like, 10
= 5s + 0.5 +
s
+ R + Comparing with
1 1
Vi R Vo Y(s) = Cs + +
R Ls
1
– – C = 5 F, R = =2
0.5
Vo 1
=0 L= = 0.1 H
Vi 10
At 0, capacitor open-circuited
5. (c)
Circuit looks like,
+ R +
+ R +
Vi Z Vo
Vi R Vo
– –
– –
RL
Z =
Vo 1 + sRLC
=0
Vi Z RL
H(s) = =
So frequency response of the circuit will be Z + R ( RL + R ) + sRRLC
If, R = RL
1
H(s) =
2 + sRC

6. (d)
1
10 × 103 +
So the circuit is bandpass filter. V2 (s) 100 × 10 6 s
=
V1 (s) 1 1
3. (d) 10 × 103 + +
100 × 10 6 s 100 × 10 6 s
Bandwidth of series RLC circuit is R/L V2 (s) s × 10 4 + 10 4 10 4 (1 + s)
Bandwidth of filter 1; = =
V1 (s) s × 10 4 + 10 4 + 10 4 10 4 ( s + 2)
R V2 (s) s+1
B1 = =
L1 V1 (s) s+2
GATE Previous Years Solved Paper 167

Answers
EE Network Functions

1. (8) 2. (b) 3. (1/9) 4. (b) 5. (c) 6. (c) 7. (d) 8. (c)

9. (c) 10. (b) 11. (35) 12. (b) 13. (0) 14. (b) 15. (10)

Solutions
EE Network Functions

1. Sol. and putting, V = 220 V


and L1 = L2 = L
L1
A 200
M12 we have, 10 = ...(i)
(2L 2 M12 )

M13
200
L2 8 = ...(ii)
(2 L + 2 M12 )
M23
On solving equation (i) and equation (ii),
B 1
L3 we get, M12 = L
9
L = L1 + L2 + L3 – 2M12 + 2M23 – 2M13 Which can be written as,
= 4 + 4 + 4 – (2 × 2) + (2 × 1) – (2 × 1) 1 1
= 8H M12 = L L= L L
9 9
2. (b) 1
Hence, coefficient of coupling = .
Tree must not contain any closed loop. 9
Hence option (b) is correct.
4. (b)
3. Sol. Inductive coils are bulky in nature.
Case-I Case-II
5. (c)
L1 L1
L1
A
220 V M 12 220 V M12
L2 L2 M12
50 Hz 50 Hz
M13 L2

V M23
I1 = B
(L1 + L2 2 M12 ) L3

V L = L1 + L2 + L3 – 2M12 + 2M23 – 2M13


I2 =
(L1 + L2 + 2 M12 ) = 4 + 5 + 6 – (2 × 1) + (2 × 2) – (2 × 3)
From above expressions, it is clear that, = 11 H

I1 > I 2 6. (c)
2
Taking, I1 = 10 A 1
Z = (4 j 2) + × 10 30°
4
and I2 = 8 A
= (4.54 – j1.69)
168 Electronics Engineering Network Theory

7. (d) 100
Hence, VWX 2 = = 80 V
1.25
M = K L1 L2
VYZ1 100
Where, K = coefficient of coupling =
VWX1 100
Q 0 < K<1
VYZ2 80
M L1 L2 =
VWX2 100
8. (c)
11. Sol.
Input ac voltage and current will be in phase
only at resonance condition. The two possible series connection are shown
i.e., XC = XL below:
Let the mutual inductance be M
j12 = j 8 + j 8 + 2 k ( j 8) × ( j 8)
L1 L2
I
12 = 8 + 8 + 16 k
4 1
k= = = 0.25 M
16 4
(i) Additive connection,
Hence coupling will be opposite.
Leq. = L1 + L2 + 2M = 380 µH
Dot will be at Q.
L1 L2
I
9. (c)
R M
+ + (ii) Subtractive connection,
Leq. = L1 + L2 – 2M = 240 µH
Vi C Vo
Thus, L1 + L2 + 2M = 380 µH ...(i)
– – and L1 + L2 – 2M = 240 µH ...(ii)
Solving equations (i) and (ii), we get,
Vo 1/ j C 1 4M = 10 µH or M = 35 µH
T.F. = = =
Vi R + 1 1 + j CR Mutual inductance,
j C
M = 35 µH
1
Gain = 12. (b)
1 + ( CR)2
C = 10 µF L = 1 µF
1
0.25 =
6
1 + ( × 5 × 10 × 50)2
On solving, R = 2.5
Zin
= 15.49 × 103 rad/sec.
15.49 A 1:b B 1:a C
f= = 2.46 kHz
2 2.5
(Req)B =
10. (b) a2

VYZ1 = 100 × 1.25 × 0.8 2.5


(2eq)B = + j (1 × 10 3 )
= 100 V a2
In second case when 100 V is applied at YZ 2.5 3
(2eq)A = 2
+ j (1 × 10 )
terminals, this whole 100 V will appear across a
the secondary winding. b2
GATE Previous Years Solved Paper 169

2.5 j (10 3 ) 1 14. (b)


Zin = 2 2
+ 2
+ 5
a b b j (10 × 10 )
n:1
2.5 5 × 10 3 × 10 3
1
Zin = +j L
a2 b 2 b2 5 × 10 3 × 10 × 10 6

...(i)
From problem,
Zin = 2.5 ...(ii) Port-1 Port-2
From equation (i) and (ii), At port 1 i.e., high voltage side impedance will
a2 b 2 = 1 ...(ii) be high and current will be low, So n2L.
5 1
2 =0 15. Sol.
b 5 × 10 2
The above circuit can be drawn by transferring
5 × 5 × 10–2 = b2
secondary circuit to primary side.
b = 0.5 and a = 2
10j
13. Sol.
I
Case-I: 80000
Leff. = L1 + L2 = 2L, a = 2 (100)2
100 V
Case-II: 40000
j
L1L2 2 (100)2
L L
Leff. = = = , a = 0.5
L1 + L2 2L 2
100 V 100 V
Case-III: I= =
(8 + 10 j 4 j ) (8 + 6 j )
If both are differentially coupled then,
So the rms value of I will be 10 A.
Leff. = 0
Minimum value = 0
GATE-2023
Electronics Engineering

Signals & Systems


Chapterwise & Topicwise

Contents
S.No. Topic Page No.

1. Basics of Signals & Systems .............................................................................................................. 1-11

2. LTI Systems Continuous and Discrete (Time Domain) ........................................................ 12-29

3. Fourier Series ..................................................................................................................................... 30-44

4. Fourier Transforms, Frequency Response and Correlation ................................................ 45-63

5. Laplace Transform ............................................................................................................................ 64-87

6. Z-Transform ..................................................................................................................................... 88-109

7. DTFS, DTFT and DFT ................................................................................................................... 110-121

8. Sampling ........................................................................................................................................ 122-131

9. Digital Filters ................................................................................................................................. 132-133


1 Basics of Signals & Systems

ELECTRO NICS EN GINEERIN G E


(a) E (b)
(GATE Previous Years Solved Papers) 2
(c) 2E (d) 4E
Q.1 An excitation is applied to a system at t = T and [EC-2001 : 1 Mark]
its response is zero for – < t < T. Such a system
is a Q.6 Let P be linearity, Q be time-invariance, R be
causality and S be stability. A discrete-time
(a) non-causal system
system has the input-output relationship,
(b) stable system
(c) causal system x(n) n 1
(d) unstable system (n) = 0 n=0
[EC-1991 : 2 Marks] x(n + 1) n 1

where x(n) is the input and y(n) is the output.


Q.2 Which of the following signals is/are periodic?
The above system has the properties.
(a) s(t) = cos2t + cos3t + cos5t
(a) P, S but not Q, R
(b) s(t) = exp(j8 t)
(b) P, Q, S but not R
(c) s(t) = exp(–7t) sin10 t
(c) P, Q, R, S
(d) s(t) = cos2t cos4t
(d) Q, R, S but not P
[EC-1992 : 2 Marks]
[EC-2003 : 2 Marks]
Q.3 A system with an input x(t) and output y(t) is
Q.7 Consider the sequence
described by the relation: y(t) = tx(t). This system
is x [n] = [ 4 j 5, 1 + j 2, 4]
(a) linear and time-invariant
The conjugate antisymmetric part of the
(b) linear and time varying
sequence is
(c) non-linear and time-invariant
(a) [–4 –j2.5, j2, 4 – j2.5]
(d) non-linear and time-varying
(b) [–j2.5, 1, j2.5]
[EC-2000 : 1 Mark]
(c) [–j5, j2, 0]
Q.4 Let (t) denote the delta function. The value of (d) [–4, 1, 4]
[EC-2003 : 2 Marks]
3t
the integral (t ) cos dt is
2 Q.8 The function x(t) is shown in the figure. Even
and odd parts of a unit-step function u(t) are
(a) 1 (b) –1 respectively.
x(t)
(c) 0 (d)
2
1
[EC-2001 : 1 Mark]
t
Q.5 If a signal f(t) has energy E, the energy of the 0
singal f(2t) is equal to –1
2 Electronics Engineering Signals & Systems

1 1 1 1 Which of the following descriptions


(a) , x(t ) (b) , x(t ) corresponds to a causal system?
2 2 2 2
1 1 1 1 (a) y(t) = x(t – 2) + x(t + 4)
(c) , x(t ) (d) , x(t )
2 2 2 2 (b) y(t) = (t – 4) + x(t + 1)
[EC-2005 : 1 Mark] (c) y(t) = (t + 4) + x(t – 1)
(d) y(t) = (t + 5) + x(t + 5) [EC-2008 : 1 Mark]
Q.9 The power in the signal
Q.14 Let x(t) be the input and y(t) be the output of a
s(t ) = 8 cos 20 t + 4 sin (15 t ) is continuous time system. Match the system
2
properties P1, P2 and P3 with system relations
(a) 40 (b) 41
R1, R2, R3, R4.
(c) 42 (d) 82
[EC-2005 : 1 Mark] Properties:
P1 : Linear but not time-invariant
Q.10 The dirac delta function (t) is defined as,
P2 : Time-invariant but not linear
1 t=0 P3 : Linear and time-invariant
(a) (t ) =
0 otherwise
Relation:
t=0 R1 : y(t) = t2 x(t)
(b) (t ) =
0 otherwise
R2 : y(t ) = t x(t )
1 t=0
(c) (t ) = and (t ) dt = 1 R3 : y(t ) = x(t )
0 otherwise
R4 : y(t) = x(t – 5)
t=0 (a) (P1, R1) (P2, R3), (P3, R4)
(d) (t ) = and (t ) dt = 1
0 otherwise (b) (P1, R2) (P2, R3), (P3, R4)
[EC-2006 : 1 Mark] (c) (P1, R3) (P2, R1), (P3, R2)
(d) (P1, R1) (P2, R2), (P3, R3)
Q.11 A system with input x[n] and output y[n] is given
[EC-2008 : 2 Marks]
as,
5 Q.15 The input x(t) and output y(t) of a system are
y [n] = sin n x (n)
6 related as,
The system is t
y(t ) = x ( ) cos(3 ) d
(a) linear, stable and invertible.
(b) non-linear, stable and non-invertible. The system is
(c) linear, stable and non-invertible. (a) time-invariant and stable.
(d) linear, unstable and invertible. (b) stable and non time-invariant.
[EC-2006 : 2 Marks] (c) time-invariant and not stable.
(d) not time-invariant and not stable.
Q.12 A Hilbert transformer is a
[EC-2012 : 2 Marks]
(a) non-linear system
(b) non-causal system Q.16 For a periodic signal,
(c) time-varying system v(t ) = 30 sin 100t + 10 cos 300t + 6 sin 500t +
(d) low-pass system 4
The fundamental frequency (in rad/sec) is
[EC-2007 : 2 Marks]
(a) 100 (b) 300
Q.13 The input and output of a continuous time (c) 500 (d) 1500
system are respectively denoted by x(t) and y(t). [EC-2013 : 1 Mark]
GATE Previous Years Solved Paper 3

Q.17 The impulse response of a continuous time (a) T = 2 Ts (b) T = 1.2Ts


system is given by
(c) Always (d) Never
h(t) = (t – 1) + (t – 3)
[EC-2016 : 1 Mark]
The value of the step response at t = 2 is
(a) 0 (b) 1 Q.22 Consider a single input single output discrete-
(c) 2 (d) 3 time system with x[n] as input and y[n] as output,
[EC-2013 : 2 Marks] where the two are related as,

n x[n ] for 0 n 10
Q.18 A discrete-time signal x[n] = sin( 2n), n being an y[n] =
x[n] x[n 1] otherwise
integer, is
(a) periodic with period Which one of the following statements is true
(b) periodic with period 2 about the system?
(c) periodic with period /2 (a) It is causal and stable.
(d) not periodic (b) It is causal but not stable.
[EC-2014 : 1 Mark] (c) It is not causal but stable.
(d) It is neither causal nor stable.
Q.19 The waveform of a periodic signal x(t) is shown
[EC-2017 : 1 Mark]
in the figure.
Q.23 The input x(t) and the output y(t) of a
x(t)
continuous-time system are related as,
3 t
y(t ) = x(u) du
–2 1 4 t T
t
–4 –1 2 3
The system is
–3
(a) linear and time-variant.
(b) linear and time-invariant.
t 1 (c) non-linear and time-variant.
A signal g(t) is defined by g(t ) = x .
2 (d) non-linear and time-invariant.
The average power of g(t) is ________ . [EC-2017 : 1 Mark]
[EC-2015 : 1 Mark]
Q.24 Let the input be ‘u’ and the output be ‘y’ of a
Q.20 Two sequence x1[n] and x2[n] have the same system, and the other parameters are real
energy. Suppose x1[n] = 0.5n u[n], where is a constants. Identify which among the following
positive real number and u[n] is the unit step systems is not a linear system:
sequence. Assume, d3 y d2 y dy
(a) 3
+ a1 2
+ a2 + a3 y
1.5 for n = 0, 1 dt dt dt
x2 [ n] =
0 otherwise du d2u
b3 u + b2 + b1 2
Then the value of is _______ . dt dt
[EC-2015 : 2 Marks] (with initial rest conditions)
t
Q.21 A continuous time function x(t) is periodic with (t )
(b) y(t ) = e u( ) d
period T. The function is sampled uniformly 0
with a sampling period Ts. In which one of the (c) y = au + b, b 0
following cases is the sampled signal periodic? (d) y = au [EC-2018 : 1 Mark]
4 Electronics Engineering Signals & Systems

ELECTRICAL EN GINEERIN G (a) 5t (b) 5 u(t) – C

(GATE Previous Years Solved Papers) 5 5 u(t )


(c) t (d)
C C
+6 [EE-2002 : 1 Mark]
2t
Q.1 The value of the integral e (t 1) dt is
5 Q.5 Which of the following is true?
(a) A finite amplitude signal is always time
equal to ________ .
bounded.
[EE-1994 : 1 Mark]
(b) A time bounded signal always possesses
Q.2 The rms value of the periodic waveform e(t), finite energy.
shown in figure is (c) A time bounded signal is always zero
e(t ) outside the interval [–t0, t0] for some t0.
(d) A time bounded signal is always finite in
+A
amplitude.
[EE-2006 : 1 Mark]
0 t
T/2 T
Q.6 A continuous-time system is described by
–A y(t) = e–|x(t)|, where y(t) is the output and x(t) is
the input, y(t) is bounded

3 2 (a) only when x(t) is bounded.


(a) A (b) A (b) only when x(t) is non-negative.
2 3
(c) only for t 0 if x(t) is bounded for t 0.
1
(c) A (d) 2A (d) even when x(t) is not bounded.
3
[EE-2006 : 2 Marks]
[EE-1995 : 1 Mark]
Q.7 Given a sequence x[n], to generate the sequence
Q.3 What is the rms value of the voltage waveform y[n] = x[3 – 4n], which one of the following
shown in figure? procedures would be correct?
(a) First delay x[n] by 3 samples of generate
v(t )
z1[n], then pick every 4th samples of z1[n] to
+100 V
generate z2[n], and then finally time reverse
z2[n] to obtain y[n].
0 t (b) First advance x[n] by 3 samples of generate
/3 2 /3 2
z1[n], then pick every 4th samples of z1[n] to
generate z2[n], and then finally time reverse
–100 V
z2[n] to obtain y[n].
(c) First pick every 4th sample of x[n] to generate
200 100 v1[n], time-reverse v1[n] to obtain v2[n], and
(a) V (b) V
finally advance v 2[n] by 3 rd samples to
(c) 200 V (d) 100 V obtain y[n].
[EE-2002 : 1 Mark] (d) First pick every 4th sample of x[n] to generate
v1[n], time-reverse v1[n] to obtain v2[n], and
Q.4 A current impulse of 5 (t), is forced through a
finally delay v2[n] by 3rd samples to obtain
capacitor C. The voltage V c(t), across the
y[n].
capacitor is given by
[EE-2006 : 2 Marks]
GATE Previous Years Solved Paper 5

Q.8 The function shown in the figure can be


f(t)
represented as,
4

2.7
t
–3.3 –1.3 –0.3 0.7 3.7 4.7

t –2
0 T 2T

(t T ) (t 2T ) [EE-2017 : 1 Mark]
(a) u(t ) u(t T ) + u(t T ) u(t 2T )
T T
Q.12 The signal energy of the continuous-time signal,
t t
(b) u(t ) + u(t T ) u(t 2T ) x(t) = [(t – 1) u(t – 1)] – [(t – 2) u(t – 2)]
T u
– [(t – 3) u(t – 3)] + [(t – 4) u(t – 4)], is
(t T ) (t 2T )
(c) u(t ) u(t T ) + u(t ) u(t )
11 7
T T
(a) (b)
(t T ) (t 2T ) 3 3
(d) u(t ) + u(t T ) 2 u(t 2T )
T T 1 5
(c) (d)
[EE-2014 : 2 Marks] 3 3
[EE-2018 : 2 Marks]
+
Q.9 The value of t Q.13 The symbol a and T, represent positive
e (2t 2) dt , where (t) is the
quantities, and u(t) is the unit step function.
Which one of the following impulse responses
Dirac delta function, is
is not the output of a causal linear time-invariant
1 2 system?
(a) (b)
2e e
(a) 1 + e–at u(t) (b) e+at u(t)
1 1 (c) e–a(t – T) u(t) (d) e–a(t + T) u(t)
(c) 2 (d) 2
e 2e [EE-2019 : 1 Mark]
[EE-2016 : 1 Mark]
Q.14 xR and xA are, respectively, the rms and average
Q.10 Consider the system with following input- values of x(t) = x(t – T), and similarly, yR and yA
output relation y[n] = (1 + (–1)n) x[n], where x[n] are, respectively, the rms and average values of
is the input and y[n] is the output. The system is y(t) = kx(t) k, T are independent of t.
(a) invertible and time invariant Which of the following is true?
(b) invertible and time varying (a) yA = kxA ; yR = kxR
(c) non-invertible and time invariant (b) yA = kxA ; yR kxR
(d) non-invertible and time varying (c) yA kxA ; yR kxR
[EE-2017 : 1 Mark] (d) yA kxA ; yR = kxR
[EE-2020 : 1 Mark]
Q.11 The mean square value of the given periodic
waveform f(t) is _______ .
6 Electronics Engineering Signals & Systems

Electronics & Electrical Engineering


GATE Previous Years Solved Paper

A n swe rs & Expl a n a t i o n s

Answers
EC Basics of Signals & Systems

1. (c) 2. (a, b, d) 3. (b) 4. (a) 5. (b) 6. (a) 7. (a) 8. (a)

9. (a) 10. (d) 11. (c) 12. (b) 13. (c) 14. (a) 15. (d) 16. (a)

17. (b) 18. (d) 19. (2) 20. (1.5) 21. (b) 22. (a) 23. (b) 24. (c)
25. (0) 26. (c, d)

Solutions
EC Basics of Signals & Systems

1. (c) 4. (a)
For the given system if the response is zero prior 3t 3× 0
to the application of the excitation. Then such a (t ) cos dt = f (0) = cos = cos 0 = 1
2 2
system is called causal system.
5. (b)
2. (a, b, d)
(a) s(t) is periodic as the ratio of any two E= f (t )2 dt
frequencies = p/q is rotational.
Where p and q are integers.
E = f (2t )2 dt
(b) s(t) is periodic with =8
(d) 2 cosA cosB = cos(A – B) + cos(A + B)
dp dp
= f ( p )2 2t = p ; dt =
1 2 2
s(t ) = [cos 2t + cos 6t ]
2
E
So, s(t) is periodic with fundamental E =
2
frequency 2 rad/sec.
6. (a)
3. (b)
y(n – n0) = x(n – n0 + 1) (time varying)
ay1(t) = atx1(t) y(n) = x(n + 1) (depends on future)
ay2(t) = atx2(t) i.e., y(1) = x(2) (non-causal)
a[y1(t) + y2(t)] = a[tx1(t) + tx2(t)] For bounded input, system has bounded output.
So it is stable,
System is linear,
y(n) = x(n); n 1
y(t – t0) = (t – t0) x(t – t0) = 0; n=0
Delay is introduced. So, time varying. = x(n + 1), n –1
So, system is linear.
GATE Previous Years Solved Paper 7

7. (a) 10. (d)


x(n) = [ 4 j 5, 1 + 2 j , 4]
(t )

x (–n) = [4, 1 2 j , 4 + j 5]
t

x( n ) x ( n )
xCAS (n) = 11. (c)
2
= [ 4 2.5 j , 2 j , 4 2.5 j ] 5
y(n) = sin n x(n)
6
Let, x(n) = (n)
8. (a)
y(n) = sin0 = 0 (Bounded)
u(t ) + u( t ) BIBO stable.
Even part =
2
u(t ) u( t ) 12. (b)
Odd part = The Hilbert transformer is characterised by the
2
impulse response,
u(t ) u(–t)
1
h(t) = t (– , )
1 1 t
t t
h(t) 0 for t < 0
Thus, the Hilbert transformer is a non-causal
system.
1
ue(t) = 13. (c)
2
A system is causal if the output at any time
ue(t )
depends only on values of the input at the
1/2 present time and in the past.

15. (d)
t
t
y = x ( ) cos(3 ) d
x(t )
uo(t) =
2 t t0
uo(t) y(t – t0) = x ( ) cos(3 ) d

1/2
y (t) for input x(t – t0) is
t t
0
–1/2 y (t) = x( t0 )cos 3 d

(t t0 )
9. (a) y (t) = x ( )cos 3( + t0 ) d

s(t) = 8 cos 20 t + 4 sin 15 t y (t) y(t – t0) so system is not time invariant for
2
input x( ) = cos(3 ) (bounded input),
= 8 sin20 t + 4 sin15 t
t
82 4 2 y(t) = cos2 (3) d as t
P= + = 32 + 8 = 40
2 2
So, for bounded input-output is not bounded
therefore system is not stable.
8 Electronics Engineering Signals & Systems

16. (a) Energy of x2[n] = 1.5 + 1.5 = 3

1 = 100, 2 = 300, 3 = 500 2 4


=3
HCF of ( 1, 2 and 3) = HCF (100, 300, 500),
3
= 100 rad/sec. 9
2 = = 1.5
4
17. (b)
Step response = Integration of impulse response, 21. (b)
t y(t ) A signal is said to be periodic if T/Ts is a
(t 1) dt = u(t – 1) rational number.
2
Here, T = 1.2 Ts
t 1
(t 3) dt = u(t – 3) T 6
t =
1 2 3 Ts 5
At, t=2 Which is a rational number.
y(t) = 1
22. (a)
18. (d) Since present output does not depend upon
x[n] = sin( 2 n) future values of input, the system is causal and
2
o = also every bounded input produces bounded
2 output, so it is stable.
N= m
o
23. (b)
where, m is the smallest integer that converts
t
2 / o into a integer value.
Given that, y(t) = x(u) du
2 2
N= 2
m= m t T

• Since the given system satisfies both


So, there exists no such integer value of m which
homogeneity and additivity properties, the
could make the N integer, so the system is not
system is linear.
periodic.
• Check for time invariance:
19. Sol. t to
x(t) = –3t, –1 < t < 1 y(t – t0) = x(u) du
t to T
t 1 3
x = (t 1) –1 < t < 1
2 2 • When the applied input is x(t – t0),
and T=6 t t to

2 y1(t) = x(u to ) du = x( ) d
3
1 3 t T t to T
Average power = (t 1) dt = 2
6 2 = y(t – t0)
1
System is time invariant.
20. Sol.

2
24. (c)
Energy of x1[n] = x1 [ n ]
y = au + b, b 0 is a non-linear system.
n=

2 1 n 1
= = 2
4 1
n=0 1
4
2 4
=
3
GATE Previous Years Solved Paper 9

Answers
EE Basics of Signals & Systems

2. (b) 3. (d) 4. (d) 5. (c) 6. (d) 7. (b) 8. (a) 9. (a)

10. (d) 11. (6) 12. (d) 13. (a) 14. (b)

Solutions
EE Basics of Signals & Systems

1. Sol. Now we will take every 4th samples of z1[n]


Using, i.e. z2[n] = z1[4n] = x[4n + 3]
Now reverse (time reverse) z2[n] will give
x(t ) (t t0 ) dt = x(t0) y[n] = z2[–n] = x[–4n + 3]

8. (a)
We have,
6
2t
e (t t1 ) dt = e–2 1
5

2. (b) t
0 T 2T
1/2
T /2 2 T
1 2A 2 The given function can be realized as follows:
Rms value = t dt + A dt
T T
0 T /2 u(t) u(t – T)

1 1
2
= A
3

t t
3. (d) 0 0 T
u(t) u(t – T)
Rms value (Unit step function right shifted)
(Unit step function)
2 (i) (ii)
/3 2 /3
1
= (100)2 dt + ( 100)2 dt + (100)2 dt r(t)
0 /3 2 /3
1 1
1/2
1
= 100 2 = 100 V
t t
0 0 T
4. (d) r(t) = tu(t) (t T )
(Ramp function) u( t T )
T
t t
1 1 5 (Right shifted unit ramp function)
Vc (t ) = i(t ) dt = 5 (t ) dt = u(t ) (iii)
C C C
1
7. (b)
y[n] = x[3 – 4n] = x[–4n + 3]
t
So to obtain y[n] we first advance x[n] by 3 unit. 0 T 2T
i.e., z1[n] = x[n + 3] (t T )
u(t 2T )
T
(iv)
10 Electronics Engineering Signals & Systems

Combining the functions (i), (ii), (iii) and (iv), Invertibility test:
we get the given function. x(n) y( n )
(n 1) [1 + ( 1)n ] ( n 1) = [1 + ( 1)1 ( n 1) = 0 (n 1) = 0
f(t)
2 (n 1) [1 + ( 1)n ]2 ( n 1) = [1 + ( 1)1 ]2 ( n 1) = 0.2 ( n 1) = 0

1 Thus, we are getting many to one mapping


between input and output. So, the system is non-
invertible.

11. Sol.
t Mean square value = Power of f(t)
0 T 2T
1 2
Mean square value = f (t ) dt
Therefore, T0 T0

(t T ) 1 2
f(t) = u(t ) u(t T ) + = [4 × 1 + 2 2 × 2]
T 4
(t 2t ) 16 + 8 24
u(t T ) u(t 2T ) = = =6
T 4 4
1 1
u(t ) u(t T ) + r (t T ) r(t 2T ) 12. (d)
T T
1 x(t) = r(t – 1) – r(t – 2) – r(t – 3) + r(t – 4)
or u(t ) u(t T ) + (t T ) u(t 2T ) u(t 2T )
T x(t)

9. (a) 1

t–1 –(t – 4)
t
To find value of e (2t 2) dt
t
1 2 3 4
1
Since, (t 1)
(2t – 2) =
2 2
x(t ) dt
Ex(t) =
Above integral can be written as,

1 1 t 1 2 3 4
e t
(t 1) dt = e =
2 2e = (t 1)2 dt + 12 dt + { (t 4)} 2 dt
2
1 2 3

10. (d) 1 1 2 5
= + 1+ = +1 =
3 3 3 3
Given relationship,
y(n) = [1 + (–1)n] x(n) 13. (a)
The invariant test: a and T represents positive quantities.
Delay of
u(t) is unit step function,
x( n) System y(n) y( n – 1)
‘1’
= [1 + (–1)
n–1
]
h(t) = 1 + e–at u(t), is non-causal
x(n – 1) Here ‘1’ is a constant and two sided so the
system will be non-causal, because for causal
Delay of System y (n) = [1 + (–1) ]
n system,
‘1’ x(n – 1) x(n – 1) h(t) = 0, t < 0
Since, y(n – 1) y (n) 0, t > 0
So, the system is time variant.
GATE Previous Years Solved Paper 11

14. (a, b) Case (ii): When K is imaginary or complex or


Given, y(t) = Kx(t) ...(i) real and negative then,
then, Average of y(t) = K × Average of x(t) K K
Y A = KXA and YR KXR
From equation (i), Thus, option (b) is satisfied, option (a) and (b)
Power of y(t) = K 2 power of x(t ) both satisfies the given condition.

YR2 = K 2 X R2 [Q Power = rms2]

YR = K X R
Case (i): When K is real and positive then,
K =K
and YR = KXR
Thus option (a) is satisfied.
2 LTI Systems Continuous and
Discrete (Time Domain)
ELECTRO NICS EN GINEERIN G 1 at
(a) ae–at (b) (1 e )
(GATE Previous Years Solved Papers) a
(c) a(1 – eat) (d) 1 – e–at
Q.1 The impulse response and the excitation
[EC-1998 : 1 Mark]
function of a linear time invariant causal system
are shown in Fig. (a) and (b) respectively. The Q.4 Let u(t) be the step function. Which of the
output of the system at t = 2 sec. is equal to waveform in the figure corresponds to the
h(t ) x(t)
convolution of u(t) – u(t – 1) with u(t) –u(t – 2)?
1 1

1 1/2
(a) (b)
0 6 t(sec) 0 2 6 t(sec)
0 1 2 0 1 2 3
1 t t
(a) 0 (b)
2
1 1
3
(c) (d) 1
2
[EC-1990 : 2 Marks] (c) (d)

Q.2 Let h(t) be the impulse response of a linear time 0 1.5 2 3 0 1 3


invariant system. Then the response of the t t
system for any input u(t) is [EC-2000 : 2 Marks]
t
(a) Q.5 The impulse response functions of four linear
h ( ) u(t )d
0 systems S1, S2, S3, S4 are given respectively by
h1(t) = 1 ; h2(t) = u(t)
t
d
(b) h ( ) u(t )d u(t )
dt h3 (t ) = ; h4(t) = e–3t u(t)
0 t+1
t t where u(t) is the unit step function. Which of
(c) h ( ) u(t )d dt these systems is time invariant, causal, and
0 0 stable?
t (a) S1 (b) S2
(d) h 2 ( ) u(t )d (c) S3 (d) S4
0
[EC-2001 : 2 Marks]
[EC-1995 : 1 Mark]
Q.6 Convolution of x(t + 5) with impulse function
Q.3 The unit impulse response of a linear time (t – 7) is equal to
invariant system is the unit step function u(t). (a) x(t – 12) (b) x(t + 12)
For t > 0, the response of the system to an
(c) x(t – 2) (d) x(t + 2)
excitation e–at u(t), a > 0 will be
[EC-2002 : 1 Mark]
GATE Previous Years Solved Paper 13

Q.7 The impulse response h[n] of a linear time- (a) is positive and is positive.
invariant system is given by (b) is negative and is negative.
h[n] = u[n + 3] + u[n –2] – 2u[n – 7] (c) is positive and is negative.
where u[n] is the unit step sequence. (d) is negative and is positive.
The above system is [EC-2008 : 1 Mark]
(a) stable but not causal.
Q.10 A discrete-time linear shift-invariant system has
(b) stable and causal.
an impulse response h[n] with h[0] = 1, h[1] = –1,
(c) causal but unstable. h[2] = 2, and zero otherwise. The system is given
(d) unstable and not causal. an input sequence x[n] with x[0] = x[2] = 1 and
[EC-2004 : 1 Mark] zero otherwise. The number of non-zero samples
in the output sequence y[n], and the value of
Q.8 Which of the following can be impulse response
y[2] are, respectively
of a causal system?
(a) 5, 2 (b) 6, 2
h (t )
(c) 6, 1 (d) 5, 3
[EC-2008 : 2 Marks]
(a)
Q.11 A system is defined by its impulse response
t h(n) = 2n u(n – 2). The system is
(a) stable and causal.
h (t ) (b) causal but not stable.
(c) stable but not causal.
(b) (d) unstable and non-causal.
t [EC-2011 : 1 Mark]

Q.12 Let y[n] denotes the convolution of h[n] and g[n],


h (t ) where h[n] = (1/2)n u[n] and g[n] is a causal
sequence. If y[0] = 1 and y[1] = 1/2, then g[1]
(c) equals
1
t (a) 0 (b)
2
3
h (t ) (c) 1 (d)
2
[EC-2012 : 2 Marks]
(d)
Q.13 Two systems with impulse responses h1(t) and
t h2(t) are connected in cascade. Then the overall
impulse response of the cascaded system is
[EC-2005 : 1 Mark] given by

Q.9 The impulse response h(t) of a linear time- (a) product of h1(t) and h2(t)
invariant continuous time system is described (b) sum of h1(t) and h2(t)
by h(t) = exp( t) u(t) + exp( t) u(–t), where u(t) (c) convolution of h1(t) and h2(t)
denotes the unit step function, and and are (d) subtraction of h2(t) and h1(t)
real constants. This system is stable if [EC-2013 : 1 Mark]
14 Electronics Engineering Signals & Systems

Q.14 Consider a discrete-time signal The impulse responses of the systems are
n for 0 0 10 h1(t) = 2 (t + 2) – 3 (t + 1)
x[n ] = h2(t) = (t – 2)
0 otherwise
If y[n] is the convolution of x[n] with itself, the If the input x(t) is a unit step signal, then the
value of y[4] is ________ . energy of y(t) is __________ .
[EC-2014 : 2 Marks] [EC-2017 : 2 Marks]

Q.15 The result of the convolution x(–t) (–t –t0) is Q.20 The output of y[n] of a discrete-time system for
(a) x(t + t0) (b) x(t – t0) an input x[n] is
(c) x(–t + t0) (d) x(–t – t0) y[n ] = max x [ k]
k n
[EC-2015 : 1 Mark]
The unit impulse response of the system is
Q.16 The impulse response of an LTI system can be
(a) 0 for all n.
obtained by
(b) 1 for all n.
(a) differentiating the unit ramp response
(c) unit step signal u[n].
(b) differentiating the unit step response
(d) unit impulse signal [n].
(c) integrating the unit ramp response
(d) integrating the unit step response [EC-2020 : 1 Mark]

[EC-2015 : 1 Mark] Q.21 For a unit step input u[n], a discrete-time LTI
system produces an output signal (2 [n + 1] +
Q.17 Which one of the following is an eigen function
of the class of all continuous-time, linear, time- [n] + [n – 1]). Let y[n] be the output of the system
invariant systems (u(t) denotes the unit-step n
1
function)? for an input u(n) . The value of y[0] is
2
(a) e j ot u(t ) (b) cos( ot)
________ .
(c) e j ot (d) sin ( ot) [EC-2021 : 2 Marks]
[EC-2017 : 1 Mark] Q.22 The output of four systems (S1, S2, S3 and S4)
Q.18 Two discrete-time signals x[n] and h[n] are both corresponding to the input signal sin(t) for all
non-zero only for n = 0, 1, 2 and are zero time t, are shown in the figure.
otherwise. It is given that,
sin(t ) S1 sin(–t)
x[0] = 1, x[1] = 2, x[2] = 1, h[0] = 1
Let y[n] be the linear convolution of x[n] and sin(t ) S2 sin(t + 1)
h[n]. Given that, y[1] = 3 and y[2] = 4, the value
sin(t ) S3 sin(2t)
of the expression (10 y[3] + y[4]) is ________ .
[EC-2017 : 2 Marks] sin(t ) S4
2
sin (t)

Q.19 Consider the parallel combination of two LTI


Based on the given information, which of the
systems shown in the figure.
four systems is/are definitely not LTI (linear and
h1(t) time-invariant)?
(a) S1 (b) S2
x(t) y(t) (c) S3 (d) S4
[EC-2022]
h2(t)
GATE Previous Years Solved Paper 15

ELECTRICAL EN GINEERIN G t
(a) s (t ) u( ) d
(GATE Previous Years Solved Papers) 0

Q.1 s(t) is step response and h(t) is impulse response t


d
of a system. Its response y(t) for any input u(t) is (b) s (t ) u( ) d
dt
0
given by
t t t
d s(t ) u(
(a) s(t ) u( ) d (c) 1) d 1 d
dt 0 0
0
t t
(b) s (t ) u( ) d (d) s (t )2 u( ) d
0 0
t [EE-2000 : 1 Mark]
(c) s (t ) u( 1) d 1 d
00 Q.7 x[n] = 0, n < –1, n > 0, x[–1] = –1, x[0] = 2 is the
t input and y[n] = 0, n < –1, n > 2, y[–1] = –1 =
d
(d) h( t ) u( ) d [EE-1993 : 1 Mark] y[1], y[0] = 3, y[2] = –2 is the output of a discrete-
dt
0
time LTI system.
Q.2 If f(t) is the step-response of a linear time- The system impulse response h[n] will be
invariant system, then its impulse response is (a) h[n] = 0, n < 0, n > 2, h[0] = 1, h[1] = h[2] = –1
given by ______ . (b) h[n] = 0, n < –1, n > 1, h[–1] = 1, h[0] = h[1] = 2
[EE-1994 : 1 Mark] (c) h[n] = 0, n < 0, n > 3, h[0] = –1, h[1] = 2,
Q.3 The convolution of the function f1(t) = e–2t u(t) h[2] = 1
and f2(t) = et u(t) is equal to ______. (d) h[n] = 0, n < –2, n > 1, h[–2] = h[1] =
[EE-1995 : 1 Mark] h[–1] = –h[0] = 3
[EE-2006 : 2 Marks]
Q.4 The impulse response of an initially relaxed
linear system is e–2t u(t). To produce a response Q.8 If u(t), r(t) denote the unit step and unit ramp
of te–2t u(t), the input must be equal to functions respectively and u(t) r(t) their
convolution, then the function u(t + 1) r(t – 2)
1 2t
(a) 2 e–t u(t) (b) e u(t ) is given by
2
(c) e–2t u(t) (d) e–t u(t) 1
(a) (t 1) u (t 2)
2
[EE-1995 : 1 Mark]
1
Q.5 The unit impulse response of a system is given (b) (t 1) u (t + 2)
2
as,
1
c(t) = –4e–t + 6e–2t (c) (t 1)2 u(t 1)
2
The step response of the same system for t 0 is
(d) None of the above
equal to
(a) –3e–2t – 4e–t + 1 (b) –3e–2t + 4e–t + 1 Q.9 Let a signal a1 sin( 1t + 1) be applied to a stable
linear time invariant system. Let the
(c) –3e–2t – 4e–t – 1 (d) 3e–2t + 4e–t + 1
corresponding steady-state output be
[EE-1996 : 2 Marks]
represented as a2 F( 2t + 2). Then which of the
Q.6 Let s(t) be the step response of a linear system following statements is true?
with zero initial conditions. Then the response (a) F is not necessarily a ‘sine’ or ‘cosine’
of this system to an input u(t) is function but must be periodic with 1 = 2.
16 Electronics Engineering Signals & Systems

(b) F must be a ‘sine’ or ‘cosine’ function with Q.13 A linear time invariant system with an impulse
a1 = a2. response h(t) produces output y(t) when input
(c) F must be a ‘sine’ function with 1 = 2 and x(t) is applied. When the input x(t – ) is applied
1 = 2. to a system with impulse response h(t – ), the
(d) F must be a ‘sine’ or ‘cosine’ function with output will be
1 = 2. (a) y( ) (b) y(2(t – ))
[EE-2007 : 1 Mark] (c) y(t – ) (d) y(t – 2 )
Q.10 The impulse response of a causal linear time- [EE-2009 : 1 Mark]
invariant system is given as h(t). Now consider
Q.14 A cascade of three linear time invariant systems
the following two statements:
is causal and unstable. From this, we concludes
Statement (I) : Principle of superposition holds.
that
Statement (II) : h(t) = 0 for t < 0.
(a) each system in the cascade is individually
(a) Statement (I) is correct and statement (II) is
causal and unstable.
wrong.
(b) Statement (II) is correct and statement (I) is (b) at least one system is unstable and at least
wrong. one system is causal.
(c) Both statement (I) and statement (II) are (c) at least one system is causal and all systems
wrong. are unstable.
(d) Both statement (I) and statement (II) are (d) the majority are unstable and the majority
correct. are causal.
[EE-2008 : 1 Mark] [EE-2009 : 2 Marks]
Q.11 A signal e– t sin ( t) is the input to a real linear Q.15 The system represented by the input-output
time invariant system. Given K and are
5t
constants, the output of the system will be of the
relationship y(t ) = x( ) d , t > 0 is
from Ke– t sin(vt + ) where
(a) need not be equal to but v equal to .
(b) V need not be equal to but equal to . (a) linear and causal
(c) equal to and v equal to . (b) linear but not causal
(d) need not be equal to and v need not to be (c) causal but not linear
equal to . (d) neither linear nor causal
[EE-2008 : 1 Mark] [EE-2010 : 1 Mark]
Q.12 A system with x(t) and output y(t) is defined by Q.16 Given the finite length input x[n] and the
the input-output relation: corresponding finite length output y[n] of an
2t LTI system as shown below, the impulse
Y (t ) = x( t ) d response h[n] of the system is

The system will be h[n]


x[n] = {1, –1} y[n] = {1, 0, 0, 0, –1}
(a) causal, time-invariant and unstable.
(b) causal, time-invariant and stable.
(c) non-causal, time-invariant and unstable. (a) h[n] = {1, 0, 0, 1} (b) h[n ] = {1, 0, 1}
(d) non-causal, time-variant and unstable.
(c) h[n ] = {1, 1, 1, 1} (d) h[n] = {1, 1, 1}
[EE-2008 : 2 Marks]
[EE-2010 : 2 Marks]
GATE Previous Years Solved Paper 17

Q.17 Given two continuous time signals x(t) = e–t and Q.22 The impulse response of a continuous time
y(t) = e–2t which exist for t > 0, the convolution system is given by
z(t) = x(t) y(t) is h(t) = (t – 1) + (t – 3)
(a) e–t – e–2t (b) e–3t The value of the step response at t = 2 is
(c) e +t (d) e–t + e–2t (a) 0 (b) 1
[EE-2011 : 1 Mark] (c) 2 (d) 3

Q.18 Let y[n] denote the convolution of h[n] and g[n], [EE-2013 : 2 Marks]
where h[n] = (1/2)n u[n] and g[n] is a causal
Q.23 x(t) is non-zero only for Tx < t < Tx , and
sequence. If y[n] = 1 and y[1] = 1/2, then g[1]
equals similarly, y(t) is non-zero only for Ty < t < Ty .
(a) 0 (b) 1/2
Let z(t) be convolution of x(t) and y(t). Which
(c) 1 (d) 3/2
one of the following statements is true?
[EE-2012 : 2 Marks]
(a) z(t) can be non-zero over an unbounded
Q.19 The input x(t) and output y(t) of a system are interval.
related as, (b) z(t) is non-zero for t < Tx + Ty .
t
(c) z(t) is zero outside of Tx + Ty < t < Tx + Ty .
y (t ) = x( ) cos(3 ) d

(d) z(t) is non-zero for t > Tx + Ty .


The system is
(a) time-invariant and stable. [EE-2014 : 1 Mark]
(b) stable and not time-invariant.
Q.24 Consider an LTI system with impulse response
(c) time-invariant and not stable. h(t) = e–5t u(t). If the output of the system is
(d) not time-invariant and not stable. y(t) = e–3t u(t) – e–5t u(t) then the input, x(t) is
[EE-2012 : 2 Marks] given by
Q.20 The impulse response of a system is h(t) = t u(t). (a) e–3t u(t) (b) 2 e–3t u(t)
For an input u(t – 1), the output is (c) e–5t u(t) (d) 2 e–5t u(t)
[EE-2014 : 1 Mark]
t2 t (t 1)
(a) u(t ) (b) u(t 1)
2 2 Q.25 The impulse response g(t) of a system, G is as
shown in Fig. (a). What is the maximum value
(t 1)2 t2 1
(c) u(t 1) (d) u(t 1) attained by the impulse response of two
2 2
cascaded blocks of G as shown in Fig. (b)?
[EE-2013 : 1 Mark]

Q.21 Two systems are impulse response h1(t) and 1


h2(t) are connected in cascaded. Then the overall G G
impulse response of the cascaded system is
t (b)
given by 0 1
(a)
(a) product of h1(t) and h2(t)
(b) sum of h1(t) and h2(t) 2 3
(a) (b)
(c) convolution of h1(t) and h2(t) 3 4
(d) subtraction of h2(t) from h1(t) 4
(c) (d) 1
[EE-2013 : 1 Mark] 5
[EE-2015 : 1 Mark]
18 Electronics Engineering Signals & Systems

Q.26 A moving average function is given by Q.30 Consider a causal LTI system characterized by
t differential equation,
1
y( t ) = u( ) d
T dy(t ) 1
t T + y(t ) = 3x(t )
dt 6
If the input ‘u’ is a sinusoidal signal of frequency
The response of the system to the input
1 x(t) = 3e–t/3 u(t), where u(t) denotes the unit step
Hz, then the steady-state, the output ‘y’ will
2T function, is
lag ‘u’ (in degree) by _______ . (a) 9 e–t/3 u(t)
[EE-2015 : 1 Mark] (b) 9 e–t/6 u(t)
(c) 9 e–t/3 u(t) – 6 e–t/6 u(t)
Q.27 For linear time invariant systems, that are (d) 54 e–t/6 u(t) – 54 e–t/3 u(t)
bounded input bounded output stable, which
[EE-2016 : 1 Mark]
one of the following statement is true?
(a) The impulse response will be integrable, but Q.31 Let z(t) = x(t) y(t), where ‘ ’ denotes
may not be absolutely integrable. convolution. Let c be a positive real-valued
(b) The unit impulse response will have finite constant. Choose the correct expression for z(ct).
support. (a) c x(ct) y(t) (b) x(ct) y(ct)
(c) The unit step response will be bounded. (c) c x(ct) y(ct) (d) c x(ct) y(t)
(d) The unit step response will be bounded. [EE-2017 : 1 Mark]
[EE-2015 : 2 Marks] Q.32 A continuous-time input signal x(t) is an eigen
Q.28 Consider a continuous-time system will input function of an LTI system, if the output is
x(t) and output y(t) given by (a) kx(t), where k is an eigen value.
y(t) = x(t) cos(t) (b) ke j t x(t), where k is an eigen value and e j t
The system is is a complex exponential signal.
(c) x(t) ej t, where e j t is a complex exponential
(a) linear and time-invariant.
signal.
(b) non-linear and time-invariant.
(d) k H( ), where k is an eigen value and H( ) is
(c) linear and time-varying
a frequency response of the system.
(d) non-linear and time-varying.
[EE-2018 : 1 Mark]
[EE-2016 : 1 Mark]
Q.33 Suppose for input x(t) a linear time-invariant
Q.29 The output of a continuous-time, linear time-
system with impulse response h(t) produces
invariant system is denoted by T[x(t)] where x(t)
output y(t), so that x(t) h(t) = y(t). Further, if
is the input signal. A signal z(t) is called eigen
signal of the system T, when T[z(t)] = yz(t), where x(t ) h(t ) = z(t ), which of the following
y is a complex number, in general, and is called
statements is true?
an eigen value of T. Suppose the impulse
(a) For some but not all t (– , ), z(t) y(t)
response of the system T is real and even. Which
of the following statements is true? (b) For all t (– , ), z(t) y(t)
(a) cos(t) is an eigen-signal but sin(t) is not. (c) For all t (– , ), z(t) y(t)
(b) cos(t) and sin(t) are both eigen-signals but (d) For some but not all t (– , ), z(t) y(t)
with different eigen values. [EE-2020 : 2 Marks]
(c) sin(t) is an eigen-signal but cos(t) is not.
(d) cos(t) and sin(t) are both eigen-signals with
identical eigen values.
[EE-2016 : 2 Marks]
GATE Previous Years Solved Paper 19

Electronics & Electrical Engineering


GATE Previous Years Solved Paper

A n swe rs & Expl a n a t i o n s

Answers
EC LTI Systems Continuous and Discrete (Time Domain)

1. (b) 2. (a) 3. (b) 4. (b) 5. (d) 6. (c) 7. (a) 8. (b)

9. (d) 10. (d) 11. (b) 12. (a) 13. (c) 14. (10) 15. (d) 16. (b)

17. (c) 18. (31) 19. (7) 20. (c) 21. (0) 22. (c, d)

Solutions
EC LTI Systems Continuous and Discrete (Time Domain)

1. (b) 2

For causal LTI system, y(2) = x( ) h(2 )d


0
h(t) = 0 for t < 0
y(t) = x(t) h(t) 1 1 1
So, y(2) = × 2× =
2 2 2
y(t) = x( ) h(t )d
2. (a)
t For LTI system,
y(t) = x( ) h(t )d y(t) = u(t) h(t)
0
As, x(t) = 0 ; t<0 y(t) = h( ) u(t )d
and h(t) = 0 ; t<0
Q u(t) = 0 ; for t < 0
h(2 – )
t
y(t) = h( ) u(t )d
1 0

3. (b)
–4 0 2
Given that:
Impulse response = h(t) = u(t)
x( )
So, transfer function,
1
H(s) =
s
1/2
Y (s )
H(s) =
2 X( s )
Y(s) = H(s) × H(s)
20 Electronics Engineering Signals & Systems

Given that, 6. (c)


x(t) = e–at u(t) x(t + 5) (t – 7) = x(t + 5 – 7)
1 = x(t – 2)
X(s) =
(s + a)
7. (a)
1 1
Y(s) = ×
s (s + a ) h( k ) =
1 1 1 k=
Y(s) =
a s s+a
u( k + 3) + u( k 2)
Taking inverse Laplace transform,
k= 3 k=2
1 at
y(t) = [1 e ]
a 2 u( k 7)
k =7
4. (b)
6 6
1 s = 1+ 1
u(t) – u(t – 1) = [1 e ]
s k= 3 k=2

1 = 10 + 5 = 15 <
[1 e 2 s ]
u(t) – u(t – 2) =
s For bounded input, bounded output. So system
Convolution in the domain = Multiplication in is stable.
s-domain Response depends on future value of input
1 signal i.e., u(n + 3). So, system is not causal.
[1 e s ] [1 e 2s
]
s2 8. (b)
1 2s s 3s For a causal system,
= [1 e e +e ]
s2 h(t) = 0 ; for t < 0
= tu(t) – (t – 1) u(t – 1) – (t – 2) u(t – 2) + (t – 3)
9. (d)
u(t – 3)
h(t) = e t u(t) + e t u(–t)
= r(t) – r(t – 1) – r(t – 2) + r(t – 3)

5. (d) For the system to be stable, h(t ) dt < .


h1(t) 0 ; for t < 0
For the above condition, h(t) should be as shown
Therefore s1 is non-causal
below.
h2(t) = u(t) = 0 for t < 0
h (t )
h2 (t ) dt = u(t ) dt = dt =
0 t – t
e u(–t) e u (t)
Therefore s2 is unstable and causal,
u(t ) t
h3(t) =
t+1
Therefore, < 0 and > 0.
At, t = –1
h3(t) = 10. (d)
h3(t) = 0 for t < 0 L1 = 3, L2 = 3
Therefore s3 is unstable and causal, Number of samples in output
h4(t) = e–3t u(t) = L1 + L2 – 1 = 5
s4 is time invariant, causal and stable.
GATE Previous Years Solved Paper 21

h[–k]
y[n] = x[ k ] h[n k ]
k= 1/2 1

1/4
y[2] = x[ k ] h[2 k ]
k=

y(2) = 3

x [k ] h[k] y[0] = h [ k ] g[ k ]
k=
1 1 2
y[0] = h[0] g[0]
1
1 = 1 g[0]
k k g[0] = 1
0 1 2 0 1 2
–1
y[1] = h [1 k ] g[ k ]
k=
h[–k] h[2 – k]
y[1] = h[1] g[0] + h[0] g[1]
2 2
h[1 – k] will be zero for k > 1 and g[k] will be
1 1
zero. For k < 0 as it is causal sequence,
–1
k k 1 1
–2 0 0 1 2
= × 1 + 1 g[1]
–1 –1 2 2
g[1] = 0
11. (b)
13. (c)
h(n) = 2n u(n – 2)
The overall impulse response h(t) of the
For causal system,
cascaded system is given by
h(n) = 0 ; for n < 0
h(t) = h1(t) h2(t)
Hence given system is causal.
For stability: 14. Sol.
n ; for 0 n 10
2n = x[n] =
n=2 0; otherwise

So, given system is not stable.


y[n] = x( k ) x( n k )
12. (a) k=

n 10
1 x( k ) x(4 k )
h[n] = u [ n] y[4] =
2 k=0
g[n] = ?? = x(0) x(4) + x(1) x(3) + x(2) + x(3)
x(1) + 0
g(0) g(1)
= 0 + (1 × 3) + (2 × 2) + (3 × 1)
= 10

15. (d)
x(–t) (–t – t0) = x(–t) (t + t0)
y[n] = h [n k ] g[ k ] = x(–t – t0)
k=
22 Electronics Engineering Signals & Systems

16. (b) By plotting y(t), we get


Step response, y(t )
t
y(t) = h( ) d 2

d
h(t) = y (t ) –2 –1 0 2
dt t(sec)

17. (c)
–1
If the input to the system is eigen signal output
is also the same eigen signal. The energy of y(t) can be given as,

18. Sol.
Ey = y 2 (t ) dt
x[n] = {1, 2, 1}
h[n] = {1, a, b} By plotting y2(t), we get
y[n] = {A, 3, 4, B, C}
y2(t)
h
x 1 a b 4

1 1 a b
1
2 2 2a 2b
–2 –1 0 2 t(sec)
1 1 a b

Ey = Area under the plot of y2(t)


y[0] = 1; y[1] = 2 + a
= (4 × 1) + (3 × 1) = 7 J
y[2] = 1 + 2a + b
y[3] = a + 2b, y[4] = b 21. (0)
Given, y[1] = 2+a=3 a=1 1
y[2] = 1 + 2a + b = 4 b = 1 x(n) = u(n) X( z) = 1
1 z
So, y[3] = a + 2b = 3
y(n) = 2 [n + 1] + [n] + [n – 1]
and y[4] = b=1
Y(z) = 2z + 1 + z–1
So, 10y[3] + y[4] = 10 × 3 +1 = 31
Transform function,
19. Sol. Y ( z) 2 z + 1 + z 1
H(z) = =
Given that, X( z) 1
h1(t) = 2 (t + 2) – 3 (t + 1) 1 z 1
h2(t) = (t – 2) = (2z + 1 + z–1) (1 – z–1)
Overall impulse response is, = 2z + 1 + z–1 – 2 – z–1 – z–2
h(t) = h1(t) + h2(t) H(z) = 2z – 1 – z–2
= 2 (t + 2) – 3 (t + 1) + (t – 2) n
1
If input, x(t) = u(t), then the output will be If x(n) = u(n) for this system with
2
y(t) = x(t) h(t)
= u(t) * [2 (t + 2) – 3 (t + 1) H ( z) = 2z – 1 – z–2 then,
+ (t – 2)] find y (n )I.Z.T.
= 2u(t + 2) – 3u(t + 1) + u(t – 2)
h(n) = 2 (n + 1) – (n) – (n – 2)
GATE Previous Years Solved Paper 23

22. (c, d)
2z 1 z 2
Y(z) = X ( z) H ( z) =
1 1 sin t S1 sin(–t) = –sin t
1 z
2
sin t S2 sin(t + 1)
y(n) = x(n) h(n)
n+1 n sin t S3 sin(2t)
1 1
= 2 u(n + 1) u(n)
2 2 1 cos 2 t
sin t S4 sin 2 (t ) =
2
n 2
1 Since, LTI system does not change the frequency
u(n 2)
2 of sinusoidal input. So, S3 and S4 are definitely
As we want y(0), it is due to first 2 terms of y(n), not LTI as input and output sinusoidal
1 0+1
1 0 frequencies are different.
y(0) = 2
2 2
= 1–1=0

Answers
EE LTI Systems Continuous and Discrete (Time Domain)

1. (a) 4. (c) 5. (b) 6. (b) 7. (a) 8. (c) 9. (d) 10. (d)

11. (c) 12. (d) 13. (d) 14. (b) 15. (b) 16. (c) 17. (a) 18. (a)

19. (d) 20. (c) 21. (c) 22. (b) 23. (c) 24. (b) 25. (d) 26. (90)

27. (d) 28. (c) 29. (d) 30. (d) 31. (a) 32. (a) 33. (b)

Solutions
EE LTI Systems Continuous and Discrete (Time Domain)

1. (a) 2. Sol.
y(t) = u(t) h(t) (always) d
f (t )
where h(t) is impulse response dt
Q s(t) is step response (given)
3. Sol.
d
s(t ) = h(t) Q f1(t) = e–2t u(t)
dt
d 1
F1(s) =
y(t) = u(t ) s(t ) s+2
dt
d and f2(t) = et u(t)
= [u(t ) s(t )] (Using property)
dt 1
F2(s) =
s 1
d
= u( ) s(t )d
dt 1
F1(s) F2(s) =
(s + 2) (s 1)
(By definition of convolution)
t 1/3 1/3
d = +
= u( ) s(t )d s+2 s 1
dt
0 Q F1(s) F2(s) = f1(t) f2(t)
(Q u(t) and s(t) are causal functions)
24 Electronics Engineering Signals & Systems

Multiplication in s-domain results in 8. (c)


convolution in time domain.
u(t + 1) r(t – 2) = r( 2) u(t + 1 )d
1/3 1/3
f1(t) f2(t) = L 1 +
s+2 s 1 ( t + 1)
z(t) = r( 2) d
1 2t 1
= e + et u(t ) (t + 1)
3 3 z(t) = ( 2) d
2
For, (t + 1) 2
4. (c)
2 ( t + 1)
1 1 (t + 1)
H(s) = L [ e 2t u(t )] = z(t) = 2 = [ 2
4 ]2
s+2 2 2
2
2t 1
Y(s) = L [t e u(t )] = 1
(s + 2)2 For t 1= [(t + 1)2 4(t + 1) 4 + 8]
2
Y (s ) 1
X(s) = = 1 2
X( s ) s + 2 z(t) = [t + 2t + 1 4t 4 + 4]
2
Hence, input x(t) = L –1 [X(s) = e–2t u(t)]
For t 1
5. (b) 1
z(t) = (t 1)2
Impulse response = 4 e–t +6 e–2t 2
t t t For t 1
Step response = c(t ) dt = 4 e t dt + 6 e 2t
dt 1
0 0 z(t) = (t 1)2 u(t 1)
2
= 4e–t – 3e–2t – 1
Hence, option (b) is correct. 9. (d)

7. (a) Input |H| fH Output


For finite duration convolution,
x[n] have M terms The effect of linear time invariant system is best
h[n] have N terms understood by considering that, at any given
then, y[n] should have terms (M + N – 1) input frequency 1, the output is sinusoidal at
Here, x[n] = { 1, 2} same frequency, as the input applied, with
amplitude given by output = H × input and
y[n] = { 1, 3, 1, 2}
phase equal to 2 = H + 1, where H is the
So, h[n] should have only 3 terms and h[n] have
magnitude of the frequency response and H is
value starting from origin or [n = 0] only because
y[n] have value start from [n = –1]. its phase angle. Both H and H are functions
Consider option (I). of frequency.
–1 1
1 –1 2 10. (d)

–1 1 –2 Since system is causal therefore its impulse


response will be zero for t < 0.
–1 1 –2 So statement-II is correct.
since system is linear it also satisfies the
principle of superposition.
y[n] = { 1, 3, 1, 2}
So both the statement are correct.
So option (a) is correct.
GATE Previous Years Solved Paper 25

11. (c)
e at j H( + j 0 ) j 0t
= H( +j 0 ) [e e
t LTI system t 2j
e sin( ot ) Ke sin( vt + )
h(t ) = real signal
j H( +j 0) j 0t
For real h(t): e e ]
h(t) = h (t) = Ke– t sin(vt + )
Applying Laplace transform,
12. (d)
H(s) = H (s )
H(s + j ) = H ( – j ) [Put s = + j ] First of all we will check for causality,
= 0, = –a 2t
y(t) = x( ) d
H(– + j 0) = H (– – j 0)
So for, t = –2
+ j 0)
H( + j 0 ) e j H(
4
y(–2) = x( ) d
j H( j 0)
= H( j 0) e
So output depends on future values of input
By comparison,
along with past and present values of input so
H( +j 0) = H( j 0) ...(i) system is non-causal.
and H(– + j 0) = – H(– – j 0) ...(ii) Let us find output for shifted input x(t – t0),
Now system input is, 2t
y (t) = x( t0 ) d
x(t) = e– t sin 0t
(2 t t0 )
1 t y (t) = x( ) d ...(i)
= e ( e j 0t e j 0t
)
2j
Now shift the output by t0
1 t( + j 0) (2 t t0 )
= e et ( j 0 )
then, y(t – t0) = x( ) d ...(ii)
2
1 so from equations (i) and (ii),
= [ x1 (t ) x2 (t )] y (t) y(t – t0)
2j
Therefore system is time variant.
+ j 0)
where, x1(t) = et ( Non-causal and time variant is present only in
t( j 0)
option (d).
and x2(t) = e
For x1(t) system output is, 13. (d)
Case-1 : Y(s) = H(s) X(s)
y1(t) = H ( s ) s = j 0
e( j 0 )t
Case-2 : Input = x(t – ) X(s) e–s
= H( j 0) e( j 0 )t Impulse response = h(t – ) H(s) e–s
Thus, the overall output y(t) is, Output : Y (s) = X(s) e–s H(s) e–s
Y (s) = X(s) H(s) e–s2t = Y(s) e–2sT
1
y(t) = [ y1 (t ) y2 (t )] y(t) = y(t – 2 )
2j
[Q By using property of linear system] 14. (b)
1
[H( + j 0 ) e( + j 0 )t
H( j 0 ) e( j 0 )t
] Since in cascade overall impulse response,
= 2j
h(t) = h1(t) h2(t) h3(t)
e at h 1 (t), h 2 (t), h 3 (t) are impulse response of
+ j 0)
= [ H( +j 0 ej H(
ej 0t
individual systems.
2j
Since initial point where h(t) is non-zero is t 0
H( j 0 ) e j H( j 0)
e j 0t ] and since in convolution initial point,
= t1 + t2 + t3
26 Electronics Engineering Signals & Systems

where, t1, t2, t3 are initial points of h1(t), h2(t), 17. (a)
h3(t) respectively. 1
1
So for it to be greater than or equal to zero at x(s) = , y( s ) =
s+1 s+2
least one of them t1, t2 or t3 must be +ve i.e. greater
z(s) = x(s) y(s)
than zero so atleast one of them must be causal.
1 1 1 1
Similarly if one (atleast) of the system become = × =
s+1 s+2 s+1 s+2
unstable then overall system will become
z(t) = e–t – e–2t
unstable.
18. (a)
15. (b)
Given, g[n] is a causal sequence.
Integrator is always a linear system.
g[n] will be zero for n < 0,
5t
Since, y(t) = x( ) d t>0
y[n] = g[ k ] h[n k ]
For t = 1 k=
5 n
y(t) = x( ) d 1
and h[n] = u [ n]
2
here value at t = 1 depends on future values like
at t = 2, 3, .... of input x(t). n
1
u[n]
So, it is a non-causal system. h[n] = 2
0 n<0
16. (c)
1 1
x[n] = {1, 1}, M = 2 h[0] = 1, h [1] = , h [2] =
2 4
y[0] = g[0] h[0] + g[1] h[–1] + 1...
y[n] = {1, 0, 0, 0, 1}, N 1 = 5 1 = g[0] × 1 + 0 + 0 + ...
g[0] = 1
Since output has number of elements, y[1] = g[0] h[1] + g[1] h[0] + g[2] h[–1] +...
N1 = M + N – 1 1 1
= 1 + g[1] 1 + 0
where N is number of elements in impulse 2 2
response h[n]. g[1] = 0
5 = 2+N–1
19. (d)
N=4
t
Let it be a1, a2, b1, b2 y = x( ) cos(3 ) d
y[n] = [ a1 ,( a2 a1 ), (b1 a2 ), (b2 b1 ), b2 ] t t0
y(t – t0) = x( ) cos(3 ) d
comparing with,
y (t) for input x(t – t0) is
y[n] = [1, 0, 0, 0, 1] t
y (t) = x( t0 ) cos 3 d
a1 = 1
( t t0 )
a2 = a1 = 1 y (t) = x( ) cos 3( + t0 ) d
b1 = a1 = 1
y (t) y(t – t0) so system is not time invariant for
b2 = b1 = 1
input x( ) = cos(3 ) bounded input,
Therefore,
t
y(t) = cos2 (3 ) d as t
h[n] = {1, 1, 1, 1}
So, for bounded input, output is not bounded
therefore system is not stable.
GATE Previous Years Solved Paper 27

20. (c) Also, h(t) = e–5t u(t)


h(t) = t u(t) 1
H(s) =
1 (s + 5)
H(s) =
s2 Y (s)
=
2
× (s + 5)
X(s) =
Y (s) 1 H (s) (s + 3) (s + 5)
= 2
U (s) s 2
= (s + 3)
1 e s
Y(s) = Input = x(t) = 2e–3t u(t)
s2 s
(t 1)2 25. (d)
y(t) = u (t 1)
2 g(t) = u(t) – y(t –1)
1 e s
21. (c) G(s) =
s s
In cascade connection,
or, G(s) × G(s) = g(t) g(t)
H1(s) H2(s) g(t) g(t) g(t)

H(s) = H1(s) H2(s)


1 1
h(t) = h1(t) h2(t)

22. (b)
t
For step response, impulse response can be 1
0 1 2
integrated.
Maximum value = 1
y(t) = u(t – 1) + u(t – 3)
y(2) = u(1) + u(–1) 26. Sol.
= 1+0=1 System input: sin 0t

23. (c)
1
f0 = Hz
2T
x(t) is non-zero for Tx < t < Tx and y(t) is non- Therefore,

zero for Ty < t < Ty 2


0 = 2 0= = rad/sec.
2T T
Q z(t) = x(t) y(t) t
1 1 t
then the limits of the resultant signal is, y(t) = sin 0 d = [ cos 0 t ]t T
T T
Tx + Ty < t < Tx + Ty t T

i.e. z(t) is non-zero for Tx + Ty < t < Tx + Ty .


1
= [cos 0 t cos[ 0 (t T )]]
T
24. (b) t
0 =
Impulse response of an LTI system = transfer T
1 t
Y (s ) = cos at cos
function = = H (s) T T
X( s )
1 2
where, y(t) = e–3t u(t) – e–5t u(t) = [cos 0t + cos 0 t ] = cos 0 t
T T
1 1 2 2
Y(s) = = = sin( 0 t 90°)
(s + 3) ( s + 5) (s + 3) (s + 5) T
= Output is lagging by 90° w.r.t. input.
28 Electronics Engineering Signals & Systems

28. (c) 30. (d)


y(t) = x(t) cos(t) The differential equation,
To check linearity, dy(t ) 1
+ y(t ) = 3 x(t)
y1(t) = x1(t) cos(t) dt 6
[y1(t) is output for x1(t)]
1
y2(t) = x2(t) cos(t) So, sY (s ) + Y (s ) = 3 X(s)
6
[y2(t) is output for x2(t)]
3 X(s )
So, the output for (x1(t) + x2(t)) will be Y(s) =
1
y(t) = [x1(t) + x2(t)] cos(t) s+
6
= y1(t) + y2(t)
So, the system is linear to check time invariance. 9
X(s) =
The delayed output, 1
s+
y(t – t0) = x(t – t0) cos(t – t0) 3
The output for delayed input, 9
y(t, t0) = x(t – t0) cos(t) So, Y(s) =
1 1
s+ s+
Since, y(t – t0) y(t, t0) 3 6
System is time varying.
54 54
=
29. (d) 1 1
s+ s+
6 3
Given that impulse response is real and even.
Thus, H(j ) will also be real and even. So, y(t) = (54e–16t – 54e–1/3t) u(t)

j ot LTI j ot
31. (a)
e H(j o) e
system Time scaling property of convolution.
If, x(t) y(t) = z(t)
–j o t LTI –j o t
e H(–j o) e 1
system
Then, x(ct) y(ct) = z(ct )
c
Since, H(j ) is real and even. z(ct) = c × x(ct) y(ct)
Thus, H(j 0) = H(–j 0)
32. (a)
e jt + e jt
Now cos(t) is input i.e. is input/ Eigen function is a type of input for which output
2
is constant times of input.
output will be
i.e.,
H ( j 1) e jt + H ( j 1) e jt e jt + e jt
= H ( j 1)
at
e = x(t) H(s) y(t )
2 2
= H(j1) cos(t) where, x(t) = system input = eigen function
H(s) = transfer function of system
e jt e jt y(t) = system output
If sin(t) is input i.e. is input/output
2
Here, y(t) = H ( s ) s = a e at
will be
= K x(t)
H ( j 1) e jt + H ( j 1) e jt e jt + e jt
= H ( j 1) 2j
where, K = eigen value = H (s) s = a
2
= H(j1) sint x(t) = eigen function input = H (s) s = a
So, sin(t) and cos(t) are eigen signals with same
eigen values.
GATE Previous Years Solved Paper 29

33. (b) Case-2:


Since, y(t) = x(t) + h(t) x (t ) h (t )

and z(t) = x(t ) × h(t ) 1 1

Case-1:
t t
0 1 0 1
x(t ) h(t )

1
then, y(t) = z(t)
0 1
t t
0 1
–1
1

then, y(t) and z(t)


t
0 1 2
y(t ) z (t )

1 2 Thus, z(t) y(t) for all ‘t’


–1
t
0

t
–1 0 1 2
3 Fourier Series

ELECTRO NICS EN GINEERIN G Q.5 The trigonometric Fourier series of a periodic


time function can have only
(GATE Previous Years Solved Papers)
(a) cosine terms
Q.1 A half-wave rectified sinusoidal waveform has (b) sine terms
a peak voltage of 10 V. Its average value and the (c) cosine and sine terms
peak value of the fundamental component are (d) dc and cosine terms
respectively given by
[EC-1998 : 1 Mark]
20 10 10 10
(a) V, V (b) V, V Q.6 A periodic signal x(t) of period T0 is given by
2 2
10 20 1, t < T1
(c) V, 5 V (d) V, 5 V x( t ) = T0
0 , T1 < t <
[EC-1987 : 2 Marks] 2
The dc component of x(t) is
Q.2 Fourier series of the periodic function (period 2 )
defined by T1 T1
(a) (b)
T0 (2T0 )
0, <x<0
f (x) =
x, 0<x< 2T1 T0
(c) (d)
T0 T1
1 1
is + [cos(n ) 1] cos(nx ) cos(n )sin(nx ) [EC-1998 : 1 Mark]
4 n=1 n2 n
Q.7 The Fourier series representation of an impulse
By putting x = in the above, one can deduce train denoted by
1 1 1
that the sum of the series 1 + 2
+ 2
+ + ... s(t ) = (t nT0 )
3 5 72
n=
is
1 j 2 nt
[EC-1993 : 2 Marks] (a) exp
T0 n= T0
Q.3 The Fourier series of an odd periodic function,
contains only 1 j nt
(b) exp
(a) odd harmonics (b) even harmonics T0 n= T0
(c) cosine terms (d) sine terms
1 j nt
[EC-1994 : 1 Mark] (c) exp
T0 n= T0
Q.4 The trigonometric Fourier series of an even
function of time does not have 1 j 2 nt
(d) exp
(a) the dc term T0 n= T0
(b) cosine terms [EC-1999 : 2 Marks]
(c) sine terms
(d) odd harmonic terms Q.8 One period (0, T) each of two periodic
[EC-1996 : 1 Mark] waveforms W1 and W2 are shown in the figure.
GATE Previous Years Solved Paper 31

The magnitudes of the nth Fourier series Q.12 The Fourier series of a real periodic function
coefficients of W1 and W2, for n 1, n odd, are has only
respectively proportional to P. Cosine terms if it is even.
Q. Sine terms if it is even.
R. Cosine terms if it is odd.
1 1
S. Sine terms if it is odd.
W1 W2 Which of the above statements are correct?
T (a) P and S (b) P and R
0 T/2 T 0 T/2
(c) Q and S (d) Q and R
[EC-2009 : 1 Mark]
–1 –1
Q.13 The trigonometric Fourier series of an even
function does not have the
(a) n 3 and n 2 (b) n 2 and n 3 (a) dc term (b) cosine terms
(c) sine terms (d) odd harmonic terms
1 2 4 2
(c) n and n (d) n and n [EC-2011 : 1 Mark]

[EC-2000 : 2 Marks] Q.14 Consider the periodic square wave in the figure
shown.
Q.9 Which of the following cannot be the Fourier
x(t)
series expansion of a periodic signal?
(a) x(t) = 2 cost + 3 cos3t 1

(b) x(t) = 2 cos t + 7 cost


(c) x(t) = cost + 0.5 t
0 1 2 3 4
(d) x(t) = 2 cos 1.5 t + sin3.5 t
[EC-2002 : 1 Mark]
–1
Q.10 The Fourier series expansion of a real periodic
signal with fundamental frequency fo is given by The ratio of the power in the 7th harmonic to the
power in the 5th harmonic for this waveform is
g p (t ) = cn e j 2 nf ot
closest in value to ______ .
n=
[EC-2014 : 1 Mark]
It is given that, c3 = 3 + j5. Then c–3 is
Q.15 The magnitude and phase of the complex
(a) 5 + j3 (b) –3 – j5
Fourier series coefficient ak of a periodic signal
(c) –5 + j3 (d) 3 – j5
x(t) are shown in the figure. Choose the correct
[EC-2003 : 1 Mark] statement from the four choices given.
Q.11 Choose the function f(t), – < t < , for which a Notation: C is the set of complex, number, R is
Fourier series cannot be defined. the set of purely real numbers, and P is the set of
(a) 3 sin(25t) purely imaginary numbers.
(b) 4 cos(20t + 3) + 2 sin(710t) ak
3 3
(c) exp ( t ) sin (25t ) 2 2

(d) 1 1

[EC-2005 : 1 Mark] k
–5 –4 –3 –2 –1 0 1 2 3 4
32 Electronics Engineering Signals & Systems

ak Q.18 Let x(t) be a periodic function with period T = 10.


–5 –4 –3 –2 –1 1 2 3 4 The Fourier series coefficients for this series are
k
0 denoted by ak, that is
2
– jk t
x( t ) = ak e T
(a) x(t) R k=
(b) x(t) P The same function x(t) can also be considered
(c) x(t) (C – R) as a periodic function with period T = 40. Let bk
(d) the information given is not sufficient to be the Fourier series coefficients when period is
draw any conclusion about x(t). taken as T .
[EC-2015 : 1 Mark]
If ak = 16 , then bk is equal to
Q.16 A periodic signal x(t) has a trigonometric Fourier k= k=
series expansion:
(a) 256 (b) 64
x(t ) = a0 + ( an cos n o t + bn sin n ot )
(c) 16 (d) 4
n=1 [EC-2018 : 1 Mark]
If x(t) = –x(–t) = –x(t – / o), we can conclude
Q.19 The exponential Fourier series representation
that
of a continuous-time period signal x(t) is defined
(a) an are zero for all n and bn are zero for n
as,
even.
(b) an are zero for all n and bn are zero for n odd. ak e jk ot
x(t) =
(c) an are zero for n even and bn are zero for n k=
odd. where, o is the fundamental angular frequency
(d) an are zero for n odd and bn are zero for n of x(t) and the coefficients of the series are ak.
even. The following information is given about x(t)
[EC-2017 : 1 Mark] and ak :

Q.17 Let x(t) be a continuous time periodic signal with I. x(t) is real and even, having a fundamental
fundamental period T = 1 seconds. Let {ak} be period of 6.
the complex Fourier series coefficients of x(t), II. The average value of x(t) is 2.
where k is integer valued. Consider the k, 1 k 3
following statements about x(3t): III. ak =
0, k>3
I. The complex Fourier series coefficients of
x(3t) are {ak} where k is integer valued. The average power of the signal x(t) (Rounded
off to one decimal place) is ______ .
II. The complex Fourier series coefficients of
x(3t) are {3ak} where k is integer valued. [EC-2021 : 2 Marks]
III. The fundamental angular frequency of x(3t) Q.20 The frequency response H(f) of a linear time
is 6 rad/s. invariant system has magnitude as shown in
For the three statements above, which one of the the figure.
following is correct?
|H(f )|
(a) Only II and III are true.
1
(b) Only I and III are true.
(c) Only III is true. f (Hz)
– 0
(d) Only I is true. [EC-2017 : 2 Marks]
GATE Previous Years Solved Paper 33

Statement-I : The system is necessarily a pure Q.3 Fourier series for the waveform, f(t) shown in
delay system for inputs which are band limited figure is
to – f .
f (t )
Statement-II : For any wide sense stationary
input process with power spectral density SX(f),
the output power spectral density SY(f) obeys
–1 1 2 3
SY(f ) = SX(f) for – f . Which one of the t
following combinations is true? –1
(a) Statement I is correct, Statement II is correct.
(b) Statement I is correct, Statement II is
8 1 1
incorrect. (a) 2
sin( t ) + sin(3 t ) + sin(5 t ) + ...
9 25
(c) Statement I is incorrect, Statement II is
correct. 8 1 1
(b) 2
sin( t ) cos(3 t ) + sin(5 t ) + ...
(d) Statement I is incorrect, Statement II is 9 25
incorrect.
8 1 1
[EC-2022] (c) 2
cos( t ) + cos(3 t ) + cos(5 t ) + ...
9 25

ELECTRICAL EN GINEERIN G 8 1 1
(d) 2
cos( t ) sin(3 t ) + sin(5 t ) + ...
(GATE Previous Years Solved Papers) 9 25
[EE-2002 : 1 Mark]
Q.1 A periodic rectangular signal, x(t) has the
waveform as shown in figure. Frequency of the Q.4 The rms value of the periodic waveform given
fifth harmonic of its spectrum is in figure is
x(t)
I

+6 A

T
t
–4 –2 0 2 4 t(ms) T/2

–6 A

(a) 40 Hz (b) 200 Hz


(c) 250 Hz (d) 1250 Hz (a) 2 6 A (b) 6 2 A
[EE-1996 : 1 Mark]
4
Q.2 If an a.c. voltage wave is corrupted with an (c) A (d) 1.5 A
3
arbitrary number of harmonics, then the overall
[EE-2004 : 2 Marks]
voltage waveform differs from its fundamental
frequency component in terms of Q.5 The rms value of the resultant current in a wire
(a) only the peak values which carries a d.c. current of 10 A and a
(b) only the rms values sinusoidal alternating current of peak value 20 A
(c) peak and rms values is

(d) all the three measures (peak, rms and (a) 14.1 A (b) 17.3 A
average values) (c) 22.4 A (d) 30.0 A
[EE-2000 : 1 Mark] [EE-2004 : 2 Marks]
34 Electronics Engineering Signals & Systems

Q.6 For the triangular wave form shown in the Q.10 Let x(t) be a periodic signal with time period T.
figure, the rms value of the voltage is equal to Let, y(t) = x(t – t0) + x(t + t0) for some t0. The
V(t) Fourier series coefficients of y(t) are denoted by
bk. If bk = 0 for all odd K. Then t0 canbe equal to
1V
T T
(a) (b)
8 4

T/2 T 3T/2 2T
t T
(c) (d) 2T
2
1 1 [EE-2008 : 2 Marks]
(a) V (b) V
6 3
Q.11 The Fourier series coefficients, of a periodic
1 2 signal x(t) expressed as,
(c) V (d) V
3 3
x(t) = k=
ak e j 2 kt /T
[EE-2004 : 2 Marks]
are given by
Q.7 The Fourier series for the function f(x) = sin2x is a –2 = 2 – j1
(a) sinx + sin2x (b) 1 – cos2x a –1 = 0.5 + j0.2
(c) sin2x + cos2x (d) 0.5 – 0.5 cos2x a0 = j2
[EE-2005 : 2 Marks] a1 = 0.5 – j0.2
a2 = 2 + j1
Q.8 x(t) is real valued function of a real variable with
period T. Its trigonometric Fourier series and ak = 0 for k > 2
expansion contains no terms of frequency Which of the following is true?
= 2 (2k)/T, k = 0, 1, 2, ..... Also, no sine terms (a) x(t) has finite energy because only finitely
are present. Then x(t) satisfies the equation many coefficients are non-zero.
(a) x(t) = –x(t – T)
(b) x(t) has zero average value because it is
(b) x(t) = x(T – t) = –x(t) periodic.
T (c) The imaginary part of x(t) is constant.
(c) x(t ) = x(T t ) = x t
2 (d) The real part of x(t) is even.
T [EE-2009 : 2 Marks]
(d) x(t ) = x(t T ) = x t
2
Q.12 The second harmonic component of the periodic
[EE-2006 : 1 Mark]
waveform given in the figure has an amplitude
Q.9 A signal x(t) is given by of
T 3T
1, <t
4 4
x( t ) = +1
3T 7T
1, <t
4 4 t
T/2 T
Which among the following gives the
fundamental Fourier term of x(t)? –1
4 t t
(a) cos (b) cos + (a) 0 (b) 1
T 4 4 2T 4
2
4 t t (c) (d) 5
(c) sin (d) sin
T 4 4 2T 4
[EE-2010 : 1 Mark]
[EE-2007 : 2 Marks]
GATE Previous Years Solved Paper 35

Q.13 The Fourier expansion: Q.17 The signum function is given by


x
f (t ) = a0 + an cos n t + bn sin n t , x 0
n=1 sgn ( x ) = x
0, x=0
of the periodic signal shown below will contain
the following non-zero terms: The Fourier series expansion of sgn (cos(t)) has
(a) only sine terms with all harmonics.
f (t )
(b) only cosine terms with all harmonics.
(c) only sine terms with even numbered
t harmonics.
0
(d) only cosine terms with odd numbered
harmonics.
(a) a0 and bn, n = 1, 3, 5, ....
[EE-2015 : 2 Marks]
(b) a0 and an, n = 1, 2, 3, ....
(c) a0, an and bn, n = 1, 2, 3, .... Q.18 Let f(x) be a real, periodic function satisfying
f(–x) = –f(x). The general form of its Fourier series
(d) a0 and an, n = 1, 3, 5, ....
representation would be
[EE-2011 : 1 Mark]
(a) f ( x ) = a0 + a
k =1 k
cos( kx )
Q.14 For a periodic signal

v(t ) = 30 sin 100t + 10 cos 300t + 6 sin 500t + , (b) f (x) = b


k =1 k
sin ( kx )
4
the fundamental frequency in rad/sec is (c) f ( x ) = a0 + a
k = 1 2k
cos( kx )
(a) 100 (b) 300
(d) f ( x ) = a
k = 0 2k + 1
sin (2 k + 1) x
(c) 500 (d) 1500
[EE-2013 : 1 Mark] [EE-2018 : 1 Mark]

Q.15 Let, g : [0, ) [0, ) be a function defined by


+
k
g(x) = x – [x], where [x] represents the integer Q.19 Let the signal x(t ) = ( 1)k t be
k= 2000
part of x. (That is, it is the largest integer which
is less than of equal to x.) The value of the
passed through an LTI system with frequency
constant term in the Fourier series expansion of
response H( ), as given in the figure.
g(x) is _______ .
H( )
[EE-2014 : 2 Marks]
1
Q.16 For a periodic square wave, which one of the
following statements is true?
(a) The Fourier series coefficients do not exist.
–5000 5000
(b) The Fourier series coefficient exist but the
The Fourier series representation of the output
reconstruction converges at most point. is given as,
(c) The Fourier series coefficient exist and the (a) 4000 + 4000 cos(2000 t) + 4000 cos(4000 t)
reconstruction converges at no points.
(b) 2000 + 2000 cos(2000 t) + 2000 cos(4000 t)
(d) The Fourier series coefficient exist and the
(c) 4000 cos(2000 t)
reconstruction converges at every point.
(d) 2000 cos(2000 t)
[EE-2014 : 1 Mark]
[EE-2017 : 2 Marks]
36 Electronics Engineering Signals & Systems

t t , t 0 If, f (t ) = a0 + an cos nt + bn sin nt


Q.20 Consider g(t ) = , where
t t , otherwise n=1 n=1

A sin t , 0 t
t R. Here t represents the largest integer less f (t ) =
0, <t<2
than or equal to t and t denotes the smallest The Fourier series coefficients a1 and b1 of f(t)
integer greater than or equal to t. The coefficient are:
of the second harmonic component of the A A
(a) a1 = 0 ; b1 = (b) a1 = ; b1 = 0
Fourier series representing g(t) is _______ .
[EE-2017 : 1 Mark] A A
(c) a1 = 0 ; b1 = (d) a1 = ; b1 = 0
2 2
Q.21 A periodic function f(t), with a period of 2 , is
[EE-2019 : 2 Marks]
represented as its Fourier series.

Electronics & Electrical Engineering


GATE Previous Years Solved Paper

A n swe rs & Expl a n a t i o n s

Answers
EC Fourier Series

1. (c) 2. (Sol.) 3. (d) 4. (c) 5. (c) 6. (c) 7. (d) 8. (c)

9. (b) 10. (d) 11. (c) 12. (a) 13. (c) 14. (0.51) 15. (a) 16. (a)

17. (b) 18. (c) 19. (32) 20. (a)

Solutions
EC Fourier Series

1. (c) Time period = To


V(t) 1
fo = = fundamental frequency
To
Vm
= 1st harmonics
t Fourier series of v(t) is given by
–T0 –T0/4 T0/4 T0
v(t) = a0 + a1 cos( ot) + a2 cos(2 ot) + ....
To a 0 = dc value = average value
V(t) = Vm cos( ot) t
4
GATE Previous Years Solved Paper 37

1 2. Sol.
a0 = v(t ) dt
To f(x)
To

To /4
1
= Vm cos( o t ) dt
To
To /4 x

To /4
1 Vm [sin o t ] To /4 Time period = T = – ( ) = 2
a0 = At the point of discontinuity,
To o
x =
Vm 2 To 2 To
a0 = sin sin f(x) expressed in Fourier series converge to the
2 To 4 To 4
To media value = /2. From the given trigonometric
To
form of Fourier series.
Vm
a0 = sin sin At, x =
2 2 2
2
f( ) = + s=
Vm V 4 2
= [1 ( 1)] = m × 2
2 2 1 1
where, s = 1+ 2
+ + ....
Vm 3 52
a0 =
2
s = =
Vm 10 2 4 4
So, average value = a0 = = V
2
s=
2 8
a1 = v(t ) cos( o t ) dt
To
To 3. (d)
To /4 The Fourier series of an odd periodic function
2
a1 = Vm cos( o t ) cos( o t ) dt does not contain the dc term (ao = 0) and cosine
To
To /4 terms. The Fourier series of an odd periodic
function contains only same terms.
To /4
2
a1 = Vm cos 2 ( o t ) dt 4. (c)
To
To /4
The trigonometric Fourier series of an even
To /4 function of time contains only dc term and
2Vm (1 + cos 2 ot)
a1 = dt cosine term. The trigonometric Fourier series of
To 2
To /4 an even function of time does not contain sine
terms.
To /4 To /4
2Vm
a1 = dt + cos 2 ot dt 5. (c)
To
To /4 To /4
The trigonometric Fourier series of a periodic
Vm To To V T time function can have only cosine and sine
a1 = × = m× o terms.
To 4 4 To 2

Vm 6. (c)
a1 =
2 x( t)
Vm = 10
10 1
So, a1 = =5V
2 t
–To/2 –T1 T1 To/2
38 Electronics Engineering Signals & Systems

Given periodic signal x(t) has time period To , Again integrating,


To /2
1
xdc = x(t ) dt
To
To /2

T
1 1 1 T1
= 1 dt = [t ] T 1
To To 1
T1 Cn
n2
1 2T
xdc = [Tt1 ( T1 )] = 1 9. (b)
To To
x(t) = 2 cos t + 7 cost
7. (d) 2
T1 = =2

s(t) = c k e jk ot
2
k= T2 = =2
1
To /2 T1 1
1 jk ot 1 = = irrational
ck = (t ) e dt = T2
To To
To /2
x(t) is not periodic and does not satisfy
1 Dirchlet condition.
s(t) = e jk ot
To k= 10. (d)

8. (c) c–k = c k
For impulse train the Fourier series coefficient c3 = 3 + j5
are: c–3 = c 3 = 3 j5

11. (c)
All other functions are either periodic or
constant function.

12. (a)
1 The Fourier series of a real periodic function
Cn =
Ts
has only cosine terms if it is even and only sine
terms if it is odd.

13. (c)
Trigonometric Fourier series of an even function
has dc and cosine terms only.

14. Sol.
A sin n o /2
Cn = x(t )
T n o /2
1
2A
= sin n o
Tn o 2
t
0 1 2 3 4
1
Cn
n
–1
GATE Previous Years Solved Paper 39

From Parseval’s theorem,


X( ) = 2 ak ( k ) T +
k= 1 2 2
Pav = x(t ) dt = ak
T
1 0 k=
For rectangle pulse, ak
k 3
2
7th harmonic power = (2 a7)2 = ak
k= 3
5th harmonic power = (2 a5)2
Ratio = 3
2 2
2 = a0 +2 ak
2
a7 5 25 k =1
= = = 0.51
a5 7 49 = (2)2 + 2[(1)2 + (2)2 + (3)2]
= 4 + 28 = 32 Watts
15. (a)
20. (a)
ak – even symmetry
ak – odd symmetry ( can be – ) x(t ) Delay y(t ) = x(t – Td)
Td
x(t) is real.

16. (a) Y(f) = X ( f ) e j 2 fTd


Signal has odd and half wave symmetries.
Y( f ) j 2 fTd
So all an are zero and bn are zero for n even. H(f) = =e
X( f )
17. (b)
H( f ) = 1
Initially, T = 1 sec, so o = 2 rad/sec.
When x(t) is compressed by 3, frequency will H(f) = –2fT d
expand by same factor but there is no change in Given that, input is band limited to – f .
values of ak. Magnitude response of the system given,
So, both statement I and II are correct. |H(f )|

18. (c)
1
Change in only time period or frequency does
not change in the value of Fourier series f (Hz)
– 0
coefficients.
So, bk = ak Based on the magnitude response given,
statement (I) is correct.
bk = ak = 16 Note : Phase response of the system is not given
k= k= in the question without knowing phase
response of the system we can not comment
19. (32)
about exact nature of the system,
From II, a0 = 2
2
From III, a 1 = 1, a2 = 2, a3 = 3 (PSD)output = ( PSD)input H ( f )
From I, since x(t) is real and even ak = a–k SY(f) = SX(f ); – f
a 1 = a–1 = 1 Statement (II) is correct).
and a 2 = a–2 = 2 Hence option (a) is correct.
and a 3 = a–3 = 3
40 Electronics Engineering Signals & Systems

Answers
EE Fourier Series

1. (d) 2. (c) 3. (c) 4. (a) 5. (b) 6. (a) 7. (d) 8. (c)

9. (a) 10. (b) 11. (c) 12. (a) 13. (d) 14. (a) 15. (0.5) 16. (b)

17. (d) 18. (b) 19. (c) 21. (c)

Solutions
EE Fourier Series

1. (d) 5. (b)
1 Rms value of d.c. current = 10 A
Q Fundamental frequency = Hz
4 × 10 3 Rms value of alternating current

5 20
= = 10 2 A
Frequency of 5th harmonic = 3 2
4 × 10
= 1250 Hz Rms value of resultant current

= 102 + (10 2 )2 A
3. (c)
Q f(t) is an even function with half wave = 17.32 A
symmetry. 6. (a)
d.c. term as well as sine terms = 0
From the wave symmetry,
Only the cosine terms with odd harmonics will
1/2
be present. T /2 T
1 2t 2
Option (c) is correct. Vrms = dt + 0 dt
T T
0 T /2
4. (a)
T /2
1 /2 2 1 4t 2 4 T3 1
1
T Vrms = = =
Irms = i 2 (t ) T
0 T2 3T 3 8 6
T
0
1
T T
Vrms = V
2 6
1 12t
2
I rms = dt + 6 2 dt
T T
0 T /2 7. (d)
f(n) = sin 2x
T /2 2 T
1 144(t ) 36 For finding the Fourier series expansion,
= 2
dt + dt
T T T
0 T /2 2 1 cos 2 x
f(x) = sin x =
2
T /2
144 t 3 36 T = 0.5 – 0.5 cos2x
= +
T3 3 0
T 2
8. (c)
144 36 Since trigonometric Fourier series has no sine
= + = 24
8× 3 2 terms and has only cosine terms therefore this
will be an even signal i.e. it will satisfy,
Irms = 2 6 A
x(t) = x(–t)
GATE Previous Years Solved Paper 41

or we can write, 1
C1 = [2 e j /4 + 2 e j /4 ]
x(t – T) = x(–t + T) j2
But signal is periodic with period T.
2 j /4 2 j /4 2 1 j
Therefore, C1 = e = e =
j 2 2
x(t – T) = x(t)
Therefore, Comparing it with
x(t) = x(T – t) ...(i) a1 jb1
C1 =
Now since signal contains only odd harmonics 2 2
i.e. no terms of frequency, 4 4
a1 = , b1 =
2 ×2k 2 2
=
; k = 0, 1, 2, 3, 4
T Fundamental Fourier term will be
i.e. no even harmonics. = a1 cos 0t + b1 sin 0t
This means signal contains half wave symmetry 4 4
this implies that, = cos t + sin t
2 T 2 T
T
x(t) = x t ...(ii) 4
2 = cos t
T 4
From equations (i) add (ii),
T 10. (b)
x(t) = x(T t ) = x t
2 y(t) = x(t – t0) + x(t + x0)
9. (a) Since, x(t) is periodic with period T.
Therefore, x(t – t0) and x(t + t0) will also be
According to definition of signal given in
periodic with period T.
question the x(t) will be as,
jk 0t0
bk = ak e + ak e jk 0t0
x(t )
(By property)
1
ak is Fourier series coefficient of signal x(t).

Therefore, bk = ak [ e jk 0t0 + e jk 0t0 ]


t
–T/4 3T/4 7T/4
bk = 2 ak cosk 0t0
since, bk = 0 for odd k
–1

i.e., k 0t0 = k
So, it is periodic with period, 2
where, k = ±1, ±3, ±5....
7T T
T0 = = 2T
4 4
0t0 =
2
Fundamental angular frequency,
2 2 T T
= = t0 = ×
=
0 = 2 2 4
T0 2T T
Now, 11. (c)
C1 = (exponential series coefficient for k = 1)
j 2 0t
x(t) = a 2 e + a 1 e j 0t + a0
1 j 0t
C1 = x (t ) e dt
T0
T0
+ a1 e j 0t + a2 e j 2 0t

j 2 0t
1 3T /4 j 0t 7T /4 j 0t
= (2 j) e + (2 + j ) e j 2 0t
C1 = e dt + 1e dt
T0 T /4 3T /4
+ (0.5 + j 0.2) e j 0t + (0.5 j0.2) e j 0t + 2 j
42 Electronics Engineering Signals & Systems

j 2 0t
+ e j 2 0t ) + j [ e j 2 0t e j 2 0t ] Then function g(x) will be
= 2 (e
+ 0.5( e j 0t +e j 0t
) j 0.2 ( e j 0t e j 0t
)+2j g (x )

= 4 cos2 0t – 2 sin2 0t + cos 0t


1
+ 0.4 sin 0t + 2j
= Real [x(t)] + j Img[x(t)]
where, x
–1 0 1 2 3
Real [x(t)] = 4 cos2 0t – 2 sin2 0t + cos 0t
+ 0.4 sin 0t The value of the constant term (or) dc term in
= neither even nor odd signal the Fourier series expansion of g(x) is
Img [x(t)] = 2 = constant T
1
a0 = f ( x ) dx
12. (a) T
0
The given signal is odd as well as having half Area in one period
wave symmetry. =
one period
So it has only sine terms with odd harmonics.
1
So for second harmonic term amplitude = 0. = × 1 × 1 = 0.5
2
13. (d)
16. (b)
Let, x(t) = Even and Hws
x(t)

1 t
t
1

T/2 –T/2 0 T/2 T


0 T
Reconstruction of signal by its Fourier series
T/4 3T/4
coefficient is not possible at those points where
Fs expansion of x(t) contains cos terms with odd signal is discontinuous.
harmonics. In the above figure, at integer multiples of ‘T/2’,
f (t )
signal recovery is not possible by using its
coefficient.
Therefore, reconstruction of x(t) by using its
2 coefficient is possible at most of the points except
t
those instants where x(t) is discontinuous.
Now, f(t) = 1 + x(t) 17. (d)
Fs of f(t) contains dc and cos terms with odd
Sgn(x)
harmonics.

15. Sol.
Given function, x
g(x) = x – [x]
where [x] is a integer part of x.
GATE Previous Years Solved Paper 43

So, cos(t) is Since, x(t) is even-half wave symmetric. So, its


expansion will contain only odd harmonics of
cos(t)
cos. Therefore, coefficient of fundamental
1 harmonic is
T0 /2
t 2
a1 = x(t ) cos 0t dt
T
T0 /2
–1
Sgn cos(t)
T /2
1 4 0
= (t ) cos 0 t dt
T0
0
T0 /2
t 4
= (t ) cos 0 dt
T0
–1 0

T0 /2
4 4
So, sgn (cost) is a rectangular signal which is = (t ) dt = = 4000
T0 T0
even and has half wave symmetry. 0
So, Fourier series will have only cosine terms Now, frequency components available in
with odd harmonics only. expansion are:
0, 3 0,.....
18. (b)
2000 , 6000 , .....
Given that, f(–x) = –f(x) As, LTI system given in the question will pass
So, function in an odd function. upto 5000 rad/sec frequency component of
So the Fourier series will have sine term only so, input.
So, output will have only one component of
f(x) = bx sin( kx )
frequency 2000 rad/sec.
k=1
Thus, y(t) = expansion of output
19. (c) = a1 cos 0t
= 4000 cos2000 t
K
x(t) = ( 1)K t
k= 2000 20. Sol.

t t , t 0
K Given that, g(t) =
= ( 1) (t KT ) t t , otherwise
k=
where,
1
where, T= t = greatest integer less than or equal to ‘t’.
2000
x(t) t = smallest integer greater than or equal to ‘t’.
Now,
–T T 3T 1
0 2T t 0 t<0
1
g(t ) t t g(t ) t t
Time period: t–0 0, 1 0 t+0 –1, 0 0

1 t–1 1, 2 1 t+1 –2, –1 –1


T0 = 2T = t–2 2, 3 2 t+2 –3, –2 –2
1000
t–3 3, 4 3 t+3 –4, –3 –3
2
0 = = 2000 rad
T0
44 Electronics Engineering Signals & Systems

g(t) A A
= 2 2 sin 2 t dt = [1 cos 2t ] dt
2
1 2 3 0 0
– – –

t
–4 –3 –2 –1
t A sin 2t
0 4 3 2 1 = t
+
3
+
2
+
1 2 2 0
t
t

A A
Since, g(t) is non-periodic. So there is no Fourier = [( 0) (0 0)] =
2 2
series expansion of this signal and hence no
T
need to calculate harmonic here. 2 0
a1 = f (t ) cos 0 t dt
T0
0
21. (c)
2
2 2
T0 = 2 0 = =1 = f (t ) cos t dt
T0 2
0
T0
2 1
Now, b1 = f (t ) sin 0t dt = A sin t cos t dt
T0
0 0

2
2 A
= f (t ) sin t dt [Q = sin 2t dt = 0
2 0 = 1] 2
0 0

1
= A sin t sin dt
0
4 Fourier Transforms, Frequency
Response and Correlation
ELECTRO NICS EN GINEERIN G List-II

(GATE Previous Years Solved Papers) 1. Gaussian function


2. Rectangular pulse
Q.1 Specify the filter type if its voltage transfer 3. Triangular pulse
function H(s) is given by 4. Ramp function
2
K(s + 2 5. Zero
o)
H (s) =
[EC-1995 : 2 Marks]
s2 + o s + 2o
Q
Q.5 The Fourier transform of a real valued time
(a) all pass filter (b) low pass filter signal has
(c) band pass filter (d) notch filter (a) odd symmetry
[EC-1988 : 2 Marks] (b) even symmetry
(c) conjugate symmetry
Q.2 The magnitude and phase functions for a
distortionless filter should respectively be (d) no symmetry

(Magnitude) (Phase) [EC-1996 : 1 Mark]

(a) Linear Constant Q.6 The function f(t) has the Fourier transform g( ).
(b) Constant Constant
(c) Constant Linear The Fourier transform of g(t ) = g(t ) e j t
dt
(d) Linear Linear
[EC-1990 : 2 Marks] is
Q.3 If G(f ) represents the Fourier’s transform of a 1 1
(a) f( ) (b) f( )
signal g(t) which is real and odd symmetric in 2 2
time, then (c) 2 f (– ) (d) none of these
(a) G(f ) is complex. [EC-1997 : 1 Mark]
(b) G(f ) is imaginary.
Q.7 If the Fourier transform of a deterministic signal
(c) G(f ) is real.
g(t) is G(f ), then
(d) G(f ) is real and non-negative.
1. the Fourier transform of g(t – 2) is
[EC-1992 : 2 Marks]
2. The Fourier transform of g(t/2) is
Q.4 Match each of the items, A, B and C with an (a) G(f) e–j(4 f) (b) G(2f )
appropriate item from 1, 2, 3, 4 and 5. (c) 2G(2f) (d) G(f – 2)
List-I [EC-1997 : 2 Marks]
A. Fourier transform of a Gaussian function
Q.8 The amplitude spectrum of a Gaussian pulse is
B. Convolution of a rectangular pulse with
(a) uniform (b) a sine function
itself
(c) Gaussian (d) an impulse function
C. Current through an inductor for a step input
[EC-1998 : 1 Mark]
voltage
46 Electronics Engineering Signals & Systems

Q.9 The Fourier transform of a function x(t) is X(f ). Q.14 A linear phase channel with phase delay Tp and
The Fourier transform of dx(t)/dt will be group delay Tg must have
dX( f ) (a) Tp = Tg = constant
(a) (b) j2 fX(f)
dt (b) Tp f and Tg f
X( f ) (c) Tp = constant and Tg f
(c) jfX(f ) (d)
jf (d) Tp f and tg = constant (f denotes frequency)
[EC-1998 : 1 Mark] [EC-2002 : 1 Mark]

Q.10 The Fourier transform of a voltage signal x(t) is Q.15 The Fourier transform F{e–t u(t)} is equal to

X(f ). The unit of X( f ) is 1 1


. Therefore, F is
(a) Volt (b) Volt-sec 1 + j2 f 1 + j2 t
(c) Volt/sec (d) Volt2
(a) e f u(f ) (b) e –f u(f )
[EC-1998 : 1 Mark]
(c) e f u(–f ) (d) e –f u(–f )
Q.11 A signal x(t) has a Fourier transform X( ). If x(t) [EC-2002 : 1 Mark]
is a real and odd function of t, then X( ) is
Q.16 Let x(t) be the input to a linear, time-invariant
(a) a real and even function of .
system. The required output is 4x(t – 2). The
(b) an imaginary and odd function of .
transfer function of the system should be
(c) an imaginary and even function of .
(a) 4 e j4 f (b) 2e –j8 f
(d) a real and odd function of .
(c) 4 e –j4 f (d) 2e j8 f
[EC-1999 : 1 Mark]
[EC-2003 : 1 Mark]
Q.12 The input to a channel is a bandpass signal. It
is obtained by linearly modulating a sinusoidal Data given below for two questions. Solve the problem
carrier with a single-tone signal. The output of and choose the correct answer.
the channel due to this input is given by The system under consideration is an RC low-pass filter
1 (RC-LPF) with R = 1.0 k and C = 1.0 µF.
y (t ) = cos(100t 10 6 ) cos(106 t 1.56)
100 Q.17 Let H(f ) denote the frequency response of the
The group delay (tg) and the phase delay (tp) in RC-LPF. Let f1 be the highest frequency such
seconds, of the channel are
H( f1 )
(a) tg = 10–6, tp = 1.56 that, 0 f f1 ; 0.95. Then f1 (in Hz)
H (0)
(b) tg = 1.56, tp = 10–6
(c) tg = 10–8, tp = 1.56 × 10–6 is
(d) tg = 108, tp = 1.56 (a) 327.8 (b) 163.9
[EC-1999 : 1 Mark] (c) 52.2 (d) 104.4
[EC-2003 : 2 Marks]
3t 2
Q.13 The Fourier transform of the signal x(t ) = e
Q.18 Let tg(f) be the group delay function of the given
is of the following form, where A and B are RC-LPF and f2 = 100 Hz. Then tg(f2) (in ms), is
constants: (a) 0.717 (b) 7.17
Bf 2 Bf 2
(a) Ae (b) Ae (c) 71.7 (d) 4.505
2 [EC-2003 : 2 Marks]
(c) A + B f (d) Ae–Bf
[EC-2000 : 1 Mark]
GATE Previous Years Solved Paper 47

Q.19 The Fourier transform of a conjugate symmetric Q.23 The output y(t) of a linear time invariant system
function is always is related to its input x(t) by the following
(a) imaginary equation:
(b) conjugate antisymmetric y(t) = 0.5x(t – td + T) + x(t – td) + 0.5x(t – td – T)
(c) real The filter transfer function H( ) of such a system
(d) conjugate symmetric is given by
[EC-2004 : 1 Mark] j td
(a) (1 + cos T ) e
Q.20 A rectangular pulse train s(t) as shown in the (b) (1 + 0.5 cos T ) e j td
figure is convolved with the singal
cos2(4 × 103t). The convolved signal will be a (c) (1 cos T ) e j td

s (t ) (d) (1 0.5 cos T ) e j td


1 [EC-2005 : 2 Marks]

Q.24 Let x(t) X(j ) be Fourier transform pair. The


t Fourier transform of the signal x(5t – 3) in terms
0
0.1 msec of X(j ) is given as,
(a) DC (b) 12 kHz sinusoid j3 j3
1 5
j 1 5 j
(c) 8 kHz sinusoid (d) 14 kHz sinusoid (a) e X (b) e X
5 5 5 5
[EC-2004 : 2 Marks]
1 j3 j 1 j3 j
Q.21 Let x(t) and y(t) (with Fourier transforms X(f) (c) e X (d) e X
5 5 5 5
and Y(f) respectively) be related as shown in
[EC-2006 : 1 Mark]
the figure. Then Y(f) is
y( t)
Q.25 The 3-dB bandwidth of the low-pass signal
x (t ) e–t u(t), where u(t) is the unit step function, is
1
–2 –1 0 given by
t
1 1
(a) Hz (b) 2 1 Hz
2 2
(c) (d) 1 Hz
t –1
–2 0 2 [EC-2007 : 2 Marks]

1 f 1 f Q.26 The signal x(t) is described by


(a) X e j 2 f (b) X e j2 f
2 2 2 2 1 for 1 t +1
x( t ) =
f f j2 f 0 otherwise
(c) X e j2 f
(d) X e
2 2
Two of the angular frequencies at which its
[EC-2004 : 2 Marks] Fourier transform becomes zero are
Q.22 For a signal x(t) the Fourier transform is X(f). (a) ,2 (b) 0.5 , 1.5
Then the inverse Fourier transform of X(3f + 2) (c) 0, (d) 2 , 2.5
is given by [EC-2008 : 2 Marks]
1 t j3 t 1 t j 4 t /3 Statement for Linked Answer Questions (27 and 28):
(a) x e (b) x e
2 2 3 3
The impulse response h(t) of a linear time-invariant
(c) 3x(3t) e–j4 t (d) x(3t + 2) continuous time system is given by h(t) = exp(–2t) u(t),
[EC-2005 : 2 Marks] where u(t) denotes the unit step function.
48 Electronics Engineering Signals & Systems

Q.27 The frequency response H( ) of this system in Causal : The system is causal.
terms of angular frequency , is given by H( ) = LP : The system is low pass.
1 LTI : The system is linear and time-invariant.
sin( )
(a) (b) (a) Causal, LP
1 + j2
(b) BIBO, LTI
1 j
(c) (d) (c) BIBO, Causal LTI
2+ j 2+ j
(d) LP, LTI [EC-2009 : 2 Marks]
[EC-2008 : 2 Marks]
Q.31 The Fourier transform of a signal
Q.28 The output of this system, to the sinusoidal input
h(t) is H(j ) = (2 cos ) (sin2 )/
x(t) = 2 cos(2t) for all time t, is
The value of h(0) is
(a) 0
(b) 2–0.25 cos(2t – 0.125 ) 1 1
(a) (b)
4 2
(c) 2–0.5 cos(2t – 0.125 )
(c) 1 (d) 2
(d) 2–0.5 cos(2t – 0.25 )
[EC-2012 : 2 Marks]
[EC-2008 : 2 Marks]
Q.32 A continuous, linear time-invariant filter has
Q.29 A function is given by
an impulse response h(t) described by
f(t) = sin2t + cos2t
Which of the following is true? 3 for 0 t 3
h( t ) =
(a) f has frequency components at 0 and 0 otherwise
1/2 Hz. When a constant input of value 5 is applied to
(b) f has frequency components at 0 and 1/ Hz. this filter, the steady-state output is ______ .
(c) f has frequency components at 1/2 and [EC-2014 : 1 Mark]
1/ Hz.
Q.33 For a function g(t), it is given that,
(d) f has frequency components at 0, 1/2
+
and 1/ Hz. j t 2 2
g(t ) e dt = e
[EC-2009 : 1 Mark]

Q.30 Consider a system whose input and output y t


for any real value . If y(t ) = g(t ) d , then
are related by the equation,

+
y(t ) = x(t ) h(2 ) d
y(t ) dt _______ .

h (t ) (a) 0 (b) –j
j j
(c) (d)
2 2
t
0 [EC-2014 : 2 Marks]

Q.34 The value of the integral sin c 2 (5t ) dt is


where h(t) is shown in the graph.
Which of the following four properties are
possessed by the system? BIBO : Bounded input ________ .
gives a bounded output. [EC-2014 : 2 Marks]
GATE Previous Years Solved Paper 49

Q.35 The phase response of a passband waveform at 3


the receiver is given by (c) ak cos( k ot + k )
k=1
(f) = –2 (f – fc) – 2 fc
where fc is the center frequency, and and are 2
positive constants. The actual signal (d) ak cos( k ot + k )
k=1
propagation delay from the transmitter to
receiver is [EC-2016 : 2 Marks]

(a) (b) sin(t ) sin(t )


+ + Q.39 If the signal x(t ) = with
t t
(c) (d)
denoting the convolution operation, then x(t) is
[EC-2014 : 1 Mark]
equal to
Q.36 A real-valued signal x(t) limited to the frequency sin(t ) sin(2t )
(a) (b)
W t 2 t
band f is passed through a linear time
2 2
2 sin(t ) sin(t )
(c) (d)
invariant system whose frequency response is t t
W [EC-2016 : 1 Mark]
e j4 f , f
2
H( f ) = Q.40 A continuous time signal x(t) = 4 cos(200 t) +
W
0, f >
2 8 cos(400 t), where ‘t’ is in seconds, is the input
The output of the system is to a linear time invariant (LTI) filter with the
impulse response,
(a) x(t + 4) (b) x(t – 4)
(c) x(t + 2) (b) x(t – 2) 2 sin (300 t )
, t 0
[EC-2014 : 1 Mark] h (t ) = t
600 , t=0
sin(4 t ) Let y(t) be the output of this filter. The maximum
Q.37 The energy of the signal x(t ) = is ___ .
4 t
value of y(t ) is ________ .
[EC-2016 : 1 Mark]
[EC-2017 : 2 Marks]
Q.38 A network consisting of a finite number of linear
resistor (R), inductor (L), and capacitor (C) Q.41 Consider an LTI system with magnitude
elements, connected all in series of all in response,
parallel, is excited with a source of the form
f
3 1 , f 20
where, ak 0, 0 H( f ) = 20
ak cos( k ot ) o
k =1 0, f > 20
The source has non-zero impedance. Which one and phase response,
of the following is a possible form of the output arg {H(f )} = –2f
measured across a resistor in the network? If the input to the system is
3
x(t ) = 8 cos 20 t + + 16 sin 40 t + + 24 cos 80 t +
(a) bk cos( k ot + k ) where, bk ak, k 4 8 16
k=1
then the average power of the output signal y(t)
3 is _______ .
(b) bk cos( k ot + k ) where, bk 0, k
k=1
[EC-2017 : 2 Marks]
50 Electronics Engineering Signals & Systems

Q.42 The input 4 sinc(2t) is fed to a Hilbert ELECTRICAL EN GINEERIN G


transformer to obtain y(t), as shown in the figure
(GATE Previous Years Solved Papers)
below:
sin ( x ) 1
Here, sin c( x ) = Q.1 Let, x(t ) = rect t (where, rect(t) = 1 for
x 2
The value (accurate to two decimal places) of
1 1
x and zero otherwise). Then
2 2 2
y(t ) dt is ______ .
sin( x )
sin c( x ) = , the Fourier transform of
Hilbert y (t )
x
4 sinc(2t)
transform x(t) + x(–t) will be given by
[EC-2018 : 2 Marks]
(a) sin c
2
Q.43 X( ) is the Fourier transform of x(t) shown
(b) 2 sin c
2 2
below. The value of X( ) d (rounded off
(c) 2 sin c cos
2 2
to two decimal places) is _______ .

(d) sin c sin [EE-2008 : 2 Marks]


x (t ) 2 2
3
Q.2 A signal x(t) = sinc( t) where is a real constant
2
1 sin( x )
sin c( x ) = is the input to a linear time
t x
–3 –2 –1 0 1 2 3 4
invariant system, whose impulse response
[EC-2020 : 2 Marks] h(t) = sinc( t), where is a real constant. If min
( , ) denotes the minimum of and and
Q.44 Consider a real valued base band signal x(t),
similarly max( , ) denotes the maximum of
band limited to 10 kHz. The Nyquist rate for the
and , and K is a constant, which one of the
t following statements is true about the output of
signal, y(t ) = x(t ) x 1 + is
2 the system?
(a) It will be of the form K sinc ( t) where
(a) 20 kHz (b) 30 kHz
= min ( , ).
(c) 15 kHz (d) 60 kHz
(b) It will be of the K sinc( t) where = max( , ).
[EC-2021 : 1 Mark]
(c) It will be of the form K sinc( t).
Q.45 The Fourier transform X(j ) of the signal, (d) It cannot be a sinc type of signal.
[EE-2008 : 2 Marks]
t
x( t ) = is ________ .
(1 + t 2 )2 Q.3 x(t) is a positive rectangular pulse from t = –1 to
t = +1 with unit height. The value of
(a) e (b) e
2j 2 2
X( ) d {where X( ) is the Fourier
(c) e (d) e [EC-2022]
2j 2 transform of x(t)} is
GATE Previous Years Solved Paper 51

(a) 2 (b) 2 (c) g(t) would be proportional to f(t) only if f(t)


(c) 4 (d) 4 is a sinusoidal function.
[EE-2010 : 2 Marks] (d) g(t) would never be proportional to f(t).
[EE-2014 : 2 Marks]
Q.4 The Fourier transform of a signal h(t) is H(j ) =
(2 cos ) (sin2 )/ . The value of h(0) is Q.7 A function f(t) is shown in the figure.

1 1 f(t)
(a) (b)
4 2 1/2
(c) 1 (d) 2
[EE-2012 : 2 Marks] –T/2
t
–T/2 0
Q.5 A continuous time LTI system with system
function H( ) has the following pole-zero plot. –1/2
For this system, which of the alternatives is true?
The Fourier transform F( ) of f(t) is
(a) real and even function of .
2 (b) real and odd function of .
(c) imaginary and odd function of .
1 (d) imaginary and even function of .
(0, 0) [EE-2014 : 1 Mark]

Q.8 A signal is represented by


1, t <1
x( t ) =
0, t >1
(a) H (0) > H ( ) ; >0 The Fourier transform of the convolved signal
(b) H ( ) has multiple maxima, at 1 and 2 t
y(t ) = x(2t ) x is
2
(c) H (0) < H ( ) ; >0

(d) H ( ) = constant ; – < < 4


(a) 2
sin sin (2 )
2
[EE-2014 : 2 Marks]
4
Q.6 Let f (t) be a continuous time signal and let F( ) (b) 2
sin
2
be its Fourier transform defined by
4
(c) 2
sin (2 )
j t
F( ) = f (t ) e dt
4
(d) 2
sin 2
jut
and g(t) = F (u) e du [EE-2014 : 1 Mark]

Q.9 A differentiable non-constant real even function


What is the relationship between f(t) and g(t)?
x(t) has a derivative y(t), and their respective
(a) g(t) would always be proportional to f(t).
Fourier transforms are X( ) and Y( ). Which of
(b) g(t) would be proportional to f(t) if f(t) is an
the following statements is true?
even function.
52 Electronics Engineering Signals & Systems

(a) X( ) and Y( ) are both real. Which one of the following statements is true?
(b) X( ) is real and Y( ) is imaginary. (a) x1(t) and x2(t) are complex and x1(t), x2(t) is
(c) X( ) and Y( ) are both imaginary. also complex with non-zero imaginary part.
(d) X( ) is imaginary and Y( ) is real. (b) x1(t) and x2(t) are real and x1(t), x2(t) is also
[EE-2014 : 2 Marks] real.
(c) x1(t) and x2(t) are complex but x1(t), x2(t) is
Q.10 Consider a signal defined by
real.
e j 10t for t t (d) x1(t) and x2(t) are imaginary but x1(t), x2(t)
x(t ) =
0 for t > 1 is real.

Its Fourier transform is [EE-2016 : 2 Marks]

2 sin( 10) sin( 10) Q.12 Suppose the maximum frequency in a brand-
(a) (b) 2 e j 10
10 10 limited signal x(t) is 5 kHz. Then, the maximum
2 sin j 10 2 sin frequency in x(t) cos(2000 t), in kHz is _____ .
(c) (d) e
10 [EE-2016 : 1 Mark]
[EE-2015 : 2 Marks]

Q.11 Suppose x 1 (t) and x 2 (t) have the Fourier sin 2 t


Q.13 The value of the integral 2 dt is
transforms as shown below. t

X1(j ) equal to
(a) 0 (b) 0.5
1
(c) 1 (d) 2

0.5
[EE-2016 : 2 Marks]

0.3
Q.14 The Fourier transform of a continuous time
signal x(t) is given by
1
–1 0 1 2 X( ) = < <
(10 + j )2

X2(j ) where j = 1 and denotes frequency. Then

1 the value of ln x(t ) at t = 1 is ______ .


(upto one decimal place). (ln denoets the
0.5
logarithm to base e).
0.3 [EE-2018 : 2 Marks]

–2 –1 0 2
GATE Previous Years Solved Paper 53

Electronics & Electrical Engineering


GATE Previous Years Solved Paper

A n swe rs & Expl a n a t i o n s

Answers
EC Fourier Transforms, Frequency Response and Correlation

1. (d) 2. (c) 3. (b) 4. (A-1, B-3, C-4) 5. (c) 6. (c) 7. (Sol.)

8. (c) 9. (b) 10. (b) 11. (b) 12. (c) 13. (b) 14. (a) 15. (c)

16. (c) 17. (c) 18. (a) 19. (c) 20. (a) 21. (b) 22. (b) 23. (a)

24. (a) 25. (a) 26. (a) 27. (c) 28. (d) 29. (b) 30. (b) 31. (c)

32. (45) 33. (b) 34. (0.2) 35. (c) 36. (d) 37. (0.25) 38. (a) 39. (a)

40. (8) 41. (8) 42. (8) 43. (56.61) 44. (b) 45. (a)

Solutions Fourier Transforms, Frequency Response and Correlation


EC

1. (d) 2. (c)
For distortionless transmission,
K (s 2 + 2o )
Given that, H(s) = H ( f ) = constant
s2 + o s + 2o
Q and H(f ) = –kf where k is a constant
S=j So, the magnitude function of a distortionless
filter should be constant.
2
K (0 + o) The phase function of a distortionless filter
H(0) = 2
=K
0+0+ o should be linear.
H(0) K
3. (b)
2
Fourier transform of real and odd symmetric
Ks 2 1 + o
s signal is imaginary and odd function of
H( ) = 2 frequency.
1
s2 1+ o + 2o
Q s s 4. Sol.
(A) – 1,(B) – 3, (C) – 4
K (1 + 0)
H( ) = =K (A) Fourier transform of a Gaussian function is
1+ 0+ 0
H( ) 0 also a Gaussian function,
Here, the given transfer function is a notch filter. t2 F.T. f2
e e
54 Electronics Engineering Signals & Systems

(B) Convolution of a rectangular pulse with 9. (b)


itself is a triangular pulse.
F.T.
(C) Current through an inductor for a step input x( t ) X( f )
voltage is ramp function, d F.T.
x (t ) ( j 2 f ) X( f )
1 dt
i= V dt
L
1 10. (b)
i= u(t ) dt
L By definition Fourier transform,
1
i = r (t ) j 2 ft
L X(f ) = x(t ) e dt

5. (c)
x(t) Volts
Fourier transform of real valued signal has
So, X(f) Volt-sec
conjugate symmetry,
11. (b)
j t Example:
[X(j ) ] = x( t ) e dt
A sin ot = x(t)
(Real and Odd function)
j( )t
X (j ) = x(t ) e dt x( ) = Aj [ ( + o) – ( – o)]
(Imaginary any odd function of )
X (j ) = X(–j )
12. (c)
6. (c)
1 6
According to the duality property of Fourier y(t) = cos(100t 10 )
100
transform,
cos(106t – 1.56)
F.T.
f (t ) g( ) Comparing with,
F.T. y(t) = cos k(t – tg) cos[ c(t – tp)]
g( t ) 2 f( )
1
y(t) = cos[100(t 10 8 )]
7. Sol. 100
1-A, 2-C cos[106 (t – 1.56 × 10–6)]
F.T. tp = 1.56 × 10–6 sec
g( t ) G( f )
tg = 10–8 sec
F.T.
g(t 2) e j 2 2 f G( f ) = G( f ) e j (4 f )
13. (b)
t F.T. 1 f
g G = 2 G(2 f ) Normalized Gaussian function has Gaussian
2 1/2 1/2
Fourier transform.
8. (c) 14. (a)
A normalized Gaussian pulse is defined as, ( ) = – to
t2 ( )
x(t) = e = to
tp =
t2 F.T. f2
x(t) = e e = X( f )
d ( )
tg = = to
t2 d
So, X(f ) = e is also a Gaussian pulse in
tp = tg = to = constant
frequency domain.
GATE Previous Years Solved Paper 55

15. (c) 18. (a)


f(t) = e–t u(t) 1
H( ) =
1 1 + j RC
F(f ) =
1 + j2 f ( ) = –tan–1 RC
From duality,
10 3
F.T. tg = 6
F (t ) f( f) 1 + 10 × 4 2 × 10 4
1 = 0.717 ms
F(t) =
1 + j2 t
20. (a)
f(–f) = ef u(–f)
To = 0.1 × 10–3 = 10–4
F.T. f
F (t ) e u( f ) 1
fo = = 10 4 = 10 kHz
To
16. (c)
cos2(4 × 103t) has frequency = 4 kHz
y(t) = 4x(t – 2)
Y(s) = 4e–2s X(s)
Y (s )
= 4e –2s
X( s )
H(f ) = 4e–2xj 2 f = 4e–j4 f –4 kHz 0 4 kHz

17. (c)
R
+ +
0 10 kHz 30 kHz
Vi C Vo
So, only at ‘0’, we get output after convolution.
– – (Only odd harmonics are present).

1
H(f ) =
1 + j 2 fRC
H(0) = 1
H ( f1 ) 1
= 0.95 Constant in time domain.
H (0) 1+ 4 2 2 2 2
f1 R C
1 21. (b)
2 2 0.9025
1+ 4 f 1 ( RC )2 y(t) = –x[2(t + 1)]

x(t – to) j 2 fto


2 2 X( f ) e
1.108 1+ 4 f 1 ( RC )2
to = –1
2 2
0.108 4 f 1 ( RC )2
1 f
x (at) = X
0.108 a a
f 12 2
4 ( RC )2 a = –2
0.329 1 f
f1 Y(f ) = X e j2 f
3 2 2
2 × 10
f1 max = 52.2 Hz
56 Electronics Engineering Signals & Systems

22. (b) X( ) = 0

2 1 t j 4 t /3 ej e j
=0
X 3 f+ = x e
3 3 3 1
ej =0
Applying scaling and shifting property. ej
e2j – 1 = 0
23. (a)
e2j = 1
y(t) = 0.5x(t – td + T) + 0.5x(t – td – T) + x(t – td) ej = ±1
Taking Fourier transform, = ,2
j ( td + T )
Y( ) = 0.5 e + 0.5 e j ( td T ) + e j td X( )
27 (c)
Y( ) j td h(t) = exp(–2t) u(t)
= e 0.5 e j T + 0.5 e j T + 1
X( )
j t
Y( ) H( ) = h (t ) e dt
j td
= e [cos T + 1]
X( )
2t j t (2 + j )t
H( ) = (1 + cos T ) e j td = e e dt = e dt
0 0
24. (a)
1 (2 + j )t 1
x(t) X(j ) = e =
2+ j 2+ j
j3
3 1 j 5
x 5 t = X e 28. (d)
5 5 5
Input, x(t) = 2 cos(2t)
Using scaling and shifting property.
Frequency response,
25. (a) X( ) = 2 [ ( – 2) + ( + 2)]
1 1
Laplace transform of e t u(t ) = H( ) =
2+ j
s+1
1 Output,
Q magnitude at 3-dB frequency = Y( ) = H( ) X( )
2
1 1 1
= = 2 [ ( 2) + ( + 2)]
2 2 2+ j
1+
= 1 rad 2 2
= ( 2) + ( + 2)
2 + j2 2 j2
1
f= Hz
2
= [ ( 2) + ( + 2)] j [ ( 2) ( + 2)]
2 2
26. (a)
cos 2t sin 2t
y(t) = +
2 2
j t
X( ) = x( t ) e dt
2 1 1
= cos 2t + sin 2t
2 2 2
1 j t 1
j t e
= e dt = 1
j = cos(2t 0.25 )
1 1 2
j
e ej 1 j = 2–0.5 cos(2t – 0.25 )
= = = (e e j )
j j j
GATE Previous Years Solved Paper 57

29. (b) 32. Sol.


1 3 , for 0 t 3
(1 cos 2t ) + cos 2t
f(t) = h(t) =
2 0, otherwise
Frequency components are h(t) = 3[u(t) – u(t – 3)]
f1 = 0
3s
1 e
2 1
2 H(s) = 3
f2 = = = Hz s s
2 2
h(t )
30. (b)
3
y(t) = x(t ) h(2 ) d

t
It is not low pass filter. 0 3
But the system is LTI and BIBO.
Q x(t) = 5
31. (c) 5
Then, X(s) =
2 cos sin 2 s
= H(j ) ...(1) We know that,
3s
2 sin 2 1 e 5
H(j ) = 2 cos Y(s) = H (s) X(s ) = 3
2 s s s
= H1(j ) H2(j ) ...(2) Steady-state output
where, H1(j ) = 2 cos 3s
15 (1 e )
2 sin 2 = lim s Y (s ) = lim s
H2(j ) = = 2 Sa(2 ) s 0 s 0 s s
2
3s
h2(t) = 15 lim 3e [Q L-Hospital’s rule]
s 0

1/2 = 45
2 Sa[2 ]
33. (b)
t
–2 2 2
G( j ) = e 2
H1(j ) = 2 cos = + (ej
) e–j
So, H( j ) = 2 cos H2(j )
g(t ) dt = G(j0) = 0
= (ej + e–j ) H2(j )
H( j ) = ej H2( j ) + e–j H2(j ) y(t) = g(t) u(t)
h(t) = h2(t + 1) + h2(t – 1) Y(j ) = G(j ) U(j )
h( t) 1
Y(j ) = G( j ) + ( )
1 j

2 2
1/2 e 2 2
h(t) = = + e ( )
j
t
–3 –1 1 3
1
So, h(0) = 1 y(t ) dt = Y ( j 0) = = j
j
58 Electronics Engineering Signals & Systems

34. Sol. 37. Sol.

X( )
sin c 2 (5t ) dt
1
Let, f(t) = sinc(5t) sin at
= Sa (5 t) [Q sinc(t) = Sa( t)] t
–a a
1
F( ) = Rect
5 10 X( )

2 1/4
ESDf = F( )
sin 4 t 1 sin 4 t
= ×
F ( ) = G10 ( ) = F( ) 4 t 4 t
–4 4

1/5
1 2
Energy, Ex(t) = X( ) d
2
–5 5
4 2
1 1
1 = d
Ef = f 2 (t ) dt = ESD d 2
4
4
2
1 1 1
5 2 = × [8 ] = = 0.25 J
1 1 2 16 4
sin c 2 (5t ) dt = d
2 5
5
38. (a)
1 1 1
= × × 10 =
2 25 5 A sin ot LTI system B sin( ot + )

35. (c) When a sinusoidal input is given to LTI system,


Group delay is given by the output is also a sinusoidal with change in
magnitude and the phase shift offered by LTI
1 d (f) 1
= = [ 2 ] system.
2 df 2
Group delay = 39. (a)

36. (d) X1( )

j4 f 1
e , f
2 sin at
Given, H(f ) = x1 (t ) =
t
0, f > –a a
2
So, H( f ) = 1
X1( )

H( ) = e–j2 f 1
2
h(t) = (t – 2) sin t
x1 ( t ) =
As we know that, t
–1 1
y(t) = x(t) h(t)
or, y(t) = X( ) H( ) x(t) = x1(t) x1(t)
y(t) = x(t) (t – 2) X( ) = X1( ) X1( )
y(t) = x(t –2)
GATE Previous Years Solved Paper 59

X1( ) X1 ( ) CTFT 4 f f
4 sin c(2t ) rect = 2 rect
1 1 2 2 2

× 2
X( f ) df = 2 × (2)2 = 8
–1 1 –1 1

X( ) 2
So, y(t ) dt = 8
1
X(f )
=
2
–1 1

sin t f (Hz)
x(t) = –1 0 1
t

40. Sol. 43. Sol.

x(t) = 4 cos200 t + 8 cos400 t y (t )


H(f )
3
2
2 sin 300 t F.T. 2
h(t) = (rad/sec)
t
–300 0 300 1
So, y(t) = 8 cos200 t t
–3 –2 –1 0 1 2 3 4
y(t ) max = 8

2
41. Sol. X( ) d

H(f )
2 2
1 = 2 x(t ) dt = 2 y(t ) dt
Since,
0
2
f = 2×2 y(t ) dt
–20 0 20
2

1 1 0
So, y(t) = × 8 cos(20 t + ) = 2×2 (t + 2)2 dt + (2t + 3)2 dt
2
2 1
= 4 cos(20 t + ),
1 0
42 (t + 2)3 (2t + 3)3
So, Py = =8W = 4 +
2 3 3× 2
2 1
42. Sol.
1 0 33 1 1 26
Hilbert transform does not alter the amplitude = 4 + =4 +
3 6 3 6
spectrum of the signal.
1 26 1 13
2
y(t ) dt =
2
x(t ) dt =
2
X ( f ) df = 4 + =4 × ×
So, 3 6 3 3

x(t) = 4 sinc(2t) 14 56
= 4 × =
3 3
CTFT
sin c(t ) rect (f )
60 Electronics Engineering Signals & Systems

44. (b) Maximum frequency of y(t) is,


Given, x(t) band limited to 10 kHz, fm = 15 kHz
Nyquist rate = 2fm = 30 kHz
X( f )
45. (a)
t
Consider, x(t) = e
By taking Fourier transform,
f
–10 kHz 10 kHz t F.T. 2
e
1+ 2
t
Given, y(t) = x(t ) x 1 + Applying differentiation in frequency,
2
F.T. d
Assume, x1(t) = x(t) t x( t ) j X( )
d
t
and x2(t) = x 1 + F.T. d 2
2 te t j
d 1+ 2
y(t) = x1(t) x2(t)
Apply Fourier transform from convolution in t F.T. [0 2(2 )
te j 2 2
frequency domain property, (1 + )
Y(f ) = X1(f) X2(f )
t F.T. 4
X1(f ) = X(f ) te 2 2
(1 + )
t
X2(t) = x +1 Applying duality property,
2
4 jt F.T.
X2(f ) = 2ej4 f X(2f) 2 2
2 ( )e
(1 + t )
X1 ( f ) X2 ( f )
t F.T. 2 e
2 2 4j
(1 + t )
t F.T.
f f 2 2
e
–10 kHz 0 10 kHz –5 kHz 0 5 kHz (1 + t ) 2j

Y(f )

f
–15 kHz 0 15 kHz
(–10 kHz – 5 kHz) (10 kHz + 5 kHz)
GATE Previous Years Solved Paper 61

Answers
EE Fourier Transforms, Frequency Response and Correlation

1. (c) 2. (a) 3. (d) 4. (d) 5. (c) 6. (b) 7. (c) 8. (a)

9. (b) 10. (a) 11. (c) 12. (6) 13. (d) 14. (10)

Solutions Fourier Transforms, Frequency Response and Correlation


EE
1. (c)
x(t) = rect [t – 1/2]
1/
x(t) = 1 ; 0 t 1
= 0; otherwise H(j ) =

j t
F[x(t)] = x(t ) e dt
y(t) = h(t) x(t)
1
Y(j ) = X(j ) H(j )
j t 1 j
= 1e dt = (1 e )
j
0 1/

2 ej /2
ej /2
j t /2 Y(j ) = if, <
= xe
2j

sin /2 j /2
= e
/2
1/
F.T.
F[ x(t ) + x( t ) X( ) + X( ) or Y(j ) = if, <
sin /2 j /2
= [e + ej /2
] –
/2
1
sin /2 so, y(t) = Sa ( t) if, <
= 2 cos
/2 2
1
y(t) = sin c ( t ) if, <
= 2 sin c cos
2 2
1
2. (a) or y(t) = sin c ( t ) if, <
x(t) = sinc( t) = Sa[ t]
So, output is of the form k sinc( t)
h(t) = sinc( t) = Sa[ t]
where, = min( , )
x(t) = Sa[ t]
3. (d)
1/ Symmetrically located pole and zero
All pass filter
X(j ) =
(Constant magnitude)

(– )
h(t) = Sa[ t]
62 Electronics Engineering Signals & Systems

4. (d) 7. (c)
By using Parseval’s theorem, Given signal f(t) is an odd signal. Hence F( ) is
2 1 imaginary and odd function of .
X( ) d = 2 1 dt = 2 × 2
1
8. (a)
2
X( ) d =4 Given signal can be drawn as,
x(t)
5. (c)
2 sin 2 1
H(j ) = cos

t
ej + e j
2 sin 2 –1 1
=
2 2 For signal,
1 j 2 sin 2 j 2 sin 2
= e +e f (t )
2
1
2 sin 2
Let, X( ) =

t
1; 2<t<2 –T/2 T/2
Then, x(t) =
0 ; otherwise F( ) is given by
By time shifting property,
Gate width
1 2
h(t) = ( x (t + 1) + x(t 1))
2
T
1 1 F( ) = T Sa
h(0) = [ x(1) + x( 1) = [1 + 1] = 1 2
2 2
Area of gate
6. (b) pulse

Given that, f (t ) F( ) Therefore,


x(t) X( ) = 2 Sa( )
jut Now, x(t) X( )
and g(t) = F( u) e du
Then by time scaling,

1 1
F( ) e j t
d x( at ) X
Now, f(t) = a a
2

x(2t ) Sa ...(i)
2 f(t) = F( ) e j t
d 2
t
u, and x 4 Sa (2 ) ...(ii)
2

2 f(t) = F(u) e jut du t


Now, y(t) = x(2t ) x
2
= g(t) Convolution in time domain multiplication in
g(t) = 2 f(–t) = 2 f(t) frequency domain,
If f(t) is even then,
Y( ) = 4 Sa Sa (2 )
f(t) = f(–t) 2
g(t) f(t) if f(t) is even.
GATE Previous Years Solved Paper 63

By observing X1( ) and X2( ), we can say,


4 sin X2( ) = X1(– )
2 sin (2 )
Y( ) = IFT
2
2 x2(t) = x1(–t) ...(i)
Now, X1( ) is real. Therefore, x1(t) will be
4
Y( ) = 2
sin sin(2 ) conjugate symmetric.
2
i.e., x1(t) = x1 ( t )
9. (b)
t = –t
For real even function x(t) the Fourier transform
x1(–t) = x1 (t )
X( ) is always real even. y(t) is a derivative of
x(t) which is a real odd function because Now, x1(t) x2(t) = x1(t) x2(–t) [From (i)]
derivative of even function is an odd function = x 1 (t ) x 1 ( t )
and hence, Fourier transform Y( ) is imaginary 2
odd. = x1 (t ) = real function

10. (a) 12. Sol.


Maximum possible frequency of x(t) (2000 t)
e j 10t for t 1 = (f1 + f2)
Since, x(t) =
0 for t > 1 = 5 kHz + 1 kHz
= 6 kHz
j t
as, X( ) = x(t ) e dt
13. (d)
The Fourier transform of
1 1
so, X( ) = j 10t j t jt(10 )
e e dt = e dt 2 sin(t /2)
2 rect
1 1 t
1 sin(2 t )
= [ e j(10 )
e j(10 )
] rect
j(10 ) t 4

2 sin( 10) sin (2 t ) j t


X( ) = So, e dt = rect
( 10) t 4
Putting = 0 in above equation,
11. (c)
sin (2 t )
By observing X1(j ) and X2(j ) we can say that dt = 1
t
they are not conjugate symmetric. Since the
Fourier transform is not conjugate symmetric sin (2 )
the signal will not be real. So x1(t), x2(t) are not 2 dt = 2
t
real. Now the Fourier transform of x1(t) x2(t)

1 14. Sol.
will be X1 ( j ) X 2 ( j ) and by looking at
2 By taking inverse Fourier transform,
x(t) = te–10t u(t)
X1(j ) and X2(j ) we can say that X1(j ) X2(j )
Now, x(t)t = 1 = 1 × e–10 × 1 = e–10
will be conjugate symmetric and thus x1(t) × x2(t)
10
will be real. Thus, ln{ x(t )} = ln ( e ) = 10 = 10
5 Laplace Transform

ELECTRO NICS EN GINEERIN G (a) 2[1 – e–2t] u(t) (b) 4[e–t – e–2t] u(t)

(GATE Previous Years Solved Papers) (c) sin2t (d) (1 – 4e–4t) u(t)
[EC-1990 : 2 Marks]
Q.1 Laplace transform of the functions tu(t) and u(t)
sin(t) are respectively. Q.6 The pole-zero pattern of a certain filter is shown
in figure. The figure must be of the following
1 s 1 1
(a) 2
, 2 (b) , 2 type.
s s +1 s s +1
1 1 s j
(c) 2
, 2 (d) s , 2
s s +1 s +1 +j2
[EC-1987 : 2 Marks]
+j1
Q.2 The Laplace transform of a function f(t) u(t),
–1
where f(t) is periodic with period T, is A(s) times
the Laplace transform of its first period. Then, +1

(a) A(s) = s –j 1

1
(b) A(s) = –j 2
(1 exp( Ts ))
1 (a) low-pass (b) high-pass
(c) A(s) =
(1 + exp( Ts)) (c) all-pass (d) band-pass
(d) A(s) = exp (Ts) [EC-1991 : 2 Marks]
[EC-1988 : 2 Marks] Q.7 The voltage across an impedance in a network
Q.3 The transfer function of a zero-order hold is is V(s) = Z(s) I(s), where V(s), Z(s) and I(s) are the
Laplace transform of the corresponding time
1 exp( Ts) 1
(a) (b) functions v(t), z(t) and i(t). The voltage v(t) is
s s
(a) v(t) = z(t) i(t)
1
(c) 1 (d) t
[1 exp( Ts)]
(b) v(t ) = i( ) z(t )d
[EC-1988 : 2 Marks] 0
t
Q.4 The transfer function of a zero-order hold is
(c) v(t ) = i( ) z(t + ) d
1 exp( Ts ) 1 0
(a) (b)
s s (d) v(t) = z(t) + i(t)
1 [EC-1992 : 2 Marks]
(c) 1 (d)
[ exp( Ts)]
Q.8 The Laplace transform of te periodic function
[EC-1988 : 2 Marks] f(t) described by the curve below, i.e.,
Q.5 The response of an initially relaxed linear sin t if (2n 1) t 2n ( n = 1, 2, 3...)
f (t ) =
constant parameter network to a unit impulse 0 otherwise
applied at t = 0 is 4e–2 u(t). The response of this is ________ .
network to a unit step function will be
GATE Previous Years Solved Paper 65

f(t) (c) ratio of the Laplace transform of the output


and that of the input with all initial
conditions zeros.
2 3 4 5 6 7 8
t
0 (d) none of these
[EC-1995 : 1 Mark]
[EC-1993 : 2 Marks] Q.14 The final value theorem is used to find the
(a) steady state value of the system output.
K
Q.9 If F(s ) = L [ f (t ) = then, lim f (t ) (b) initial value of the system output.
( s + 1) ( s 2 + 4) t
(c) transient behaviour of the system output.
is given by (d) none of these
K [EC-1995 : 1 Mark]
(a) (b) zero
4
Q.15 The inverse Laplace transform of the function,
(c) infinite (d) undefined
[EC-1993 : 2 Marks] s+5
is
(s + 1) (s + 3)
Q.10 The Laplace transform of a unit ramp function
starting at t = a, is (a) 2e–t – e–3t (b) 2e–t + e–3t
as (c) e–t – 2e–3t (d) e–t + 2e–3t
1 e
(a) (b) [EC-1996 : 2 Marks]
( s + a )2 ( s + a )2
Q.16 The Laplace transform of e t cos( t) is equal to
e as a
(c) (d) (s + ) (s + )
s2 s2 (a) (b)
[EC-1994 : 1 Mark] (s 2
) + 2 (s + )2 + 2

Q.12 Indicate whether the following statement is 1


(c) (d) none of these
True/False. Give reason for your answer. If G(s) (s )2
[EC-1997 : 1 Mark]
1
is a stable transfer function, the F(s) = is
G( s) Q.17 Match each of the items 1, 2 on the left with
always a stable transfer function. most appropriate item A, B, C or D on the right.
In the case of a linear time invariant system
[EC-1994 : 1 Mark]
1. Poles in the right half plane implies.
(2 s + 1) 2. Impulse response zero for t 0 implies
Q.12 If L [ f (t )] = 2
, then f(0+) and f( ) are
s + 2s + 5 (a) Exponential decay of output

given by (b) System is causal.

(a) 0, 2 respectively (b) 2, 0 respectively (c) No stored energy in the system.

(c) 0, 1 respectively (d) 2/5, 0 respectively (d) System is unstable.

[Note : ‘L’ stands for ‘Laplace transform of’] [EC-1997 : 2 Marks]

[EC-1995 : 1 Mark]
Q.18 If L[ f (t )] = , then the value of
Q.13 The transfer function of a linear system is the (s2 + 2
)
(a) ratio of the output, vo(t) and input vi(t).
(b) ratio of the derivatives of the output and lim f (t )
t
the input.
66 Electronics Engineering Signals & Systems

(a) cannot be determined Q.23 The transfer function of a system is given by


(b) is zero 1
H (s) = 2
(c) is unity s (s 2)
(d) is infinite The impulse response of the system is
[EC-1998 : 1 Mark] (a) (t2 e–2t) u(t) (b) (t e–2t) u(t)

Q.19 The transfer function of a zero-order hold system (c) (te–2t) u(t) (d) (te–1t) u(t)
is ( denotes convolution, and u(t) is unit step
function)
1 sT 1 sT
(a) (1 + e ) (b) (1 e ) [EC-2001 : 1 Mark]
s s
Q.24 The Laplace transform of a continuous-time
1 1 sT
(c) 1 e sT (d) 1 + e
s s 5 s
signal x(t) is X (s ) = . If the Fourier
2
[EC-1998 : 1 Mark] s s 2
transform of this signal exists, then x(t) is
Q.20 If L [f(t) = F(s), then L [ f(t – T)] is equal to
(a) e2t u(t) – 2e–t u(t)
(a) esT F(s) (b) e–sT F(s)
(b) –e2t u(–t) + 2e–t u(t)
F(s ) F(s)
(c) (d) (c) –e2t u(–t) – 2e–t u(t)
1 + esT 1 e sT
(d) e2t u(–t) – 2e–t u(t)
[EC-1999 : 1 Mark]
[EC-2002 : 2 Marks]
Q.21 Given that,
Q.25 The Laplace transform of i(t) is given by
2
s+2 s +1 2
L [ f (t )] = 2
, L [ g(t )] = , I (s) =
s +1 (s + 3) (s + 2) s (1 + s )
t As t , the value of i(t) tends to
h(t ) = f ( ) g(t )d
(a) 0 (b) 1
0
(c) 2 (d)
L[h(t)] is
[EC-2003 : 1 Mark]
2
s +1
(a) Q.26 Consider the function f(t) having Laplace
s+3
transform:
1
(b) F(s ) = o , Re[s] > 0
s+3
s2 + 2
o
s2 + 1 s+2 The final value of f(t) would be
(c) +
(s + 3) (s + 2) s2 + 1 (a) 0 (b) 1
(d) None of the above (c) –1 f( ) 1 (d)
[EC-2000 : 1 Mark] [EC-2006 : 2 Marks]

Q.22 A linear time invariant system has an impulse Q.27 If the Laplace transform of a signal y(t) is
response e2t, for t > 0. If initial conditions are
1
zero and the input is e3t, the output for t > 0 is Y (s) = , then its final value is
s (s 1)
(a) e3t – e2t (b) e5t
(a) –1 (b) 0
(c) e3t + e2t (d) none of these
(c) 1 (d) unbounded
[EC-2000 : 2 Marks]
[EC-2007 : 1 Mark]
GATE Previous Years Solved Paper 67

Q.28 Given that, F(s) is the one-side Laplace transform Q.33 The unilateral Laplace transform of f(t) is

t 1
. The unilateral Laplace transform of
of f(t), the Laplace transform of f ( ) d is s2 + s + 1
0
t f(t) is
1 s 2s + 1
(a) sF(s) – f(0) (b) F (s ) (a) (b)
s 2
(s + s + 1) 2
(s + s + 1)2
2

s
1 s 2s + 1
(c) F( ) d (d) [ F( s ) f (0)] (c) (d)
s 2
(s + s + 1) 2 (s + s + 1)2
2
0
[EC-2009 : 2 Marks] [EC-2012 : 1 Mark]

Q.29 A continuous-time LTI system is described by Q.34 The impulse response of a system is h(t) = tu(t).
For an input u(t – 1), the output is
d 2 y(t ) dy(t ) dx(t )
+4 + 3 y(t ) = 2 + 4x(t )
dt 2 dt dt t2 t (t 1)
(a) u(t ) (b) u (t 1)
Assuming zero initial conditions, the response 2 2
y(t) of the above system for the input (t 1)2 t2 1
x(t) = e–2t u(t) is given by (c) u (t 1) (d) u (t 1)
2 2
(a) (et – e3t) u(t) (b) (e–t – e–3t) u(t) [EC-2013 : 1 Mark]
(c) (e–t + e–3t) u(t) (d) (et + e3t) u(t)
Q.35 Assuming zero initial condition, the response
[EC-2010 : 2 Marks]
y(t) of the system, given below to a unit step
Q.30 If the unit step response of a network is (1 – e– t), input u(t) is
then its unit impulse response is
U(s ) 1/s Y(s )
(a) e– t (b) –1 e– t

(c) (1 – –1) e– t (d) (1 – ) e– t


(a) u(t) (b) tu(t)
[EC-2011 : 1 Mark]
t2
(c) u(t ) (d) e–t u(t)
Q.31 An input x(t) = exp(–2t) u(t) + (t – 6) is applied 2
to an LTI system with impulse response [EC-2013 : 1 Mark]
h(t) = u(t). The output is
Q.36 Which one of the following statements is not
(a) [1 – exp(–2t)] u(t) + u(t + 6)
true for a continuous time causal and stable LTI
(b) [1 – exp(–2t)] u(t) + u(t – 6) system?
(c) 0.5[1 – exp(–2t)] u(t) + u(t + 6)
(a) All the poles of the system must lie on the
(d) 0.5[1 – exp(–2t)] u(t) + u(t – 6) left side of the j -axis.
[EC-2011 : 2 Marks] (b) Zero of the system can lie anywhere in the
s-plane.
2 (s + 1)
Q.32 If F(s ) = L [ f (t )] then the initial and (c) All the poles must lie within s = 1.
s 2 + 4s + 7
final values of f(t) are respectively (d) All the roots of the characteristic equation
must be located on the left side of the j -axis.
(a) 0, 2 (b) 2, 0
[EC-2013 : 1 Mark]
2 2
(c) 0, (d) ,0
7 7 Q.37 A system is described by the differential
[EC-2011 : 2 Marks] equation,
68 Electronics Engineering Signals & Systems

For the above system


d2 y dy
+5 + 6 y(t ) = x(t ) (a) only S1 and S2 are true.
dt 2 dt
(b) only S2 and S3 are true.
Let x(t) be a rectangular pulse given by
(c) only S1 and S3 are true.
1, 0<t<2
x (t ) = (d) S1, S2 and S3 are true.
0 , otherwise
[EC-2014 : 2 Marks]
dy
Assuming that, y(0) = 0 and = 0 at t = 0, the Q.41 The unilateral Laplace transform of f (t) is
dt
1
Laplace transform of y(t) is . Which one of the following is the
2
s +s+1
e 2s 1 e 2s
(a) (b) unilateral Laplace transform of g(t) = t f(t) ?
s (s + 2) (s + 3) s (s + 2) (s + 3)
s (2s + 1)
e 2s 1 e 2s (a) (b)
(c) (d)
2
(s + s + 1) 2 (s + s + 1)2
2
(s + 2) (s + 3) (s + 2) (s + 3)
s 2s + 1
[EC-2013 : 2 Marks] (c) (d)
(s 2 + s + 1)2 (s + s + 1)2
2

Q.38 A system is described by the following [EC-2014 : 2 Marks]


differential equation, where u(t) is the input to
the system and y(t) is the output of the system, Q.42 A stable linear time invariant (LTI) system has

y (t ) + 5 y ( t ) = u ( t ) 1
a transfer function H (s ) = . To make
2
s +s 6
When y(0) = 1 and u(t) is a unit step function,
y(t) is this system causal it needs to be cascaded with
(a) 0.2 + 0.8e–5t (b) 0.2 – 0.2e–5t another LTI system having a transfer function
(c) 0.8 + 0.2e–5t (d) 0.8 – 0.8e–5t H1(s). A correct choice for H1(s) among the
[EC-2014 : 2 Marks] following options is
(a) s + 3 (b) s – 2
Q.39 The input –3e–2t u(t), where u(t) is the unit step
(c) s – 6 (d) s + 1
function, is applied to a system with transfer
[EC-2014 : 2 Marks]
s 2
function . If the initial value of the output Q.43 A causal LTI system has zero initial conditions
s+3
and impulse response h(t). Its output x(t) and
is –2, then the value of the output at steady state input y(t) are related through the linear constant
is _______ .
coefficient differential equation,
[EC-2014 : 1 Mark]
d 2 y(t ) dy(t ) 2
Q.40 Let h(t) denote the impulse response of a causal 2
+ + y(t ) = x(t )
dt dt
1 Let another signal g(t) be defined as,
system with transfer function . Calculate
s+1 t
2 dh(t )
the following three statements: g( t ) = h( ) d + + h (t )
dt
S1 : The system is stable. 0

h (t + 1) If G(s) is the Laplace transform of g(t), then the


S2 : is independent of t for t > 0.
h(t ) number of poles of G(s) is ________ .
S3 : A non-causal system with the same transfer [EC-2014 : 2 Marks]
function is stable.
GATE Previous Years Solved Paper 69

Q.44 Input x(t) and output y(t) of an LTI system are Q.48 Let x(t) = s(t) + s(–t) with s(t) = e–4t u(t), where
related by the differential equation y (t) – y (t) – u(t) is unit step function. If the bilateral Laplace
6y(t) = x(t). If the system is neither causal nor transform of x(t) is
stable, the impulse response h(t) of the system is 16
X (s) = 2
4 < Re { s} < 4
1 3t 1 2t s 16
(a) e u( t) + e u( t )
5 5 Then the value of is _______ .
1 3t 1 2t [EC-2015 : 2 Marks]
(b) e u ( t) + e u( t )
5 5
Q.49 Consider the function g(t) = e–t sin(2 t) u(t)
1 3t 1 2t where u(t) is the unit step function. The area
(c) e u( t) e u(t )
5 5 under g(t) is _______ .
1 3t 1 2t [EC-2015 : 1 Mark]
(d) e u ( t) e u(t )
5 5
Q.50 The Laplace transform of the causal periodic
[EC-2015 : 2 Marks]
square wave of period T shown in the figure
Q.45 Consider the differential equation, below is

dx f (t )
= 10 0.2 x with initial conduction x(0) = 1.
dt
1
The response x(t) for t > 0 is
(a) 2 – e–0.2t (b) 2 – e0.2t
(c) 50 – 49e–0.2t (d) 50 – 49e0.2t
t
[EC-2015 : 2 Marks] 0 T/2 T 3T/2 2T

Q.46 The bilateral Laplace transform of a function, 1


(a) F(s ) = sT /2
1, if a t b 1+ e
f (t ) = is
0 , otherwise 1
(b) F(s ) =
s (1 + e sT /2 )
a b z
e ( a b)
(a) (b) 1
s s (c) F(s ) =
s (1 e sT /2 )
as bs ( a b)
e e e
(c) (d) 1
s s (d) F (s ) = sT
1 e
[EC-2015 : 1 Mark]
[EC-2016 : 2 Marks]
Q.47 Let the signal f(t) = 0 outside the interval [T1, T2]
Q.51 A first order low-pass filter of time constant T is
where T 1 and T 2 are finite. Furthermore,
excited with difference input signals (with zero
f (t ) < . The region of convergence (ROC) of initial conditions upto t = 0). Match the
the signal’s bilateral Laplace transform F(s) is excitation signals X, Y, Z with the
corresponding time responses for t 0:
(a) a parallel strip containing the j axis.
List-I List-II
(b) a parallel strip not containing the j axis.
X. Impulse P. 1 – e–t/T
(c) the entire s-plane.
Y. Unit step Q. t – T(1 – e–t/T)
(d) a half-plane containing the j axis.
Z. Ramp R. e–t/T
[EC-2015 : 1 Mark]
70 Electronics Engineering Signals & Systems

(a) X - R, Y - Q, Z - P
G(s )
(b) X - Q, Y - P, Z - R that is, Y (s ) = . The forced response of the
s
(c) X - R, Y - P, Z - Q
system is
(d) X - P, Y - R, Z - Q
(a) u(t)
[EC-2016 : 2 Marks]
(b) 2u(t)
(c) u(t) – 2e–t u(t) + e–3t u(t)
2
Q.52 A signal 2 cos t cos( t ) is the input to (d) 2u(t) – 2e–t u(t) + e–3t u(t)
3
[EC-2019 : 1 Mark]
an LTI system with the transfer function,
H(s) = es + e–s Q.56 A system with transfer function
If Ckdenote the kth coefficient in the exponential 1
G(s) = ,a>0
Fourier series of the output signal, then c3 is (s + 1) (s + a)
equal to is subjected an input 5 cos3t. The steady-state
(a) 0 (b) 1
1
(c) 2 (d) 3 output of the system is cos(3t 1.892).
10
[EC-2016 : 2 Marks]
The value of a is ________ .
Q.53 Consider the following statements for [EC-2020 : 2 Marks]
continuous-time linear time invariant (LTI)
systems: Q.57 Let x1(t) = e–t u(t) and x2(t) = u(t) – u(t – 2), where
I. There is no bounded input bounded output u( ) denotes the unit step function. If y(t) denotes
(BIBO) stable system with a pole in the right the convolution of x 1 (t) and x 2 (t), then
half of the complex plane. lim y(t ) = ___ (Rounded off to 1 decimal place).
t
II. There is no causal and BIBO stable system
with a pole in the right half of the complex [EC-2022]
plane.
Which one among the following is correct? ELECTRICAL EN GINEERIN G
(a) Both I and II are true. (GATE Previous Years Solved Papers)
(b) Both I and II are not true.
Q.1 The Laplace transforma.tion of f(t) is F(s).
(c) Only I is true.
(d) Only II is true. Given, F( s ) = , the final value of f(t) is
s2 + 2
[EC-2017 : 1 Mark]
(a) infinity (b) zero
Q.54 The transfer function of a causal LTI system is
(c) one (d) none of these
H(s) = 1/s. If the input to the system is
[EE-1995 : 1 Mark]
x(t) = [sin(t)/ t] u(t), where u(t) is a unit step
function, the system output y(t) as t is Q.2 The Laplace transform of (t2 – 2t) u(t – 1) is
________ .
2 s 2 s 2 2s 2 s
[EC-2017 : 2 Marks] (a) e e (b) 3
e e
s 3
s 2 s s2
Q.55 Let Y(s) be the unit-step response of a causal
2 s 1 s
system, having a transfer function, (c) e e (d) None of the above
s3 s
3 s
G(s) = [EE-1998 : 2 Marks]
(s + 1) (s + 3)
GATE Previous Years Solved Paper 71

Q.3 The output of a linear time invariant control 5


(a) 5 (b)
system is c(t) for a certain input r(t). If r(t) is 2
modified by passing it through a block whose 5
transfer function is e–s and then applied to the (c) (d) 0
3
system, the modified output of the system [EE-2004 : 1 Mark]
would be
Q.8 The Laplace transform of a function f(t) is
r(t ) r (t )
(a) (b) t 5s 2 + 23s + 6
1 + et 1+ e F( s ) =
s (s 2 + 2 s + 2)
(c) c(t – 1) (d) r(t) u(t – 1)
[EE-1998 : 1 Mark] As t , f(t) approaches
(a) 3 (b) 5
Q.4 A rectangular current pulse of duration T and
17
magnitude 1 has the Laplace transform (c) (d)
2
1 1 [EE-2005 : 2 Marks]
(a) (b) exp( Ts)
s s
Q.9 The integrator given by
1 1 t
(c) exp(Ts) (d) [1 exp( Ts)]
s s y(t ) = x( ) d
[EE-1999 : 2 Marks]
(a) has no finite singularities in its double sided
Q.5 Given the relationship between the input u(t) Laplace transform H(s).
and the output y(t) to be (b) produces a bounded output for every causal
t bounded input.
t( t )
y(t ) = (2 + t )e u( ) d
(c) produces a bounded output for every anti-
0
causal bounded input.
the transfer function Y(s)/U(s) is (d) has no finite zeroes in its double sided
2s Laplace transform H(s).
2e s+2
(a) (b)
s+3 (s + 3)2 [EE-2006 : 2 Marks]
2s + 5 2s + 7 Common Data Questions (10 and 11):
(c) (d)
s+3 (s + 3)2 Given: f(t) and g(t) as shown below.
[EE-2001 : 2 Marks] f (t ) g (t )

Q.6 Let Y(t) be the Laplace transform of the function 1 1


y(t), then the final value of the function is
(a) LimY (s) (b) Lim Y ( s)
s 0 s t t
0 1 0 3 5

(c) Lim sY (s) (d) Lim sY ( s)


s 0 s Q.10 g(t) can be expressed as:
[EE-2002 : 1 Mark] t
(a) g(t) = f(2t – 3) (b) g(t ) = f 3
2
5
Q.7 Consider the function, F( s ) = 3 t 3
2
s (s + 3s + 2) (c) g(t ) = f 2t (d) g(t ) = f
2 2 2
where F(s) is the Laplace transform of the [EE-2010 : 2 Marks]
function f(t). The initial value of (t) is equal to
72 Electronics Engineering Signals & Systems

Q.11 The Laplace transform of g(t) is Q.15 Assuming zero initial condition, the response
1 3s 1 y(t) of the system given below to a unit step input
(a) (e e 5s ) (b) (e 5s
e 3s
)
s s u(t) is
3s 1 5s
e
(c) (1 e 2s
) (d) (e e 3s ) U(s) 1/s Y(s )
s s
[EE-2010 : 2 Marks] (a) u(t) (b) t u(t)
Q.12 Let the Laplace transform of a function f(t) which t2
(c) u(t ) (d) e–t u(t)
exists for t > 0 be F1(s) and the Laplace transform 2
of its delayed version f(t – ) be F2(s). F1 (s) be [EE-2013 : 1 Mark]

the complex conjugate to F1(s) with the Laplace Q.16 Which one of the following statements is not
true for a continuous time causal and stable LTI
F (s) F1 (s) system ?
variable set as s = + j . If G(s) = 2 2
,
F1 (s) (a) All the poles of the system must lie on the
left side of the j -axis.
then the inverse Laplace transform of G(s) is (b) Zero of the system can lie anywhere in the
(a) an ideal impulse (t). s-plane.
(b) an ideal delayed impulse (t – ).
(c) All the poles must lie within s = 1.
(c) an ideal step function u(t).
(d) All the roots of the characteristic equation
(d) an ideal delayed step function u(t – ).
must be located on the left side of the j -axis.
[EE-2011 : 2 Marks]
[EE-2013 : 1 Mark]
Q.13 The unilateral Laplace transform of f(t) is
Q.17 Consider an LTI system with transfer function:
1 1
2
. The unilateral Laplace transform of H (s) =
s +s+1 s (s + 4)
t f(t) is If the inputs to the system is cos(3t) and the
s 2s + 1 steady-state output is A sin(3t + ), then the
(a) (b)
2
(s + s + 1) 2
(s + s + 1)2
2 value of A is
1 1
s 2s + 1 (a) (b)
(c) (d) 30 15
(s 2 + s + 1)2 (s 2 + s + 1)2
3 4
[EE-2012 : 1 Mark] (c) (d)
4 3
Q.14 Consider the differential equation: [EE-2014 : 2 Marks]

d 2 y(t ) dy(t )
2
+2 + y(t ) = (t ) with t
dt dt Q.18 The Laplace transform of f (t ) = 2 is s–3/2.
dy
y(t ) t = 0 = 2 and =0
dt t = 0 The Laplace transform of g(t ) = 1/ t is
dy 5/2
The numerical value of is 3s
dt t = 0+ (a) (b) s–1/2
2
(a) –2 (b) –1 (c) s1/2 (d) s3/2
(c) 0 (d) 1 [EE-2015 : 1 Mark]
[EE-2012 : 2 Marks]
GATE Previous Years Solved Paper 73

Q.19 The Laplace transform of f(t) = e2t sin(5t) u(t) is Q.24 The output response of a system is denoted as
5 5 y(t), and its Laplace transform is given by
(a) 2 (b) 2
s 4s + 29 s 5 10
Y (s) = 2
s 2 5 s(s + s + 100 2 )
(c) 2 (d)
s 4s + 29 s+5 The steady-state value of y(t) as,
[EE-2016 : 1 Mark]
1
(a) 100 2 (b)
10 2
Y (s) s
Q.20 The transfer function of system is = .
R(s ) s + 2 1
(c) 10 2 (d)
The steady-state output y(t) is A cos(2t + ) for 100 2
the input cos(2t). The value of A and , [EE-2019 : 1 Mark]
respectively are
Q.25 A system transfer function is
1 1
(a) , 45° (b) , + 45°
2 2 a1 s 2 + b1s + c1
H (s) =
(c) 2 , 45° (d) 2 , + 45° a2 s 2 + b2 s + c 2
[EE-2016 : 1 Mark] If a1 = b1 = 0, and all other coefficient are positive,
Q.21 Consider a causal LTI system characterized by the transfer function represents a
(a) high pass filter (b) low pass filter
dy(t ) 1
differential equation + y(t ) = 3x(t ). The (c) notch filter (d) band pass filter
dt 6
[EE-2019 : 1 Mark]
response of the system to the input
x(t) = 3e–t/3 u(t), where u(t) denotes the until step Q.26 The inverse Laplace transform of
function, is
s+3
(a) 9 e–t/3 u(t) H (s) = 2
for t 0 is
s + 2s + 1
(b) 9 e–t/6 u(t)
(c) 9 e–t/3 u(t) – 6 e–t/6 u(t) (a) 2t e–t + e–t (b) 3 e–t
(d) 54 e–t/6 u(t) – 54 e–t/3 u(t) (c) 3t e–t + e–t (d) 4t e–t + e–t
[EE-2016 : 1 Mark] [EE-2019 : 1 Mark]

Q.22 The solution of the differential equation, for t > Q.27 Consider a linear time-invariant system whose
0, y (t) + 2y (t) + y(t) = 0 with initial conditions input r(t) and output y(t) are related by the
y(0) = 0 and y (0) = 1, is (u(t) denotes the unit following differential equation,
step function).
d 2 y(t )
(a) t e–t u(t) (b) (e–t – te–t) u(t) + 4 y(t ) = 6 r (t )
dt 2
(c) (–e–t – te–t) u(t) (d) e–t u(t)
[EE-2016 : 1 Mark] The poles of this system are at
(a) +4j, –4j (b) +4, –4
Q.23 Consider a linear time-invariant system with
(c) +2, –2 (d) +2j, –2j
transfer function:
[EE-2020 : 1 Mark]
1
H (s ) =
(s + 1) Q.28 Which of the following statements is the about
If the input is cos(t) and the steady-state output the two sided Laplace transform?
is A cos(t + ), then the value of A is ______ . (a) It has no poles for any bounded signal that
[EE-2016 : 1 Mark] is non-zero only inside a finite time interval.
74 Electronics Engineering Signals & Systems

(b) If a signal can be expressed as a weighted Q.29 The causal realization of a system transfer
sum of shifted one sided exponential, then function H(s) having poles at (2, –1), (–2, 1) and
its Laplace transform will have no poles. zeroes at (2, 1), (–2, –1) will be
(c) It exists for every signal that may or may (a) unstable, complex, all pass
not have a Fourier transform. (b) unstable, real, high pass
(d) The number of finite poles and finite zeroes (c) stable, complex, low pass
must be equal. (d) stable, real, all pass
[EE-2020 : 1 Mark] [EE-2020 : 2 Marks]

Electronics & Electrical Engineering


GATE Previous Years Solved Paper

A n swe rs & Expl a n a t i o n s

Answers
EC Laplace Transform

1. (c) 2. (b) 3. (a) 4. (a) 5. (a) 6. (c) 7. (b) 8. (Sol.)

9. (d) 10. (c) 11. (Sol.) 12. (b) 13. (c) 14. (a) 15. (a) 16. (a)

17. (1-D, 2-B) 18. (a) 19. (b) 20. (b) 21. (b) 22. (a) 23. (b)

24. (c) 25. (c) 26. (c) 27. (d) 28. (d) 29. (b) 30. (a) 31. (d)

32. (b) 33. (d) 34. (c) 35. (b) 36. (c) 37. (b) 38. (a) 39. (0)

40. (a) 41. (d) 42. (b) 43. (1) 44. (b) 45. (c) 46. (c) 47. (c)

48. (–2) 49. (0.155) 50. (b) 51. (c) 52. (b) 53. (d) 54. (0.5) 55. (a, c)
56. (4) 57. (0)

Solutions
EC Laplace Transform

1. (c) 2. (b)
L.T. 1 The given function represents a causal periodic
tu(t )
s 2 signal,
f(t) u(t) = 0 ; t<0
L.T. a
u(t )sin at Period of f(t) = T ; for t > 0
s + a2
2
Let, f1(t) = f(t) u(t); 0 t T
L.T. 1 1
u(t )sin t = = 0; otherwise
s 2 + 12 s2 + 1
GATE Previous Years Solved Paper 75

f(t) u(t) 4. (a)


The impulse response h(t) of zero order hold
system is
h(t) = u(t) – u(t – T)
t
0 T 2T where, T is the sampling period,
1 1 Ts
f1(t) H(s) = e
s s
1 exp( Ts )
H(s) =
s
t
0 T 5. (a)
Given that, h(t) = 4e–2t
f(t) u(t) = f 1 (t nT ) 1
n=0 So, H(s) = 4
(s + 2)
Let, f1(t) F1(s)
Given that, x(t) = u(t)
nTs
f1(t – nT) e F1 (s ) 1
So, X(s) =
s
nTs
f(t) u(t) F( s ) = e F1 (s) We know that,
n=0
Y (s )
H(s) =
F1 (s ) X( s )
= Ts
1 e Y(s) = H(s) Y(s)
st
1 Transform of the 1 4 1 1 1
f(t) u(t) ×
Y(s) = × =2
1 e Ts period of f (t ) u(t )
(s + 2) s s s+2
1 Taking inverse Laplace transform of both side,
A(s) = Ts
1 e y(t) = 2[1 – e–2t] u(t)

3. (a) 6. (c)
A zero order hold system holds the input signal In the given pole-zero pattern, poles and zeros
value for a period of T, i.e. for an input of short are symmetrical about imaginary axis. This is
duration ( ) pulse, it produces an output pulse the case of all-pass filter.
of duration T, the sampling period.
7. (b)
T
s (t ) Zero order We know that, multiplication of two functions
hold in frequency (s) domain is equivalent to the
convolution in time domain.
Input, x(t) = (t)
t
So, X(s) = 1
So, v(t) = i( ) z(t )d
Output, y(t) = u(t) – u(t – T) 0
Ts Ts
1 e 1 e
So, Y(s) = = 8. Sol.
s s s
The given signal f(t) is a causal periodic signal
Transfer function,
with period,
Ts
Y (s) 1 e To = 2
H(s) = =
X( s ) s f1(t) = f(t) ; 0<t<2
= 0; otherwise
76 Electronics Engineering Signals & Systems

s
f(t) s 1+ e 1
F(s) = e 2
× 2 s
s +1 (1 e )
2 s
t s (1 + e )
0 F(s) = e 2 s s
( s + 1) (1 + e ) (1 e )
s
e
F(s) =
(s 2 + 1) (1 e s
)
sin(t ) u (t )

9. (d)
K
F(s) =
0 2 3 4
t (s + 1) ( s 2 + 4)
So, poles = –1; ±2j
Since all the poles are not in the left half of
s-plane. So, the final value theorem cannot be
sin(t – ) u (t – )
applied.

So, lim f (t ) is indeterminate.


t
t
0 2 3 4 5
10. (c)
L.T. 1
r (t )
s2
sin(t – 2 ) u(t – 2 )

L.T. 1 e as
r (t a ) e as × 2 = 2
s s
t
0 2 3 4 5 6 11. Sol.
If G(s) stable then all its poles must lie in the left
half of s-plane and there is no restriction on its
So, f1(t) = –[sin(t – ) u(t – ) + sin(t – 2 )
zeros, which can lie also in the right half of
1
sin(t ) u(t ) 2 1
s +1 s-plane. The inverse function F(s) = may
G( s)
e s
sin(t ) u(t ) or my not be stable. The zeros of G(s) may lie in
s2 + 1
the right half of s-plane.
e 2 s Hence the given statement is not true.
sin(t 2 ) u(t 2 )
s2 + 1
1
s 2 s
So, F(s) = is not always a stable transfer
e e G( s)
F1(s) = 2
+
s +1 s2 + 1 function.
s
s 1+e 12. (b)
F1(s) = e
s2 + 1 Initial value theorem,

F1 (s ) F1 (s ) 2s(s + 1)
F1(s) = = f(0+) = lim 2
1 e Tos
1 e 2 s s s + 2s + 5
GATE Previous Years Solved Paper 77

2s2 + 2s 18. (a)


f(0+) = lim 2
s s + 2s + 5
F(s) =
2 s2 + 2
s2 2 +
s f(t) = sin t
f(0+) = lim
s 2 2 5 lim f (t ) = lie between – 1 to +1
s 1+ + 2
s s t

2 So, it can not be determined.


2+
2+0 s
f(0+) = lim = =2
s 2 5 1+0+0 19. (b)
1+ + 2
s s
The impulse response h(t) of zero-order hold
Final value theorem,
system is given by
s 2 (s + 1) h(t) = u(t) – u(t – T)
f( ) = lim sF(s ) = lim
s s 0 s2 + 2s + 5 where T is the sampling period,
f( ) = 0
1 1 sT 1 sT
H(s) = e = [1 e ]
13. (c) s s s

Y (s ) 20. (b)
H(s) =
X( s ) f(t – T) = F(s) e–sT
(Time shifting property of Laplace transform)
14. (a)
Final value theorem is used to find the final value 21. (b)
of the steady-state value of the system output, Convolution in time domain is multiplication
f( ) = lim sF(s) in s-domain.
s 0
1
L[h(t)] = L[ f (t )] × L[ g(t )] =
15. (a) s+3

(s + 5) 22. (a)
F(s) =
(s + 1) (s + 3) h(t) = e2t
2 1 1
F(s) = H(s) =
s+1 s+3 s+2
Taking inverse Laplace transform on both side, x(t) = e3t
f(t) = 2e–t – e–3t
1
X(s) =
16. (a) s 3

s Output = x(t) h(t)


L.T.
cos( t ) u(t )
s2 + 2 1
= X(s) H (s) =
(s )
(s 2) (s 3)
t L.T.
e cos( t ) u(t ) 2 2
(s + ) + 1 A B
= +
(s 2) (s 3) s 2 s 3
17. Sol.
A = –1
1-D, 2-B
B=1
(i) Poles lie in the right half of s-plane Output = [e3t – e2t]
unstable system.
(ii) h(t) = 0 for t 0 causal system.
78 Electronics Engineering Signals & Systems

23. (b) 28. (d)


Impulse response of system L–1[H(s)] t
1
1 1 1 L f( )d = [F(s) f (0)]
+2 t s
= × = (t e ) u(t ) 0
s 2 (s 2) s2 s 2
29. (b)
24. (c)
d 2 y(t ) 4 dy(t ) 2 dx(t )
5 s 5 s + + 3 y(t ) = + 4 x(t )
X(s) = = dt 2 dt dt
s2 s 2 (s + 1) (s 2)
Taking Laplace transform on both sides
5 s A B
= + (assuming zero initial conditions),
(s + 1) (s 2) s+1 s 2
s2Y(s) + 4sY(s) + 3Y(s) = 2sX(s) + 4X(s)
B=1 Y (s ) 2s + 4 2(s + 2)
A = –2 or, = 2 =
X( s ) s + 4s + 3 (s + 1) (s + 3)
x(t) = –2e–t u(t) – e2t u(–t)
Given that, x(t) = e–2t u(t)
ROC : –1 < < 2
1
Note : ROC of Laplace transform should include X(s) =
s+2
= 0 line.
2(s + 2)
Y(s) =
25. (c) (s + 1) (s + 3) (s + 2)
lim i(t ) = lim sI (s) 2 1 1
t s 0 = =
(s + 1) (s + 3) s + 1 s + 3
2 Taking inverse Laplace transform on both sides,
= lim s =2
s 0 s (1 + s ) y(t) = (e–t – e–3t) u(t)

26. (c) 30. (a)

f (t )
Unit step response,
s(t) = (1 – e– t)
So, unit impulse response is
t ds(t ) d t t
h(t) = = (1 e )= e
dt dt

31. (d)
L–1[F(s)] = sin ot
x(t) = e–2t u(t) + (t – 6)
f(t) = sin ot
–1 f( ) 1 1 6s
X(s) = +e
s+2
27. (d) 1
Final value theorem is applicable only when all H(s) =
s
the poles of system lies in left half of s-plane. Y(s) = X(s) H(s)
Q s = 1 is right s-plane pole. Therefore, final
1 1
value theorem cannot be applied, Y(s) = + e 6s
s (s + 2) s
1 1
Y(s) = 1 1 1 1 6s
s 1 s Y(s) = + e
2 s s+2 s
y(t) = (et – 1) u(t)
Taking inverse Laplace transform, we have
Q Unbounded.
y(t) = 0.5(1 – e–2t) u(t) + u(t – 6)
GATE Previous Years Solved Paper 79

32. (b) Taking Laplace transform


s
2 (s + 1) e
F(s) = L[ f (t )] = 2 X(s) =
s + 4s + 7 s
Initial value, Y (s )
= H(s)
2(s + 1) X( s )
lim f (t ) = lim sF(s) = lim s
t 0 s s 2
s + 4s + 7 Y(s) = H(s) X(s)
1 e s e s
1 2 Y(s) = = 3
2s 1 + s2 s s
s
= lim
s 4 7 Taking the inverse Laplace transform,
s2 1 + + 2
s s (t 1)2
y(t) = u (t 1)
2 (1 + 0) 2
= =2
(1 + 0 + 0) 35. (b)
Final value, x(t) = u(t)
2(s + 1) Apply Laplace transform,
lim f (t ) = lim sF(s) = lim s 2
t s 0 s 0 s + 4s + 7 1
X(s) =
s
33. (d)
1
If, L[f(t)] = f (s) H(s) = (given)
s
dn Y(s) = H(s) X(s)
then, L[tn f(t)] = ( 1)n f (s )
dsn 1 1 1
× ==
In this problem: s s s2

1 Taking inverse Laplace transform


Given, L[f(t)] = = f (s) y(t) = tu(t)
s2 + s + 1
We need, 36. (c)
1
d For a causal and stable LTI system the ROC
L[tf(t)] = ( 1)n 1
f (s)
ds must be right sided and it must include the s = j
line ( = 0). For all the poles must lie within
d d 1
= f (s ) =
ds ds s 2 + s + 1 s = 1 then all poles will lie between –1 to +1.
We know that ROC does not include any pole,
1
= × (2s + 1) so in this case ROC will not be able to include
(s2 + s + 1)2
s = j line ( = 0). Thus the system will not be
2s + 1 stable. Hence this statement is not true.
=
(s + s + 1)2
2
37. (b)
x(t) = u(t) – u(t – 2)
34. (c)
h(t) = tu(t) 2s 2s
1 e 1 e
X(s) = =
Taking Laplace transform, s s s
1 Given that,
H(s) =
s2
d2 y dy
x(t) = u(t – 1) +5 + 6 y(t ) = x(t )
2 dt
dt
80 Electronics Engineering Signals & Systems

Taking Laplace transform on both the sides Y(s) = X(s) H(s)


dy 3(s 2)
y(0) = 0 and = 0 at t = 0 Y(s) =
dt (s + 3) (s + 2)
s2Y(s) + 5sY(s) + 6Y(s) = X(s) Using final value theorem, the steady-state value
2s of Y(s) is
1 e
Y(s) [s2 + 5s + 6] = y ( ) = lim sY (s) = 0
s
s 0
2s
1 e
Y(s) = 40. (a)
s (s 2 + 5s + 6)
1
1 e 2s Given, H(s) =
= (s + 1)
s (s + 2) (s + 3)
h(t) = e–t u(t)
At t = , h(t) = 0
38. (a)
h(t)
y(t ) + 5 y(t ) = u(t)
Taking the Laplace transform of the above
equation,
1 t
sY(s) – y(0) + 5Y(s) =
s
i.e. the system is stable as it converges to zero as
1 t .
(s + 5) Y(s) – 1 =
s • The system is causal as it can be seen from
s+1 the graph that, h(t) = 0 for t < 0.
(s + 5) Y(s) =
s
h (t + 1) e (t + 1) e t e 1
• = =
(s + 1) h( t ) e t e t
Y(s) =
s(s + 5)
= e–1 i.e. independent of t.
A B (s + 1) So, statements S1 and S2 are true.
+ =
s (s + 5) s(s + 5)
41. (d)
A = 0.2
1
4 Given, F(s) =
B= = 0.8 s +s+1 2
5
Laplace transform of g(t) = t f(t)
0.2 0.8
Y(s) = + dF(s ) d 1
s (s + 5) = =
ds ds s 2 + s + 1
y(t) = 0.2 + 0.8e–5t
(2s + 1)
=
39. Sol. (s + s + 1)2
2

Given, x(t) = –3e–2t


Laplace transform of 42. (b)
1
x( t ) X(s) H(s) = 2
(s + s 6)
3
where, X(s) = The system is said to be causal if the output at
s+2
any time depends only on present and/or past
s 2
Also, H(s) = values of input or h(t) = 0 for t > 0 i.e. all poles
(s + 3)
must lie on LHS of s-plane.
GATE Previous Years Solved Paper 81

1 45. (c)
H(s) =
(s + 3) (s 2) Applying Laplace transform,
From the above expression it can be seen that 10
sX(s) – x(0) = 0.2 X(s)
s = 2 is a pole which lies on RHS of s-plane. So, s
H1(s) system that needs to be cascaded should 10 1
have one zero at 2. X(s) = +
s (s + 0.2) (s + 0.2)
1 Applying inverse Laplace transform, we get
Thus, H(s) H1(s) = (s 2)
(s + 3) (s 2) x = 50 – 49 e(–0.2)t
1
= causal system 46. (c)
(s + 3)
1, if a t b
43. Sol. f(t) = = u(t – a) –u(t – b)
0 , otherwise
d 2 y(t ) dy 2 e as
e bs
+ + y ( t ) = x (t )
dt 2 dt F(s) =
s
(s2 + s + 2) Y(s) = X(s)

Y (s) 1 47. (c)


H(s) = = 2 2
X (s) (s + s + ) ROC of a finite duration signal is entire s-plane.
Let another signal g(t) be defined as, 48. Sol.
2
G(s) = H (s ) + sH (s ) + H (s ) x(t) = e–4t u(t) + e4t u(–t)
s
16
2 Given, X(s) = 2 ; –4 < Re[s] < 4
s 16
G(s) = +s+ H (s )
s 16 2 2
= = +
2 (s + 4) (s 4) s + 4 s 4
+ s2 + s 1
= 2 2 x(t) = –2e–4t u(t) – 2e4t u(–t)
s (s + s + )
On comparison,
(s 2 + s + 2
) 1 = –2
= =
s(s 2 + s + 2
) s
So, number of poles of G(s) is 1. 49. Sol.
g(t) = e–t sin (2 t) u(t)
44. (b)
y (t) – y (t) – 6y(t) = x(t) Area = g(t ) dt
Y (s )
H(s) =
X( s )
1 1 = sin(2 t ) u(t ) e t dt = G(s) s = 0
= =
s2 s 6 (s 3) (s + 2)
1 1 2
G(s) =
5 5 (s + 1)2 + 4 2
H(s) = +
s 3 s+2
If the system is neither causal not stable, then, 2
Area = = 0.155
1 3t 1 (s + 1)2 + 4 2 s = 0
2t
h(t) = e u( t ) + e u( t )
5 5
ROC : < –2
82 Electronics Engineering Signals & Systems

50. (b) 52. (b)


T /2 Given, H(s) = es + e–s
1 st
L [f(t)] = sT
e dt H(ej ) = ej + e–j = 2 cos
1 e 0
j
x(t ) H(e ) y(t )
T /2
st
1 e
= sT 2
1 e s If, x1(t) = 2 cos t
0
3
1
= sT
(1 e sT /2 ) 2
s (1 e ) o =
3
1 1 e sT /2 2 1
= H (j o) = 2 cos =2 = 1
s (1 e sT /2 ) (1 + e sT /2 ) 3 2
1 1
= 2
s 1 + e sT /2 y1(t) = 2 cos t + 180°
3
51. (c) If, x2(t) = cos t
For 1st order system, o =
j
H (e ) = 2 cos( ) = –2
o
1
G(s) = ; H (s) = 1 y2(t) = 2 cos( t + 180°)
sT
Impulse response, 2
y(t) = 2 cos t+ 2 cos( t + )
R(s) = 1 3

G(s ) 2
Y(s) = R(s ) = , 2 =
1 + G(s) H (s) 1 3
T1 = 3, T2 = 2
1 1
= = e t /T for t 0 T0 = 6
1 + sT T
2
Unit step response, o = =
T0 3
1
R(s) = y(t) = 2 cos(2 t + ) – 2 cos(3
s ot + )
e j(2 ot + ) +e j(2 ot + )
e j(3 ot + ) j(3 ot + )
1 (1 + sT ) ( sT ) y(t) = e
Y(s) = =
s (1 + sT ) s (1 + sT ) y(t) = e
j(2 ot ) e j(2 ot )
+ e j(3 ot ) +e j(3 ot )

1 T 1 T C3 = 1
= =
s (1 + sT ) s 1
T s+ 53. (d)
T
• A BIBO stable system can have poles in right
y(t) = 1 – e–t/T for t 0
half of complex plane, if it is a non-causal
Ramp response,
system. So, statement-I is wrong.
1
R(s) = • A causal and BIBO stable system should
s2
have all poles in the left half of complex
1 1 T T plane. So, statement-II is correct.
Y(s) = = +
s 2 (1 + sT ) s2 s s+ 1
• So, option (d) is correct.
T
y(t) = t – T(1 – e–t/T) for t 0
GATE Previous Years Solved Paper 83

54. Sol. 56. Sol.


1 Given that,
H(s) = Integrator
s 1
G(j ) =
sin t (1 + j ) ( + j )
x(t) = u(t )
t 1
G( j ) =
2
t ( + 1) ( 2 + 2 )
sin
So, y(t) = d
According to question,
0
1
G( j ) =3 =
1 sin 5 10
y(t)t = d
0 1 1
=
( 2
+ 1) ( 2
+ 2
) 5 10
sin
d =
2 1 1
0 =
10( a + 9) 2 5 10
1 1
So, y(t)t = × = = 0.5 2 + 9 = 25
2 2
2 = 16

55. (a, c) =4
3 s
Given, G(s) = 57. (0)
(s + 1) (s + 3)
Given that, x1(t) = e–2t u(t)
G(s) 3 s x2(t) = u(t) – u(t – 2)
Y(s) = =
s s(s + 1) (s + 3) y(t) = x1(t) x2(t)
Using partial fractions, we get, By applying Laplace transform,
Y(s) = X1(s) X2(s)
A B C
Y(s) = + + 2s
s (s + 1) ( s + 3) 1 (1 e )
=
A = 1, B = –2 and C = 1 (s + 1) s
1 2 1 By applying final value theorem,
So, Y(s) = +
s (s + 1) (s + 3)
y(t ) t = 1 e 2s
= lim sY (s) = lim =0
and y(t) = u(t) – 2e–t u(t) + e–3t u(t) s 0 s 0 s+1
Forced response,
yf (t) = u(t) option (a)
Some authors also consider the zero state
response (ZSR) as the forced response and
according to them the correct answer will be
option (c).
84 Electronics Engineering Signals & Systems

Answers
EE Laplace Transform

1. (d) 2. (c) 3. (c) 4. (d) 5. (d) 6. (c) 7. (d) 8. (a)

9. (d) 10. (d) 11. (c) 12. (b) 13. (d) 14. (d) 15. (b) 16. (c)

17. (b) 18. (b) 19. (a) 20. (b) 21. (d) 22. (a) 23. (0.707) 24. (b)

25. (b) 26. (a) 27. (d) 28. (a) 29. (a)

Solutions
EE Laplace Transform

1. (d) 2 1
H(s) = +
s + 3 (s + 3)2
Q F(s) =
s2 + 2 2s + 7
T.F. = H(s) =
f(t) = sin( t) u(t) (s + 3)2
which is a periodic signal so final value theorem
6. (c)
is not applicable on it. Final value of f(t) may be
any value between-1 and +1. Final value = lim sY (s)
s 0

2. (c)
7. (d)
L (t2 – 2t) u(t – 1) = L [(t – 1)2 u(t – 1) – u(t – 1)]
5
F(s) =
2e s e s 2
s (s + 3s + 2)
=
s3 s
By initial value theorem,
3. (c) lim f (t ) = lim sF(t )
t 0 s

r (t )
LTI system
c(t ) 5×s
= lim 2
=0
s s (s + 3s + 2)
Unit delay block LTI system
r (t ) r(t 1) C (t 1)
e- s
8. (a)
4. (d) 5s 2 + 23s + 6
F(s) =
f( t) s(s 2 + 2s + 2)
f(t) = u(t) – u(t – T) A F(s) has only left hand poles.
By final value theorem,
6
lim F(t ) = lim sF(s) = =3
t t 0 s 0 2
T
Ts Ts
1 e 1 e 10. (d)
L [u(t) – u(t – T)] = =
s s s
f( t) g (t )
5. (d)
1
1
t
3(t )
y(t) = (2 + t )e u( ) d
0 t t
1 3 5
y(t) = [(2 + t) e–3t] u(t)
GATE Previous Years Solved Paper 85

Since, g(t) has width of 2 unit and f(t) has 1 unit 14. (d)
therefore we have to first expand f(t) by 2 unit
and for this we have to a scale f(t) by 1/2. d 2 y(t ) dy(t )
2
+2 + y (t ) = ( t )
dt dt
1
i.e., f1(t) = f t
2 s2Y(s) – sy(0) – y (0) + 2[sY(s) – y(0)]+Y(s) = 1
Now shift it by three unit to get g(t), Y(s) [s2 + 2s + 1] – (s × –2) –y (0)–(2 × –2) = 1
g(t) = f1(t – 3) Y(s) [s2 + 2s + 1] = –2s – 3
t 3 t 3 2s 3 2 1
g(t) = f =f Y(s) = =
2 2 2 s2 + 2s + 1 s + 1 (s + 1)2
y(t) = –2e–t – te–t
11. (c)
dy (t )
1, 3<t<5 = 2e–t – (–te–t + e–t) = e–t + te–t
g(t) = dt
0 , t < 3, t > 5
dy(t )
g(t) = u(t – 3) – u(t – 5) = 1+0=1
dt t = 0 +
L{g(t)} = L{u(t – 3)} – L{u(t – 5)}
1 3s 1 5s
= e e 15. (b)
s s
1 1 1
= (e 3s
e 5s
) Y(s) = U (s) = 2
s s s
3s y(t) = t u(t)
e 2s
= (1 e )
s 16. (c)
12. (b) All poles must lie within Z = 1.
F2(t) = L{f(t – )} = e–s F1(s)
s
17. (b)
e F1 ( s ) F1 (s ) s
G(s) = 2
=e 1
F1 (s ) Given, H(s) =
s ( s + 4)
G(t) = (t – )
r(t) = input = cos(3t) = cos t
13. (d) = 3 rad/sec
If, L[f(t)] = F(s) 1
H(j ) =
d n ( j ) ( j + 4)
then, L[tn f(t)] = ( 1)n F( s )
dsn 1 1 1
Now, H( j ) = = =
In this problem: 2
+4 2 3 25 15
1 (at = 3)
d d
Given, L[f(t)] = ( 1)1 1
F(s) = F(s )
ds ds 1
H(j ) = 90° tan
d 1 4
= 2
ds s + s + 1 1 3
= 90° tan
4
1
= × (2s + 1) = –126.86° (at = 3)
(s + s + 1)2
2

2s + 1
=
(s + s + 1)2
2
86 Electronics Engineering Signals & Systems

1 Q Hence, A = 1 × H ( j ) = 2
c(t) = cos(3t 126.86°)
15
1
1 = 1× = 0.707
= sin(3t 36.86°) ...(i) 2
15 and = +45°
or, c(t) = A sin(3t + ) ...(ii)
Comparing equations (i) and (ii), we have 21. (d)

1 The differential equation,


A=
15 dy(t ) 1
+ y(t ) = 3x(t)
dt 6
18. (b)
1
So, sY (s ) + Y (s ) = 3X(s)
t 3/2 6
Given that, f(t) = 2 F( s ) = s
3X(s)
By using property of differentiation in time, Y(s) =
1
df (t ) s+
sF(s ) 6
dt
3
2 1 1/2 X(s) =
t sF(s ) 1
2 s+
3
1 3 /2
s s
t 9 54 54
So, Y (s) = =
1 1 1 1 1
s 1 /2 s+ s+ s+ s+
t 3 6 6 3
So, y(t) = (54e–t/6 – 54e–t/3) u(t)
19. (a)
5 22. (a)
Laplace transform of sin 5t u(t )
s 2 + 25 The differential equation is,
5 5 y (t) + 2y (t) + y(t) = 0
e 2 t sin 5t u(t ) 2
= 2 2
So, (s Y(s) – sy(0) – y (0)) + 2[sY(s) – y(0)] + Y(s)
(s 2) + 25 s 4s + 29
=0
20. (b) sy(0) + y (0) + 2 y(0)
So, Y(s) =
Y (s ) s (s 2 + 2 s + 1)
=
R( s ) s+2 Given that, y (0) = 1, y(0) = 0
y(t) = A cos(2t + ), r(t) = cos2t 1
So, Y(s) =
s (s + 1)2
Q H(s) =
(s + 2) So, y(t) = te–t u(t)
j
H(j ) = ; H( j ) = 23. Sol.
j +2 2
+4
1
H(s) =
H(j ) = 90° tan 1 (s + 1)
2
Put, s=j
Q = 2 (Given)
1
2 2 1 H(j ) =
H( j ) = = = j +1
4+4 2 2 2
1
H ( j ) = 90° – tan–1(1) = 45° H( j ) =
2
+1
GATE Previous Years Solved Paper 87

Q Input, x(t) = cos(t) 27. (d)


Here, = 1 rad/sec.
dy(t )
and x(t ) = 1 + 4 y(t ) = 6 r(t)
dt 2
Hence, steady state output, [s2 + 4] Y(s) = 6 R(s)
y(t) = H ( j ) = 1 cos(t + H ( j ) = 1 ) Y (s ) 6
= 2
1 R(s ) s +4
A = H( j ) =1
= = 0.707
2 Poles: s2 + 4 = 0
s = ±j2
24. (b)
Steady state value of y(t) 29. (a)
= lim sY (s) Pole locations: (2, –1) and (–2, 1)
s 0 Zero locations: (2, 1) and (–2, 1)
10s j
= lim 2
s 0 s(s + s + 100 2 )
1
10 1
= =
100 2 10 2

25. (b) –2 2
c1
H(s) = 2 (as a1 = b1 = 0)
a2 s + b2 s + c 2
–1
c1
=
(1 + s 1 ) (1 + s 2) Filter is all pass since for each pole, there is a
c1 mirror image zero.
Put s = 0, H(0) =
c2 System is unstable because for stability of causal
Put s = , H( ) = 0 system all poles should lie in the L.H.S. of
s-plane and here one pole is lying in the R.H.S.
At s = 0 At s = Type of filter
[s (2 + j ) [s ( 2 j )]
Constant 0 LPF H(s) =
[s (2 + j ) [s ( 2 + j )]
0 Constant HPF
[s (2 + j ) [s + (2 + j )]
0 0 BPF =
[s (2 j ) [s + ( 2 j )]
1 1 BSF
s2 (2 + j )2 s 2 (4 1 + 4 j )
which represents 2nd order low pass filter. = =
s 2 (2 j )2 s2 (4 1 4 j )
26. (a)
s2 3 4j
=
s+3 s + 1+ 2 s 2
3+ 4j
L 1
= L 1
s2 + 2s + 1 (s + 1)2 i.e. transfer function is complex.

1 1 2
= L +
s + 1 (s + 1)2
= e–t + 2te–t
6 Z-Transform

ELECTRO NICS EN GINEERIN G Q.4 The z-transform of the time function

(GATE Previous Years Solved Papers)


(n k ) is
k=0
Q.1 Consider the system shown in the figure below.
z 1 z
The transfer function Y(z)/X(z) of the system is (a) (b)
z z 1
y (k ) z
x(k) ( z 1)2
(c) 2 (d)
( z 1) z
Z–1
[EC-1998 : 1 Mark]

Q.5 The z-transform F(z) of the function f(nT) = anT


a is
–b
z z
(a) T (b)
z a z + aT
1 + az 1 1 + bz 1
(a) (b)
1 + bz 1 1 + az 1 z z
(c) T (d) T
1 1 z a z+a
1 + az 1 bz
(c) (d) [EC-1999 : 1 Mark]
1 bz 1 1 + az 1
[EC-1988 : 2 Marks] Q.6 The z-transform of a signal is given by

Q.2 The Z-transform of the following real 1 z 1 (1 z 4 )


C ( z) =
exponential sequence: 4 (1 z 1 )2
x(nT) = an, nT 0 Its final value is
= 0 , nT < 0, a > 0
1
1 1 (a) (b) zero
(a) ; z >1 (b) ; z >a 4
1 1
1 z 1 az (c) 1.0 (d) infinity
1 [EC-1999 : 2 Marks]
(c) 1 for all z (d) 1
; z <a
1 az
Q.7 The region of convergence of the z-transform of
[EC-1990 : 2 Marks] a unit step function is
Q.3 A linear discrete-time system has the (a) z >1
characteristic equation, z3 – 0.81z = 0. The
(b) z <1
system
(c) (Real part of z) > 0
(a) is stable.
(d) (Real part of z) < 0
(b) is marginally stable.
[EC-2001 : 1 Mark]
(c) is unstable.
(d) stability cannot be assessed from the given Q.8 If the impulse response of a discrete-time system
information. is h[n] = –5n u[–n –1], then the system function
[EC-1992 : 2 Marks] H(z) is equal to
GATE Previous Years Solved Paper 89

z Q.12 The region of convergence of z-transform of the


(a) and the system is stable.
z 5 n n
5 6
z sequence u(n) u ( n 1) must be
(b) and the system is stable. 6 5
z 5
z 5 5
(c) and the system is unstable. (a) z < (b) z >
z 5 6 6

z 5 6 6
(c) < z< (d) < z<
(d) and the system is unstable. 6 5 5
z 5
[EC-2005 : 1 Mark]
[EC-2002 : 2 Marks]
Q.13 If the region of convergence of x1[n] + x2[n] is
Q.9 A sequence x(n) with the z-transform
X(z) = z4 + z2 – 2z + 2 – 3z–4 is applied as an 1 2
< z < , then the region of convergence of
input to a linear, time-invariant system with the 3 3
impulse response h(n) = 2 (n – 3) where,
x1[n] – x2[n] includes
1, n=0 1 2
(n) = (a) < z <3 (b) < z <3
0 , otherwise 3 3
The output at n = 4 is 3 1 2
(c) < z <3 (d) < z<
(a) –6 (b) zero 2 3 3
(c) 2 (d) –4 [EC-2006 : 1 Mark]
[EC-2003 : 1 Mark] Q.14 The z-transform X[z] of a sequence x[n] is given

z 0.5
Q.10 The z-transform of a system is H ( z) = . If by X [ z] = . It is given that the region of
1
z 0.2 1 2z
convergence of X[z] includes the unit circle. The
the ROC is z < 0.2, then the impulse response
value of x[0] is
of the system is
(a) –0.5 (b) 0
(a) (0.2)n u[n]
(c) 0.25 (d) 0.5
(b) (0.2)n u[–n – 1]
[EC-2007 : 2 Marks]
(c) –(0.2)n u[n]
(d) –(0.2)n u[–n – 1] Statement for Linked Answer Questions (15 and 16):
[EC-2004 : 1 Mark] In the following network, the switch is closed at t = 0–
and the sampling starts from t = 0. The sampling
Q.11 A causal LTI system is described by the frequency is 10 Hz.
difference equation,
S 10 µF
2y[n] = y[n – 2] – 2x[n] + x[n – 1]
x(n) X(z )
The system is stable only if 5V 200 k
Sampler
z-transform
(fs = 10 Hz)
(a) = 2, <2

(b) > 2, >2


Q.15 The sample x(n) (n = 0, 1, 2,....) are given by
(c) < 2, any value of
(a) 5(1 – e–0.05n) (b) 5 e–0.05n
(d) < 2, any value of
(c) 5(1 – e–5n) (d) 5 e–5n
[EC-2004 : 2 Marks] [EC-2008 : 2 Marks]
90 Electronics Engineering Signals & Systems

Q.16 The expression and the region of convergence Consider the following statements:
of the z-transform of the sampled signal are S1 : The system is stable and causal for
5z 5 1
(a) 5
, z <e ROC : z >
z e 2
5z 0.05 S2 : The system is stable but not causal for
(b) 0.05
, z <e
z e
1
5z ROC : z <
0.05 4
(c) 0.05
, z >e
z e S3 : The system is neither stable nor causal for
5z 5
(d) , z >e 1 1
z e 5 ROC : < z<
4 2
[EC-2008 : 2 Marks]
Which one of the following statement is valid?
Q.17 The ROC of z-transform of the discrete-tme (a) Both S1 and S2 are true.

n n
(b) Both S2 and S3 are true.
1 1
sequence, x(n) = u(n) u ( n 1) is (c) Both S1 and S3 are true.
3 2
(d) S1, S2 and S3 are all true.
1 1 1 [EC-2010 : 2 Marks]
(a) < z< (b) z >
3 2 2
Q.21 Two systems H1(z) and H2(z) are connected in
1 cascade as shown below. The overall output
(c) z < (d) 2 < z < 3
3 y(n) is the same as the input x(n) with a one unit
[EC-2009 : 1 Mark] delay. The transfer function of the second system
H2(z) is
Q.18 Consider the z-transform, X(z) = 5z2 + 4z–1 + 3,
1
0 < z < . The inverse z-transform x[n] is x(n) H 1 ( z) =
(1 0.4z )
H2(z) y(n)
1
(1 0.6z )
(a) 5 [n + 2] + 3 [n] + 4 [n – 1]
(b) 5 [n – 2] + 3 [n] + 4 [n + 1]
(1 0.6 z 1 ) z 1 (1 0.6 z 1 )
(c) 5u[n + 2] + 3u[n] + 4u[n – 1] (a) (b)
z 1 (1 0.4 z 1 ) (1 0.4 z 1 )
(d) 5u[n – 2] + 3u[n] + 4u[n + 1]
[EC-2010 : 1 Mark] z 1 (1 0.4 z 1 ) (1 0.4 z 1 )
(c) (d)
Q.19 Two discrete-time systems with impulse (1 0.6 z 1 ) z 1 (1 0.6 z 1 )
responses h1[n] = [n – 1] and h2[n] = [n – 2] are [EC-2011 : 2 Marks]
connected cascade. The overall impulse
response of the cascaded system is n
Q.22 If x [n] = (1/3) (1/2)n u[ n], then the region
(a) [n – 1] + [n – 2] (b) [n – 4]
of convergence (ROC) of its z-transform in the
(c) [n – 3] (d) [n – 1] [n – 2]
z-plane will be
[EC-2010 : 1 Mark]
1 1 1
(a) < z <3 (b) < z<
Q.20 The transfer function of a discrete-time LTI 3 3 2
system is given by
1 1
(c) < z <3 (d) < z
3 1 2 3
2 z
H ( z) = 4 [EC-2012 : 1 Mark]
3 1 1 2
1 z + z
4 8
GATE Previous Years Solved Paper 91

Q.27 Let H1(z) = (1 – pz–1)–1, H2(z) = (1 – qz–1)–1,


n n
1 1 H(z) = H1(z) + rH2(z). The quantities p, q, r are
Q.23 Let x [n] = u(n) u ( n 1). The
9 3
1 1
region of convergence (ROC) of the z-transform real numbers. Consider p = ,q= , r < 1.
2 4
of x[n] is
If the zero of H(z) lies on the unit circle, then
1 1 r = ________ .
(a) z > (b) z <
9 3
[EC-2014 : 2 Marks]
1 1
(c) > z > (d) does not exist Q.28 The z-transform of the sequence x[n] is given by
3 9
[EC-2014 : 2 Marks] 1
X ( z) = , with the region of
Q.24 Let x[n] = x[–n], X(z) be the z-transform of x[n]. If (1 2 z 1 )2
0.5 + j0.25 is a zero of X(z), which one of the
convergence z > 2. Then, x[2] is ________ .
following must also be a zero of X(z).
1 [EC-2014 : 2 Marks]
(a) 0.5 – j0.25 (b)
(0.5 + j 0.25) Q.29 For the discrete-time system shown in the figure,
1 the poles of the system transfer function are
(c) (d) 2 + j4
(0.5 j 0.25) located at
[EC-2014 : 1 Mark] x[ n] y[n]

Q.25 The input-output relationship of a causal stable


LTI system is given as, 1 5
6 6
y[n] = y[n – 1] + x[n]
If the impulse response h[n] of this system –1 –1
Z Z

satisfies the condition h[n] = 2, the 1


(a) 2, 3 (b) ,3
n=0 2
1 1 1
relationship between and is (c) , (d) 2,
2 3 3
(a) =1 (b) = 1+ [EC-2015 : 2 Marks]
2 2
(c) =2 (d) = –2 Q.30 The pole-zero diagram of a causal and stable
[EC-2014 : 2 Marks] discrete-time system is shown in the figure. The
zero at the origin has multiplicity 4. The impulse
1 response of the system is h[n]. If h[0] = 1, we can
(z b)
Q.26 For an all-pass system H ( z) = , where conclude
(1 az 1 )
Im(z)

j
H(e ) = 1, for all . If Re(a) 0, Im(a) 0,
0.5
then b equals
(a) a (b) a Re(z)
–0.5 0.5
1 1
(c) (d) –0.5
a a
[EC-2014 : 1 Mark]
92 Electronics Engineering Signals & Systems

(a) h[n] is real for all n. The condition on the filter coefficient that results
(b) h[n] is purely imaginary for all n. in a null at zero frequency is
(c) h[n] is real for only even n. (a) 1 = 2 = 0; 0 =– 3

(d) h[n] is purely imaginary for only odd n. (b) 1= 2 = 1; 0=– 3

[EC-2015 : 2 Marks] (c) 0= 3 = 0; 1= 2


(d) 1= 2 = 0; 0= 3
Q.31 Two causal discrete-time signals x[n] and y[n]
[EC-2015 : 1 Mark]
n
are related as y[n] = x [m]. If the Q.34 Suppose x[n] is an absolutely summable
m=0 discrete-time signal. Its z-transform is a rational
function with two poles and two zeroes. The
2 poles are at z = ±2j. Which one of the following
z-transform of y[n] is , the value of x[2]
z ( z 1)2 statements is true for the signal x[n]?
(a) It is a finite duration signal.
is ________ .
(b) It is a causal signal.
[EC-2015 : 1 Mark]
(c) It is a non-causal signal.
Q.32 A realization of a stable discrete-time system is (d) It is a periodic signal.
shown in figure. If the system is excited by a
[EC-2015 : 2 Marks]
unit step sequence input x[n], the response y[n] is
Q.35 Consider the sequence x[n] = an u[n] + bn u[n],
x[n] where u[n] denotes the unit-step sequence and

–1
0 < a < b < 1. The region of convergence (ROC)
Z
of the z-transform of x[n] is
1 –5/3
y[n] (a) z > a (b) z > b

–1
(c) z < a (d) a < z< b
Z
[EC-2016 : 1 Mark]

–2/9 5/3 Q.36 The ROC (region of convergence) of the


z-transform of a discrete-time signal is
1 n
2 n represented by the shaded region in the z-plane.
(a) 4 u [ n] 5 u [ n] If the signal,
3 3
n n x[n] = (2.0) n , <n<+
2 1
(b) 5 u [ n] 3 u [n] then the ROC of its z-transform is represented
3 3
by
n n
1 2
(c) 5 u [ n] 5 u [ n]
3 3 Img
Unit circle
n n z-plane
2 1
(d) 5 u [ n] 5 u [ n]
3 3
[EC-2015 : 2 Marks] (a) Re
0.5 2

Q.33 Consider a four point moving average filter


3
defined by the equation y[n] = i=0 i x [n i ].
GATE Previous Years Solved Paper 93

Img Q.39 Let H(z) be the z-transform of a real-valued

Unit circle z-plane 1


discrete-time signal h[n]. If P( z ) = H ( z) H
z

(b) Re 1 1
0.5 2 has a zero at z = + j , and P(z) has a total of
2 2
four zeros, which one of the following plots
represents all the zeros correctly?

Img Img
Unit circle z-plane 2
z-plane
z =1
(c) Re 0.5
0.5 2
(a) Re
–2 –0.5 0.5 2
–0.5

Img
Unit circle z-plane –2

(d) Re Img
0.5 2
2

(ROC does not exist) z-plane


z =1
[EC-2016 : 2 Marks] 0.5

Q.37 A discrete-time signal x[n] = [n – 3] + 2 [n – 5] (b) Re


–2 0.5 2
has z-transform X(z). If Y(z) = X(–z) is the
–0.5
z-transform of another signal y[n], then
(a) y[n] = x[n] (b) y[n] = x[–n]
–2
(c) y[n] = –x[n] (d) y[n] = –x[–n]
[EC-2016 : 1 Mark]

Q.38 A discrete-time all pass system has two of its Img

poles at 0.25 0° and 2 30°. Which one of the 2


following statements about the system is true? z-plane
z =1
(a) It has two more poles at 0.5 30° and 4 0°. 0.5
(b) It is stable only when the impulse response
(c) Re
is two-sided. –2 –0.5 0.5 2
(c) It has constant phase response over all
frequenicies.
(d) It has constant phase response over the
–2
entire z-plane.
[EC-2018 : 1 Mark]
94 Electronics Engineering Signals & Systems

Img
2 Img
z-plane
1 1
3rd order pole
z =1
0.5

(d) Re –1 1
–2 0.5 1 2
(d) Re

–0.5

–1
–1
–2

[EC-2019 : 1 Mark]
[EC-2020 : 1 Mark]
Q.40 Which one of the following pole-zero
corresponds to the transfer function of an LTI Q.41 The transfer function of a stable discrete-time
system characterized by the input-output K( z )
LTI system is H ( z) = , where K and
difference equation given below? z + 0.5
3
are real numbers. The value of (rounded off to
y[n] = ( 1)k x [n k ]
k=0 one decimal place) with > 1, for which the
Img magnitude response of the system is constant
1 over all frequencies, is ________ .
4th order pole
[EC-2020 : 2 Marks]

–1 1
(a) Re ELECTRICAL EN GINEERIN G
(GATE Previous Years Solved Papers)

–1 Q.1 If u(t) is the unit step and (t) is the unit impulse

1
Img function, the inverse z-transform of R( z ) =
z+1
1
3rd order pole
for k > 0 is
(a) (–1)k (k) (b) (k) – (–1)k u(k)
–1 1
(b) Re (c) (–1)k u(k) (d) u(k) – (–1)k (k)
[EE-2005 : 2 Marks]

Q.2 A discrete real all pass system has a pole at


–1
z = 2 30°, it therefore,
(a) also has a pole at 0.5 30°
Img
1 (b) has a constant phase response over the
3rd order pole
z-plane: arg |H(z)| = constant
(c) is stable only if it anti-causal
–1 1
(c) Re (d) has a constant phase response over the
unit circle: arg |H(e j )| = constant
[EE-2006 : 1 Mark]
–1
GATE Previous Years Solved Paper 95

Q.3 The discrete-time signal Linked Answer Questions (6 and 7):

3n 2 n Q.6 A signal is processed by a causal filter with


x[n ] X ( z) = n=0 2 + n
z transfer function G(s). For a distortion free output
signal waveform, G(s) must
where denotes a transform-pair relationship,
is orthogonal to the signal. (a) provide zero phase shift for all frequency.
(b) provide constant phase shift for all
n
2 n frequency.
(a) y1 [n] Y1 ( z) = n=0
z
3 (c) provide linear phase shift that is
proportional to frequency.
(b) y2 [n] Y2 ( z) = n=0
(5n n) z (2 n + 1)
(d) provide a phase shift that is inversely
n n proportional to frequency.
(c) y3 [n] Y3 ( z) = n=
2 z
[EE-2007 : 2 Marks]
(d) y4[n] Y4(z) = 2z–4 + 3z–2 + 1
Q.7 G(z) = z–1 + z–3 is a digital band stop filter
[EE-2006 : 2 Marks]
with a phase characteristics same as that of the
Q.4 Consider the discrete-time system shown in the above question if
figure where the impulse response of G(z) is (a) = (b) =–
g(0) = 0, g(1) = g(2) = 1, g(3) = g(4) = .... = 0. (c) = (1/3) (d) = (–1/3)

+
G(z ) [EE-2007 : 2 Marks]
+ Q.8 H(z) is a transfer function of a real system when
a signal x[n] = (1 + j)n is the input to such a
K system, the output is zero.
Further, the region of convergence (ROC) of
The system is stable for range of values of K

1 1 1
(a) 1, (b) (–1, 1) 1 z H ( z) is the entire z-plane
2 2

(except z = 0). It can then be inferred that H(z)


1 1
(c) ,1 (d) ,2 can have a minimum of
2 2
(a) one pole and one zero
[EE-2007 : 2 Marks]
(b) one pole and two zeroes
Q.5 X(z) = 1 – 3z–1, Y(z) = 1 + 2z2 are Z-transforms of (c) two poles and one zero
two signals x[n], y[n] respectively. A linear time (d) two poles and two zeroes
invariant system has the impulse response h[n]
[EE-2008 : 2 Marks]
defined by these two signals as h[n] = x[n – 1]
y[n] where denotes discrete time convolution.
z
Then the output of the system for the input Q.9 Given, X ( z) = with z > a , the residue
[n –1]. ( z a )2
of X(z) zn – 1 at z = a for n 0 will be
(a) has Z-transform z–1 X(z) Y(z)
(a) an – 1 (b) an
(b) equals [n – 2] – 3 [n – 3] + 2 [n – 4] – 6 [n – 5]
(c) nan (d) nan – 1
(c) has Z-transform 1 – 3z–1 + 2z–2 – 6z–3
[EE-2008 : 2 Marks]
(d) does not satisfy any of the above three
[EE-2007 : 2 Marks] Q.10 The Z-transform of a signal x[n] is given by
4z–3 + 3z–1 + 2 – 6z2 + 2z3
96 Electronics Engineering Signals & Systems

It is applied to a system, with a transfer function N


Y ( z) 1 1 z
H(z) = 3z–1 – 2. Let the output be y(n). Which of = 1
X ( z) N 1 z
the following is true?
(a) y(n) is non-causal with finite support. where, N represents the number of samples per
(b) y(n) is causal with infinite support. cycle. The output y(n) of the system under steady-
(c) y(n) = 0, |n| > 3 state is
(a) 0 (b) 1
(d) Re[Y ( z)] = Re[Y ( z)] ;
z=ej z=e j (c) 2 (d) 5
Im[Y ( z)] = Im[Y ( z)] ;– < [EE-2014 : 2 Marks]
z=ej z=e j

[EE-2009 : 2 Marks] Q.15 Consider a discrete time signal given by:


x[n] = (–0.25)n u[n] + (0.5)n u[–n – 1]
n The region of convergence of its Z-transform
1 1
Q.11 If x[n] = u [n], the region of
3 2 would be
(a) the region inside the circle of radius 0.5 and
convergence (ROC) of its Z-transform in the
centered at origin.
z-plane will be
(b) the region outside the circle of radius 0.25
1 1 1
(a) < z <3 (b) < z< and centered at origin.
3 3 2
(c) the annular region between the two circles,
1 1
(c) < z <3 (d) < z <3 both centered at origin and having radii
2 3
0.25 and 0.5.
[EE-2012 : 1 Mark]
(d) the entire z-plane.
1 [EE-2015 : 2 Marks]
Q.12 Let, X ( z ) = be the Z-transform of a
3
1 z Q.16 The Z-transform of a sequence x[n] is given as
causal signal x[n]. Then, the values of x[2] and 4 3
X ( z) = 2 z + 4 +
x[3] are z z2
(a) 0 and 0 (b) 0 and 1 If y[n] is the first difference of x[n], then Y[z] is
(c) 1 and 0 (d) 1 and 1 given by
[EE-2014 : 2 Marks] 8 7 3
(a) 2 z + 2 +
Q.13 A discrete system is represented by the difference z z2 z3
equation, 6 1 3
(b) 2z + 2 +
X1 ( k + 1) a a 1 X1 ( k ) z z2 z3
=
X 2 ( k + 1) a+1 1 X2 ( k ) 8 7 3
(c) 2z 2 + 2
+
It has initial conditions X1(0) = 1, X2(0) = 0. The z z z3
pole locations of the system for a = 1, are 8 1 3
(d) 4 z 2 2
+
(a) 1 + j0 (b) –1 – j0 z z z3
(c) –1 + j0 (d) 0 + j1 [EE-2015 : 2 Marks]
[EE-2014 : 2 Marks]

Q.17 Let, S = n where < 1. The value of a


Q.14 An input signal x(t) = 2 + 5 sin(100 t) is sampled n
with a sampling frequency of 400 Hz and n=0
in the range of 0 < < 1, such that S = 2a is ___.
applied to the system whose transfer function
is represented by [EE-2016 : 2 Marks]
GATE Previous Years Solved Paper 97

Q.18 Consider a causal and stable LTI system with Q.20 A cascade system having the impulse responses
rational transfer function H(z), whose
h1 (n) = {1 , 1} and h2 (n) = {1 , 1} is shown in
corresponding impulse response begins at n = 0.

5 the figure below, where symbol denotes the time


Furthermore, H (1) = . The poles of H(z) are:
4 origin.

1 2 k 1) x(n) h1(n) h2(n) y(n)


pk = exp j for k = 1, 2, 3, 4
2 4
The input sequence x(n) for which the cascade
The zeros of H(z) are all at z = 0. Let g[n] = jn h[n]. system produces an output sequence
The value of g[8] equals _____.
y(n) = {1, 2, 1, 1, 2, 1} is
(Given the answer upto 3 decimal places.)
[EE-2017 : 2 Marks]
(a) x(n ) = {1, 2, 1, 1} (b) x(n ) = {1, 1, 2, 2}
Q.19 The pole zero plots of three discrete-time systems
(c) x(n) = {1, 1, 1, 1} (d) x(n ) = {1, 2, 2, 1}
P, Q and R on the z-plane are shown below.
[EE-2017 : 2 Marks]
Im(z) Im(z)

2 poles P Q n
1
0.5 Q.21 Consider a signal x[n] = 1[n], where
2
Re(z) Re(z)
–0.5 1[n] = 0 if n < 0 and 1[n] = 1 if n 0. The
Unit circles Unit circles
z k
(i) (ii) Z-transform of x[n – k], k > 0 is with
1 1
1 z
2
Im(z) region of convergence being
R 1 1
(a) z< (b) z >
2 2
(c) z <2 (d) z >2
Re(z)
[EE-2020 : 1 Mark]
Unit circles
Q.22 Which of the following options is true for a linear
(iii) time-invariant discrete time system that obeys
the difference equation:
Which one of the following is true about the
y[n] – ay[n – 1] = b0 x[n] – b1x[n – 1]
frequency selectivity of these systems?
(a) Where x[n] = 0, n < 0 the function y[n], n > 0
(a) All three are high-pass filters.
is solely determined by the function x[n].
(b) All three are band-pass filters.
(b) The system is necessarily causal.
(c) all three are low-pass filters. (c) y[n] is unaffected by the values of x[n – k],
(d) P is a low-pass filter, Q is a band-pass filter k > 2.
and R is a high-pass filter. (d) The system impulse response is non-zero
[EE-2017 : 1 Mark] at infinitely many instants.
[EE-2020 : 2 Marks]
98 Electronics Engineering Signals & Systems

Electronics & Electrical Engineering


GATE Previous Years Solved Paper

A n swe rs & Expl a n a t i o n s

Answers
EC Z-Transform

1. (a) 2. (b) 3. (a) 4. (b) 5. (a) 6. (c) 7. (a) 8. (b)

9. (b) 10. (d) 11. (c) 12. (c) 13. (d) 14. (b) 15. (b) 16. (c)

17. (a) 18. (a) 19. (c) 20. (c) 21. (b) 22. (c) 23. (c) 24. (b)

25. (a) 26. (b) 27. (–0.5) 28. (12) 29. (c) 30. (a) 31. (0) 32. (c)

33. (a) 34. (c) 35. (b) 36. (d) 37. (c) 38. (b) 39. (d) 40. (c)

41. (–2)

Solutions
EC Z-Transform

1. (a) 2. (b)
Given that,
+ y1(z) +
x(z) y (z ) x(n) = an u(n) a>0
+ +
Z.T. z
x( n ) X(z) = ; z > a
Z–1 z a
1
X(z) = 1
; z >a
–b a 1 az

y1(z) = x(z) – bz–1 y1(z) 3. (a)


y1(z)[1 + bz–1] = x(z) Given that:
x( z ) Characteristic equation,
y1(z) = z3 – 0.81z = 0
(1 + bz 1 )
z(z2 – 0.81) = 0
y(z) = y1(z) + az–1 y1(z)
z(z – 0.9) (z + 0.9) = 0
y(z) = y1(z) [1 + az–1]
So, poles are z = 0, 0.9 and –0.9
x( z) As all the three poles are inside the unit circle,
y(z) = × (1 + az 1 )
(1 + bz 1 ) so the system is stable.
Transfer function,
4. (b)
1
y( z) 1 + az
H(z) = =
x( z) 1 + bz 1 Given that, x(n) = (n k )
k=0
GATE Previous Years Solved Paper 99

x(n) = (n) + (n – 1) + (n – 2) +... 8. (b)


x(n) = u(n) h(n) = –5n u[–n – 1]
X(z) = Z.T. [u(n)]
n
z H(z) = h(n) z
X(z) = n=
z 1
1
5. (a) 5n z n
H(z) = Let, n = –k
Z-transform is given by n=

n z z
z[f(nT)] = f (nT ) z = = z<5
n= 5 z z 5
The system is stable because ROC includes unit
= a nT
z n
=
T 1 n
(a z ) circle.
n= n=
9. (b)
1 z
F(z) = = Y(z) = H(z) X(z)
1 aT z 1
z aT
H(z) = 2z –3
6. (c) Y(z) = 2z–3 (z4 + z2 – 2z + 2 – 3z–4)
= 2(z + z–1 – 2z–2 + 2z–3 – 3z–7)
Final value theorem,
Taking inverse of z-transform,
lim x[ N ] = lim (1 z 1 ) X( z) y(n) = 2[ (n + 1) + (n – 1) – 2 (n – 2)
N z 1
+ 2 (n – 3) – 3 (n – 7)]
1 z 1 (1 z 4 )
lim (1 z 1 ) At n = 4,
z 1 4 (1 z 1 )2 y(4) = 0
1 z 1 (1 z 4 )
= lim 10. (d)
z 1 4 (1 z 1 )
z z
H(z) = =
1 ( z2 1) ( z2 + 1) z 0.2 z(1 0.2 z 1 )
= lim
z 1 4 z 4 ( z 1)
1
2 H(z) = 1
1 ( z + 1) ( z 1) ( z + 1) 1 0.2z
= lim
z 1 4 z 4 ( z 1) ROC : z < 0.2
1 Comparing with,
= ×4 =1
4 1
an u( n 1) ; z <a
7. (a) 1 az 1
h(n) = u(n) we get, h(n) = –(0.2)n u(–n – 1)

n 11. (c)
H(z) = 1 z
n=0 2y[n] = y[n – 2] – 2x[n] + x[n – 1]
For the convergence of H(z), Taking z-transform,
2Y(z) = Y(z)z–2 – 2X(z) + X(z)z–1
( z 1 )n < Y(z) [2 – z–2] = X(z) [ z–1 – 2]
n=0
1
Y ( z) z 2
ROC is the range of values of z for which, = 2
X( z ) 2 z
1
z < 1 or z > 1 For system to be stable, can be of any value,
100 Electronics Engineering Signals & Systems

2 – z–2 = 0 16. (c)


2z2 – = 0
0.05n n
X(z) = 5e z
z= <1 n=0
2
<2 = 5 (e 0.05
z 1 )n
For system to be stable all poles should be inside n=0
unity circle. 0.05 1
For e z or z > e 0.05
12. (c)
5z
n n X(z) = 0.05
5 6 z e
z-transform of u(n) u( n 1)
6 5
17. (a)
n n
1 1
1 1 x(n) = u(n) u( n 1)
= + 1 3 2
5 1 6 1
1 z 1 z n
6 5 1
u(n) is right sided signal so ROC will be
(ROC) (ROC) 3

5 6 1
= z > = z < z > ...(i)
6 5 3
5 6
< z< 1 n
6 5 u( n 1) is left sided signal so ROC
2
13. (d) will be
ROC remains the same for addition and 1
z < ...(ii)
subtraction in z-domain. 2
From equation (i) and (ii) we see that ROC of the
14. (b) function will be
0.5 1 1
X(z) = 1 < z <
1 2z 3 2
Q ROC includes unit circle 18. (a)
Left handed system
z
x(n) = –(0.5) (2)n u(–n – 1) [ n + n0 ] zn0
x(0) = 0 X(z) = 5z2 + 4z–1 + 3; 0< z <

15. (b) x(n) = 5 (n + 2) + 4 (n – 1) + 3 (n)

19. (c)
200 × 10 3 5
VR(s) = z
1 s h1[n] = [n 1] H 1 ( z) = z 1
200 × 10 3 +
10 × 10 6 s z
h2[n] = [ n 2] H 2 ( z) = z 2
5 6
5 × 2 × 10 × 10 × 10
= Overall impulse response in z-domain,
2 × 10 5 × 10 5 s + 1
H(z) = H1(z) H2(z)
10 5
= = = z–1 z–2 = z–3
2s + 1 s + 0.5
Overall impulse response in discrete-time
Therefore samples,
domain, h[n] = (n – 3)
x(n) = 5e–0.5n/10 = 5e–0.05n
GATE Previous Years Solved Paper 101

20. (c) 22. (c)


1. A discrete-time LTI system is causal if and n n
1 1
only if the ROC of its system function is the x[n] = u [ n]
3 2
exterior of circle, including infinity.
n n n
2. A discrete-time LTI system is stable if and 1 1 1
x[n] = u [n] + u [ n 1] u[n]
3 3 2
only if the ROC of its system function
includes the unit circle, 1 n 1 1
u [n] ROC : z >
3 1 1; 3
1 1 1 1 1 z
1 z + 1 z 3
4 2
H(z) =
1 1 1 1 1 n
1
1 z 1 z u [ n 1] ; ROC : z < 3
4 2 3 1
1 3z
1 1
or, H(z) = + 1 n 1 1
1 1 u [n] ROC : z >
1 z 1
1 z 1
2 1 1 ; 2
4 2 1 z
2
1 So overall ROC will be intersection of there ROCs
For ROC : z > , the system is stable and
i.e.,
2
causal. 1
< z <3
2
1
For ROC : z > , ROC does not include 23. (c)
4
unit circle. So, system is not stable. 1 n 1 n
x(n) = u(n) u( n 1)
1 1 9 3
For ROC : < z < , ROC does not x1 ( n ) x2 ( n )
4 2
n
include unit circle. So, system is not stable. 1
x1(n) = u(n) Right sided signal
9
1
Also ROC is not the exterior of z = . So, it
1
2
X1(z) =
1
is not causal. 1 z 1
9
21. (b)
1 1
y[n] = x[n – 1] ROC : z or z >
9 9
Taking z-transform of both sides,
n
Y(z) = z–1 X(z) 1
x2(n) = u ( n 1) Right sided signal
Y ( z) 3
= z –1
X( z ) 1 1
X2(z) = ; z
For cascaded system, 1 3
1 z 1
H(z) = H1(z) H2(z) 3
(1 0.4 z 1 ) Now, when both the transforms are added using
z –1 = H 2 ( z)
(1 0.6 z ) 1 linearity property.
1 1
z 1 (1 0.6 z 1 ) ROC must be between > z > .
H2(z) = 3 9
(1 0.4 z 1 )
102 Electronics Engineering Signals & Systems

24. (b) 1 1 1
= j a
=
Given, x[n] = x[–n] b ae a
As we know that, or, b=a
z
x[ n] X[ z ]
27. Sol.
z 1
x[ n] X[ z ] 1
Here, H1(z) =
So, when (0.5 + j0.25) is a zero fo X(z). 1 pz 1
Now, x[ n] X( z 1 ) 1
H2(z) =
1 1 qz 1
Zero of X ( z) =
(0.5 + j0.25)
1 1
Also, p = ; q=
25. (a)
2 4
H(z) = H1(z) + rH2(z)
Input-output relation is given as,
y[n] = y(n – 1) + x[n] 1 r
= 1
+
Taking z-transform, 1 pz 1 qz 1
Y(z) = z–1 Y(z) + X(z) 1 r
Y(z) [1 – z–1] = X(z) = +
1 1 1 1
1 z 1+ z
Y ( z) 2 4
= H ( z) = 1
X( z ) 1 z
1 1 1 1
h[n] = an u[n] 1+ z +r 1 z
4 2
H(z) = ...(i)
1 1 1
Alos, h[n] = 2 n
u[n] = 2 1 z 1+ z 1
2 4
n=0 n=0
zero of H(z) (1 + r) – (q + pr) z–1 = 0 ...(ii)
=2 =2–2 q + pr
1 or, z=
2 = 2– 1+r
Since, the zero of H(z) lies on the unit circle.
= 1
2 Therefore, z = 1 or z = ±1
Taking, z = 1, we get
26. (b)
q + pr
For the given system, z= =1 ...(iii)
1+r
1
(z b)
H(z) = 1 ...(i) 1
(1 az ) By solving above expression with p = and
2
1
The pole lie at z = a and the zero lie at z = . 1
b q= , we get
Also ‘a’ is complex in nature. 4

j a r = –2.5
Thus, a= ae
But, r < 1 or –1 < r < 1
For an all pass system where H ( e j ) = 1, for Taking, z = –1 in equation (iii), we get
r = –0.5
all , if the pole lies at ‘a’ then the zero must be
present at 1/a .
GATE Previous Years Solved Paper 103

28. Sol. 2 z
Given, Y(z) = 2
= X ( z)
1 z( z 1) z 1
Given, X(z) = 1
; z >2
(1 2 z )
2 z 2 z
X(z) = (1 – 2z–1)–2 X(z) = = 2z 3
z 1 z 1
X(z) = 1 + 4z–1 + 12z–2 +...
x[n] = 2u[n – 3]
(using binomial expansion)
Therefore, x[2] = 0
Taking inverse z-transform, we get
x[n] = {1, 4, 12, ... 32. (c)

x[2] = 12 5
Y ( z) [1 z]
3
29. (c) =
X( z ) 2
z2 z +
The difference equation of the system, 9
1 5 (1 z)
x( n ) y(n 2) + y(n 1) = y(n) 5
6 6 Y(z) =
3 1 2
z z
Y ( z) 1 3 3
H(z) = =
X ( z) 1 + 1 z 2 5 z 1
6 6 5z 5z
Y(z) =
1 2
1 1 z z
Poles are at, z= , 3 3
2 3
n n
30. (a) 1 2
y(n) = 5 u(n) 5 u(n)
3 3
z4
H(z) =
1
2
1 1
2
1 33. (a)
z + z+ +
2 4 2 4 3
y(n) = i x(n i )
1 1 1 1 i=0
Poles = ± j , ±j
2 2 2 2 = 0x[n]
+ 1x[n – 1] + 2x[n – 2]
z4 +3 x[n – 3]
=
1 1 Y(z) = 0X(z) + 1X(z)z–1 + 2X(z)z–2
z2 z+ z2 + z +
2 2 + 2X(z)z–2 + 3X(z)z–3
z4 H(z) = 0 + 1z–1 + 2z–2 + 3z–3
= 1 2 Null at zero frequency
z 4 + z2 + z
4 j
z= e
=0
z4 1 4
H(z) = =1 z 0 = 0+ 1+ 2+ 3
1 4
z4 +
4 Option (a) satisfies the condition.
So answer is (a).
34. (c)
31. Sol. Poles are at z = ±2j
n Since x[n] is absolutely summable hence, DTFT
y[n] = x[m] = x[n] u[n] exists which implies ROC must include unit
m=0 circle.
z x(t) is a non-causal signal.
Y(z) = X( z)
z 1
104 Electronics Engineering Signals & Systems

35. (b) 39. (d)


Given, x[n] = an u[n] + bnu[n] 1
P(z) = H ( z) H
Also given, 0 < a < b <1 z

ROC = ( z > a ) and ( z > b ) (i) h(n) is real. So, p(n) will be also real.
(ii) P(z) = P(z–1)
ROC = z > b
From (i) : If z1 is a zero of P(z), then z1 will be
36. (d)
also a zero of P(z).
n
Given, x[n] = (2.0) , <n<+ From (ii) : If z1 is a zero of P(z), then 1/z1 will be
n also a zero of P(z).
1
= 2 n u [ n] + u [ n 1] So, the 4 zeros are,
2
1 1
1 z1 = + j
ROC : z > 2 and z < 2 2
2
1 1
No ROC z 2 = z1 = j
2 2
37. (c) 1 1
z3 = = =1 j
Given, x[n] = [n – 3] + 2 [n – 5] z1 1 1
+ j
2 2
X(z) = z–3+ 2z–5
X(–z) = (–z)–3 + 2(–z)–5 1
Y(z) = X(–z) = –z–3 – 2z–5 z4 = = z3 = 1 + j
z1
= –[z–3 + 2z–5]
So, option (d) is correct.
y[n] = –x[n]
40. (c)
38. (b)
3
y(n) = ( 1)k x (n k )
Im
k=0
2 = x(n) – x(n – 1) + x(n – 2) – x(n – 3)
1 Y(z) = X(z) – z–1 X(z) + z–2 X(z) – z–3 X(z)
Y ( z) 1 2 3
H(z) = =1 z +z z
30° X( z)
Re
0.25
z3 z2 + z 1 ( z 1) ( z 2 + 1)
= =
z3 z3
Unit circle Pole zero plot :

Img
1
The ROC should include unit circle to make the 3rd order pole

system stable. From the given pole pattern it is


clear that, to make the system stable, the ROC –1 1
Re
should be finite circular strip or finite annular
strip and hence the impulse response of the
system should be also two sided.
–1
GATE Previous Years Solved Paper 105

41. Sol. By given transfer function,


System is all-pass filter. Zero =
For digital all-pass filter, condition is Using condition (i),

1 1
Zero = ...(i) = = 2
Pole 0.5

Answers
EE Z-Transform

1. (b) 2. (c) 3. (b) 4. (a) 5. (b) 6. (c) 7. (a) 8. (d)

9. (d) 10. (a) 11. (c) 12. (b) 13. (a) 14. (c) 15. (c) 16. (a)

17. (0.29) 18. (0.097) 19. (b) 20. (d) 21. (b) 22. (d)

Solutions
EE Z-Transform

1. (b)
(2 n + 1)
y( n) Y2 ( z) = (5n n) z
z 1 n=0
F(z) = 1 =1 1
z+1 1+ z In Y2(z), powers of ‘z’ are odd. Therefore samples
= (k) – (–1)k u(k) in y2(n) are available only at odd instants of
time.
2. (c)
For causal system, if all the poles are inside the, Hence, x(n) y2 (n) = 0
n=

2 Thus, x(n) is orthogonal to y2(n).


r 2 30°
4. (a)
Given, g(1) = g(2) = 1, otherwise 0
i.e., g[n] = [n – 1] + [n – 2]
Therefore, G(z) = z–1 + z–2
Therefore, overall transfer function of closed-
Unit circle then system is stable, and converse loop system,
in true for anti-causal system. G( z)
T(z) =
3. (b) 1 k G( z)
G( z)
3n 2n T(z) =
x(n) X( z) = z 1 k G( z)
n=0 2+n
z+1
In X(z), power of ‘z’ are even. Therefore, samples or, T(z) = 2
z k( z + 1)
in x(n) are available at even instants of time. By
So, the system will be stable if it’s outer most
observing all the options.
pole will lie inside the unit circle.
106 Electronics Engineering Signals & Systems

Location of poles, For linear phase FIR filter, g(n) should be either
even symmetric or odd symmetric about virtual
+ k ± k2 + 4 k
z= y-axis.
2
To, satisfy the above condition.
k + k2 + 4 k Case (i): =–
<1
2 (odd symmetric about virtual y-axis)
But in this case,
k + k2 + 4 k < 2 G(1) = G(–1) = 0
i.e. at low frequency G(z) = 0
k 2 + 4 k < (2 – k)
and at high frequency G(z) = 0
k2 + 4 k < (2 – k)
Thus in this case filter will be band-pass.
k2 + 4 k < 4 + k2 – 4 k
8k < 4 Case (ii): =
(even symmetric about virtual y-axis)
1
k< In this case:
2
At low frequency (z = 1) G(1) = 2
This condition is satisfied in only option (a).
At low frequency (z = –1) G(–1) = –2
5. (b) Filter will be either all pass or band stop.
X(z) = (1 – 3z–1) Now, G(e j ) = (e–j + e–j3 )
Y(z) = (1 + 2z–1) = e–j2 (ej + e–j )
h[n] = x(n – 1) y[n] = 2 e–j 2 cos
H[z] = z–1 X(z) Y(z)
9. (d)
H[z] = z–1 (1 – 3z–1) (1 + 2z–2)
H[z] = z–1 (1 + 2z–2 – 3z–1 – 6z–3) z
x(z) =
H[z] = (z–1 – 3z–2 + 2z–3 – 6z–4) ( z a)2
when input, zn
I(n) = [n – 1] zn – 1 X(z) =
( z a)2
then, I[z] = z –1
Since, z = a is a pole of second order therefore
Therefore output,
residue at z = a,
P(n) = h[n] I[n]
P(z) = H(z) I(z) 1 d zn
= ( z a)2
P(z) = (z–1 – 3z–2 + 2z–3 – 6z–4) × z– 1 dz ( z a )2
at z = a
1
= [nzn – 1]z = a = nan – 1
P(z) = z–2 – 3z–3 + 2z–4 – 6z–5
P(n) = [n – 2] – 3 [n – 3] + 2 [n – 10. (a)
4] y[n] = x[n] H[n]
– 6 [n – 5] y[z] = x[z] H[z]
y[z] = (4z–3 + 3z–1 + 2 – 6z2 + 2z3)
7. (a)
(3z–1 – 2)
G(z) = z–1 + z–3
= 12z–4 + 9z–2 + 6z–1 – 18z + 6z2
For, low frequency (z = 1) G(1) = + ...(i)
–8z–3 – 6z–1 – 4 + 12z2 – 4z3
For, low frequency (z = 1) G(–1) = –( + )
= –4z3 + 18z2 – 18z + 12z–4 + 9z–2
...(ii)
– 4 – 8z–3
For now, g(n) = {0, , 0, } y[n] 0 for n < 0
Therefore it is non-causal with finite support.
GATE Previous Years Solved Paper 107

11. (c) Applying z-transform on equation (i),


n
zX1(z) – zX1(0) = X1(z)
1
Let, x1[n] = X1(z) [z – 1] = zX1(0)
3
z
n X1(z) = [Given, X2(0) = 0]
1 z 1
and x2[n] = u [ n]
2 ...(iii)

1 n
1 n X 2 ( z) 2
x1[n] = u [ n] + u [ n 1] =
3 3 X1 ( z) z 1
Thus, transfer function,
1 n z 1 1
u [n] ; ROC : z > 2
3 1 1 3 H(z) =
1 z z 1
3
n
Therefore, pole location is z = 1.
1 z 1
u [ n 1] 1
; ROC : z < 3
3 1 1 14. (c)
1 z
3
LTI
x(t) Sampler y(n)
1 n z 1 1 x(n) system
and x2 [n] = u [ n]
1 1
; ROC : z >
2 1 z 2 H(z )
2
f s = 400 Hz
1
ROC is < z < 3. x(t) = 2 + 5 sin(100 t)
2
n n
t xTs = =
12. (b) f s 400
1 n
X(z) = 3
x(n) = 2 + 5sin 100
1 z 400
From z-transform definition,
= 2 + 5 sin n
4
n
X(z) = x[n] z
n= Here, 0 =
4
= x(0) + x(1) z–1 + x(2) z–2 + x(3) x–3... 2 2
Therefore, N = time period of x(n) = = =8
1 /4
X(z) = = 1 + 0 z 1 + 0 z 2 + 1 z 3 ... 0
1 z 3 N
1 1 z
By comparison x(2) = 0 and x(3) = 1. Now, H(z) = 1
N 1 z
13. (a) Applying, z = ej
Given that,
1 1 e j N
X1 (K + 1) a a 1 X1 ( K ) H(ej ) =
N 1 e j
X 2 (K + 1) = a+1 a X 2 (K )
For sinusoidal part of x(n):
With initial conditions,
X1(0) = 1; X2(0) = 0 1 1 e j 0N
H(e j 0 ) =
For a = 1 we can write, N 1 e j 0

X1 (K + 1) 1 0 X1 (K )
= Note: 0N = ×8 = 2
X 2 (K + 1) 2 1 X 2 (K ) 4
X1(K + 1) = X1(K) ...(i) j2 N
1 1 e
= =0
X2(K + 1) = 2X1(K) + X2(K) ...(ii) N 1 e j /40
108 Electronics Engineering Signals & Systems

Therefore for sinusoidal part of input, system


n
output is zero. For dc part of input, Since, n = 2 =
n=0 (1 )2
y(n) = Output
= H(ej0) × Input dc value 1
So, 2 = = 0.29
(1 )2
H(e j ) =1
=0
18. Sol.
= 1×2=2
Pole location of H(z) are given as,
Thus, steady-state output = 2
1 j(2 K 1) /4
z= e ; K = 1, 2, 3, 4
15. (c) 2
x[n] = (–0.25)n u(n) + (0.5)n u(–n – 1) 1 j /4 1
z1 = e = (1 + j )
Signal x[n] is sum of two signals, one is right 2 2
sided [(–0.25)n u(n)] and other is left sided 1 j3 /4 1
z2 = e = [ 1 + j]
[(0.5)n u(–n – 1)]. The right sided signal will have 2 2
pole at location with magnitude 0.25. So, ROC
1 j5 /4 1
z3 = e = [ 1 j]
is z > 0.25. The left sided signal will have pole 2 2
at location with magnitude 0.5. So, ROC is 1 j7 /4 1
z4 = e = [1 j ]
z < 0.5. So, ROC of X(z) (Z-transform of x(n) 2 2
Now,
will be) 0.25 < z < 0.5.
K z4
H(z) = ...(i)
16. (a) ( z z1 ) ( z z2 ) ( z z3 ) ( z z4 )
y(n) is first difference of x(n). [As h(n) is causal and it starts from n = 0, so
So, y(n) = x(n) – x(n – 1) numerator will have same order as denominator
is having].
So, Y(z) = X(z) – Z–1 X(z)
By solving equation (i),
Y(z) = (2z + 4 – 4z–1 + 3z–2)
–(2 + 4z–1 – 4z–2 + 3z–3) Kz 4
H(z) = ...(ii)
1
Y(z) = 2z + 2 – 8z–1 + 7z–2 – 3z–3 z4 +
4
17. Sol. 5
Given that, H(1) =
The Z-transform of 4
From equation (ii),
n 1
u(n) K 5 K
(1 Z 1) H(1) = =
1 4 5
1+
Z 1 4 4
n
and n u(n )
(1 Z 1 )2 25
K =
16
1
Z n n 25 4
so, = n Z z
(1 Z 1 )2 n=0 16
H(z) = 1
If we put Z = 1 in above equation we get, z4 +
4
n n By using division rule,
2 =
(1 ) n=0 25 1 4 1 8
H(z) = 1 z + z + ...
16 4 16
GATE Previous Years Solved Paper 109

1 25 Now, h(n) = overall system impulse response


Thus, h(8) = × = 0.097
16 16 = h1(n) h2(n)
It is given that, = {1, 1} {1, 1}
g(n) = jn h(n)
2
g(8) = j8 h(8) = h(8) = {1, 0, 1} H ( z) = 1 z
g(8) = h(8) = 0.097
At we know,
19. (b) Y ( z) Y ( z)
H(z) = X( z) =
For Fig. (i): X( z) H ( z)

K ( z2 1) 1 + 2z 1 + z 2 z 2 2z 4 z 5
H(z) = =
z2 1 z 2
At low frequency, i.e. z = 1 = 1 + 2z–1 + 2z–2 + z–3
H(1) = 0 x(n) = {1, 2, 2, 1}

At high frequency, i.e. z = –1 21. (b)


H ( 1) = 0 n
1 1
Hence filter type is BPF. x(n) = u(n) ; ROC of x(n) : z >
2 2
For Fig. (ii):
z k
2 x( n k ) X( z) =
K(z 1) 1 1
H(z) = 2
1 z
z + 0.25 2
At low frequency, i.e. z = 1 1
ROC of x(n – k) : z >
H(1) = 0 2
At high frequency, i.e. z = –1 1
For x(n – k) ROC will be z > .
2
H ( 1) = 0
Hence filter type is BPF. 22. (d)
y(n) – ay(n – 1) = b0 x(n) – b1 x(n – 2)
For Fig. (iii):
By applying ZT,
z2 1 Y(z) – az–1 Y(z) = b0 X(z) – b1 z–1 X(z)
H(z) = 2
z +1
Y ( z) b0 b1 z 1
At low frequency, i.e. z = 1 H(z) = =
X( z) 1 az 1
H(1) = 0
By taking right sided inverse ZT,
At high frequency, i.e. z = –1 h(n) = b0an u(n) – b1an – 1 u(n – 1)
H ( 1) = 0 By taking left sided inverse ZT,
h(n) = –b0an u(–n – 1) + b1an – 1 u(–n)
Hence filter type is BPF.
Thus, system is not necessarily causal.
20. (d) The impulse response is non-zero at infinitely
many instants.
x(n) h1(n) h2(n) y(n)

h(n)
7 DTFS, DTFT and DFT

ELECTRO NICS EN GINEERIN G (a) E-3, F-2, G-4, H-1

(GATE Previous Years Solved Papers) (b) E-1, F-3, G-2, H-4
(c) E-1, F-2, G-3, H-4
Q.1 The impulse response h[n] of a linear time (d) E-2, F-1, G-4, H-3
invariant system is given as, [EC-2005 : 2 Marks]
2 2, n = 1, 1
Statement of Linked Answer Questions (4 and 5):
h[n ] = 4 2, n = 2, 2 A sequence x(n) has non-zero values as shown in the
0, otherwise figure.
If the input to the above system is the sequence x(n)
e j n/4, then the output is
2
(a) 4 2 e j n /4 (b) 4 2 e j n /4 1 1

(c) 4e j n/4 (d) –4e j n/4 1/2 1/2


[EC-2004 : 2 Marks]
n
n –2 –1 0 1 2
1 2
Q.2 Let x (n) = u (n), y(n) = x (n) and Y(ej ) be
2
Q.4 The sequence,
the Fourier transform of y(n). Then Y(ejo) is
n
(a) 1/4 (b) 2 x 1 ; for n even
y(n ) = 2
(c) 4 (d) 4/3 0; for n odd
[EC-2005 : 1 Mark]
will be
Q.3 Match the following and choose the correct
combination:
2
Group-I
1 1
E. Continuous and aperiodic signal
F. Continuous and periodic signal 1/2 1/2
(a)
G. Discrete and aperiodic signal
n
H. Discrete and periodic signal –2 0 2 4 6
Group-II
1. Fourier representation is continuous and
aperiodic. 2

2. Fourier representation is discrete and 1 1

aperiodic. 1/2 1/2


(b)
3. Fourier representation is continuous and
periodic. n
–3 –1 1 3 5
4. Fourier representation is discrete and
periodic.
GATE Previous Years Solved Paper 111

Q.8 {x(n)} is a real-valued periodic sequence with a


period N. x(n) and X(k) form N-point Discrete
2
Fourier Transform (DFT) pairs. The DFT Y(k) of
1 1
N 1
1/2 1/2 1
(c) the sequence, y (n) = x(r ) x(n + r ) is
N r =0
n
–6 –4 –2 0 2
2
(a) X( k )

N 1
2 1
(b) X(r ) X( k + r )
1 1 N r =0

1/2 1/2 N 1
(d) 1
(c) X(r ) X( k + r )
N r =0
n
–5 –3 –1 1 3
(d) 0 [EC-2008 : 2 Marks]
[EC-2005 : 2 Marks]
Q.9 A system with transfer function H(z) has
Q.5 The Fourier transform of y(2n) will be impulse response h(n) defined as h(2) = 1,
(a) e–j2 [cos4 + 2 cos2 + 2] h(3) = –1 and h(k) = 0 otherwise. Consider the
(b) [cos2 + 2 cos + 2] following statements:
(c) e–j [cos2 + 2 cos + 2] S1 : H(z) is low-pass filter.
(d) e–j /2 [cos2 + 2 cos + 2] S2 : H(z) is an FIR filter.
[EC-2005 : 2 Marks] Which of the following is correct?
(a) Only S2 is true.
Q.6 A signal x(n) = sin( 0n + ) is the input to a linear
(b) Both S1 and S2 are false.
time-invariant system having a frequency
(c) Both S1 and S2 are true, and S2 is a reason
response H(e j ). If the output of the system
for S1.
Ax(n – n0), then the most general form of H(e j )
will be (d) Both S1 and S2 are true, and S2 is not a reason
for S1.
(a) –n0 0+ for any arbitrary real .
[EC-2009 : 2 Marks]
(b) –n0 0+2 k for any arbitrary integer k.
(c) n0 0+2 k for any arbitrary integer k. Q.10 The 4-point Discrete Fourier Transform (DFT)
(d) –n0 0
of a discrete time sequence {1, 0, 2, 3} is
[EC-2005 : 2 Marks] (a) [0, –2 + 2j, 2, –2 – 2j]
(b) [2, 2 + 2j, 6, 2 – 2j]
Q.7 A 5 point sequence x[n] is given as, x[–3] = 1,
(c) [6, 1 – 3j, 2, 1 + 3j]
x[–2] = 1, x[–1] = 0, x[0] = 5, x[1] = 1. Let X(ej )
denote the discrete-time Fourier transform of (d) [6, –1 + 3j, 0, –1 – 3j]
[EC-2009 : 2 Marks]

x[n]. The value of X( e j ) d is Q.11 For an N-point FFT algorithm with N = 2m, which
one of the following statements is true?

(a) 5 (b) 10 (a) It is not possible to construct a signal flow


graph with both input and output in normal
(c) 16 (d) 5 + j10
order.
[EC-2007 : 2 Marks]
112 Electronics Engineering Signals & Systems

(b) The number of butterflies in the mth state is Q.16 The sequence x[n] = 0.5n u[n] where u[n] is the
N/m. unit step sequence, is convolved with itself to
(c) In place computation requires storage of
only 2 N node data. +
obtain y[n]. Then y[n] is ______ .
(d) Computation of a butterfly requires only one n=
complex multiplication.
[EC-2014 : 1 Mark]
[EC-2010 : 1 Mark]
Q.17 A Fourier transform pair is given by
Q.12 The first six points of the 8-point DFT of a real
Ae + j 6
valued sequence are 5, 1 –j3, 0, 3, –j4, 0 and n f
2 F.T.
u [n + 3]
3 – j4. The last two points of the DFT are 3 2 j2 f
1 e
respectively. 3
(a) 0, 1 – j3 (b) 0, 1 + j3
when u[n] denotes the unit step sequence. The
(c) 1 + j3, 5 (d) 1 – j3, 5
value of A is _______ .
[EC-2011 : 2 Marks]
[EC-2014 : 1 Mark]
Q.13 The DFT of a vector [a b c d] is the vector [ ].
Q.18 The N-point DFT of a sequence x[n], 0 n N – 1
Consider the product:
is given by
a b c d
N 1 2
d a b c 1 j nK
[p q r s] = [ a b c d ] X[K ] = x [n] e N , 0 K N 1
c d a b N n=0
b c d a
Denote this relation as X = DFT(x). For N = 4,
The DFT of the vector [p q r s] is a scaled version
which one of the following sequence satisfies
of
DFT (DFT (x)) = x.
(a) [ 2 2 2 2]
(a) x = [1 2 3 4] (b) x = [1 2 3 2]
(b) (c) x = [1 3 2 2] (d) x = [1 2 2 3]
(c) [ + + + + ] [EC-2014 : 2 Marks]
(d) [ ] [EC-2013 : 2 Marks] Q.19 Two sequence [a, b, c] and [A, B, C] are related
Q.14 Consider a discrete time periodic signal as,

A 1 1 1 a
n
x [n] = sin . Let ak be the complex Fourier 1 2
5 B 1 W3 W3 2 b j
where, W3 = e 3
C 1 W3 2 W3 4 c
series coefficients of x[n]. The coefficients {ak}
are non-zero when k = Bm ± 1, where m is any If another sequence [p, q, r] is derived as,
integer. The value of B is ______ .
[EC-2014 : 2 Marks] p 1 1 1 1 0 0 A /3
q = 1 W31 W32 0 W32 0 B /3
Q.15 An FIR system is described by the system
c 1 W32 W34 0 0 W34 C /3
function:
7 1 3 2 Tthen the relationship between the sequences
H ( z) = 1 + z + z
2 2 [p, q, r] and [a, b, c] is
The system is (a) [p, q, r] = [b, a, c] (b) [p, q, r] = [b, c, a]
(a) maximum phase (b) minimum phase (c) [p, q, r] = [c, a, b] (d) [p, q, r] = [c, b, a]
(c) mixed phase (d) zero phase [EC-2015 : 2 Marks]
[EC-2014 : 1 Mark]
GATE Previous Years Solved Paper 113

Q.20 Consider two real sequences with time-origin Q.25 A continuous-time speech signal xa(t) is sampled
marked by the bold value: at a rate of 8 kHz and the samples are
x1[n] = {1, 2, 3, 0}, x2[n] = {1, 3, 2, 1} subsequently grouped in blocks, each of size N.
Let X1(k) and X2(k) be 4-point DFTs of x1[n] and The DFT of each block to be computed in real
x2[n], respectively. time using the radix-2 decimation-in-frequency
Another sequence x3[n] is derived by taking FFT algorithm. If the processor performs all
4-point inverse DFT of X3(k) = X1(k) X2(k). The operations sequentially, and takes 20 µs for
value of x3[2] is _______ . computing each complex multiplication
(including multiplications by 1 and –1) and the
[EC-2015 : 2 Marks]
time required for addition/subtraction is
n negligible, then the maximum value of N is
1
Q.21 The value of n is ________ . _________ .
n=0 2
[EC-2016 : 2 Marks]
[EC-2015 : 1 Mark]
Q.26 Let h[n] be the impulse response of a discrete-
time linear time invariant (LTI) filter. The
n
Q.22 Let x [n] = 1 + cos be a periodic signal impulse response is given by
8
1 1 1
with period 16. Its DFS coefficients are defined h [0] = ; h [1] = ; h [2] =
3 3 3
15 and h[n] = 0 for n < 0 and n > 2
1
by ak = x [n] exp j kn for all k. The
16 n=0 8 Let H( ) be the Discrete-Time Fourier Transform
(DTFT) of h[n], where is the normalized
value of the coefficient a31 is ______ . angular frequency in radians. Given that,
[EC-2015 : 2 Marks] H( o) = 0 and 0 < o < , the value of o
(in radians) is equal to ________ .
Q.23 Consider the signal:
[EC-2017 : 2 Marks]
x[n] = 6 [n + 2] + 3 [n + 1] + 8 [n] + 7 [n – 1]
+ 4 [n – 2] Q.27 An LTI system with unit sample response
If X(e j ) is the discrete-time Fourier transform of h[n] = 5 [n] – 7 [n – 1] + 7 [n – 3] – 5 [n – 4] is a

1 (a) low pass filter


x[n], then X ( e j ) sin 2 (2 ) d is equal to (b) high pass filter
(c) band pass filter
_______ .
(d) band stop filter
[EC-2016 : 2 Marks]
[EC-2017 : 1 Mark]
Q.24 The Discrete Fourier Transform (DFT) of the
4-point sequence: Q.28 Let X[k] = k + 1, 0 k 7 be 8-point DFT of a

x[n] = {x[0], x[1], x[2], x[3]} = {3, 2, 3, 4} is N 1


j 2 nk / N
sequence x[n], where X [ k ] = x [ n] e .
X[k] = {X[0], X[1], X[2], X[3]} = {12, 2j, 0, –2j} n=0
If X1[k] is the DFT of the 12-point sequence
x1[n] = {3, 0, 0, 2, 0, 0, 3, 0, 0, 4, 0, 0}, the value of The value (correct to two decimal places) of

3
X1 [8]
x [2n] is _________ .
X1 [11] is ________ . n=0

[EC-2016 : 2 Marks] [EC-2018 : 2 Marks]


114 Electronics Engineering Signals & Systems

Q.29 It is desired to find a three-tap causal filter 2


(a) a1 = 1, a2 = W6 , a3 = W6
which gives zero signal as an output to an input
2
of the form (b) a1 = 1, a2 = W6 , a3 = W6

j n j n 2
x [n] = c1 exp + c2 exp (c) a1 = 1, a2 = W6 , a3 = W6
2 2
2
(d) a1 = 1, a2 = W6 , a3 = W6
where c1 and c2 are arbitrary real numbers. The
[EC-2019 : 2 Marks]
desired three-tap filter is given by
h[0] = 1, h[1] = a, h[2] = b Q.31 A finite duration discrete-time signal x[n] is
and h[n] = 0 for n < 0 or n > 2 obtained by sampling the continuous-time
What are the values of the filter taps a and b if signal x(t) = cos(200 t) at sampling instants
the output y[n] = 0 for all n, where x[n] is as t = n/400, n = 0, 1, ...., 7. The 8-point Discrete
given above? Fourier Transform (DFT) of x[n] is defined as,

7 nkn
n=0 j ,
x[n] y[n] = 0 X[ k ] = x [n] e 4 k = 0, 1, ..., 7
h[n] = {1, a, b} n=0

(a) a = 0, b = –1 (b) a = 1, b = 1 Which one of the following statement is true?


(c) a = –1, b = 1 (d) a = 0, b = 1 (a) Only X[2] and X[6] are non-zero.
[EC-2019 : 2 Marks] (b) Only X[3] and X[5] are non-zero.
(c) All X[k] are non-zero.
Q.30 Consider a six-point decimation-in-time Fast
(d) Only X[4] is non-zero.
Fourier Transform (FFT) algorithm, for which
[EC-2020 : 2 Marks]
the signal-flow graph corresponding of X[1] is
Q.32 Consider two 16 point sequences x[n] and h[n].
j2
shown the figure. Let W6 = exp . In the Let the linear convolution of x[n] and h[n] be
6
denoted by y[n], while z[n] denotes the 16 point
figure, what should be the values of the Inverse Discrete Fourier Transform (IDFT) of the
coefficients a1, a2, a3 in terms of W6 so that X[1] is product of the 16 point DFTs of x[n] and h[n].
obtained correctly? The value(s) of ‘k’ for which z[k] = y[k] is/are
(a) k = 15 (b) k = 0, 1, 2, ...., 15
x[0] X[0] (c) k = 0 (d) k = 0 and k = 15
[EC-2021 : 1 Mark]
a1
x[3] X[1]
–1 Q.33 Consider the signals, x[n] = 2n – 1 u[–n + 2] and
y[n] = 2–n + 2 u[n + 1), where u[n] is the unit step
x[1] a2 X[2]
sequence. Let X(ej ) and Y(ej ) be the discrete
time Fourier transform of x[n] and y[n],
x[4] a3 X[3] respectively. The value of the integral,
–1
2
1
x[2] X[4] X( e j )Y ( e j
)d
2
0

(Rounded off to one decimal place) is ______ .


x[5] X[5]
–1 [EC-2021 : 2 Marks]
GATE Previous Years Solved Paper 115

Q.34 For a vector x = [ x[0], x[1],..., x[7]], the 8-port If, x = [1, 0, 0, 0, 2, 0, 0, 0]
discrete Fourier transform (DFT) is denoted by and y = DFT(DFT ( x ))
X = DFT ( x ) = [ X[0], X[1],..., X[7]] then the value of y[0] is ______ (Rounded off to
one decimal place).
7
2
where, X[ k ] = x[n] exp j nk [EC-2022]
n=0 8

Here, j= 1

Electronics & Electrical Engineering


GATE Previous Years Solved Paper

A n swe rs & Expl a n a t i o n s

Answers
EC DTFS, DTFT and DFT

1. (d) 2. (d) 3. (c) 4. (a) 5. (c) 6. (b) 7. (b) 8. (a)

9. (a) 10. (d) 11. (d) 12. (b) 13. (a) 14. (10) 15. (c) 16. (4)

17. (3.375) 18. (b) 19. (c) 20. (11) 21. (2) 22. (0.5) 23. (8) 24. (6)

25. (4096) 26. (2.10) 27. (c) 28. (3) 29. (d) 30. (d) 31. (a) 32. (a)

33. (8) 34. (8)

Solutions
EC DTFS, DTFT and DFT

1. (d) x(n) (n – a) x(n – a)

h(n) = 4 2 (n + 2) 2 2 (n + 1) y(n) = 4 2 [ e j /4( n+ 2) + e j /4( n 2) ]

/4( n + 1)
2 2 (n 1) + 4 2 (n 2) 2 2 [e j + ej /4( n 2)
]

x(n) = ej n/4 jn /4
= e [4 2 ( e j /2 + e j /2 )
y(n) = x(n) h(n)
= 4 2 (n + 2) e jn /4 2 2 (e j /4
+e j /4
)]

+ 4 2 (n 2) e jn /4
= e
jn /4
[0 2 2 × 2 cos /4]

2 2 [ ( n + 1) e jn /4 y(n) = –4e jn /4

+ (n 1) e jn /4
]
116 Electronics Engineering Signals & Systems

2. (d) j 1 2j 1
= e e + ej + 2 + e j
+ e 2j
n 2 2
1
x(n) = u(n)
2 j e2 j + e 2 j
= e + ej + e j + 2
2n 2
1
y(n) = x 2 (n) = u2 ( n )
2 f(n) = e–j [cos2 + 2cos + 2]
2 n
1 6. (b)
= u(n)
2 • sin( on + ) = x(n) X(ej )
n • y(n) = Ax(n – no)
1
y(n) = u(n)
4 Y(ej ) = Ae jno X ( e j )

1 Y(e j ) jno
Y(z) = • H(ej ) = = Ae
1 1 X( e j
)
1 z
4
• H(ej ) = –no o+2 k (at = o)
Put, z = e j
1 7. (b)
Y(e j ) =
1 j X(ej ) = e3j + e2j + 0 + 5 + e–j
1 e
4
1 4 Q e aj d =0 if a 0
Y(ejo) = =
1 3
1
4
e3 j e2 j e j
X( e j ) d = + +5 +
4. (a) 3j 2j j

n = 5 + 5 = 10
y(n) = x 1 , n even
2
9. (a)
= 0 for n odd
h(2) = 1
n = 0, y(n) = x(–1) = 1
h(3) = –1
n = 2, y(n) = x(0) = 2
h(k) = 0 otherwise
n = 4, y(n) = x(1) = 1
n = 6, y(n) = x(2) = 1/2 h(k)

5. (c) 1

y(2n) = x(n – 1) seeing in graph k


2
f(n) = y(2n) –1

1
= (n + 1) + (n) + 2 (n 1) It is finite impulse response. It is not low-pass
2
filter.
1
+ (n 2) + (n 3)
2 10. (d)
Taking z-transform, 4-point DFT of sequence [1, 0, 2, 3] is given as,
1 1
F(z) = z + 1 + 2z 1 + z 2 + z 3 1 1 1 1 1 1+2 +3 6
2 2
1 j 1 j 0 1 2 + 3j 1+ 3j
z = ej = =
1 1 1 1 2 1+2 3 0
1 j j 2j 1 3j 1 j 1 j 3 1 2 3j 1 3j
F(e j ) = e + 1 + 2e +e + e
2 2
GATE Previous Years Solved Paper 117

11. (d) 15. (c)


For a N-point FFT algorithm with N = 2 m, Minimum phase all zeros are inside the unit
computation of a butterfly requires only one circle.
complex multiplication and two complex • For maximum phase, all the zeroes are
additions. outside the unit circle.

12. (b) 7 1 3 2
• H ( z) = 1 + z + z =0
x[n] real 2 2
X[k] conjugate symmetric 2z2 + 6z + z + 3 = 0
X[k] X [N – k] 1
z1 = (inside the unit circle)
N = 8 2
X[k] = X [8 – k] z2 = –3 (outside the unit circle)
X[6] = X [8 – 6] = X [2] = 0 Mixed phase
X[7] = X [8 – 7] = X [1] = 1 + j3
16. Sol.
13. (a) Given, x[n] = 0.5n u[n]
DFT [p q r s] = [a b c d] [a b c d] Z-transform of x(n) is
So, DFT [p q r s] = [ 2 2 2 2]
1
X(z) =
14. Sol. (1 0.5 z 1 )
From discrete Fourier series definition, and Z-transform of y(n) = x(n) x(n) is
N 1 2
jk
N
n 1
x[n] = ak e Y(z) =
k=0 (1 0.5 z 1 )2

2 2
j n j n
= a 1e N + a0 + a1 e N ...(i) y( n) = Y ( z ) z = 1 = 4
n=
1 j n 1 j 5n
x[n] = e 5 + e
2j 2j 17. Sol.
Also given, n
2 Ae j 6 f
u(n + 3)
n 3 2
x[n] = sin 1 e j2 f
5 3
2 n
where, N = 10 = N = 10 2
N 5 Let, x[n] = u(n + 3)
3
By comparison,
Discrete Fourier transform is given as,
1 1
a1 = and a9 = a 1 =
2j 2j j n
X(e j ) = x[n] e
As the TFS coefficient ak is also periodic with n=

period N = 10, n
2 j n
a 1 = a11 = a21 .... = e
n= 3 3
a –1 = a9 = a19 ....
Given that, ak is non-zero for k = Bm ± 1. Put, n+3 = m
To satisfy this condition B must be ‘10’ where
‘m’ is any integer.
118 Electronics Engineering Signals & Systems

m 3 2
2 j ( m 3) j
So, X(e j ) = e where, W31 = e 3
m=0 3
4
j
3 m
2 2 j m W32 = e 3
= e ej 3
3 m=0 3 2
j
3 m
W3 1 = e 3
3 2 j
= e ej 3
j
4
2 3 2 3
m=0 W3 = e
j2 f 3 j6 f 2
3.375 e 3.375 e j
= = W3 4 = W3 1 = e 3
2 j2 f 2 j2 f
1 e 1 e
3 3
p c
So, A = 3.375
q = a
18. (b) r b
DFT (DFT (x)) = x
Let, x = [1 2 3 2] 20. Sol.

1 1 1 1 1 x1[n] = {1, 2, 3, 0}
x2[n] = {1, 3, 2, 1}
1 1 j 1 j 2
DFT (x) = X(k)3 = X(k)1 X(k)2
4 1 1 1 1 3
x3[n] = circular convolution of
1 j 1 j 4
x1[n] and x2[n]
8 4 = x1[n] x2[n]
1 2 1
= = x3 (0) 1 0 3 2 1 9
2 0 0
x3 (1) 2 1 0 3 3 8
2 1 = =
x3 (2) 3 2 1 0 2 11
1 1 1 1 4 x3 (3) 0 3 2 1 1 14
1 1 j 1 j 1
DFT (DFT (x)) = x3[2] = 11
4 1 1 1 1 0
1 j 1 j 1 21. Sol.

2 1 n n
1 1
= n u(n)
1 4 2 2 n= 2
= = n=0
2 6 3
4 2 n
1
= Z T n u(n)
2
z=1
19. (c)

A a+b+c
1 1 z
z
B = a + bW3 1 + cW3 2 2 2
= = 2
1 1
C a + bW3 2 + cW3 1 1 z z
1
2 2 z=1
a+b+c
W32 W31 3
p 1
a + bW3 1 + cW3 2 1/2
q = 1 W31 W32 W32 W31 = =2
3 1/4
r 1 W32 W32 W31 W31
a + bW3 2 + cW3 1
3
GATE Previous Years Solved Paper 119

22. Sol. 24. Sol.


n From the given question,
x [n] = 1 + cos
8 n
x1[n] = x
n n 3
j j
e 8 e 8
= 1+ + (N = 16) X1[k] = {12, 2j, 0, –2j, 12, 2j, 0, –2j}
2 2 X1[8] = 12 ; X1(11) = –2j
2 2
j n j n X1 (8) 12
e 16 e 16 = =6
= 1+ + X1 (11) 2j
2 2
1 1 25. Sol.
a1 = and also a31 = a15 = a 1 =
2 2
f s = 8000 samples/sec
23. Sol. 1 1
Ts = = sec.
From the definition of DTFT, f s 8000

j n
Time for each multiplication = Tm = 20 µsec
X(ej ) = x[n] e Tm = 20 × 10–6 sec
n=
Suppose block is size N, then time taken to
1
x[n] = X( e j ) e j n
d 1
2 generate each block = N × Ts = N × .
8000
1 Number of multiplication that can be performed
x[0] = X( e j ) d by processor in the time taken for each block =
2
N × Ts
1 j j
.
X( e ) Y(e )d = [x(n) y(n)]n = 0 Tm
2
Number of multiplication required to compute
Y(ej ) = sin2 (2 )
N
1 cos 4 DFT by Radix – 2 FFT algorithm = log 2 N .
= 2
2
Number of multiplication required by FFT
1 1 4j 1 4j
= e e Number of multiplication that can be performed
2 4 4
by the processor in the time taken in each block.
1 1 1
y[n] = [ n] [n + 4] [n 4] [Reason : Real time]
2 2 4
N N × Ts
log 2 N
1 1 1 2 Tm
y[n] = , 0, 0, 0, , 0, 0, 0,
4 2 4 1

log2 N 8000 = 12.5
1 20 × 10 6
y[0] =
2 N 212.5 N 212 = 4096
x[n] = {6, 3, 8, 7, 4} ; x[0] = 8
26. Sol.
1 1 1 1
X( e j ) Y ( e j ) d = 2 x[0] y[0] Since, h[n] = [ n] + [n 1] + [n 2]
3 3 3
n=
1 j
1 So, H(e j ) = e [1 + 2 cos ]
= 2 ×8× = 8 3
2
H(e j o) = 0 ; for (1 + 2 cos o) = 0
120 Electronics Engineering Signals & Systems

1 To get y(n) = 0,
or, cos o =
2 = H(e j )
H(e j o) =0
2 = /2
or, o = = 2.10 radians
3 j j2
1 + ae + be 2 = 0
2
27. (c) 1 – ja – b = 0
h[n] = 5 [n] – 7 [n – 1] + 7 [n – 3] – 5 [n – 4] From the given options, a = 0 and b = 1.
Now, H(e j ) = 5 – 7e–j + 7e–3j – 5e–4j
Now for = 0, 30. (d)
H(e jo) = 5 – 7 + 7 – 5 = 0 N 1 2
j kn
and for = , X(k) = x(n) e N
H(e j ) = 5 – 7(–1) + 7(–1) – 5(1) n=0
= 5+7–7–5=0 5
System is attenuating low and high frequencies X(1) = x(n) W6n
whereas passing the mid frequencies. So, its a n=0

BPF. 2 3
= x(0) + x(1) W6 + x(2) W6 + x(3) W6
28. Sol. + x(4) W64 + x(5) W65 ...(i)
X(k) = {1, 2, 3, 4, 5, 6, 7, 8} From the given flow graph,
7 2 X(k) = [x(0) – x(3)] a1 + [x(1) – x(4)] a2
j kn
X(k) = x[n] e 8
+ [x(2) – x(5)] a3
n=0
By comparing equations (i) and (ii), we get,
7 j kn 2
a 1 = 1, a2 = W6 , a3 = W6
= x[n] e 4
n=0
31. (a)
3
x[2n] = x[n] x(t) = cos200 t
n=0 n = 0, 2, 4, 6 n
t=
400
1 7
= ( x[n] + ( 1)n x[n]) n
2 n=0 x(n) = cos 200 = cos n
400 2
7
n = 0, 1, ... , 7
x[n] = X(0) = 1
n=0 3
= cos 0, cos , cos , cos ;
2 2
7 7 j 4n
n x[n] e 4 = X(4) = 5 5 7
( 1) x[n] = cos 2 , cos , cos 3 , cos
n=0 n=0 2 2
3 DFT
1 = {1, 0, –1, 0, 1, 0, –1, 0} X( k )
x[2n] = [1 + 5] = 3
n=0
2 Suppose,
DFT
29. (d) y(n) = {1, –1, 1, –1} Y( k )
[Y(k)] = [ WN ] N = 4 [ y(n)]
j n j n
x(n) = c1 e 2 + c2 e 2
1 1 1 1 1
rad/sec.
= 1 j 1 j 1
o 2 = = [0, 0, 4, 0]
1 1 1 1 1
H(e j ) = 1 + ae–j + be–j2
1 j 1 j 1
GATE Previous Years Solved Paper 121

Now, as we know, if for 1 4 3 2 0


DFT 2 1 4 3 1
{a, b, c, d} { A, B, C , D}
=
3 2 1 4 2
DFT
Then for {a, 0, b, 0, c, 0, d, 0} {A, B, C, D, 4 3 2 1 3
A, B, C, D} z(n) = {16, 18, 16, 10 }
Similarly, for z(3) = 10
DFT Here, y(n) and z(n) are equal for n = 3
y(n) = {1, –1, 1, –1} Y(k) = {0, 0, 4, 0}
[n = N – 1 = 4 – 1]
Here, for x(n) = {1, 0, –1, 0, 1, 0, –1, 0}
where, ‘N’ is length of the sequences.
X(k) = {0, 0, 4, 0, 0, 0, 4, 0}
33. (8)
Given:
32. (a)
F.T.
Given, y(n) = x(n) h(n) x(n) = 2 n 1
u( n + 2) X( e j )
where, ‘ ’ represents circular convolution n+2 F.T.
y(n) = 2 u( n + 1) Y(e j )
Given, z(n) = IDFT [X(k) H(k)]
For any real signal,
z(n) = x(n) N h(n) X(e j ) = X (e–j )
X (e j ) = X(e–j )
where, N represents circular convolution 2
1
Value of X( e j )Y ( e j
)d
The length of linear convolution is N1 + N2 – 1. 2
0
Given, 16 point sequences 2
1
So, N1 + N2 – 1 = 31 = X( e j )Y ( e j ) d
y(n) length is 31. 2
0
The length of circular convolution is max(N1, N2) From Plancherel’s theorem
= max(16, 16) = 16 +
So, z(n) length is 16. = x(n) y (n)
n=
z(k) and y(k) are equal for k = 15[16 – 1]
i.e., z(15) = y(15) Overlap of the signals x(n) and y(n) in the range
For your understanding purpose, consider two –1 to +2,
4-point sequences as: x(n) = 2n – 1; n 2
x(n) = {1, 2, 3, 4} y(n) = 2–n + 2; n –1
h(n) = {0, 1, 2, 3} 2 2
n+2
= 2n 1
2 = 2
y(n) = x(n) h(n) n= 1 n= 1
0 1 2 3 = 2[1 + 1 + 1 + 1] = 8
1 0 1 2 3
34. (8)
2 0 2 4 6
= y(n) = DFT [DFT (x(n))]
3 0 3 6 9
Using duality property,
4 0 4 8 12
DFT DFT
x( n ) Nx( k ) = y( n)
y(n) = {0, 1, 4, 10 , 16, 17, 12}
y(n) = Nx(–k) = Nx(N – k)
y(3) = 10 y(n) = 8(1, 0, 0, 0, 2, 0, 0, 0)
z(n) = x(n) N h(n) At x = 0, y(0) = 8 × 1 = 8
8 Sampling

ELECTRO NICS EN GINEERIN G (b) 800 Hz and 900 Hz components.


(c) 800 Hz and 1000 Hz components.
(GATE Previous Years Solved Papers)
(d) 800 Hz, 900 Hz and 1000 Hz components.
Q.1 A signal containing only two frequency [EC-1995 : 1 Mark]
components (3 kHz and 6 kHz) is sampled at
the rate of 8 kHz, and then passed through a Q.4 Flat top sampling of low pass signals
low pass filter with a cut-off freuqency of 8 kHz. (a) gives rise to aperture effect
The filter output (b) implies oversampling
(a) is an undistorted version of the original (c) leads to aliasing
signal. (d) introduces delay distortion
(b) contains only the 3 kHz component. [EC-1998 : 1 Mark]
(c) contains the 3 kHz component and a
Q.5 Let x(t) = 2 cos(800 t) + cos(1400 t). x(t) is
spurious component of 2 kHz.
sampled with the rectangular pulse train shown
(d) contains both the components of the original
in the figure. The only spectral components
signal and two spurious components of
(in kHz) present in the sampled signal in the
2 kHz and 5 kHz.
frequency range 2.5 kHz to 3.5 kHz are
[EC-1988 : 2 Marks]
p (t )
Q.2 Increased pulse-width in the flat-top sampling,
leads to 3

m(t ) M(f )

t
–T0 –T0/6 0 T0/6 T0
–3
f T = 10 sec
–W 0 W
(a) 2.7, 3.4
(a) attenuation of high frequencies in
reproduction. (b) 3.3, 3.6
(b) attenuation of low frequencies in (c) 2.6, 2.7, 3.3, 3.4, 3.6
reproduction. (d) 2.7, 3.3
(c) greater aliasing errors in reproduction. [EC-2003 : 2 Marks]
(d) no harmful effects in reproduction.
Q.6 A 1 kHz sinusoidal signal is ideally sampled at
[EC-1994 : 1 Mark] 1500 samples/sec and the sampled signal is
Q.3 A 1.0 kHz signal is flat-top sampled at the rate passed through an ideal low-pass filter with
of 1800 samples/sec and the samples are cut-off frequency 800 Hz. The output signal has
applied to an ideal rectangular LPF with cut-off the frequency
frequency of 1100 Hz, then the output of the (a) 0 Hz (b) 0.75 Hz
filter contains (c) 0.5 kHz (d) 0.25 kHz
(a) only 800 Hz component. [EC-2004 : 2 Marks]
GATE Previous Years Solved Paper 123

Q.7 A signal m(t) with bandwidth 500 Hz is first (a) 5 Hz and 15 Hz only
multiplied by a signal g(t) where, (b) 10 Hz and 15 Hz only
(c) 5 Hz, 10 Hz and 15 Hz only
g(t ) = ( 1)k (t 0.5 × 10 4
k) (d) 5 Hz only
k=
[EC-2014 : 1 Mark]
The resulting signal is then passed through an
ideal low pass filter with bandwidth 1 kHz. The Q.12 For a given sample-and-hold circuit, if the value
output of the low pass filter would be of the hold capacitor is increased, then
(a) (t) (b) m(t) (a) drop rate decreases and acquisition time
decreases.
(c) 0 (d) m(t) (t)
(b) drop rate decreases and acquisition time
[EC-2006 : 2 Marks]
increases.
Q.8 An LTI system having transfer function (c) drop rate increases and acquisition time
decreases.
s2 + 1
and input x(t) = sin(t + 1) is in steady- (d) drop rate increases and acquisition time
s2 + 2s + 1
increases.
state. The output is sampled at a rate s rad/sec [EC-2014 : 1 Mark]
to obtain the final output {y(k)}. Which of the
following is true?
Q.13 The signal cos 10 t + is ideally sampled at
(a) y is zero for all sampling frequencies s. 4
(b) y is non-zero for all sampling frequencies
a sampling frequency of 15 Hz. The sampled
s. signal is passed through a filter with impulse
(c) y is non-zero for s > 2 but zero for s < 2.
(d) y is non-zero for > 2 but non-zero for sin( t )
s response cos 40 t . The filter
t 2
s < 2.
[EC-2009 : 2 Marks] output is
Q.9 A band-limited signal with a maximum 15
(a) cos 40 t
frequency of 5 kHz is to be sampled. According 2 4
to the sampling theorem, the sampling frequency
15 sin( t )
which is not valid is (b) cos 10 t +
2 t 4
(a) 5 kHz (b) 12 kHz
15
(c) 15 kHz (d) 20 kHz (c) cos 10 t
2 4
[EC-2013 : 1 Mark]
15 sin( t )
Q.10 Consider two real valued signals, x(t) band- (d) cos 10 t
2 t 2
limited to [–500 Hz, 50 Hz] and y(t) band-limited
[EC-2015 : 1 Mark]
to [–1 kHz, 1 kHz]. For z(t) = x(t) y(t), the Nyquist
sampling frequency (in kHz) is ______ . Q.14 Consider a continuous time signal defined as,
[EC-2014 : 1 Mark]
sin t /2
x(t ) = (t 10 n)
Q.11 Let x(t) = cos(10 t) + cos(30 t) be sampled at t /2 n=
20 Hz and reconstructed using an ideal low-
where ‘ ’ denotes the convolution operation
pass filter with cut-off frequency of 20 Hz. The
and ‘t’ is in seconds. The Nyquist sampling rate
frequency/frequencies present in the
(in samples/sec) for x(t) is _______ .
reconstructed signal is/are
[EC-2015 : 2 Marks]
124 Electronics Engineering Signals & Systems

Q.15 A continuous-time sinusoid of frequency 33 Hz ELECTRICAL EN GINEERIN G


is multiplied with a periodic Dirac impulse
(GATE Previous Years Solved Papers)
train of frequency 46 Hz. The resulting signal is
passed through an ideal analog low-pass filter Q.1 The frequency spectrum of a signal is shown in
with a cut-off frequency of 23 Hz. The the figure. If this is ideally sampled at intervals
fundamental frequency (in Hz) of the output is of 1 ms, then the frequency spectrum of the
________ . sampled signal will be
[EC-2016 : 1 Mark] |U(j )|
Q.16 Consider the signal x(t) = cos(6 t) + sin(5 t),
where ‘t’ is in seconds. The Nyquist sampling
rate (in samples/second) for the signal
y(t) = x(2t + 5) is 1 kHz
(a) 8 (b) 12
(c) 16 (d) 32 |U(j )|

[EC-2016 : 1 Mark]

Q.17 The signal x(t) = sin(14000 t), where ‘t’ is in (a)


seconds is sampled at a rate of 9000 samples
per second. The sampled signal is the input to
an ideal low-pass filter with frequency response
H(f ) as follows:
1, f 12 kHz (b)
H( f ) =
0, f > 12 kHz
What is the number of sinusoids in the output
and their frequencies (in kHz)?
(a) Number = 1, frequency = 7
(b) Number = 3, frequencies = 2, 7, 11
(c)
(c) Number = 2, frequencies = 2, 7
(d) Number = 2, frequencies = 7, 11
[EC-2017 : 2 Marks]

Q.18 A band-limited low-pass signal x(t) of


bandwidth 5 kHz is sampled at a sampling rate
(d)
fs. The signal x(t) is reconstructed using the
reconstruction filter H(f ) whose magnitude
response is shown below.
[EE-2007 : 1 Mark]
H( f )
K Q.2 A band-limited singal with a maximum
frequency of 5 kHz is to be sampled. According
to the sampling theorem, the sampling frequency
in kHz which is not valid is
f (kHz)
–8 –6 0 6 8 (a) 5 (b) 12
The minimum sampling rate fs (in kHz) for (c) 15 (d) 20
perfect reconstruction of x(t) is ______ . [EE-2013 : 1 Mark]
[EC-2018 : 2 Marks]
GATE Previous Years Solved Paper 125

Q.3 A sinusoidal x(t) of unknown frequency is (a) 2B1 (b) 2(B1 + B2)
sampled by an impulse train of period 20 ms. (c) 4(B1 + B2) (d)
The resulting sample train is next applied to an [EE-2016 : 2 Marks]
ideal low-pass filter with a cut-off at 25 Hz. The
filter output is seen to be a sinusoid of frequency Q.6 The output y(t) of the following system to be
20 Hz. This means that x(t) has a frequency of sampled, so as to reconstruct it from its samples
uniquely. The required minimum sampling rate
(a) 10 Hz (b) 60 Hz
is
(c) 30 Hz (d) 90 Hz
X( )
[EE-2014 : 2 Marks]

Q.4 For the signal f(t) = 3 sin8 t + 6 sin12 t + sin14 t,


the minimum sampling frequency (in Hz)
–1000 1000
satisfying the Nyquist criterion is _____ . sin (1500 t ) y(t)
h (t ) =
x (t ) X( ) t
[EE-2014 : 1 Mark]

Q.5 Let x1(t) X1( ) and x2(t) X2( ) be two cos(100 t)


signals whose Fourier transforms are shown in (a) 1000 samples/sec
the figure below. In the figure, h(t ) = e 2 t (b) 1500 samples/sec
(c) 2000 samples/sec
denotes the impulse response.
(d) 3000 samples/sec
X1( ) X 2( )
[EE-2017 : 2 Marks]

Q.7 Consider the two continuous-time signals


defined below:
These signals are sampled with a sampling
–B1 B1 B1 B1 –B2 B2
period of T = 0.25 seconds to obtain discrete-
2 2
time signals x1[n] and x2[n], respectively. Which
X1(t ) one of the following statements is true?
–2|t| (a) The energy of x1[n] is greater than the energy
h (t ) = e y (t )
of x2[n].
X2(t ) (b) The energy of x2[n] is greater than the energy
For the system shown above, the minimum of x1[n].
sampling rate required to sample y(t), so that (c) x1[n] and x2[n] have equal energies.
y(t) can be uniquely reconstructed from its (d) Neither x1[n] nor x2[n] is finite energy signal.
samples, is [EE-2018 : 2 Marks]
126 Electronics Engineering Signals & Systems

Electronics & Electrical Engineering


GATE Previous Years Solved Paper

A n swe rs & Expl a n a t i o n s

Answers
EC Sampling

1. (d) 2. (a) 3. (c) 4. (a) 5. (d) 6. (c) 7. (b) 8. (a)

9. (a) 10. (3) 11. (a) 12. (b) 13. (a) 14. (0.4) 15. (13) 16. (c)

17. (b) 18. (13)

Solutions
EC Sampling

1. (d) 3. (c)
f s = 8000 samples/sec f s = 1800 samples/sec
fm1 = 3 kHz fm = 1000 Hz
fm2 = 6 kHz The spectrum of sampled signal would have
The spectrum of sampled signal would have nfs ± fm
nfs ± fm So, 1000 Hz, 1800 ± 1000 Hz, 3600 ± 1000 Hz ...
So, 3 kHz, 8 ± 3, 16 ± 3, ... = 3 kHz, 5 kHz, So, 1000 Hz, 800 Hz, 2800 Hz, 2600 Hz,
11 kHz, ... 4600 Hz, ...
6 kHz, 8 ± 3, 16 ± 6, ... = 6 kHz, 2 kHz, The cut-off frequency of LPF is 1100 Hz.
14 kHz, ... So, the output of filter will contains 800 Hz and
Cut-off frequencies of LPF = 8 kHz 1000 Hz components.
So, the filter output would have 3 kHz, 6 kHz,
4. (a)
2 kHz and 5 kHz.
Flat top sampling of low pass signals gives rise
2. (a) to aperture effect.

x(t ) X( f ) = sin c ( f ) 5. (d)


From Fourier series expansion,
1 F.T.
To /6
1 jn ot
t f Cn = Ae dt
To
To /6
As pulse width is increased, the width 1/ of
the first lobe of the spectrum is decreased. A n
= sin
Hence, increased pulse-width in the flat-top n 3
sampling, leads to attenuation of high From Cn its clear that 1, 2, 4, 5, 7,... harmonics
frequencies in reproduction. are present.
GATE Previous Years Solved Paper 127

Frequency of p(t) corresponding to 1, 2, 4, 5, 7, ... x(t) = sin(t + 1)


are 103, × 2 × 103, 4 × 103.... = 12 rad/sec
x(t) has frequency components 0.7 k and 0.4 k 1 es
p(t) × x(t) gives (1 ± 0.7) k, (2 ± 0.7) k, (4 ± 0.7) k, X(s) = 2
es = = 2
+ s2 12 + s 2 s +1
(1 ± 0.4) k, (2 ± 0.4) k, ....
Y (s )
Frequency present in range of 2.5 k to 3.5 are H(s) =
X( s )
2.7, 3.3.
Y(s) = H(s) X(s)
6. (c)
s2 + 1 es
f s = 1.5 kHz Y(s) = ×
s2 + 2s + 1 s2 + 1
fm = 1 kHz
es
Available frequency components are nfs ± fm. =
So, 1 kHz, 2.5 kHz, 0.5 kHz, .... s2 + 2s + 1
Q LPF has fc = 0.8 k es
Y(s) =
only 0.5 k will appear at output. (s + 1)2
y(t) = (t + 1) e–(t + 1)
7. (b)
ses
M(f ) y( ) = lim sY (s) = lim =0
s 0 s 0 ( s + 1)2

So, at steady-state ‘y’ remains zero for all


sampling frequencies s.
f
–500 500
9. (a)
m(t) g(t) M(f ) G(f)
(fs)min = 2fm
1 (fs)min = 2 × 5 = 10 kHz
4 = 20 kHZ
0.5 × 10 So, f s 10 kHz
G(f ) 10. Sol.
Multiplication in time domain = convolution in
frequency domain,
x1(t) x2(t) X1( ) X2( )
f
–20 kHz 20 kHz So, highest frequency component contained by
the convolved signal z(t) = 1500 Hz.
M(f ) G(f ) Nyquist rate = 2 × 1500
= 3000 Hz = 3 kHz

11. (a)
f
x(t) = cos(10 t) + cos(30 t)
–20 kHz 20 kHz
Given sampling frequency,
After low pass filtering with fc = 1 kHz f s = 20 Hz
Output is zero.
s = 40 rad/sec.

8. (a) X( )

s2 + 1
X(s) H (s ) = X(s )
2
s + 2s + 1
–30 –10 10 30
128 Electronics Engineering Signals & Systems

1 (f 20) (f + 20) j
Y( ) = X( k o) H(f ) = rect + rect e 2
2 2 2
k=
After sampling waveform will be Output, Y(f ) = Xs(f ) × H(f )
15
Y( ) y(t) = cos 40 t
2 4

14. Sol.
–70 –50 –30 –10 10 30 50 70 t
sin
–40 40 2
x(t) = [t 10 n]
t n=
Applying an ideal low-pass filter of cut-off
2
frequency of 20 Hz or 40 rad/sec, we get the
frequencies in reconstructed signal as, 10 and
30 rad/sec. or 5 Hz and 15 Hz.
X(f ) = ×
12. (b) f
–0.1 0 0.1 0.2 0.3
–1/4 1/4
We know that, fmax = 0.2 Hz
Q = CV = i t ...(i) f s = 2fmax = 0.4 Hz
CV
t= ...(ii) 15. Sol.
i
If x(t) is a message signal and y(t) is a sampled
From equation (ii),
signal, then y(t) is related to x(t) as,
t C and it is clear that if value of capacitor
increases then the acquisition time increases.
y(t) = x(t ) (t nTs )
For capacitor the drop rate is given as dv/dt, n=
dv
i= C
dt Y(f ) = f s X( f nTs )
dv i n=
=
dt C Spectrum of X(f ) and Y(f ) are as shown
dv 1 X(f )
dt C
From above relation it is clear that if capacitor
value increases drop rate decreases.
f
13. (a) –33 Hz 0 33 Hz

x(t) = cos 10 t +
4 Y(f )

1
[ f 5) + ( f + 5)] e j
X(f ) =
/4
2
After sampling x(t) by 15 Hz, we get –79 Hz –33 Hz –13 Hz 13 Hz 33 Hz 79 Hz
f

Xs(f ) = 15 X( f 15n) –23 Hz 23 Hz


n=
Cut-off frequency of LPF = 23 Hz
sin( t ) Hence, frequency at the output is 13 Hz.
Given, h(t) = cos 40 t
t 2
GATE Previous Years Solved Paper 129

16. (c) So, three sinusoids will be there at the output of


X(t) = cos(6 t) + sin(8 t) the LPF and the frequencies of those sinusoids
Y(t) = x(2t + 5) are 2 kHz, 7 kHz and 11 kHz.
Y(t) = cos[6 (2t + 5) + sin(8 (2t + 5)]
18. Sol.
= cos(12 t + 30 ) + sin(16 t + 40 )
Let us assume an arbitrary spectrum for x(t) as
fm1 = 6 HZ, fm2 = 8 Hz
shown below.
Nyquist sampling rate,
f s = 2 fmax X(f )

= 16 samples/second

17. (b)
x(t) = sin(14000 t) f (kHz)
–5 0 5
fm = 7 kHz
f s = 9000 samples per second The spectrum of the sampled signal can be given
= 9 kHz as,
The spectrum of the sampled signal can be given
as shown below.
S(f )
Frequency response of
LPF with fc = 12 kHz f (kHz)
1 –fs – 5 –fs –fs + 5 –8 –6 –5 0 5 6 8 fs – 5 –fs fs + 5

For proper reconstruction of the signal,


fs – 5 8
–25 –16 –11 –7 –2 0 2 7 11 16 25 f(kHz) f s 8 + 5 = 13 kHz
–12 12
So, fs(min) = 13 kHz

Answers
EE Sampling

1. (b) 2. (a) 3. (c) 4. (14) 5. (b) 6. (b) 7. (a)

Solutions
EE Sampling

1. (b) Therefore sampling frequency,


1
|U(j )| fs =
Ts
1
= 1 kHz
=
10 3
After sampling new signal in frequency domain,
–1 kHz 1 kHz
Given that, sampling interval = 1 msec 1
UT(f ) = U( f nf s )
i.e., Ts = 1 msec = 10–3 sec Ts n=
130 Electronics Engineering Signals & Systems

Spectrum of sampled signal will be Then the minimum sampling frequency


satisfying the Nyquist criterion is = 14 Hz.
|UT(j )|
Method-II:
Sampling frequency = 2 (Highest frequency
component of f(t)).
Given, f(t) = 3 sin8 t + 6 sin12 t + sin14 t
Different frequency components are:
2. (a) 1=8 :

(fs)min = 2 fm 8
f1 = = 4 Hz
(fs)min = 2 × 5 = 10 kHz 2
So, f s 10 kHz 2 = 12 :
12
3. (c) f2 = = 6 Hz
2
Given, impulse train of period 20 ms. and = 14 :
2
1 14
Then, sampling frequency = 3
= 50 Hz f3 = = 7 Hz
20 × 10 2
If the input signal x(t) = cos m (t) having Clearly, highest frequency component of the
spectrum. signal,
X(f ) f(t) = f3 = 7 Hz
Minimum sampling frequency,
f s = 2 × f3
= 2 × 7 = 14 Hz
f
–fm fm 5. (b)
The filtered out sinusoidal signal has 20 Hz Given that:
frequency has sampling must bounded Bandwidth of X1( ) = B1
sampling. The output signal which is an under Bandwidth of X2( ) = B2
sampled signal with sampling frequency
System has h(t ) = e 2 t and input to the system
50 Hz is
is x1(t) x2(t).
X(f )
The bandwidth of x1(t) x2(t) is B1 + B2.
The bandwidth of output will be B1 + B2.
So sampling rate will be 2(B1 + B2).

f
6. (b)
–fm –50 + fm 50 + fm fm
sin(1500 t ) y(t)
x(t) h (t ) =
and 50 – fm = 20 Hz t
X( )
fm = 30 Hz
1 cos(1000 t)
4. Sol.
Method-I: –1000 1000

fm1 = 4 Hz From the above block diagram,


fm 2 = 6 Hz Z(t) = x(t) cos1000 t
fm 3 = 7 Hz By using modulation property of Fourier
transform,
GATE Previous Years Solved Paper 131

1
Z( ) = [ X( + 1000 ) + X( 1000 )]
2 1
Z( ) 0.75
0.5
1/2 0.25
t

0
–2000 0 2000

sin 1500 t –0.25 0.25


Now, h(t) = = 1500 Sa (1500 t ) –0.5 0.5
t
–0.75 0.75
H( )
–1 1
1

1 t; 1 t 1
Now, x2(t) =
–1500 0 1500 0; otherwise
Thus, H( ) is a low pass filter and it will pass x2(t)
frequency, component of Z( ) upto 1500 rad/s. 1
Y( )

t
–1 1
(rad/sec)
x2(n) = {0, 0.25, 0.5, 0.75, 1, 0.75, 0.5, 0.25, 0}
–1500 1500
Therefore, maximum frequency component of
1
y(t) is
0.75
m = 1500 rad/sec 0.5
or, fm = 750 Hz 0.25
So, the minimum sampling rate for y(t) is t
fs min = 2fm = 2 × 750
0
= 1500 Hz
= 1500 samples/sec. –0.25 0.25
–0.5 0.5
7. (a)
–0.75 0.75
t; 1 t 1
x1(t) = –1 1
0; otherwise
Since x1(n) is having one more non-zero sample
x1(t)
of amplitude ‘1’ as compared to x2(n). Therefore,
1 energy of x1(n) is greater than energy of x2(n).

t
–1 1
Ts = sampling time-period
= 0.25 sec.
x1(n) = {1, 0.75, 0.5, 0.25, 0, 0.25, 0.5, 0.75, 1}
9 Digital Filters

ELECTRO NICS EN GINEERIN G The filter can be used to approximate a

(GATE Previous Years Solved Papers) (a) low-pass filter (b) high-pass filter
(c) band-pass filter (d) band-stop filter
Q.1 A continuous-time filter with transfer function [EC-2016 : 2 Marks]
2s + 6 Q.3 Let h[n] be a length 7 discrete-time finite impulse
H (s) = 2
is converted to a discrete time
s + 6s + 8 response filter, given by:
filter with transfer function, h[0] = 4, h[1] = 3, h[2] = 2, h[3] = 1
h[–1] = –3, h[–2] = –2, h[–3] = –1
2 z 2 0.5032 z
G( s ) = 2 . So that the impulse
z 0.5032 z + k and h[n] is zero for n 4. A length-3 finite
impulse response approximation g[n] of h[n] has
response of the continuous-time filter, sampled
to be obtained such that,
at 2 Hz, is identical at the sampling instants to
the impulse response of the discrete time filter. 2
E( h , g ) = H ( e j ) G( e j ) d
The value of ‘k’ is ______ .
[EC-2016 : 2 Marks]
is minimized, where H(e j ) and G(e j ) are the
Q.2 The direct form structure of an FIR (Finite discrete time Fourier transforms of h[n] and g[n],
Impulse Response) filter is shown in the figure. respectively. For the filter that minimizes E(h, g),
the value of 10g[–1] + g[1] rounded off to two
Unit Unit
x[n]
delay delay decimal places, is _______ .
[EC-2019 : 2 Marks]

5 5


+
y[n]
GATE Previous Years Solved Paper 133

Answers
EC Digital Filters

1. (0.049) 2. (c) 3. (–27)

Solutions
EC Digital Filters

1. Sol.
= 0; H(e j ) = 0
2s + 6 1 1
Given, H(s) = 2
= +
s + 6s + 8 s+2 s+4 = ; H ( e j ) = 10
2
h(t) = e–2t u(t) + e–4t u(t)
Given, f s = 2 Hz = , H(e j ) = 0
For discrete time,
The given one is band-pass filter.
n
t = nTs =
2 3. Sol.
–n –2n
h[n] = (e + e ) u[n] From Parseval’s theorem,
1 1
H(z) = + 2 1 2
1 e 1
Z 1
1 e 2Z 1 x[n] = X( e j ) d
n=
2
Z Z
= 1
+ 2 2 3
2
Z e Z e So, H ( e j ) G( e j ) d = 2 h( n ) g( n )
n= 3
2Z 2 0.5032 Z
= The solution of g(n) that minimizes E(h, g) also
Z2 0.5032 Z + 0.049
3
K = 0.049 2
minimizes, h(n) g(n) .
n= 3
2. (c)
y[n] = 5x[n] – x[n – 2]] 3
2
h(n) g(n) = 4 g(0) 2 + 3 g(1) 2
Y ( z) 2
H(z) = = 5[1 z ] n= 3
X( z)
2
But, z = ej + 3 g( 1) + 10
H(e ) = 5[1 – e–j2 ]
j
The solution of g(n) that minimizes the above
equation is,
H(e j )
g(n) = { 3, 4, 3}
10
So, 10g(–1) + g(1) = 10(–3) + 3
= –27

0 /2
GATE-2023
Electronics Engineering

Electronic Devices & Circuits


Chapterwise & Topicwise

Contents
S.No. Topic Page No.

1. Basic Semiconductor Physics .......................................................................................................... 1-17

2. PN-Junction Diodes and Special Diodes .................................................................................. 18-38

3. BJT and FET Basics ............................................................................................................................ 39-55


1 Basic Semiconductor Physics

Q.1 Consider two energy levels: E1, E ev above the Q.5 Due to illumination by light, the electron and
Fermi level and E2, E ev below the Fermi level. P1 hole concentrations in a heavily doped N type
and P2 are respectively the probability of E1 semiconductor increases by n and p
being occupied by an electron and E2 being respectively if ni is the intrinsic concentration
empty. Then, then,
(a) P1 > P2 (a) n< p (b) n> p
(b) P1 = P2
(c) n= p (d) n × p = ni2
(c) P1 < P2
(d) P1 and P2 depend on number of free electrons [EC-1989 : 2 Marks]

[EC-1987 : 2 Marks] Q.6 The concentration of ionized acceptors and


donors in a semiconductor are N A , N D
Q.2 In an intrinsic semiconductor the free electron
respectively. If NA > ND and ni is the intrinsic
concentration depends on
concentration, the position of the Fermi level
(a) effective mass of electrons only
with respect to the intrinsic level depends on
(b) effective mass of holes only
(a) NA – ND (b) NA + ND
(c) temperature of the semiconductor
N A ND
(d) width of the forbidden energy band of the (c) (d) ni
ni2
semiconductor
[EC-1987 : 2 Marks] [EC-1989 : 2 Marks]

Q.3 According to the Einstein relation, for any Q.7 Under high electric fields, in a semiconductor
semiconductor the ratio of diffusion constant to with increasing electric field,
mobility of carriers (a) the mobility of charge carriers decreases.
(a) depends upon the temperature of the (b) the mobility of charge carriers increases.
semiconductor. (c) the velocity of the charge carriers saturates.
(b) depends upon the type of the semiconductor. (d) the velocity of the charge carriers increases.
(c) varies with life time of the semiconductor. [EC-1990 : 2 Marks]
(d) is a universal constant.
Q.8 A silicon sample is uniformly doped with 1016
[EC-1987 : 2 Marks] phosphorus atoms/cm3 and 2 × 1016 boron
Q.4 Direct band gap semiconductors atoms/cm3. If all the dopants are fully ionized,
(a) exhibit short carrier life time and they are the material is
used for fabricating BJT’s. (a) n-type with carrier concentration of
(b) exhibit long carrier life time and they are 1016/cm3.
used for fabricating BJT’s. (b) p-type with carrier concentration of
(c) exhibit short carrier life time and they are 1016/cm3.
used for fabricating lasers. (c) p-type with carrier of 2 × 1016/cm3.
(d) exhibit long carrier life time and they are (d) n-type with a carrier concentration of
used for fabricating lasers. 2 × 1016/cm3.
[EC-1987 : 2 Marks] [EC-1991 : 2 Marks]
2 Electronics Engineering Electronic Devices & Circuits

Q.9 A semiconductor is irradiated with light such Q.14 A small concentration of minority carries is
that carriers are uniformly generated throughout injected into a homogeneous semiconductor
its volume. The semiconductor is n-type with crystal at one point. An electric field of 10 V/cm
N D = 10 19 /cm 3 . If the excess electron is applied across the crystal and this moves the
concentration in the steady-state is minority carries a distance of 1 cm in 20 µsec.
n = 1016/cm3 and if p = 10 µsec. (minority The mobility (in cm2/V-sec) will be
carriers life time) the generation rate due to (a) 1,000 (b) 2,000
irradiation (c) 5,000 (d) 500,000
(a) is 1020 e-h pairs/cm3/s. [EC-1995 : 1 Mark]
(b) is 1024 e-h pairs/cm3/s.
Q.15 The units of q/KT are
(c) is 1010 e-h pairs/cm3/s.
(a) V (b) V–1
(d) cannot be determined, the given data is
(c) J (d) J/K
insufficient.
[EC-1997 : 1 Mark]
[EC-1992 : 2 Marks]
Q.16 The intrinsic carrier density at 300 K is
Q.10 A p-type silicon sample has a higher
1.5 × 1010/cm3, in silicon for n-type doped to
conductivity compared to an n-type silicon
2.25 × 1015 atoms/cm3, the equilibrium electron
sample having the same dopant concentration.
and hole densities are
(True/False)
(a) n = 1.5 × 1015, p = 1.5 × 1010/cm3
[EC-1994 : 1 Mark]
(b) n = 1.5 × 1010, p = 2.25 × 1015/cm3
Q.11 The drift velocity of electrons, in silicon (c) n = 2.25 × 1015, p = 1.0 × 105/cm3
(a) is proportional to the electric field for all
(d) n = 1.5 × 1010, p = 1.5 × 1010/cm3
values of electric field.
[EC-1997 : 2 Marks]
(b) is independent of the electric field.
(c) increases at low values of electric field and Q.17 The electron concentration in a sample of non-
decreases at high values of electric field uniformly doped n-type silicon at 300 K varies
exhibiting negative differential resistance. linearly from 1017/cm3 at x = 0 to 6 × 1016/cm3
(d) increases linearly with electric field at low at x = 2 µm. Assume a situation that electrons
values of electric field and gradually are supplied to keep this concentration gradient
saturates at higher values of electric field. constant with time. If electronic charge is
[EC-1995 : 1 Mark] 1.6 × 10–19 Coulomb and the diffusion constant
Dn = 35 cm2/s, the current density in the silicon,
Q.12 The probability that an electron in a metal
if no electric field is present is
occupies the Fermi level, at any temperature.
(a) zero (b) 120 A/cm2
(T > 0 K)
(c) +1120 A/cm2 (d) –1120 A/cm2
(a) 0 (b) 1
(c) 0.5 (d) 1.0 [EC-1997 : 2 Marks]
[EC-1995 : 1 Mark] Q.18 An n-type silicon bar 0.1 cm long and 100 µm2
in cross-sectional area has a majority carrier
Q.13 In a p-type Si simple the hole concentration is
concentration of 5 × 1020/m3 and the carrier
2.25 × 10 15 /cm 3 . The intrinsic carrier
mobility is 0.13 m2/V-s at 300 K. If the charge of
concentration is 1.5 × 1010/cm3 the electron
an electron is 1.6 × 10–19 Coulomb, then the
concentration is
resistance of the bar is
(a) zero (b) 1010/cm3
(a) 106 (b) 104
(c) 105/cm3 (d) 1.5 × 1025/cm3 –1
(c) 10 (d) 10–4
[EC-1995 : 1 Mark] [EC-1997 : 2 Marks]
GATE Previous Years Solved Paper 3

Q.19 The unit of q/KT are Q.26 The neutral base width of a bipolar transistor,
(a) V (b) V–1 biased in the active region, is 0.5 µm. The
(c) J (d) J/K maximum electron concentration and the
[EC-1998 : 1 Mark] diffusion constant in the base are 1014/cm3 and
Dn = 125 cm 2/sec respectively. Assuming
Q.20 n-type silicon is obtained by doping silicon with negligible recombination in the base, the collector
(a) Germanium (b) Aluminium current density is (the electron charge is
(c) Boron (d) Phosphorus 1.6 × 10–19 Coulomb)
[EC-2003 : 1 Mark] (a) 800 A/cm2 (b) 8 A/cm2

Q.21 The band gap of silicon at 300 K is (c) 200 A/cm2 (d) 2 A/cm2

(a) 1.36 eV (b) 1.10 eV [EC-2004 : 2 Marks]

(c) 0.80 eV (d) 0.67 eV Q.27 The bandgap of silicon at room temperature is
[EC-2003 : 1 Mark] (a) 1.3 eV (b) 0.7 eV
Q.22 The intrinsic carrier concentration of silicon (c) 1.1 eV (d) 1.4 eV
sample at 300 K is 1.5 × 1016/m3. If after doping, [EC-2005 : 1 Mark]
the number of majority carriers is 5 × 1020/m3,
Q.28 The primary reason for the widespread use of
the minority carrier density is
silicon in semiconductor device technology is
(a) 4.50 × 1011/m3 (b) 3.33 × 104/m3
(a) abundance of silicon on the surface of the
(c) 5.00 × 1020/m3 (d) 3.00 × 10–5/m3 Earth.
[EC-2003 : 1 Mark] (b) larger bandgap of silicon in comparison to
Q.23 The impurity commonly used for realizing the Germanium.
base region of a silicon n-p-n transistor is (c) favorable properties of silicon-dioxide
(a) Gallium (b) Indium (SiO2).
(c) Boron (d) Phosphorus (d) lower melting point.
[EC-2004 : 1 Mark] [EC-2005 : 1 Mark]

Q.24 The resistivity of a uniformly doped n-type Q.29 A silicon sample A is doped with 1018 atoms/
silicon sample is 0.5 -cm. If the electron cm3 of Boron. Another sample B of identical
mobility (µn) is 1250 cm2/V-sec and the charge dimensions is doped with 1018 atoms/cm3 of
of an electron is 1.6 × 10–19 Coulomb, the donor phosphorus. The ratio of electron to hole
impurity concentration (ND) in the sample is mobility is 3. The ratio of conductivity of the
(a) 2 × 1016/cm3 (b) 1 × 1016/cm3 sample A to B is
(c) 2.5 × 1015/cm3 (d) 2 × 1015/cm3 1
(a) 3 (b)
[EC-2004 : 2 Marks] 3

Q.25 The longest wavelength that can be absorbed 2 3


(c) (d)
by silicon, which has the bandgap of 1.12 eV, is 3 2
1.1 µm. If the longest wavelength that can be [EC-2005 : 2 Marks]
absorbed by another material is 0.87 µm, then
Q.30 The concentration of minority carriers in an
the bandgap of this material is
extrinsic semiconductor under equilibrium is
(a) 1.416 eV (b) 0.886 eV
(a) directly proportional to the doping
(c) 0.854 eV (d) 0.706 eV concentration.
[EC-2004 : 2 Marks]
4 Electronics Engineering Electronic Devices & Circuits

(b) inversely proportional to the doping (a) ni (b) ni + NA


concentration.
ni2
(c) directly proportional to the intrinsic (c) NA – ni (d)
NA
concentration.
[EC-2007 : 1 Mark]
(d) inversely proportional to the intrinsic
concentration. Q.35 Which of the following is true?
[EC-2006 : 1 Mark] (a) A silicon wafer heavily doped with boron
is a p+ substrate.
Q.31 Under low level injection assumption, the
(b) A silicon wafer light doped with boron is a
injected minority carrier current for an extrinsic
p+ substrate.
semiconductor is essentially the
(a) diffusion current (c) A silicon wafer heavily doped with arsenic
is a p+ substrate.
(b) drift current
(d) A silicon wafer lightly doped with arsenic
(c) recombination current
is a p+ substrate.
(d) induced current
[EC-2008 : 1 Mark]
[EC-2006 : 1 Mark]
Q.36 Silicon is doped with boron to a concentration
Q.32 The majority carriers in an n-type semiconductor
of 4 × 1017 atoms/cm3. Assume the intrinsic
have an average drift velocity ‘v’ in a direction
carrier concentration of silicon to be
perpendicular to a uniform magnetic field B. The
1.5 × 1010/cm3 and the value of kT/q to be 25 mV
electric field E induced due to hall effect acts in
at 300 K. Compared to undoped silicon, the
the direction
Fermi level of doped silicon
(a) v × B (b) B × v
(a) goes down by 0.13 eV
(c) along v (d) opposite to v
(b) goes up by 0.13 eV
[EC-2006 : 2 Marks]
(c) goes down by 0.427 eV
Q.33 A heavily doped n-type semiconductor has the (d) goes up by 0.427 eV
following data: [EC-2008 : 2 Marks]
Hole-electron mobility ratio : 0.4
Doping concentration : 4.2 × 108 atoms/m3 Q.37 In an n-type silicon crystal at room temperature
Intrinsic concentration : 1.5 × 104 atoms/m3 which of the following can have a concentration
The ratio of conductance of the n-type of 4 × 1019 cm–3 ?
semiconductor to that of the intrinsic (a) Silicon atoms (b) Holes
semiconductor of same material and at the same (c) Dopant atoms (d) Valence electrons
temperature is given by [EC-2009 : 1 Mark]
(a) 0.00005 (b) 2.000
Q.38 The ratio of the mobility to the diffusion
(c) 10.000 (d) 20.000 coefficient in a semiconductor has the unity
[EC-2006 : 2 Marks] (a) V–1 (b) cm V–1
Q.34 The electron and hole concentrations in an (c) V cm–1 (d) V s
intrinsic semiconductor are ni per cm3 at 300 K. [EC-2009 : 1 Mark]
Now, if acceptor impurities are introduced with
Linked Answer Questions (39 and 40):
a concentration of NA per cm3 (where NA >> ni)
The silicon sample with unit cross-sectional areas
the electron concentration per cm3 at 300 K
shown below is in thermal equilibrium. The following
will be
information is given:
GATE Previous Years Solved Paper 5

T = 300 K, electronic charge = 1.6 × 10–19 C, thermal Q.43 A silicon bar is doped with donor impurities
voltage = 26 mV and electron mobility = 1350 cm2/V-s. ND = 2.25 × 1015 atoms/cm3. Given the intrinsic
carrier concentration of silicon at T = 300 K is
n i = 1.5 × 10 10 cm –3 . Assuming complete
1V
impurity ionization, the equilibrium electron
16 3
ND = 10 /cm and hole concentrations are:
(a) n0 = 1.5 × 1016 cm–3, p0 = 1.5 × 105 cm–3
x=0 x = 1 µm (b) n0 = 1.5 × 1010 cm–3, p0 = 1.5 × 1015 cm–3
(c) n0 = 2.25 × 1015 cm–3, p0 = 1.5 × 1010 cm–3
Q.39 The magnitude of the electric field at x = 0.5 µm
is (d) n0 = 2.25 × 1015 cm–3, p0 = 1 × 105 cm–3

(a) 1 kV/cm (b) 5 kV/cm [EC-2014 : 1 Mark]


(c) 10 kV/cm (d) 26 kV/cm Q.44 Assume electronic charge q = 1.6 × 10–19 C,
[EC-2010 : 2 Marks] kT/q = 25 mV and electron mobility
µn = 1000 cm2/V-s. If the concentration gradient
Q.40 The magnitude of the electron drift current
of electrons injected into a p-type silicon sample
density at x = 0.5 µm is
is 1 × 1021 cm–3, the magnitude of electron
(a) 2.16 × 104 A/cm2
diffusion current density (in A/cm2) is ____ .
(b) 1.08 × 104 A/cm2
[EC-2014 : 2 Marks]
(c) 4.32 × 103 A/cm2
(d) 6.48 × 102 A/cm2 Q.45 When a silicon diode having a doping
[EC-2010 : 2 Marks] concentration of NA = 9 × 1016 cm–3 on p-side
and ND = 1 × 1016 cm–3 on n-side is reverse
Q.41 Drift current in semiconductors depends upon
biased, the total depletion width is found to be
(a) only the electric field.
3 µm. Given that the permittivity of silicon is
(b) only the carrier concentration gradient. 1.04 × 10–12 F/cm, the depletion width on the
(c) both the electric field and the carrier p-side and the maximum electric field in the
concentration. depletion region, respectively, are
(d) both the electric field and the carrier (a) 2.7 µm and 2.3 × 105 V/cm
concentration gradient.
(b) 0.3 µm and 4.15 × 105 V/cm
[EC-2011 : 1 Mark]
(c) 0.3 µm and 0.42 × 105 V/cm
Q.42 The doping concentrations on the p-side and (d) 2.1 µm and 0.42 × 105 V/cm
n-side of a silicon diode are 1 × 1016 cm–3 and [EC-2014 : 2 Marks]
1 × 1017 cm–3, respectively. A forward bias of
0.3 V is applied to the diode. At T = 300 K, the Q.46 A thin p-type silicon sample is uniformly
intrinsic carrier concentration of silicon illuminated will light which generates excess
carriers. The recombination rate is directly
kT proportional to
n i = 1.5 × 10 10 cm –3 and = 26 mV. The
q
(a) the minority carrier mobility.
electron concentration at the edge of the (b) the minority carrier recombination lifetime.
depletion region on the p-side is (c) the majority carrier concentration.
(a) 2.3 × 109 cm–3 (b) 1 × 1016 cm–3 (d) the excess minority carrier concentration.
(c) 1 × 1017 cm–3 (d) 2.25 × 106 cm–3 [EC-2014 : 1 Mark]
[EC-2014 : 2 Marks]
6 Electronics Engineering Electronic Devices & Circuits

Q.47 At T = 300 K, the hole mobility of a Q.51 Consider a silicon sample doped with
semiconductor µ p = 500 cm/V-s and ND = 1 × 1015/cm3 donor atoms. Assume that
the intrinsic carrier concentration
kT
= 26 mV. The hole diffusion constant Dp in ni = 1.5 × 1010/cm3. If the sample is additionally
q
doped with NA = 1 × 1018/cm3 acceptor atoms,
cm2/s is ______ .
the approximate number of electrons/cm3 in the
[EC-2014 : 1 Mark]
sample, at T = 300 K, will be ______ .
Q.48 A T = 300 K, the bandgap and the intrinsic [EC-2014 : 2 Marks]
carrier concentration of GaAs are 1.42 eV and
106 cm–3, respectively. In order to generate Q.52 An N-type semiconductor having uniform
electron hole pairs in GaAs, which one of the doping is biased as shown in the figure.
wavelength ( C) ranges of incident radiation, is
most suitable? (Given that: Plank’s constant is V
6.62 × 10–34 J-s, velocity of light is 3 × 1010 cm/s
and charge of electron is 1.6 × 10–19 C) N-type semiconductor

(a) 0.42 µm < C < 0.87 µm


If EC is the lowest energy level of the conduction
(b) 0.87 µm < C < 1.42 µm
band, EV is the highest energy level of the
(c) 1.42 µm < C < 1.62 µm valance band and EF is the Fermi level, which
(d) 1.62 µm < C < 6.62 µm one of the following represents the energy band
[EC-2014 : 1 Mark] diagram for the biased N-type semiconductor?
Q.49 In the figure, in ( i) is plotted as a function of EC EC
1/T, where i is the intrinsic resistivity of EF
(a) (b) EF
silicon, T is the temperature, and the plot is
almost linear. EV EV

ln ( i) EC
EF
EC
(c) EV (d)
EF

EV
[EC-2014 : 2 Marks]
1/T

The slope of the line can be used to estimate Q.53 A silicon sample is uniformly doped with donor
(a) bandgap energy of silicon (Eg). type impurities with a concentration of
(b) sum of electron and hole mobility in silicon 1016/cm3. The electron and hole mobilities in
(µn + µp). the sample are 1200 cm2/V-s and 400 cm2/V-s
(c) reciprocal of the sum of electron and hole respectively. Assume complete ionization of
mobility in silicon (µn + µp)–1. impurities. The charge of an electron is
(d) intrinsic carrier concentration of silicon (ni). 1.6 × 10 –19 C. The resistivity of the sample
(in -cm) is ________ .
[EC-2014 : 1 Mark]
[EC-2015 : 1 Mark]
Q.50 The cut-off wavelength (in µm) of light that can
be used for intrinsic excitation of a Q.54 A piece of silicon is doped uniformly with
semiconductor material of bandgap, Eg = 1.1 eV phosphorous with a doping concentration of
is _______ . 1016/cm3. The expected value of mobility versus
[EC-2014 : 1 Mark] doping versus doping concentration for silicon
GATE Previous Years Solved Paper 7

assuming full dopant ionization is shown (a) –4.4 × 10–2 (b) –2.2 × 10–2
below. The charge of an electron is 1.6 × 10–19 C. (c) 0 (d) 2.2 × 10–2
The conductivity (in S cm –1) of the silicon [EC-2015 : 2 Marks]
sample at 300 k is ______ .
Q.57 A dc voltage of 10 V is applied across an n-type
Hole and Electron Mobility in Silicon at 300 K
silicon bar having a rectangular cross-section
1400
Electron Hole and length of 1 cm as shown in figure. The donor
1200
doping concentration ND and the mobility of
s )
–1

1000
–1

electrons µn are 1016 cm–3 and 1000 cm–2 V–1 s–1,


V

800
2

respectively. The average time (in µs) taken by


Mobility (cm

600

400
the electrons to move from one end of the bar to
200
other end is _______ .

1.E + 13 1.E + 14 1.E + 15 1.E + 16 1.E + 17 1.E + 18 1.E + 19 1.E + 20


Doping concentration (cm–3 )
10 V

[EC-2015 : 1 Mark] n-Si


1 cm
Q.55 An n-type silicon sample is uniformly
illuminated with light which generates 10 20 [EC-2015 : 2 Marks]
electron-hole pairs per cm3 per second. The Q.58 A small percentage of impurity is added to an
minority carrier lifetime in the sample is 1 µs. In intrinsic semiconductor at 300 K. Which one of
the steady-state, the hole concentration in the the following statements is true for the energy
sample is approximately 10x, where x is an band diagram shown in the following figure?
integer. The value of x is _______ .
[EC-2015 : 1 Mark] Conduction band
EC
Q.56 The energy band diagram and the electron
0.01 eV
density profile n(x) in a semiconductor are
New energy level
shown in the figures.
EV
q x Valence band
Assume that, n( x ) = 1015 e kT cm –3, with
(a) Intrinsic semiconductor doped with
= 0.1 V/cm and x expressed in cm. Given pentavalent atoms to form n-type
D kT semiconductor.
kT
= 0.026 V, Dn = 36 cm2 s–1 and = .
q µ q (b) Intrinsic semiconductor doped with
trivalent atoms to form n-type
The electron current density (in A/cm2) at x = 0 semiconductor.
is
(c) Intrinsic semiconductor doped with
E(eV) log( n(x)) pentavalent atoms to form p-type
semiconductor.
Slope = –0.1 eV/cm
(d) Intrinsic semiconductor doped with
trivalent atoms to form p-type
EC semiconductor.

EV
[EC-2016 : 1 Mark]
x=0 x x=0 x
8 Electronics Engineering Electronic Devices & Circuits

Q.59 The figure below shows the doping distribution (c) Silicon atoms acts as p-type dopants in
in a p-type semiconductor in log scale. Arsenic as well as Gallium sites.
(d) Silicon atoms acts as n-type dopants in
Arsenic as well as Gallium sites.
16
–3
10 [EC-2017 : 1 Mark]
NA (cm )
14
10 Q.62 As shown, a uniformly doped Silicon (Si) bar of
1 2
length L = 0.1 µm with a donor concentration
Position (µm) ND = 1016 cm–3 is illuminated at x = 0 such that
The magnitude of the electric field (in kV/cm) electron and hole pairs are generated at the rate of
in the semiconductor due to non-uniform x
doping is _______ . GL0 1 , 0 x L
L
[EC-2016 : 2 Marks]
where GL 0 = 1017 cm 3
s 1 . Hole lifetime is
Q.60 Consider a silicon sample at T = 300 K, with a
uniform donor density Nd = 5 × 10 16 cm–3, 10–4 s, electronic charge q = 1.6 × 10–19 C, hole
illuminated uniformly such that the optical diffusion coefficient Dp = 100 cm2/s and low
generation rate is Gopt = 1.5 × 1020 cm–3 s–1 level injection condition prevails. Assuming a
throughout the sample. The incident radiation linearly decaying steady-state excess hole
is turned-off at t = 0. Assume low-level injection concentration that goes to 0 at x = L, the
to be valid and ignore surface effects. The carrier magnitude of the diffusion current density at
lifetimes are p0 = 0.1 µs and n0 = 0.5 µs . x = L/2, (in A/cm2), is _______ .
Light
16 –3
Si (ND = 10 cm )

x=0 L = 0.1 µm
n-type Si
[EC-2017 : 2 Marks]

The hold concentration at t = 0 and the hole Q.63 The dependence of drift velocity of electrons on
concentration at t = 0.3 µs, respectively are electric field in a semiconductor is shown below.
(a) 1.5 × 1013 cm–3 and 7.47 × 1011 cm–3 The semiconductor has a uniform electron
(b) 1.5 × 1013 cm–3 and 8.23 × 1011 cm–3 concentration of n = 1 × 1016 cm–3 and electronic
(c) 7.5 × 1013 cm–3 and 3.73 × 1011 cm–3 charge q = 1.6 × 10–19 C. If a bias of 5 V is applied
(d) 7.5 × 1013 cm–3 and 4.12 × 1011 cm–3 across a 1 µm region of this semiconductor, the
[EC-2016 : 2 Marks] resulting current density in this region,
(in kA/cm2), is _______ .
Q.61 A bar of Gallium Arsenide (GaAs) is doped with
Drift velocity (cm/s)
Silicon such that the Silicon atoms occupy
Gallium and Arsenic sites in the GaAs crystal. Constant
7
Which one of the following statements is true? 10

(a) Silicon atoms acts as p-type dopants in


r a
ne

Arsenic sites and n-type dopants in Gallium


Li

sites.
(b) Silicon atoms acts as n-type dopants in 0 5 × 10
5
Electric field (V/cm)
Arsenic sites and p-type dopants in Gallium
sites. [EC-2017 : 2 Marks]
GATE Previous Years Solved Paper 9

Q.64 A single crystal intrinsic semiconductor is at a respectively. The left side of the bar (x = 0) is
temperature of 300 K with effective density of uniformly illuminated with a laser having
states for holes twice that of electrons. The photon energy greater than the bandgap of the
thermal voltage is 26 mV. The intrinsic Fermi semiconductor. Excess electron hole pairs are
level is shifted from mid-bandgap energy generated only at x = 0 because of the laser. The
level by steady-state electron density at x = 0 is 1014 cm–3
(a) 13.45 meV (b) 18.02 meV due to laser illumination. Under these
conditions and ignoring electric field, the closest
(c) 26.90 meV (d) 9.01 meV
approximation (among the given options) of the
[EC-2020 : 1 Mark]
steady-state electron density at x = 2 µm, is
Q.65 A bar of silicon is doped with boron _________ .
concentration of 1016 cm–3 and assumed to be (a) 0.37 × 1014 cm–3 (b) 0.63 × 1013 cm–3
fully ionized. It is exposed to light such that (c) 3.7 × 1014 cm–3 (d) 103 cm–3
electron hole pairs are generated throughout the [EC-2022]
volume of the bar at the rate of 1020 cm–3 s–1. If
Q.67 Select the correct statement(s) regarding
the recombination lifetime is 100 µs, intrinsic
semiconductor devices:
carrier concentration of silicon is 1010 cm–3 and
assuming 100% ionization of boron, then the (a) Electrons and holes are of equal density in
approximate product of steady-state electron an intrinsic semiconductor at equilibrium.
and hole concentrations due to this light (b) Collector region is generally more heavily
exposure is, doped than base region in a BJT.
(a) 1032 cm–6 (b) 2 × 1020 cm–6 (c) Total current is spatially constant in a two
(c) 2× 1032 cm–6 (d) 1020 cm–6 terminal electronic device in dark under
steady-state condition.
[EC-2021 : 1 Mark]
(d) Mobility of electrons always increases with
Q.66 Consider a long rectangular bar of direct temperature in silicon beyond 300 K.
bandgap p-type semiconductor. The equilibrium
[EC-2022]
hole density is 1017 cm–3 and the intrinsic carrier
concentration is 1010 cm–3. Electron and hole
diffusion lengths are 2 µm and 1 µm,
10 Electronics Engineering Electronic Devices & Circuits

Answers Basic Semiconductor Physics

1. (b) 2. (c) 3. (a) 4. (c) 5. (c) 6. (a) 7. (a, c) 8. (b)

9. (a) 10. (False) 11. (d) 12. (c) 13. (c) 14. (c) 15. (b) 16. (c)

17. (d) 18. (a) 19. (b) 20. (d) 21. (b) 22. (a) 23. (c) 24. (b)

25. (a) 26. (b) 27. (c) 28. (a) 29. (b) 30. (b) 31. (a) 32. (b)

33. (d) 34. (d) 35. (a) 36. (c) 37. (c) 38. (a) 39. (c) 40. (a)

41. (c) 42. (a) 43. (d) 44. (4000) 45. (b) 46. (d) 47. (13) 48. (a)

49. (a) 50. (1.12) 51. (225.2) 52. (d) 53. (0.52) 54. (1.92) 55. (14) 56. (c)

57. (100) 58. (a) 59. (1.198) 60. (a) 61. (a) 62. (16) 63. (1.6) 64. (d)
65. (c) 66. (a) 67. (a, c)

Solutions Basic Semiconductor Physics

2. (c) µ = Mobility
By mass action law, Dn = Electron diffusion constant
Dp = Hole diffusion constant
n p = ni2
µn = Electron mobility
ni = intrinsic carrier concentration
µp = Hole mobility
p = hole concentration
n = electron concentration 4. (c)

ni2 T3 DBG (Direct Band Gap) semiconductors exhibit


short carrier life time they are used for
ni T3/2
fabricating lasers.
For intrinsic semiconductor,
In DBG semiconductor during the
n = p = ni
recombination the energy is released in the form
n T3/2
of light.
3. (a)
5. (c)
We know that,
n= p
Einstein equation
Due to illumination by light EHP (electron-hole
D Dn Dp T (°K ) pair) generation occurs.
= = = VT =
µ µn µp 11600 So, n= p
where,
where,
n = increase in electron concentration due
T = Temperature in °K to illumination by light
VT = Thermal voltage p = increase in hole concentration due to
D = Diffusion constant illumination by light
GATE Previous Years Solved Paper 11

7. (a, c) 10. Sol.


(False)
µ
The given statement is false, because for a given
µ E
–1/2 semiconductor the electron mobility (µn) is
always higher than the hole mobility (µp).
µn > µp (for a given semiconductor) we know
µ 1/E
that, the conductivity of a given n-type
semiconductor,
n = nqµn
3 4 7
10 10 10 E(V/m)
the conductivity of a given p-type
v d = µE semiconductor,
where,
p = pqµp
v d = Drift velocity Given that, n = p
µ = Mobility = same dopant concentration
E = Applied electric field q = 1.602 × 10–19 Col.
For high electric field, with increasing electric So, n > p
field:
1. The mobility of charge carriers decreases 11. (d)
as electric field increases, v d = µE
1 where,
µ v d = Drift velocity
E
µ = Mobility
2. The velocity (drift velocity) of charge
E = Applied electric field
carriers saturates.
µ
8. (b)
–1/2
p-type with carrier concentration of 1016/cm3. µ E
Given that,
ND = n = phosphorus atoms = 1016/cm3 µ 1/E
NA = p = Boron atoms = 2 × 1016/cm3
Q NA >> ND
3 4 7
So, the resultant material will be p-type 10 10 10 E(V/m)
semiconductor carrier concentration
1. For smaller electric field applied, mobility
= NA – ND
of change carrier will remain almost
= 2 × 1016 – 1016
constant. So for smaller field applied, drift
= 1016/cm3
velocity (vd) increases linearly with electric
9. (a) field.
1020 e-h pairs/cm3/s 2. For large electric field applied, mobility of
Given that, n = 1015/cm3 charge carrier will be very small, so the drift
velocity gradually saturates at higher
p = 10 µ-sec = 10 × 10 sec.
–6

values of electric field.


P 1015
Generation rate = =
p 10 × 10 6 13. (c)

= 1020 e-h pairs/cm3/s By Mass Action Law:


n p = ni2
12 Electronics Engineering Electronic Devices & Circuits

where, By Mass Action Law:


n = electron concentration n p = ni2
p = hole concentration
ni = intrinsic carrier concentration ni2 (1.5 × 10 10 )2
P= = = 105 atoms/cm3
p = 2.25 × 1015/cm3 n 2.25 × 10 15
ni = 1.5 × 1010/cm3 17. (d)
ni2 (1.5 × 1010 )2 dn
n= = Jn = nq µn E + Dn q
p 2.25 × 1015 dx
Given, E=0
2.25 × 10 20 5 3
= = 10 /cm dn
2.25 × 10 15 Jn = Dn q
dx
14. (c) dn 6 × 10 16 1017
= 4
= 2 × 10 20
Distance dx 2 × 10 0
Velocity =
Time Jn = (35) (1.6 × 10–19) (–2 × 1020)
= –1120 A/cm2
1
vd = 6
20 × 10 18. (a)
= 50,000 cm/sec Given that, l = 0.1 cm = 10–3 m
Drift velocity = vd = µE A = 100 µm2 = 100 × 10–12 m2
where, n = 5 × 1020/m3
n = Mobility µn = 0.13 m2/V-sec
E = Electric field q = 1.6 × 10–19 Col.
vd 50, 000 We know that,
µ= = = 5,000 cm3/volt-sec
E 10 Conductivity = = nqµn
1
15. (b) Resistivity = =
Thermal voltage or volt-equivalent of l l
temperature, Resistance = =
A A
3
KT R=
10
= 106
VT = = Volts (5 × 10 20
) × (1.6 × 10 19
) (0.13) (100 × 10 12
)
q

q 19. (b)
So, V–1
KT KT
Thermal voltage = VT =
q
16. (c)
q
n = 2.25 × 1015 So, V 1
KT
p = 1.0 × 105/cm3
For n-type semiconductor, 22. (a)
Electron density = n = ND
ni2 = np
= 2.25 × 1015 atoms/cm3
Given that, intrinsic carrier concentration = ni ni = Intrinsic concentration
= 1.5 × 10/cm3 ni2 1.5 × 10 16 × 1.5 × 10 16
p = =
Q n>>ni n 5 × 10 20
= 45 × 1010 = 4.5 × 1011/cm3
GATE Previous Years Solved Paper 13

24. (b) 33. (d)


1 For n-type semiconductor n = nqµn
= For intrinsic semiconductor,
nq µ n
n = ND i = ni q(µn + µp)

n nµn
1 1 =
ND = = 19 i ni (µn + µ p )
q µn p 1.6 × 10 × 1250 × 0.5
= 1016/cm3 4.2 × 108 × µn
=
µp
25. (a) 1.5 × 10 4 × µn 1 +
µn
1.24
Eg = eV
(µm) 4.2 × 108
= 4
= 2 × 10 4
1.5 × 10 × 1.4
1.24
= eV = 1.425 eV
0.87 µm 34. (d)
By the law of electrical neutrality,
26. (b)
p + ND = n + NA
dn As, ND = 0
JC = qDn
dx NA >> ni 0,
19 10 14 p = NA
= 1.6 × 10 × 25 ×
0.5 × 10 4 Using mass action law,
JC = 8 A/cm2 np = ni2
29. (b) ni2 n2
So, n= = i
n = nqµn p NA
p µp 1
= = 36. (c)
n µn 3
Since, boron is p-type impurity, therefore, Fermi-
30. (b) level goes down.
2 NA
np = ni Ei – Ef = kT ln
ni
ni = constant
For n-type ‘p’ is minority carrier concentration, 3 4 × 1017
= 25 × 10 ln
1.5 × 1010
ni2
p = = 0.427 eV
n
1 37. (c)
p
n In an N-type silicon crystal at room temperature,
n Nd
32. (b) and by mass action law,
Hall effect:
n p = ni2
Electric force + magnetic force = 0
qE + qv + B = 0 (1.5 × 10)2
p = = 5.625 cm 3
E = –v × B 4 × 1019
E = B×v and n>>p
14 Electronics Engineering Electronic Devices & Circuits

38. (a) 43. (d)


D Since, ND >> ni therefore equilibrium electron
= VT concentration is,
µ
n ND = 2.25 × 105 cm–3
µ 1
= units : V–1 and equilibrium hole concentration is given by
D VT
mass action law,
39. (c) ni2 (1.5 × 1010 )2
p= = = 1 × 105 cm–3
The sample is in thermal equilibrium. ND 2.25 × 1015
V 1
So, E= = 44. Sol.
d 1 × 10 6
= 106 V/m = 10 kV/cm dn
J = qDn
dx
40. (a) where, Dn = µnVT
J= E,
1000 cm 2
= nqµn = ND qµn Dn = × 25 × 10 3 V
Vs
J= ND qµnE
Dn = 25 cm2/s
J= 1016 × (1.6 × 10–19) (1350) (10 × 103)
then, J = 1.6 × 10–19 × 25 × 1 × 1021
J= 2.16 × 104 A/cm2
J = 4000 A/cm2
41. (c)
45. (b)
J = n e vd
We know that,
Put, v d = µE
J = n e µE NA xn
=
ND xp
Hence, I = n e µ EA
So, I depends upon carrier concentration and NA x
So, +1 = n +1
electric field. ND xp

42. (a) N A + ND xn + x p
=
The electron concentration at the edge of the ND xp
depletion region on the p-side is given by 1017 3 µm
qV f 16 =
10 xp
np = npo = exp
kT x p = 0.3 µm
qN A x p qN D xn
ni2 and Emax = =
where, npo =
NA s s
19
1.6 × 10 × 9 × 1016 × 0.3 × 10 4
(1.5 × 1010 )2 Emax =
npo = 1.04 × 10 12
1016
= 4.15 × 105 V/cm
= 2.25 × 104 cm–3
we then have, 47. Sol.
0.3 Dp
np = 2.25 × 10 4 exp
0.026 = VT
µp
np = 2.25 × 104
× 1.025 × 105
Dp = µpVT
np = 2.306 × 10 cm–3
9
= 500 × 26 × 10–3
Dp = 13 cm2/s
GATE Previous Years Solved Paper 15

48. (a) 56. (c)


1.24 The concentration of doping is not uniform
c
µm throughout the semiconductor, thus to maintain
Eg ( eV )
equilibrium internal electric field is generated
1.24 due to which the band of the semiconductor is
c
0.87 µm
1.42 varying with slop –0.1 eV/cm.
The generated electric field opposes the
50. Sol.
diffusion of carriers due to concentration
1.24 1.24 gradient and thus there will be not current flow
c = µm = µm
Eg ( eV ) 1.1 inside the semiconductor.

c = 1.12 µm 57. Sol.

51. Sol. Electric field inside semiconductor bar,


E = 10 V/cm
For a compensated p-type semiconductor, the
Velocity of electron,
minority carrier electron concentration is
V = µE = 1 × 104 cm/sec
given by
The average time (in µs) taken by the electrons
ni2 to move from one end of the bar to other end is
no =
(N A ND ) L
= 100 µsec
2.25 × 10 20 V
no =
(10 18 1015 ) 58. (a)
= 225.2 cm–3 Pentavalent impurity when introduced in
intrinsic SC, a new discrete energy level called
53. Sol.
donor energy level is created just below the
1 1 conduction band.
P= =
N ND qµn
59. Sol.
1
=
10 16 × 1.6 × 10 19
× 1200
–3
NA(cm )

= 0.52 -cm 16 36.84


10

54. Sol. 32.24


14
As per the graph, mobility of electrons at the 10
1 2 0.001 0.01
concentration 1016/cm3 is 1200 cm2/V-s. µm (log scale)
So, µn = 1200 cm2/V-s
Applying the current density equation,
N = ND qµn J = Jdrift + Jdrift
= 1016 × 1.6 × 10–19 × 1200 J=0
= 1.92 S cm–1 dp
0 = qDp + q µ p pE
dx
55. Sol.
dp
The concentration of hole-electron per in 1 µsec qDp = qµppE
dx
= 1020 × 10–6 = 1014/cm3
So, the power of 10 is 14, VT dp
E=
x = 14 p( x ) dx
16 Electronics Engineering Electronic Devices & Circuits

V dp Slope of the curve,


E =
p( x ) dx 107 0
m = = 20
5 (y = mx)
d 5 × 10
E = VT ln[ N A ( x )]
dx vd = 20 × E = 20 × 5 × 104
= 106 V/cm
ln (1016 ) ln (1014 )
E = VT J = nevd
4
(2 1) × 10
= 1 × 1016 × 1.6 × 10–19 × 106
36 32.23 = 1.6 × 103 A/cm2
= 0.026
1 × 10 4 = 1.6 kA/cm2
= 1198.6 V/cm 64. (d)
= 1.198 kV/cm
EC EV kT NC
EFi = ln
60. (a) 2 2 NV
Given, Gopt = 1.5 × 1020/cm3/sec
NV
NA NC =
Gopt = R = 2
P
0.026
NA = ln 0.5 = 9.01 meV
1.5 × 102 = 2
0.1 × 10 6
NA = 1.5 × 1013/cm3 S65. (c)
NA = 1016 per cm3
t/ p
P(t) = Pno e G1 = 1020 per cm3-S
= 1.5 × 1013 e–0.3/0.1 = 100 µs
= 7.46 × 1011/cm3 ni = 1010 per cm3
Steady-state excess carriers
61. (a) p = n=
Si acts as p-type dopant in As sites. = G1 = 1020 × 100 × 10–6
Si acts as n-type dopant in As sites. = 1016 per cm3
p p = ppo + = 2 × 1016 per cm3
62. Sol.
np = npo + = 1016 per cm3
Net hole density varying in the direction of x is, pp np = 2 × 1032 per cm6
pn(x) = pno + p = pno + GL p
66. (a)
x
= pno + GLD p 1 Given, P = 1017 cm–3 = P0
L
ni2 1020
dp GLo p n0 = = = 103 cm 3
Jp, drift = eDp = eDp P0 1017
dx L
Ln = 2 µm
19
1.6 × 10 × 100 × 10 17 × 10 4
= 4
A/cm 2 nP0 = 1014 cm–3
0.1 × 10
= 16 A/cm2 nP(x)

63. Sol.
n P0
V 5
E = = = 5 × 10 4 V/cm
d 10 4 nP0 3 –3
nP0 = n0 = 10 cm
x
GATE Previous Years Solved Paper 17

Excess electron concentration at any distance • Total current is spatially constant in a two
‘x’ is, terminal electronic device, however
x / Ln individual currents vary spatially under
nP(x) = nP 0 e
dark and steady-state condition.
= 1014 e–2/2 = 1014 e–1 • Beyond 300 K, mobility of electron decreases
= 3.67 × 1013 cm–3 with increases in temperature.
= 0.367 × 1014 cm–3
Hence, statement(s): (a) and (c) are correct.
67. (a, c)
• In intrinsic semiconductor at equilibrium,
n = p = ni.
• Collector region is generally lightly doped
than base region in a BJT.
2 PN - Junction Diodes and
Special Diodes
Q.1 The diffusion capacitance of a p-n junction KT I
(c) V = sin h 1
(a) decreases with increasing current and q 2
increasing temperature.
KT
(b) decreases with decreasing current and (d) V = [exp( I ) 1]
q
increasing temperature.
[EC-1988 : 2 Marks]
(c) increases with increasing current and
increasing temperature. Q.4 The switching speed of P+N junction (having a
(d) does not depend on current and heavily doped P region) depends primarily on
temperature. (a) the mobility of minority carriers in the P+
[EC-1987 : 2 Marks] region.
(b) the lifetime of minority carriers in the P+
Q.2 For an pn-junction match the type of breakdown region.
with phenomenon:
(c) the mobility of majority carriers in the N
1. Avalanche breakdown region.
2. Zener breakdown (d) the lifetime of majority carriers in the N
3. Punch through region.
A. Collision of carriers with crystal ions [EC-1989 : 2 Marks]
B. Early effect
Q.5 In a Zener diode
C. Rupture of covalent bond due to strong (a) only the P region is heavily doped.
electric field.
(b) only the N region is heavily doped.
(a) 1-B, 2-A, 3-C (b) 1-C, 2-A, 3-B
(c) both P and N regions are heavily doped.
(c) 1-A, 2-B, 3-C (d) 1-A, 2-C, 3-B (d) both P and P regions are lightly doped.
[EC-1988 : 2 Marks] [EC-1989 : 2 Marks]
Q.3 In the circuit shown below the current voltage Q.6 In a junction diode
relationship when D1 and D2 are identical is (a) the depletion capacitance increases with
given by (Assume Ge diodes) increase in the reverse bias.
+ (b) the depletion capacitance decreases with
D1 increase in the reverse bias.
I
(c) the depletion capacitance increases with
D2
increase in the forward bias.
(d) the depletion capacitance is much higher
V than the depletion capacitance when it is
+ forward biased.
[EC-1990 : 1 Mark]
KT I
(a) V = sin h
q 2 Q.7 In a uniformly doped abrupt p-n junction the
doping level of the n-side is four (4) times the
KT I doping level of the p-side the ratio of the
(b) V = ln
q Io depletion layer width of n-side verses p-side is
GATE Previous Years Solved Paper 19

(a) 0.25 (b) 0.5 (c) does not depend on doping concentrations.
(c) 1.0 (d) 2.0 (d) increases with increase in doping
[EC-1990 : 2 Marks] concentration.
[EC-1995 : 1 Mark]
Q.8 The small signal capacitance of an abrupt P+ n
junction is 1 nf/cm2 at zero bias. If the built-in Q.12 A Zener diode works on the principle of
voltage is 1 volt, the capacitance at a reverse (a) tunneling of charge carriers across the
bias voltage of 99 volts in junction.
(a) 10 (b) 0.1 (b) thermionic emission.
(c) 0.01 (d) 100 (c) diffusion of charge carriers across the
[EC-1991 : 2 Marks] junction.
(d) hopping of charge carriers across the
Q.9 Referring to the below figure the switch ‘S’ is in
junction.
position 1 initially and steady-state condition
exist from time t = 0 to t = t0 , the switch is [EC-1995 : 1 Mark]
suddenly thrown into position 2. The current Q.13 The depletion capacitance, CJ of an abruptly p-n
‘I’ through the 10 K resistor as a function of time junction with constant doping on either side
t, from t = 0 is (Given the sketch showing the varies with R.B. VR as,
magnitudes of the current at t = 0, t = t0 and
t = ). (c) CJ VR (b) C J VR 1

(c) C J VR 1/2 (d) C J VR 1/3


1 2
10 k I [EC-1995 : 1 Mark]

20 V 20 V Q.14 The static characteristic of an adequately


forward biased p-n junction is a straight line, if
the plot is of
(a) log I vs log V (b) log I vs V
[EC-1991 : 2 Marks]
(c) I vs log V (d) I vs V
Q.10 The built-in potential (diffusion potential) in a [EC-1998 : 1 Mark]
p-n junction
Q.15 A pn-junction in series with a 100 resistor, is
(a) is equal to the difference in the Fermi-level
forwarded biased. So that a current of 100 mA
of the two sides, expressed in volts.
flows. If the voltage across this combination is
(b) increases with the increase in the doping instantaneously reversed at t = 0 current through
levels of the two sides. diodes is approximately given by
(c) increases with the increase in temperature. (a) 0 mA (b) –100 mA
(d) is equal to the average of the Fermi-level of (c) 100 mA (d) 50 mA
the two sides.
[EC-1998 : 2 Marks]
[EC-1993 : 2 Marks]
Q.16 In the figure, silicon diode is carrying a constant
Q.11 The diffusion potential across a pn-junction current of 1 mA. When the temperature of the
(a) decreases with increasing doping diode is 20°C, VD is found to be 700 mV. If the
concentration. temperature rises to 40°C, V D becomes
(b) increases with decreasing bandgap. approximately equal to
20 Electronics Engineering Electronic Devices & Circuits

+ Q.20 A particular green LED emits light of


wavelength 5490 Å. The energy bandgap of the
semiconductor material used there is
VD
(Plank’s constant = 6.626 × 10–34 J-s)
(a) 2.26 eV (b) 1.98 eV

(c) 1.17 eV (d) 0.74 eV
(a) 740 mV (b) 660 mV
[EC-2003 : 2 Marks]
(c) 680 mV (d) 700 mV
[EC-2002 : 1 Mark] Q.21 In a abrupt p-n junction, the doping
concentrations on the p-side and n-side are
Q.17 Choose proper substitute for X and Y to make
NA = 9 × 10 16/cm and ND = 1 × 10 16/cm 3
the following statement correct tunnel diode
respectively. The p-n junction is reverse biased
and avalanche photodiode are operated in X
and the total depletion width is 3 µm. The
bias and Y bias respectively.
depletion width on the p-side is
(a) X : reverse, Y : reverse
(a) 2.7 µm (b) 0.3 µm
(b) X : reverse, Y : forward
(c) 2.25 µm (d) 0.75 µm
(c) X : forward, Y : reverse
[EC-2004 : 2 Marks]
(d) X : forward, Y : forward
[EC-2003 : 1 Mark] Q.22 Consider an abrupt p-junction. Let Vbi be the
built-in potential of this junction and VR be the
Q.18 Match items in Group-I with items in Group-II,
applied reverse bias. If the junction capacitance
most suitably:
(C j ) is 1 pF for V bi + V R = 1 V, then for
Group-I Group-II Vbi + VR = 4 V, Cj will be
P. LED 1. Heavy doping
(a) 4 pF (b) 2 pF
Q. Avalanche 2. Coherrent radiation
(c) 0.25 pF (d) 0.5 pF
photodiode
[EC-2004 : 2 Marks]
R. Tunnel diode 3. Spontaneous emission
S. LASER 4. Current gain Q.23 A silicon pn-junction at a temperature of 20°C
has a reverse saturation current of 10 pico-
(a) P-1, Q-2, R-4, S-3
amperes (pA). The reverse saturation current at
(b) P-2, Q-3, R-1, S-4
40°C for the same bias is approximately
(c) P-3, Q-4, R-1, S-2
(a) 30 pA (b) 40 pA
(d) P-2, Q-1, R-4, S-3
(c) 50 pA (d) 60 pA
[EC-2003 : 2 Marks]
[EC-2005 : 1 Mark]
Q.19 At 300 K, for a diode current of 1 mA, a certain
germanium diode requires a forward bias of Q.24 A silicon pn-junction diode under reverse bias
0.1435 V, whereas a certain silicon diode has depletion region of width 10 µm. The
requires a forward bias of 0.718 V. Under the relative permittivity of silicon, r = 11.7 and the
conditions stated above, the closest permittivity of free space 0 = 8.85 × 10–12 F/m.
approximation of the ratio of reverse saturation The depletion capacitance of the diode per
current in germanium diode to that in silicon square meter is
diode is (a) 100 µ (b) 10 µ
(a) 1 (b) 5 (c) 1 µ (d) 20 µ
(c) 4 × 103 (d) 8 × 103 [EC-2005 : 2 Marks]
[EC-2003 : 2 Marks]
GATE Previous Years Solved Paper 21

Q.25 The values of voltage (VD) across a tunnel-diode (b) the edge of the depletion region on the
corresponding to peak and valley currents are n-side.
Vp and Vv respectively. The range of tunnel- (c) the p+n junction.
diode voltage VD for which the slope of its I – VD (d) the center of the depletion region on the
characteristics is negative would be n-side.
(a) VD < 0 (b) 0 VD < Vp [EC-2007 : 1 Mark]
(c) Vp VD < VV (d) VD VV
Q.29 Group-I lists four types of p-n junction diodes.
[EC-2006 : 1 Mark]
Match each device in Group-I with one of the
Q.26 In the circuit shown below, the switch was options in Group-II to indicate the bias
connected to position 1 at t < 0 and at t = 0, it is condition of that device in its normal mode of
changed to position 2. Assume that the diode operation:
has zero voltage drop and a storage time ts. For Group-I Group-II
0 < t ts, vR is given by (all in Volts) P. Zener diode 1. Forward bias
1 Q. Solar cell 2. Reverse bias
R. LASER diode
+
2 S. Avalanche photodiode
(a) P-1, Q-2, R-1, S-2
5V
1k vR
(b) P-2, Q-1, R-1, S-2
(c) P-2, Q-2, R-2, S-1
– (d) P-2, Q-1, R-2, S-2
[EC-2007 : 2 Marks]
(a) vR = –5 (b) vR = +5
(c) 0 vR < 5 (d) –5 < vR < 0 Q.30 Group-I lists four different semiconductor
[EC-2006 : 2 Marks] devices. Match each device in Group-I with its
characteristic property in Group-II:
Q.27 Find the correct match between Group-I and
Group-I Group-II
Group-II:
P. BJT 1. Population inversion
Group-I Group-II
Q. MOS capacitor 2. Pinch-off voltage
E. Varactor diode 1. Voltage reference
R. LASER diode 3. Early effect
F. PIN diode 2. High-frequency switch
S. JFET 4. Flat-band voltage
G. Zener diode 3. Tuned circuits
(a) P-3, Q-1, R-4, S-2
H. Schottky diode 4. Current controlled
(b) P-1, Q-4, R-3, S-2
attenuator
(c) P-3, Q-4, R-1, S-2
(a) E-4, F-2, G-1, H-3
(d) P-3, Q-2, R-1, S-4
(b) E-2, F-4, G-1, H-3
[EC-2007 : 2 Marks]
(c) E-3, F-4, G-1, H-2
(d) E-1, F-3, G-2, H-4 Q.31 A p+n junction has a built-in potential of 0.8 V.
[EC-2006 : 2 Marks] The depletion layer width at a reverse bias of
1.2 V is 2 µm. For a reverse bias of 7.2 V, the
Q.28 In a p+n junction diode under reverse bias, the depletion layer width will be
magnitude of electric field is maximum at
(a) 4 µm (b) 4.9 µm
(a) the edge of the depletion region on the
(c) 8 µm (d) 12 µm
p-side.
[EC-2007 : 2 Marks]
22 Electronics Engineering Electronic Devices & Circuits

Q.32 Which of the following is not associated with a (c) 1.80 MV-cm–1, directed from p-region to
p-n junction? n-region.
(a) Junction capacitance (d) 1.80 MV-cm–1, directed from n-region to
(b) Charge storage capacitance p-region.
(c) Depletion capacitance [EC-2009 : 2 Marks]
(d) Channel length modulation
Q.36 Compared to a p-n junction with
[EC-2008 : 1 Mark]
NA = ND = 1014/cm3, which one of the following
Q.33 Consider the following assertions: statements is true for a p-n junction with
S1 : For Zener effect to occur, a very abrupt NA = ND = 1020/cm3?
junction is required. (a) Reverse breakdown voltage is lower and
S2 : For quantum tunneling to occur, a very depletion capacitance is lower.
narrow energy barrier is required. (b) Reverse breakdown voltage is higher and
Which of the following is correct? depletion capacitance is lower.
(a) Only S2 is true. (c) Reverse breakdown voltage is lower and
(b) S1 and S2 are both true but S2 is not a reason depletion capacitance is higher.
for S1. (d) Reverse breakdown voltage is higher and
(c) S1 and S2 are both true and S2 is a reason for depletion capacitance is higher.
S1. [EC-2010 : 2 Marks]
(d) Both S1 and S2 are false.
Q.37 A silicon pn-junction is forward biased with a
[EC-2008 : 2 Marks]
constant current at room temperature. When the
Common Data for Questions (34 and 35): temperature is increased by 10°C, the forward
Consider a silicon p-n junction at room temperature bias voltage across the pn-junction
having the following parameters: (a) increases by 60 mV
Doping on the n-side = 1 × 1017 cm–3 (b) decreases by 60 mV
Depletion width on the n-side = 0.1 µm (c) increases by 25 mV
Depletion width on the p-side = 1.0 µm (d) decreases by 25 mV
Intrinsic carrier concentration = 1.4 × 1010 cm–3 [EC-2011 : 1 Mark]
Thermal voltage = 26 mV
Q.38 A Zener diode, when used in voltage
Permittivity of free space = 8.85 × 10–14 F-cm–1
stabilization circuits, is biased in
Dielectric constant of silicon = 12
(a) reverse bias region below the breakdown
Q.34 The built-in potential of the junction voltage.
(a) is 0.70 V. (b) reverse breakdown region.
(b) is 0.76 V. (c) forward bias region.
(c) is 0.82 V. (d) forward bias constant current mode.
(d) cannot be estimated from the data given. [EC-2011 : 1 Mark]
[EC-2009 : 2 Marks]

Q.35 The peak electric field in the device is Q.39 In a forward biased pn-junction, the sequence
(a) 0.15 MV-cm–1, directed from p-region to of events that best describes the mechanism of
n-region. current flow is
(b) 0.15 MV-cm–1, directed from n-region to (a) injection, and subsequent diffusion and
p-region. recombination of minority carriers.
GATE Previous Years Solved Paper 23

(b) injection, and subsequent drift and (a) both the P-region and the N-region are
generation of minority carriers. heavily doped.
(c) extraction, and subsequent diffusion and (b) the N-region is heavily doped compared to
generation of minority carriers. the P-region.
(d) extraction, and subsequent drift and (c) the P-region is heavily doped compared to
recombination of minority carriers.
the N-region.
[EC-2013 : 1 Mark]
(d) an intrinsic silicon region is inserted
Q.40 When the optical power incident on a between the P-region and the N-region.
photodiode is 10 µW and the respectively is [EC-2015 : 1 Mark]
0.8 A/W, the photocurrent generated (in µA), is
________ . Q.44 The built-in potential of an abrupt p-n junction
[EC-2014 : 1 Mark] is 0.75 V. If its junction capacitance (CJ) at a
reverse bias (VR) of 1.25 V is 5 pF, the value of CJ
Q.41 Consider an abrupt PN junction (at T = 300 K)
(in pF) when VR = 7.25 V is ______ .
shown in the figure. The depletion region width
[EC-2015 : 2 Marks]
xn on the N-side of the junction is 0.2 µm and the
permittivity of silicon ( si) is 1.044 × 10–12 F/cm. Q.45 For a silicon diode with long P and N regions,
At the junction, the approximate value of the the acceptor and donor impurity concentrations
peak electric field (in kV/cm) is _______ . are 1 × 1017 cm–3 and 1 × 1015 cm–3, respectively.
The lifetimes of electrons in P-region and holes
in N-region are both 100 µs. The electron and
+
P -region N-region
xn 16 3 hole diffusion coefficients are 49 cm2/s and
NA >> ND NA = 10 /cm
36 cm2/s, respectively. Assume kT/q = 26 mV,
[EC-2014 : 2 Marks] the intrinsic carrier concentration is
1 × 1010 cm–3, and q = 1.6 × 10–19 C. When a
Q.42 The donor and acceptor impurities in an abrupt
forward voltage of 208 mV is applied across the
junction silicon diode are 1 × 1016 cm–3 and
diode, the hole current density (in nA/cm)2
5 × 1018 cm–3, respectively. Assume that the
injected from P-region to N-region is ______ .
intrinsic carrier concentration in silicon
[EC-2015 : 2 Marks]
kT
ni = 1.5 × 1010 cm–3 at 300 K, =26 mV & the Q.46 The electric field profile in the depletion region
q
of a p-n junction in equilibrium is shown in the
permittivity of silicon si = 1.04 × 10–12 F/cm.
The built-in potential and the depletion width figure. Which one of the following statements is
of the diode under thermal equilibrium not true?
conditions, respectively, are E(V/cm)
(a) 0.7 V and 1 × 10–4 cm 10
4

(b) 0.86 V and 1 × 10–4 cm


(c) 0.7 V and 3.3 × 10–5 cm
(d) 0.86 V and 3.3 × 10–5 cm
[EC-2014 : 2 Marks]
X(µm)
–0.1 0 0.5 1.0
Q.43 A region of negative differential resistance is
observed in the current voltage characteristic of (a) The left side of the junction is n-type and
the right side is p-type.
a silicon PN junction if
24 Electronics Engineering Electronic Devices & Circuits

(b) Both the n-type and p-type depletion regions I


are uniformly doped.
(c) The potential difference across the depletion
region is 700 mV. X Y Z
(d) If the p-type region has a doping
concentration of 1015 cm–3, then the doping
concentration in the n-type region will be V
1016 cm–3.
[EC-2015 : 2 Marks] (a) EgX > EgY > EgZ
(b) EgX = EgY = EgZ
Q.47 Consider a silicon p-n junction with a uniform
(c) EgX < EgY < EgZ
acceptor doping concentration of 1017 cm–3 on
(d) no relationship among these band gaps
the p-side and a uniform donor doping
concentration of 1016 cm–3 on the n-side. No exists
external voltage is applied to the diode. [EC-2016 : 1 Mark]
Given: kT/q = 26 mV, ni = 1.5 × 1010 cm–3, Q.50 The figure shows the I-V characteristics of a
–14 F/m & q = 1.6 × 10–19 C.
si = 12 0, 0 = 8.85 × 10 solar cell illuminated uniformly with solar light
The charge per unit junctions area (nC cm–2) in
of power 100 mW/cm2. The solar cell has an
the depletion region on the p-side is ______ .
area of 3 cm2 and a fill factor of 0.7. The maximum
[EC-2016 : 2 Marks]
efficiency (in %) of the device is ______ .
Q.48 Consider avalanche breakdown in a silicon p+n I
junctions. The n-region is uniformly doped with
a donor density ND. Assume that breakdown ISC = 180 mA
occurs when the magnitude of the electric field
at any point in the device becomes equal to the
critical field Ecrit. Assume Ecrit to be independent
of ND. If the built-in voltage of the p+n junction is
much smaller than the breakdown voltage, VBR, V
(0, 0) VOC = 0.5 V
the relationship between VBR and ND is given by
[EC-2016 : 1 Mark]
(a) VBR × N D = constant
Q.51 Consider a region of silicon devoid of electrons
(b) N D × VBR = constant
and holes, with an ionized donor density of
(c) ND × VBR = constant
(d) ND/VBR = constant N d+ = 1017 cm 3 . The electric field at x = 0 is
[EC-2016 : 2 Marks] 0 V/cm and the electric filed at x = L is 50 kV/cm
in the positive x-directions. Assume that the
Q.49 The I-V characteristics of three types of diodes
electric field is zero in the y and z-directions at
at the room temperature, made of
all points.
semiconductors X, Y and Z are shown in the
figure. Assume that the diodes are uniformly
doped and identical in all respects except their
materials. If EgX, EgY, and EgZ are the band gaps N d+ = 1017 cm 3

of X, Y and Z, respectively, then

x=0 x=L
GATE Previous Years Solved Paper 25

Given: q = 1.6 × 10–19 Coulomb, 0 = 8.85 × 10–14 Q.55 A abrupt pn-junction (located at x = 0) is
F/cm, r = 11.7 for silicon, the value of L (in nm) uniformly doped on both p and n sides. The
is _______ . width of the depletion region is W and the
[EC-2016 : 2 Marks] electric field variation in the x-direction is E(x).
Which of the following figures represents the
Q.52 An n + -n silicon device is fabricated with
electric field profile near the pn-junction?
uniform and non-degenerate donor doping
E(x )
concentrations of N D1 = 1 × 10 18 cm–3 and
ND2 = 1 × 1015 cm–3 corresponding to the n+ and n-side p-side
n regions respectively. At the operational
(a)
temperature T, assume complete impurity
ionization, kT/q = 25 mV, and intrinsic carrier (0, 0)
x
concentration to be ni = 1 × 1010 cm–3. What is
W
the magnitude of the built-in potential of this
device? E(x )
(a) 0.748 V (b) 0.460 V
n-side p-side
(c) 0.288 V (d) 0.173 V
[EC-2017 : 1 Mark] (b)

Q.53 As shown, two silicon (Si) abrupt p-n junction (0, 0)


x
diodes are fabricated with uniform donor W
doping concentrations of ND1 = 1014 cm–3 and
E(x )
ND2 = 1016 cm–3 in the n-regions of the diodes ,
and uniform acceptor doping concentrations of
n-side p-side
NA1 = 1014 cm–3 and NA2 = 1016 cm–3 in the
p-regions of the diodes, respectively. Assuming W
(c) x
that the reverse bias voltage is >> built-in (0, 0)
potential of the diodes, the ratio C2/C1 of their
reverse bias capacitances for the same applied
reverse bias, is _______ .

p n p n E(x )
14 –3 14 –3 16 –3 16 –3
10 cm 10 cm 10 cm 10 cm
n-side p-side

C1 C2
W
Diode 1 Diode 2 (d) x
(0, 0)
[EC-2017 : 2 Marks]

Q.54 For a particular intensity of incident light on a


silicon pn-junction solar cell, the photocurrent
density (JL) is 2.5 mA/cm2 and the open-circuit [EC-2017 : 2 Marks]
voltage (VOC) is 0.451 V. Consider thermal
voltage (VT) to be 25 mV. If the intensity of the Q.56 In a p-n junction diode at equilibrium, which
incident light is increased by 20 times, assuming one of the following statements is not true?
that the temperature remains unchanged, VOC (a) The hole and electron diffusion current
(in Volts) will be ________ . components are in the same direction.
(b) The hole and electron drift current
[EC-2017 : 2 Marks]
components are in the same direction.
26 Electronics Engineering Electronic Devices & Circuits

(c) On an average, holes and electrons drift in Q.61 Which one of the following options describes
opposite direction. correctly the equilibrium band diagram at
(d) On an average, electrons drift and diffuse T = 300 K of a silicon pnn+ p++ configuration
in the same direction. shown in the figure?
[EC-2018 : 1 Mark]
p n n+ p++
Q.57 A p-n step junction diode with a contact
potential of 0.65 V has a depletion width of 1 µm EC
at equilibrium. The forward voltage (in Volts),
correct to two decimal places at which this EF
(a)
width reduces to 0.6 µm is ______ .
EV
[EC-2018 : 1 Mark]

Q.58 Red (R), Green (G) and Blue (B) Light Emitting
Diodes (LEDs) were fabricated using p-n EC
junctions of there different inorganic
semiconductors having different band-gaps. (b)
EF
The built-in voltages of red, green and blue
diodes are VR, VG, VB respectively. EV

Assume donor and acceptor doping to be the


same (NA and ND respectively) in the p and n
EC
sides of all the three diodes.
Which one of the following relationships about EF
(c)
the built-in voltages is true?
(a) VR > VG > VB (b) VR < VG < VB
(c) VR = VG = VB (d) VR > VG < VB EV
[EC-2018 : 2 Marks]
EC
Q.59 p–
A junction is made between Si with doping
EV
density NA1 = 1015 cm–3 and p Si with doping
(d)
density NA2 = 1017 cm–3. EF
Given: Boltzmann constant K = 1.38 × 10–23 J-K–1,
electronic charge q = 1.6 × 10–19 C. Assume 100%
acceptor ionization. [EC-2019 : 1 Mark]
At room temperature (T = 300 K), the magnitude
of the built-in potential (in Volts, correct to two Q.62 In an ideal pn-junction with an ideality factor of
decimal places) across this junction will be ____. 1 at T = 300 K, the magnitude of the reverse-bias
voltage required to reach 75% of its reverse
[EC-2018 : 2 Marks]
saturation current, rounded off to 2 decimal
Q.60 A solar cell of area 1.0 cm2, operating at 1.0 sun places, is ______ mV. [k = 1.38 × 10–23 J-K–1,
intensity, has a short-circuit current of 20 mA, h = 6.625 × 10–34 J-s, q = 1.602 × 10–19 C]
and an open-circuit voltage of 0.65 V. Assuming [EC-2019 : 2 Marks]
room temperature operation and thermal and
equivalent voltage of 26 mV, the open-circuit Q.63 A Germanium sample of dimensions 1 cm × 1 cm
voltage (in Volts, correct to two decimal places) is illuminated with a 20 mW. 600 mm laser light
at 0.2 sun intensity is _______ . source as shown in the figure. The illuminated
[EC-2018 : 2 Marks] sample surface has a 100 nm of loss-less silicon
GATE Previous Years Solved Paper 27

dioxide layer that reflects one-fourth of the Q.66 A one-sided abrupt pn-junction diode has a
incident light. From the remaining light, one- depletion capacitance of CD of 50 pF at a reverse
third of the power is reflected from the silicon 2 versus the
bias of 0.2 V. The plot of 1/C D
dioxide Germanium interface, one-third is
absorbed in the Germanium layer, and one-third applied voltage V for this diode is a straight
is transmitted through the other side of the line as shown in the figure below. The slope of
sample. If the absorption coefficient of the plot is _____ × 1020 F–2 V–1.
Germanium at 600 nm is 3 × 104 cm–1 and the 2
1/C D
bandgap is 0.66 eV, the thickness of the
Germanium layer, rounded off to 3 decimal
places, is ______ µm.

20 mW, 600 nm

1 cm V
0
1 cm
(a) –1.2 (b) –5.7
Silicon dioxide 100 nm (c) –3.8 (d) –0.4
Germanium T [EC-2020 : 2 Marks]

[EC-2019 : 2 Marks] Q.67 A pn-junction solar cell of area 1.0 cm 2 ,


illuminated uniformly with 100 mW cm–2, has
Q.64 The quantum efficiency ( ) and responsively (R) the following parameters: Efficiency = 15%,
at a wavelength (in µm) in a p-i-n photodetector open-circuit voltage = 0.7 V, fill factor = 0.8 and
are related by thickness = 200 µm. The charge of an electron is
1.24 × × 1.6 × 10–19 C. The average optical generation
(a) R = (b) R =
1.24 rate (in cm–3 S–1) is
1.24 (a) 1.04 × 1019 (b) 83.60 × 1019
(c) R = (d) R =
× 1.24 × (c) 0.84 × 1019 (d) 5.57 × 1019
[EC-2019 : 2 Marks] [EC-2020 : 2 Marks]

Q.65 Consider the recombination process via bulk Q.68 The energy band diagram of a p-type
traps in a forward biased pn-homojunction rate semiconductor bar of length ‘L’ under
is Umax. If the electron and the hole capture cross- equilibrium condition (i.e., the Fermi energy level
section are equal, which one of the following is EF is constant) is shown in the figure. The
false? valance band EV is sloped since doping is non-
(a) With all other parameters unchanged, Umax uniform along the bar. The difference between
decreases if the intrinsic carrier density is the energy levels of the valence band at the two
reduced. edges of the bar is .
(b) With all other parameters unchanged, Umax
p-type
increases if the thermal velocity of the
carriers increases. EF
EV
(c) Umax is occurs at the edges of the depletion
region in the device.
(d) Umax depends exponentially on the applied
z=0 z=L
bias.
[EC-2020 : 1 Mark]
28 Electronics Engineering Electronic Devices & Circuits

If the change of an electron is q, then the (a) 226 meV (b) 174 meV
magnitude of the electric field developed inside (c) 218 meV (d) 182 meV
this semiconductor bar is [EC-2022]
2 3
(a) (b) Q.71 A p-type semiconductor with zero electric field
qL 2qL
is under illumination (low level injection) in
steady-state condition. Excess minority carrier
(c) (d)
2qL qL density is zero at x = ±2In, where ln = 10–4 cm is
[EC-2021 : 1 Mark] the diffusion length of electrons. Assume
electronic charge, q = –1.6 × 10–19 C. The profiles
Q.69 A silicon P-N junction is shown in the figure. of photo-generation rate of carriers and the
the doping in the ‘P’ region is 5 × 1016 cm–3 and recombination rate of excess minority carriers
doping in the ‘N’ region is 10 × 1016 cm–3. The (R) are shown. Under these conditions, the
parameters given are: magnitude of the current density due to the
Built-in voltage ( bi) = 0.8 V photo-generated electrons at x = ±2ln is ______
Electron charge (q) = 1.6 × 10–19 C mA/cm2 (Rounded off to two decimal places).
Vacuum permittivity ( 0) = 8.85 × 10–12 F/m
Relative permittivity of silicon ( si) = 12 Semiconductor

20
P N Photo generation 10

–3
Rate (cm /s) 0
1.2 µm 0.2 µm x
–2ln –ln 0 ln 2ln
The magnitude of reverse bias voltage that
would completely deplete one of the two regions
(P or N) prior to the other (Rounded off to one R = 1020 exp
x
Excess minority ln
decimal place) is _______ V. carrier recombination 10
20

–3
[EC-2021 : 2 Marks] rate (cm /s)

Q.70 In a non-degenerate bulk semiconductor with


0
electron density n = 1016 cm–3, the value of x
–2ln –ln 0 ln 2ln
EC – EFn = 200 meV, where EC and EFn denote the
bottom of the conduction band energy and
[EC-2022]
electron Fermi level energy, respectively.
Assume thermal voltage as 26 meV and the
intrinsic carrier concentration is 1010 cm–3. For
n = 0.5 × 1016 cm–3, the closest approximation of
the value of (EC – EFn), among the given options,
is ______ .
GATE Previous Years Solved Paper 29

Answers PN - Junction Diodes and Special Diodes

1. (b) 2. (d) 3. (b) 4. (*) 5. (c) 6. (b) 7. (a) 8. (b)

9. (Sol.) 10. (a, b) 11. (d) 12. (a) 13. (c) 14. (b) 15. (c) 16. (b)

17. (c) 18. (c) 19. (c) 20. (a) 21. (b) 22. (d) 23. (b) 24. (b)

25. (c) 26. (a) 27. (c) 28. (c) 29. (b) 30. (c) 31. (a) 32. (d)

33. (a) 34. (b) 35. (b) 36. (c) 37. (d) 38. (b) 39. (a) 40. (8)

41. (–30.65) 42. (d) 43. (a) 44. (2.5) 45. (0.5147) 46. (c) 47. (–48) 48. (c)

49. (c) 50. (21) 51. (32.36) 52. (d) 53. (10) 54. (0.526) 55. (a) 56. (d)

57. (0.42) 58. (b) 59. (0.12) 60. (0.61) 61. (d) 62. (35.83) 63. (0.231) 64. (b)

65. (c) 66. (*) 67. (c) 68. (d) 69. (8.2 V) 70. (c) 71. (0.59)

Solutions PN - Junction Diodes and Special Diodes

1. (b) Diode D1 is in forward bias.


Decreases with decreasing current and Diode D2 is in reverse bias.
increasing temperature. So, the current through diode D1 is forward bias
current If and current through diode D2 is
Diffusion capacitance = CD = reverse current Io.
r
So, total current = I = If + Io
VT
r= V / VT
If I = Io ( e d 1) = I o eVd / VT
Io

If If
= I = ( I o eVd / VT Io ) + Io
CD =
VT kT
CD If I = I o eVd / VT
1 I
CD eVd / VT =
T Io
2. (d) Vd I
= ln
1-A, 2-C, 3-B VT Io
Avalanche breakdown Collision of carriers
with crystal ions. I
Vd = VT ln
Io
Zener breakdown Rupture of covalent bond
due to strong electric field. KT I
Punch through Early effect Vd = ln [For Ge, = 1]
q Io
3. (b) Vd = V

KT I KT I
V= ln V= ln
q Io q Io
30 Electronics Engineering Electronic Devices & Circuits

4. (*)
9. Sol.
The lifetime of majority carriers in the N-region.
When diode instantaneously switched from a
The switching speed of a P+N (heavily doped
conduction state it needs some time to return to
p-region) junction depends on the lifetime ( ) of
non-conduction state, so diode behaves as short
minority carriers (holes) in the N-region (lightly
circuit for the little time, even in reverse direction.
doped region).
This is due to accumulation of stored excess
5. (c) minority carrier charge when diode is forward
Both P and N-regions are heavily doped. biased.
In a Zener diode P and N both the regions are Time required to return back to state of non-
heavily doped. conduction is ‘Reverse recovery time’ which is
Doping level of Zener diode is 1 : 105. ‘storage time’ and ‘transition time’.
• Storage time is the period for which diode
6. (b) remains in conduction state even in reverse
The depletion capacitance decreases with direction.
increase in the reverse bias. • Transition time is time elapsed in returning
Depletion width = W back to state of non-conduction.
W VRB For 0 < t < to, Vm = 20 V, ‘D’ is forward biased.

W Reverse bias voltage Vm 20


i= = = 2 mA
R 10 k
A
Capacitance = C = For t > to, Vm = –20
W
1 During storage time: to < t < to + ts,
C Vm
W 20
i= = = 2 mA
1 R 10 k
C
Reverse bias voltage During transition time: to + ts < t < to + ts + tt
‘i’ decreases exponentially to (–Io)
7. (a) For t > to + ts + tt,
In the step graded diode, by using charge i = (–Io)
density condition or charge neutrality condition, I
WN NA
= 2 mA
WP ND
WN NA 1
= = = 0.25 to to + ts to + ts + t1
WP 4N A 4 t
–Io

8. (b)
–2 mA
For abrupt p-n junction,
C j V–1/2
1 10. (a, b)
Cj
V Increases with the increase in the doping levels
C2 V1 1+0 1 1 of the two sides. Built in potential or diffusion
= = = = potential across a p-n junction diode,
C1 V2 1 + 99 100 10
C1 1 N A ND
C2 = = = 0.1 nF/cm 2 Vo = kT ln
10 10 ni2
GATE Previous Years Solved Paper 31

So, m = VT
So, Vo ni2
x = ln (I)
Vo NA ND
c = – VT ln (Io)
So, option (a) and option (b) both are correct.
y = Vd
11. (d)
ln (If)
Increases with increase in doping concentration
contact potential of diffusion potential across a
pn-junction,

N A ND
Vo = kT ln
ni2
Vd
Vr
at constant temperature,
Vo NAND
15. (c)
12. (a) Diode is forward bias in the starting.
Tunneling of charge carriers across the junction Diode is reverse bias later.
a Zener diode works on the principle of But at t = 0 diode is reverse bias.
tunneling of charge carriers across the junction So, I = –100 mA
which leads the junction to breakdown.
16. (b)
13. (c) dV
For either Si or Ge 2 mV / °C
dT
Cj VR 1 /2
In order to maintain a constant value of I.
The depletion layer capacitance of diode is T2 – T1 = 40 – 20 = 20°C
given, 2 × 20 mV = 40 mV
C T V–n Therefore, VD = 700 – 40 = 660 mV
1
n= for step graded or abrupt p-n junction 19. (c)
2
= 1 for Germanium
CT VR 1 /2 = 2 for Silicon at low value of current

I = I o Si ( eVD1 / VT
1) ...(i)
14. (b)
(Log I versus V)
I = I o Ge ( eVD2 / VT
1) ...(ii)
I = Io [ eVd / VT 1] = forward current
LHS of (i) = (ii),
Q eVd / VT >>1
VD1 VD2
I = Io eVd / VT I o Si e VT
1 = I o Ge e VT
1
I
eVd / VT =
Io VD1 0.718
3
I oGe e VT
1 e 2 × 26 × 10 1
Vd I = =
= ln = ln ( I ) ln ( I o ) I oSi VD2
e
0.1435
1
VT Io e VT
1 26 × 10 3
e
Vd = VT ln(I) – VT ln(Io)
= 4 × 103
y = mx + C
32 Electronics Engineering Electronic Devices & Circuits

20. (a) 26. (a)


1.24 When diode is instantaneously switched from
Eg = eV
(µm) a conduction state it needs time to return to non-
conduction state, so behaves short-circuited for
1.24
= 4
= 2.26 eV a small time period in reverse direction.
5490 × 10 µm
Storage time: The time period for which diode
21. (b) remains in conduction state even in the reverse
direction so,
n NA
= ; n ( n >> p)
VR = –5 V
p ND
For t < 0,
n × ND 3 µm × 1016
= = = 0.3 µm
p NA 9 × 1016
+ +
5V 1k VR = 5 Volt
22. (d) – –

Cj Vj 1/2
For 0 < t < ts,
C j2 V1 1 1
= = =
C j1 V2 4 2
– +
C j1 1 pF
C j2 = = = 0.5 pF 5V 1k VR = –5 Volt
I
2 2 + –

23. (b)
We know that, 28. (c)
(T2 T1 ) Electrical field always maximum at the junction.
I o(T ) = I o(T ) × 2 10
2 1
31. (a)
I o(40°C) = I o
(20°C)
× 22 Junction potential = Built-in potential + Reverse
I o(40°C) = 40 pA bias voltage,
Vj = Vo + VR
24. (b) Now for abrupt pn-junction depletion width,

A W V j1/2
o r
C=
d 1/2
W = KVj
C 8.85 × 10 12 × 11.7 2 µm = K(0.8 + 1.2)1/2 ...(i)
= = 10.35 µF
A 10 × 10 6 x = K(0.8 + 7.2)1/2 ...(ii)
From equation (i) and (ii),
25. (c)
x = 4 µm
I
e

34. (b)
nc
ta

IP
is

N side is heavily doped,


es
er
–v

1/2
2 Vj
WN =
IV eN D
V
VP VV
eN D WN2
Vj = 0.76 Volts
2 o r
GATE Previous Years Solved Paper 33

35. (b) 40. Sol.


qN o Ip
Electric field, E= (x ) Respectively (R) =
Po
At the center, x = 0 where, Ip = Photo current
qN o Po = Incident power
So, Emax = ( n) Ip = R × Po = 8 µA

1.6 × 10 19 × 1017 × 10 5 41. Sol.


= 14
12 × 8.85 × 10 For abrupt P+N junction
= –0.15 MV/cm Emax is given:
qN D xn
36. (c) Emax =
s
With increase in doping, the reverse breakdown
voltage decreases and depletion width ‘d’ also [Q NA >> ND, W xn]
decreases. Since depletion capacitance, 1.6 × 10 19 × 10 16 × 0.2 × 10 4
Emax =
C = A/d, so depletion capacitance increases 1.044 × 10 12
with increase in doping. Emax = –30.65 × 103 V/cm
Emax = –30.65 kV/cm
37. (d)
dV 42. (d)
= –2.5 mV/°C Built-in potential is given by,
dT
For 10°C voltage across pn-junction will N AND
decreases by 25 mV. Vbi = VT ln
ni2
38. (b)
1016 × 5 × 1018
Vbi = 0.026 ln = 0.86 V
Break-down voltage V 2.25 × 10 20
and depletion width W is given by,

2 1 1
6V W= + Vbi
V q N A ND

2 × 1.04 × 10 12 1 1
W= 19 18
+ × 0.86
1.6 × 10 5 × 10 10 16
Reverse Reverse bias region below W = 3.3 × 10–5 cm
breakdown the breakdown voltage
region 44. Sol.

In breakdown region only, Zener diode is useful. 1


Cj
In reverse bias region below the breakdown Vbi + VR
voltage it will behave like an open-circuit. C2 j Vbi + VR1
=
39. (a) C1 j Vbi + VR2
In a forward biased pn-junction diode, the 2 C1 j
current flow is due to diffusion of majority C 2j = C1 j = = 2.5 pF
8 2
carriers and recombination of minority carriers.
So, answer is 2.5.
34 Electronics Engineering Electronic Devices & Circuits

45. Sol. WN D 3.3255 × 10 6 × 1016


WP = =
The hole current density injected from P region N A + ND 1016 + 1017
to N region is given by = 0.3023 µ-cm
q ni2 Dp Charge per unit junction area in the depletion
VFB
exp 1 layer on p-side is
N D Lp VT
= –qNA + WP
where, = –1.6 × 10–19 × 1017 × 0.3023 × 10–6
q = Charge on electron = –4.8368 nC/cm2
ni = Intrinsic carrier concentration is silicon
ND = Donor doping 48. (c)
Dp = Hole diffusion coefficient In any type of PN-junction,
Lp = Mean diffusion length of hole 1
VBr
VFB = Forward voltage applied across diode Doping concentration
kT 1
VT = = 26 mV i.e., VBr
q ND

Lp = p Dp E2
or, VBr =
2qND
= 6
100 × 10 × 36 = 0.06 cm VBr × ND is a constant.
Using the above values, we get hole current
49. (c)
density injected from P region to N region is
X Ge diode ............... EG = 0.2 V
= 0.5147 nA/ cm2
Y Si diode ............... EG = 0.7 V
46. (c) Z GaAs diode (or) LED ............... EG = 1.3 V
Built-in potential,
50. Sol.
1
0 = 2
× (106 V/m) × (1.1 × 10 6 m) Maximum obtained power
Fill factor =
VOC I SC
= 0.55 Volts
Maximum obtained power
But in question [option (c)] is given as, 700 mV. 0.7 = 3
0.5 × 180 × 10
47. Sol. Maximum obtained power,
N A ND Pm = 0.063 W
Vo = VT ln
ni2 Pm 0.063
Efficiency, = = = 21%
G A 100 × 10 3 × 3
3 1016 × 1017 G Input light in W/cm2
= 26 × 10 ln
(1.5 × 10 10 )2
51. Sol.
Vo = 0.757 V
q
= ND XN
2 1 1
W= + Vo
q N A ND
1.6 × 10 19
5 × 103 = × 1017 × X N
2 × 8.854 × 10 16
× 12 1 1
8.85 × 10 14 × 11.7
= + 0.757
1.6 × 10 19 1016 10 17 On solving, XN = 3.2356 × 10–6 cm
= 3.3255 µ-cm = 3.236 × 10–8 m
XN = L = 32.36 nm
GATE Previous Years Solved Paper 35

52. (d)
E(x )
n N n-side p-side
Vo = VT ln 1 = VT ln D1
n2 N D2 x NDq
xn =
N Aq
xp
v (x )
E( x ) = dx s s
18 s
10
= 0.25 ln 15
= 0.025 ln (1000)
10 Electric field
= 0.173 V profile x
–xn 0 xp

53. Sol. W

2 VR 1 1
W= + (Q VR>>Vbi) 56. (b)
e N A ND
Hole diffusion
Under reverse biased condition, p n
Electron diffusion
A 1
C = C Hole drift
W W
Electron drift
2 VR 2
C2 Hole diffusion current
W1 e 10 14
So, C 1 = W2 = = 10 Electron diffusion current
2 VR 2
e 10 16 Hole drift current
Electron drift current
54. Sol.
57. Sol.
J
VOC = VT ln L (JL Light intensity)
JS 2 1 1
Wdeap = + (Vbi VAK )
q N A ND
JL2
VOC2 VOC1 = VT ln
JS (0.65 VAK ) 0.6 µm
= = 0.6
VOC2 VOC1 = 25 ln20 75 mV = 0.075 V 0.65 1 µm

VOC2 = 0.451 V + 0.075 V = 0.526 V VAK


1 = 0.36
0.65
55. (a) VAK = 0.65 (1 – 0.36)
= 0.65 × 0.64
ND – NA = 0.416 V
n-side p-side 0.42 V
Doping ND
profile x 58. (b)

–NA R > G> B

1
Energy gap, Eg =
v (x )
So, EgR < EgG < EgB
n-side NDq p-side
Materials with high energy gap will have high
Charge xp built-in voltages, when doping concentration
density x
–xn
are same.
–NAq So, VR < VG < VB
36 Electronics Engineering Electronic Devices & Circuits

59. Sol. 64. (b)


Built-in potential, I out h f
= ×
kT N A2 q Pin
Vbi = ln
q N A1 I out
R=
1.38 × 3 Pin
= ln (100) V
1.6 × 100 q q
So, R= × = ×
= 0.1192 V 0.12 V hf hc

60. Sol. If is given in µm, then

For solar cell, q × 10 6


R= ×
hc
I SC
VOC = VT ln
Io q × 10 6
1.24
hc
I SC 2 0.20
VOC2 VOC1 = VT ln = VT ln
I SC1 1.0 So, R=
1.24
VOC2 = VOC1 0.026 ln (5) 66. ( )
= 0.65 – 0.041845
= 0.608 V 1
y= 2
CD
62. Sol. 1
y2 = = 4 × 1020
23 (50 × 10 12 )2
kT 1.38 × 10 × 300
VT = = V
q 1.602 × 10 19
= 25.843 mV
y1 = 0
V /VT 3
I = Io ( e 1) = Io x2 = –0.2 V x1 = Vbi x=V
4
V = VT ln 0.25 = –35.83 mV Depletion or transition capacitance is,
VR = V = 35.83 mV A
CD =
W
63. Sol.
For one-sided PN-junction (Ex: P+N junction)
Pabsorbed = Pincident (1 – e– T)
2 VB 2 (Vbi V )
1 2 T W= =
= (1 e ) eN D eN D
3 3
where V is anode to cathode applied potential.
2 T 1
e = A
3 3 CD =
2 (Vbi V )
where, = 3 × 104 cm–1, absorption coefficient
eN D
of Ge sample.
1 1 2
T = ln (2) = (Vbi V )
2 2
CD A eN D
1
= ln(2) cm 1
3 × 10 4 2
becomes zero at V = Vbi
CD
= 0.231 µm
GATE Previous Years Solved Paper 37

From above graph, We know that,


1 xnND = xpNA
y = 2
= 0 at x1 = Vbi
CD ND xp
=
1 NA xn
and y2 = 2
= 4 × 10 20 at x1 = –0.2 V
CD ND
=2 xp = 2xn
y2 y1 4 × 10 20 NA
0
Slope = =
x2 x1 0.2 Vbi As, xn = 0.2 µm xp = 0.4 µm
Q Vbi is not provided, slope cannot be found. So, width of depletion region W = 0.6 µm,

67. (c) 2 (Vo + VR ) 1 1


W= +
q N A ND
(FF ) VOC I SC
= 14
Pin 0.6 × 10–4 =
2 × 12 × 8.85 × 10
19
(0.8 + VR ) 1
+
1
1.6 × 10 5 × 10 16 10 × 1016
0.8 × 0.7 × I SC
0.15 = 0.36 × 10 8 × 1.6 × 10 19 × 5 × 10 16
100 mW = (VR + 0.8)
2 × 12 × 8.85 × 10 14 × 1.5
15
ISC = mA VR = 8.2395 Volts
0.56
I SC 70. (c)
GL =
q × Area × Thickness NC
Given, (EC – EFn)1 = kT ln = 200 meV
15 × 10 3 N D1
= 19 4
0.56 × 1.6 × 10 × 1 × 200 × 10 ...(1)
15
= × 1019 NC
0.56 × 32 (EC – EFn)2 = kT ln ...(2)
N D2
= 0.837 × 1019/cm3/second
N D1 = 1016 cm–3
68. (d)
Relation between electric field and potential is N D2 = 0.5 × 1016 cm–3
given by
(EC – EFn)1 – (EC – EFn)2
d
E =
dx NC NC
= kT ln kT ln
N D1 N D2
d E
E = E Energy 200 meV – (EC – EFn)2
dx q
N D2
1 dE 1 = kT ln
E = = N D1
q dx q L
*Non-uniform doping in a semiconductor 1016 × 0.5
= 0.026 ln
results in built-in electric field. 1016
69. (8.2) = 0.026 ln(0.5)
NA = 5 × 1016 per cm3 = –0.01802 Volt
ND = 10 × 1016 per cm3 = –18.02 meV
(EC – EFn)2 = 200 meV + 18.02 meV
bi = 0.8 V
= 218.02 meV
38 Electronics Engineering Electronic Devices & Circuits

71. (0.59) At x = ln :
Equation (1) = Equation (2)
n(x) = R n = 1020 e x /ln n
ln
n(ln) = 1020 e–1 n ...(1) 1020 e–1 n = B 1
2 ln
ln x 2ln
Continuity equation in steady-state, B = 2 × 1020 e–1 n

2 x
Dn
n
+G R = 0 n(x) = 2 × 10 20 e 1 n 1
2 2 ln
x
G=0 ln x 2ln
Since, ln x 2ln Electron diffusion current density:
R=0
dn
2
n Jn diff. = qDn
Dn = 0 dx
x2
20 1 1
Whose solution is, = qDn × 2 × 10 × e × n 0
2ln
n(x) = Ax + B
19
Since at x = 2ln : 1.6 × 10 × ln2 × 2 × 10 20 × e 1
=
n(2ln) = 0 (Given) 2ln
0 = A(2ln) + B = 1.6 × 10–19 × ln × 1020 × e–1
B = 1.6 × 101 × 1 × 10–4 × e–1
A=
2 ln (ln = 10–4 cm)
= 0.588 mA/cm 2
B x
n(x) = x+B= B 1 = 0.59
2 ln 2ln
...(2)
3 BJT and FET Basics

Q.1 In MOSFET devices the n-channel type is better Q.6 Channel current is reduced on application of a
than the P-channel type in the following respects more positive voltage to the GATE of the
(a) it has better noise immunity. depletion mode n-channel MOSFET.
(b) it is faster. (True/False)
(c) it is TTL compatible. [EC-1994 : 1 Mark]
(d) it has better drive capability.
Q.7 The breakdown voltage of a transistor with its
[EC-1988 : 2 Marks]
bass open is BVCEO and that with emitter open
Q.2 In a MOSFET, the polarity of the inversion layer is BVCBO, then
is the same as that of the
(a) BVCEO = BVCBO
(a) charge on the GATE-EC-electrode
(b) BVCEO > BVCBO
(b) minority carries in the drain
(c) BVCEO < BVCBO
(c) majority carriers in the substrate
(d) BVCEO is not related to BVCBO
(d) majority carriers in the source
[EC-1995 : 1 Mark]
[EC-1989 : 2 Marks]
Q.8 Match the following:
Q.3 Which of the following effects can be caused by
a rise in the temperature? List-I
(a) Increase in MOSFET current (IDS) A. The current gain of a BJT will be increased.
(b) Increase in BJT current (IC) B. The current gain of a BJT will be reduced.
(c) Decrease in MOSFET current (IDS) C. The break-down voltage of a BJT will be
(d) Decrease in BJT current (IC) reduced.
[EC-1990 : 2 Marks] List-II

Q.4 In a transistor having finite , forward bias 1. The collector doping concentration is
across the base emitter junction is kept constant increased.
and the reverse bias across the collector base 2. The base width is reduced.
junction is increased. Neglecting the leakage 3. The emitter doping concentration to base
across the collector base junction and the doping concentration ratio is reduced.
depletion region generations current, the base 4. The base doping concentration is increased
current will _____ (increase/decrease/remains
keeping the ratio of the emitter doping
constant).
concentration to base doping concentration
[EC-1992 : 2 Marks]
constant.
Q.5 The threshold voltage of an n-channel MOSFET 5. The collector doping concentration is
can be increased by reduced.
(a) increasing the channel dopant [EC-1994 : 2 Marks]
concentration.
(b) reducing the channel dopant concentration. Q.9 The transit time of a current carriers through
(c) reducing the GATE oxide thickness. the channel of an FET decides it ______
(d) reducing the channel length. characteristic.

[EC-1994 : 1 Mark] [EC-1994 : 1 Mark]


40 Electronics Engineering Electronic Devices & Circuits

Q.10 A BJT is said to be operating in the saturation Q.15 The effective channel length of a MOSFET in
region if saturation decreases with increase in
(a) both the junctions are reverse biased. (a) gate voltage (b) drain voltage
(b) base-emitter junction is reverse biased and (c) source voltage (d) body voltage
base-collector junction is forward biased. [EC-2001 : 1 Mark]
(c) base-emitter junction is forward biased and
Q.16 For an n-channel enhancement type MOSFET,
base-collector junction is reverse biased.
if the source is connected at a higher potential
(d) both the junctions are forward biased. than that of the bulk (i.e. VSB > 0), the threshold
[EC-1995 : 1 Mark] voltage VT of the MOSFET will
Q.11 The early-effect in a bipolar junction transistor (a) remain unchanged
is caused by (b) decrease
(a) fast-turn-on (c) change polarity
(b) fast-turn-off (d) increase
(c) large collector-base reverse bias [EC-2003 : 1 Mark]
(d) large emitter-base forward bias Q.17 When the gate-to-source voltage (VGS) of a
[EC-1995; 1999 : 1 Mark] MOSFET with threshold voltage of 400 mV,
working in saturation is 900 mV, the drain
Q.12 If a transistor is operating with both of its
current is observed to be 1 mA. Neglecting the
junctions forward biased, but with the collector
channel width modulation effect and assuming
base forward bias greater than the emitter-base
that the MOSFET is operating at saturation, the
forward bias, then it is operating in the
drain current for an applied VGS of 1400 mV is
(a) forward active mode.
(a) 0.5 mA (b) 2.0 mA
(b) reverse saturation mode.
(c) 3.5 mA (d) 4.0 mA
(c) reverse active mode.
[EC-2003 : 2 Marks]
(d) forward saturation anode.
Q.18 If for a silicon n-p-n transistor, the base-to-
[EC-1996 : 2 Marks]
emitter voltage (VBE) is 0.7 V and the collector-
Q.13 In a bipolar transistor at room temperature, if to-base voltage (VCB) is 0.2 V, then the transistor
the emitter current is doubled, the voltage across is operating in the
its base-emitter junction (a) normal active mode
(a) doubles (b) saturation mode
(b) halves (c) inverse active mode
(c) increases by about 20 mV (d) cut-off mode
(d) decreases by about 20 mV [EC-2004 : 1 Mark]
[EC-1997 : 2 Marks]
Q.19 Consider the following statements S1 and S2:
Q.14 MOSFET can be used as a S1 : The of a bipolar transistor reduces if the
(c) current controlled capacitor. base width is increased.
(b) voltage controlled capacitor. S2 : The of a bipolar transistor increases if the
(c) current controlled inductor. doping concentration in the base is
(d) voltage controlled inductor. increased.
Which one of the following is correct?
[EC-2001 : 1 Mark]
GATE Previous Years Solved Paper 41

(a) S1 is false and S2 is true. (c) the forward biasing of emitter-base junction.
(b) Both S1 and S2 are true. (d) the early removal of stored base charge
(c) Both S1 and S2 are false. during saturation-to-cut-off switching.
(d) S1 and true and S2 is false. [EC-2006 : 1 Mark]
[EC-2004 : 1 Mark]
Common Data Questions (24 to 26):
Q.20 Consider the following statements S1 and S2: The figure shows the high frequency capacitance
S1 : The threshold voltage (V T ) of a MOS voltage (C-V) characteristics of a Metal/SiO2/Silicon
capacitor decreases with increase in gate (MOS) capacitor having an area of 1 × 10–4 cm2. Assume
oxide thickness. that the permittivities ( 0 r) of silicon and SiO2 are
S2 : The threshold voltage (V T ) of a MOS 1× 10–12 F/cm and 3.5 × 10 –13 F/cm respectively.
capacitor decreases with increase in
substrate doping concentration. C
Which one of the following is correct? 7 pF
(a) S1 is false and S2 is true.
(b) Both S1 and S2 are true.
(c) Both S1 and S2 are false.
(d) S1 and true and S2 is false. 1 pF
V
0
[EC-2004 : 2 Marks]

Q.21 The drain of an n-channel MOSFET is shorted


Q.24 The gate oxide thickness in the MOS capacitor
to the gate so that VGS = VDS. The threshold
is
voltage (VT) of MOSFET is 1 V. If the drain
current (ID) is 1 mA for VGS = 2 V, then the (a) 50 nm (b) 143 nm
VGS = 3 V, ID is (c) 350 nm (d) 1 µm
(a) 2 mA (b) 3 mA [EC-2007 : 2 Marks]
(c) 9 mA (d) 4 mA Q.25 The maximum depletion layer width in silicon
[EC-2004 : 2 Marks] is
(a) 0.143 µm (b) 0.857 µm
Q.22 A MOS capacitor made using p-type substrate
is in the accumulation mode. The dominant (c) 1 µm (d) 1.143 µm
charge in the channel is due to the presence of [EC-2007 : 2 Marks]
(a) holes Q.26 Consider the following statements about the C-V
(b) electrons characteristics plot:
(c) positively charged ions S1 : The MOS capacitor has as n-type substrate.
(d) negatively charged ions S2 : If positive charges are introduced in the
[EC-2005 : 2 Marks] oxide, the C-V plot will shift to the left.
Then which of the following is true?
Q.23 The phenomenon known as “Early effect” in a
(a) Both S1 and S2 are true.
bipolar transistor refers to a reduction of the
(b) S1 is true and S2 is false.
effective base-width caused by
(c) S1 is false and S2 is true.
(a) effective-hole recombination at the base.
(a) Both S1 and S2 are false.
(b) the reverse biasing of the base-collector
[EC-2007 : 2 Marks]
junction.
42 Electronics Engineering Electronic Devices & Circuits

Q.27 The drain current of MOSFET in saturation is (a) 450 cm2/V-s (b) 1350 cm2/V-s
given by ID = K(VGS – VT)2 where K is a constant. (c) 1800 cm2/V-s (d) 3600 cm2/V-s
The magnitude of the trans-conductance gm is [EC-2010 : 1 Mark]
2
K(VGS VT )
(a) (b) 2K(VGS – VT) Q.31 In a uniformly doped BJT, assume that NE, NB
VDS
and NC are the emitter, base and collector
ID K(VGS VT )2 dopings in atoms/cm3, respectively. If the
(c) V (d)
GS VDS VGS emitter injection efficiency of the BJT is close to
[EC-2008 : 1 Mark] unity, which one of the following conditions is
true?
Q.28 The measured trans-conductance g m of an
(a) NE = NB = NC
NMOS transistor operating in the linear region
(b) NE >> NB and NB > NC
is plotted against the gate voltage VG at a
constant drain voltage V D . Which of the (c) NE = NB and NB < NC
following figures represents the expected (d) NE < NB < NC
dependence of gm on VG? [EC-2010 : 2 Marks]

Q.32 For a BJT, the common-base current gain = 0.98


gm gm and the collector base junction reverse bias
(a) (b) saturation current ICO = 0.6 µA. This BJT is
connected in the common emitter mode and
VG VG
operated in the active region with a base drive
current IB = 20 µA. The collector current IC for
gm gm this mode of operation is
(c) (d) (a) 0.98 mA (b) 0.99 mA
(c) 1.0 mA (d) 1.01 mA
VG VG
[EC-2011 : 2 Marks]
[EC-2008 : 2 Marks]
Q.33 The source of a silicon (ni = 1010/cm3) n-channel
Q.29 Consider the following two statements about
MOS transistor has an area of 1 square µm and
the internal conditions in an n-channel
a depth of 1 µm. If the dopant density in the
MOSFET operating in the active region:
source is 1019/cm3, the number of holes in the
S1 : The inversion charge decreases from source
source region with the above volume is
to drain.
approximately.
S2 : The inversion charge increases from source
to drain. (a) 10 7 (b) 100
Which of the following is correct? (c) 10 (d) 0
(a) Only S2 is true. [EC-2012 : 2 Marks]
(b) Both S1 and S2 are false.
Common Data for Questions (34 and 35):
(c) Both S1 and S2 are true, but S2 is not a reason
for S1. In the three dimensional view of silicon n-channel MOS
(d) Both S1 and S2 are true, but S2 is a reason transistor shown below, = 20 nm. The transistor is of
for S1. width 1 µm. The depletion width formed at every p-n
[EC-2009 : 2 Marks] junctions is 10 nm. The relative permittivities of Si and
SiO 2 , respectively, are 11.7 and 3.9 and
Q.30 A room temperature, a possible value for the –12 F/m.
0 = 8.9 × 10
mobility of electrons in the inversion layer of a
silicon n-channel MOSFET is
GATE Previous Years Solved Paper 43

Q.39 An ideal MOSFET capacitor has boron doping


1 µm
concentration of 1015 cm–3 in the substitute.
When a gate voltage is applied, a depletion
G
region of width 0.5 µm is formed with a surface
1 nm (channel) potential of 0.2 V. Given that,
D
–14 F/cm and the relative
0 = 8.854 × 10
0.2 µm
D S
0.2 µm permittivities of silicon and silicon dioxide are
12 and 4, respectively, the peak electric field
0.2 µm p substrate 0.2 µm
(in V/µm) in the oxide region is ______ .
B [EC-2014 : 2 Marks]

Q.34 The source body junction capacitance is Q.40 Consider two BJTs biased at the same collector
approximately current with area A1 = 0.2 µm × 0.2 µm and
(a) 2 fF (b) 7 fF A2 = 300 µm × 300 µm. Assuming that all other
(c) 2 pF (d) 7 pF device parameters are identical kT/q = 26 mV,
the intrinsic carrier concentrations is
[EC-2012 : 2 Marks]
1 × 1010 cm–3, and q = 1.6 × 10–19 C, the difference
Q.35 The gate source overlap capacitance is between the base-emitter voltages (in mV) of the
approximately two BJTs (i.e., VBE1 – VBE2) is ________ .
(a) 0.7 fF (b) 0.7 pF [EC-2014 : 2 Marks]
(c) 0.35 fF (d) 0.24 pF
[EC-2012 : 2 Marks]
Q.41 For the MOSFET in the circuit shown, the
Q.36 A BJT is biased in forward active mode. Assume
threshold voltage is VTh, where VTh > 0. The
VBE = 0.7 V, kT/q = 25 mV and reverse saturation
source voltage VGS is varied from 0 to VDD.
current Is = 10–13 mA. The trans-conductance of
Neglecting the channel length modulation, the
the BJT (in mA/V) is _______ .
drain current I D as a function of V SS is
[EC-2014 : 2 Marks]
represented by
Q.37 A depletion type N-channel MOSFET is biased VDD
in its linear region for use as a voltage controlled
resistor. Assume threshold voltage,
VTH = –0.5 V, VGS = 2.0 V, VDS = 5. V, W/L = 100,
Cox = 10–8 F/cm2 and µn = 800 cm2/V-s. The
value of the resistance of the voltage controlled
resistor (in ) is _______ .
Vss
[EC-2014 : 2 Marks]

Q.38 The slope of the ID vs VGS curve on an n-channel


MOSFET in linear region is 10 –3 –1 at ID
VDS = 0.1 V. For the same device, neglecting
channel length modulation, the slope of the
(a)
I D vs VGS ( A /V ) under saturation region

is approximately ______ .
Vss
[EC-2014 : 2 Marks] VDD – VTh
44 Electronics Engineering Electronic Devices & Circuits

ID Q.45 If the base width in a bipolar junction transistor


is doubled, which one of the following
statements will be true?
(b) (a) Current gain will increase.
(b) Unity gain frequency will increase.
Vss (c) Emitter base junction capacitance will
VTh increase.
ID (d) Early voltage will increase.
[EC-2015 : 1 Mark]

Q.46 An n-p-n BJT having reverse saturation current


(c)
Is = 10–15 A is biased in the forward active region
with VBE = 700 mV. The thermal voltage (VT) is
Vss 25 mV and the current gain ( ) may vary from
VDD – VTh
50 to 150 due to manufacturing variations. The
ID maximum emitter current (in µA) is ______ .
[EC-2015 : 2 Marks]

(d) Q.47 Consider the following statements for a Metal


Oxide Semiconductor Field Effect Transistor
Vss (MOSFET):
VDD – VTh
P : As channel length reduces, OFF-state
[EC-2015 : 2 Marks] current increases.
Q.42 A MOSFET in saturation has a drain current of Q : As channel length reduces, output
1 mA for VDS = 0.5 V. If the channel length resistance increases.
modulation coefficient is 0.05 V–1, the output R : As channel length reduces, threshold
resistance (in k ) of the MOSFET is _______ . voltage remains constant.
[EC-2015 : 2 Marks] S : As channel length reduces, ON current
increases.
Q.43 In MOS capacitor with an oxide layer thickness Which of the above statements are incorrect?
of 10 mm. The maximum depletion layer
(a) P and Q (b) P and S
thickness is 100 mm. The permittivities of the
(c) Q and R (d) R and S
semiconductor and the oxide layer are s and ox
[EC-2016 : 1 Mark]
respectively. Assuming s/ ox = 3 the ratio of the
maximum capacitance to the minimum Q.48 Consider an n-channel metal oxide semi-
capacitance of this MOS capacitor is ______ . conductor field effect transistor (MOSFET) with
[EC-2015 : 2 Marks] a gate-to-source voltage of 1.8 V. Assume that
W/L = 4, µnCox = 70 × 10–6 AV–2, the threshold
Q.44 The current in an enhancement mode NMOS voltage is 0.3 V, and the channel length
transistor biased in saturation mode was modulation parameter is 0.09 V –1 . In the
measured to be 1 mA at a drain source voltage saturation region, the drain conductance (in µS)
was increased to 6 V while keeping gate source is ______ .
voltage same, the drain current increased to [EC-2016 : 2 Marks]
1.02 mA. Assume that drain to source saturation
voltages is much smaller than the applied drain Q.49 A long channel N-MOS transistor is biased in
source voltage. The channel length modulation the linear region with VDS = 50 mV and is used
parameter (in V–1 ), is _______ . as a resistance. Which one of the following
[EC-2015 : 2 Marks] statements is not correct?
GATE Previous Years Solved Paper 45

(a) If the device width ‘W’ is increased, the (a) inversion (b) accumulation
resistance decreases. (c) depletion (d) flat band
(b) If the threshold voltage is reduced, the [EC-2016 : 1 Mark]
resistance decreases.
(c) If the device length ‘L’ is increased, the Q.53 Figures I and II show two MOS capacitors of
resistance increase. unit area. The capacitor in Fig. (I) has insulator
(d) If VGS is increased, the resistance increases. materials X (of thickness t1 = nm and dielectric
[EC-2016 : 1 Mark] constant 1 = 4) and Y (of thickness t2 = 3 nm
and dielectric constant 2 = 20). The capacitor
Q.50 A voltage VG is applied across a MOS capacitor
in Fig. (II) has only insulator material X of
with metal gate and p-type silicon substrate at
thickness tEq. If the capacitors are of equal
T = 300 K. The inversion carrier density
capacitance, then the value of tEq (in nm) is
(in number of carriers per unit area) for VG = 0.8 V
_________ .
is 2 × 1011 cm–2. For VG = 1.3 V, the inversion
carrier density is 4 × 1011 cm–2. What is the value
Metal
of the inversion carrier density for VG = 1.8 V?
t2 2
(a) 4.5 × 1011 cm–2 (b) 6.0 × 1011 cm–2
t1 1
(c) 7.2 × 1011 cm–2 (d) 8.4 × 1011 cm–2
[EC-2016 : 2 Marks] Si

Q.51 Consider a long channel NMOS transistor with


Figure-I
source and body connected together. Assume
that the electron mobility is independent of VGS
and VDS. Given: gm = 0.5 µA/V for VDS = 50 mV Metal
and VGS = 2 V, gd = 8 µA/V for VGS = 2 V and 1 tEq
VDS = 0 V where,
ID ID
gm = and gd = Si
VGS VDS
The threshold voltage (in Volts) of the transistor
is _______ . Figure-II
[EC-2016 : 2 Marks]
[EC-2016 : 2 Marks]
Q.52 The figure shows the band diagram of a Metal
Q.54 The injected excess electron concentration
Oxide Semiconductor (MOS). The surface region
profile in the base region of an n-p-n BJT, biased
of this MOS is in,
in the active region, is linear, as shown in the
figure. If the area of the emitter base junction is
0.001 cm2, µn = 800 cm2/(V-s) in the base region
and depletion layer widths are negligible, then
SiO2
the collector current I C (in mA) at room
EM temperature is (Given: thermal voltage,
B EC VT = 26 mV at room temperature, electronic
EFS
B charge, q = 1.6 × 10–19 C).
Ei

EV
46 Electronics Engineering Electronic Devices & Circuits

Q.57 Consider an n-channel MOSFET having width


IB
W, length L, electron mobility in the channel mn
n p n
14 and oxide capacitance per unit area Cox. If gate-
10 cm–3
Excess to-source voltage VGS = 0.7 V drain-to-source
electron
IE
profile
IC voltage VDS = 0.1 V. (µn Cox) = 100 µA/V2,
threshold voltage VTh = 0.3 V and (W/L) = 50,
then the trans-conductance gm (in mA/V) is
________. [EC-2017 : 1 Mark]
0
0.5 µm Q.58 Two n-channel MOSFETs, T1 and T2 are
[EC-2016 : 2 Marks] identical in all respects except that the width of
T2 is double that of T1. Both the transistors are
Q.55 For a narrow base PNP BJT, the excess minority biased in the saturation region of operation, but
carrier concentrations ( nE for emitter, pB for the gate overdrive voltage (VGS – VTH) of T2 is
base, n C for collector) normalized to double that of T1, where VGS and VTH are the
equilibrium minority carrier concentrations gate-to-source voltage and threshold voltage of
(nE0 for emitter, pB0 for base, nC0 for collector) in the transistors, respectively. If the drain current
the quasi-neutral emitter, base and collector and transconductance of T1 are ID1 and gm1
regions are shown below. Which one of the respectively, the corresponding values of these
following biasing modes is the transistor two parameters for T2 are
operating in?
(a) 8 ID1 and 2 gm1 (b) 8 ID1 and 4 gm1
(c) 4 ID1 and 4 gm1 (d) 4 ID1 and 2 gm1
PB 5
10
carrier concentration

[EC-2017 : 2 Marks]
Normalized excess

PB0

Q.59 A MOS capacitor is fabricated on p-type Si


nC
nC 0
(Silicon) where the metal work function is 4.1 eV
0
and electron affinity of Si is 4.0 eV. EC – EF = 0.9 eV,
nE
where EC and EF are the conduction band
nE0 1
minimum and the Fermi energy levels of Si
Emitter (P) Base (N) Collector (P)
respectively. Oxide r = 3.9, o = 8.85 × 10–14 F/cm,
(a) Forward active (b) Saturation oxide thickness tox = 0.1 µm and electronic
(c) Inverse active (d) Cut-off charge q = 1.6 × 10–19 C. If the measured flat band
[EC-2017 : 1 Mark] voltage of this capacitor is –1 V, then the
magnitude of the fixed charge at the oxide
Q.56 An n-p-n bipolar junction transistor (BJT) is
semiconductor interface, (in nC/cm2), is _____ .
operating in the active region. If the reverse bias
across the base-collector junction is increased, [EC-2017 : 2 Marks]
then Q.60 The figure shows the high frequency C-V curve
(a) the effective base width increases and of a MOS capacitor (at T = 300 K) with ms = 0 V
common-emitter current gain increases. and no oxide charges. The flat band, inversion,
(b) the effective base width increases and and accumulation conditions are represented,
common-emitter current gain decreases. respectively, by the points.
(c) the effective base width decreases and C
common-emitter current gain increases. Q
P
(d) the effective base width decreases and
common-emitter current gain decreases. R
[EC-2017 : 1 Mark]
VG
0
GATE Previous Years Solved Paper 47

(a) Q, R, P (b) P, Q, R
17 –3
10 (cm )
(c) Q, P, R (d) R, P, Q
[EC-2019 : 1 Mark] NB(x)

Q.61 Consider a long channel MOSFET with a 14 –3


10 (cm )
channel length 1 µm and width 10 µm. The
device parameters are acceptor concentration Emitter Base Collector
0 W
N A = 5 × 10 16 cm –3 , electron mobility
µn = 800 cm 2/V-s, oxide capacitance/area (a) approximately 0.3 times that of T1
Cox = 3.45 × 10 –7 F/cm2, threshold voltage (b) approximately 0.7 times that of T1
VT = 0.7 V. The drain saturation current (IDsat) (c) approximately 2.5 times that of T1
for a gate voltage of 5 V is ______ mA (rounded (d) approximately 2.0 times that of T1
off to two decimal places). [EC-2020 : 2 Marks]
[ o = 8.854 × 10–14 F/cm, Si = 11.9]
Q.64 For an n-channel silicon MOSFET with 10 nm
[EC-2019 : 2 Marks]
gate oxide thickness, the substrate sensitivity
Q.62 The band diagram of a p-type semiconductor ( VT / VBS ) is found to be 50 mV/V at a
with a bandgap of 1 eV is shown. Using this
substrate voltage VBS = 2 V, where VT is the
semiconductor, a MOS capacitor having VTh of
threshold voltage of the MOSFET. Assume that,
–0.16 V, C ox of 100 nF/cm2 and a metal work
VBS >> 2 B, where q B is the separation
function of 3.87 eV is fabricated. There is no
between the Fermi energy level EF and the
charge within the oxide. If the voltage across
intrinsic level Ei in the bulk. Parameters given
the capacitor is VTh. The magnitude of depletion
are:
charge per unit area (in C/cm2) is
Electron charge (q) = 1.6 × 10–19 C
Vacuum level Vacuum permittivity ( 0) = 8.85 × 10–12 F/m
Relative permittivity of silicon ( si) = 12
Relative permittivity of oxide ( ox) = 4
4 eV
The doping concentration of the substrate is
(a) 4.37 × 1015 cm–3 (b) 7.37 × 1015 cm–3
EC (c) 2.37 × 1015 cm–3 (d) 9.37 × 1015 cm–3
0.5 eV [EC-2021 : 2 Marks]
Ei
Q.65 An ideal MOS capacitor (p-type) semiconductor)
EFS
0.2 eV is shown in the figure. The MOS capacitor is
EV
under strong inversion with VG = 2 V. The
(a) 0.52 × 10–8 (b) 0.93 × 10–8 corresponding inversion charge density (Qin) is
(c) 1.41 × 10–8 (d) 1.70 × 10–8 2.2 µC/m2. Assume oxide capacitance per unit
area as Cox = 1.7 µF/cm2. For VG = 4 V, the value
[EC-2020 : 2 Marks]
of Qin is _______ µC/cm2 (Rounded off to one
Q.63 The base of an n-p-n BJT T1 has a linear doping decimal place).
profile NB(x) as shown below. The base of Metal Oxide Semiconductor

another n-p-n BJT T2 has a uniform doping NB


of 1017 cm–13. All other parameters are identical VG

for both the devices. Assuming that the hole


density profile is the same as that of doping, the [EC-2022]
common-emitter current gain of T2 is
48 Electronics Engineering Electronic Devices & Circuits

Answers BJT and FET Basics

1. (b) 2. (d) 3. (b, c) 4. Sol. 5. (b) 6. (False) 7. (c) 8. (A-2, B-3, C-1)
9. Sol. 10. (d) 11. (c) 12. (b) 13. (c) 14. (b) 15. (b) 16. (d)
17. (d) 18. (a) 19. (d) 20. (d) 21. (d) 22. (a) 23. (b) 24. (a)
25. (b) 26. (c) 27. (b) 28. (c) 29. (d) 30. (b) 31. (b) 32. (d)
33. (d) 34. (a) 35. (a) 36. (5.76) 37. (500) 38. (0.0707)39. (2.4) 40. (380.28)
41. (a) 42. (20) 43. (4.33) 44. (0.02) 45. (d) 46. (1475) 47. (c) 48. (28.35)
49. (d) 50. (b) 51. (1.2) 52. (a) 53. (1.6) 54. (6.656) 55. (c) 56. (c)
57. (0.5) 58. (b) 59. (6.903) 60. (a) 61. (25.52) 62. (d) 63. (*) 64. (b)
65. (5.6)

Solutions
BJT and FET Basics
1. (b) 4. Sol.
n-channel MOSFET is faster than the p-channel (Decreases)
MOSFET. As the reverse bias increase at CB (collector base)
junction then the collector current (IC) increases
Mobility of electrons is always higher than the
and the effective base width decreases, so the
mobility of holes,
recombination in base decreases.
µn > µp
In n-channel the charge carriers are electrons 5. (b)
whereas in p-channel MOSFET the charge For the NMOS threshold voltage is given by
carriers are holes. VT = VTO + [ 2 f + VSB 2 ]
f

2. (d) 2 qN A tox 2qN A


s s
= =
In a MOSFET, the polarity of the inversion layer C ox 3.45 × 10 11

is the same as that of the majority carriers in the So, the threshold voltage of an n-channel
source. MOSFET can be increased by reducing the
channel dopant concentration.
3. (b, c)
IC = Ib + (1 + ) ICO 6. Sol.
as temperature increases, ICO increases, so the (False)
current (I C ) increases in BJT with rise in For depletion mode n-channel MOSFET, if the
temperature. GATE terminal is made more positive then the
Q mobility decreases as temperature increases, channel becomes more and more n-type hence
T µ the drain current will increase.
So, in MOSFET, current (IDS) decreases with rise 7. (c)
in temperature, The relationship between open base breakdown
T IDS voltage (BVCEO) of BJT with open emitter voltage
(BVCBO) is given by
GATE Previous Years Solved Paper 49

1 12. (b)
VBCEO = BVCBO
In a transistor when both the junction (JC and JE)
Q >1 are forward bias and if collector junction voltage
So, BVCEO < BVCBO is greater than emitter junction voltage then this
transistor is in reverse saturation region.
8. Sol.
13. (c)
(A-2, B-3, C-1)
• As the base width of the BJT is reduced then Ie = I o eVbe / VT 1
the recombination current (base current IB)
V / VT
decreases as a result collector current (IC) I o e be2 1
2I 1
increases. So, the current gain of the BJT = V / VT
I1 I o e be1 1
increases,
I 2
V
e be2
/ VT
= C 1
E = V / VT
1 e be1 1
• If the emitter doping concentration to base
Vbe2 / VT >>1
doping concentration ratio is reduced then e
the emitter injection efficiency decreases, so Vbe1 / VT >>1
e
the current gain ( ) of BJT reduces. Vbe / VT
e 2
• If the collector doping concentration is =2
Vbe / VT
e 1
increased then the breakdown (VBR) of a BJT
will be reduced. (Vbe2 Vbe1 ) VT =2
e
9. Sol. Vbe2 Vbe1
= ln2
The transit of a current carriers through the VT
channel of an FET decides its switching Vbe2 Vbe1 = VT (0.693)
characteristics. Take, =1
Vbe2 Vbe1 = 1 × 0.026 × 0.693
10. (d)
Vbe2 Vbe1 = 18 mV
Both the junctions are forward biased.
When emitter base (E-B) junction and collector 14. (b)
base (C-B) junction both are forward biased then
Voltage controlled capacitor.
the BJT is said to be operating in saturation
region. 15. (b)
As the edge of saturation i.e. when drain to
11. (c)
source voltage reaches VDsat the inversion layer
The process where the effective base width of charge at the drain end becomes zero (ideally).
the transistor is altered by varying the collector The channel is said to be pinched-off at the drain
junction voltage is called base width end. If the drain to source voltage V DS is
modulation of early effect. In a transistor by increased even further beyond the saturation
changing the collector junction voltage, is seen edge so that VDS > VDsat, an even larger portion
that the base width of the transistor is altered of the channel becomes pinched-off and effective
and this property is called as early effect. channel length is reduced.
50 Electronics Engineering Electronic Devices & Circuits

16. (d) 21. (d)


ID = K(VGS – VT)2
VT = VTO + 2 F + VSB 2 F
1 mA = K(2 – 1)2
= substrate bias coefficient K = 1 mA/V2
VSB = substrate bias voltage Again, ID = K(VGS – VT)2 = 4 mA
17. (d) 22. (a)
ID = K(VGS – VT)2 In accumulation mode for NMOS having
1 mA = K(900 – 400)2 p-substrate VG is –ve. When negative VG is
1 mA/V 2 applied to the gate electrode, the holes in the
K=
10 4 × 25 p-type substrate are attached to the
when, VGS = 1400 mV semiconductor oxide interface. This condition
ID = K(VGS – VT)2 is called carrier accumulation on the surface.
1 mA
= (1400 400)2 V 2 24. (a)
25 × 10 4 V 2
A
1 mA × 100 4 C=
= × 10 d
25 × 10 4
ID = 4 mA A 3.5 × 10 13 × 10 4 × 10 4
d = =
C 7 × 10 12 × 10 2
19. (d) = 5 × 10–8 m = 50 nm
IC
= = 25. (b)
IB 1
si = 10–12 F/cm
When base width increases, recombination in
= 10–10 F/cm
base region increase and decreases and hence
decreases. Cmax C d
Cmin =
If doping in base region increases, then Cmax + C d
recombination in base increases and a decreases Cmax Cmin
Cd =
thereby decreasing . Cmax Cmin

20. (d) 7×1


Cd = = 1.166 pF
7 1
S2 : VT of N-MOS is increased by adding extra
si A
p-type impurities i.e. VT of a MOS increases Cd =
w
by increase in substrate doping
concentration. (True) 10 10 × 10 4 × 10 4
w=
1.166 × 10 12
S1 : C ox = ox ; tox =gate oxide thickness
tox = 0.857 µm

QR Qox 27. (b)


VT = GC 2 F
C ox C ox
id
gm =
Q Q VGS
If Cox increases B and ox decreases VDS
C ox Cox
K (VGS VT )2
and VT increases. =
VGS
Cox decreases when tox increases.
= 2K(VGS – VT)
GATE Previous Years Solved Paper 51

28. (c) 34. (a)


ID is given by, Source body junction capacitor,
2 A A
VDS Cj = = o r
ID = Kn (Vgs Vt ) VDS d d
2
Here, r= 11.7 as there is Si only
– in linear region A= 1 µm × 0.2 µm
ID = 0.2 × 10–12 m2
and gm = at constant VDS
Vgs d = depletion width of p-n junction
d = 10 nm = 10–8 m
ID
gm = = K n (VGS Vt ) VDS
Vgs 8.9 × 10 12 × 11.7 × 0.2 × 10 12
So, Cj =
10 8
gm VGS
So, linear relation. C j = 2.082 × 10–15 F
C j 2 fF
31. (b)
35. (a)
1
= Gate source capacitance Cg is,
N
1+ B A1
NE 1 A1 =
o r1
Cg =
d1 d1
To achieve, =1
NE >> NB r1 = 3.9 as between gate and source there is

32. (d) SiO2,


IC = IB + (1 + ) ICO A1 = 1 µm ×
= 1 × 10–6 × 20 × 10–9
0.97
= = = 49 = 2 × 10–14 m2
1 1 0.98
d1 = 1 nm = 10–9 m
IC = 49 × 20 + 50 × 0.6
= 980 + 30 = 1010 µA 8.9 × 10 12 × 3.9 × 2 × 10 14
So, Cg =
or, IC = 1.01 mA 10 9
Cg = 69.42 × 10–17 F
33. (d)
Cg = 0.69 × 10–15 = 0.7 F
ni = 1010/cm3
A = 1 × 10–12 m2 36. Sol.
d = 10–6 m We know that,
V = Ad = 10–18 m3 IC
= 10–12 cm3 gm =
VT
ND = n = 1019/cm3
VBE
ni2 10 20 IC = Is e
VT
p = = 19 = 10/cm 3 where,
N D 10
0.7
So, holes in volume V is, 13
So, IC = 10 × e 0.025
H = pV = 10–11 = 0.144 mA
Since, holes number can not be a decimal
0.144
number. Hence, gm = = 5.76 mA/V
0.025
So, H=0
52 Electronics Engineering Electronic Devices & Circuits

37. Sol. VBE1


IC1 = Is exp ...(i)
Voltage controlled resistance rDS is given by 1 VT
1 VBE2
rDS =
W IC2 = Is2 exp ...(ii)
(µnC ox ) (VGS VT ) VT
L
According to given condition,
1
rDS =
800 × 10 8
× 100 × (2 + 0.5) IC1 = IC
2
= 500 VBE1 VBE2
Is1 exp = Is2 exp
38. Sol. VT VT
We known that for linear region, I s2
2
VBE1 VBE2 = VT ln
VDS I s1
ID = Kn (VGS VT )VDS
2
300 × 300
VBE1 VBE2 = 26 ln
ID 0.2 × 0.2
= Kn VDS
VGS Q = 2 for Si BJT
3
10 VBE1 VBE2 = 380.28 mV
= Kn
0.1
i.e., Kn = 10 –2 41. (a)
and for saturation region, VGS = VDS
Kn Hence MOS transistor is in saturation.
ID = (VGS VT )2
2 In saturation, ID = K(VGS – VT)2
Kn = K(VDD – VSS – VT)2
ID = (VGS VT )
2 As, VSS increases ID decreases
ID (Note linearly because square factor)
Kn
= Hence option (a) is correct.
VGS 2
42. Sol.
ID 10 2 A
So, = = 0.0707 Under channel length modulation,
VGS 2 V
ID = IDsat (1 + VDS)
39. Sol. dI D 1
= = IDsat
Peak electric field in the oxide region is, dVDS ro

Eox =
s
Es 1 1
ro = = = 20 k
ox I Dsat 0.05 × 10 3
0.2 × 2
and Es = = 0.8 V/µm 43.. Sol.
0.5
12 Cmax ox /tox X d max ox
Eox = × 0.8 = 2.4 V/µm = = 1+ ×
4 Cmin ox s tox
× s
tox X d max
40. Sol.
ox + s
IC is given by, tox X d max
VBE 100 1
IC = Is exp = 1+ × = 4.33
VT 10 3
GATE Previous Years Solved Paper 53

44. Sol. On solving,


VT = 0.3 V
ID 0.02 mA
= = = 0.02 Now consider,
ID VDS 1 mA × 1 V
VG 2 VT Q
= 2
45. (d) VG 3 VT Q3
WB doubled (increased) early effect is still
present but its effect less severe relative to
1.3 0.3 4 × 1011
=
1.8 0.3 Q3
previous WB. Slope IC versus VCE decreases.
Inverse charge density with VG = 1.8 V
46. Sol.
Q3 = 6 × 1011/cm2
IC = Is eVBE /VT
51. Sol.
+1 +1 Since VGS > VDS, MOSFET is in linear operation,
IE = IC = I s eVBE /VT
ID = K N [VGS VT ] VDS
IE will be maximum when is 50
dID
700×10 3
= K NVDS
dVGS
9 6
= (1.02) (10 × 10 ) e 25×10 3
gm = K NVDS
= 1475 µA
0.5 × 10–6 = K N [50 × 10 3 ]
48. Sol.
K N = 10 5
In the saturation region,
g d = IDS ID = K N [VGS VT ]VDS
1 W dI D
= µnC ox (VGS VT )2 = K N [VGS VT ]
2 L dVDS
g d = 28.35 µs g d = K N [VGS VT ]
49. (d) 8 × 10–6 = 10–5 [2 – VT]
On solving, VT = 1.2 V
1
rds =
W 52. (a)
µ oC ox (VGS VT )
L Substrate is N-type SC.
rds = channel resistance
W r A correct 53. Sol.
VT r B correct From the Fig. (I)
L r C correct Both the capacitors C1 and C2 are in series.
VGS r Q D wrong statement Total capacitance,
C1C 2
50. (b) C=
C1 + C 2
In a MOS-capacitor,
(VG1 – VT) Q A 1 A 2 A × 4 A × 20
(VG 1 VT ) Q t1 t2 10 9 3 × 10 9
= 1 C= =
A 1 A 2 A × 4 A × 20
(VG 2 VT ) Q2 + +
t1 t2 10 9 3 × 10 9
0.8 VT 2 × 10 11
= C = 2.5 × 109 × A
1.3 VT 4 × 10 11
54 Electronics Engineering Electronic Devices & Circuits

From Fig. (II), W


So, gm = 2 KnVDS = µnC ox VDS
A L
C=
t eq = 100 × 50 × 0.1 mA/V
Since both capacitors must be equal in Fig. (I) = 0.5 mA/V
and (II),
58. (b)
A× 4
2.5 × 109 ×A = W2 = 2 W
t eq
VGS2 VT2 = (VGS1 VT1 )
teq = 1.6 nm
2
54. Sol. I D2 W2 VGS2 VT2
= ×
I D1 W1 VGS1 VT1
dn
IC = AqDn where, Dn = µnVT
dx = 2×4=8
19 3 10 14 ID2 = 8ID1
= 0.001(1.6 × 10 ) 800 × 26 × 10 4
0.5 × 10 gm2 W2 VGS2 VT2
= × = 2×2 = 4
= 66.56 × 10–4 A gm1 W1 VGS1 VT1
= 6.656 mA
gm2 = 4 gm1
55. (c)
Emitter-base junction (JE) is in RB. 59. Sol.
Collector-base junction (JC) is in FB. EVac EVac
Hence, inverse active mode.
e m e s e
56. (c)
EFm EC
When a BJT is in active region, as the reverse bias Metal 0.9 eV
voltage across collector-base junction increased, EFP
Semiconductor
the width of depletion region increases, which
(Isolated case)
results in decrease of effective base width. This
e m = 4.1 eV
decrease in effective base width reduces the
e = 4 eV
recombinations in base region, hence, common-
EC – EFP = 0.9 eV
emitter current gain will increase.
e s = e + 0.9 = 4.9 eV
57. Sol. m = 4.1 V

Given that: VGS = 0.7 V, VTH = 0.3 V, s = 4.9 V

VDS = 0.1 V ms = m – s = 4.1 – 4.9 = –0.8 V

VGS – VTH = 0.4 > VDS 14


ox 3.9 × 8.85 × 10
Cox = =
MOSFET is in linear region. tox 0.1 × 10 4
In linear region, = 34.515 × 10–9 F/cm2
2
ID = Kn 2(VGS VTH )VDS VDS ox
VFB = ms
C ox
Transconductance,
ox
ID = –0.8 + 1 = 0.2 V
gm = = 2 KnVDS C ox
VGS
ox = 0.2 × Cox
K W µ C W
Kn = n = n ox = 0.2 × 34.515 × 1019 C/cm2
2 L 2 L
= 6.903 nC/cm2
GATE Previous Years Solved Paper 55

60. (a)
VT = VTO + VBS + 2 B 2 B
Since, ms = 0
The MOS capacitor is ideal. VTO = VT V = 0
BS
Point P represents accumulation
Point Q represents flat band 2 qN A s
= Body effect parameter
Point R represents inversion C ox
Since,
61. Sol.
1 W VBS >>2 B
ID(sat) = µnC ox (VGS VT )2
2 L
VT VTO + VBS
1 7 10 2
= × 800 × 3.45 × 10 × (5 0.7) A
2 1 VT 1
=
= 25.5162 mA 25.52 mA VBS 2 VBS

62. (d) 1
50 × 10–3 =
MOS capacitance, 2 2

m = 3.87, s = 4.8, ms = –0.93 = 2 2 × 50 × 10 3

Qox Qd
VT = ms +2 Fp = 141.42 × 10 3
V
C ox C ox
14
= Ei – EF = 0.5 – 0.2 = 0.3 ox 4 × 8.85 × 10
Fp Cox = = 7
Qd tox 10 × 10
–0.16 = 0.93 0 + 2 × 0.3
C ox = 3.54 × 10–7 F/cm2
Qd 2qN A s
= 0.6 + 0.16 – 0.93 = –0.17 =
C ox C ox
Qb = –0.17 × Cox = –0.17 × 100 × 10–9 2 2
Cox
= –1.7 × 10–8 C/cm2 NA =
2q s
3 2 7 2
63. ( ) (141.42 × 10 ) × (3.54 × 10 )
NA = 19 14
W
2 × 1.6 × 10 × 12 × 8.85 × 10
N A2 ( x ) dx = 7.37 × 1015 per cm3
1 0
= W 65. (5.6)
2
N A1 ( x ) dx Qin = Cox(VGS – VT) or Cox(VG – VT)
0
Qin
W × 1017 2 × 1017
= VG – VT
= 2 C ox
= 1
× W × (1017 1014 ) 1017 + 1014
2 Qin 2.2
VT = VG =2 = 2 1.294
C ox 1.7
2 = 0.5 1
Hence no option is matching. = 0.706 Volt
Now, Qin at
64. (b)
VG = 4V
tox = 10 nm Qin = Cox(VG – VT)
VT = 1.7 × 10–6 × (4 – 0.706)
= 50 mV/V
VBS Qin = 5.5998 µC/cm2 5.6 µC/cm2

VBS = 2 V, VBS >> 2 B


GATE-2023
Electronics Engineering

Communication Systems
Chapterwise & Topicwise

Contents
S.No. Topic Page No.

1. Analog Communication Systems .................................................................................................. 1-23

2. Random Signals and Noise ........................................................................................................... 24-51

3. Digital Communication Systems ................................................................................................ 52-86

4. Information Theory & Coding ...................................................................................................... 87-97


1 Analog Communication Systems

Q.1 In a superheterodyne AM receiver, the image (Picture) (Speech)


channel selectivity is determined by (a) VSB and VSB
(a) the preselector and RF stages (b) VSB and SSB
(b) the preselector, RF and IF stages (c) VSB and FM
(c) the IF stages (d) FM and VSB
(d) all the stages [EC-1990 : 2 Marks]
[EC-1987 : 2 Marks] Q.6 The maximum power efficiency of an AM
Q.2 A carrier Ac cos ct is frequency modulated by a modulator is
signal Em cos mt. The modulation index is mf. (a) 25% (b) 50%
The expression for the resulting FM signal is (c) 33% (d) 100%
(a) Ac cos[ ct + mf sin [EC-1992 : 2 Marks]
mt]
(b) Ac cos[ ct + mf cos mt] Q.7 Which of the following demodulator(s) can be
(c) Ac cos[ ct + 2 mf sin mt] used for demodulating the signal
2 m f Em x(t) = 5(1 + 2 cos200 t) cos20000 t
(d) Ac cos ct + cos mt (a) envelope demodulator
m
(b) square-law demodulator
[EC-1989 : 2 Marks] (c) synchronous demodulator
Q.3 Which of the following schemes suffer(s) from (d) none of the above
the threshold effect? [EC-1993 : 2 Marks]
(a) AM detection using envelope detection. Q.8 A superheterodyne radio receiver with an
(b) AM detection using synchronous detection. intermediate frequency of 455 Hz is tuned to a
(c) FM detection using a discriminator. station operating at 1200 kHz. The associated
(d) SSB detection with synchronous detection. image frequency is ______ kHz.
[EC-1989 : 2 Marks] [EC-1993 : 2 Marks]

Q.4 A signal x(t) = 2 cos( 104t) Volts is applied to Q.9 v(t) = 5[cos(10 6 t) – sin(10 3 t) × sin(10 6 t)]
an FM modulator with the sensitivity constant represents
of 10 kHz/Volt. Then the modulation index of (a) DSB suppressed carrier signal
the FM wave is (b) AM signal
(a) 4 (b) 2 (c) SSB upper sideband signal
(d) Narrow band FM signal
4 2
(c) (d) [EC-1994 : 1 Mark]

[EC-1989 : 2 Marks] Q.10 A 10 MHz carrier is frequency modulated by a


sinusoidal signal of 500 Hz, the maximum
Q.5 In commercial TV transmission in India, picture frequency deviation being 50 kHz. The
and speech signals are modulated respectively bandwidth required, as given by the Carsons’s
as: rule is ______ .
[EC-1994 : 1 Mark]
2 Electronics Engineering Communication Systems

Q.11 Match List-I with List-II and select the correct Q.16 An FM signal with a modulation index 9 is
answer using the codes given below the lists: applied to a frequency tripler. The modulation
List-I List-II index in the output signal will be
A. SSB 1. Envelope detector (a) 0 (b) 3
B. AM 2. Integrate and dump (c) 9 (d) 27
C. BPSK 3. Hilbert transform [EC-1996 : 2 Marks]
4. Ratio detector Q.17 The image channel selectivity of
5. PLL superheterodyne receiver depends upon
Codes: (a) IF amplifiers only
A B C (b) RF and IF amplifiers only
(a) 3 1 2
(c) Preselector, RF and IF amplifiers
(b) 3 2 1
(d) Preselector and RF amplifiers only
(c) 2 1 3
[EC-1998 : 1 Mark]
(d) 1 2 3
[EC-1994 : 2 Marks] Q.18 A DSB-SC signal is generated using the carrier
cos( ct + ) and modulating signal x(t). The
Q.12 The image (second) channel selectivity of a
envelope of the DSB-SC signal is
superheterodyne communication receiver is
(a) x(t)
determined by
(a) antenna and preselector (b) x(t )
(b) the preselector and RF amplifier (c) only positive portion of x(t)
(c) the preselector and IF amplifier (d) x(t) cos
(d) the RF and IF amplifier [EC-1998 : 1 Mark]
[EC-1995 : 1 Mark]
Q.19 A modulated signal is given by:
Q.13 A PLL can be used to demodulate s(t) = m1(t) cos(2 fc t) + m2(t) sin(2 fct)
(a) PM signals (b) PCM signals where the baseband signal m1(t) and m2(t) have
(c) FM signals (d) DSB-SC signals bandwidths of 10 kHz and 15 kHz, respectively.
[EC-1995 : 1 Mark] The bandwidth of the modulated signal
(in kHz), is
Q.14 A PAM signal can be detected by using
(a) 10 (b) 15
(a) an ADC
(c) 25 (d) 30
(b) an integrator
[EC-1999 : 1 Mark]
(c) a band pass filter
(d) a high pass filter Q.20 A modulated signal is given by:
[EC-1995 : 1 Mark] s(t) = e–at cos[( c+ )t] u(t)
where a, c and are positive constants, and
Q.15 The image channel rejection in a
c >> . The complex envelope of s(t) is given
superheterodyne receiver comes from
by
(a) IF stages only
(a) exp(–at) exp[ j( c+ )t] u(t)
(b) RF stages only
(b) exp(–at) exp( j )t) u(t)
(c) detector and RF stages only
(c) exp( j )t) u(t)
(d) detector, RF and IF stages
(d) exp[( j c+ )t]
[EC-1996 : 1 Mark]
[EC-1999 : 1 Mark]
GATE Previous Years Solved Paper 3

Q.21 The amplitude modulated waveform (a) an RC filter


s(t) = Ac[1 + Kam(t)] cos ct is fed to an ideal (b) an envelope detector
envelope detector. The maximum magnitude of (c) a PLL
Kam(t) is greater than 1. Which of the following (d) an idea low-pass filter with the appropriate
could be the detector output? bandwidth
(a) Acm(t) (b) Ac2 [1 + K a m(t )]2 [EC-2001 : 1 Mark]

2 Q.26 A 1 MHz sinusoidal carrier is amplitude


(c) [ Ac 1 + K a m(t ) ] (d) Ac 1 + K a m(t )
modulated by a symmetrical square wave of
[EC-2000 : 1 Mark] period 100 µ-sec. Which of the following
frequencies will not be present in the modulated
Q.22 A message m(t) band-limited to the frequency fm
signal?
has a power of Pm. The power of the output
(a) 990 kHz (b) 1010 kHz
signal in the figure is
(c) 1020 kHz (d) 1030 kHz
Ideal low [EC-2002 : 2 Marks]
(Multiply) pass filter
m(t) cos ot Q.27 An angle modulated signal is given by
cut-off f = fm Output
passband signal s(t) = cos2 (2 × 106t + 30 sin150t + 40 cos150t)
gain = 1 ( o > 2 fm)
cos( The maximum frequency and phase deviations
ot + )
of s(t) are
Pm cos Pm (a) 10.5 kHz, 140 rad
(a) (b)
2 2 (b) 6 kHz, 80 rad
(c) 10.5 kHz, 100 rad
Pm sin 2 Pm cos2
(c) (d) (d) 7.5 kHz, 100 rad
4 4
[EC-2002 : 2 Marks]
[EC-2000 : 2 Marks]

Q.23 The Hilbert transform of cos 2t + sin 2t is 2 sin 2 t


Q.28 In the figure m(t ) = , s(t) = cos200 t
(a) sin 1t – cos 2t (b) sin 1t + cos 2t t
(c) cos 1t – sin 2t (d) sin 1t + sin 2t
sin 199 t
[EC-2000 : 2 Marks] and n(t ) = .
t
Q.24 In a FM system, a carrier of 100 MHz is
Low pass
modulated by a sinusoidal signal of 5 kHz. The m(t ) y(t)
filter
bandwidth by Carson’s approximation is cut-off frequency = 1 Hz
1 MHz. If y(t) = (modulated waveform)3, then by s(t) n(t ) s( t )
pass band gain = 1

using Carson’s approximation, the bandwidth


The output y(t) will be
of y(t) and the spacing of spectral components
sin 2 t
are, respectively. (a)
t
(a) 3 MHz, 5 kHz (b) 1 MHz, 15 kHz
sin 2 t sin t
(c) 3 MHz, 15 kHz (d) 1 MHz, 5 kHz (b) + cos 3 t
t t
[EC-2000 : 2 Marks]
sin 2 t sin 0.5 t
(c) + cos 1.5 t
Q.25 A band-limited signal is sampled at the Nyquist t t
rate. The signal can be recovered by passing the sin 2 t sin t
(d) + cos 0.75 t
samples through t t
[EC-2002 : 2 Marks]
4 Electronics Engineering Communication Systems

Q.29 The input to a coherent detector is DSB-SC 5


(a) 5 J4(3) (b) J8 (3)
signal plus noise. The noise at the detector 2
output is
5
(a) the in phase component (c) J8 (4) (d) 5 J4(6)
2
(b) the quadrature component
[EC-2003 : 2 Marks]
(c) zero
Q.33 Choose the correct one from among the
(d) the envelope [EC-2003 : 1 Mark]
alternative a, b, c, d after matching an item in
Q.30 A DSB-SC signal is to be generated with a carrier Group-1 with the most appropriate item in
frequency, fc = 1 MHz using a non-linear device Group-2.
with the input-output characteristic Group-1
V0 = a0 vi + a1 vi3 P. Ring modulator
where, a0 and a1 are constants. The output of Q. VCO
the non-linear device can be filtered by an R. Foster-Seely discriminator
appropriate band-pass filter. S. Mixer
Group-2
Let, Vi = Aci cos(2 fci t ) + m(t ) where m(t) is the
1. Clock recovery
message signal. Then the value of fci (in MHz) 2. Demodulation of FM
is 3. Frequency conversion
(a) 1.0 (b) 0.333 4. Summing the two inputs
(c) 0.5 (d) 3.0 5. Generation of FM
[EC-2003 : 2 Marks] 6. Generation of DSB-SC
(a) P-1, Q-3, R-2, S-4
Common Data for Questions (31 and 32):
(b) P-6, Q-5, R-2, S-3
Let, m(t) = cos[(4 × 103)t] be the message signal and
(c) P-6, Q-1, R-3, S-2
c(t) = 5 cos[(2 × 106)t] be the carrier.
(d) P-5, Q-6, R-1, S-3
Q.31 c(t) and m(t) are used to generate an AM signal. [EC-2003 : 2 Marks]
The modulation index of the generated AM
signal is 0.5. Q.34 A superheterodyne receiver is to operate in the
frequency range 550 kHz-1650 kHz with the
Total sideband power
Then the quantity is intermediate frequency of 450 kHz. Let,
Carrier power
Cmax
1 1 R= denote the required capacitance ratio
(a) (b) Cmin
2 4
1 1 of the local oscillator and I denote the image
(c) (d)
3 8 frequency (in kHz) of the incoming signal. If the
[EC-2003 : 2 Marks] receiver is tuned to 700 kHz, then
(a) R = 4.41, I = 1600
Q.32 c(t) and m(t) are used to generate an FM signal.
(b) R = 2.10, I = 1150
If the peak frequency deviation of the generated
FM signal is three times the transmission (c) R = 3.0, I = 1600
bandwidth of the AM signal, then the coefficient (d) R = 9.0, I = 1150
of the term cos[(2 (1008 × 103t)] in the FM signal [EC-2003 : 2 Marks]
(in terms of the Bessel coefficients) is
GATE Previous Years Solved Paper 5

Q.35 An AM signal is detected using an envelope (a) 0.1 kHz sinusoid


detector. The carrier frequency and modulating (b) 20.1 kHz sinusoid
signal frequency are 1 MHz and 2 kHz (c) a linear function of time
respectively. An appropriate value for the time (d) a constant
constant of the envelope detector is
[EC-2004 : 2 Marks]
(a) 500 µsec (b) 20 µsec
(c) 0.2 µsec (d) 1 µsec Q.39 Consider a system shown in the figure. Let X(f )
and Y(f ) denote the Fourier transforms of x(t)
[EC-2004 : 1 Mark]
and y(t) respectively. The ideal HPF has the
Q.36 An AM signal and a narrow-band FM signal cut-off frequency 10 kHz.
with identical carriers, modulating signals and
modulation indices of 0.1 are added together. Balanced HPF Balanced y (t )
x(t)
modulator 10 kHz modulator
The resultant signal can be closely
approximated by
(a) broadband FM (b) SSB with carrier
(c) DSB-SC (d) SSB without carrier 10 kHz 13 kHz
[EC-2004 : 1 Mark]
X(f )
Q.37 A 100 MHz carrier of 1 V amplitude and a 1 MHz
modulating signal of 1 V amplitude are fed to a
balanced modulator. The output of the
f (kHz)
modulator is passed through an ideal high-pass –3 –1 1 3
filter with cut-off frequency of 100 MHz. The The positive frequencies where Y( f ) has spectral
output of the filter is added with 100 MHz peaks are
signal of 1 V amplitude and 90° phase shift as (a) 1 kHz and 24 kHz
shown in the figure. The envelope of the (b) 2 kHz and 24 kHz
resultant signal is
(c) 1 kHz and 14 kHz
Balanced HPF + (d) 2 kHz and 14 kHz
1 MHz, 1 V
modular 100 MHz
+
[EC-2004 : 2 Marks]

100 MHz, 1 V 100 MHz, 1 V, 90° Q.40 Find the correct match between Group-1 and
Group-2.
(a) constant
Group-1
(b) 1 + sin(2 + 10 6 t ) P. {1 + km(t)} A sin( ct)
5 Q. km(t) A sin( ct)
(c) sin(2 + 106 t )
4 R. A sin{ ct + km(t)}

5 t
(d) + cos(2 + 106 t )
4 S. A sin ct + k m(t ) dt

[EC-2004 : 2 Marks]
Group-2
Q.38 Two sinusoidal signals of same amplitude and W. Phase modulation
frequencies 10 kHz and 10.1 kHz are added X. Frequency modulation
together. The combined signal is given to an
Y. Amplitude modulation
ideal frequency detector. The output of the
Z. DSB-SC modulation
detector is
6 Electronics Engineering Communication Systems

(a) P-Z, Q-Y, R-X, S-W Q.45 A message signal with bandwidth 10 kHz is
(b) P-W, Q-X, R-Y, S-Z Lower-Side Band SSB modulated with carrier
(c) P-X, Q-W, R-Z, S-Y frequency fc = 106 Hz. The resulting signal is
1
(d) P-Y, Q-Z, R-W, S-X then passed through a Narrow-Band Frequency
Modulator with carrier frequency fc = 106 Hz.
[EC-2005 : 1 Mark] 2
The bandwidth of the output would be
Q.41 Which of the following Analog Modulation (a) 4 × 104 Hz (b) 2 × 106 Hz
scheme requires the minimum transmitted
(c) 2 × 109 Hz (d) 2 × 1010 Hz
power and minimum channel bandwidth?
[EC-2006 : 2 Marks]
(a) VSB (b) DSB-SC
(c) SSB (d) AM Common Data for Questions (46 and 47):
[EC-2005 : 1 Mark] Consider the following Amplitude Modulated (AM)
signal, where fm < B:
Q.42 A device with input x(t) and output y(t) is xAM(t) = 10 (1 + 0.5 sin2 fmt) cos2 fct
characterized by
Q.46 The average side-band power for the AM signal
y(t) = x2(t)
given above is
An FM signal with frequency deviation of
(a) 25 (b) 12.5
90 kHz and modulating signal bandwidth of
(c) 6.25 (d) 3.125
5 kHz is applied to this device. The bandwidth
of the output signal is [EC-2006 : 2 Marks]

(a) 370 kHz (b) 190 kHz Q.47 The AM signal gets added to a noise with Power
(c) 380 kHz (d) 95 kHz Spectral Density Sn(t) given in the figure below.
[EC-2005 : 2 Marks] The ratio of average sideband power to mean
noise power would be
Q.43 A carrier is phase modulation (PM) with
Sn(f )
frequency deviation of 10 kHz by a single tone
frequency of 1 kHz. If the single tone frequency
is increased to 2 kHz, assuming that phase
deviation remains unchanged, the bandwidth
f
of the PM signal is –fe – B –fe –fe + B fe – B fe fe + B
(a) 21 kHz (b) 22 kHz
25 25
(c) 42 kHz (d) 44 kHz (a) (b)
8N 0 B 4N 0 B
[EC-2005 : 2 Marks]
25 25
Q.44 The diagonal clipping in Amplitude (c) (d)
2N 0 B N0 B
Demodulation (using envelope detector) can be [EC-2006 : 2 Marks]
avoided if RC time-constant of the envelope
detector satisfies the following condition, (here Q.48 In the following scheme, if the spectrum M(f ) of
W is message bandwidth and is carrier m(t) is as shown, then the spectrum Y(f ) of y(t)
frequency both in rad/sec) will be

1 1 M(f )
(a) RC < (b) RC >
W W
1 1
(c) RC < (d) RC >
f
–B 0 B
[EC-2006 : 2 Marks]
GATE Previous Years Solved Paper 7

Q.50 Consider the frequency modulated signal,


cos(2 Bt)
10 cos[2 × 10 5 t + 5 sin(2 × 1500t)
+ 7.5 sin(2 × 1000t)] with carrier frequency of
m(t)
+
105 Hz. The modulation index is
y (t ) (a) 12.5 (b) 10
+ (c) 7.5 (d) 5
Hilbert
transformer [EC-2008 : 2 Marks]

sin(2 Bt) Q.51 The signal cos ct + 0.5 cos mt sin ct is


(a) FM only (b) AM only
Y(f ) (c) Both AM and FM(d) Neither AM nor FM
[EC-2008 : 2 Marks]
(a) Q.52 For a message signal m(t) = cos(2 fmt) and
carrier of frequency fc, which of the following
f
–B 0 +B represents a single side-band (SSB) signal?
(a) cos(2 fmt) cos(2 fct)
Y(f )
(b) cos(2 fct)
(c) cos[2 ( fc + fm)t]
(b) (d) [1 + cos(2 fmt)] cos(2 fct)
[EC-2009 : 1 Mark]
f
–2B –B 0 +B +2B
Q.53 A message signal given by
Y(f ) 1 1
m(t ) = cos 1t sin 2t
2 2
(c) is amplitude-modulated with a carrier of
frequency c to generate, s(t) = [1 + m(t)] cos ct.
f What is the power efficiency achieved by this
–B 0 +B
modulation scheme?
Y(f ) (a) 8.33% (b) 11.11%
(c) 20% (d) 25%

(d) [EC-2009 : 2 Marks]

Q.54 Suppose that the modulating signal is


f
–2B –B 0 +B +2B m(t) = 2 cos(2 fmt) and the carrier signal is
xc(t) = Ac cos(2 fct). Which one of the following
[EC-2007 : 2 Marks]
is a conventional AM signal without over-
Q.49 Consider the amplitude modulated (AM) signal modulation?
Ac cos ct + 2 cos mt cos ct. For demodulating (a) x(t) = Ac m(t) cos(2 fct)
the signal using envelope detector, the (b) x(t) = Ac [1 + m(t)] cos(2 fct)
minimum value of Ac should be
A
(a) 2 (b) 1 (c) x(t ) = Ac cos(2 f c t ) + c m(t )cos(2 f c t )
4
(c) 0.5 (d) 0 (d) x(t) = Ac cos(2 fmt) cos(2 fct) + Ac sin(2 fmt)
[EC-2008 : 1 Mark] sin(2 fct)
[EC-2010 : 1 Mark]
8 Electronics Engineering Communication Systems

Q.55 Consider an angle modulation signal: Q.58 The signal m(t) as shown is applied both to a
x(t) = 6 cos[2 × 106t + 2 sin(8000 t) phase modulator (with kp as the phase constant)
+ 4 cos(8000 t)] V and a frequency modulator with (kf as the
The average power of x(t) is frequency constant) having the same carrier
(a) 10 W (b) 18 W frequency.
(c) 20 W (d) 28 W m(t)
[EC-2010 : 1 Mark]
2
Q.56 The List-I (Lists the attributes) and the List-II
(Lists of the modulation systems). Match the
attribute to the modulation system that best –2 0 2 4 6 8 t(sec)
meet it.
List-I –2
A. Power efficient transmission of signals
B. Most bandwidth efficient transmission of The ratio k p /k f (in rad/Hz) for the same
voice signals maximum phase deviation is
C. Simplest receiver structure (a) 8 (b) 4
D. Bandwidth efficient transmission of signals (c) 2 (d)
with significant dc component [EC-2012 : 2 Marks]
List-II
1. Conventional AM Q.59 Consider sinusoidal modulation in an AM
system. Assuming no over-modulation, the
2. FM
modulation index (µ) when the maximum and
3. VSB
minimum values of the envelope, respectively,
4. SSB-SC
are 3 V and 1 V, is _______ .
Codes:
[EC-2014 : 1 Mark]
A B C D
(a) 4 2 1 3 Q.60 In the figure, M(f ) is the Fourier transform of the
(b) 2 4 1 3 message signal m(t) where A = 100 Hz and
(c) 3 2 1 4 B = 40 Hz. Given v(t) = cos(2 fct) and (t) =
(d) 2 4 3 1 cos(2 (f c + A)t), where f c > A. The cut-off
[EC-2011 : 1 Mark] frequencies of both the filters are f c .
Q.57 A message signal, M (f )

m(t) = cos2000 t + 4 cos4000 t 1


modulates the carrier c(t) = cos2 fct where
fc = 1 MHz to produce an AM signal. For
demodulating the generated AM signal using –A –B B A
f
an envelope detector, the time constant RC of
the detector circuit should satisfy High pass Low pass s(t )
m(t)
filter filter
(a) 0.5 ms < RC < 1 ms
(b) 1 µs << RC < 1 ms
v ( t) (t)
(c) RC << 1 µs
(d) RC >> 0.5 µs The bandwidth of the signal at the output of the
modulator (in Hz) is ______ .
[EC-2011 : 2 Marks]
[EC-2014 : 2 Marks]
GATE Previous Years Solved Paper 9

Q.61 A modulated signal is y(t) = m(t) cos(4000 t),


H(f )
where the baseband signal m(t) has frequency
components less than 5 kHz only. The minimum
required rate (in kHz) at which y(t) should be
sampled to recover m(t) is _______ . –1700 –700 0 700 1700 f (Hz)
Fig. (b)
[EC-2014 : 1 Mark]
[EC-2015 : 2 Marks]
Q.62 Consider an FM signal:
f(t) = cos[2 fct + 1 sin2 f1t + 2 sin2 f2t] Q.66 A message signal m(t) = Am sin(2 fmt) is used to
The maximum deviation of the instantaneous modulate the phase of carrier Ac cos(2 fct) to get
frequency from the carrier frequency fc is the modulated signal y(t) = Ac cos(2 fct + m(t)).
(a) The bandwidth of y(t)
1 f1 + 2 f2
(b) (a) depends on Am but not on fm
1 f2 + 2 f1
(c) (b) depends on fm but not on Am
1+ 2
(d) f1 + f2 (c) depends on both Am and fm
[EC-2014 : 1 Mark] (d) does not depends on Am or fm
[EC-2015 : 1 Mark]
Q.63 In a double side-band (DSB) full carrier Am
transmission system, if the modulation index is Q.67 The complex envelope of the bandpass signal
doubled, then the ratio of total sideband power
sin( t /5)
to the carrier power increases by a factor of x( t ) = 2 sin t , centered
t /5 4
______ .
[EC-2014 : 1 Mark]
1
about f = Hz, is
Q.64 Consider the signal: 2
ˆ (t ) sin(2 fc t )
s(t ) = m(t ) cos(2 fc t ) + m
sin( t /5 j 4
ˆ (t ) denotes the Hilber transform of m(t) (a) e
where, m t /5
and the bandwidth of m(t) is very small
compared to fc. The signal s(t) is a sin( t /5 j
4
(b) e
(a) high-pass signal t /5
(b) low-pass signal
sin( t /5 j 4
(c) band-pass signal (c) 2 e
t /5
(d) double sideband suppressed carrier signal
[EC-2015 : 1 Mark] sin( t /5 j
(d) 2 e 4
t /5
Q.65 In the system shown in Fig. (a), m(t) is a low-
pass signal with bandwidth W Hz. The [EC-2015 : 2 Marks]
frequency response of the band-pass filter H(f)
Q.68 The block diagram of a frequency synthesizer
is shown in Fig. (b). If it is desired that the output
consisting of a Phase Locked Loop (PLL) and a
signal z(t) = 10x(t), the maximum value of
divide by N counter (comprising ÷ 2, ÷ 4, ÷ 8,
W (in Hz) should be strictly less than _______ .
÷ 16 outputs) is sketched below. The synthesizer
x( t) = m( t) cos(2400 t)
Amplifier
y(t) = 10x(t) + x2(t) H(f )
band-pass z (t )
is excited with a 5 kHz signal (input 1). The free
Fig. (a)
filter running frequency of the PLL is set to 20 kHz.
10 Electronics Engineering Communication Systems

Q.72 A modulating signal given by


+VCC
x(t) = 5 sin(4 103t – 10 cos2 103t) V
Input 1 Phase Low pass is fed to a phase modulator with phase deviation
detector filter constant kp = 5 rad/V. If the carrier frequency is
Amplifier 20 kHz, the instantaneous frequency (in kHz)
VCO at t = 0.5 ms is _______ .
Counter [EC-2017 : 2 Marks]
1 +2
2 +4 R –VCC Q.73 The unmodulated carrier power is an AM
3 +8 transmitter is 5 kW. This carrier is modulated
4 +16 by a sinusoidal modulating signal. The
Synthesizer maximum percentage of modulation is 50%. If
output
it is reduced to 40%, then the maximum
Assume that the commutator switch makes unmodulated carrier power (in kW) that can be
contacts repeatedly in the order 1 - 2 - 3 - 4. used without overloading the transmitter is __ .
The corresponding frequency synthesized are: [EC-2017 : 2 Marks]
(a) 10 kHz, 20 kHz, 40 kHz, 80 kHz Q.74 Consider the following amplitude modulated
(b) 20 kHz, 40 kHz, 80 kHz, 160 kHz signal:
(c) 80 kHz, 40 kHz, 20 kHz, 10 kHz s(t) = cos(2000 t) + 4cos(2400 t) + cos(2800 t)
(d) 160 kHz, 80 kHz, 40 kHz, 20 kHz The ratio (accurate to three decimal places) of
[EC-2016 : 1 Mark] the power of the message signal to the power of
the carrier signal is ________ .
Q.69 A superheterodyne receiver operates in the
[EC-2018 : 1 Mark]
frequency range of 58 MHz – 68 MHz. The
intermediate frequency fIF and local oscillator Q.75 Let c(t) = Ac cos(2 fct) and m(t) = cos(2 fmt). It is
frequency fLO are chosen such that fIF fLO. It is given that, fc >> 5fm. The signal c(t) + m(t) is
required that the image frequencies fall outside applied to the input of a non-linear device,
the 58 MHz – 68 MHz band. The minimum whose output vo(t) is related to the input vi(t) as
required fIF (in MHz) is _______ .
vo (t ) = avi (t ) + bvi2 (t ), where a and b are positive
[EC-2016 : 1 Mark]
constants. The output of the non-linear device
Q.70 The amplitude of a sinusoidal carrier is is passed through an ideal band-pass filter with
modulated by a single sinusoid to obtain the center frequency fc and bandwidth 3fm, to
amplitude modulated signal: produce an amplitude modulated (AM) wave.
s(t) = 5 cos1600 t + 20 cos1800 t + 5 cos2000 t If it is desired to have the sideband power of the
The value of the modulation index is ______ . AM wave to be half of the carrier power, then
[EC-2016 : 1 Mark] a/b is
(a) 0.25 (b) 0.5
Q.71 For a superheterodyne receiver, the intermediate
frequency is 15 MHz and the local oscillator (c) 1 (d) 2
frequency is 3.5 GHz. If the frequency of the [EC-2018 : 2 Marks]
received signal is greater than the local oscillator Q.76 The baseband signal m(t) shown in the figure is
frequency, then the image frequency (in MHz) phase-modulated to generate the PM signal
is ______ . (t) = cos(2 fct + km(t)). The time ‘t’ on the x-axis
[EC-2016 : 1 Mark] in the figure is in milliseconds. If the carrier
GATE Previous Years Solved Paper 11

frequency is fc = 50 kHz and k = 10 , then the (a) cos(1000 t) (b) cos(540 t)


ratio of the minimum instantaneous frequency (c) cos(920 t) (d) cos(460 t)
(in kHz) to the maximum instantaneous [EC-2020 : 2 Marks]
frequency (in kHz) is ______ (rounded off to
Q.79 A sinusoidal message signal having root mean
two decimal places).
square value of 4 V and frequency of 1 kHz is
m(t)
fed to a phase modulator with phase deviation
1 constant 2 rad/volt. If the carrier signal is,
c(t) = 2 cos(2 106t), the maximum instantaneous
0 3 5 6 8 9
1 2 4 7 t(in ms)
frequency of the phase modulated signal
(Rounded off to one decimal place) is _____ Hz.
–1
[EC-2021 : 1 Mark]
[EC-2019 : 1 Mark]
Q.80 Consider a superheterodyne receiver tuned to
Q.77 SPM(t) and SFM(t) as defined below, are the phase 600 kHz. If the local oscillator feeds a 1000 kHz
modulated and the frequency modulated signal to the mixer, the image frequency
waveforms, respectively, corresponding to the (in integer) is ______ kHz.
message signal m(t) shown in the figure, [EC-2021 : 2 Marks]
SPM(t) = cos(1000 t + Kpm(t)) Q.81 Consider a carrier signal which is amplitude
t modulated by a single-tone sinusoidal message
and SFM(t) = cos 1000 t + K f m( ) d
signal with a modulation index of 50%. If the
carrier and one of the sidebands are suppressed
where Kp is the phase deviation constant in
in the modulated signal, the percentage of power
radians/volt and Kf is the frequency deviation
saved (Rounded off to one decimal place) is
constant in radians/second/volt. If the highest
________ .
instantaneous frequencies of SPM(t) and SFM(t)
are same, then the value of the ratio Kp/Kf is [EC-2021 : 1 Mark]
_______ seconds. Q.82 Consider an FM broadcast that employs the
V pre-emphasis filter with frequency response:
m(t)
10 j
H pe ( ) = 1 +
o
where, o = 104 rad/sec
t(sec) For the network shown in the figure to act as a
0 1 3 6 7
corresponding de-emphasis filter, the
[EC-2020 : 2 Marks]
appropriate pair(s) of (R, C) values is/are ___ .
Q.78 For the modulated signal x(t) = m(t) cos{2 fct},
+ +
the message signal m(t) = 4 cos(1000 t) and the R
carrier frequency fc is 1 MHz. The signal x(t) is Input C Output
passed through a demodulator, as shown in the
– –
figure below. The output y(t) of the
demodulator is (a) R = 1 k , C = 0.1 µF
(b) R = 2 k , C = 1 µF
Ideal LPF with cut-off 510 Hz
(c) R = 1 k , C = 2 µF
x(t) y(t)
(d) R = 2 k , C = 0.5 µF [EC-2022]
f (Hz)
–510 510

cos(2 (fc + 40)t)


12 Electronics Engineering Communication Systems

Answers Analog Communication Systems


1. (a) 2. (a) 3. (c) 4. (a) 5. (c) 6. (b) 7. (c) 8. (2110)

9. (d) 10. (101) 11. (a) 12. (b) 13. (c) 14. (b) 15. (b) 16. (d)

17. (d) 18. (b) 19. (d) 20. (b) 21. (c) 22. (d) 23. (a) 24. (a)

25. (d) 26. (c) 27. (d) 28. (c) 29. (a) 30. (c) 31. (d) 32. (d)

33. (b) 34. (a) 35. (b) 36. (b) 37. (c) 38. (d) 39. (b) 40. (d)

41. (c) 42. (a) 43. (d) 44. (a) 45. (b) 46. (c) 47. (b) 48. (a)

49. (a) 50. (b) 51. (a) 52. (c) 53. (c) 54. (c) 55. (b) 56. (b)

57. (b) 58. (b) 59. (0.50) 60. (60) 61. (10) 62. (a) 63. (4) 64. (c)

65. (350) 66. (c) 67. (c) 68. (a) 69. (5) 70. (0.5) 71. (3485) 72. (20)

73. (5.208) 74. (0.125) 75. (d) 76. (0.75) 77. (2) 78. (c) 79. (1011313.7)

80. (1400) 81. (94.44) 82. (a)

Solutions
Analog Communication Systems
1. (a) 4. (a)
The image rejection should be achieved before k f Am
IF stage because ones it enters into IF amplifier Modulation index = M f = =
m m
it becomes impossible to remove it from wanted
signal. So image channel selectivity depends (2 × 10 × 10 3 ) × (2)
Mf = =4
upon preselector and RF amplifiers only. The IF × 10 4
amplifiers helps in rejection of adjacent channel
5. (c)
frequency and not image frequency.
In commercial TV transmission in India, picture
2. (a) signal is modulated using VSB modulation and
speech or audio signal is modulated using FM
XFm(t) = Ac cos[ ct + k f m(t ) dt ]
modulation.
XFm(t) = Ac cos[ ct + k f Em cos m t dt ] 6. (b)
k f Em When the message signal is symmetrical square
XFm(t) = Ac cos ct + sin mt wave, 50% power efficiency can be achieved by
m
an AM modulator.
Modulation index,
k f Em 7. (c)
mf =
m Given that,
XFm(t) = [Ac cos[ ct + mf sin x(t) = 5(1 + 2 cos200 t) cos20000 t ...(i)
mt)]
The standard equation for AM signal is,
3. (c)
XAM(t) = Ac (1 + m cos mt) cos ct ...(ii)
FM detection using a discriminator suffers from
By comparing the equation (i) and equation (ii),
the threshold effect.
we have m = 2.
GATE Previous Years Solved Paper 13

Since the modulation index is more than 1 here, 14. (b)


so it is the case of over modulation. When the An integrator.
modulation index of AM wave is more than 1
(over modulation) then the detection is possible 15. (b)
only with synchronous modulator only. Such The image rejection should be achieved IF stage
signals can not be detected with envelope because ones it enters into IF amplifier it becomes
detector. impossible to remove it from wanted signal. So
the image channel rejection in a super-
8. Sol. heterodyne receiver comes from RF stages only.
f si = fs + 2IF
16. (d)
f si = 1200 + 2(455)
f si = 2110 kHz In a frequency multiplier circuit, the modulation
index is multiplied by n.
9. (d)
So, BFM = n FM
6 5
v(t) = 5 cos(10 t) cos(106 103 ) t = 3 × 9 = 27
2
5 17. (d)
+ (106 + 10 3 ) t
2 The image rejection should be achieved before
So, carrier and upper side band are in phase IF stage because ones it enters into IF amplifier
and lower side band is out of phase with carrier it becomes impossible to remove it from wanted
and upper side band. signal. So image channel selectivity depends
So, the given signal is narrow band FM signal. upon preselector and RF amplifier only. The IF
amplifier helps in rejection of adjacent channel
10. Sol.
frequency and not image frequency.
By Carson’s rule:
BW = 2( f + fm) 18. (b)

BW = 2(50 + 0.5) DSB-SC signal = x(t) cos( ct + )


BW = 101 kHz Envelope of DSB-SC signal = x(t )

11. (a)
19. (d)
SSB Hilbert transform B.W. = 2fm
AM Envelope detector = 2 × 15 kHz
BPSK Integrate and dump = 30 kHz

12. (b) 20. (b)


The image rejection should be achieved before Complex envelope,
IF stage because ones it enters into IF amplifier j t
g( t ) = g + ( t ) e c
it becomes impossible to remove it from wanted
where, g+(t) is pre-envelope given as,
signal. So image channel selectivity depends
upon preselector and RF amplifiers only. The IF g+(t) = g(t ) + j gˆ (t )
amplifier helps in rejection of adjacent channel and gˆ (t ) = e–at sin[( c+ )t] u(t)
frequency and not image frequency. at + j( c + )t
g+(t) = e e u(t )
13. (c) at + j( c + )t j ct
Hence, g( t ) = e e e u(t )
PLL (Phase Locked Loop) is used to demodulate
the FM signals. = e at e j t
u(t )
14 Electronics Engineering Communication Systems

21. (c) T0 = 100 µs


When the modulation index of AM wave is less 1
f0 = = 10 kHz
than unity the output of the envelope detector is T0
envelope of the AM wave but when the Fourier series representation of m(t) will have
modulation index is greater than unity then the only odd harmonics.
output of the envelope detector is not envelope So, m(t) contains 10 kHz, 30 kHz, 50 kHz, ....
but mode of the envelope of the AM wave. Thus Carrier frequency,
the detector output in given case would be f c = 1 MHz = 1000 kHz
Ac[1 + ka m(t)]. AM signal contains ...., 950 kHz, 970 kHz,
990 kHz, 1000 kHz, 1010 kHz, 1030 kHz, 1050
22. (d)
kHz, ....
m(t ) So, option (c) is correct.
Output signal = cos
2
m(t) has power Pm, 27. (d)
Power of m(t) a2 Pm s(t) = cos[2 × 2 × 106t + 2 × 30 sin150t
1 + 2 × 40 cos150t]
Here, a= cos
2 d i
= 2 × 2 × 106 + 2 × 4500 cos150t
Pm dt
Power of output signal = cos2
4 + 2 × 6000 (–sin150t)
= 1 – c = 2 (45002 + 60002)1/2
23. (a)
= 2 × .75 k-rad/sec
H.T.
cos 1t sin 1t
f= = 7.5 kHz
H.T. 2
sin 2 t cos 2 t
= 2 30 2 + 40 2 = 100
24. (a)
28. (c)
x(t) = Ac cos[ ct + sin mt]
y(t) = K cos[3 ct + 3 sin 2 sin 2 t
mt] m(t) × s(t) = × cos 200 t
t
[After passing from y(t) = [x(t)]3]
1=3 1
= (sin 202 t sin 198 t )
i.e. f 1 = 3 × f = f >> fm t
B.W. = 2 f 1 × 3 = 3 MHz The signal at the input of the LPF will be
fm remains same. x(t) = [m(t) s(t) + n(t)] s(t)
fm = 5 kHz 1
= (sin 202 t sin 198 t + sin 199 t ) cos 200 t
t
26. (c) So, after passing x(t) through the LPF, we get
m(t) 1 1
y(t) = (sin 2 t ) + (sin 2 t + sin 2 t sin t )
2t 2t
sin 2 t 1
= + (2 sin 0.5 t cos 1.5 t )
t t 2t
sin 2 t sin 0.5 t
y(t) = + cos 1.5 t
t t
Message signal m(t) is a symmetrical square
wave of period 100 µs.
GATE Previous Years Solved Paper 15

29. (a) 2 × 106 + n 4 × 103 = 2 × 1008 × 103


The coherent detector rejects the quadrature n=4
component of noise and therefore noise at the Bessel coefficient 5J4(6).
output has inphase component only. The
34. (a)
inphase component of noise and output are
2
additive at output of detector. Cmax f
R= = max
Cmin f min
30. (c)
2
3
V0 = a0 vi + a1 vi 1650 + 450
= = 4.41
550 + 420
i3
= a0 ( Aci cos i
c t ) + a0 m(t ) + a1 Ac cos3 i
ct I = fs + 2fif
3 2
+ a1 m (t ) + 3 ai m (t ) Aci cos i
c t = 700 + 2 × 450 = 1600

+ 3a1 Aci cos 2 i


c t m(t ) 35. (b)
For DSB-SC, we are connected with only last 1 1
term, < RC
fc fm
m(t) cos ct fc = 1 MHz
(where, 1/fc = to avoid fluctuations at recovered
For cos2 term: i = 2 × 1 MHz output and 1/fm = to avoid diagonal clipping)
2 c
1
For cos2 term: 2 i = 2 × 1 MHz 1 µsec < RC
c 2 kHz
1 µsec < RC 0.5 ms
31. (d)
36. (b)
ma2
Pt = Pc 1 +
2 At
s(t) = Ac cos c t + c cos( c + m ) t
2
Ac2
Pc = Ac
2 cos( c m)
2
Ac2 ma2 Ac ma
Side-band power = s(t)AM = Ac cos c t +
2 2 2
Ac2 ma2 A m
cos( c + m ) t + c a cos( c m)
Ps 4 ma2 1 2
= = = s(t) + s(t)AM = Carrier + USB
Pc Ac2 2 8
= SSB with carrier
2
Note : –s(t) + s(t)AM = SSB + SC

32. (d) 37. (c)


99, MHz,1/2 V
xFM(t) = Ac Jn ( ) cos( c +n m )t
n=
Balanced HPF
1 MHz, 1 V 101, MHz,1/2 V
B.W. = 2 n modular 100 MHz

= 3 × B.W. = 6 m
100 MHz, 1 V 101, MHz,1/2 V
= =6
m 101, MHz,1/2 V Output = y(t)

c+n m = 2 × 1008 × 103


(Given in question) 100 MHz, 1 V, 90°
16 Electronics Engineering Communication Systems

1 39. (b)
y(t) = cos{2 × 101 × 106 } t + sin(2 × 100 + 106 ) t
2
Spectrum at the output of the first balanced
1 6 6 modulator,
= [cos(2 100 × 10 ) t cos 10 t 2
2

sin(2 × 100 × 106 ) t sin 2 10 6 t


+ sin2 × 100 × 106t
1
= cos 2 × 100 × 106 t [cos 2 × 106 t ] –13 –11 –10 –9 –7 0 7 9 10 11 13 f (kHz)
2
After passing through HPF with a cut-off
1
+ sin 2 × 100 × 106 t [ sin 2 × 106 t + 2] frequency of 10 kHz,
2
= A cos2 × 106t + Bcos2 × 106t
Envelope
= A 2 + B2
1 1 –13 –11 –10 0 10 11 13 f (kHz)
= cos 2 2 × 106 t + [sin 2 × 106 t 2]2
4 4
After passing through the second balanced
Envelope modulator, the spectrum of y(t) for positive
5 frequencies can be drawn as,
= sin 2 × 106 t
4

38. (d)
x(t) = cos(2 × 10kt) + cos(2 × 10.1kt)
= cos(2 × 10kt) + cos[2 (10 k + 0.1k)t]
0 2 3 13 23 24 26 f (kHz)
= cos(2 × 10kt) + cos(2 × 10kt) cos(200 t)
–sin(2 × 10kt) sin(200 t) So, the spectral peaks exists at f = 2 kHz and
= cos(2 × 10kt) [1 + cos(200 t)] 24 kHz.
–sin(2 × 10kt) sin(200 t)
42. (a)
= [1 + cos(200 t )]2 + sin 2 (200 t ) cos[2 × 10 kt + (t )] B.W. = 2( f + fm)
= 2(180 + 5)
1 sin (200 t )
(t) = tan = 370 K
1 + cos(200 t )
Note : When FM signal is applied to doubler
Frequency frequency deviation doubles but fm remains the
detector d (t )
Ac cos[2 fct + (t)] Output same.
dt

d (t ) 1 1 + cos(200 t )] (200 cos 200 t )


43. (d)
Output = 2
×
dt sin(200 t ) [1 + cos(200 t )]2 f = mf
1+
1 + cos(200 t )
mf remains same for both cases,
sin(200 t ) [ 200 sin 200 t ]
[1 + cos(200 t )]2 f 10
=
200 cos(200 t ) + 200 2 1
[1 + cos(200 t )]2 + sin 2 (200 t ) f = 20
B.W. = 2( f + fm)
200 [1 + cos(200 t )]
= = 2(20 + 2)
2[1 + cos(200 t )]
= 44 K
Output 100 (constant)
GATE Previous Years Solved Paper 17

45. (b) 50. (b)


Spectrum of lower sideband SSB signal is, Modulation index,

SSB
mf = f
m
where, = maximum frequency deviation
fm = maximum frequency component
Given that, fm = 1500 Hz
–fc1 0 fc1 Deviation,
10 kHz 10 kHz = 5sin(2 × 1500t) + 7.5 sin(2 × 1000t)
d
fmax = fc1 = 106 Hz = = 5 × 2 × 1500 cos(2 × 500t)
dt
After passing the SSB signal through an NBFM
+ 7.5 × 2 × 1000 cos(2 × 1000t)
modulator,
= 2 (7500 + 7500)
(BW)out = 2fmax = 2 × 106 Hz max

max
= = 15000 Hz
46. (c) 2

ma2 Ac2 ma2


15000
mf = = = 10
Ps = Pc = × fm 1500
2 2 2
100 0.25 51. (a)
Ps = × = 6.25 Watts
2 2 s(t) = cos( ct) + 0.5 cos mt sin ct
47. (b) In this signal, the carrier components are in
phase quadrature. So, this signal is a narrow-
25
PS = Watts band FM (NBFM) signal.
4
PN = NO B 52. (c)
PS 25 cos[2 (f c + f m ) t] represents only USB of
SNR = =
PN 4 NO B AM-SSB/SC signal.

49. (a) 53. (c)

Modulated signal, ma2


= × 100%
AM(t) = Ac cos ct + 2 cos mt cos ct 2 + ma2
2 Vm 1
= Ac 1 + cos mt cos ct
ma1 = = = ma2
Ac Vc 2

2 ma = ma21 + ma22
µ=
Ac 0.25 × 2
= × 100% = 20%
Condition for envelope detection of an AM 2 + 0.25 × 2
signal is,
54. (c)
µ 1
Conventional AM signal is,
2
1 or Ac 2 x(t) = Ac [1 + m(t)] cos(2 fct)
Ac
= Ac cos(2 fct) + Ac m(t) cos(2 fct)
Therefore minimum value of Ac should be 2.
In option (b),
2
Modulation index = =2
1
18 Electronics Engineering Communication Systems

So, it is conventional AM signal but with over Given, ( D)max = ( D)max


modulation. 8 k f = 2 kp
In option (c), kp
A =4
x(t) = Ac cos(2 f c t ) + c m(t )cos(2 fc t ) kf
4
2 1 59. Sol.
Here, modulation index = =
4 2
Amax Amin
Therefore, it is conventional AM signal without µ=
Amax + Amin
over modulation.
3 1 2
= = = 0.50
55. (b) 3+1 4
Average power of angle modulated signal x(t)
60. Sol.
is Ac2 /2.
m(f )
Here, Ac = 6
1
2
6
Average power = = 18 W
2
f
–A –B 0 B A
56. (b)
After passing through first multiplier, the output
AM has simple receiver and VSB is bandwidth
is
efficient transmission of signals with significant
m(t) × v(t) = x(t)
dc component. SSB-SC is the most bandwidth
Therefore,
efficient transmission of voice signals.
X (f )
57. (b)
I 1
<< RC <<
fc fm –fc – A –fc – B –fc + B –fc + A 0 fc – A fc – B fc + B fc + A

I 1 The Fourier transform of output y(t) of the high


<< RC <<
1 MHz 2 kHz pass filter centered at fc is
1 µs << RC << 0.5 ms Y (f )

58. (b)
For phase modulator,
(t) = 2 fct + kp m(t) –fc – A –fc – B 0 fc + B fc + A
Maximum phase deviation is,
Now, the output of second multiplier is,
( D)max = kp max[m(t)] = 2kp ...(i)
y(t) w(t) = z(t)
For frequency modulator,
Therefore,
t
(t) = 2 fc t + 2 k f m(t ) dt Z (f )
0
t
( D)max = 2 kft 0
m(t ) dt
max
2 –2fc – 2A –2fc – A – B B–A 0 A – B 2fc + A + B 2fc + 2A
( D)max = 2 kf m(t ) dt
0 After passing through the low pass filter centred
( D)max = 8 kf ...(ii) at fc the Fourier transform of output s(t) is
GATE Previous Years Solved Paper 19

63. Sol.
S (f )
Ratio of PSB to PC is,
PC µ 2
PSB =
2
B–A 0 A–B PSB µ2
=
Thus, B.W. = (A – B) – 0 PC 2
= 100 Hz – 40 Hz PSB µ2
and = 1
= 60 Hz PC 1 2

61. Sol. PSB µ 22


PC =
2 2
M(f ) SY (f )
where, µ2 = 2µ1
( PSB / PC )1 1
Hence, =
( PSB / PC )2 4
f
–5 0 5 f(kHz) 0 fc
PSB P
B = 4 SB
mfs PC 2 PC 1
Four times.
To recover m(t) from the samples of y(t), the
following conditions must be satisfied, 64. (c)
4000 Given s(t) is an SSB modulated signal.
mfs = f c =
= 20 kHz ...(i)
2 Alternate solution:
fs B = 10 kHz ...(ii) It is the canonical representation of a bandpass
‘m’ is any arbitrary integer which satisfies the signal.
above two conditions.
So, f s 10 kHz [From (ii)] 65. Sol.
Let the spectrum of message signal as shown
20 kHz
fs = below.
m
= 20 kHz, 10 kHz, 5 kHz ...[From(i)] M(f )

From (i) and (ii),


fs(min) = 10 kHz
f
–W 0 W
62. (a)
x(t) = m(t) cos2400 t
Maximum frequency deviation is
Thus, y(t) = 10 x(t) + x2(t)
1 d y(t) = 10 m(t) cos(2400 t)
=
2 dt max + m2(t) cos2(2400 t)
Phase deviation = 10 m(t) cos(2400 t)
1 1 [1 + cos(4800 t )]
= 12 f 1 cos 2 f 1t + 22 f 2 cos 2 f 2 t max + m 2 (t )
2 2 2
1 Drawing the spectrum set spectrum of x(t) be
= (2 1 f1 +2 2 f2 )
2 X(f )

= 1 f1 + 2 f2

f
–1200 – W –1200 –1200 + W 1200 – W –1200 1200 + W
20 Electronics Engineering Communication Systems

H(f )
VCO output Divide by
fn
(Nfin) N counter
5 kHz 10 kHz 2
f(Hz)
–1700 700 0 700 1700 5 kHz 20 kHz 4
y(t) = 10x(t) + x2(t) 5 kHz 40 kHz 8
z(t) = 10x(t) = 10m(t) cos(2400 t)
Y(f ) 5 kHz 80 kHz 16
BPF 1 2
2
m (t) m (t )cos(4800 t )
2
69. Sol.

2W 2W 700 1200 1700 2400


f f s = 58 MHz – 68 MHz
1200 – W 1200 + W 2400 – 2W 2400 + 2W
fsi should fall outside the range 58 MHz-68 MHz
To recover z(t), Hence, fs min = 58 MHz
1200 – W > 700 W < 500 f si = fs + 2 IF > 68 MHz
1200 + W < 1700 W < 500 58 MHz + 2IF > 68 MHz
2 W < 700 W < 350 IF > 5 MHz
2400 – 2W > 1700 W < 350 (IF)min = 5 MHz
So, that maximum value of W should be less
than 350 Hz. 70. Sol.
s(t) = 5 cos1600 t + 20 cos1800 t + 5 cos2000 t
66. (c) s(t) = 20 cos1800 t + 5 cos1600 t + 5 cos2000 t
The bandwidth of a PM signal is given by Ac µ
B = 2( f + fm) s(t) = Ac cos 2 fc t + cos 2 ( fc fm ) t
2
Now, for single tone PM signal, Ac µ
f = Am Kp fm + cos 2 ( f c + f m ) t
2
B = 2(KpAm + 1) fm Comparing, we get
67. (c) Ac µ
Ac = 20 V; =5V
2
t
sin 10
5 µ= = 0.5
x(t) = 2 sin t
t 4 20
5
71. Sol.
x(t) = Re[ x (t ) e j 2 fc t ] IF = 15 MHz
x (t ) = Complex envelope f l = 3.5 GHz
t f si = fl – IF
sin
5 = 3500 – 15 MHz
= 2 ej /4
t = 3485 MHz
5
72. Sol.
68. (a) Given that, the modulating signal is,
x(t) = 5 sin(4000 t – 10 cos 2000 t) V)
fin Phase Amplifier and
The standard phase modulated signal can be
Input detector filter
given as,
s(t) = Ac cos( ct + kp x(t))
Divide by
N counter
VCO Instantaneous angle of the modulated signal is,
(t) = c t + kp x(t)
Output (Nfin)
GATE Previous Years Solved Paper 21

Instantaneous frequency is, 75. (d)


d (t ) dx(t ) vi(t) = Ac cos(2 fct) + cos(2 fmt)
i(t) =
= c + kp
dt dt
vo(t) = avi (t ) + bvi2 (t )
25
fi (t) = fc + [cos(4000 t 10 cos 2000 t ) = [aAc cos(2 fct) + acos(2 fmt)]
2
(4000 + 20000 2 sin2000 t)] Ac2 cos2 (2 fc t ) + cos2 (2 f mt )
At t = 0.5 ms, +b
+ 2 Ac cos(2 fc t ) cos(2 f mt )
25
fi(t) = f c + [cos(2 10 cos( ))] After passing through the given BPF,
2
2 sin(
y(t) = aAc cos2 fct + 2 bAc cos(2 fct) cos(2 fmt)
(4000 + 20000 ))
2b
25 = aAc 1 + cos(2 f m t ) cos(2 fc t )
= fc + [cos(12 )] (4000 ) a
2
Modulation index,
= fc + 50 kHz
= 70 kHz 2b
µ=
Given that, a
f c = 20 kHz µ2 1
PSB = PC = PC
2 2
73. Sol. So, µ2 = 1 µ = 1
Given that, 2b
PC = 5 kW for µ(max) = 0.5 =1
a
(0.5)2 0.25 a
So, Pt(max) = PC 1 + = 5 1+ kW =2
2 2 b
= 5.625 kW 76. Sol.
For µ = 0.4,
1 d (t ) k dm(t )
fi = = fc +
µ2 2 dt 2 dt
PC (max) 1+ = Pt(max)
2 dm(t )
= fc + 5
5.625 dt
PC(max) = kW dm(t)/dt
(0.4)2
1+
2
2000
= 5.208 kW

74. Sol.
0 1 3 4 6 t(ms)
The power of the carrier signal is,
–1000
(4)2
Pc = =8W So, fi(min) = 50 kHz – (5 × 1000 Hz) = 45 kHz
2
The power of message signal can be taken as fi(max) = 50 kHz + (5 × 2000 Hz) = 60 kHz
the power carried by the sidebands. f i(min) 45
= = 0.75
So, the power of the message signal is, f i(max) 60
(1)2
Pm = 2 =1W 77. Sol.
2
S(t)pm = Ac cos[2 fct + kpm(t)]
Required ratio,
Pm 1 S(t)Fm = Ac cos 2 f c t + k f m(t ) dt
= = 0.125
Pc 8 0
22 Electronics Engineering Communication Systems

V 79. (1011313.7)

m(t ) Vm
10 Vrms = 4 = Vm = 4 2
2
fm = 1 kHz
Kp = 2 rad/volt
t(sec) c(t) = 2 cos(2 106t)
0 1 3 6 7
SPM(t) = Ac cos(2 fct + Kpm(t))
Instantaneous frequency are equal,
K p dm(t )
1 d fi,max = f c +
fi = (t ) 2 dt max
2 dt
kp d m(t) = 4 2 cos(2 10 3 t )
fiPM = f c + m(t )
2 dt
dm(t )
Kf = 4 2 × 2 × 10 3 sin(2 10 3 t )
f i FM = f c + m(t ) dt
2
dm(t )
Given that, = 8 2 10 3 sin(2 10 3 t )
dt max max
(fiPM)max = (fiFM)max
kp d kf = 8 2 10 3
fc + m(t ) = fc + [m(t )]max
2 dt max 2 2
f i max = 106 + × 8 2 10 3
kp d kf 2
m(t ) = m(t )
2 dt 2 = 1011313.7 Hz
max max
d 80. (1400)
m(t ) = 5 m(t ) max = 10
dt max fs = 600 kHz
5 kp = 10 kf fLO = 1000 kHz
kp fSI = Image frequency = fs + 2fIF
=2
kf fIF = fLO – fs = 1000 k – 600 k = 400 k
fSI = fs + 2fIF = 600 k + (2 × 400 k)
78. (c) = 1400 kHz
Output of multiplier
81. (94.44)
= x(t) cos2 (fc + 40) t
µ = 0.5
= m(t) cos2 fct × cos2 (fc + 40) t
Carrier and one sideband suppressed,
m(t )
= [cos 2 (2 f c + 40) t + cos 2 (40) t ] Power saved
2 % power saving = × 100
Given, Total power
m(t) = 4 cos1000 t PC + PUSB
= × 100
So, output of multiplier PT
= 2cos2 (500)t[cos2 (2fc + 40) t
+ cos2 (40) t] µ2
PC + PC
= cos2 (2fc + 540)t + cos2 (2fc – 460) t = 4 × 100
2
+ cos2 (540)t + cos2 (460) t µ
PC 1 +
2
Output of low pass filter
= cos[2 (460)] t
4 + 0.52
= cos920 t = × 100 = 94.44%
2(2 + 0.52 )
GATE Previous Years Solved Paper 23

82. (a) Relationship between pre-emphasis and


Given, frequency response of pre-emphasis De-emphasis systems is
filter, 1
H Pe ( ) =
j H De ( )
Hpe( ) = 1 +
o 2 1
H Pe ( ) = 2
where, o = 104
rad/sec H De ( )
De-emphasis filter given as,
2
R
1+ = 1+ ( RC)2
+ + o

Input C Output 1
= RC RC = 10–4
o
– – Option (a) only satisfies the required condition.
1
HDe( ) = H pe ( ) =
1 + j RC
1
HDe ( ) =
1 + ( RC )2
2 Random Signals and Noise

cable with a loss of 3 dB. Assuming the ambient


Q.1 The variance of a random variable X is 2x . Then
temperature to be 300°K, we have for this part of
the variance of –kx (where k is an positive the communication system,
constant) is (a) effective noise temperature = 30°K
2 2 (b) effective noise temperature = 36°K
(a) x (b) k x
(c) noise figure = 0.49 dB
(c) k 2x (d) k 2 2x
(d) noise figure = 1.61 dB
[EC-1987 : 2 Marks]
[EC-1991 : 2 Marks]
Q.2 White Gaussian noise is passed through a linear
Q.6 Two resistors R1 and R2 (in ) at temperatures
narrow band filter. The probability density
T1 °K and T2 °K respectively are connected in
function of the envelope of the noise at the filter
series. Their equivalent noise temperature is
output is
_______ °K.
(a) Uniform (b) Poisson
[EC-1991 : 2 Marks]
(c) Gaussian (d) Rayleigh
[EC-1987 : 2 Marks] Q.7 For a random variable ‘X’ following the
probability density function p(x), shown in
Q.3 In a radar receiver the antenna is connected to figure, the mean and the variance are,
the receiver through a waveguide. Placing the respectively
pre-amplifier on the antenna side of the
p( x)
waveguide rather than of the receiver side leads
to
1/4
(a) a reduction in the overall noise figure.
(b) a reduction in interference. x
–1 0 3
(c) an improvement in selectivity
characteristics.
1 2 4
(d) an improvement in directional (a) and (b) 1 and
2 3 3
characteristics.
2 4
[EC-1987 : 2 Marks] (c) 1and (d) 2 and
3 3
Q.4 Zero mean Gaussian noise of variance N is [EC-1992 : 2 Marks]
applied to a half wave rectifier. The mean
squared value of the rectifier output will be Q.8 For a narrow band noise with Gaussian
quadrature components, the probability density
(a) Zero (b) N/2
function of its envelope will be
N
(c) (d) N (a) Uniform (b) Gaussian
2
(c) Exponential (d) Rayleigh
[EC-1989 : 2 Marks]
[EC-1995 : 1 Mark]
Q.5 A part of a communication system consists of
Q.9 The auto-correlation function of an energy
an amplifier of effective noise temperature,
signal has
Te = 21°K, and a gain of 13 dB, followed by a
GATE Previous Years Solved Paper 25

(a) no symmetry 1 2
(a) (b)
(b) conjugate symmetry 2
(c) even symmetry 1
1
(d) both (b) and (c) (c) (d)
2 2
[EC-1996 : 2 Marks] [EC-1998 : 1 Mark]
Q.10 The power spectral density of a deterministic Q.14 The amplitude spectrum of a Gaussian pulse is
signal is given by [sin(f )/f ] 2, where ‘f ‘ is (a) uniform (b) a sine function
frequency. The autocorrelation function of the (c) Gaussian (d) an impulse function
signal in the time domain is
[EC-1998 : 1 Mark]
(a) a rectangular pulse
Q.15 The probability density function of the envelope
(b) a delta function
of narrow band Gaussian noise is
(c) a sine pulse
(a) Poisson (b) Gaussian
(d) a triangular pulse
(c) Rayleigh (d) Rician
[EC-1997 : 2 Marks]
[EC-1998 : 1 Mark]
Q.11 An amplifier A has 6 dB gain and 50 input
Q.16 The PDF of a Gaussian random variable X is
and output impedances. The noise figure of this
given by:
amplifier as shown in the figure is 3 dB. A
cascade of two such amplifiers as in the figure ( x 4)2
1
will have a noise figure of Px ( x ) = e 18
3 2
The probability of the even {X = 4} is
A A A
1 1
(a) (b)
2 3 2
50 50 50 50
1
(c) 0 (d)
4
[EC-2001 : 1 Mark]
(a) 6 dB (b) 8 dB
(c) 12 dB (d) none of the above Q.17 The PSD and the power of a signal g(t) are
[EC-1997 : 1 Mark] respectively, Sg( ) and Pg. The PSD and the
power of the signal ag(t) are respectively
Q.12 The ACF of a rectangular pulse of duration T is
(a) a2 Sg( ) and a2Pg
(a) a rectangular pulse of duration T
(b) a2 Sg( ) and aPg
(b) a rectangular pulse of duration 2T (c) aSg( ) and a2Pg
(c) a triangular pulse of duration T (d) aSg( ) and aPg
(d) a triangular pulse of duration 2T [EC-2001 : 2 Marks]
[EC-1998 : 1 Mark]
Q.18 If the variance 2X of d(n) = x(n) – x(n – 1) is one-
Q.13 A probability density function is given by
tenth the variance 2X of a stationary zero-mean
x2
P( x ) = K exp ; <x<
2 discrete-time signal x(n), then the normalized
2
The value of K should be autocorrelation function RXX ( k )/ X at k = 1 is
26 Electronics Engineering Communication Systems

(a) 0.95 (b) 0.90 Q.23 The distribution function FX(x) of a random
(c) 0.10 (d) 0.05 variable X is shown in the figure. The
[EC-2002 : 2 Marks] probability that, X = 1 is

Q.19 The noise at the input to an ideal frequency FX(X)


detector is white. The power spectral density of
1.0
the noise at the output is
0.55
(a) Raised-cosine (b) Flat
(c) Parabolic (d) Gaussian 0.25

[EC-2003 : 1 Mark] x
–2 0 1 3

Q.20 Let X and Y be two statistically independent


(a) zero (b) 0.25
random variables uniformly distributed in the
(c) 0.55 (d) 0.30
ranges (–1, 1) and (–2, 1) respectively. Let
Z = X + Y. Then the probability that, (Z –2) is [EC-2004 : 1 Mark]

1 Q.24 A 1 mW video signal having a bandwidth of


(a) zero (b)
6 100 MHz is transmitted to a receiver through a
cable that has 40 dB loss. If the effective one-
1 1
(c) (d) sided noise spectral density at the receiver is
3 12
10–20 Watt/Hz, then the signal-to-noise ratio at
[EC-2003 : 2 Marks] the receiver is
Common Data for Questions (21 and 22): (a) 50 dB (b) 30 dB
Let X be the Gaussian random variable obtained by (c) 40 dB (d) 60 dB
sampling the process at t = ti and let [EC-2004 : 2 Marks]

y2 Q.25 A random variable X with uniform density in


1
Q( ) = e 2 dy the interval 0 to 1 is quantized as follows:
2
If 0 X 0.3; xq = 0
If 0.3 < X 1; xq = 0.7
Autocorrelation function, RXX ( ) = 4( e 0.2 + 1) and
where, xq is the quantized value of x.
mean = 0
The root-mean square value of the quantization
Q.21 The probability that, [x 1] is noise is
(a) 1 – Q(0.5) (b) Q(0.5) (a) 0.573 (b) 0.198
(c) 2.205 (d) 0.266
1 1
(c) Q (d) 1 Q [EC-2004 : 2 Marks]
2 2 2 2
[EC-2003 : 2 Marks] Q.26 Noise with uniform spectral density of N0 W/Hz
is passed through a filter H( ) = 2 exp(–j td)
Q.22 Let Y and Z be the random variable obtained by
followed by an ideal low-pass filter of
sampling X(t) at t = 2 and t = 4 respectively. Let
bandwidth B Hz. The output noise power
W = Y – Z. The variance of W is
(in Watts) is
(a) 13.36 (b) 9.36
(a) 2 N0B (b) 4 N0B
(c) 2.64 (d) 8.00
(c) 8 N0B (d) 16 N0B
[EC-2003 : 2 Marks]
[EC-2005 : 2 Marks]
GATE Previous Years Solved Paper 27

Q.27 An output of a communication channel is a Q.30 A zero-mean white Gaussian noise is passed
random variable ‘v’ with the probability density through an ideal low-pass filter of bandwidth
function as shown in the figure. The mean 10 kHz. The output is then uniformly sampled
square value of ‘v’ is with sampling period t s = 0.03 msec. The
samples so obtained would be
p(v)
(a) correlated
(b) statistically independent
k
(c) uncorrelated
(d) orthogonal
v [EC-2006 : 2 Marks]
0 4
Common Data for Questions (31 and 32):
(a) 4 (b) 6
The following two questions refer to wide sense
(c) 8 (d) 9
stationary stochastic processes.
[EC-2005 : 2 Marks]
Q.31 It is desired to generate a stochastic process (as
Common Data for Questions (28 and 29):
voltage process) with power spectral density,
Asymmetrical three-level midtread quantizer is to be
16
designed assuming equiprobable occurrence of all S( ) = 2
16 +
quantization levels.
by driving a linear-time-invariant system by zero
Q.28 If the probability density function is divided into mean while noise (as voltage process) with
three regions as shown in the figure, the value power spectral density being constant equal to 1.
of ‘a’ in the figure is The system which can perform the desired task
could be
1/4 (a) first order low-pass R-L filter.
(b) first order high-pass R-C filter.
Region 1 (c) tuned L-C filter
1/8
(d) series R-L-C filter
Region 2 Region 3
[EC-2006 : 2 Marks]
–a +a +1
–3 –1 +3
Q.32 The parameters of the system obtained in the
1 2 above question would be
(a) (b)
3 3 (a) first order R-L low-pass filter would have
1 1 R = 4 , L = 1 H.
(c) (d)
2 4 (b) first order R-C high-pass filter would
[EC-2005 : 2 Marks] have R = 4 , C = 0.25 F.
(c) tuned L-C filter would have L = 4 H, C = 4 F.
Q.29 The quantization noise power for the
quantization region between –a and +a in the (d) series R-L-C low-pass filter would have
figure is R = 1 , L = 4 H, C = 4 F.

4 1 [EC-2006 : 2 Marks]
(a) (b)
81 9 Q.33 A uniformly distributed random variable X with
5 2 probability density function,
(c) (d)
81 81 1
f X (x ) = (u( x + 5) u( x 5))
[EC-2005 : 2 Marks] 10
28 Electronics Engineering Communication Systems

where u( ) is the unit step function is passed PDF


through a transformation given in the figure
1
below. The probability density function of the
transformed random variable Y would be
y
X
1 –1 0 +1

The corresponding Cumulative Distribution


x
–2.5 2.5 Function (CDF) has the form
1 CDF
(a) fY ( y ) = (u( y + 2.5) u( y 2.5))
5 1
(b) fY(y) = 0.5 (y) + 0.5 (y – 1)
(c) fY(y) = 0.25 (y + 2.5) + 0.25 (y – 2.5) + 0.5 (y) (a)
(d) f Y (y) = 0.25 (y + 2.5) + 0.25 (y – 2.5)

1 x
+ (u( y + 2.5) u( y 2.5)) –1 0
10

[EC-2006 : 2 Marks] CDF

Q.34 If E denotes expectation, the variance of a 1

random variable X is given by


(b)
(a) E[X2] – E2[X] (b) E[X2] + E2[X]
(c) E[X2] (d) E2[X]
x
[EC-2007 : 1 Mark] –1 0 1

Q.35 If R( ) is the autocorrelation function of a real,


CDF
wide-sense stationary random process, then
1
which of the following is not true?
(a) R( ) = R(– ) (c)
(b) R( ) R(0)

(c) R( ) = –R(– ) x
–1 0 1
(d) The mean square value of the process is R(0)
[EC-2007 : 1 Mark]
CDF
Q.36 If S(f ) is the power spectral density of a real,
1
wide-sense stationary random process, then
which of the following is always true?
1
(a) S(0) S(f ) (b) S(f) 0 (d) x
–1 0

(c) S(–f) = –S(f) (d) S( f ) df = 0


–1
[EC-2007 : 1 Mark]

Q.37 The Probability Density Function (PDF) of a


[EC-2008 : 2 Marks]
random variable X is as shown below.
GATE Previous Years Solved Paper 29

Q.38 Px ( x ) = M exp( 2 x ) + N exp( 3 x ) is the R( )

probability density function for the real random


(c)
variable X, over the entire x-axis. M and N are
both positive real numbers. The equation
relating M and N is
R( )
2 1
(a) M + N = 1 (b) 2 M + N = 1
3 3
(c) M + N = 1 (d) M + N = 3
(d)
[EC-2008 : 2 Marks]

Q.39 Noise with double-sided power spectral density


[EC-2009 : 1 Mark]
of K over all frequencies is passed through a RC
low-pass filter with 3 dB cut-off frequency of fc. Q.42 A discrete random variable X takes values from
The noise power at the filter output is 1 to 5 with probabilities as shown in the table.
(a) K (b) Kf c A student calculates the mean X as 3.5 and her
(c) K fc (d) teacher calculates the variance of X as 1.5.
[EC-2008 : 2 Marks] Which of the following statements is true?

Q.40 A white noise process x(t) with two-sided power k 1 2 3 4 5


spectral density 1 × 10–10 W/Hz is input to a P (X = k ) 0.1 0.2 0.4 0.2 0.1
filter whose magnitude squared response is
(a) Both the student and the teacher are right.
shown below.
(b) Both the student and the teacher are wrong.
H( f )
2 (c) The student is wrong but the teacher is right.
1 (d) The student is right but the teacher is wrong.
x( t) y(t)
[EC-2009 : 2 Marks]
f Q.43 X(t) is a stationary with the power spectral
–10 kHz 10 kHz
density SX(f ) > 0 for all f. The process is passed
The power of the output process y(t) is given by through a system shown below.
(a) 5 × 10–7 W (b) 1 × 10–6 W
(c) 2 × 10–6 W (d) 1 × 10–5 W +
d
[EC-2009 : 1 Mark] X(t ) Y(t)
dt
+
Q.41 If the power spectral density of stationary
Delay = 0.5 ms
random process is a sinc-squared function of
frequency, the shape of its autocorrelation is Let Sy(f ) be the power spectral density of Y(t).
R( )
Which one of the following statements is
correct?
(a) (a) SY(f ) > 0 for all f
(b) SY(f ) > 0 for f > 1 kHz
R( ) (c) SY(f ) > 0 for f = nf0, f0 = 2 kHz, n any integer
(d) SY(f ) > 0 for f = (2n + 1) f0, f0 = 1 kHz, n any
(b) integer
[EC-2010 : 2 Marks]
30 Electronics Engineering Communication Systems

Statement for Linked Answer Questions (44 and 45): Q.47 Two independent random variables X and Y
Consider a baseband binary PAM receiver shown are uniformly distributed in the interval [–1, 1].
below. The additive channel noise n(t) is white with The probability that max[X, Y] is less than 1/2 is
power spectral density SN(f) = N0/2 = 10–20 W/Hz. The 3 9
low-pass filter is ideal with unity gain and cut-off (a) (b)
4 16
frequency 1 MHz. Let Yk represent the random variable
1 2
y(tk): (c) (d)
4 3
Yk = Nk if transmitted bit bk = 0
[EC-2012 : 1 Mark]
Yk = a + Nk if transmitted bit b = 1
where, Nk represents the noise sample value. The noise Q.48 A power spectral density of a real process X(t)
sample has a probability density function, for positive frequencies is shown below. The
n values of E[X2(t)] and E[ X(t ) , respectively are
PN k (n) = 0.5 e (This has mean zero and variance

2/ 2). Assume transmitted bits to be equiprobable and SX( )


threshold z is set to a/2 = 10–6 V. 400 ( – 104)
n(t) 6

r(t) y( t ) y(tk )
x(t) LPF S/H bˆ k = 1 if y(t k ) z
0 if y (t k ) < z
Sampling Threshold 0 9 10 11 (103 rad/s)
time z
Receiver
6000 6400
(a) ,0 (b) ,0
Q.44 The value of the parameter (in V–1) is
(a) 10 10 (b) 10 7 6000 20
6400 20
(c) , (d) ,
(c) 1.414 × 10–10 (d) 2 × 10–20 ( / 2) ( 2)
[EC-2010 : 2 Marks]
[EC-2012 : 1 Mark]
Q.45 The probability of bit error is
Q.49 Let U and V be two independent zero mean
(a) 0.5 × e–3.5 (b) 0.5 × e–5
Gaussian random variables of variance 1/4 and
(c) 0.5 × e–7 (d) 0.5 × e–10 1/9 respectively. The probability P(3V 2U) is
[EC-2010 : 2 Marks] 4 1
(a) (b)
Q.46 X(t) is a stationary random process with 9 2
autocorrelation function RX( ) = exp(– 2). This 2 5
(c) (d)
process is passed through the system below. The 3 9
power spectral density of the output process [EC-2013 : 2 Marks]
Y(t) is
Q.50 Consider two identically zero-mean random
H(f ) = j2 f variables U and V. Let the cumulative
+ distribution functions of U and 2 V be F(x) and
X(t) Y(t)
G(x) respectively. Then, for all values of x

(a) F(x) – G(x) 0
(a) (4 2f 2 + 1) exp(– f 2) (b) F(x) – G(x) 0
(b) (4 2f 2 – 1) exp(– f 2) (c) (F(x) – G(x)) x 0
(c) (4 2f 2 + 1) exp(– f ) (d) (F(x) – G(x)) x 0
(d) (4 2f 2 – 1) exp(– f ) [EC-2011 : 2 Marks] [EC-2013 : 2 Marks]
GATE Previous Years Solved Paper 31

Q.51 Let, x1, x2 and x3 be independent and identically where ‘f ’ is the frequency expressed in Hz. The
distributed random variables with the uniform signal x(t) modulates a carrier cos16000 t and
distribution on [0, 1]. The probability P{x1 is the the resultant signal is passed through an ideal
largest} is ______ . band-pass filter of unity gain with center
[EC-2014 : 1 Mark] frequency of 8 kHz and bandwidth of 2 kHz.
The output power (in Watts) is ______ .
Q.52 Let X be a real-valued random variable with E[X]
[EC-2014 : 2 Marks]
and E[X2] denoting the mean values of X and
X2, respectively. The relation which always Q.57 Let X1, X2 and X3 be independent and identically
holds distributed random variables with the uniform
(a) (E[X])2 > E[X2] (b) E[X2] (E[X])2 distribution on [0, 1].
(c) E[X2] = (E[X])2 (d) E[X2] > (E[X])2 The probability P{X1 + X2 X3} is ______ .
[EC-2014 : 2 Marks] [EC-2014 : 2 Marks]

Q.58 A binary random variable X takes the value of 1


Q.53 Consider a random process X(t ) = 2 sin(2 t + ),
with probability 1/3. X is input to a cascade of
where the random phase is uniformly
2 independent identical Binary Symmetric
distributed in the interval [0, 2 ]. The
Channels (BSCs) each with crossover
autocorrelation E[X(t1) X(t2)] is
probability 1/2. The output of BSCs are the
(a) cos(2 (t1 + t2)) (b) sin(2 (t1 – t2)) random variables Y1 and Y2 as shown in the
(c) sin(2 (t1 + t2)) (d) cos(2 (t1 – t2)) figure.
[EC-2014 : 2 Marks]
Y1
X BSC BSC Y2
Q.54 The input to a 1-bit quantizer is a random
variable X with pdf fX(x) = 2e–2x for x 0 and
The value of H(Y1) + H(Y2) in bits is ______ .
fX(x) = 0 for x < 0. For outputs to be of equal
[EC-2014 : 2 Marks]
probability, the quantizer threshold should be
_______ . Q.59 Let X(t) be a Wide Sense Stationary (WSS)
[EC-2014 : 2 Marks] random with power spectral density SX(f). If Y(t)
is the process defined as Y(t) = X(2t – 1), the
Q.55 The power spectral density of a real stationary
power spectral density SY(f) is
random process X(t) is given by
1 f
1 (a) SY ( f ) = SX e j f
; f W 2 2
Sx ( f ) = W
0; f >W 1 f j f /2
(b) SY ( f ) = SX e
The value of the expectation 2 2

1 f
1 (c) SY ( f ) = SX
E x(t ) x t is ________ . 2 2
4W
1 f j2 f
[EC-2014 : 2 Marks] (d) SY ( f ) = SX e
2 2
Q.56 A real band-limited random process X(t) has [EC-2014 : 2 Marks]
two-sided power spectral density,
Q.60 Let X be a zero mean unit variance Gaussian
10 6 (3000 f ) Watts/Hz for f 3 kHz
Sx ( f ) = random variable. E[ X ] is equal to ______ .
0 otherwise
[EC-2014 : 1 Mark]
32 Electronics Engineering Communication Systems

Q.61 Consider a communication scheme where the When, = –0.3 the BER is closest to
binary values signal X satisfies P{X = 1} = 0.75 (a) 10 –7 (b) 10 –6
and P{X = –1} = 0.25. The received signal (c) 10 –4 (d) 10 –2
Y = X + Z, where Z is a Gaussian random [EC-2014 : 2 Marks]
variable with zero mean and variance 2. The
received signal Y is fed to the threshold detector.
Q.64 {Xn }nn == is an independent and identically
The output of the threshold detector X̂ is
distributed (i.i.d) random process Xn equally
+1 , Y >
Xˆ n=
1, Y likely to be +1 or –1. {Yn }n = is another
To achieve a minimum probability of error
random process obtained as Yn = Xn + 0.5Xn – 1.
P{ Xˆ X }, the threshold should be
The autocorrelation function of {Yn }nn ==
(a) strictly positive
(b) zero denoted by Ry[k], is
(c) strictly negative R Y [k ]
(d) strictly positive, zero or strictly negative 1
depending on the non-zero value of 2
(a)
[EC-2014 : 2 Marks]
k
–3 –2 –1 0 1 2 3
Q.62 Consider the Z-channel given in the figure. The
input is 0 or 1 with equal probability. R Y [k ]

1.0 1.25
0 0
0.5 0.5
(b)

Input Output k
0.25 –3 –2 –1 0 1 2 3

1 1 R Y [k ]
0.75 1.25
If the output is 0, the probability that the input (c) 0.5 0.5
0.25 0.25
is also 0 equals _______ .
k
[EC-2014 : 2 Marks] –3 –2 –1 0 1 2 3

Q.63 Consider a discrete-time channel Y = X + Z, R Y [k ]


where the additive noise Z is signal-dependent. 1.25
In particular, given the transmitted symbol
X {–a, + a} at any instant, the noise sample Z is (d) 0.25 0.25

chosen independently from a Gaussian k


–3 –2 –1 0 1 2 3
distribution with mean X and unit variance.
[EC-2015 : 2 Marks]
Assume a threshold detector with zero threshold
at the receiver. Q.65 A zero mean white Gaussian noise have power
When, = 0 the BER was found to be spectral density N0/2 is passed through an LTI
Q(a) = 1 × 10–8 filter whose impulse response h(t) is shown in
the figure. The variance of the filtered noise at
1 u 2 /2 v 2 /2
Q( v ) = e du , and for v > 1 , use Q( v ) e
2 t = 4 is
v
GATE Previous Years Solved Paper 33

(a) Te = 169.36 K and Pa0 = 3.73 × 10–10 W


h(t)
(b) Te = 170.8 K and Pa0 = 4.56 × 10–10 W
A (c) Te = 182.5 K and Pa0 = 3.85 × 10–10 W
(d) Te = 160.62 K and Pa0 = 4.6 × 10–10 W
t
0 1 2 3 [EC-2016 : 2 Marks]

–A Q.69 An information source generates a binary


sequence { n} n can take one of the two

3 2 3 2 possible values –1 and +1 with equal probability


(a) A N0 (b) A N0 and are statistically independent and
2 4
identically distributed. This sequence is
1 2
(c) A2N0 (d) A N0 precoded to obtain another sequence { n}, as
2
n = n + k n – 3. The sequence { n} is used to
[EC-2015 : 2 Marks]
modulate a pulse g(t) to generate the baseband
Q.66 A random binary wave y(t) is given by signal,

y (t ) = Xn p (t nT ) X(t) = n g (t nT )
n= n=

where p(t) = u(t) – u(t – T), u(t) is the unit step 1, 0 t T


function and is an independent random where, g(t) =
0 , otherwise
variable with uniform distribution in [0, T]. The
sequence consists of independent and 1
If there is a unit at f = in the power spectral
identically distributed binary valued random 3T
variables with P{Xn = +1} = P{Xn = –1} = 0.5 for density of X(t), then k is ______ .
each n. The value of the autocorrelation [EC-2016 : 2 Marks]
3T 3T
Ryy E y (t ) y t equals _____ . Q.70 Consider a random process X(t) = 3V(t) – 8,
4 4
where V(t) is a zero mean stationary random
[EC-2015 : 2 Marks]
process with autocorrelation Rv ( ) = 4 e 5 .
Q.67 The variance of the random variable X with
The power in X(t) is ______ .
1 [EC-2016 : 2 Marks]
probability density function f ( x ) = x e x is
2
Q.71 A wide sense stationary random process X(t)
______ .
passes through the LTI system shown in the
[EC-2015 : 2 Marks]
figure. If the autocorrelation function of X(t) is
Q.68 An antenna pointing in a certain direction has RX( ), then the autocorrelation function RY( ) of
a noise temperature of 50 K. The ambient the output Y(t) is equal to
temperature is 290 K. The antenna is connected
to a pre-amplifier that has a noise figure of 2 dB +
and an available gain of 40 dB over an effective X(t) Y(t )
bandwidth of 12 MHz. The effective input noise –

temperature Te for the amplifier and the noise Delay = T0

power Pa0 at the output of the pre-amplifier,


(a) 2RX( ) + RX( – T0) + RX( + T0)
respectively are
34 Electronics Engineering Communication Systems

(b) 2RX( ) – RX( – T0) – RX( + T0) distributed between 0 and 2. Assume and U and
(c) 2RX( ) + 2RX( – 2T0) V are statistically independent. The mean value
(d) 2RX( ) – 2RX( – 2T0) of the random process at t = 2 is _____ .
[EC-2016 : 2 Marks] [EC-2017 : 1 Mark]

Q.72 Let X(t) ne a code sense stationary random Q.74 Consider a white Gaussian noise process N(t)
process with the power spectral density SX(f ) with two-sided power spectral density
as shown in Fig. (a), where f is in Hertz (Hz). SN(f ) = 0.5 W/Hz as input to a filter with
The random process X(t) is input to an ideal t 2 /2
impulse response 0.5 (where t is in
low-pass filter with the frequency response
seconds) resulting in output Y(t). The power in
1
1, f Hz Y(t) in Watts is
2
H( f ) = (a) 0.11 (b) 0.22
1
0, f > Hz
2 (c) 0.33 (d) 0.44

as shown in Fig. (b). The output of the low-pass [EC-2018 : 2 Marks]


filter is Y(t). Q.75 Let a random process Y(t) be described as
SX(f ) Y(t) = h(t) X(t) + Z(t), where X(t) is a white
noise process with power spectral density
SX(f ) = 5 W/Hz. The filter h(t) has a magnitude
exp( f )
response given by H ( f ) = 0.5 for –5 f 5, and

f zero elsewhere, Z(t) is a stationary random


0
process, uncorrelated with X(t), with power
(a)
spectral density as shown in the figure. The
Ideal low pass power in Y(t), in Watts, is equal to W _____
X(t) filter h(t) Y(t ) (rounded off tow two decimal places).
cut-off =1/2 Hz
SZ(f ) (W/Hz)
(b) 1
Let E be the expectation operator and consider
the following statements:
I. E(X(t)) = E(Y(t)) f (Hz)
–5 5
II. E(X2(t)) = E(Y2(t))
[EC-2019 : 2 Marks]
III. E(Y2(t)) = 2
Select the correct option: Q.76 The random variable,
(a) Only I is true. Y= W (t ) (t ) dt
(b) Only II and III are true.
1, 5 t 7
(c) Only I and II are true. where, (t) =
0 , otherwise
(d) Only I and III are true.
and W(t) is a real white Gaussian noise process
[EC-2017 : 2 Marks]
with two-sided power spectral density
Q.73 Consider the random process, SW(f ) = 3 W/Hz, for all f. The variance of Y is
X(t) = U + Vt _______ .
where U is a zero mean Gaussian random [EC-2020 : 1 Mark]
variable and V is a random variable uniformly
GATE Previous Years Solved Paper 35

Q.77 In a high school having equal number of boy a uniform quantizer. The PCM output is
students and girl students, 75% of the students transmitted over a channel that can support a
study Science and the remaining 25% students maximum transmission rate of 50 kbps.
study Commerce. Commerce students are two Assuming that the quantization error is
times more likely to be a boy than are Science uniformly distributed, the maximum signal to
students. The amount of information gained in quantization noise ratio that can be obtained
knowing that a randomly selected girl student by the PCM system (Rounded off to two decimal
studies Commerce (Rounded off to 3 decimal places) is ________ .
places) is ______ bits. [EC-2021 : 2 Marks]
[EC-2021 : 2 Marks]
Q.81 The auto-correlation in function RX( ) of a wide
Q.78 A box contains the following three coins: sense stationary random process X(t) is shown
I. A fair coin with head on one face and tail in the figure.
on the other face. RX( )
II. A coin with heads on both the faces. 2
III. A coin with tails on both the faces.
A coin is picked randomly from the box and
tossed. Out of the two remaining coins in the
box, one coin is then picked randomly and –2 0 2
tossed. If the first toss results in a head, the
The average power of X(t) is _______ .
probability of getting a head in the second toss
[EC-2021 : 1 Mark]
is
2 2 Q.82 Consider a real valued source whose samples
(a) (b) are independent and identically distributed
5 3
random variables with the probability density
1 1
(c) (d) function f(x), as shown in the figure.
3 2
[EC-2021 : 2 Marks] f( x )

Q.79 Two continuous random variable ‘X’ and ‘Y’


are related as:
Y = 2X + 3
x
2
Let X and Y2 denote the variances of X and –2 0 1

Y, respectively. The variances are related as, Consider a 1-bit quantizer that maps positive
2 2 2 2 samples to value and others to value . If
(a) Y =2 X (b) Y =4 X
and are the respective choices for and
(c) 2 2 (d) 2 2 that minimize the mean square quantization
Y =5 X Y = 25 X
[EC-2021 : 1 Mark] error, then ( – ) = ________ (Rounded off to
two decimal places).
Q.80 A message signal having peak-to-peak value of [EC-2022]
2 V, root mean square value of 0.1 V and
bandwidth of 5 kHz is sampled and fed to a
pulse code modulation (PCM) system that uses
36 Electronics Engineering Communication Systems

Answers Random Signals and Noise

1. (d) 2. (d) 3. (a) 4. (b) 5. (b, c) 6. (Sol.) 7. (b) 8. (d)

9. (d) 10. (d) 11. (d) 12. (d) 13. (a) 14. (c) 15. (c) 16. (c)

17. (a) 18. (a) 19. (c) 20. (d) 21. (d) 22. (c) 23. (d) 24. (a)

25. (b) 26. (b) 27. (c) 28. (b) 29. (a) 30. (a) 31. (a) 32. (a)

33. (b) 34. (a) 35. (c) 36. (b) 37. (a) 38. (a) 39. (c) 40. (b)

41. (b) 42. (b) 43. (d) 44. (b) 45. (d) 46. (a) 47. (b) 48. (a)

49. (b) 50. (d) 51. (0.33) 52. (b) 53. (d) 54. (0.346) 55. (4) 56. (2.5)

57. (0.16) 58. (2) 59. (c) 60. (0.8) 61. (c) 62. (0.8) 63. (c) 64. (b)

65. (a) 66. (0.25) 67. (6) 68. (a) 69. (–1) 70. (100) 71. (b) 72. (a)

73. (2) 74. (b) 75. (17.5) 76. (6) 77. (3.3219) 78. (c) 79. (b) 80. (30.72)
81. (2) 82. (1.167)

Solutions
Random Signals and Noise
1. (d) Therefore placing the pre-amplifier on the
Var(–Kx) = E[(–Kx)2 – (E[–Kx)])2 antenna side of the waveguide will result in the
2 = E[K2 x2] – K2 E[Kx] reduction of overall noise figure of the system.
2 = K2(E2 [x2] – E[x]2)
4. (b)
2 = K 2 2
x Half wave rectification can be represented as,
y = x for x 0
2. (d) = 0 for x < 0
Narrow-band representation of noise is, 1 1 y 2 /2 N
So, f(y) = (y) + e
n(t) = nc(t) cos ct – ns(t) sin ct 2 2 N
Its envelope is,
R(t) = nc2 (t ) + ns2 (t ) E[Y2] = y 2 f ( y ) dy
0
where, nc(t) and ns(t) are two independent, zero
mean Gaussian processes, with same variance.
1 1 y 2 /2 N
The resulting envelope is Rayliegh variable. E[Y2] = y2 (y) + e dy
2 2 N
0
3. (a)
A pre-amplifier is a very large gain amplifier y2 y 2 /2 N
E[Y2] = 0 + e dy
with low noise figure. 0
2 N
Noise figure of cascaded amplifier can be given
y
as: Let, =t
N
F 1 F3 1 Fn 1
F = F1 + 2 + + ... + dy =
G1 G1G2 G1G2 G3 ....Gn 1 N dt
GATE Previous Years Solved Paper 37

So,
1 2 t 2 /2
E[Y2] = Nt e N dt 4 KTe B(R1 + R2) = 4KT1BR1 + 4KT2BR2
2 N 0 4 KTe B(R1 + R2) = 4KB(R1T1 + R2T2)
Te (R1 + R2) = R1T1 + R2T2
N
E[Y2] =
2 R1T1 + R2T2
Te =
R1 + R2
5. (b, c)
Given that, Te = 21 °K, Ta = 300°K 7. (b)
Gain = (G)dB = 13
13 = 10 log10 (G) Mean = µX = E[ X } = xp( x ) dx
G = 19.95
Cable loss = (3)dB = 10 log10(L) 3 3
L = 1.995 1 1 x2
µX = x dx =
For a cable: 4 4 2
1 1
Noise in figure = F2 = cable loss =L
1 1 1
F2 = 1.995 µX = × [9 ( 1)2 ] = × 8
4 2 8
T µX = 1
Noise figure of amplifier = F1 = 1 + e
Ta Variance =
21 2
= E[(x – µX)]2
F1 = 1 + = 1.07 X
300
Noise figure of cascaded amplifier, ( x µ X )2
= X ( x ) dx
F 1
F = F1 + 2
G 3 3
1 1
1.995 1
2
X = ( x 1)2 dx = ( x 1)2 dx
= 1.07 + = 1.12 4 4
1 1
19.95
F = 0.49 dB 3
1
2
X = ( x 2 + 1 2 x ) dx
Te of the cascaded amplifier, 4
1

Te = (F – 1) Ta 3
2 1 x3 2x2
X = +x
Te = (1.12 – 1) 300 = 36°K 4 3 2
1

6. Sol. 2 1 27 1
X = +3 9 1 1
4 3 3

R1 R2 1 1 1 1
2 3 + + 2 = × [9 + 1 + 6]
Vn21 Vn22 X =
4 3 4 3
2 2
Vn2 = Vn1 + Vn2 2 4
X =
3
Vn2 = 4 KTe BR
8. (d)
= 4 KTe B(R1 + R2)
For a narrow-band noise with Gaussian
Vn21 = 4 KT1BR1 quadrature components the probability density
function (PDF) of its envelope will be Reyleigh.
Vn22 = 4 KT2BR2
38 Electronics Engineering Communication Systems

9. (d) when, = 1 and µ = 0 (zero mean)


Autocorrelation function of energy signal has 1 x2
X(x) =
exp ...(i)
conjugate symmetry, 2 2
Rx( ) = Rx* ( ) Given that,
If the function is real, then the autocorrelation x2
X(x) = k exp ...(ii)
function has even symmetry, 2
Rx( ) = Rx(– ) By comparing equation (i) and equation (ii), we
10. (d) have

Autocorrelation function and power spectral 1


k=
density makes the Fourier transfer pair, 2

RX ( )
F.T.
GX ( ) 14. (c)
The Fourier transform of a Guassian signal in
2
sin f f time domain is also Gaussian signal in the
RX( ) = F 1 =F 1
sin c 2
f frequency domain,
t2 F.T. f2
Inverse Fourier transform of square of sinc e e
function is always a triangular signal in time
15. (c)
domain.
The probability density function (PDF) of the
11. (d) envelope of narrow-band Gaussian noise is
10 log10 (2) = 3 dB Rayleigh.
10 log10 (4) = 6 dB
16. (c)
So, F1 = 2 and G1 = 4
Probability of a Gaussian random variable is
For cascaded system,
defined for an internal and not at a point. So at
F1 1
F2 = F1 + = 2.25 X = 4, it is zero.
G1
10 log10(2.25) = 3.5 dB 17. (a)

SX ( ) H( ) SY ( )
12. (d)
2
SY( ) = H ( ) SX ( )
Rx( ) = x (t + ) x(t ) dt
18. (a)
= x( t ) x( t ) t = E [d2(n)] = E[{x(n) – x(n – 1)}2]
2 = E[x(n)]2 + E[x(n – 1)]2
The autocorrelation function (ACF) of a d
rectangular pulse of duration T is a triangular –2E[x(n) x(n – 1)]
pulse of duration 2 T. 2
X 2 2
= X+ X 2 RXX (1)
13. (a) 10
Gaussian probability density of random Autocorrelation at k = 1,
variable ‘x’ is given by 19 2
2RXX(1) = X
2 10
1 ( x µ)
X(x) = exp 2 RXX (1) 19
2 2 = = 0.95
2
X 20
GATE Previous Years Solved Paper 39

19. (c) 22. (c)


Frequency detector can be modelled as a W = Y–Z
differentiator, 2 = E[Y2] + E[Z2] – 2E[Y Z]
W
N
SX ( f ) = 0 H(f ) = j2 f SY ( f ) = SX ( f ) H ( f )
2
Y is sampled at t = 2 and Z at t = 4
2
2 2 2
N0 = + 2 RXX (2)
SY ( f ) = 4 2f2 Parabolic W Y Z
2
= 8 + 8 2[4 e0.2121 + 1]
20. (d) 2
W = 2.64
fX(x)

K 23. (d)
Area under fX(x) = 1 P(x = 1) = FX(x = 1+) – FX(x = 1–)
1
x K= = 0.55 – 0.25 = 0.30
–1 1 2
24. (a)
f Y(y )
3
K
1
Ps 10
1 SNR = = 20
K1 = N 0 B 10 × 100 × 106
3
–2 1
y = 10 9
(SNR)dB = 10 log10 9 = 90 dB
fX(x) fY(y) Cable loss = 40 dB
1/3 SNR at RX = (90 – 40) dB = 50 dB
1
= K 11 –1/6
6 25. (b)

z fX(x)
–3 –2 –1 2
1
1
K 11 =
6
1 x
. 0 1
Since it lies in middle of
3
Qe = s.v. – q.v. = x – xq
Prob [z –2] = Area of graph [z –2]
Mean square value of quantization noise
1 1 1
= × ×1 = = E [( x xq )2 ]
2 6 12
1
21. (d) = ( x xq )2 f X ( x ) dx
0
RXX( ) = 4[ e 0.2 + 1]
1
P(X 1) = FX(1) = ( x xq )2 1 dx
0
X µ
= 1 Q at X = 1 0.3 1
= ( x 0)2 dx + ( x 0.7)2 dx
µ = 0 (Given) 0 0.3
2 = 8
= 0.039
= 2 2 Root mean square value of the quantization
noise
1
P(X 1) = 1 Q
2 2 = 0.039 = 0.198
40 Electronics Engineering Communication Systems

26. (b)
RX( )
PSDout = H ( )2 PSDin = 4N 0 IFT

PN = B.W. × SNout = 4N0B

27. (c) 0.05 ms

4
E[X2] = x 2 f X ( x ) dx
0.03 ms
0
RX( ) 0 at = 0.03 m-sec. So correlated.
x
f X ( x ) = y = mx + c =
8 31. (a)
K×4
= Area = 1 16
2 S( ) = ...(i)
2
1 16 +
K=
2 4
H(s) =
4 4+s
4
2 x x4 which is a LPF.
E[X2] = x dx = =8
8 4×8 It is a low pass RL filter.
0 0

28. (b) 32. (a)

Probability of occurrence of each region is, L


a
1
f X ( x ) dx =
3 R
a
a
1 1
x dx =
4 3
a R
2a 1 H(j ) =
R+ j L
=
4 3
Comparing with (i) and (ii),
2 R = 4, L = 1
a=
3
33. (b)
29. (a)
1
a a fX(x) = [u ( x + 5) u ( x 5)]
1 10
E[X2] = x 2 f X ( x ) dx = 2 x 2 dx
4 y fX(x)
a 0

2 /3 2 /3 1/10
1 2 1 x3
E[X2] = 2 x dx =
4 2 3
0 0 x x
–2.5 2.5 –5 5
1 8 4
E[X2] = × =
6 27 81 Y is a discrete random variable takes the values
of 0 and 1,
30. (a)
Y = 1; 2.5 x 2.5
s(f ) s0(f )
Y = 0; x 2.5
N0/2 LPF N0/2
fc = 10 kHz 1
f f
P(Y = 0) = P( 2.5 x 2.5) =
–10 K 10 K 2
GATE Previous Years Solved Paper 41

1 fc2
P(Y = 1) = P ( x 2.5) = Output PSD = H ( f ) 2 Input PSD = K
2 f 2 + fc2
1 1
fY(y) = (y) + ( y 1)
2 2 Output noise power = (output PSD) df

34. (a)
f c2
2 = K df
x = E[x2] – E2[x] f 2 + f c2
AC power = Total power – DC power = K fc (By substitution f = fc tan )

35. (c) 40. (b)


Autocorrelation function is an even function. PSD of white noise = 1 × 10–10 W/Hz ( K)
PSD of output,
36. (b)
2
PSD is always a positive quantity. G0(f) = H ( f ) G( f )
2
37. (a) = k H( f )

PDF : fX(x) = (t + 1) u(t + 1)–2tu(t) + (t – 1) u(t – 1) Output noise power,


x + f0
CDF : FX(x) = f X ( x ) dx N0 = G0 ( f ) df
f0
Integral of increasing ramp signal is increasing
= k × (area under H ( f ) 2 curve)
parabola and integral of decreasing ramp signal
is decreasing parabola. 1
= k×2 bh = k f0 × 1
2
35. (a) = 1 × 10–10 × 10 × 103
PDF, PX(x) = M exp ( 2 x ) + N exp ( 3 x ) = 10–6 W

41. (b)
PX ( x ) dx = 1
PSD = FT [R( )]
R( )
2x 3x
or, {M e +Ne } dx = 1
= FT

2x 3x 1
or, (M e +Ne ) dx =
2
sinc2 function
M N 1
or, + =
2 3 2 42. (b)
2N
or, M+ =1 Mean = X = xi p( xi )
3
= (1 × 0.1) + (2 × 0.2) + (3 × 0.4)
39. (c) + (4 × 0.2) + (5 × 0.1) = 3
1 1
H(f ) = = X2 = x12 P( xi )
1 + j 2 fRC 1 + j f
fc = (1 × 0.1) + (4 × 0.2) + (9 × 0.4)
fc2 + (16 × 0.2) + (25 × 0.1) = 10.2
2
H( f ) =
f 2 + fc2 2 = X 2 ( X )2 = 1.2
42 Electronics Engineering Communication Systems

43. (d) 45. (d)


When a ‘1’ is transmitted:
+
YK = a + NK
y1(t) d
x(t) y (t ) a
dt Threshold, Z= = 10 6
+ 2
Delay = 0.5 ms a = 2 × 10–6
For error to occur,
y1(t) = x(t) + x(t – 0.5 × 10–3)
YK < 10 –6
3 2 × 10–6 + NK < 10 –6
Y1(t) = X ( f )[1 + e j 2 f (0.5 × 10 ) ]
NK < –10 –6
Y1 ( f ) j f × 10 3
H1(f) = = 1+ e 0 10 6
X( f ) P = PNK (n ) dn
1
H(f ) = H1(f ) H2(f )

j ( f × 10 3 ) 10 6
= ( j 2 f ) [1 + e ] n
= 0.5 e dn with = 107
2
H( f ) = 4 2f 2 [2 + 2 cos( f × 10–3)]
0
2f2 P = 0.5 e–10
=8 [1 + cos( f × 10–3)] 1
2 When a ‘0’ is transmitted:
SY(f ) = H ( f ) SX ( f )
YK = NK
= 8 2 f 2 [1 + cos( f × 10–3)] SX(f )
For error to occur,
For f = (2n + 1) f0 with f0 = 1 kHz
–3 YK > 10 –6
f × 10 is an odd multiple of .
SY(f ) = 0 1
P = PNK (n) dn = 0.5 × e 10
0
44. (b) 10 6

2 Since, both bits are equiprobable,


Output noise power = H( f ) (Input noise PSD)
1
2 P(0) = P(1) =
SN0 ( f ) = SN ( f ) H( f ) 2
20 2 The probability of bit error
SN0 ( f ) = 10 H( f )
0 1
H(f ) SN0(f ) = P(1) P + P(0) P
1 0
–20
1 10
0 1
P +P
1 0 10
f f = = 0.5 × 10
–1M 0 1M –1M 0 1M 2

46. (a)
Output noise power = SN 0 ( f ) df
d/dt
W +
= 10 20 × 2 × 106 Hz x(t ) y(t )
Hz

= 2 × 10–14 W
Since mean square value = Power
d
2 y(t) = x(t ) x(t )
= 2 × 10–14 = 107 dt
2
GATE Previous Years Solved Paper 43

FT : Y(f ) = 2j f X(f ) – x(f ) So, P = E[X2 (t)]


Y(f ) = [j2 f – 1] x(f ) 11 × 103
1
2 = 2× SX ( ) d
PSD : SY(f ) = j 2 f 1 SX ( f ) 2
9 × 103
SY(f ) = (4 2f 2 + 1) SX(f )
1 1
SX(f ) = FT [RX( )] E[X2 (t)] = 400 + × 6 × 2 × 10 3
2
f2
SX(f ) = e 6400
E[X2 (t)] =
F.T.
Q [Gaussian function Gaussian
At = 0, there is no any frequency component
function] present, hence dc value of the process is zero.
2
SY(f ) = [4 2 f 2 + 1] e f 49. (b)
The probability,
47. (b)
P(3V – 2U) = P(3V – 2U 0)
–1 x 1 and –1 y 1 is the entire rectangle.
= P(W 0)
The region in which maximum of {x, y} is less
where, W = 3V – 2U
than 1/2 is shown below as shaded region
U and V are independent random variables and
inside this rectangle,
can be expressed in terms of mean and variance
as shown below,
(–1, 1) 1 (1, 1)
1
U = N 0,
4
1/2
1
U = N 0,
–1 1/2 1
x
9
0 Q W = 3V – 2U
1 1
W = N 0, 9 × + 4 ×
9 4
(–1, –1) –1 (1, –1) = N(0, 2)
Hence W is Gaussian variable with mean 0 and
1 Area of shaded region
p max { x , y} < = variance 2.
2 Area of entire rectangle
fW(W)
3 3
×
9
= 2 2 = Shaded area = P(W 0)
2 × 2 16

48. (a)
w
SX( ) 0
400 ( – 104)
1
6 P(W 0) =
2

50. (d)

0
F(x) = P{X x}
9 10 11 (103 rad/s)
G(x) = P(2x x)
We know that, E[X2(t)] represent the total power
x
in random signal. = P x
2
44 Electronics Engineering Communication Systems

For positive values of x, 55. Sol.


F(x) – G(x) 0
1
For negative values of x, , f W
SX(f) = W
F(x) – G(x) < 0 0, f >W
But, [F(x) – G(x)] x 0 ...(for all values of x)
SX(f )
51. Sol.
If multiple independent random variables are 1/W
uniformly distributed in the same interval then
each random variable will have equal chances f
–W 0 W
to be largest and to be lowest.
Q RX( ) [SX(f )]
1
P(X1 is the largest) = = 2 sinc[2W ]
3
Also,
52. (b) 1 1
E x( t ) x t = E x(t ) x t
2 4W 4W
Variance, X = E(X2) – [E(X)]2

2 1 1
Q X can never be negative. = RX = 2 sin c ...(i)
4W 2
E(X2) [E(X)2]
We know that,
53. (d) sin x
sinc(x) =
E[X(t1) X(t2)] x
= E[A sin(2 t1 + )] × A sin(2 t2 + )] From equation (i), we get,

A2 sin
= E [cos[2 (t1 t2 )] cos[2 (t1 + t2 + 2 )]
2 2 2
=4
2
A 2
= cos[2 (t1 t2 )
2
56 Sol.
54. Sol.
10 6 (300 f ) W/Hz for f 3 kHz
Assume quantization voltage is VTh. SX ( f ) =
0 otherwise
For outputs of quantizer to have equal
probability. SX(f )
P(X VTh) = P(X VTh) –3
3 × 10
VTh
f X ( x ) dx = f X ( x ) dx 2 × 10
–3

VTh
f (kHz)
VTh –3 –1 1 3
2x 2x
2e dx = 2e dx
VTh PSD
0 If, x( t ) SX ( f )

1 Then,
2 × VTh =
e 2 SX ( f fc ) + SX ( f + fc )
PSD
x(t ) cos 2 f c t
VTh = 0.346 4
PSD of x(t) cos2 × 8000t
GATE Previous Years Solved Paper 45

3
3 × 10 1
4

0 2 –1 0

–11 –8 –5 0 –11 –8 –5 f (kHz) 0


( y + 1)2
So, P(Y 0) = dy
Now, BPF is given by 2
1
H(f ) 0
Y3
P(Y 0) = = 0.16
1 6
1

f (kHz) 58. Sol.


–9 –8 –7 0 7 8 9
1 2
We know that, P(X = 1) = and P(X = 0) =
3 3
2
SX(f ) H(f ) H ( f ) SX ( f ) 1
and H(Y1) = P(Y1 )log 2
P(Y1 )
PSD of BPF output,
1
H(Y2) = P(Y2 )log 2
P(Y2 )
3 × 10 3
4
where, P(Y1 = 1) = P(Y1 = 0)
1 1 2 1 1
= × + × =
3 2 3 2 2
2 × 10 3
4
Similarly,
f (kHz) 1
–9 –8 –7 0 7 8 9 P(Y2 = 1) = P(Y2 = 0) =
2
Output power = Total area of output PSD Here,
1 1
2 × 10 3 1 10 3 H(Y1) + H(Y2) = log 2 2 + log 2 2
= 2 (2000) + 2 × 1000 × 2 2
4 2 4
1 1
+ log 2 2 + log 2 2
1 2 2
= 2 1+ = 2.5 Watts H(Y1) + H(Y2) = 2 bits
4
59. (c)
57. Sol.
Shifting operation does not change the power
X1 + X2 X3
spectral density but scaling operation change
X1 + X2 – X3 0
the power spectral density:
So, P{X1 + X2 – X3 0} = P{Y 0}
y(t) = x(2t – 1)
and Y is another random variable whose
probability density function is given by x(t) Rx ( z ) Sx ( f )
convolution of X1, X2 and X3. 1 f
x(2t) Rx (2 z) Sx
X1 X2 X3 2 2
1 1 1
1 f
= x(2t – 1) Rx (2 z) Sx
2 2
0 1 0 1 0 2
1 f
So, Y(t) Sy ( f ) = Sx
2 2
46 Electronics Engineering Communication Systems

60. Sol. P(X1) = 0.75, P(X2) = 0.25


For a Gaussian random variable, 2 2
0.25
0 = ln =
2 2 0.75 2
µ x
1 2 2 So, 0 should be strictly negative.
fX(x) = e
x 2
62. Sol.
2
For mean (µ) = 0 and variance ( X)= 1,
x1 0
1.0
y1 0
1 x 2 /2
fX(x) = e
2 Input Output
0.25

E[ x ] = x f X ( x ) dx x2 1 y2 1
0.75
Given, P(X) = [0.5 0.5]
1 x 2 /2
= x e dx y1 y2
2
Y x1 1 0
P =
0
1 X x2 0.25 0.75
x 2 /2
= x e dx
2 0.5 0
Then, P[X, Y] = 0.125 0.375
1 x 2 /2 and P(Y1) = 0.625
+ x e dx
0
2 P(Y2) = 0.375
and the conditional probability matrix.
1 x 2 /2
= 2 x e dx X
0
2 P is given by,
Y

2 y1 y 2
x 2 /2
= x e dx x1 0.8 0
2 X
0 P = x
Y 2 0.33 1
x2
Let, =t x1 0
2 Hence, P = P = 0.8
y1 0
2 x dx
= dt
2 63. (c)
Hence, Y = X+Z
2 Z is Gaussian RV with mean x,
t
e dt = 0.8 x {–a, +a}
2 0
when = 0;
E [ x ] = 0.8 E[Y] = E[X] + E[Z]
E[Y] = E[X] = +a
61. (c) BER = Q(a) = 1 × 10–8
The optimum threshold value is,
1 v2 /2 v2 /2
Q(v) = e du e
2
P( x2 ) x12 x22 2
0 = ln + 2
v
x1 x2 P( x1 ) 2
8 a2 /2
Q(a) = 1 × 10 e
x 1 = 1, x2 = –1
a=6
GATE Previous Years Solved Paper 47

when, = –0.3 mean = 6 × –0.3 = –1.8 R Y [k ]


So, E[Y] = E[X] + E[Z] 1.25
= 6 – 1.8 = 4.2
0.5 0.5
a2 /2
So, BER = Q (4.2) e
k
0.001 10–4 –3 –2 –1 0 1 2 3

64. (b) 65. (a)


RY(K) = RY(n, n + K) Variance of a process = RX(0)
= E[Y(n) Y[n + K]] Let variance (output) = RY(0)
Given, Q It is zero mean.
1 RY( ) = h( ) h(– ) RN( )
P[x(n) = 1] = where, h( ) = filter response and RN( ) is the
2
input noise,
1
P[x(n) = –1] = N0
2 RN( ) = ( )
Y[n] = X[n] + 0.5X[n]
2
N0
RY (K) = E[x[n] + 0.5x[n – 1]] RN( ) = [ h( ) × h( )]
2
[x[n + K] + 0.5x[n + K – 1]]
= E[x[n] x[n + K]] N0
RY( ) = h( ) h( + z) d
+ 0.5E[x[n] x[n + K – 1]] 2
+ 0.5E[x[n – 1] x[n + K]]
N0
+ 0.25E[x[n – 1] x[n + K – 1]] RY(0) = h2 ( ) d
2
= RX(K) + 0.5RX(K – 1)
N0
+0.5RX(K + 1) + 0.25RX(K) = (energy)
2
= 1.25 RX(K) + 0.5 RX(K – 1)
N0 3
+ 0.5RX(R + 1) = (3 A2 ) = N0 A2
2 2
= 1.25 RX(K) + 0.5 RX(K – 1)
+ 0.5RX(K + 1) 66. Sol.
Now, if K = 0, For a binary symmetric pulse.
1 1
RX(0) = E[ X 2 [n]] = 1 × + 1 × = 1 RYY(t) = 1
2 2 T
K 0 3T 3 /4 1
RYY = 1 = = 0.25
RX(K) = E[X(n) X(n – K)] 4 4
= E[X(n)] E[X(n – K)] = 0
67. Sol.
1 1
Because, E [ X(n)] = 1 × 1× = 0 1
2 2 x
fX(x) = xe
2
RY(0) = 1.25 RX(0) + 0.5 RX(–1)
fX(x) is an even function.
+ 0.5 RX(1)
= 1.25 So, E(X) = X = xf ( x ) dx = 0
Similarly, RY(1) = 0.5
2
RY(–1) = 0.5 Variance, X = E [ X 2 ] ( X )2 = E[ X 2 ]
48 Electronics Engineering Communication Systems

1 + k2 + 2k = 0
2 2 2
So, X = E[X ] = x f X ( x ) dx (k + 1)2 = 0
k = –1
1
= x2 x e x
dx = x3 e x
dx 70. Sol.
2 0 X(t) = 3V(t) – 8
and E[V(t)] = 0
= e x ( x 3 + 3x 2 + 6 x + 6) =6
0 5
Rv( ) = f = 4 e
68. (a) Power of X(t) = E[X2(t)]
(i) Te = (F – 1) T0 = E[9V2(t)] + 64 – 48 E[V(t)]
= (102/10 – 1) 290 = 9E[V2(t)] + 64 – 48 E[V(t)]
2
E[V (t)] = Rv(0) = 4
= 169.6 K
(ii) Ni = k(Tant + Te) B Power of X(t) = ((9 × 4) + 64) = 100
= 1.38 × 10–23 × (50 + 169.6) 71. (b)
× 12 × 106
Y(t) = X(t) – (t – T0)
= 3.63 × 10–14 W
RY( ) = E[Y(t + ) Y(t)]
N0 = Ni × Gain
= E[X(t + ) – X(t + – T0)) (X(t) – X(t – T0))]
= 3.63 × 10–14 × 104
= E[X(t + ) X(t) – X(t) X(t + – T0)
= 3.63 × 10–10 W
–X(t + ) X(t – T0) + X(t + – T0) X(t – T0)]
69. Sol. Since, X(t) is a WSS process,
Power spectral density of x(t) = SX(f ) RY( ) = RX( ) – RX( – T0) – RX( + T0) + RX( )
2
= 2RX( ) – RX( – T0) – RX( + T0)
G( f )
SX(f ) = Rb ( ) e j 2 fn
T 72. (a)
n=
The given input power spectral density is as
Rb( ) = E[ n n – t]
follows:
= E[( n + k n – 3) ( n – t + k n – – 3)]
= E[ n n – ] + kE[n – 3 n – ] + kE[ n n – – 3] SX (f )

+ k2E[ n – 3 n – – 3] 1
f
= E[ n n – ] + kE[ n – 3 n – – 3 + 3] SX ( f ) = e
+ kE[ n n – – 3]
= R( ) + kR( – 3) + kR( + 3) + k2R( ) f
0
= (1 + k2) R( ) + kR( + 3) + kR( – 3)
Hence autocorrelation function can be defined Frequency response of the low pass filter is as
as: follows:
H(f )
1 + k2 =0
1
Rb( ) = k = ±3
0 otherwise
f (Hz)
Power spectral density, –1/2 0 1/2

sb(f ) = 1 + k2 + 2k cos(2 f 3T) E[Y(t)] = H(0) E[X(t)]


1 H(0) = 1
Null will occur at, f =
3T So, E[Y(t)] = E[X(t)]
sb(f ) = 1 + k2 + 2k cos(2 ) = 0 E[Y2(t)] E[X2(t)]
GATE Previous Years Solved Paper 49

Since, LPF does not allow total power from input


1 t2
to output, = e dt
8

E[X2(t)] = SX ( f ) df = 2 W
= = 0.2215 W 0.22 W
0 8
As, E[Y 2(t)] E[X2(t)]
75. Sol.
E[Y 2(t)] 2
So, only statement-1 is correct. Let, X1(t) = h(t) X(t)
2
73. Sol. SX1 ( f ) = H ( f ) SX ( f )
X(t) = U + Vt SX1(f ) S Z( f )
At t = 2, X(t) = X(2) = U + 2V 2 1
(0.5) 5 = 1.25
E[X(t)] = E[U + 2V]
= E[U] + 2E[V]
f (Hz) f (Hz)
Given that, E[U] = 0 –5 5 –5 5

Given that, Z(t) and X(t) are uncorrelated.


fv(v)
So, SY(f ) = SX1 ( f ) + Sz ( f )
1/2 Power in y(t),
PY = [Area under SX1 ( f )] + [Area under Sz(f )]
v
0 2 = (10 × 1.25) + (5 × 1)
Also given that, = 17.5 W
V is uniformly distributed between 0 and 2. 76. Sol.
2
1 1; 5 t 7
So, E[V] = f v (V ) dv = dv = 1
2 Given, (t) =
0 0 ; otherwise
So, E[X(t)] = 0 + 2(1) = 2 S (f ) = 3 Watts/Hz
R () = 3 ( ) = 3 (t1 – t2)
74. (b)
Var [y] = E[y2] {E[y]}2
PSD of noise input, {E[W(t)]}2 = DC power
SN(f ) = 0.50 W/Hz = Area under PSD at
Power of y(t), f = 0;
2 {E[W(t)]}2 = 0
Py = SN ( f ) H ( f ) df
E[W(t)] = 0

2 y = W (t ) (t ) dt
= 0.50 H ( f ) df

2
E[Y] = E [ W (t )] (t ) dt = 0
= 0.50 h(t ) dt

1 t 2 /2 y = W (t ) (t ) dt E[Y 2 ]
Given that, h(t) = e
2
2 = S (f) Energy [ (t)]
1 1 t 2 /2 = 3×2=6
So, Py = e dt
2 2 Var [Y] = 6 – 0 = 6
50 Electronics Engineering Communication Systems

Detailed explanations for: 4 1


C ×
P( B /C ) P(C ) 5 4 2
P = = =
y = W (t ) (t ) dt B P( B) 1/2 5
C C
E(y2) = E[Y Y] P(C) = P P( B) + P P(G )
B G
= E W (t1 ) (t1 ) dt1 W (t 2 ) (t2 ) dt 2 1 2 1 C 1
= × +P
4 5 2 G 2

= E W (t1 ) W (t2 ) (t1 ) (t2 ) dt1 dt2 1 1 1


C 1
P = 4 5 = 20 =
G 1 /2 1/2 10
= E [ W (t1 ) W (t2 )] (t1 ) (t2 ) dt1 dt2 C 1
I= log 2 P = log 2 = log 2 10
G 10
= 3.3219 bits
= RW (t1 t2 ) (t1 ) (t2 ) dt1 dt2
78. (c)
Let ‘A’ be the even of getting head in first toss
= 3 (t1 t2 ) (t1 ) (t2 ) dt1 dt2 and ‘B’ be the event of getting head in second
toss. We need to find P(B/A),
Above integration exists provided,
P( A B )
t1 = t2 = t P(B/A) =
P( A )
3 (0) (t ) (t ) dt dt Getting head
= P(A) = P(Selected coin in fair) × P
Selected coin is fair
Getting head
+ P(Selected coin in double headed) × P
Selected coin is doubled headed
2
= 3 (0) dt (t ) dt 1 1 1 1
× + 1=
=
3 2 3 2
= 3 × 1 × Energy [ (t)] Let us now find probability of getting heads in
E[y2] =6 both the tosses.
77. (3.3219) Case-I : When the first tossed coin is a fair coin
1 1 1 1
1 1 1 =
P(B) = , P(G) = 2 3 2 12
2 2
3 1 Case-II : When the first tossed coin is a double
P(S) = , P(C ) = headed coin.
4 4
B B 1 1 1
P = 2P P(A B) = + =
C S 12 12 6
Required probability
B B
P(B) = P P(C ) + P P(S ) P( A B) 1/6 1
C S = = =
P( A) 1/2 3
1 B 1 B 3
= P +P
2 C 4 S 4 79. (b)
B B B 3 B Y = 2X + 3
2 = P + 3P =P + P
C S C 2 C 2
Var (Y) = Y = E[Y 2 ] (E(Y ))2
B 2 4
P = = = E((2X + 3)2) – (E(2X + 3))2
C 2.5 5
= E(4X2 + 9 + 12X) – (2mX + 3)2
GATE Previous Years Solved Paper 51

= 4E(X2) + 9 + 12mX – 4mx2 9 12 mx Quantizer output,


xq = ; for 0 x 1
= 4E( X 2 ) 4mx2 = 4 2X
xq = ; for –2 x 0
2 = 4 2X
Y Mean square quantization error,
If, Y = aX + b
MSQ [Qe] = E [Qe2 ]
2
Y = a 2 2X Qe = (Sampled value) –
(Quantized value)
80. (30.72)
= X – xq
Given that, Vrms = 0.1 V
MSQ [Qe] = E[(X – xq)2]
2
Signal power = Vrms = 0.01 Quantization noise power,
Bit rate = nfs = 50 kbps No = MSQ [Qe]
n × 2 × 5 × 103 = 50 × 103 = ( X xq )2 f X ( x ) dx
n=5 For –2 x 0
Peak to peak voltage 0
= 1 1
2 n NQ = (x )2 × x+ dx
4 2
2V 1 2
= 5
= 4 0
2 2 x 1
= (x 2 + 2
2x ) + dx
2
1 1 4 2
2
Noise power = = 4 2
= 8
12 12 × (2 ) 12 × 2 2 1 2
NQ = +
Signal power 2 3 3
SNR =
Noise power To find value for which NQ will be minimum
= 0.01 × 12 × 28 = 30.72 dNQ 1 2
=0 ×2 + = 0
d 2 3
81. (2)
2
Average power, RX(0) = 2 =
3
82. (1.167) For 0 x 1;
fX(x) 1
1
NQ = (x )2 × dx
K 2
0
2 1
K/ NQ = [(1 )3 + 3 ]
=
m 6
x To find ‘ ’ value for which NQ is minimum,
–2 0 1
dNQ 1
1 1 =0 [3(1 )2 ( 1) + 3 2
]=0
× K × 2 + 1× K = 1 K = = 0.5 d 6
2 2
–2 x 0 fX(x) = mx + C 1
=
fX(x) = 0.25x + C 2
Chosen and for which mean square
When x = –2 fX(x) = 0;
quantization error (NQ) is minimum will be 1/3
0 = 0.25 × –2 + C
and –2/3 respectively,
C = 0.25
1 2 7
1 1 – = + = = 1.167
fX(x) = x+ = 2 x 0 2 3 6
4 2
fX(x) = 0.5; 0 x 1
3 Digital Communication Systems

Q.1 Companding in PCM system lead to improved Q.5 In binary data transmission DPSK is preferred
signal to quantization noise ratio. This to PSK because
improvement is for (a) a coherrent carrier is not required to be
(a) lower frequency components only generated at the receiver.
(b) higher frequency components only (b) for a given energy per bit, the probability of
(c) lower amplitudes only error is less.
(c) the 180° phase shifts of the carrier are
(d) higher amplitudes only
unimportant.
[EC-1987 : 2 Marks]
(d) more protection is provided against impulse
Q.2 A signal having uniformly distributed noise.
amplitude in the interval (–V to +V), is to be [EC-1989 : 2 Marks]
encoded using PCM with uniform quantization.
Q.6 A 4 GHz carrier is DSB-SC modulated by a low
The signal to quantizing noise ratio is
pass message signal with maximum frequency
determined by the
of 2 MHz. The resultant signal is to be ideally
(a) dynamic range of the signal
sampled. The minimum frequency of the
(b) sampling rate
sampling impulse train should be
(c) number of quantizing levels
(a) 4 MHz (b) 8 MHz
(d) power spectrum signal
(c) 8 GHz (d) 8.004 GHz
[EC-1988 : 2 Marks]
[EC-1990 : 2 Marks]
Q.3 The message bit sequence to a DPSK modulator
Q.7 In a BPSK signal detector, the local oscillator
is 1, 1, 0, 0, 1, 1. The carrier phase during the
has a fixed phase error of 20°. This phase error
reception of the first two message bits is , . The
deteriorates the SNR at the output by a factor of
carrier phase for the remaining four message
(a) cos 20° (b) cos2 20°
bits is
(c) cos 70° (d) cos2 70°
(a) , , 0, (b) 0, 0, ,
[EC-1990 : 2 Marks]
(c) 0, , , (d) , , 0, 0
[EC-1988 : 2 Marks] Q.8 A signal has frequency components from
300 Hz to 1.8 kHz. The minimum possible rate
Q.4 In a digital communication system,
at which the signal has to be sampled is ____ .
transmissions of successive bits through a noisy
[EC-1991 : 2 Marks]
channel are assumed to be independent events
with error probability p. The probability of at Q.9 For the signal constellation shown in the figure,
most one error in the transmission of an 8-bit the type of modulation is _______ .
sequence is
cos2 (n/T)t
7(1 p)
(a) 7 (1 p )/8 S2 S2
p
8+ sin2 (n/T)t
8
T = symbol duration
(b) (1 – p)8 + 8p(1 – p)7 S3 S4
(c) (1 – p)8 + (1 – p)7
(d) (1 – p)8 + p(1 – p)7 [EC-1988 : 2 Marks]
[EC-1991 : 2 Marks]
GATE Previous Years Solved Paper 53

Q.10 A signal has frequency components from (b) attenuation of low frequencies in
300 Hz to 1.8 kHz. The minimum possible rate reproduction.
at which the signal has to be sampled is ____ (c) greater aliasing errors in reproduction.
(fill in the blank). (d) no harmful effects in reproduction.
[EC-1991 : 2 Marks] [EC-1994 : 1 Mark]

Q.11 The bit stream 01001 is differentially encoded Q.16 The bandwidth required for the transmission
using ‘Delay and EX-OR’ scheme for DPSK of a PCM signal increases by a factor of ____
transmission. Assuming the reference bit as a when the number of quantization levels is
‘1’ and assigning phases ‘0’ and ‘ ’ for 1’s and increased from 4 to 64.
0’s respectively, in the encoded sequence, the [EC-1994 : 1 Mark]
transmitted phase sequence becomes
Q.17 If the number of bits per sample in a PCM system
(a) 0 0 (b) 0 00
is increased from a n to n + 1, the improvement
(c) 0 0 (d) 0
in signal to quantization nose ratio will be
[EC-1992 : 2 Marks]
(a) 3 dB (b) 6 dB
Q.12 Coherrent demodulation of FSK signal can be (c) 2 n dB (d) n dB
detected using [EC-1995 : 1 Mark]
(a) correlation receiver
Q.18 A 1.0 kHz signal is flat top sampled at the rate
(b) bandpass filters and envelope detectors
of 1800 samples/sec and the samples are
(c) matched filters applied to an ideal rectangular LPF with cut-off
(d) discriminator detection frequency of 1100 Hz, then the output of the
[EC-1992 : 2 Marks] filter contains
(a) only 800 Hz component
Q.13 Source encoding in a data communication
(b) 800 Hz and 900 Hz components
system is done in order to
(c) 800 Hz and 1000 Hz components
(a) enhance the information transmission
(d) 800 Hz, 900 and 100 Hz components
(b) bandpass filters and envelope rate detectors
[EC-1995 : 1 Mark]
(c) conserve the transmitted power
(d) discriminator detection Q.19 The signal to quantization noise ratio in an n-bit
[EC-1992 : 2 Marks] PCM system
(a) depends upon the sampling frequency.
Q.14 Sketch the waveform (with properly marked
(b) is independent of the value of ‘n’.
axes) at the output of a matched filter matched
(c) increasing with increasing value of ‘n’.
for a signal s(t), of duration T, given by
(d) decreases with the increasing value of ‘n’.
2
A, for 0 t< T [EC-1995 : 1 Mark]
3
s(t ) =
2 Q.20 For a given data rate, the bandwidth Bp of a
0, for T t <T
3 BPSK signal and the bandwidth B0 of the OOK
[EC-1993 : 2 Marks] signal are related as
B B
Q.15 Increased pulse width in the flat top sampling, (a) Bp = 0 (b) Bp = 0
leads to 4 2
(a) attenuation of high frequencies in (c) Bp = B0 (d) Bp = 2B0
reproduction. [EC-1995 : 1 Mark]
54 Electronics Engineering Communication Systems

Q.21 A rectangular pulse of duration T is applied to (a) 9 (b) 8


a filter matched to this input. The output of the (c) 4 (d) 2
filter is a [EC-1998 : 1 Mark]
(a) rectangular pulse of duration T.
Q.26 Flat top sampling of low pass signals
(b) rectangular pulse of duration 2T.
(a) gives rise to aperture effect
(c) triangular pulse. (b) implies over sampling
(d) sine function. (c) leads to aliasing
[EC-1996 : 1 Mark] (d) introducing delay distortion
[EC-1998 : 1 Mark]
Q.22 The number of bits in a binary PCM system is
increased from n to n + 1. As a result, the signal Q.27 Quadrature multiplexing is
to quantization noise ratio will improve by a (a) the same as FDM.
factor (b) the same as TDM.
(a) (n + 1)/n (c) a combination of FDM and TDM.
(d) quite different from FDM and TDM.
(b) 2(n + 1)/n
[EC-1998 : 1 Mark]
(c) 22(n + 1)/n
(d) which is independent of n Q.28 Compression in PCM refers to relative
[EC-1996 : 2 Marks] compression of
(a) higher signal amplitudes
Q.23 The line code that has zero dc component for (b) lower signal amplitudes
pulse transmission of random binary data is (c) lower signal frequencies
(a) non-return to zero (NRZ) (d) higher signal frequencies
(b) return to zero (RZ) [EC-1998 : 1 Mark]
(c) alternate mark inversion (AMI)
Q.29 The Nyquist sampling frequency (in Hz) of a
(d) none of the above signal given by
[EC-1997 : 1 Mark] 6 × 104 sinc3(400t) 106 sinc3(100t) is
Q.24 A deterministic signal has the power spectrum (a) 200 (b) 300
given in figure. The minimum sampling rate (c) 1500 (d) 1000
needed to completely represent signal is [EC-1999 : 2 Marks]

S( f ) Q.30 The peak-to-peak input to an 8-bit PCM coder is


2 Volts. The signal power-to-quantization noise
power ratio (in dB) for an input of 0.5 cos( mt) is
(a) 47.8 (b) 43.8
(c) 95.6 (d) 99.6
[EC-1999 : 2 Marks]
f (kHz)
–2 –1.5 –1 0 1 1.5 2
Q.31 The input to a matched filter is given by
(a) 1 kHz (b) 2 kHz
10 sin(2 × 106 ) 0 < t < 10 4 sec
(c) 3 kHz (d) none of these s(t ) =
0 otherwise
[EC-1997 : 1 Mark]
The peak amplitude of the filter output is
Q.25 In a PCM system with uniform quantization, (a) 10 volts (b) 5 volts
increasing the number of bits from 8 to 9 will (c) 10 millivolts (d) 5 millivolts
reduce the quantization noise power by a
[EC-1999 : 2 Marks]
factor of
GATE Previous Years Solved Paper 55

Q.32 Four independent messages have bandwidths Q.37 Consider a sample signal,
of 100 Hz, 100 Hz, 200 Hz, and 400 Hz +
y(t ) = 5 × 10 6 x(t ) n=
(t nTs )
respectively. Each is sampled at the Nyquist rate
and the samples are Time Division Multiplexed where x(t) = 10 cos(8 × 103)t and Ts = 100 msec.
(TDM) and transmitted. The transmitted sample When y(t) is passed through an ideal lowpass
rate (in Hz) is filter with a cut-off frequency of 5 kHz, the
(a) 1600 (b) 800 output of the filter is
(c) 400 (d) 200 (a) 5 × 10–6 cos(8 × 103) t
[EC-1999 : 2 Marks] (b) 5 × 10–5 cos(8 × 103) t
(c) 5 × 10–1 cos(8 × 103) t
Q.33 In a digital communication system employing
(d) 10 cos(8 × 103) t
Frequency Shift Keying (FSK), the 0 and 1 bit are
[EC-2002 : 1 Mark]
represented by sine waves of 10 kHz and 25 kHz
respectively. These waveforms will be Q.38 For a bit-rate of 8 kbps, the best possible values
orthogonal for a bit interval of of the transmitted frequencies in a coherrent
(a) 45 µsec (b) 200 µsec binary FSK system are
(c) 50 µsec (d) 250 µsec (a) 16 kHz and 20 kHz
[EC-2000 : 2 Marks] (b) 20 kHz and 32 kHz
(c) 20 kHz and 40 kHz
Q.34 A video transmission system transmits 625
(d) 32 kHz and 40 kHz
picture frames per second. Each frame consists
[EC-2002 : 1 Mark]
of a 400 × 400 pixel grid with 64 intensity levels
per pixel. The data rate of the system is Q.39 A signal x(t) = 100 cos(24 × 103) t is ideally
(a) 16 Mbps (b) 100 Mbps sampled with a sampling period of 50 µsec and
(c) 600 Mbps (d) 6.4 Gbps then passed through an ideal lowpass filter
with cut-off frequency of 15 kHz. Which of the
[EC-2001 : 2 Marks]
following frequencies is/are present at the filter
Q.35 The Nyquist sampling interval, for the signal output?
sinc(700t) + sinc(500t) is (a) 12 kHz only
1 (b) 8 kHz only
(a) sec (b) sec
350 350 (c) 12 kHz and 9 kHz
1 (d) 12 kHz and 8 kHz
(c) sec (d) sec
700 175 [EC-2002 : 2 Marks]
[EC-2001 : 2 Marks]
Q.40 At a given probability of error, binary coherrent
Q.36 During transmission over a communication FSK is inferior to binary coherrent PSK by
channel, bit errors occur independently with (a) 6 dB (b) 3 dB
probability p. If a block of n-bits is transmitted, (c) 2 dB (d) 0 dB
the probability of at most one bit error is equal to
[EC-2003 : 1 Mark]
(a) 1 – (1 p)n
(b) p + (n – 1) (1 – p) Q.41 A sinusoidal signal with peak-to-peak
amplitude of 1.536 V is quantized into 128 levels
(c) np(1 – p)n – 1
using a mid-rise uniform quantizer. The
(d) (1 – p)n + np(1 – p)n – 1
quantization noise power is
[EC-2001 : 2 Marks]
56 Electronics Engineering Communication Systems

(a) 0.768 V (b) 48 × 10–6 V2 Q.46 In a PCM system, if the code word length is
(c) 12 × 10–6 V2 (d) 3.072 V increased from 6 to 8 bits, the signal to
[EC-2003 : 2 Marks] quantization noise ratio improves by the factor
8
Q.42 If Eb, the energy per bit of a binary digital signal, (a) (b) 12
6
is 10 –5 watt-sec and the one-sided power
(c) 16 (d) 8
spectral density of the white noise,
[EC-2004 : 1 Mark]
N0 = 10–6 W/Hz, then the output of SNR of the
matched filter is Q.47 In the output of a DM speech encoder, the
(a) 26 dB (b) 10 dB consecutive pulses are of opposite polarity
during time interval t1 t t2. This indicates
(c) 20 dB (d) 13 dB
that during this interval
[EC-2003 : 2 Marks]
(a) the input to the modulator is essentially
Q.43 The input to a linear delta modulator having a constant.
step-size = 0.628 is a sinewave with frequency (b) the modulator is going through slope
fm and peak amplitude Em. If the sampling overload.
frequency fs = 40 kHz, the combination of the (c) the accumulator is in saturation.
sinewave frequency and the peak amplitude, (d) the speech signal is being sampled at the
where slope overload will take place is Nyquist rate.
Em fm [EC-2004 : 1 Mark]
(a) 0.3 V 8 kHz
Q.48 A source produces binary data at the rate of
(b) 1.5 V 4 kHz
10 kbps. The binary symbols are represented as
(c) 1.5 V 2 kHz shown in the figure.
(d) 3.0 V 1 kHz
Binary 1
[EC-2003 : 2 Marks] 1V

Q.44 If S represents the carrier synchronization at the


0 0.1 t(msec)
receiver and represents the bandwidth
efficiency, the correct statement for the coherrent
Binary 0
binary PSK is
0 0.1
(a) = 0.5, S is required. t(msec)
(b) = 1.0, S is required. –1 V
(c) = 0.5, S is not required.
The source output is transmitted using two
(d) = 1.0, S is not required.
modulation schemes, namely Binary PSK (BPSK)
[EC-2003 : 2 Marks]
and Quadrature PSK (QPSK). Let B1 and B2 be
Q.45 A signal is sampled at 8 kHz and is quantized the bandwidth requirements of BPSK and QPSK
using 8-bit uniform quantizer. Assuming SNRq respectively. Assuming that the bandwidth of
for a sinusoidal signal, the correct statement for the above rectangular pulses is 10 kHz, B1 and
PCM signal with a bit rate of R is B2 are
(a) R = 32 kbps, SNRq = 25.8 dB (a) B1 = 20 kHz, B2 = 20 kHz
(b) R = 64 kbps, SNRq = 49.8 dB (b) B1 = 10 kHz, B2 = 20 kHz
(c) R = 64 kbps, SNRq = 55.8 dB (c) B1 = 20 kHz, B2 = 10 kHz
(d) R = 32 kbps, SNRq = 49.8 dB (d) B1 = 10 kHz, B2 = 10 kHz
[EC-2003 : 2 Marks] [EC-2004 : 2 Marks]
GATE Previous Years Solved Paper 57

Q.49 Consider a binary digital communication system x(t)


with equally likely 0’s and 1’s. When binary 0
1
is transmitted the detector input can lie between
the levels –0.25 V and +0.25 V with equal
probability. When binary 1 is transmitted, the 1 2 3 4 t(sec)
voltage at the detector can have any value
between 0 and 1 V with equal probability. If the –1
detector has a threshold of 0.2 V (i.e., if the
received signal is greater than 0.2 V, the bit is 1
(a) sec 1 (b) –1 sec–1
taken as 1), the average bit error probability is 2
(a) 0.15 (b) 0.2 1
(c) sec 1 (d) 1 sec–1
(c) 0.05 (d) 0.5 2
[EC-2004 : 2 Marks] [EC-2004 : 2 Marks]

Q.50 Choose the correct one from among the Q.53 A signal as shown in the figure is applied to a
alternatives a, b, c, d after matching an item from matched filter. Which of the following does
Group-1 with the most appropriate item in represents the output of this matched filter?
Group-2. Input
Group-1 Group-2
1
1. FM P. Slope overload
2. DM Q. µ-law 2 3
t
1
3. PSK R. Envelope detector
4. PCM S. Capture effect –1
T. Hilbert transform
Output
U. Matched filter
(a) 1-T, 2-P, 3-U, 4-S 1

(b) 1-S, 2-U, 3-P, 4-T (a) 2


t
(c) 1-S, 2-P, 3-U, 4-Q 1 3

(d) 1-U, 2-R, 3-S, 4-Q [EC-2004 : 2 Marks] –1

Q.51 Three analog signals, having bandwidths Output


1200 Hz, 600 Hz and 600 Hz are sampled at
1
their respective Nyquist rates, encoded with
12-bit words, and time division multiplexed. (b)
t
The bit rate for the multiplexed signal is 1 2 3

(a) 115.2 kbps (b) 28.8 kbps –1

(c) 57.6 kbps (d) 38.4 kbps


Output
[EC-2004 : 2 Marks]
2
Q.52 Consider the signal x(t) shown in the figure. Let
h(t) denote the impulse response of the filter (c)
matched to x(t), with h(t) being non-zero only in 1 3
t
the interval 0 to 4 sec. The slope of h(t) in the 2
interval 3 < t < 4 sec is –1
58 Electronics Engineering Communication Systems

Output Common Data for Questions (57 and 58):


Let g(t) = p(t) p(t), where ‘ ’ denotes convolution and
2
p(t) = u(t) – u(t – 1) with u(t) being the unit step function.
(d)
Q.57 The impulse response of filter matched to the
t signal, s(t) = g(t) – (t – 2) g(t) is given as,
1 2 3
(a) s(1 – t) (b) –s(1 – t)
[EC-2005 : 2 Marks] (c) –s(t) (d) s(t)
[EC-2006 : 2 Marks]
Q.54 The minimum sampling frequency
(in samples/sec) required to reconstruct the Q.58 An amplitude modulated signal is given as,
following signal from its samples without xAM(t) = 100(p(t) + 0.5g(t)) cos ct in the interval
3 2 0 t 1. One set of possible values of the
sin 2 1000t sin 2 1000t
distortion x(t ) = 5 +7 modulating signal and modulation index
t t
would be
would be
(a) t, 0.5 (b) t, 1.0
(a) 2 × 103 (b) 4 × 103
(c) t, 2.0 (d) t2, 0.5
(c) 6 × 103 (d) 8 × 103
[EC-2006 : 2 Marks]
[EC-2006 : 2 Marks]
Q.59 In delta modulation, the slope overload
Q.55 The minimum step-size required for a delta- distortion can be reduced by
modulator operating at 32 K samples/sec to
(a) decreasing the step size
track the signal (here u(t) is the unit-step
(b) decreasing the granular noise
function), x(t) = 125t (u(t) – u(t – 1) + (250 – 125t)
(c) decreasing the sampling rate
(u(t – 1) –u(t – 2)) so that slope-overloaded is
avoided, would be (d) increasing the step size

(a) 2 –10 (b) 2 –8 [EC-2007 : 2 Marks]

(c) 2 –6 (d) 2 –4
Q.60 The raised cosine pulse p(t) is used for zero S
[EC-2006 : 2 Marks]
in digital communications. The expression for
Q.56 In the following figure the minimum value of p(t) with unity roll-off factor is given by
the constant ‘C’, which is to be added to y1(t) sin 4 Wt
such that y1(t) and y2(t) are different, is p(t ) =
4 Wt (1 16 W 2 t 2 )
Quantizer Q with L 1
levels, step size y1(t) Same The value of p(t) at t = is
allowable signal
quantizer 4W
+ Q y2(t)
dynamic range [–V, V] (a) –0.5 (b) 0
x(t)
with range V V C
, (c) 0.5 (d)
2 2
[EC-2007 : 2 Marks]

(a) (b) Q.61 During transmission over a certain binary


2
communication channel, bit errors occur
2 independently with probability p. The
(c) (d)
12 L probability of at most one bit in error in a block
[EC-2006 : 2 Marks] of n-bits is given by
GATE Previous Years Solved Paper 59

(a) pn Q.64 The ratio of the average energy of


(b) 1 – pn Constellation-1 to the average energy of
Constellation-2 is
(c) np(1 – p)n – 1 + (1 – p)n
(a) 4a 2 (b) 4
(d) 1 – (1 – p)n [EC-2007 : 2 Marks]
(c) 2 (d) 8
Q.62 In a GSM system, 8 channels can co-exist in [EC-2007 : 2 Marks]
200 kHz bandwidth using TDMA. A GSM based
cellular operator is allocated 5 MHz bandwidth. Q.65 If these constellations are used for digital
Assuming a frequency reuse factor of 1/5, i.e. a communications over an AWGN channel, then
five-cell repeat pattern, the maximum number which of the following statements is true?
of simultaneous of channels that can exist in (a) Probability of symbol error for
one cell is Constellation-1 is lower.
(a) 200 (b) 40 (b) Probability of symbol error for
(c) 25 (d) 5 Constellation-1 is higher.
[EC-2007 : 2 Marks] (c) Probability of symbol error is equal for both
the constellations.
Q.63 In a direct sequence CDMA system the chip rate (d) The value of N0 will determine which of the
is 1.2288 × 10 6 chips per second. If the two constellations has a lower probability
processing gain is desired to the at least 100, of symbol error.
the data rate
[EC-2007 : 2 Marks]
(a) must be less than or equal to 12.288 × 103
bits per sec. Statement for Linked Answer Questions (66 and 67):
(b) must be greater than or equal to 12.288 × An input to a 6-level quantizer has the probability density
103 bits per sec. function f(x) as shown in the figure. Decision boundaries
(c) must be exactly equal to 12.288 × 103 bits of the quantizer are chosen so as to maximize the entropy
per sec. of the quantizer output. It is given that three consecutive
decision boundaries are ‘–1’, ‘0’ and ‘1’.
(d) can take any value less than 122.88 × 103
bits per sec. f (x)

[EC-2007 : 2 Marks] a

Common Data for Questions (64 and 65):


Two 4-ary signal constellations are shown. It is given b
that f1 and f2 constitute an orthonormal basis for the
x
two constellation. Assume that the four symbols in both –5 –1 0 1 5
the constellations are equiprobable. Let N0/2 denote
Q.66 The values of a and b are
the power spectral density of white Gaussian noise.
(a) a = 1/6 and b = 1/12
2 2
(b) a = 1/5 and b = 3/40
2a
a (c) a = 1/4 and b = 1/16
(d) a = 1/3 and b = 1/24
0
1
–a a 1 [EC-2007 : 2 Marks]
2 2a 2a

–a Q.67 Assuming that the reconstruction levels of the


2a quantizer are the mid-points of the decision
boundaries, the ratio of signal power to
Constellation-1 Constellation-2
quantization noise power is
60 Electronics Engineering Communication Systems

152 64 Q.72 The number of quantization levels required to


(a) (b) reduce the quantization noise by a factor of 4
9 3
would be
76
(c) (d) 28 (a) 1024 (b) 512
3
(c) 256 (d) 64
[EC-2007 : 2 Marks]
[EC-2008 : 2 Marks]
Q.68 Consider a Binary Symmetric Channel (BSC)
with probability of error being p. To transmit a Common Data for Questions (73 and 74):
bit, say 1, we transmit a sequence of three 1’s. The amplitude of a random signal is uniformly
The receiver will interpret the received sequence distributed between –5 V and 5 V.
to represent 1 if at least two bits are 1. The Q.73 If the signal to quantization noise ratio required
probability that the transmitted bit will be in uniformly quantizing the signal is 43.5 dB,
received in error is the step size of the quantization is
(a) p3 + 3p2 (1 – p) (b) p 3 approximately
(c) (1 – p)3 (d) p3 + p2(1 – p) (a) 0.0333 V (b) 0.05 V
[EC-2008 : 2 Marks] (c) 0.0667 V (d) 0.10 V
Q.69 Four messages band limited to W, W, 2 W and [EC-2009 : 2 Marks]
3 W respectively are to be multiplexed using Q.74 If the positive values of the signal are uniformly
Time Division Multiplexing (TDM). The quantized with a step size of 0.05 V, and the
minimum bandwidth required for transmission negative values are uniformly quantized with a
of this TDM signal is step size of 0.1 V, the resulting signal to
(a) W (b) 3 W quantization noise ratio is approximately
(c) 6 W (d) 7 W (a) 46 dB (b) 43.8 dB
[EC-2008 : 2 Marks] (c) 42 dB (d) 40 dB
Common Data for Questions (70 and 72): [EC-2009 : 2 Marks]
A speech signal, band-limited to 4 kHz and peak Q.75 Consider the pulse shape s(t) as shown. The
voltage varying between +5 V and –5 V, is sampled at impulse response h(t) of the filter matched to
the Nyquist rate. Each sample is quantized and this pulse is
represented by 8 bits.
s(t )
Q.70 If the bits 0 and 1 are transmitted using bipolar
1
pulses, the minimum bandwidth required for
distortion free transmission is
(a) 64 kHz (b) 32 kHz
t
(c) 8 kHz (d) 4 kHz 0 T
[EC-2008 : 2 Marks]
h(t)
Q.71 Assuming the signal to be uniformly distributed
between its peak-to-peak value, the signal to 1
noise ratio at the quantizer output is
(a)
(a) 16 dB (b) 32 dB
(c) 48 dB (d) 64 dB
t
[EC-2008 : 2 Marks] –T 0
GATE Previous Years Solved Paper 61

h(t )
Q
1
d
(b) r2
d r1
I I
t
0 T

h(t )

1
Q.78 For the constraint that the minimum distance
(c) between pairs of signal points be d for both
constellations, the radii r1 and r2 of the circles
t are
0 T
(a) r1 = 0.707 d, r2 = 2.782 d

h(t ) (b) r1 = 0.707 d, r2 = 1.932 d


(c) r1 = 0.707 d, r2 = 1.545 d
1
(d) r1 = 0.707 d, r2 = 1.307 d
(d) [EC-2011 : 2 Marks]

t Q.79 Assuming high SNR and that all signals are


0 T 2T equally probable, the additional average
transmitted signal energy required by the 8-PSK
[EC-2010 : 1 Mark]
signal to achieve the same error probability as
Q.76 The Nyquist sampling rate for the signal the 4-PSK signal is
sin (500 t ) sin (700 t ) (a) 11.90 dB (b) 8.73 dB
s(t ) = × is given by
t t (c) 6.79 dB (d) 5.33 dB
[EC-2011 : 2 Marks]
(a) 400 Hz (b) 600 Hz
(c) 1200 Hz (d) 1400 Hz Q.80 In a baseband communications link, frequencies
[EC-2010 : 2 Marks] upto 3500 Hz are used for signalling. Using a
raised cosine pulse with 75% excess bandwidth
Q.77 An analog signal is band-limited to 4 kHz, and for no inter-symbol interference, the
sampled at the Nyquist rate and the samples maximum possible signalling rate in symbols
are quantized into 4 levels. The quantized levels per second is
are assumed to be independent and equally
(a) 1750 (b) 2625
probable. If we transmit two quantized samples
(c) 4000 (d) 5250
per second, the information rate is
[EC-2012 : 1 Mark]
(a) 1 bit/sec (b) 2 bit/sec
(c) 3 bit/sec (d) 4 bit/sec Q.81 A Binary Symmetric Channel (BSC) has a
[EC-2011 : 1 Mark] transition probability of 1/8. If the binary
transmit symbol X is such that, P(X = 0) = 9/10,
Linked Data Questions (78 and 79): then the probability of error for an optimum
A four-phase and an eight-phase signal constellation receiver will be
are shown in the figure below.
62 Electronics Engineering Communication Systems

7 63 Common Data for Questions (85 and 86):


(a) (b)
80 80 Bits 1 and 0 are transmitted with equal probability. At
the receiver, the pdf of the respective received signals
9 1
(c) (d) for both bits are as shown below.
10 10
[EC-2012 : 2 Marks] pdf of received
1 signal for bit 0
pdf of received
Q.82 A BPSK scheme operating over an AWGN 0.5 signal for bit 1
channel with noise power spectral density of
N0/2, uses equiprobable signals,
2E –1 0 1 2 4
s1(t) = sin( c t)
T
Q.85 If the detection threshold is 1, the BER will be
2E 1 1
and s2(t) = sin( c t )
T (a) (b)
2 4
over the symbol internal (0, T). If the local 1 1
oscillator in a coherrent receiver is ahead in (c) (d)
8 16
phase by 45° with respect to the received signal, [EC-2013 : 2 Marks]
the probability of error in the resulting system is
Q.86 The optimum threshold to achieve minimum Bit
2E E Error Rate (BER) is
(a) Q (b) Q
N0 N0
1 4
(a) (b)
2 5
E E
(c) Q (d) Q 3
2 N0 4N 0 (c) 1 (d)
2
[EC-2012 : 2 Marks] [EC-2013 : 2 Marks]
Q.83 The bit rate of digital communication system is
R K-bits/s. The modulation used is 32-QAM. Q.87 Let Q( ) be the BER of a BPSK system over an
The minimum bandwidth required for ISI free AWGN channel with two-sided noise power
transmission is spectral density N0/2. The parameter is a
R R function of bit energy and noise power spectral
(a) Hz (b) kHz
10 10 density. A system with two independent and
R R identical AWGN channels with noise power
(c) Hz (d) kHz
5 5 spectral density N0/2 is shown in the figure.
[EC-2013 : 1 Mark] The BPSK demodulator receives the sum of
outputs of both the channels.
t2
Q.84 Let g(t ) = e and h(t) is a filter matched to AWGN
channel-1
g(t). If g(t) is applied as input to h(t), then the
0/1 BPSK BPSK 0/1
Fourier transform of the output is modulator demodulator

f2 f 2 /2
(a) e (b) e AWGN
channel-2

f 2 f2
(c) e (d) e If the BER of this system is Q (b ), then the
[EC-2013 : 1 Mark] value of b is ______ .
[EC-2014 : 2 Marks]
GATE Previous Years Solved Paper 63

Q.88 Coherrent orthogonal binary FSK modulation 1 1


is used to transmit two equiprobable symbol (a) (b)
2
waveforms s1(t) = cos2 f1t & s2(t) = cos2 f 2t,
2
where = 4 mV. Assume an AWGN channel (c) (d)
with two-sided noise power spectral density
[EC-2015 : 1 Mark]
N0
= 0.5 × 10 12 W/Hz. Using an optimal Q.93 The input X to Binary Symmetric Channel (BSC)
2
shown in the figure is ‘1’ with probability of
1 u2 /2 0.8. The crossover probability is 1/7. If the
receiver and the relation Q ( v ) = e du,
2 v received bit Y = 0, the conditional probability
that ‘1’ was transmitted is ______ .
the bit error probability for a data rate of 50 kbps
is X Y
6/7
0 0
(a) Q(2) (b) Q (2 2 )
P[X = 0] = 0.2
(c) Q(4) (d) Q (4 2 ) 1/7 1/7
[EC-2014 : 2 Marks]

Q.89 An analog voltage in the range 0 to 8 V is divided P[X = 1] = 0.8


in 16 equal intervals for conversion to 4-bit
1 1
digital output. The maximum quantization error 6/7
(in Volts) is _____ .
[EC-2014 : 1 Mark] [EC-2015 : 2 Marks]

Q.90 In a PCM system, the signal, Q.94 The transmitted signal in a GSM system is of
m(t) = sin(100 t) + cos(100 t) 200 kHz bandwidth and 8 users share a
is sampled at the Nyquist rate. The samples are common bandwidth using TDMA. If at a given
processed by a uniform quantizer with step size time 12 users are talking in a cell, the total
0.75 V. The minimum data rate of the PCM bandwidth of the signal received by the base
system in bits per second is ______ . station of the cell will be at least (in kHz) ____ .
[EC-2014 : 2 Marks] [EC-2015 : 2 Marks]

Q.91 An M-level PSK modulation scheme is used to Q.95 A source emits bit 0 with probability 1/3 and
transmit independent binary digits over a bit 1 with probability 2/3. The emitted bits are
bandpass channel with bandwidth 100 kHz. communicated to the receiver. The receiver
The bit rate is 200 kbps and the system decides for either 0 or 1 based on the received
characteristic is a raised-cosine spectrum with value R. It is given that the conditional density
100% excess bandwidth. The minimum value functions of R as,
of M is _______ .
1
[EC-2014 : 2 Marks] , 3 r 1
f R 0 (r ) = 4
Q.92 A sinusoidal signal of 2 kHz frequency is 0 , otherwise
applied to a delta-modulator. The sampling rate
1
and step size of the delta-modulator are , 1 r 5
f R 1 (r ) = 6
20,000 samples per second and 0.1 V,
0 , otherwise
respectively. To prevent slope overload, the
maximum amplitude of the sinusoidal signal The minimum decision error probability is
(in Volts) is
64 Electronics Engineering Communication Systems

1 Q.100 An analog pulse s(t) is transmitted over an


(a) 0 (b)
12 Additive White Gaussian Noise (AWGN)
1 1 channel. The received signal is r(t) = s(t) + n(t)
(c) (d)
9 6 where n(t) is additive white Gaussian noise with
[EC-2015 : 2 Marks] power spectral density N0/2. The received
signal is passed through a filter with impulse
Q.96 A sinusoidal signal of amplitude A is quantized
response h(t). Let Es and Eh denote the energies
by a uniform quantizer. Assume that the signal
of the pulse s(t) and the filter h(t), respectively.
utilizes all the representation levels of the
When the Signal to Noise Ratio (SNR) is
quantizer. If the signal to quantization noise ratio
maximized at the output of the filter (SNRmax),
is 31.8 dB, the number of levels in the quantizer
is ______ . which of the following holds?
[EC-2015 : 1 Mark] 2 Es
(a) Es = Eh ; SNR max =
N0
Q.97 Consider a binary, digital communication
Es
system which uses pulses g(t) and –g(t) for (b) Es = Eh ; SNR max =
transmitting bits over an AWGN channel. If the
2 N0
receiver uses a matched filter, which one of the 2 Es
(c) Es > Eh ; SNR max >
following pulses will give the minimum N0
probability of bit error? 2 Eh
(d) Es < Eh ; SNR max =
g(t) g(t ) N0
1 1 [EC-2016 : 2 Marks]
(a) (b)
Q.101 An ideal bandpass channel 500 Hz-2000 Hz is
t t deployed for communication. A modem is
1 0 1
g(t)
designed to transmit bits at the rate of 4800 bits/s
g(t)
using 16-QAM. The roll-off factor of a pulse with
1 1
(c) (d) a raised cosine spectrum that utilizes the entire
frequency band is ______ .
t t [EC-2016 : 2 Marks]
1 0 1
[EC-2015 : 2 Marks] Q.102 A speech signal is sampled at 8 kHz and
encoded into PCM format using 8 bits/sample.
Q.98 The modulation scheme commonly used for
The PCM data is transmitted through a
transmission from GSM mobile terminals is
baseband channel via 4-level PAM. The
(a) 4-QAM
minimum bandwidth (in kHz) required for
(b) 16-PSK transmission is ______ .
(c) Walsh-Hadamard orthogonal codes
[EC-2016 : 1 Mark]
(d) Gaussian Minimum Shift Keying (GMSK)
[EC-2015 : 1 Mark] Q.103 A binary baseband digital communication
system employs the signal,
Q.99 Consider a binary data transmission at a rate of
1
56 kbps using baseband binary Pulse , 0 t Ts
Amplitude Modulation (PAM) that is designed p(t ) = Ts
to have a raised-cosine spectrum. The 0, otherwise
transmission bandwidth (in kHz) required of a for transmission of bits. The graphical
roll-off factor of 0.25 is ______ . representation of the matched filter output y(t)
[EC-2016 : 1 Mark] for this signal will be
GATE Previous Years Solved Paper 65

P(f )
y(t)
1
1/Ts
(a)
(a)
f (kHz)
–1.2 0 1.2
t
0 Ts 2Ts
P(f )

1
y(t )
(b)
0.5
(b) f (kHz)
–1.2 –0.8 0 0.8 1.2

P(f )
t
0 Ts 2Ts
1
(c)
y(t )
f (kHz)
–1.2 –1 0 1 1.2
1
(c) P(f )
1

t
0 Ts 2Ts (d)
f (kHz)
–1.2 0 1.2
y(t )
[EC-2017 : 1 Mark]
1
Q.106 Which one of the following statements about
(d)
Differential Pulse Code Modulation (DPCM) is
true?
t
0 Ts/2 Ts (a) The sum of message signal sample with its
prediction is quantized.
[EC-2016 : 1 Mark] (b) The message signal sample is directly
Q.104 The bit probability of a memoryless binary quantized, and its prediction is not used.
symmetric channel is 10–5. If 105 bits are sent (c) The difference of message signal sample
over this channel, then the probability that not and a random signal is quantized.
more than one bit will be in error is ______ . (d) The difference of message signal sample
with its prediction is quantized.
[EC-2016 : 2 Marks]
[EC-2017 : 1 Mark]
Q.105 In a digital communication system, the overall
pulse shape p(t) at the receiver before the sampler Q.107 In binary Frequency Shift Keying (FSK), the
given signal waveforms are:
has the Fourier transform P(f ). If the symbols
are transmitted at the rate of 2000 symbols per u0(t) = 5 cos(20000 t); 0 t T
second, for which of the following cases is the and u1(t) = 5 cos(22000 t); 0 t T
inter symbol interference zero? where T is the bit duration interval and t is in
seconds. Both u0(t) and u1(t) are zero outside the
66 Electronics Engineering Communication Systems

interval 0 t T. With a matched filter N0/2. Binary signalling, with 0 p(t) and
(correlator) based receiver, the smallest positive 1 q(t), is used for the transmission, along with
value of T (in milliseconds) required to have u0(t) an optimal receiver that minimizes the bit error
and u1(t) uncorrelated is probability. Let 1(t), 2(t) form an orthonormal
(a) 0.25 ms (b) 0.5 ms signal set. If we choose p(t) = 1(t) and q(t) = – 1(t),
(c) 0.75 ms (d) 1.0 ms we would obtain a certain bit error
[EC-2017 : 2 Marks] probability Pb.
If we keep p(t) = but take q(t ) = E 2 (t ),
1(t),
Q.108 A sinusoidal message signal is converted to a
for what value of E would be obtain the same bit
PCM signal using a uniform quantizer. The
required signal to quantization noise ratio error probability Pb?
(SQNR) at the output of the quantizer is 40 dB. (a) 0 (b) 3
The minimum number of bits per sample needed (c) 1 (d) 2
to achieve the desired SQNR is ______ . [EC-2019 : 2 Marks]
[EC-2017 : 1 Mark]
Q.112 A voice signal m(t) is in the frequency range
Q.109 A random variable X takes values –0.5 and 0.5 5 kHz to 15 kHz. The signal is amplitude
with probabilities 1/4 and 3/4, respectively. The modulated to generate an AM signal
noisy observation of X is Y = X + Z, where Z has f(t) = A(1 + m(t)) cos2 fc(t), where fc = 600 kHz.
uniform probability density over the interval The AM signal f(t) is to be digitized and
(–1, 1). X and Z are independent. If the MAP achieved. This is done by first sampling f(t) at
1.2 times the Nyquist frequency, and then
rule based detector outputs X̂ is
quantizing each sample using a 256 level
0.5 , Y < quantizer. Finally, each quantized sample is
Xˆ =
0.5 , Y binary coded using K bits, where K is the
then the value of (accurate to two decimal minimum number of bits required for the
places) is _______ . encoding. The rate, in Megabits per second
[EC-2018 : 2 Marks] (rounded off to 2 decimal places), of the resulting
stream of coded bits is ______ Mbps.
Q.110 A binary source generates symbols X {–1, 1}
[EC-2019 : 2 Marks]
which are transmitted over a noisy channel. The
probability of transmitting X = 1 is 0.5. Input to Q.113 A random variable X takes values –1 and +1
the threshold detector is R = X + N. The with probabilities 0.2 and 0.8 respectively. It is
probability density function fN(n) of the noise N transmitted across a channel which adds noise
is shown below. N, so that the random variable at the channel
fN(n) output is Y = X + N. The noise N is independent
0.5 of X, and is uniformly distributed over the
interval [–2, 2]. The receiver makes a decision
n 1 , if Y
–2 2 Xˆ =
If the detection threshold is zero, then the +1 , if Y <
probability of error (correct to two decimal where the threshold [–1, 1] is chosen so as to
places) is _______ .
minimize the probability of error Pr[ Xˆ X ].
[EC-2018 : 1 Mark]
The minimum probability of error (rounded off
Q.111 A single bit, equally likely to be 0 and 1, is to be to 1 decimal place) is ______ .
sent across an additive white Gaussian noise
[EC-2019 : 2 Marks]
(AWGN) channel with power spectral density
GATE Previous Years Solved Paper 67

Q.114 In a digital communication system, a symbol S Q.116 Consider a polar non-return to zero (NRZ)
randomly chosen from the set (s1, s2, s3, s4) is waveform, using +2 V and –2 V for representing
transmitted. It is given that, s1 = –3, s2 = –1, s3 = +1 binary ‘1’ and ‘0’ respectively, is transmitted in
and s4 = +2. The received symbol is Y = S + W. W the presence of additive zero mean white
variable and is independent of S. Pi is the Gaussian noise with variance 0.4 V2. If the a
conditional probability of symbol error for the priori probability of transmission of a binary
maximum likelihood (ML) decoding when the ‘1’ is 0.4, the optimum threshold voltage for a
transmitted symbol S = si. The index ‘i’ for which maximum a posteriori (MAP) receiver (Rounded
the conditional symbol error probability Pi is off to two decimal places) is ______ V.
the highest is ______ . [EC-2021 : 2 Marks]
[EC-2020 : 2 Marks]
Q.117 A 4 kHz sinusoidal message signal having
Q.115 A speech signal, band limited to 4 kHz is amplitude 4 V is fed to a delta-modulator (DM)
sampled at 1.25 times the Nyquist rate. The operating at a sampling rate of 32 kHz. The
speech samples, assumed to be statistically minimum step size required to avoid slope
independent and uniformly distributed in the overload noise in the DM (Rounded off to two
range –5 V to +5 V, are subsequently quantized decimal places) is ______ V.
in an 8-bit uniform quantizer and then [EC-2021 : 1 Mark]
transmitted over a voice grade AWGN telephone
Q.118 The refractive indices of the core and cladding
channel. If the ratio of transmitted signal power
of an optical fiber are 1.50 and 1.48, respectively.
to channel noise power is 26 dB, the minimum
The critical propagation angle, which is defined
channel bandwidth required to ensure reliable
as the maximum angle that the light beam makes
transmission of the signal with arbitrarily small
with the axis of the optical fiber to achieve the
probability of transmission error (Rounded off
total internal reflection, (Rounded off to two
to two decimal places) is ______ kHz.
decimal places) is ______ degree.
[EC-2021 : 1 Mark]
[EC-2021 : 1 Mark]
68 Electronics Engineering Communication Systems

Answers Digital Communication Systems

1. (c) 2. (c) 3. (c) 4. (b) 5. (a) 6. (b) 7. (b) 8. (3600)

9. (Sol.) 10. (3600) 11. (c) 12. (a) 13. (a) 14. (Sol.) 15. (a) 16. (3)
17. (b) 18. (c) 19. (c) 20. (c) 21. (c) 22. (d) 23. (c) 24. (d)
25. (c) 26. (a) 27. (d) 28. (a) 29. (c) 30. (b) 31. (d) 32. (a)
33. (b) 34. (c) 35. (c) 36. (d) 37. (c) 38. (a) 39. (d) 40. (b)
41. (c) 42. (d) 43. (b) 44. (b) 45. (b) 46. (c) 47. (a) 48. (c)
49. (a) 50. (c) 51. (c) 52. (b) 53. (c) 54. (c) 55. (b) 56. (b)
57. (c) 58. (a) 59. (d) 60. (c) 61. (c) 62. (b) 63. (a) 64. (b)
65. (a) 66. (a) 67. (d) 68. (a) 69. (d) 70. (a) 71. (c) 72. (b)
73. (c) 74. (c) 75. (c) 76. (c) 77. (d) 78. (d) 79. (d) 80. (c)
81. (d) 82. (b) 83. (d) 84. (d) 85. (d) 86. (b) 87. (1.414) 88. (c)
89. (0.25) 90. (200) 91. (17) 92. (a) 93. (0.4) 94. (400) 95. (d) 96. (32)
97. (a) 98. (d) 99. (35) 100. (a) 101. (0.25) 102. (16) 103. (c)
104. (0.7357) 105. (b) 106. (d) 107. (b) 108. (7) 109. (–0.50) 110. (0.125)
111. (b) 112. (11.81) 113. (0.10) 114. (3) 115. (9.26) 116. (0.04) 117. (3.14)
118. (9.39)

Solutions
Digital Communication Systems
1. (c) Area under pdf is unity,
Companding results in making SNR uniform, K[V – (–V)] = 1
throughout the signal, irrespective of amplitude 2VK = 1
levels. Since, in uniform quantization, step size 1
K=
is same, the quantization noise power is 2V
uniform, throughout the signal. 1
Thus, higher amplitudes of signal will have So, X(x) = –V to V
2V
better SNR than the lower amplitudes. =0 otherwise
Hence, companding is used for improving SNR
at lower amplitudes. x2
Signal power = s = x ( x ) dx

2. (c)
v V
Since the signal is uniformly distributed in the 2 1 1 x3
s= x dx =
interval –V to +V. So, pdf is 2V 2V 3
v V
x (x )
1 V3 ( V )3
K s=
2V 3 3
3
x 1 2V V2
–V 0 V s= =
2V 3 3
GATE Previous Years Solved Paper 69

In uniform quantization, P(x = at most 1) = P(x = 0) + P(x = 1)


2 = 8C0(P)0 (1 – P)8 – 0 + 8C1(P)1 (1 – P)8 – 1
Quantization noise power = QNP = = (1 – P)8 + 8P(1 – P)7
12
Vp p Vp p 5. (a)
where, step size = = = n
L 2 A coherent carrier is not required to be generated
Vp2 at the receiver.
p
QNP = 2n
12 × 2 6. (b)

(2 V )2 V2 f c = 4 GHz = 4000 MHz


= = fm = 2 MHz
12 × 2 2 n 3 × 22n
fH = fc + fm
S
SQNR = = 4000 + 2 = 4002 MHz
QNP
fL = fc – fm
V 2 3 × 2 2n = 4000 – 2 = 3998 MHz
= × = 22n
3 V2 2 fH
fs =
So, SQNR 22n K
So, signal to quantizing noise ratio is determined
fH 4002 MHz
by the number of quantizing levels. K= =
fH fL 4 MHz
3. (c) = 1000.5 = 1000
2 × 4002 MHz
bk dk PSK fs =
1000
modulation
= 8.004 MHz 8 MHz

EX-NOR 7. (b)
A B Y In BPSK if detector has a fixed phase error then
0 0 1
the output power would change by a factor
0 1 0 cos2 .
1 0 0
8. Sol.
Y = AB + AB 1 1 1
3600 samples/sec
0 is represented with a carrier has phase ‘ ’ Given that, fH = 1800 Hz,
1 is represented with a carrier has phase ‘0’ fL = 300 Hz
dk = bX dk – 1 or dk = bk dk – 1 So, B.W. = fH – fL
Logic function here is EX-NOR. = 1800 – 300
1 1 0 0 1 1 = 1500 Hz
fH 1800
K= =
1 0 1 0 0 0 B.W. 1500
0 So, K=1
Hence, the answer is 0, , , .
2 fH
(fs )min =
4. (b) K
Let getting an error be success. 2 × 1800
(fs )min =
P (success) = P 1
P (failure) = 1 – P = 3600 samples/sec
70 Electronics Engineering Communication Systems

9. Sol. 14. Sol.


For different phases are given which are 90° x(t)
apart with adjacent signal. So, it is the clear of
QPSK (Quadrature Phase Shift Keying). A

10. Sol. t
0 T/3 2T/3 T
fs = 3600 samples/sec
fH = 1800 Hz We know that impulse-response of a matched
fL = 300 Hz filter is,
B.W. = fH – fL h(t) = x(T – t)
= 1800 – 300 = 1500 Hz h(t)

fH 1800
K= = =1 A
B.W. 1500
2 f H 2 × 1800 t
fs(min) = = 0 T/3 2T/3 T
K 1
= 3600 samples/sec The output of matched filter,
y(t) = x(t) h(t)
11. (c)
dh T
(t ) = A t A (t T )
dt 3
t
Input MOD T
y(t) = x (t ) * A t x(t ) A (t T ) dt
3
Delay
t
T
EX-NOR y(t) = A x t x (t T ) dt
3
A B Y
0 0 0 t

0 1 1 Y=A B = y1 (t ) dt

1 0 1 Y = AB + AB
t
1 1 0 y(t) = y1 (t ) dt
Given that, 0 1 0 0 1
y1(t )
Reference bit = 1
logic 0 A
2
1 1 0 0 0 1
logic 1 0° 0 0
4T/3 5T/3
t
12. (a) 0 T/3 2T/3 T 2T
Coherrent demodulation of FSK signal can be 2
–A
detected using correlation receiver.

13. (a)
y(t)
The purpose of source encoding in a data 2 A2 T
3
communication system is to increase the
information transmission rate and purpose of
channel encoding is to decrease the probability t
0 T/3 T 5T/3
of error. The channel coding helps in detection
and correction of errors.
GATE Previous Years Solved Paper 71

15. (a) (SQNR)dB = 1.76 + 6n


Increased pulse width in the flat-top sampling 3 2n
SQNR = 2
leads to greater attenuation of high frequencies 2
in reproduction. This effect is known as aperture From the above equation it is clear that SQNR
effect. increases with increase in value of ‘n’.

16. Sol. 21. (c)


(B.W.)PCM = nfs Given that,
n = log2 L
n1 = log24 = 2 x(t) x(t)

n2 = log264 = 6 1
(B.W.)1 = n1 f s = 2fs
(B.W.)2 = n2 f s = 6fs t
0 T
(B.W.)2 6 fs
= = 3 times
(B.W.)1 2 fs For the matched filter,
(B.W.)2 = 3 (B.W.)1 h(t) = x(T – t)

17. (b) h ( t ) = x (T – t )

(SQNR)dB = (1.76 + 6 n)dB


1
(SQNR)1 = 1.76 + 6n
(SQNR)2 = 1.76 + 6(n + 1)
= 1.76 + 6n + 6 0
t
T
(SQNR)2 – (SQNR)1
= (1.76 + 6n + 6) – (1.76 + 6n) y(t) = x(t) h(t)
(SQNR)2 – (SQNR)1 = 6 dB
y(t)
So, for every one bit increase in bits per sample
will result in 6 dB improvement in signal to
quantization noise ratio.

18. (c) t
0 T 2T
Given that, fm = 1 kHz
f s = 1.8 K-samples/sec y(t) is a triangular pulse.
The frequency components in the sampled
22. (d)
signal are
nfs ± fm 3 2n
SQNR = 2
n = 0 fm = 1 kHz = 1000 Hz 2
n = 1 fm = 1.8 ± 1 = 800 Hz and 2800 Hz Given that, n1 = n
n = 2 3.6 ± 1 = 2600 Hz and 4600 Hz n2 = n + 1
cut-off frequency of LPF 1100 Hz. 3 2n
(SQNR)1 = 2
So, 800 Hz and 100 Hz components. 2
3 2( n + 1) 3 2 n + 2
19. (c) (SQNR)2 = 2 = 2
2 2
The signal to quantization noise ratio in an n-bit
3 2n 2
PCM system is given by = [2 2 ]
2
72 Electronics Engineering Communication Systems

(SQNR)2 22 28. (a)


So, = =4 Compression in PCM refers to relative
(SQNR)1 1
(SQNR)2 = 4(SQNR)1 compression of higher signal amplitudes.
Signal to quantization noise ratio increases by 29. (c)
a factor of 4. So, this improvement in SQNR is
sinc3(400t) fm1 = 600 Hz
independent of ‘n’.
sinc3(100t) fm2 = 150 Hz
23. (c) sinc3(400t) sinc3(100t)
Alternate Mark Inversion (AMI) code has zero fm = fm1 + fm2 = 750 Hz
dc component for pulse transmission of random fs(min) = 2fm = 1500 Hz
binary data. 30. (b)
24. (d) SNR = 1.76 + 6 × 7 = 1.76 + 42 = 43.8
fs(min) = 2f(max) As input signal is 0.5 cos mt, 128 levels are
needed to quantize them.
= 2 × 2 kHz = 4 kHz

25. (c) 1

QNP of power signal is given by, 0.5


2
128
QNP = 256 levels
12 levels
Vp –0.5
p
Step size = =
2n –1

Vp2 p 1
QNP = 2n
; QNP 31. (d)
12 × 2 2 2n
Maximum amplitude of matched filter output
(QNP )2 2 2 n1 2 2 × 8 2 16 1 1
= 2 n = 2 × 9 = 18 = 2 = is
(QNP )1 2 2 2 2 2 4
A2T 10 2 4
(QNP)1 = × 10 = 5 mV
(QNP)2 = 2 2
4
So, the QNP reduces by a factor of 4. 32. (a)
Sample rate = 2(100 + 100 + 200 + 400) Hz
26. (a)
= 1600 Hz
Flat top sampling of low pass signals gives rise
to aperture effect. 33. (b)
f 1 = 25 kHz and f0 = 10 kHz
27. (d)
f1 – f0 = 15 kHz
Quadrature carrier multiplexing utilizes carrier
For orthogonality,
phase shifting and synchronous detection to
n
permit two DSB signals to occupy the same f1 – f0 =
2 Tb
frequency band. It is the scheme where same
n
carrier frequency is used for two different DSB So, = 15 kbps
2 Tb
signals. It is also known as quadrature
amplitude modulation (QAM). So, quadrature n 100 n
Tb = ms = µs
multiplexing is quite different from FDM and 30 3
TDM. Among the given options, only 200 ms is the
integer multiple of (100/3) µs.
GATE Previous Years Solved Paper 73

34. (c) 39. (d)


Frames per second = 625 50 µs 20 kHz
Pixels per frame = 400 × 400 fm = 12 kHz
64 intensity levels per pixel can be represented f s = 20 K-samples/sec
by 6 bits per pixels. Frequency at filter output with cut-off
Date rate = 625 × 400 × 400 × 6 = 600 Mbps frequency of 15 K is 20 ± 12 = 8 K and fm = 12 kHz

35. (c) 40. (b)

sinc(700t) + sinc(500t) 2EBPSK


Pe (BPSK) = Q
Nyquist sampling rate = 2fm = 700 Hz N0
1
Sampling rate = sec 2EBFSK
700 Pe (BFSK) = Q
N0
36. (d)
To get same probability of error in both the
Probability of error = p schemes,
Probability of no error = (1 – p) 2EBPSK = EBFSK
For error in at most one bit there should either
EBFSK
be no error or error in only one bit = 2 (or) 3 dB
EBPSK
= nC0(1 – p)n p0 + nC1(1 – p)n – 1 p1
So, at a given probability of error, coherrent
= (1 – p)n + np(1 – p)n – 1
BFSK is inferior to coherrent BPSK by 3 dB.
37. (c)
41. (c)
6
5 × 10 2 Mp
Y(f) = X( f nf s ) 1.536
100 × 10 6 = =
n= L 128
1
fs = = 10 kHz 1.536 2
Ts 2
128
x(t) is a sinusoidal signal of 4 kHz. PN = = = 12 × 10 6 V 2
12 12
Since, fs > 8 kHz and cut-off frequency fc of the
LPF satisfies 4 kHz < (fc = 5 kHz) < 6 kHz, the 42. (d)
output signal of the filter will be 2Eb 2 × 10 5
1 10 SNR = = = 20
z(f ) = x (t ) = cos(8 × 10 3 t ) N0 10 6
20 20 (SNR)dB = 10 log20 = 13 dB
= 5 × 10–1 cos(8 × 103t)
43. (b)
38. (a)
For slope overload to take place,
For coherrent BFSK system,
n Am
fH – fL = = n × 4 kHz Ts m
2Tb
For best possible set of transmitted frequencies, = 0.628 × 40 × 103
fH – fL should be as minimum as possible. i.e., n Ts
can be taken as 1. = 25120 = 25.12 K
fH – fL = 4 kHz Matching with options 2 × 1.5 × 4 K = 37.7
From option (a), which is greater than /Ts.
fH – fL = 20 kHz – 16 kHz = 4 kHz
74 Electronics Engineering Communication Systems

44. (b) 0.2 0


Pe(1) = = 0.2
1 1 0
Bandwidth efficiency =
Tb × B.W. Since probability of occurrence of 0 and 1 are
1 equal,
For BPSK, B.W. = Pe (0) + Pe (1)
Tb Average = = 0.15
(with Nyquist pulse shaping of baseband data)
2
So, Bandwidth efficiency = 1 51. (c)

45. (b) f s = 2(1200 + 600 + 600) = 4.8 k


Rb = nfs = 12 × 4.8 k
Bit rate = nfs = 8 × 8 k = 64 kbps
= 57.6 kbps
SNR = 1.76 + 6n
= 1.76 + 48 = 49.8 52. (b)

46. (c) h(t) = x(4 – t)


x(–t)
For every 1-bit SNRq improves by a factor of 4.
Thus for 2-bits increase in message the SNRq
increases by a factor of 16.
–2
47. (a) –4 –3 –1 0 –t

In between the two adjacent sampled values, if


the base band signal changes an amount less x(–t + 4)
than the step size, the output of the DM is a
–t
sequence of alternate positive and negative
pulses. The small change in base band signal 2
indicated that the base band is almost constant. 0 1 3 4 –t + 4

48. (c)
Slope in region t = 3 to 4 = –1.
Rb = 10 k
B1 = 2Rb = 20 k 53. (c)
B2 = Rb = 10 k
Input x(t )
49. (a) 1
2 3
t
1
0.25 –1
Threshold
0.2 1
x(t)

0
T–t
2 1
0.2

–0.25 0
h(t)
For 0 For 1

Shaded areas show the probability of error. 1

Error 0.25 2 t
Pe(0) = = = 0.1 1 2
Total 0.25 ( 0.25) –1
GATE Previous Years Solved Paper 75

y (t ) g(t )
2
2A T 1
(2)
2
1 3 A T=2×1=2
2
2
–A T t
0 1 2
(–1)
s(t) = g(t) – (t – 2) g(t)
54. (c) = g(t) – g(t – 2)
f s = (2fm) × 3
g(t – 2)
= (2 × 1000) × 3
= 6 × 103 Hz 1

55. (b)
To avoid slope overload, t
0 2 3 4

m (t) s(t) = g(t) – g(t – 2)


Ts
s (t )
32 × 1024 125
215 = 125 1
7
2
2–8
2 15 3 4
t
0 1 2
56. (b)
–1

Impulse response of match filter,


/2
h(t) = s(T – t)
h(t )
/2
1

1
t
0 2 3 4
For y1(t) and y2(t) to be different minimum step
size of /2 is needed else they will be same. –1

57. (c) h(t) = –s(t)


p(t) = u(t) – u(t – 1)
58. (a)
p(t ) XAM(t) = 100[p(t) + 0.5g(t)] cos ct ...(i)
p(t )
1

1
t
0 1
t
g(t) = p(t) p(t) 0 1
76 Electronics Engineering Communication Systems

g(t ) 62. (b)

1
Allocated bandwidth = 5 MHz
1
Frequency reuse factor =
5
t 1
0 1 2 Bandwidth allocated for 1 cell = 5 ×
= 1 MHz
5
s(t) = 100 (1 + m(t)) cos ct
To allocate 8 channels 200 kHz bandwidth
p(t) = 1
required.
m(t) = 0.5g(t)
Number of channels in 1 MHz
Comparing AM equation with (i) = 0.5t
1 MHz
Modulating signal = t(from equation of line) = × 8 = 40
200 kHz
Modulation index = 0.5

59. (d) 63. (a)

Condition to avoid slope overload in DM, Rc


Processing gain =
d Rb
m(t )
Ts dt Rc
100
So, by increasing the step size, slope overload Rb
distortion can be avoided. Rc
Rb
100
60. (c)
Rb 12.288 × 103 bits per sec.
sin 4 Wt
p(t) = 64. (b)
4 Wt (1 16 W 2 t 2 )
Average energy of constellation 1 is,
1
sin 4 W ×
1 4W 0 + 4 a 2 + 4 a2 + 8a 2
p = E1 = = 4 a2
4W 1 1 4
4 W× 1 16 W 2 Average energy of constellation 2 is,
4W 16 W 2
a 2 + a2 + a2 + a2
As it comes in the form of 0/0 so, applying LH E2 = = a2
rule, 4

d E1 4 a2
sin 4 Wt So, = 2 =4
1 dt E2 a
p = d
4W 4 Wt (1 16 W 2 t 2 )
dt 65. (a)

cos 4 Wt The probability of error decreases with increase


= in average energy. As constellation 1 has more
1 48 W 2 t 2
average energy than that of constellation 2. So,
1
Putting, t= the probability of symbol error for constellation
4W
1 is lower.
cos
P(t) = = 0.5 66. (a)
1 3
To maximize the entropy, all the decision
61. (c)
boundaries should be equiprobable,
Probability of no error in n-bits = (1 – p)n 5
1
Probability of one error in n-bits = np(1 – p)n – 1 x ( x ) dx =
3
Total probability = np(1 – p)n – 1 + (1 – p)n 1
GATE Previous Years Solved Paper 77

5 2
b dx = 1 P( 2) = 1 P ( 1 ) =
3
1 3
1 1 12 2 22 1
b [ x ]1 =
5 So, NQ = + =
3 3 12 3 12 4
1 S
b[5 – 1] =
3 Hence, SQNR = N = 28
Q
1
4b =
3 68. (a)
1 There will be error if all the three received bits
b=
12 are 0 ro 2 of the three received bits are 0.
1
1 Therefore, probability of error in output
x ( x ) dx = = 3C3 p p p + 3C2 p p(1 – p)
3
1
= p3 + 3p2 (1 + p)
1
1
a (x) = 69. (d)
3
1
f s1 = 2 × W = 2 W
1 1
a [x ] 1 =
3 f s2 = 2 × W = 2 W
1 f s3 = 2 × 2 W = 4 W
a[1 – (–1)] =
3
1 1 f s4 = 2 × 3 W = 2 W
2a = a=
3 6
f s = fs1 + f s2 + f s3 + f s4
67. (d) For minimum bandwidth, n = 1
Signal power, Rb = nfs
Rb = 1 × 14 W = 14 W
S = E[ X 2 ] = x 2 f X ( x ) dx Rb
(B.W.)min =
2
1 5
14 W
= 2 ax 2 dx + bx 2 dx (B.W.)min = =7W
2
0 1
1 5 70. (a)
x3 x3
= 2 a +b Binary sequence represented by bipolar pulses.
3 3
0 1
So, B.W. = Rb = nfs
a 124b 1 124 = 8 × 8 K = 64 kHz
= 2 + =2 + =7
3 3 6 × 3 12 × 3
71. (c)
Quantization noise power,
Signal to noise ratio,
2 2
i
NQ = P( i) S0
i=1 12 6n dB
N0 dB
1 = 1 V; for –1 < x < 1
where, n = number of bits per sample quantized
2 = 2 V; for 1 < x < 5
S0
6 × 8 = 48 dB
1 N 0 dB
P( 1) = P{–1 < x < 1} = 2 a =
3
78 Electronics Engineering Communication Systems

72. (b) 2 2
1 2
Quantization noise, NQ = P( 1)+ P( 2)
12 12
S2 1 2
( 1 + 22 )
Nq = =
12 24
where, S = step size of quantization 125 1
= =
Nq 24 × 10000 1920
Nq = Signal power,
4
25
S2 S2 S=
= 3
12 12 × 4
S S 1920 × 25
S = SQNR = = = 16000
2 NQ 3
Vpp In decilogs,
S= n [SQNR] = 10 log10 (SQNR)
2
where, Vpp = peak-to-peak value = 42 dB
n = number of bits/sample 75. (c)
Vpp Vpp Vpp
= = s(t)
2n 2 2n 2 28
1
2n = 29 = 512
Therefore, number of quantization levels
required to reduce the quantization noise by a t
0 T
factor 4 would be 512.
Impulse response of the matched filter,
73. (c)
h(t) = s(T – t)
Signal power,
s(T + t) is the left-side shifted version of s(t) by T.
5
1 2 25
S= x 2 f X ( x ) dx = x dx = s(T + t)
10 3
5
1
SQNR = 43.5 dB (or) 10(4.35)
2
S
NQ = = t
12 SQNR –T 0

12 (S ) 12 × 25 s(T – t) is the mirror image of s(T + t) on y-axis.


So, = =
SQNR 3(10 4.35 )
h(t) = s(T – t)
= 0.0668 V
1
74. (c)
= 0.05 V (for positive values)
1
t
2 = 0.1 V (for negative values) 0 T
The amplitude of the input signal is
symmetrically distributed about zero.
76. (c)
Hence, P( 1) = P( 2) = 0.50
So, the quantization noise power will be 1 2 sin(500 t ) sin(700 t )
s(t) = 2 2
2 t
GATE Previous Years Solved Paper 79

1 79. (d)
s(t) = 2 2
[cos (700 t 500 t ]
2 t Pe Es for both cases
–cos(700 t + 500 t)]
Es1 r 0.707 d
1 So, = 1 =
s(t) = 2 2
[cos(200 t ) cos(1200 t )] Es2 r2 1.307 d
2 t
Maximum frequency component, Es2 2
1.307
1200 = = 3.42
= 600 Hz
fm = Es1 0.707
2
To achieve same error, 2nd must have 3.42 time
Nyquist sampling rate,
than 1st.
fsmin = 2fm = 1200 Hz
The value in dB = 10 log(3.42)
77. (d) = 5.33 dB
Quantized levels are equiprobable.
80. (c)
Hence, H = log2 4 = 2 bits/sample
Rb
r = 2 samples/sec B.W. = (1 + )
2
Hence, information rate
Rb
R = r H = 2 samples/sec × 2 bits/sample 3500 = (1 + 0.75)
2
R = 4 bits/sec
Rb = 4000 bits/sec
78. (d) In base band transmission,
Symbol rate = Bit rate = 4000 symbols/sec
For M-ary: d = 2 sin Es
M 81. (d)
Distance of any point from origin is Es . 7/8
P(x0) = 9/10 x0 y0
For 4-ary, r1 = Es1
1/8
For 8-ary, r2 = Es2 1/8

For 4-ary, M = 4: P(x1) = 1/10 x1 y1


7/8
d 1 = 2 sin r1
4 7 9 63
P(y0|x0) P(x0) = × =
For 8-ary, M = 8: 8 10 80
1 1 1
d 2 = 2 sin r2 P(y0|x1) P(x1) = × =
8 8 10 80
If d1 = d2 = d then, 1 9 9
P(y1|x0) P(x0) = × =
8 10 80
2 sin r1 = d
4 7 1 7
P(y0|x1) P(x1) =× =
d 8 10 80
r1 = = 0.707 d
2 P(y0|x0) P(x0) > P(y0|x1) P(x1)
So, when a symbol is received as y0, the decision
2 sin r2 = d can be made in favour of x0 in an optimum way.
8
P(y1|x0) P(x0) > P(y1|x1) P(x1)
d
r2 = = 1.307 d So, when a symbol is received as y1, the decision
2 sin can be made in favour of x0 in an optimum way.
8
80 Electronics Engineering Communication Systems

So, for the given BSC, with optimum receiver, 1


Y =0 z 1 1 1
both the received symbols will be decoded as P = f dz = × × 1 =
X=1 1 4 4 8
x0. Hence, the probability of error is equal to the 0

probability of transmitting x1. 1 1 1 1


Pe = ×0 + × =
1 2 2 8 16
So, Pe = P( x1 ) =
10
86. (b)
83. (d) Optimum threshold is given by the point of
Nyquist pulses should be used for baseband intersection of two pdf curves,
pulse shaping to avoid S , with minimum z
f = 1 z; z 1
bandwidth requirement. In this case the 0
bandwidth of the base band signal will be z z
f = ; 0<z<2
R 1 4
(B.W.)baseband = kHz
2 The point of intersection which decides
2 (B.W.)baseband optimum threshold,
(B.W.)M-QAM =
log 2 M 1
1–z =
For 32-QAM, 4
2 (B.W.)baseband R z
(B.W.)32-QAM = = kHz 1 = z+
log 2 (32) 5 4
4
z=
84. (d) 5
t2 87. Sol.
g(t) = e
f2 Since the same signal is passed through two
G(f ) = e
2 parallel channels and added at the receiver
f
H(f ) = e input, the effective distance between the adjacent
Y(f ) = G(f ) H(f )
message points will be dmin = 4 Eb and the
t2 2 2
Y(f ) = e e f =e 2 f
effective points PSD (on-sided) will be
85. (d) N0(eff) = 2N0.
BER is given as, 2
dmin 16 Eb
1 0 So, BER = Q =Q
Pe = P(0) P + P(1) P 2 N 0(eff) 4N 0
0 1
If detection threshold = 1 then, Eb 2 Eb
= Q 4 =Q 2
1 N0 N0
P(0) = P(1) =
2 For single channel case,
Y =1 z 2Eb
P = f dz = 0 )=Q
X=0 0 BER = Q (
1 N0
So, for the given problem,
0.5
1 f(z/1)
f(z/0)
2Eb
BER = Q 2 = Q( 2 )
1/4 N0

z = Q (b )
–1 0 4/5 1 2 4
So, b= 2 = 1.414
GATE Previous Years Solved Paper 81

88. (c) 92. (a)


Slope signal = Slope of delta modulator
Ac2 Tb
For FSK Bit error probability = Q Am (2 fm) = f s
2N0
Am(2 2 × 103) = 20,000 × 0.1
Ac = 4 mV 1
Am =
1 1 2
Tb = =
Rb 500 × 103
93. Sol.
N0
= 0.5 × 10–12 W/Hz P(X = 1/Y = 0)
2
P(Y = 0 / X = 1) P( X = 1)
N0 = 10–12 W/Hz = P(Y = 0 / X = 1) P(X = 1) + P(Y = 0 / X = 0) P( X = 0)
Bit error probability = Q(4)
1
89. Sol. (0.8)
7
= = 0.4
The maximum quantization error is, 1 6
(0.8) + (0.2)
7 7
Qe =
2
Dynamic range 94. Sol.
where, = In the question it is given that GSM requires
2 n or L
200 kHz for 8 users and uses TDMA scheme to
8 8
= 4
= = 0.5 accommodate them. Thus for the next users we
2 16
will required an extra of 200 kHz bandwidth.
0.5 Thus, 400 kHz bandwidth is to be used.
Qe = = 0.25
2
95. (d)
90. Sol.
According to MAP criteria, for optimum
m(t) = sin(100 t) + cos(100 t)
threshold,
Maximum peak of m(t) is 2, H0
Vmax Vmin f R (r |s0 ) P(s0 ) > f R (r |s1 ) P(s1 )
= <
n H1
2
2 2 Given that,
0.75 = n
2 1 2
P(s0) = and P(s1 ) =
3 3
2 2
2n =
0.75
n fR (r |s1) P (s1 )
2 = 3.8 n = 2
1/9
Rb = nfs = 2 × 100 fR(r|s0) P(s0)
= 200 bits/sec 1/12

91. Sol. r
–3 –1 0 1 5
Rb
B= (1 + )
log 2 M From the above figure, it is clear that,
200 × 2 f R (r |s0 ) P(s0 ) > f R (r |s1 ) P(s1 ) for r < –1
100 =
log 2 M
log2 M = 4 f R (r |s1 ) P(s1 ) > f R (r |s0 ) P(s0 ) for r > –1
So, M = 16 So, optimum threshold will be rTh = –1
82 Electronics Engineering Communication Systems

For rTh = –1, But shifting does not change the energy
P(r0|s1) P(s1) = 0 Eh = Es
1 1 2Es
P(r1|s0) P(s0) = [1 ( 1)] = and (SNR)max =
12 6 N0
Pe(min) = P(r0|s1) P(s1) + P(r1|s0) P(s0)
101. Sol.
1
= Channel spectrum 500 Hz – 2000 Hz
6
Hence, B.W. = 1500 Hz
96. Sol. Rb
B.W. = (1 + )
SNR of uniform quantized sinusoidal signal is log 2 M
given by
Rb
SNR = 1.8 + 6n 1500 = (1 + )
log 2 16
31.8 = 1.8 + 6n
n=5 4800
1500 = (1 + )
Thus total number of levels = 25 = 32. log 2 16
1500 = 1200 (1 + )
97. (a)
= 0.25
E
Since, SNR = 102. Sol.
2
Thus for having minimum error we have to f s = 8 kHz
select signal with maximum energy. n = 8 bits/sample
Energy of signal 1 = 1 M=4
Rb R
1 (B.W.)min = = b
Q Energy of signal 2 = 2 log 2 M 4
3
1 8× 8
= = 16 kHz
Energy of signal 3 = 4
3
1 103. (c)
Energy of signal 4 =
3 P(t )

98. (d) 1 / Ts
(GMSK) is used to transmit GSM signals.

99. Sol.
Bit rate, Rb = 56 kbps t
0 Ts
Roll-off factor, = 0.25
Impulse response of matched filter,
Rb
Transmission B.W. = (1 + ) h(t) = P(Ts – t)
2
P(Ts – t ) = h(t)
56
= (1.25) = 28 × 1.25
2 1 / Ts
= 35 kHz

100. (a)
When the signal to noise ratio is maximum, t
0 Ts
h(t) = s(T – t)
GATE Previous Years Solved Paper 83

Output of matched filter, For FSK waveforms to be uncorrelated,


y(t) = P(t) h(t) R
f1 – f2 = n b ; n = 1, 2, 3...
y(t) 2
2 ( f 1 f 2 ) 2000
1 Rb = = bits/sec
n n
Rb(max) = 2000 bits/sec
t (Q minimum value of n = 1)
0 Ts 2Ts
1
Tb(min) = = 0.5 ms
Rb(max)
104. Sol.
Bit error probability, 108. Sol.
p = 10 –5
For sinusoidal input is applied to a PCM
Number of bits transmitted,
system,
n = 10 5
SQNR = 6n + 1.8
If n-bits are transmitted, probability of getting
n = Number of bits per sample
error in ‘r’ bits is given by
Given that, the required SQNR = 40 dB
p e = nCr p r(1 – p)n – r
So, 6n + 1.8 40
The probability that not more than one bit will
38.2
be in error is n
6
= nC0 p0(1 – p)n – 0 + nC1(p)1 (1 – p)n – 1
nmin = 7
= (1 – p)n + np(1 – p)n – 1
(Q n is always an integer value)
= (1 – 10–5)105 + 105 × 10–5(1 – 10–5)105 – 1
= 0.7357 109. Sol.

105. (b) f z(z )


Given:
1/2
Symbol rate = 2000 symbol/sec
So, Rb = 2 kHz
z
Nyquist criterion for S free transmission is, –1 0 1

p( f nRb ) = Constant 1 3
P(x0) = ; P( x1 ) =
n= 4 4
x0
So, only option (b) satisfies this condition for
MAP criteria, fY ( y|x0 ) P( x0 ) > fY ( y|x1 ) P( x1 )
given Rb. <
x1

106. (d) fy
1 3 3
In DPCM, difference of message signal sample 1 1 1
f y ( y| x1 ) ( Px1 ) = × =
2 4 8
f y ( y |x0 ) ( Px0 ) = × =
with its prediction is quantized. 2 4 8

107. (b)
y
u0(t) = 5 cos 20000 t –1.5 –1 –0.5 0 0.5 1 1.5

u1(t) = 5 cos 22000 t Optimum threshold


f1 = 11000 Hz exists here

and f2 = 10000 Hz So, = –0.50


84 Electronics Engineering Communication Systems

110. Sol. 112. Sol.


Let s 0 and s 1 be the transmitted symbols M (f )
representing the transmitted value {–1, 1}
respectively and let r0 and r1 be the received
symbols.
f R( r ) f (kHz)
–15 –5 0 5 15
slope = 1/4
when s0 when s1
transmitted 0.5 transmitted F(f )
(One sided spectrum)

r
–3 –2 –1 0 1 2 3

P(r0|s1) P(r 1|s 0) f (kHz)


0 585 595 600 605 615
Probability of error, B = 30 kHz
Pe = P(s1) P(r0|s1) + P(s0) P(r1|s0) Nyquist rate = 2 × 615 kHz
1 1 1 = 1230 kHz = 1.23 MHz
P(r0|s1) = P(r1|s0) = × 1 × =
2 4 8 fs = 1.2 × 1.23 = 1.476 MHz
Given that, Bits/sample, n = log2(256) = 8
1 So, Rb = nfs = 8 × 1.476
P(s0) = P(s1 ) =
2 = 11.808 11.81 Mbps

1 1 1 1 113. Sol.
So, Pe = + = = 0.125
2 8 8 8 MAP criteria should be used to minimize the
probability of error,
11. (b) +1
When, p(t) = fY ( y|+1) P( +1) > fY ( y| 1) P( 1)
1(t) and q(t) = – 1(t) : <
1
dmin
1 (t )
P(+1) = 0.80 and P(–1) = 0.20
–1 1
dmin = 2
fY(y|+1) P(+1)
When, p(t) = 1(t) and q(t) = E 2 (t ) :
fY(y|–1) P(–1)

2(t) 4/20
1/20
E y
–3 –1 0 1 3
dmin
Optimum threshold exists here
1 (t )
1 1
Pe (min) = shaded area = 2 × = 0.10
20
dmin = ( E )2 + 1 = E + 1
To obtain same bit error probability, dmin should 114. Sol.
be same. Since the noise variable is Gaussian with zero
mean and ML decoding is used, the decision
So, E+1 = 2
E=3 boundary between two adjacent signal points
will be their arithmetic mean.
GATE Previous Years Solved Paper 85

In the following graphs, the shaded area 5


1
indicates the conditional probability of = E( x 2 ) = x2 dx
10
decoding a symbol correctly when it is 5

transmitted. 5
1 x3 25
= =
10 3 3
5
P1 = 1 – (shaded area)
n = 8 (8-bit uniform quantizer)
rb = nfs = 8 × 10 k = 80 kbps
–3 –1 +1 +2
For reliable transmission, R C
–2
S
R B log 1 +
N
P2 = 1 – (shaded area)
S
= 102.6 = 397.107
N
80 × 103 B log2 (1 + 398.107)
–3 –1 +1 +2
–2 0 80 × 103 B × 8.64
P3 = 1 – (shaded area)
80 × 10 3
B 9.259 = 9.26 kHz
8.64

116. (0.04)
+2 +1 –1 0 +1
1 0
1.5

0 Tb
P4 = 1 – (shaded area)
+2

–2
Tb
+2 +1 –1 +1 1.5
N0
2
Noise variance = = 2 = 0.4
2
By comparing the above graphs, we can P(1) = (0.4), P(0) = 0.6
conclude that P3 is larger among the four. Optimum threshold
2
115. (9.26) S0 + S1 P(0)
= + ln
fm = 4 kHz 2 S1 S0 P(1)
f s = 1.25 × NR
2+2 0.4 0.6
= 1.25 × 2 × fm VTh = + ln
2 2 ( 2) 0.4
= 1.25 × 2 × 4 k
= 10 k-samples/sec 0.4
= ln 1.5
4
fX(x)
= 0.0405 = 0.04 V
1/10
117. (3.14)
x fm = 4 kHz
–5 0 5
Am = 4 V
Message signal power f s = 32 kHz
86 Electronics Engineering Communication Systems

Slope overload distortion can be avoided, n1 = Refractive index of core = 1.5


n2 = Refractive index of cladding = 1.48
dm(t )
dt max Ts C = Critical angle (or) minimum angle
required for total internal reflection
Am 2 fm/fs
I = Angle w.r.t. the axis of the core
4× 2 × 4 k If C is minimum, 1 should be maximum
min = = 3.14 V
32 k
1 n2 1.48
C = sin = sin 1
118. (9.39) n1 1.5

n2 Cladding
= sin–1[0.9866] = 80.6
1 = 90 – C
C
t
Core = 9.39°
1
n1
Core axis
n2
4 Information Theory & Coding

Q.1 A source produces 4 symbols with probability and bandwidth B has capacity C1. If the SNR is
doubled keeping B constant, the resulting
1 1 1 1
, , and . For this source, a practical capacity C2 is given by
2 4 8 8
(a) C2 2C1 (b) C2 C1 + B
coding scheme has an average codeword length
(c) C2 C1 + 2B (d) C2 C1 + 0.3B
of 2 bits/symbols. The efficiency of the code is
[EC-2009 : 2 Marks]
7
(a) 1 (b) Q.6 A source alphabet consists of N symbols with
8
the probability of the first two symbols being
1 1
(c) (d) the same. A source encoder increases the
2 4
probability of the first symbol by a small amount
[EC-1989 : 2 Marks]
and decreases that of the second by . After
Q.2 An image uses 512 × 512 picture elements. Each encoding, the entropy of the source
of the picture elements can take any of the 8 (a) increases
distinguishable intensity levels. The maximum (b) remains the same
entropy in the above image will be (c) increases only if N = 2
(a) 2097152 bits (b) 786432 bits (d) decreases
(c) 648 bits (d) 144 bits [EC-2012 : 1 Mark]
[EC-1990 : 2 Marks]
Q.7 Let U and V be two independent and identically
Q.3 A source generates three symbols with distributed random variables such that,
probabilities 0.25, 0.25, 0.50 at a rate of 3000
1
symbols per second. Assuming independent P (U = +1) = P (U = 1) =
2
generation of symbols, the most efficient source
The entropy H(U + V) in bits is
encoder would have average bit rate is
(a) 6000 bits/sec (b) 4500 bits/sec
3
(a) (b) 1
4
(c) 3000 bits/sec (d) 1500 bits/sec
[EC-2006 : 2 Marks] 3
(c) (d) log2 3
2
Q.4 A memoryless source emits n symbols each with [EC-2013 : 2 Marks]
a probability p. The entropy of the source as a
function of n Q.8 In a code-division multiple access (CDMA)
(a) increases as log n system with N = 8 chips, the maximum number
of users who can be assigned mutually
(b) decreases as log (1/n)
orthogonal signature sequences is ______ .
(c) increases as n
[EC-2014 : 1 Mark]
(d) increases as n log n
[EC-2008 : 2 Marks] Q.9 The capacity of a Binary Symmetric Channel
(BSC) with crossover probability 0.5 is _____ .
Q.5 A communication channel with AWGN
[EC-2014 : 1 Mark]
operating at a signal to noise ratio SNR >> 1
88 Electronics Engineering Communication Systems

Q.10 A fair coin is tossed repeatedly until a ‘Head’ Q.14 A discrete memoryless source has an alphabet
appears for the first time. Let, L be the number of {a1, a2, a3, a4} with corresponding probabilities
tosses to get this first ‘Head’. The entropy H(L)
in bits is ________ .
[EC-2014 : 2 Marks]
{ 1 1 1 1
}
, , , . The minimum required average
2 4 8 8

codeword length in bits to represent this source


Q.11 The capacity of a band-limited additive white for error free reconstruction is _______ .
Gaussian noise (AWGN) channel is given by
[EC-2016 : 1 Mark]

P Q.15 A binary communication system makes use of


C = W log 2 1 + 2
bits per second (bps),
W the symbols “zero” and “one”. There are
channel errors. Consider the following events:
where W is the channel bandwidth, P is the
x0 : a “zero” is transmitted.
average power received and 2 is the one-sided
x1 : a “one” is transmitted.
power spectral density of the AWGN. For a fixed
y0 : a “zero” is received.
P
2
= 1000, the channel capacity (in kbps) with y1 : a “one” is received.
The following probabilities are given:
infinite bandwidth (W ) is approximately
1 3 1
(a) 1.44 (b) 1.08 , P( y0 x0 ) = and P( y0 x1 ) =
P( x0 ) =
2 4 2
(c) 0.72 (d) 0.36 The information in bits that you obtain when
[EC-2014 : 1 Mark] you learn which symbol has been received
(while you know that a “zero” has been
Q.12 Consider a discrete memoryless source with
transmitted) is ______ .
alphabet, S = {s0, s1, s2, s3, s4,....} and respective
probabilities of occurrence, [EC-2016 : 2 Marks]

P= { 1 1 1 1 1
, , , ,
2 4 8 16 32
,..... } Q.16 An analog baseband signal, band-limited to
100 Hz, is sampled at the Nyquist rate. The
The entropy of the source (in bits) is _______ . samples are quantized into four message
[EC-2016 : 2 Marks] symbols that occur independently with
probabilities, p1 = p4 = 0.125 and p2 = p3. The
Q.13 A digital communication system uses a information rate (bits/sec) of the message source
repetition code for channel encoding/decoding. is ________ .
During transmission, each bit is repeated three [EC-2016 : 1 Mark]
times (0 is transmitted as 000, and 1 is
transmitted as 111). It is assumed that the source Q.17 A voice-grade AWGN (Additive White
puts out symbols independently and with equal Gaussian noise) telephone channel has a
probability. The decoder operates as follows. In bandwidth of 4.0 kHz and two-sided noise
a block of three received bits, if the number of
zeros exceeds the number of ones, the decoder
power spectral density = 2.5 × 10 5 Watt/Hz.
2
decides in favor of a 0, and if the number of ones
If information at the rate of 52 kbps is to be
exceeds the number of zeros, the decoder decides
transmitted over this channel with arbitrarily
in favor of a 1. Assuming a binary symmetric
small bit error rate, then the minimum bit-energy
channel with crossover probability p = 0.1. The
Eb (in mJ/bit) necessary is _______ .
average probability of error is _______ .
[EC-2016 : 2 Marks]
[EC-2016 : 2 Marks]
GATE Previous Years Solved Paper 89

Q.18 Let, (X1, X2) be independent random variables. 0.25


0 0
X1 has mean 0 and variance 1, while X2 has
mean 1 and variance 4. The mutual information
0.25
I (X1, X2) between X1 and X2 in bits is ______ .
[EC-2017 : 1 Mark] 0.75

Q.19 Which one of the following graphs shows the 1 1


Shannon capacity (Channel capacity) in bits of 0.75

a memoryless binary symmetric channel with The channel is


crossover probability p ? (a) lossless (b) noiseless
(c) useless (d) deterministic
[EC-2017 : 2 Marks]
1
Q.21 Consider a binary channel code in which each
Capacity

(a)
codeword has a fixed length of 5 bits. The
Hamming distance between any pair of distinct
p
0 1 codwords in this code is at least 2. The
maximum number of codewords such a code
can contain is ________ .
[EC-2018 : 1 Mark]
1
Capacity

Q.22 A linear Hamming code is used to map 4-bit


(b)
messages to 7-bit codewords. The encoder
p
mapping is linear. If the message 0001 is
0 1 mapped to the codeword 0000111 and the
message 0011 is mapped to the codeword
1100110, then the message 0010 is mapped to
(a) 0010011 (b) 1100001
1
(c) 1111000 (d) 1111111
Capacity

(c)
[EC-2019 : 1 Mark]

p Q.23 A digital communication system transmits a


0 1
block of N bits. The probability of error in
decoding a bit is . The error event of each bit is
independent of the error events of the other bits.
1 The received block is declared erroneous if at
Capacity

least one of the its bits is decoded wrongly. The


(d)
probability that the received block is erroneous
is
p
0 1 (a) N(1 – ) (b) 1 – (1 – )N
(c) 1 – N (d) N
[EC-2017 : 1 Mark]
[EC-2020 : 1 Mark]
Q.20 Consider a binary memoryless channel
characterized by the transition probability Q.24 A digital transmission system uses a (7, 4)
diagram shown in the figure. systematic linear Hamming code for
transmitting data over a noisy channel. If three
90 Electronics Engineering Communication Systems

of the message codeword pairs in this code 1–


a a
(mi, ci), where ‘ci’ is the codeword corresponding
to the ith message ‘mi’, are known to be (1100;
0101100), (1110; 0011110) and (0110; 1000 110),
1–
then which of the following is valid codeword b b
in this code?
(a) 1011010 (b) 0110100
(c) 0001011 (d) 1101001 c c
1–
[EC-2021 : 2 Marks]
[EC-2022]
Q.25 Let H(X) denote the entropy of a discrete random
Q.28 Consider communication over a memoryless
variable ‘X’ taking ‘K’ possible distinct real
binary symmetric channel using a (7, 4)
values. Which of the following statements is/are
Hamming code. Each transmitted bit is received
necessarily true?
correctly with probability (1 – ) and flipped
(a) H(X) log2 K-bits
with probability . For each codeword
(b) H(X) H(2X) transmission, the receiver performs minimum
(c) H(X) H(X2) Hamming distance decoding, and correctly
(d) H(X) H(2X) decodes the message bits if and only if the
[EC-2022] channel introduces at most one bit error.
For = 0.1, the probability that a transmitted
Q.26 A symbol stream contains alternate QPSK and codeword is decoded correctly is _____
16-QAM symbols. If symbols from this steam (Rounded off to two decimal places).
are transmitted at the rate of 1 mega-symbols [EC-2022]
per second, the raw (uncoded data rate is ____
mega-bits per second (Rounded off to one Q.29 Consider a channel over which either symbol
decimal place). xA or symbol xB is transmitted. Let the output of
the channel ‘Y’ be the input to a maximum
[EC-2022]
likelihood (ML) detector at the receiver. The
Q.27 The transition diagram of a discrete memoryless conditional probability density functions for ‘y’
channel with three input symbols and three given xA and xB are:
output symbols is shown in the figure. The fY ( y ) = e–(y + 1) u(y + 1)
xA
transition probabilities are as marked. The
parameter lies in the interval [0.25, 1]. The fY xB
( y ) = e(y – 1) (1 – u(y – 1))
value of ___ for which the capacity of this where u( ) is the standard unit step function.
channel is maximized, is ______ (Rounded off The probability of symbol error for this system
of two decimal places). is ______ (Rounded off to two decimal places).
[EC-2022]
GATE Previous Years Solved Paper 91

Answers Information Theory & Coding

1. (b) 2. (b) 3. (b) 4. (a) 5. (b) 6. (d) 7. (c) 8. (8)

9. (0) 10. (2) 11. (a) 12. (2) 13. (0.028) 14. (1.75) 15. (0.811) 16. (362.2)

17. (31.504) 18. (0) 19. (c) 20. (c) 21. (16) 22. (b) 23. (b) 24. (c)
25. (a, b, d) 26. (3) 27. (1) 28. (0.85) 29. (0.23)

Solutions
Information Theory & Coding
1. (b) where, Pi = probability of individual symbol.
n Since probability of each symbol is same,
Entropy = H = Pi log 2 ( Pi ) therefore,
i =1 1
P1 = P2 = ... = Pn =
H=
1
log 2
1 1
+ log 2
1 1
+ log 2
1 1
+ log 2
1 n
2 2 4 4 8 8 8 8
n
1 1
1 1 1 1 H(m) = log = log n
H = + ×2+ ×3+ ×3 i=1 n n
2 4 8 8
3 7
H = 1+ = 5. (b)
4 4
SNR >> 1
H
Code efficiency = =
× 100%
L S S
C 1 = B log 2 1 + B log 2
7 /4 7 N N
= × 100% =
2 8
when SNR is doubled.
2. (b) 2S
C B log 2
n= log2 L N
n= log2 8 = 3 S
= B log 2 + B log 2 2 = C1 + B
Maximum entropy N
=B
= 512 × 512 × n = C1

= 512 × 512 × 3
6. (d)
= 786432 bits/image
We know that entropy is maximum when
3. (b) symbols are equal probable so if probability will
Bit rate = H Rb change from equal to non-equal entropy will
1 1 1 decrease.
H = 0.25 log + 0.25 log + 0.5 log
0.25 0.25 0.5
7. (c)
1 1 1 3
H= + + = = 1.5
2 2 2 2 U V (U + V )
Bite rate = 1.5 × 3000 = 4500 bits/sec
+1 +1 +2
4. (a) +1 –1 0
n –1 +1 0
Entropy, H(m) = Pi log Pi bits
i =1 –1 –1 –2
92 Electronics Engineering Communication Systems

1 1 1 11. (a)
P(U + V = +2) = =
2 2 4
P
1 1 1 C = W log 2 1 + 2
P(U + V = 0) = + = W
4 4 2
2
1 1 1 P P
= C= × W log 2 1 +
P(U + V = –2) = 2 P 2
W
2 2 4
When, W
1 1
H(U + V) = log 2 2 + 2 × log 2 4 2
2 4 P W P
C = 2 W
lim log 2 1 + 2
P W
1 3
= +1= 2
W
2 2 As, W , also
P
9. Sol.
2
P W 1
Channel capacity of BSC is, C = lim log 2 1 +
2 2W P 2 W
C = P log2 P + (1 – P) log2(1 – P) + 1 P P
C = 0.5 log2 0.5 + 0.5 log2 0.5 + 1
1
C=0 lim x log 2 1 + = log 2 e
x x
It is the case of channel with independent input
and output, hence C = 0. P
C = 2
log 2 e
10. Sol.
P
If P = 1, i.e. if 1 toss is required to gets first head. C = 1.44 2
Then probability to get the first head.
1 12. Sol.
P1 =
2 Entropy of source is given as,
If P = 2, i.e. if 2 tosses are required to get first N
1
head. H= Pi log 2 (Here, N = )
i =0 Pi
Then probability to get the first head.
1 1 1 1
1 1 1 = 2 log 2 2 + 4 log 2 4 + 8 log 2 8 + 16 log 2 16 + ...
× =
P2 =
2 2 4
...(i)
Then probability to get the first head.
k
1 1 1 1 1
= k
P3 = × × = 2
2 2 2 8 k =0
Entropy,
2 3 4
H 1 1 1
N
1 = + 2× + 3× + ... ...(ii)
H= Pi log 2 2 2 2 2
i=1 Pi
Subtracting (ii) from (i),
1 1 1 1 1 1 1 1
= log 2 + log 2 + log 2 + log 2 + ... 2 3
2 1/2 4 1/4 8 1/8 16 1/16 H 1 1 1
= + + + ...
1 1 1 1 2 2 2 2
= × 1 + × 2 + × 3 + × 4 + ...
2 4 8 16 1
1 2 3 4 H 2
= 2 + 2 + 3 + 4 + ... 2
=
1
=1
(2) (2) (2) 1
H 2 2
H = 2 bits/symbol
GATE Previous Years Solved Paper 93

13. Sol. 16. Sol.


Crossover probability, fm = 100 Hz
p = 0.1 f s = 2fm = 200 samples/sec.
Average probability of error 1
P1 = P4 =
= 3p2 – 2p3 8
= 3(0.1)2 – 2(0.1)3 P1 + P2 + P3 + P4 = 1
= 0.028 1 3
2P2 = 2 P3 = 1 =
4 4
14. Sol.
3
Minimum required average codeword length in P2 = P3 =
8
bits for error free reconstruction,
N
Lmin = H (Entropy) 1
H= Pi log
i=1 Pi
1 1 1 1
H= log 2 2 + log 2 4 + log 2 8 + log 2 8
2 4 8 8 1 1 3 8 3 8
= log 2 8 + log 2 8 + log 2 + log 2
1 1 3 3 8 8 8 3 8 3
= + + + = 1.75 H = 1.811 bits/sample
2 2 8 8
Information rate,
Lmin = 1.75 bits/word
R = H (bits/sample) × r(sample/sec)
15. Sol. = 1.811 × 200
= 362.2 bits/sec
3/4
x0 y0
1/4 17. Sol.
S
C = B log 2 1 +
1/2 N
x1 y1 S = Eb Rb
1/2
(Eb = Bit energy, Rb = Information rate bits/sec)
Information in bits when you learn which
E R
symbol has been received when zero is C = B log 2 1 + b b
N0 B
transmitted.
Case-I : When y 0 is received and x 0 is For distortionless transmission, C should be
transmitted. atleast of Rb,
Case-II : When y 1 is received and x 0 is Rb E R
= log 2 1 + b b
transmitted. B N0 B
Thus, the overall average information, we can N0 B
R /B
say, H. Eb = (2 b 1)
Rb
1 (2 52 /13 1) × 2 × 2.5 × 10 5 × 4
H = P( y0 |x0 ) log 2
P( y 0 | x 0 ) =
52
Eb = 31.504 mJ/bit
1
+ P( y1 |x0 ) log 2
P( y 1 | x 0 ) 18. Sol.
Mutual information of two random variables is
3 4 1
= log 2 + log 2 (4) bits a measure of the mutual dependence of the two
4 3 4
variables.
= 0.811 bits Given that, X and Y are independent.
Hence, I (X : Y) = 0
94 Electronics Engineering Communication Systems

19. (c) 21. Sol.


The channel capacity of a memoryless binary According to the Plotkin bound,
symmetric channel can be expressed as:
n2 k 1
C = 1 + p log2 p + (1 – p) log2(1 – p) dmin
2k 1
C n = Length of each codeword
k = Length of each message word
Given that,
dmin = 2 and n = 5

2k 2
So, k
p 2 (2 1) 5
0 0.5 1
2k 4
k 5
20. (c) 2 1
For any value of k, the above bound will be
Given that,
satisfied. But as n > k and n is fixed at 5, the
Y 0.25 0.75 maximum value of k that can be selected is 4. So,
P =
X 0.25 0.75 the maximum number of codewords possible is
If mutual information I(X : Y) = 0 for every 24 = 16.
possible input distribution, then the channel is
22. (b)
called as useless (or) zero-capability channel.
00 01 000 01 11
Let, [P(X)] = [ (1 – )]
Then, 00 11 110 01 10
H(X) = – log2 – (1 – ) log2 (1 – ) bits/symbol 00 10 110 00 01

Y
[P(Y)] = [ P(Y )] P = [0.25 0.75] 23. (b)
X
Probability of error in decoding single bit =
3 Then probability of no error will be 1 – .
4 4 Total N-bits transmitted, so that probability of
[P(X, Y)] =
(1 ) 3(1 ) no error in received block
4 4 = (1 – ) (1 – ) ... N times
= (1 – )N
X [ P( X , Y )]
P = = The probability of received block is erroneous is
Y [ P(Y )]d (1 ) (1
= 1 – (1 – )N
X xi
H = P( xi , yi ) log 2 P 24. (c)
Y i j yi
P0 P1 P2 d0 d1 d2 d3
Bits/symbol
0 1 0 1 1 0 0 C1
= – log2 – (1 – ) log2 (1 – )
0 0 1 1 1 1 0 C2
Bits/symbol
1 0 0 0 1 1 0 C3
X
I(X, Y) = H ( X ) H =0 P 0 = d0 d1 d3
Y
P 1 = d1 d2 d3
So, the given binary memoryless channel is a
P 2 = d0 d1 d2
‘useless’ channel.
GATE Previous Years Solved Paper 95

P0 P1 P2 d0 d1 d2 d3 i.e., H(X) = H(2X)


1 1 0 1 0 1 0 option 'a' Given option (c),
1 1 1 0 1 0 0 option 'b' H(X) H(X2)
Let, Y = X2
0 0 0 1 0 1 1 option 'c'
0 1 1 1 0 0 1 option 'd' X Y P (Y )
Y {yi} 0 1
So, option (c) is a valid codeword. –1 1 1/4
1 1
0 0 1/2 P(Y = yi)
25. (a, b, d) 2 2
1 1 1/4
Given that, ‘X’ is a discrete random variable
taking ‘K’ possible distinct real values. If ‘K’ 1
H(X2) = H (Y ) = PY ( yi )log 2
PY ( yi )
symbols having equal probability then entropy i

will be maximum H(X)max = log2 K. If symbols 1 1


= log 2 2 + log 2 2
having different probabilities then H(X) < log2 K. 2 2
So that, H(X) log2 K = 1 bit/symbol
Option (a) is correct. whereas, H(X) = 1.5 bits/symbol
Given option (b), Option (c) is incorrect.
H(X) H(2X) Given option (d),
Let, H(X) H(2X)
Let, Y = 2X
X {xi} –1 0 1
1 1 1 Y
PX{xi}
4 2 4

1
H(X) = PX ( xi )log 2
i PX ( xi )

1 1 1 X
= log 2 4 + log 2 2 + log 2 4 0
4 2 4
H(X) = 1.5 bits/symbol Here distinct ‘X’ values results in distinct ‘Y’
Let, Y = 2X values.
X Y P (Y ) So that, H(X) = H(Y)
–1 –2 1/4 i.e., H(X) = H(2X)
0 0 1/2 Option (d) is true.
1 2 1/4
26. (3)
Y {yi} –2 0 2 Given that symbol stream contains alternate
1 1 1 QPSK and 16-QAM symbols. Symbol rate given
PY{yi}
4 2 4 as 1 mega symbols per second,
Bit rate = Symbol rate × log2 M
1
H(2X) = H (Y ) = PY ( yi )log 2 For QPSK:
i PY ( yi )
= 1.5 bits/symbol symbol bits
Bit rate = 1 M × log 2 4
For Y = 2X, distant ‘X’ values results in distinct sec symbol
‘Y’ values so that, Bit rate = 2 Mbps
H(X) = H(Y)
So, option (b) is true.
96 Electronics Engineering Communication Systems

For 16-QAM: I(X ; Y) = H(Y) + (1 – ) log2 (1 – ) + log2


symbol bits C s = Max{(X ; Y)}
Bit rate = 1 M × log 2 16
sec symbol = Max{H(Y)} + (1 – ) log2(1 – )
+ log2
Bit rate = 4 Mbps
Since, alternate QPSK and 16-QAM used, data C s = log 2 3 + (1 )log 2 (1 ) + log 2
rate of uncoded data is
log2 + (1 – ) log(1 – )
4 Mbps + 2 Mbps
= = 3 Mbps
2
0.5 1
27. (1) 0
1–
x1 a a y1

–1

1–
x2 b b y2
Cs will be maximum at = 0 and 1.
Given, [0.25, 1]
So that, = 1 will be the correct answer.
x3 c c y3
1–
Channel capacity,
28. (0.85)
C s = Max[I(X ; Y)]
Given, (7, 4) Hamming code.
Y Number of bits in the transmitted codeword = 7
I(X ; Y)] = H (Y ) H
X Given is binary symmetric channel P(0/1) =
P(1/0),
Y 3 3 yj P(0/1) = P(1/0) = = 0.1
where, H = P(xi , y j ) log 2 P
X i =1 j=1 xi Probability of correct decoding of codeword (Pc)
= Probability of almost one bit error,
Pc = No error (or) 1 bit error
y1 y2 y3
When ‘n’ bits transmitted probability of getting
x1 1 0
Y error in ‘r’ bits is nCr Pr(1 – P)n – r.
P = x2 0 1
Where ‘p’ is bit error probability,
X
x3 0 1
P(1/0) = P(0/1) = 0.1
Pc = 7C0(0.1)0 (1 – 0.1)7 – 0 +
For simplification convenience, let [P(X)] = 7C (0.1)1 (1 – 0.1)7 – 1
1
[1 0 0], = (0.9)7 + 7 × 0.1 × (0.9)6
Y Pc = 0.8503
[P(X, Y)] = [ P( X )d P
X
29. (0.23)
1 0
[P(X, Y)] = 0 0 0
fY |XA (y) fY|XB(y )
0 0 0

Y
H = {(1 )log 2 (1 ) + log 2 }
X
y
–1 0 1
GATE Previous Years Solved Paper 97

1 1
xA ( y + 1)
PeX = fY|X A ( y ) dy = e u( y + 1) dy
> A
0 0
ML decoding fY|X A ( y ) f (y )
< Y|XB 1
xB ( y + 1) 1 2
= e dy = e e = 0.23
0
i.e., fY|X (y) > fY|X (y) Decision favour of XA
A B 0 0
i.e., fY|X (y) < fY|X (y) Decision favour of XB PeX = fY|XB ( y ) dy = e( y 1)
[1 u( y 1)] dy
A B B
For –1 < y < 0 and 1 < y < fY/X (y) > fY/X (y) 1 1
A B
For above internal decision in favour of XA. 0

For – < y < –1 and 0 < y < 1 fY|X (y) > fY/X (y)
= ey 1
dy = e 1
e 2
= 0.23
B A 1
For above internal decision in favour of XB,
Pe = P(XA) × 0.23 + P(XB) × 0.23
Pe = P( X A ) PeX + P( X B ) PeX
A B = 0.23[P(XA) + P(XB)]
PeX Probability of error XA transmitted Pe = 0.23
A

PeX Probability of error XB transmitted


B
GATE-2023
Electronics Engineering

Control Systems
Chapterwise & Topicwise

Contents
S.No. Topic Page No.

1. Basics of Control Systems, Block Diagram and SFG’s .............................................................. 1-19

2. Compensators and Controllers .................................................................................................... 20-31

3. Time Response Analysis ................................................................................................................ 32-60

4. Stability Analysis ............................................................................................................................... 61-81

5. Root Locus .......................................................................................................................................... 82-98

6. Frequency Response Analysis .................................................................................................. 99-134

7. State Space Analysis ................................................................................................................... 135-164


1 Basics of Control Systems,
Block Diagram and SFG’s
ELECTRO NICS EN GINEERIN G G1G2 G3G4
(c)
(GATE Previous Years Solved Papers) (1 + G1 + G2 ) (1 + G3 + G4 )

G1G2 G3G4
Q.1 In the signal flow graph shown in figure (d)
(1 + G1 + G2 + G3 + G4 )
X2 = TX1 where T, is equal to
[EC-1989 : 2 Marks]
0.5
Q.4 In the signal flow graph of figure the gain c/r
will be

5 5
X1 X2

(a) 2.5 (b) 5 1 2 3 4


c
r
(c) 5.5 (d) 10
[EC-1987 : 2 Marks] –1 –1 –1

Q.2 For the system shown in figure the transfer 11 22


function C(s)/R(s) is equal to (a) (b)
9 15
10 24 44
R(s) +

+
– s (s + 1)
C(s ) (c) (d)
23 23
[EC-1991 : 2 Marks]

s
Q.5 Signal flow graph is used to find
(a) stability of the system

10 10 (b) controllability of the system


(a) (b)
s 2 + s + 10 s 2 + 11 s + 10 (c) transfer function of the system
(d) poles of the system
10 10
(c) (d) [EC-1995 : 1 Mark]
s 2 + 9 s + 10 s 2 + 2 s + 10
[EC-1987 : 2 Marks] Q.6 The transfer function of a linear system is the
(a) ratio of the output, vo(t) and input vi(t).
Q.3 The C/R for the signal flow graph in figure is
(b) ratio of the derivatives of the output and
1 G1 G2 1 G3 G4 1
the input.
R C
(c) ratio of the Laplace transform of the output
–1 –1 –1 –1
and that of the input with all initial
G1G2 G3G4 conditions zeros.
(a)
(1 + G1G2 ) (1 + G3G4 )
(d) none of these.
G1G2 G3G4 [EC-1995 : 1 Mark]
(b)
(1 + G1 + G2 + G1G2 ) (1 + G3 + G4 + G3G4 )
2 Electronics Engineering Control Systems

Q.7 In the signal flow graph of figure, y/x equals


5 2 1 Z1(s) Z2(s)
x y Vo(s )

–2 I1(s) I2(s )
Vi(s) Z 3 (s ) Z4(s)
5
(a) 3 (b)
2
(c) 2 (d) None of the above
Fig. (a)
[EC-1997 : 2 Marks]

Q.8 The equivalent of the block diagram in the figure G1 I1(s ) G2 I2(s) G3
is given as, Vi(s ) Vo(s )

E G1 G2 C H
Fig. (b)

Z3 (s) Z3 (s)
(a) ,
F H Z2 (s) + Z3 (s) + Z4 (s) Z1 (s) + Z3 ( s)
Z3 (s) Z3 (s)
(b) ,
E G1 C Z2 (s) Z3 (s) + Z4 (s ) Z1 (s) + Z3 (s)
Z3 (s) Z3 (s)
(a) (c) ,
Z2 (s) + Z3 (s) + Z4 (s) Z1 (s) + Z3 ( s)
F H/G2
Z3 (s) Z3 (s)
(d) ,
Z2 (s) Z3 (s) + Z4 (s ) Z1 (s) + Z3 (s)
E G1G2 C [EC-2001 : 2 Marks]
(b) Q.10 The signal flow graph of a system is shown in
HG2 the figure. The transfer function C(s)/R(s) of the
F
system is
1 1/s 6 1/s
E G1 C R(s)
–3
(c) 1 –2
–4
F HG2
C(s )

6 6s
E G1G2 C (a) 2 (b) 2
s + 29s + 6 s + 29s + 6
(d) s (s + 2) s (s + 27)
(c) 2 (d) 2
s + 29s + 6 s + 29s + 6
F H/G2
[EC-2003 : 2 Marks]
[EC-2001 : 1 Mark] Q.11 Consider the signal flow graph shown in the
Q.9 An electrical system and its signal flow graph figure. The gain x5/x1 is
representation are shown in the Fig. (a) and (b) a x2 b x3 c x4 d x5
x1
respectively. The values of G2 and H, repectively,
are:
e f g
GATE Previous Years Solved Paper 3

1 ( be + cf + dg ) –100
(a)
abc
1/s 1/s 1/s 100
(b) u y
bedg
(b)
1 (be + cf + dg )
–20
abcd
(c) –100
1 (be + cf + dg ) + bedg
1/s 1/s 1/s 100
1 ( be + cf + dg ) + bedg (c) u y
(d)
abcd
–20
[EC-2004 : 2 Marks]
–100
Q.12 The transfer function Y(s)/R(s) of the system
shown is (d) u
1/s 1/s 1/s 100 y

+ 1 [EC-2011 : 2 Marks]
R(s ) Y(s )
s+1
– Q.14 The signal flow graph for a system is given
+ below. The transfer function Y(s)/U(s) for this
1
s+1 – system is
1

1 s–1 s–1 1
1
(a) 0 (b) U(s ) Y(s )
s+1
2 2 –4
(c) (d)
s+1 s+3
–2
[EC-2010 : 1 Mark]
s+1 s+1
Q.13 The input-output transfer function of a plant (a) 2 (b) 2
5s + 6 s + 2 s + 6s + 2
100 s+1 1
H (s) = . The plant is placed in a unity (c) (d)
s (s + 10) 2 s 2 + 4s + 2 5s 2 + 6 s + 2
[EC-2013 : 2 Marks]
negative feedback configuration as shown in
the figure below. Q.15 For the following system,
X2 ( s )
+ u 100
r H (s ) = y
s (s + 10)2 X1(s ) +
s
+
+ 1
Y(s )
– – s+1 – s
Plant

where X 1 (s) = 0, the transfer function is


The signal flow graph that does not model the Y(s)/X2(s) is
plant transfer function H(s) is s+1 1
(a) 2
(b)
1 1/s 1/s 1/s 100 s s+1
(a) u y s+2 s+1
(c) (d)
s ( s + 1) s ( s + 2)
–10 –10
[EC-2014 : 1 Mark]
4 Electronics Engineering Control Systems

Q.16 Consider the following block diagram in the 1


figure. (c) G1 (s ) + G2 (s )
G1 (s )
R(s ) G1 + G2 + C(s )
+ + 1
(d) G1 (s ) G2 (s )
G1 (s )

The transfer function C(s)/R(s) is [EC-2015 : 1 Mark]

G1G2 Q.20 The position control of a dc servo-motor is given


(a) (b) G1G2 + G1 + 1
1 + G1G2 in the figure. The values of the parameters are
G1 KT = 1 N-m A, Ra = 1 , La = 0.1 H. J = 5 kg-m2,
(c) G1G2 + G2 + 1 (d) B = 1 N-m (rad/sec) and Kb = 1 V/(rad/sec).
1 + G1G2
The steady-state position response (in radians)
[EC-2014 : 1 Mark]
due to unit impulse disturbance torque Td is __.
Q.17 Negative feedback in closed-loop control system Td(s)
does not
(a) reduce the overall gain KT – 1
+ + 1/s
(b) reduce bandwidth Va(s ) – Ra + L a s Js + B (s)

(c) improve disturbance rejection


Kb
(d) reduce sensitive to parameter variation
[EC-2015 : 1 Mark] [EC-2015 : 2 Marks]

Q.18 For the signal flow graph shown in the figure, Q.21 The block diagram of a feedback control system
the value of C(s)/R(s) is is shown in the figure. The overall closed-loop
–H 3 gain G of the system is

X + G2 + G1 Y
– –
1 X1 X3 1 X4
R(s) G1 X2 G2 G3 X5 G4 C(s ) H1

–H1 –H2 G1G2


(a) G =
G1G2 G3 G4 1 + G1 H 1
(a) 1 G G H G G H G G H + G G G G H H
1 2 1 3 4 2 2 3 3 1 2 3 4 1 2 G1G2
G1G2 G3 G4
(b) G =
(b) 1 + G1G2 + G1 H 1
1 + G1G2 H 1 + G3G4 H 2 + G2 G3 H 3 + G1G2 G3G4 H 1 H 2
G1G2
(c)
1 (c) G =
1 + G1G2 H 1 + G3G4 H 2 + G2 G3 H 3 + G1G2 G3G4 H 1 H 2 1 + G1G2 H 1
1 G1G2
(d) (d) G =
1 G1G2 H 1 G3G4 H 2 G2 G3 H 3 + G1G2 G3G4 H 1 H 2
1 + G1G2 + G1G2 H 1
[EC-2015 : 1 Mark]
[EC-2016 : 1 Mark]
Q.19 By performing cascading and/or summing/
Q.22 For the system shown in the figure, Y(s)/X(s) =
differencing operates using transfer function
_______ .
blocks G1(s) and G2(s), on cannot realize a
transfer function of the form

(a) G1(s) G2(s) X(s ) + G(s ) = 2 +
+
Y (s )

G1 (s)
(b)
G2 (s) [EC-2017 : 1 Mark]
GATE Previous Years Solved Paper 5

Q.23 The block diagram of a system is illustrated in ELECTRICAL EN GINEERIN G


the figure shown, where X(s) is the input and
(GATE Previous Years Solved Papers)
Y(s) is the output. The transfer function
Q.1 The signal flow graph of figure shown below,
Y (s )
H (s) = is has ____ forward paths and ___ feedback loops.
X( s )
c

s b d e
– +
+
X(s) 1/s Y(s ) a h
x
– +
1/s
k I f

m
y
2 g
s +1
(a) H (s ) =
2s2 + 1 n
2
s +1 [EE-1991 : 2 Marks]
(b) H (s ) =
s + 2s 2 + s + 1
3
Q.2 The overall transfer function of the system
s+1
(c) H (s ) = shown in figure is
s2 + s + 1
+
s2 + 1 G
(d) H (s ) = 3 2 +
s +s +s+1
[EC-2019 : 2 Marks] H +
w y
Q.24 The block diagram of a feedback control system H +

is shown in the figure.


+
+
G
G2

X(s) +– G1 +
+ Y(s ) G 2G
(a) (b)
1 GH 1 GH
H GH GH
(c) (d)
1 GH 1 H
Y (s ) [EE-1992 : 1 Mark]
The transfer function of the system is
X( s )
Q.3 Signal flow graph is used to obtain the
G1 + G2
(a) (a) stability of a system.
1 + G1 H
(b) transfer function of a system.
G1 + G2 + G1G2 H (c) controllability of a system.
(b)
1 + G1 H
(d) observability of a system.
G1 + G2 + G1G2 H [EE-1993 : 1 Mark]
(c)
1 + G1 H + G2 H
Q.4 The closed loop transfer function of a control
G1 + G2
(d) system is given by
1 + G1 H + G2 H
C (s) 2 (s 1)
[EC-2021 : 1 Mark] =
R(s ) ( s + 2) (s + 1)
6 Electronics Engineering Control Systems

For a unit step input the output is Q.8 A control system is defined by the following
(a) –3e–2t + 4e–t – 1 (b) –3e–2t – 4e–t + 1 mathematical relationship
(c) zero (d) infinity d2 x dx
2
+6 + 5x = 12 (1 e 2t )
[EE-1995 : 1 Mark] dt dt
The response of the system as t is,
Q.5 For block diagram shown in figure C(s)/R(s) is
given by (a) x = 6 (b) x = 2
(c) x = 2.4 (d) x = –2
H2
[EE-2003 : 1 Mark]

R(s) +

G1 + G2 G3 C(s ) Q.9 A control system with certain excitation is
governed by the following mathematical
H1 equation,

G1G2 G3 d2 x 1 dx 1
(a) 2
+ + x = 10 + 5 e 4t + 2 e 5t
1 + H 2 G2 G3 + H1G1G2 dt 2 dt 18
The nature time constants of the response of the
G1G2 G3
(b) system are
1 + G1G2 G3 H 1 H 2
(a) 2s and 5s (b) 3s and 6s
G1G2 G3 (c) 4s and 5s (d) 1/3 and 1/6s
(c)
1 + G1G2 G3 H 1 + G1G2 G3 H 2
[EE-2003 : 2 Marks]
G1G2 G3
(d) Q.10 The block diagram of a control system is shown
1 + G1G2 G3 H 1
in figure. The transfer function G(s) = Y(s)/U(s)
[EE-1998 : 2 Marks]
of the system is
Q.6 A linear time invariant system initially at rest, 9
when subjected to a unit step input, gives a
response y(t) = te–t, t > 0. The transfer function of – for for y(t)
+– 2 +–
integrator integrator
the system is
1 1
(a) (b) 3 12
(s + 1) 2 s(s + 1)2
1
s s (a)
(c) (d) s s
(s + 1)2 s ( s + 1) 18 1 + 1+
12 3
[EE-2000 : 1 Mark]
1
(b)
Q.7 The transfer function of the system described s s
27 1 + 1+
6 9
d2 y
dy du
by +
= + 2u with ‘u’ as input and ‘y’
2 dt dt 1
dt (c)
as output is s s
27 1 + 1+
(s + 1) 12 9
(s + 2)
(a) (b)
2
(s + s) (s2 + s) 1
(d)
2 2s s s
27 1 + 1+
(c) 2 (d) 2 9 3
(s + s) (s + s)
[EE-2002 : 2 Marks] [EE-2003 : 2 Marks]
GATE Previous Years Solved Paper 7

Q.11 For a tachometer, if (t) is the rotor displacement Q.15 The system shown in figure below:
in radians, e(t) is the output voltage and Kt is
the tachometer constant (in V/rad/sec), then
the transfer function, E(s)/Q(s) will be b0 c0 b1 c1

(a) Kt s2 (b) Kt/s +


(c) Kt s (d) Kt +
+ +
[EE-2004 : 1 Mark] +
1/s
+
1/s P
– –
Q.12 For the block diagram shown in figure, the
a0 a1
transfer function C(s)/R(s) is equals to

R(s) 1/s + 1/s + C(s )


+ +
can be reduced to the form
+
X Y P
+

s2 + 1 s2 + s + 1 Z
(a) (b)
s2 s2
with
s2 + s + 1 1 1
(c) (d) (a) X = c0 s + c1 , Y = , Z = b0 s + b1
s s2 + s + 1 (s 2 + a0 s + a1 )
[EE-2004 : 2 Marks] (c 0 s + c 1 )
(b) X = 1, Y = 2
, Z = b0 s + b1
Q.13 The unit impulse response of a second order (s + a0 s + a1 )
under damped system starting from rest is given (b1 s + b0 )
(c) X = c1 s + c0 , Y = ,Z=1
by 2
(s + a1s + a1 )
c(t) = 12.5 e–6t sin8t, t 0 1
(d) X = c1s + c0 , Y = 2
, Z = b1s + b0
The steady-state value of the unit step response (s + a1s + a0 )
of the system is equal to [EE-2007 : 2 Marks]
(a) 0 (b) 0.25
Q.16 A function y(t) satisfies the following differential
(c) 0.5 (d) 1.0
equation,
[EE-2004 : 2 Marks]
dy(t )
+ y (t ) = ( t )
Q.14 When subjected to a unit step input, the closed dt
loop control system shown in the figure will where, (t) is the delta function.
have a steady-state error of Assuming zero initial condition, and denoting
the unit step function by u(t), y(t) can be of the
X(s) form
– 2
R(s) + 3/s + Y(s )
– s+2 (a) et (b) e–t
(c) et u(t) (d) e–t u(t)
[EE-2008 : 1 Mark]
(a) –1.0 (b) –0.5
Q.17 The response h(t) of a linear time invariant
(c) 0 (d) 0.5
system to an impulse (t), under initially relaxed
[EE-2004 : 2 Marks]
condition is h(t) = e–t + e–2t. The response of this
system for a unit step input u(t) is
8 Electronics Engineering Control Systems

(a) u(t) + e–t + e–2t Assuming, h1 = b1 and h0 = b0 – b1a1, the input-


(b) (e–t + e–2t) u(t)
C(s)
(c) (1.5 – e–t – 0.5 e–2t) u(t) output transfer function, G(s) = of the
U(s )
(d) e–t (t) + e–2t u(t) [EE-2011 : 2 Marks]
system is given by
Q.18 The transfer function V2(s)/V1(s) on the circuit b0 s + b1
(a) G(s ) =
shown below is 2
s + a0 s + a1
100 µF a1 s + a0
(b) G(s ) =
+ + 2
s + b1s + b0
10 k
V1(s) V2(s) b1 s + b0
(c) G(s ) =
2
s + a1s + a0
100 µF
– –
a s + a1
(d) G(s ) = 2 0 [EE-2014 : 1 Mark]
0.5s + 1 3s + 6 s + b0 s + b1
(a) (b)
s+1 s+2
Q.21 For the signal flow graph shown in the figure,
s+2 s+1
(c) (d) which one of the following expressions is equal
s+1 s+2
[EE-2013 : 1 Mark] Y (s )
to the transfer function ?
Q.19 The signal flow graph for a system is given X2 (s ) X
1 (s) = 0
below. The transfer function Y(s)/U(s) for this
system is X1(s) X2(s)

1
1 G1 G2
Y(s )
1 s–1 s–1 1
U(s ) Y(s )

–4
–1 –1
–2
G1 G2
s+1 s+1 (a) (b)
(a) (b) 1 + G2 (1 + G1 ) 1 + G1 (1 + G2 )
5s 2 + 6 s + 2 s 2 + 6s + 2
s+1 1 G1 G2
(c) (d) (c) (d)
5s 2 + 6 s + 2 1 + G1 G2 1 + G1 G2
s 2 + 4s + 2
[EE-2013 : 2 Marks] [EE-2015 : 1 Mark]

Q.20 The signal flow graph of a system is shown Q.22 Find the transfer function Y(s)/X(s) on the
below. U(s) is the input and C(s) is the output. system given below.
h1 +– G1

h0 1 1/s 1 1/s 1
U(s) C(s ) X(s) + Y(s )
H
Y(s ) +
–a1


+ G2
–a0
GATE Previous Years Solved Paper 9

G1 G2 Q.25 For a system having transfer function,


(a) +
1 HG1 1 HG2 s+1
G(s ) = , a unit step input is applied time
G1 G2 s+1
(b) +
1 + HG1 1 + HG2 t = 0. The value of the response of the system at
G1 + G2 t = 1.5 sec (Rounded off to 3 decimal places), is
(c) _________ . [EE-2017 : 2 Marks]
1 + H (G1 + G2 )
G1 + G2 Q.26 Let a causal LTI system be characterized by the
(d) [EE-2015 : 2 Marks]
1 H (G1 + G2 ) following differential equation, with initial rest
condition,
Q.23 For the system governed by the set of equations:
dx1 d2 y dy dx(t )
= 2 x1 + x 2 + u 2
+7 + 10 y(t ) = 4x(t ) + 5
dt dt dt dt
dx2 where, x(t) and y(t) are the input and output
= 2 x1 + u
dt respectively. The impulse response of the system
y = 3x1 is (u(t) is the unit step function)
The transfer function Y(s)/U(s) is given by (a) 2e–2t u(t) – 7e–5t u(t)
3( s + 1) 3(2 s + 1) (b) –2e–2t u(t) + 7e–5t u(t)
(a) (b) (c) 7e–2t u(t) – 2e–5t u(t)
(s2 2 s + 2) (s2 2 s + 1)
(d) –7e–2t u(t) + 2e–5t u(t) [EE-2017 : 2 Marks]
(s + 1) 3(2 s + 1)
(c) (d)
(s2 2s + 1) (s2 2 s + 2) Q.27 Which of the options is an equivalent
[EE-2015 : 2 Marks] representation of the signal flow graph shown
here?
Q.24 In the system whose signal flow graph is shown
c
in figure, U1(s) and U2(s) are inputs. The transfer 1 a 1
function Y(s)/U1(s) is d

–R U2 e
–1
1 (a + c) d 1
1 1 (a)
U1 1/L 1/s k1 1/J 1/s Y

e
1 a( d + c) 1
(b)
–k2

k1 e
(a)
JLs 2 + JRs + k1 k2 a
c
1 1 cd 1
k1 (c)
(b)
JLs 2 JRs k1 k2
e
k1 U 2 ( R + sL )
(c) 2 d
JLs + ( JR U 2 L ) s + k1 k2 U 2 R a
1 cd
1 1
(d)
k1 U 2 (sL R )
(d) 2
JLs ( JR + U 2 L ) s k1 k2 + U 2 R
e
[EE-2017 : 2 Marks] [EE-2020 : 2 Marks]
10 Electronics Engineering Control Systems

Electronics & Electrical Engineering


GATE Previous Years Solved Paper

A n swe rs & Expl a n a t i o n s

Answers
EC Basics of Control Systems, Block Diagram and SFG’s

1. (d) 2. (b) 3. (c) 4. (d) 5. (c) 6. (c) 7. (c) 8. (d)

9. (c) 10. (d) 11. (c) 12. (b) 13. (d) 14. (a) 15. (d) 16. (c)

17. (b) 18. (b) 19. (b) 20. (–0.5) 21. (b) 22. (1) 23. (b) 24. (a)

Solutions
EC Basics of Control Systems, Block Diagram and SFG’s

1. Sol. C G1G2 G3 G4
=
X2 5 5 5 R 1 [G1 G2 G3 G4 ]
= = = = 10
X1 1 (0.5) 0.5 + [G1G3 + G1G4 + G2 G3 + G2 G4 ]

2. (b) C G1G2 G3G4


=
R 1 + G1 + G2 + G3 + G4 + G1G3
10 + G1G4 + G2 G3 + G2 G4
C (s) s (s + 1)
= G1G2 G3G4
R( s ) 10 =
1+ ×s (1 + G1 + G2 ) (1 + G3 + G4 )
s (s + 1)
10
4. (d)
s (s + 1)
1+ ×1 By using Masson’s gain formulae,
10
1+ ×s
s (s + 1) C PK K
=
R
10
C (s) s(s + 1) + 10s 10 C (1 × 2 × 3 × 4 × 1) + (1 × 5 × 1) × (1 + 3)
= = 2 =
R( s ) 10 s + s + 10s + 10 R 1 ( 2 3 4 5) + ( 2) × ( 4)
1+
s(s + 1) + 10s
C 24 + 20 44
= =
C (s) 10 R 1 + 14 + 8 23
= 2
R( s ) s + 11s + 10
5. (c)
3. (c) The signal flow graph can not be used to
By using Masson’s gain formulae, comment on stability, controllability
C PK K absorbability or pole zero locations. It only gives
= transfer function.
R
GATE Previous Years Solved Paper 11

6. (c) L1 and L3 are non-touching,


The transfer function of a linear time invariant (s + 27)
system is defined as the ratio of the Laplace s
G(s) =
transform of the output and that of the input 3 24 2 2 3
1 + ×
with all initial conditions zero. s s s s s

7. (c) (s + 27)
s
C PK K =
= 29 6
1+ +
R s s2
C 5× 2× 1 10 10
= = = =2 C (s) s (s + 27)
R 1 ( 4) 1 + 4 5 = 2
R(s ) s + 29s + 6
8. (d)
11. (c)
Take off point is moved after G2 so H/G2.
P1 = abcd, 1 = 1
9. (c) L1 = be, L2 = cf, L3 = dg
From KVL in both loops: Non-touching loops are L1 and L3 = bedg
In first loop, X5 abcd
=
Vi(s) = I1(s) Z1(s) + [I1(s) – I2(s)] Z3(s) X1 1 (be + cf + dg ) + bedg
Vi(s) = I1(s) [Z1(s) + Z3(s)] – I2(s) Z3(s)
12. (b)
Vi (s) I 2 (s) Z3 (s)
= I1 (s) ...(i)
Z1 (s) + Z3 (s) Z1 (s) + Z3 (s) + P(s ) 1
R(s ) Y(s )
In second loop, s+1

[I2(s) – I1(s)] Z3(s) + I2(s) Z2(s) + I2(s) Z4(s) = 0
+
I2(s) (Z2(s) + Z3(s) + Z4(s)) = I1(s) × Z3(s) 1
s+1 –
I 2 (s) Z3 (s)
G2 = =
I 1 (s) Z2 (s) + Z3 (s) + Z4 (s)
Q(s )
From SFG,
I1(s) = ViG1(s) + I2(s) H(s) 1 1
Q(s) = P(s) =0
1 Z3 (s ) s+1 s+1
I1(s) = Vi (s) + I 2 (s)
Z1 (s ) + Z3 (s ) Z1 (s ) + Z3 (s ) So, P(s) = R(s) – 0 = R(s)
...[(From equation (i)] P(s ) R(s )
Y(s) = =
Z3 (s) s+1 s+1
H(s) =
Z1 (s) + Z3 (s) Y (s) 1
Therefore, =
...(Comparing above two equations) U (s) s+1

10. (d) 13. (d)


P1 = 1 For option (d),
3 24 s + 27
1 = 1+ + = Y (s) 100 / s 3
s s s =
U (s) 100
P1 1 1+ 2
G(s) = 1 (loop gain) + pair of non-touching loops s
3 24 2 Which is not transfer function of H(s).
L1 = , L2 = , L3 =
s s s
12 Electronics Engineering Control Systems

14. (a) G1 G2
R(s ) C(s )
Using Mason’s gain formula,
= 1 – [–2s–1 – 2s–2 – 4 – 4s–1]
1 1
2 2 4
= 1+ + 2 + 4+
s s s Forward paths,
P1 = G1G2, P2 = G2 1
5s 2 + 6 s + 2
= P3 = 1 1 = 1
s2
So, the transfer function is,
2 1
P1 = s = C (s)
s2 = G1G2 + G2 + 1
R(s )
1
P2 = s 1 =
s 18. (b)
1 = 1; 2=1 Transfer function,
1 1 C (s) PK
×1+ ×1 K
Y (s) PK 2 s =
=s 2
K
= R(s )
U (s) 5s + 6s + 2 G1G2 G3G4
=
s2 1 (G1G2 H 1 G2 G4 H 2 G2 G3 H 3 ) + G1G2 H1 G3G4 H 2

Y (s) s+1 C (s) G1G2 G3G4


= = 1 + G1G2 H 1 + G2 G4 H 2 + G2 G3 H 3 + G1G2 H1 G3G4 H 2
U (s) 2
5s + 6 s + 2 R(s )

15. (d) 19. (b)


Redrawing the block diagram with X1(s) = 0 Given blocks has transfer function G1(s) and
G2(s).
X2(s) + 1/s C(s ) In cascade connection: G1(s) G2(s)

In parallel connection: G1(s) + G2(s)
s From the given options:
(s + 1) (For cascading/summing/differencing)
(a) G1(s) G2(s) Realization is possible.
The transfer function,
Y (s) G(s ) G1 (s)
T(s) = = ...(i) (b) Realization is not possible
X2 (s) 1 + G(s) H (s) G2 (s)
1 s because the gain of blocks given as G1(s)
Here, G(s) = and H (s ) =
s s+1 and G2(s).
Y (s ) 1/ s (s + 1) 1
= = (c) G1 (s ) + G 2 ( s ) = 1 + G1 ( s ) G 2 ( s )
X2 (s ) 1 s s(s + 2)
1+ × G1 (s )
s s+1
Realization is possible.
16. (c)
1
(d) G1 (s ) G2 (s ) = 1 G1 (s ) G2 (s )
R(s ) G1 + G2 + C(s ) G1 (s )
+ +
Realization is possible.

20. Sol.
Converting the block diagram into signal flow
The transfer function due to the disturbance
graph on,
torque Ta(s) is
GATE Previous Years Solved Paper 13

1 1 22. Sol.
×
(s ) ( Js + B) s
=
Td (s) 1 Kb KT
1+
Js + B Ra + L a s X(s ) +

G(s ) = 2 + Y(s )
X(s) – Y(s) +
1
( Ra + L a s )
s
=
( Ra + La s) ( Js + B) + Kb KT Y(s) = [X(s) – Y(s)] G(s) + X(s)
The steady value of response for unit impulse [1 + G(s)] Y(s) = [G(s) + 1] X(s)
input, Y (s )
=1
1 X( s )
s ( Ra + L a s )
s
(0) = lim Td 23. (b)
s 0 ( Ra + La s ) ( Js + B) + K b KT
Using block diagram reduction, we get
Ra
= 1
Ra B + Kb KT

Given, + 1
X(s) s+ 1/s Y(s )
s
K T = 1 N-m/A, Ra = 1 –
B = 1 N-m/rad/sec
and Kb = 1 V/rad/sec
+ s2 + 1
Substituting the given values into above X(s)
2
1/s Y(s )
s +s+1
equation, we get –

1
(0) = = 0.5 rad
2 Y (s ) s2 + 1
= H (s) = 3
21. (b) X( s ) s + 2s 2 + s + 1

G1 24. (a)
X + G2 Y
– 1 + G1 H 1
Y (s ) G + G2
= 1
X( s ) 1 + G1 H
[By using Mason’s gain formula]
G2 G1
Y (s ) 1 + G1 H 1 G2 G1
= =
X( s ) G2 G1 1 + G1G2 + G1 H 1
1+
1 + G1 H 1

Answers
EE Basics of Control Systems, Block Diagram and SFG’s

1. (4, 4) 2. (b) 3. (b) 4. (a) 5. (a) 6. (c) 7. (a) 8. (c)

9. (b) 10. (b) 11. (c) 12. (b) 13. (d) 14. (c) 15. (d) 16. (d)

17. (c) 18. (d) 19. (a) 20. (c) 21. (b) 22. (c) 23. (a) 24. (a)

25. (0.554) 26. (b) 27. (d)


14 Electronics Engineering Control Systems

Solutions
EE Basics of Control Systems, Block Diagram and SFG’s

1. Sol. t 1
Y(s) = L te =
Number of forward paths (s + 1)2
= 4 (abdfg, ahfg, aklfg, akmg)
Y (s ) s
Number of loops = 4 (c, de, lfn, mn) T.F. = =
X(s ) (s + 1)2
2. (b)
SFG : 7. (a)

G d2 y dy du
2
+ = + 2u
dt dt dt
1 H 1
w(s ) y(s) s2Y(s) + sY(s) = sU(s) + 2U(s)
H
Y (s) s+2
= 2
G U (s) (s + s)
G + GHG + G + GHG
T.F. = 8. (c)
1 [GH GH ]
Taking (LT) on both sides,
(4 forward paths are 3)
1 1 24
2G (s2 + 6s + 5) X(s) = 12 =
= s s+2 s (s + 2)
1 GH
24
X(s) =
3. (b) s(s + 2) (s + 1) (s + 5)
Signal flow graph is used to find the transfer Response at t
function between the output and input node. Using final value theorem,
Lt x(t ) Lt sX(s)
4. (a) t = s 0

C (s) 2(s 1) s × 24
= = Lt
1 /s (s + 2) (s + 1) s 0 s(s + 1) (s + 2) (s + 5)

= 2.4
1 3 4
C(s) = + +
s s+2 s+1 9. (b)
C(t) = –1 – 3e–2t + 4e–t Natural ratio constant of the response depends
only on poles of the system,
5. (a)
Number of foward paths = 1 (G1G2G3) C (s) 1
T(s) = =
Number of loops = 2 (–G1G2H1, –G2G3H2) R(s ) s 2 + s + 1
Number of non-touching loops = 0 2 18

C (s) G1G2 G3 18 1
T.F. = = = 2
=
R(s) 1 + G1G2 H 1 + G2 G3 H 2 18s + 9s + 1 (6s + 1) (3s + 1)

1
6. (c) This is in the form
(1 + sT1 ) (1 + sT2 )
1
X(s) = L u(t ) = T1, T2 = 6 sec, 3 sec.
s
GATE Previous Years Solved Paper 15

10. (b) 11. (c)


Integrator are represented as 1/s in s-domain.
(s ) T.F. E(s )
9
(t) = rotor displacement in radians
– y ( t) d
u(t) +

1/s 2 +

1/s (t ) = = angular speed in rad/sec.
dt
Output voltage,
3 12
d
As per the block diagram, the corresponding e(t) = Kt (t ) = Kt
dt
singal flow graph is drawn.
Taking Laplace transform on both sides,
–9 E(s) = Kt s (s)
E(s)
= Kt s
1 1/s 2 1/s 1 ( s)
u(s) y (s )
12. (b)
–3 –12 Using signal flow graph,
2 1/s 1/s
One forward path, P1 = 1
s2 R(s) C(s )

The individual loos pare,


1
3 12
L1 = , L2 =
s 3 1
18 Three forward paths:
and L3 =
s2
11 1
L1 and L2 are non-touching loops, P1 = =
s s s2
36
L1L2 = 1 1
s2 P2 = 1 = and P3 = 1
s s
The loops touches the forward path 1= 1.
The numbmer of individual loop = 0
The graph determinant is,
So, graph determinant = = 1
= 1 – (L1 + L2 + L3) + L1L2
and 1 = 2= 3=1
3 12 18 36 Applying Mason’s gain formula,
= 1+ + + +
s s s2 s2
C (s)
G(s) =
Applying Mason’s gain formula, R(s )
Y (s) P1 1 P1 1 + P2 2 + P3 3
G(s) = = =
U (s)

2 / s2 1
1+
1
1+1+1
= 3 12 18 36 2 s2 + s + 1
s s
1+ + + + = =
s s s2 s2 1 s2
2 2
= 2
= 13. (d)
s + 15s + 54 (s + 9) (s + 6)
Transfer function of a system is the unit impulse
1 response of the system.
=
s s
27 1 + 1+
9 6
16 Electronics Engineering Control Systems

C(s) Using Mason’s gain formula,


Transfer function = = L (12.5e 6t sin 8t )
R(s) Y (s ) P1 1 + P2 2
=
8 R(s )
= 12.5 ×
( s + 6)2 + 82 2 6
×1+ ×1
100 s+2 s(s + 2)
=
= s(s + 2) + 6
(s + 6)2 + 82
s(s + 2)
when input is unit step,
1 Y (s ) 6 2s
R(s) = =
s R(s ) s2 + 2 s + 6
1 100 For unit step unit,
C(s) =
s (s + 6)2 + 82 1
R(s) =
Steady-state value of response, using final value s
theorem, 6 2s
Csteady-state = Lt C (t ) = Lt sC(s) Y(s) = R(s ) 2
f s 0 s + 2s + 6
= Error = E(s) = R(s) – Y(s)

1 100 6 2s
Lt s =1 = R(s ) R(s ) 2
s 0 s (s + 6)2 + 82 s + 2s + 6

6 2s
14. (c) = R(s ) 1 2
s + 2s + 6
Using signal flow graph,
1 s 2 + 4s
–1 E(s) =
s s2 + 2s + 6
2
1 1 3/s s+2 1
Steady state value of error, using final value
Y(s )
theorem,
R(s)
ess = Lt sE(s)
s 0
–1
Forward path gains, 1 s 2 + 4s
= Lt s
s 0 s s2 + 2s + 6
2 2
P1 = ( 1) × =
s+2 s+2 s 2 + 4s
= Lt =0
3 2
=
6 s 0 s 2 + 2s + 6
and P2 =
s s + 1 s (s + 2)
Individual loop, 15. (d)
The block-diagram can be redrawn as,
3 2 6
L1 = 1× × =
s s + 2 s(s + 2) 2
+ +
c1 b1
Loop touches forward paths, therefore,
1 = 1 and 2 = 1
3 4
D = 1 – L1 1 + +
5 6
R(s) c1 1/s 1/s P
+ –
6 s (s + 2) + 6 1 C(s )
= 1+ =
s (s + 2) s(s + 2) b0 a0 a1
GATE Previous Years Solved Paper 17

Signal flow graph of the block-diagram, +


X Y P C(s )
+
2
c1 b1
1 Z
1 3 4 1/s 5 1
C (s) xyP
R(s ) c0 1/s P C(s ) = 1 yzP ...(ii)
R(s )
–a1
Comparing equation (i) and (ii), we get
–a0 c0 + c1s
xy = 2
b0
s + a1s + a0
b0 + b1 s
There are two forward paths: yz = 2
s + a1s + a0
1 1 c0 P
P1 = 1 + c0 × × × P = 2 Hence option (d) is correct.
s s s
1 c P 16. (d)
P2 = 1 × c1 × 1 × × P = 1
s s Taking (L.T.) on both sides,
These are four individual loops, Y(s) (s + 1) = 1
1 a1 1
L1 = a1 × = Y(s) =
s s s+1
1 1 a0 Taking inverse Laplace transform,
L2 = × × a0 =
s s s2 y(t) = e–t u(t)
1 1 b P
L3 = × × P × b0 = 02 17. (c)
s s s
Transfer function of system is impulse response
1 b P of the system with zero initial conditions.
L4 = b1 × P × = 1
s s Transfer function,
All the loops touch forwad paths, H(s) = L(e–t + e–2t)
1 = 2=1 1 1
= 1 – (L1 + L2 + L3 + L4) = +
s+1 s+2
a0 a1 b0 P b1 P
= 1+ 2
+ 2 C(s) 1 1
s s s s H(s) = = +
R(s ) s+1 s+2
Using Mason’s gain formula,
C (s) P1 + P2 1
=
1 2 R(s) = (step input)
R(s ) s s

c0 P c1 P 1 1 1
+ C(s) = R(s) H (s) = +
s s+1 s+2
s2 s
=
a a b0 P b1 P 1 1
1 + 1 + 20 2 = +
s s s s s(s + 1) s(s + 2)
C (s) c0 P + c1 Ps
= 2 1 1 1 1 1
R(s ) s + ( a0 b1 ) s + ( a0 b0 P ) C(s) = +
s s+1 2 s s+2
C (s) P(c0 + c1s )/(s 2 + a1s + a0 ) 1.5 1 0.5
= ...(i) =
R(s ) 1 (b0 + b1 s )P /(s 2 + a1s + a0 ) s s+1 s+2
18 Electronics Engineering Control Systems

Response = C(t) = L–1[C(s)] Forward paths are,

1 1.5 1 0.5 h0 h1
= L P1 = 2
, P2 =
s s+1 s+2 s s
C(t) = (1.5 – e–t – 0.5e–2t) u(t) Individual loops are:
a1 a0
18. (d) L1 = , L2
s s2
100 µF Non-touching loops = zero
+ + 1 = 1
10 k a1 a s + a1
V1(s) V2(s) 2 = 1 = 1+ 1 =
s s s
100 µF
– – a1 a0 s2 + a1s + a0
= 1 =
s s2 s2
1
R+ C(s) P1 + P2
V2 (s) 1 + RCs
Cs 1 2
= = G(s) = =
V1 (s) 1 1 2 + RCs U (s)
+R+
Cs Cs
h0 h1 s + a1
×1+ ×
1 + 10 × 10 3 × 100 × 10 6 s
1+s s 2 s s
= 3 6
= =
2 + 10 × 10 × 100 × 10 s 2 + s s 2 + a1s + a0
19. (a) s2

Using Mason’s gain formulae,


h0 + h1 (s + a1 )
= 1 – [–2s–1 – 2s–2 – 4 – 4s–1] =
s2 + a1s + a0
2 2 4
= 1+ + +4+ Given, h 1 = b1 and h0 = b0 – b1a1
s s2 s
(b0 b1 a1 ) + b1 (s + a1 )
5s 2 + 6 s + 2 Thus, G(s) =
= (s 2 + a1s + a0 )
s2
2 1 b1s + b0
P1 = s = 2 = 2
s s + a1s + a0
1 1
P2 = s = 21. (b)
s
1 = 1, 2=2 X1(s) X2(s)
1
1
×1+ ×1
Y (s) Pk k s 2 s 1 G1 G2
= = 2 Y(s )
U (s) 5s + 6s + 2
s2
Y (s) s+1
= 2
U (s) 5s + 6 s + 2 –1 –1

20. (c) Y (s) G2


=
Using Mason gain formula: X 2 (s ) x 1 ( G1G2 + G1 )
1 (s ) = 0
Transfer function,
G2
C(s) Pk =
G(s) = =
k 1 + G1 (G2 + 1)
U (s)
GATE Previous Years Solved Paper 19

22. (c) 25. Sol.


G1 s+1
G(s) =
s+1
–H
System output,
X(s) Y(s )
G2 1 s+1 1
Y(s) = G(s ) =
s s+1 s
–H
1 2
Y (s ) G1 + G2 G1 + G2 =
s s+1
= =
X( s ) 1 ( G1 H G2 H 1 ) 1 + G1 H + G2 H y(t) = u(t) – 2e–t u(t)
y(1.5) = 1 – 2e–1.5
23. (a)
= 1 – 0.446 = 0.554
dx1
= 2x1 + x2 + u ...(i) 26. (b)
dt
dx2 d2 y dy dx
= –2x1 + 4 ...(ii) +7 + 10 y = 4x + 5
dt 2 dt dt
dt
From equation (i),
(s2 + 7s + 10) Y(s) = (4 + 5s) X(s)
sX1 = 2X1 + X2 + u
Y (s ) 5s + 4
sX2 = –2X1 + u = 2
X( s ) s + 7 s + 10
2X1 + u
X2 = Impulse response = L –1 (Transfer function)
s
1 5s + 4
2 X1 + u = L
sX1 = 2 X1 + +4 (s + 2) (s + 5)
s
2 X1 4 1 2 7
sX1 2 X1 + = +4 = L +
s 5 s+2 s+5
2 1 = –2e–2t u(t) + 7e–5t u(t)
X1 s 2 + = u 1+
s s
27. (d)
1 Simplying given signal flow graph:
1+
X1 s Step-1:
=
4 2
s 2+ d
s
1 a 1 cd 1
1
3 1+
y 3 X1 s
= = e
u u 2
s 2+
s Step-2:
y 3( s + 1)
= 2 d
u a
( s 2 s + 2) 1 1 cd 1

24. (a)
Transfer function, e

K1
[1]
Y (s) LJs 2 K1
= = 2
U1 ( s) R K1K 2 LJs + RJs + K 1K 2
1 2
Ls LJs
2 Compensators and
Controllers
ELECTRO NICS EN GINEERIN G Q.5 The transfer function of a phase lead controller

(GATE Previous Years Solved Papers) 1+ 3Ts


is . The maximum value of phase
1 + Ts
Q.1 The transfer function of a simple RC network
provided by this controller is
functioning as a controller is
(a) 90° (b) 60°
s + z1
Gc (s) = (c) 45° (d) 30°
s + p1
[EC-1998 : 1 Mark]
The condition for the RC network to act as a
phase lead controller is Q.6 A PD controller is used to compensate a system.
Compared to the uncompensated system, the
(a) p1 < z1 (b) p1 = 0
compensated system has
(c) p1 = z1 (d) p1 > 0
(a) a higher type number
[EC-1990 : 2 Marks]
(b) reduced damping
Q.2 A process with open-loop mode of (c) higher noise amplification
(d) larger transient overshoot
Ke sTD
G(s ) = is controlled by a PID controller. [EC-2003 : 1 Mark]
s+1
For this process
K
(a) the integral mode improves transient Q.7 A double integrator plant, G(s ) = , H(s) = 1 is
s2
performance. to be compensated to achieve the damping ratio
(b) the integral mode improves steady-state = 0.5 and an undamped natural frequency,
performance.
n = 5 rad/s. Which one of the following
(c) the derivative mode improve transient compensator Ge(s) will be suitable?
performance. s+3 s + 9.9
(d) the derivative mode improves steady-state (a) (b)
s + 9.9 s+3
performance.
s 6 s+6
[EC-1992 : 2 Marks] (c) (d)
s + 8.33 s
Q.3 Tachometer feedback in a dc position control [EC-2005 : 2 Marks]
system enhances stability (T/F). Q.8 The tranfser function of a phase-lead
[EC-1994 : 1 Mark] compensator is given by
Q.4 The transfer function of a tachometer is of the 1 + 3Ts
Ge ( s ) = where, T > 0
form 1 + Ts

K The maximum phase-shift provided by such a


(a) Ks (b) compensator is
s
K K (a) (b)
(c) (d) 2 3
(s + 1) s ( s + 1)
(c) (d)
[EC-1998 : 1 Mark] 4 6
[EC-2006 : 2 Marks]
GATE Previous Years Solved Paper 21

Q.9 A control system with a PD controller is shown C2


in the figure. If the velocity error constant
KV = 1000 and the damping ratio = 0.5, then R.
the values of KP and KD are
R2
+ 100
r KP + KDS y
s ( s + 10) Group-II

1. PID controller
2. Lead compensator
3. Lag compensator
(a) KP = 100, KD = 0.09
(a) Q-1, R-2 (b) Q-1, R-3
(b) KP = 100, KD = 0.9
(c) Q-2, R-3 (d) Q-3, R-2
(c) KP = 10, KD = 0.09
[EC-2008 : 2 Marks]
(d) KP = 10, KD = 0.9
[EC-2007 : 2 Marks] Q.12 The magnitude plot of a rational transfer
function G(s) with real coefficients is shown
Q.10 The open-loop transfer function of a plant is
below. Which of the following compensators
1 has such a magnitude plot?
given as G(s ) = . If the plant is operated
2
s 1
G( j )
in a unity feedback configuration, then the lead
compensator that can stabilize this control 20 dB
system is
10( s 1) 10( s + 4) log
(a) (b)
(s + 2) (s + 2)
10( s + 2) 2(s + 2)
(c) (d) –40 dB
(s + 10) (s + 10)
[EC-2007 : 2 Marks] (a) Lead compensator
Q.11 Group I gives two possible choice for the (b) Lag compensator
impedance Z in the diagram. The circuit (c) PID compensator
elements in Z satisfy the condition R2C2 > R1C1. (d) Lead-Lag compensator
The transfer function Vo/Vi represents a kind of [EC-2009 : 1 Mark]
controller. Match the impedance in Group I with
Q.13 A unity negative feedback closed-loop system
the types of controllers in Group II:
has a plant with the transfer function
C1 Z
1
G(s ) = 2
and a controller G2(s) in the
s + 2s + 2
Vi –
R1 Vo feed forward path. For a unit step input, the
+
transfer function of the controller that gives
minimum steady state error is
s+1
Group-I (a) Gc (s ) =
s+2
R2 C2
Q. s+2
(b) Gc (s ) =
s+1
22 Electronics Engineering Control Systems

(s + 1) (s + 4) Q.18 The transfer function of a first order controller


(c) Gc (s ) =
(s + 2) (s + 3) is given as,
K (s + a)
2 Gc (s ) =
(d) Gc (s ) = 1 + + 3s s+b
s
where, K is a and b are positive real numbers.
[EC-2010 : 2 Marks]
The condition for this controller to act as a phase
Linked Answer Questions (14 and 15): lead compensator is
The transfer function of a compensator is given as, (a) a < b (b) a > b
s+a (c) K < ab (d) K > ab
Gc (s ) =
s+b [EC-2015 : 1 Mark]
Q.14 Gc(s) is a lead compensator if Q.19 Which of the following can be the pole-zero
(a) a = 1, b = 2 (b) a = 3, b = 2 configuration of a phase lag controller (lag
(c) a = –3, b = –1 (d) a = 3, b = 1 compensator)?
[EC-2012 : 2 Marks] j

Q.15 The phase of the above lead compensator is s-plane Pole


maximum at Zero
(a)
(a) 2 rad/s (b) 3 rad/s
1
(c) 6 rad/s (d) rad/s
3
[EC-2012 : 2 Marks]
j
Q.16 For the following feedback system,
s-plane Pole
1
G(s) = Zero
(s + 1) (s + 2) (b)
The 2% settling time of the step response is
required to be less than 2 seconds.
r + C(s ) G(s ) y

j

Which one of the following compensators C(s) s-plane Pole


achieves this? Zero
(c)
1 0.03
(a) 3 (b) 5 +1
s+5 s
s+8
(c) 2(s + 4) (d) 4
s+3
j
[EC-2014 : 2 Marks]
s-plane Pole
Q.17 A lead compensator network includes a parallel Zero
combination of R and C in the feed forward path. (d)
If the transfer function of the compensator is
s+2
Gc (s ) =
s+4
The value of RC is ______ . [EC-2017 : 1 Mark]
[EC-2015 : 2 Marks]
GATE Previous Years Solved Paper 23

Q.20 Which of the following statements is incorrect? j


(a) Lead compensator is used to reduce the
settling time.
(b) Lag compensator is used to reduce the (c)
steady-state error.
(c) Lead compensator may increase the order
of a system.
(d) Lag compensator always stabilizes an j
unstable system.
[EC-2017 : 1 Mark]
(d)
Q.21 A unity feedback system that, uses proportional
integral (PI) control is shown in the figure.

KI 2
X(s) + KP + Y(s )
– s s 3 + 4s 2 + 5s + 2
[EE-1994 : 1 Mark]

Q.2 Introduction of integral action in the forward


The stability of the overall system is controlled
path of a unity feedback system results in a
by tuning the PI control parameters KP and KI.
(a) marginally stability
The maximum value of KI that can be chosen so
as to keep the overall system stable or, in the (b) system with no steady-state error
worst case, marginally stable (Rounded off to (c) system with increased stability margin
three decimal places) is ______ . (d) system with better speed of response
[EC-2021 : 2 Marks] [EE-1997 : 1 Mark]

Q.3 A differentiator has transfer function whose


ELECTRICAL EN GINEERIN G
(a) phase increases linearly with frequency
(GATE Previous Years Solved Papers)
(b) amplitude remains constant
Q.1 The pole zero configuration of a phase lead (c) amplitude increases linearly with
compensator is given by frequency
j (d) amplitude decreases linearly with
frequency
[EE-1997 : 1 Mark]
(a)
Q.4 The phase lead configurations used to
(a) increase rise time and decrease overshoot.
(b) decrease both rise time and overshoot.

j
(c) increase both rise time and overshoot.
(d) decrease rise time and increase overshoot.
[EE-1998 : 1 Mark]
(b)
(0.5s + 1)
Q.5 G(s) = maximum phase lead of the
(0.05s + 1)

compensator is
24 Electronics Engineering Control Systems

(a) 52 deg at 4 rad/sec (b) C1 is lag compensator and C2 is a lead


(b) 52 deg at 10 rad/sec compensator.
(c) 55 deg at 12 rad/sec. (c) Both C1 and C2 are lead compensator.
(d) None of these (d) Both C1 and C2 are lag compensator.
[EE-2000 : 2 Marks] [EE-2008 : 2 Marks]

Q.6 A lead compensator used for a closed-loop Statement for Linked Answer Questions (9 and 10):
controller has the following transfer function, The transfer function of a compensator is given as,
s+a
Gc (s ) =
s s+b
K 1+
a
. For such a lead compensator Q.9 Gc(s) is a lead compensator if
s
1+
b (a) a = 1, b = 2 (b) a = 3, b = 2
(c) a = –3, b = –1 (d) a = 3, b = 1
(a) a < b (b) b < a [EE-2012 : 2 Marks]
(c) a > Kb (d) a < Kb
Q.10 The phase of the above lead compensator is
[EE-2003 : 1 Mark]
maximum at
900 (a) 2 rad/sec. (b) 3 rad/sec.
Q.7 The system is to be such that its
s (s + 1) (s + 9) 1
(c) 6 rad/sec. (d) rad/sec.
gain crossover frequency becomes same as its 3
uncompensated phase crossover frequency and [EE-2012 : 2 Marks]
provides a 45° phase margin. To achieve this,
one may use Q.11 For the network shown in the figure below, the
(a) a lag compensator that provides an frequency (in rad/sec) at which the maximum
attenuation of 20 dB and a phase lag of 45° phase lag occurs is ________ .

at the frequency of 3 3 rad/sec.


9
(b) a lead compensator that provides an 1
attenuation of 20 dB and a phase lag of 45° Vin Vo

at the frequency of 3 rad/sec. 1F


(c) a lag-lead compensator that provides an
attenuation of 20 dB and a phase lag of 45° [EE-2016 : 1 Mark]
at the frequency of 3 rad/sec.
Q.12 The transfer function C(s) of a compensator is
(d) a lag-lead compensator that provides an
given below,
attenuation of 20 dB and a phase lead of
s s
45° at the frequency of 3 rad/sec. 1+ 1+
0.1 100
C (s) =
Q.8 The transfer function of two compensators are s
(1 + s ) 1 +
given below, 10
10(s + 1) s + 10 The frequency range in which the phase (lead)
C1 = , C2 =
(s + 10) 10( s + 1) introduced by the compensator reaches the
Which one of the following statements is maximum is
correct? (a) 0.1 < <1 (b) 1 < < 10
(a) C1 is lead compensator and C2 is a lag (c) 1.0 < < 100 (d) > 100
compensator. [EE-2017 : 1 Mark]
GATE Previous Years Solved Paper 25

Q.13 The transfer function of a phase lead 3


compensator is given by (a) (b) 3T
T2
1 1
3 s+
3T (c) (d) 3T 2
D(s ) = 3T 2
1
s+ [EE-2019 : 2 Marks]
T
The frequency (in rad/sec), at which D(j ) is
maximum, is

Electronics & Electrical Engineering


GATE Previous Years Solved Paper

A n swe rs & Expl a n a t i o n s

Answers
EC Compensators and Controllers

1. (d) 2. (b, c) 3. (True) 4. (a) 5. (d) 6. (c) 7. (a) 8. (d)

9. (b) 10. (c) 11. (b) 12. (d) 13. (d) 14. (a) 15. (a) 16. (c)

17. (0.5) 18. (a) 19. (a) 20. (d) 21. (3.125)

Solutions Compensators and Controllers


EC
1. (d) 3. Sol.
(True)
1 1
= tan tan The tachometer feedback is derivative feedback.
z1 p1
So, tachometer adds zero at origin.
For phase lead controller,
Hence, type decreases and stability is improved
>0
(True).
1 1
tan tan >0 4. (a)
z1 p1
Tachometer is basically a differentiator. So, the
1 1 transfer function of a tachometer is of the
tan > tan
z1 p1 form Ks.
p 1 > z1
5. (d)
2. (b, c) For lead compensator,
The integral mode improves steady-state (1 + T s ) 1 + 3Ts
performance and the derivative mode improves T.F. = =
1+ T s 1 + Ts
the transient performance.
So, T = 3T
26 Electronics Engineering Control Systems

T =T 8. (d)
3T = T Maximum phase shift,
1 m = Ge(s)
=
3 = tan–1 3 T – tan–1 t
For maximum phase shift,
1 1 1 1 1/3
= sin = sin d
max 1+ 1 + 1/3 =0
d
1 2 /3 1 3T T
max = sin = sin 1 = 30° =
4 /3 2 1 + (3T ) 2
1 + (T )2
3[1 + (T )2] = 1 + (3T )2
6. (c)
3 + 3T2 2 = 1 + 9T2 2
• A PD controller does not effect the type of 2 = 6( T)2
the system. 1
( T)2 =
• It reduces the overshoot and increases the 3
damping. 1
T=
• It increases the bandwidth therefore SNR 3
decreases. 1 1 1 1
max = tan 3× tan
• System become more prone to noise. 3 3

7. (a) = =
3 6 6
(–2.5, 4.33j)
9. (b)
K v = lim sG(s) H (s )
5 s 0

(K P + KD s )100
60° 1000 = lim s ×
s 0 s (s + 10)
= 0.5 K P = 100
cos–1 0.5 = 60° Now characteristic equation (1 + G(s) H(s)) = 0
= 60°
(K P + KD s )100
1+ =0
K s (s + 10)
G(s) =
s 2 s = 2.5 + 4.33 j Putting, K P = 100
s2 + 10s + 104 + 100KDs = 0
1 4.33
= 2 tan ; 120° s2 + (10 + 100KD) s + 104 = 0
2.5
Comparing with standard second order
For compensated system =180 – 120; 60°
equation.
(b) and (d) are lag network and for
compensating lag network K/s2, a lead network i.e., s2 + 2 n + 2
n =0
is required.
So, = n 100
Putting, s = –2.5 + 4.33j in (a) gives, 2 n= 10 + 100KD
K (s + 3) 0.5 + j 4.33 Given, = 0.5
2 = = 53°; 60°
s (s + 9.9) 7.4 + j 4.33 2 × 0.5 × 100 = 10 + 100KD
Option (a) is the correct answer. KD = 0.9
GATE Previous Years Solved Paper 27

10. (c) s+2 1


Taking, Gc(s) = , ess =
Lead compensator is required for stability. s+1 3
Controllers given in option (a) and (b) are not (s + 1) (s + 4) 3
Taking, Gc(s) = , ess =
lead compensator. From option (c) and (d), with (s + 2) (s + 3) 5
option (d) system becomes unstable hence option 2
Taking, Gc(s) = 1 + + 3s , ess = 0
(c) is correct. s

11. (b) 14. (a)


Vi ( R1C 1s + 1)1 Vo s+a
= Gc(s) =
R1 Z s+b
Vo Z( R1C1s + 1) 1 1
= Phase, P = tan tan
Vi R1 a b

R2C2 s + 1 For lead comparators phase must be +ve.


In case of Q, Z=
C2 s For this > a<b
a b
R2
In case of R, Z= So, a = 1, b = 2
R2C2 s + 1
Considering Q, 15. (a)

Vo ( R1C1s + 1) ( R2C 2 s + 1) For phase to be maximum,


= P
Vi R1 C2 s =0
Considering R,
1/a 1 /b
Vo ( R1C1s + 1) R2 =0
= 2 2
Vi R1 ( R2C 2 s + 1) 1+ 2 1+ 2
a b
Q Given that, R2C2 > R1C1
Considering R, controller is lag 1 1/ b
2 2 =0
compensator and Considering Q, Controller is 1+
1+
PID controller. 4
1 2
13. (d) =0
2 2
1+ +4
Steady state error, 4+4–2–2 2 = 0
s R(s ) 2 = 4
ess = lim
s 0 1 + G(s ) Gc (s)
= 2 rad/sec
r(t) = u(t)
1 16. (c)
R(s) =
s
Given open-loop transfer function is
1
s 1
ess = lim s G(s) =
(s + 1) (s + 2)
s 0 1 + G( s ) Gc ( s )
T(s) =
1
ess = lim G(s) 1
s 0 1 + G(s ) Gc (s ) =
1 + G(s) (s + 1) (s + 2) + 1
s+1 2
Taking, Gc(s) = , ess = 1
s+2 3 T(s) = 2 ...(i)
s + 3s + 3
28 Electronics Engineering Control Systems

Comparing equation (i) with standard transfer 18. (a)


function, For phase lead compensator,
3 (1 + s )
n =
= 1.5 Gc(s) = ; <1
2 (1 + s )
2% settling time,
1
4 4 Here, =
= = = 2.67 > 2 sec a
s
n 1.5
1
Thus, in order to make settling time ( s) less than and =
b
2 sec, PD controller should be used. Hence, a
option (c) is the correct choice. or, = < 1 or a < b
b
Where, C(s) = 2(s + 4)
New transfer function = T (G) 19. (a)
C (s ) G(s ) Phase lag controller transfer function is,
=
1 + C (s) G(s ) s+Z
Gc(s) = ; Z > P
2 (s + 4) s+P
= 2 j
s + 3s + 2 + 2 s + 8
2 (s + 4)
or, T (s) = 2
s + 5s + 10
4 4 4
s = = = = 1.6
n 5/2 2.5 –Z –P

17. Sol. 20. (d)


s+2 In case of high type systems lag compensator
Gc(s) = ...(i)
s+4 fails to give stability.
For lead compensator,
21. (3.125)
C1
SK P + K I 2
G(s) = 3 2
S S + 4S + 5S + 2
R1 2(SK P + K I )
=
R2 S(S + 4S 2 + 5S + 2)
3

CE
1 + G(S ) = 0
CE
1+s S 4 + 4S 3 + 5S 2 + S(2 + 2 K P ) + 2 K I = 0
T.F. = ...(ii)
1+ s From necessary condition KP > –1, KI > 0
where, = lead time constant = R1C
R2 S4 1 5 2K I
and = S 3
4 2 + 2K P
R1 + R2
Comparing equation (i) and (ii), we get [20 (2 + 2K P )]
S2 2K I
4
1 1 [(18 2 K P ) (2 + 2 K P )]
=
and = S1 8K I
2 4 (18 2 K P )
1
or, = S0 2K I
2
RC time constant = 0.5
GATE Previous Years Solved Paper 29

20 2 2 K P 36 + 32 K P 4K 22
For stability, >0 KP < 9 0 < KI <
4 32
–1 < KP < 9
For stability: dK I
To get maximum value of K I =0
dK P
(18 2K P ) (2 + 2 K P )
8K I > 0 dK I
4 = 0 = [0 + 32 – 8KP] = 0
dK P
(18 – 2KP) [2 + 2KP] – 32KI > 0
KP = 4
36 + 36K P 4K P 4K P2 32 K I > 0
36 + 32(4) 4(4)2
For KP = 4, KI =
(36 + 36K P 4K P2 ) > 32 K I 32
= 3.125
36 + 32 K P 4K22 KImax = 3.125
KI <
32

Answers
EE Compensators and Controllers

1. (b) 2. (b) 3. (c) 4. (b) 5. (d) 6. (a) 7. (d) 8. (a)

9. (a) 10. (a) 11. (0.316) 12. (a) 13. (c)

Solutions
EE Compensators and Controllers

1. (b) 5. (d)

In phase lead compensator P > Z . 0.5s + 1


G(s) =
0.05s + 1
2. (b) On comparing with,
Integral controller is widely used to eliminate 1 + aTs
G(s) =
steady-state error. 1 + Ts
we get, T = 0.05 and a = 10
3. (c)
1 a 1
= sin = 55°
X(s) Differentiator Y(s ) = sX(s ) n a+1
Y (s ) 1
H(s) = =s and m = = 6.32 rad/sec.
X(s ) T a
Frequency response, H (j ) = j
6. (a)
Hence, phase is constant at 90° for all but
amplitude increases linearly with frequency. s
k 1+
a
Transfer function =
4. (b) s
1+
Lead compensator increases bandwidth, hence b
decreases rise time. As it improve damping, so Zero of TF = –a
overshoot also decreases. Poles of TF = –b
30 Electronics Engineering Control Systems

For a lead compensator, the zero is nearer to Phase margin = 180° + T ( j ) = ( gc )2


origin as compared to pole, hence the effect of
the zero is dominant, therefore, the lead 45° = 180° + T ( j ) = ( pc )2
compensator when introduced in series with
At ( gc)2 = 3 rad/sec.
forward path of the transfer function the phase
Phase angle of T(j ) is –135° and phase of
shift is increased.
uncompensated system is –180° at 3 rad/sec.
Therefore, the compensator provides phase lead
of 45° at the frequency of 3 rad/sec.
–b –a Let X dB is the gain provided by the
Pole Zero compensator, so at gain cross frequency,

T ( j ) com = 1 or 0 dB.

So, from pole-zero configuration of the Gain of uncompensated system = T ( j ) un-com


compensator a < b. 100
=
2
7. (d) 2
1+ 1+
Let uncompensated system, 9
900 T ( j ) un-com in dB
T(s) =
s (s + 1) (s + 9)
2
Phase crossover frequency of uncompensated = 40 20 log 20 log 1 +
system = ( pc), at this frequency phase of T(j ) 2
is –180°. 20 log 1 +
9
Put, s = j in T(s)
Gain of compensated system,
900
T(j ) = T ( j ) com = X + T ( j )un-com
j ( + 1) ( + 9)

T(j ) = 90° tan 1


tan 1 T ( j ) com must be zero at gain cross frequency
9
( gc)2.
At ( pc)1,
T(j ) = –180° T ( j gc )2
com
1 1
( pc )1
–180° = 90° tan ( pc )1 tan = X + 40 – 20 log( 20 log 1 + ( 2
gc )2
9 gc)2

( pc )1 ( 2
( + gc )
pc )1 20 log 1 + =0
1 9 2
+90° = tan 2 g
( pc )1
1 2
9 3
X + 40 20 log 3 20 log 1 + 32 20 log 1 +
2 9
( pc )1
1 =0 =0
9 X = –20 dB
( pc)1 = 3 rad/sec. So, the compensator provides an attenuation of
Gain cross frequency of compensated system, 20 dB. Hence option (d) is correct.
( gc ) 2 = phase cross frequency of
uncompensated system, ( pc)1. 8. (a)
( gc)2 = ( gc)1 = 3 rad/sec. 10( s + 1)
C1 =
(s + 10)
GATE Previous Years Solved Paper 31

Zero at s = –1 11. Sol.


Pole at s = –10 Assuming, R1 = 9 , R2 = 1
we can write,
1
Vo (s) R2 + 1 + R2 Cs
= sC =
Vi (s) 1 1 + ( R1 + R2 ) Cs
–10 –1 R1 + R2 +
sC
s-plane
1 + R2 Cs
=
As zero is closer origin, zero dominates pole. R1 + R2
1+ R2 Cs
Hence, C1 is lead compensator, R2
Let, R2 C = T
s + 10
C2 = R1 + R2
10( s + 1) =
R2
Zero at s = –10
Pole at s = –1 Vo (s) 1 + Ts
Hence, =
Vi (s) (1 + Ts )
which represent a lag compensator.
Q Here, T = R2C = 1 sec.
1+9
–10 –1 = 10 =
1
Maximum phase lag occurs at frequency,
As pole is closer to origin, pole dominates zero. 1 1
Hence, C2 is lag compensator. n = = = 0.316 rad/sec.
T 1 10
9. (a)
12. (a)
s+a Pole zero diagram of compensator transfer
Gc(s) =
s+b
function is shown below.
j +a
Gc(j ) =
j +b
1 1
Gc(j ) = tan tan
–100 –10 –1 –0.1
a b
Maximum phase lead is between 0.1 and 1,
1 a b 0.1 < < 1
= tan 2
1+ 13. (c)
ab
For Gc(s) to be a lead compensator, 1 + 3Ts
T(s) =
Gc(j ) > 0 1 + Ts
Frequency at which T(j ) is maximum,
> 1
a b ( m) =
b>a T
Option (a) satisfies the above equation. 1
= =3
1 /3
10. (a)
1 1
= =
m = c1 × c2 = 1 × 2 = 2 rad/sec.
m
T 3 3T 2
3 Time Response Analysis

ELECTRO NICS EN GINEERIN G If the input to the system is a unit ramp, the
steady-state error will be
(GATE Previous Years Solved Papers)
(a) 0 (b) 0.5
Q.1 The unity feedback system shown in figure has (c) 2 (d) Infinity
[EC-1991 : 2 Marks]
K
R(s ) + C(s ) Q.5 The poles of a continuous time oscillator are
– s (s + 10)
________ .
[EC-1994 : 1 Mark]

Q.6 The response of an LCR circuit to a step input is


(a) zero steady-state position error.
(a) over damped
(b) zero steady-state velocity error.
(b) critically damped
(c) steady-state position error K/10 units.
(c) oscillatory
(d) steady-state velocity error K/10 units.
If the transfer function has
[EC-1987 : 2 Marks] 1. poles on the negative real axis
Q.2 The steady-state error of a stable ‘type 0’ unity 2. poles on the imaginary axis
feedback systm for a unit step function is 3. multiple poles on the positive real axis
1 4. Multiple poles on the negative real axis
(a) 0 (b) [EC-1994 : 2 Marks]
1 + KP
1 Q.7 Match the following codes with List-I with
(c) (d)
KP List-II:
[EC-1990 : 2 Marks] List-I
(a) Very low response at very high frequencies
Q.3 A second order system has a transfer function
(b) Very shoot
given by
(c) Synchro-control transformer output
25 List-II
G( s ) =
s 2 + 8s + 25 (1) Low pass systems
If the system, initially at rest, is subjected to a (2) Velocity damping
unit step iinput at t = 0, the second peak in the (3) Natural frequency
response will occur at (4) Phase sensitive modulation
(a) sec. (b) sec. (5) Damping ratio [EC-1994 : 2 Marks]
3
2 Q.8 For a second order system, damping ratio ( ), is
(c) sec. (d) sec. 0 < < 1, the the roots of the characteristic
3 2
polynomial are
[EC-1991 : 2 Marks]
(a) real but not real
Q.4 A unit feedback control system has the open- (b) real and equal
loop transfer function, (c) complex conjugates
4 (1 + 2 s ) (d) imaginary
G( s ) = 2
s + (s + 2) [EC-1995 : 1 Mark]
GATE Previous Years Solved Paper 33

Q.14 Consider a unity feedback control system with


2 (s + 1)
Q.9 If L[ f (t )] = then f(0+) and f( ) are
s2 + 2s + 5 K
open-loop transfer function G(s) = the
given by s (s + 1)
steady-state error of the system due to a unit
(a) 0, 2 respective (b) 2, 0 respectively
step input is
(c) 0, 1 respectively (d) 2/5, 0 respectively
(a) zero (b) K
[Note: ‘L’ stands for ‘Laplace transform of’] (c) 1/K (d) infinite
[EC-1995 : 1 Mark] [EC-1998 : 1 Mark]
Q.10 The step error coefficient of a system Q.15 Tht unit impulse response of a linear time
1 invariant system is the unit step function u(t).
G(s) =
(s + 6) (s + 1) For t > 0, the response of the system to an
with unity feedback is excitation e–at u(t), a > 0 will be
1 (a) ae–at (b) (1/a) (1 – e–at)
(a) (b) (c) a(1 – e–at) (d) 1 – e–at
6
[EC-1998 : 2 Marks]
(c) 0 (d) 1
[EC-1995 : 1 Mark] Q.16 For a second order system with the closed-loop
transfer function,
Q.11 The final value theorem is used to find the
9
(a) steady-state value of the system output. T (s) = 2
s + 4s + 9
(b) intial value of the system output.
the settling time for 2 percent band (in seconds),
(c) transient behaviour of the system output.
is
(d) none of these.
(a) 1.5 (b) 2.0
[EC-1995 : 1 Mark] (c) 3.0 (d) 4.0
[EC-1999 : 1 Mark]
Q.12 If F(s ) = , then the value of lim f (t ),
s2 + 2 t Q.17 If the closed-loop transfer function T(s) of a unity
negative feedback system is given by
{where F(s) is the L[ f(t)}
(a) cannot be determined
an 1 s + an
T (s ) =
(b) is zero s n + a1s n 1
+ ... + an 1 s + an
(c) is unity then the steady-state error for a unit ramp input
(d) is infinite is
[EC-1998 : 1 Mark] an an
(a) (b)
an 1 an 2
Q.13 Consider a feedback control system with loop
transfer function, an 2
(c) (d) zero
K (1 + 0.5s) an 1
G(s ) H (s ) =
s (1 + s ) (1 + 2 s) [EC-1999 : 2 Marks]
The type of the closed-loop system is
Q.18 If the characteristic equation of a closed-loop
(a) zero (b) one system is s2 + 2s + 2 = 0, then the system is
(c) two (d) three (a) overdamped (b) critically damped
[EC-1998 : 1 Mark] (c) underdamped (d) undamped
[EC-2001 : 1 Mark]
34 Electronics Engineering Control Systems

Q.19 The open-loop dc gain of a unity negative


Step response
feedback system with closed-loop transfer 1

s+4
function is
2
s + 7 s + 13

Amplitude
4 (b) 0.5
4
(a) (b)
13 9
(c) 4 (d) 13
[EC-2001 : 2 Marks] 0
0 2 4 6
Time (sec.)
Q.20 Consider a system with the transfer function,
s+6
G( s ) =
Ks 2 + s + 6 2
Step response

Its damping ratio will be 0.5 when the value of


K is 1.5

Amplitude
2
(a) (b) 3 1
6 (c)
1 0.5
(c) (d) 6
6
[EC-2002 : 1 Mark] 0
0 5 10 15 20 25

Q.21 The transfer function of a system is, Time (sec.)

100
G(s) =
(s + 1) (s + 100) Step response
1
For a unit step input to the system the
approximate sesttling time for 2% criterion is
Amplitude

(a) 100 sec. (b) 4 sec.


0.5
(c) 1 sec. (d) 0.01 sec. (d)
[EC-2002 : 2 Marks]

Q.22 A second order system has the transfer function,


0
C (s) 4 0 5 10
= 2 Time (sec.)
R(s) s + 4s + 4
With r(t) as the unit step function, the response [EC-2003 : 2 Marks]
c(t) of the system is represented by
Q.23 A causal system having the transfer function
Step response
1.5 1
H (s ) = is excited with 10 u(t). The time at
s+2
1
Amplitude

which the output reaches 99% of its steady-state


(a)
value is
0.5 (a) 2.7 sec. (b) 2.5 sec.
(c) 2.3 sec. (d) 2.1 sec.
0 [EC-2004 : 2 Marks]
0 2 4 6
Time (sec.)
GATE Previous Years Solved Paper 35

Q.24 A system described by the following differential Q.28 The unit step response of a system starting from
rest is given by
d2 y dy c(t) = 1 – e–2t for t 0
equation 2
+3 + 2 y = x(t ) is initially at
dt dt The transfer function of the system is
rest. For input x(t) = 2u(t), the output y(t) is 1 2
(a) (b)
(a) (1 – 2e–t + e–2t) u(t) 1 + 2s 2+s
(b) (1 + 2e–t – 2e–2t) u(t)
1 2s
(c) (0.5 + e–t + 1.5e–2t) u(t) (c) (d)
2+s 1 + 2s
(d) (0.5 + 2e–t + 2e–2t) u(t)
[EC-2006 : 2 Marks]
[EC-2004 : 2 Marks]
Q.29 The unit impulse response of a system is
Q.25 Despite the presence of negative feedback,
h(t) = e–t, t 0
control systems still have problems of instability
because the For this system, the steady-state value of the
output for unit step input is equal to
(a) components used have non-linearities
(b) dynamic equations of the systems are not (a) –1 (b) 0
known exactly. (c) 1 (d)
(c) mathematical analysis involves [EC-2006 : 2 Marks]
approximations.
Q.30 The transfer function of a plant is
(d) system has large negative phase angle at
high frequencies. 5
T (s) =
[EC-2005 : 1 Mark] ( s + 5) ( s 2 + s + 1)

The second-order approximation of T(s) using


Q.26 In the derivation of expression for peak percent
dominant pole concept is
1 5
overshoot, M p = exp × 100 which (a) (b)
2 (s + 5) (s + 1) (s + 5) (s + 1)
1
5 1
one of the following conditions is not required? (c) 2 (d) 2
s +s+1 s +s+1
(a) System is linear and time invariant.
[EC-2007 : 2 Marks]
(b) The system transfer function has a pair of
complex conjugate poles and no zeroes. Q.31 Step response of a set of three-second order
(c) There is no transportation delay in the underdamped systems all have the same
system. percentage overshoot. Which of the following
(d) The system has zero initial conditions. diagrams represents the poles of three systems?
[EC-2005 : 2 Marks]
j
Q.27 A ramp input applied to an unity feedback
system results in 5% steady-state error. The type
number and zero frequency gain of the system
are respectivley (a)
(a) 1 and 20 (b) 0 and 20
1 1
(c) 0 and (d) 1 and
20 20
[EC-2005 : 2 Marks]
36 Electronics Engineering Control Systems

j Group-I
25 36
P= 2 ; Q= 2
s + 25 s + 20s + 36
36 36
(b) R= 2 ; S= 2
s + 12 s + 36 s + 7 s + 49
Group-II
y(t )
1
j 1.

(c) y(t)

1
2.

t
j
y(t )

3. 1
(d)
t

y(t )
1
[EC-2008 : 1 Mark]
4.
Q.32 A linear, time-invariant, causal continuous time
t
system has a rational transfer function with
simple poles at s = –2 and s = –4 and one simple (a) P-3, Q-1, R-4, S-2
zero at s = –1. A unit step u(t) is applied at the (b) P-3, Q-2, R-4, S-1
input of the system. At steady-state, the output
(c) P-2, Q-1, R-4, S-3
has constant value of 1. The impulse response
(d) P-3, Q-4, R-1, S-2
of this system is
[EC-2008 : 2 Marks]
(a) [exp(–2t) + exp(–4t)] u(t)
(b) [–4 exp(–2t) + 12 exp(–4t) – exp(–t)] u(t) Q.34 The unit step response of an under damped
(c) [–4 exp(–2t) + 12 exp(–4t)] u(t) second order system has steady-state value of
–2. Which one of the following transfer functions
(d) [–0.5 exp(–2t) + 1.5 exp(–4t)] u(t)
has these properties?
[EC-2008 : 2 Marks]
2.24 3.82
(a) (b)
Q.33 Group-I : Lists a set of four transfer function. s 2 + 2.59s + 1.12 s 2 + 1.91 s + 1.91
Group-II : Gives a list of possible step responses 2.24 3.82
y(t). Match the step responses with the (c) 2 (d) 2
s 2.59 s + 1.12 s 1.91 s + 1.91
corresponding transfer function:
[EC-2009 : 2 Marks]
GATE Previous Years Solved Paper 37

Q.35 The differential equation,


(c) change the initial condtion to j 2 y(0) and
2
d y dy
100 2
20 + y = x(t ) the forcing function to j 2 x(t ).
dt dt
describes a system with an input x(t) and an (d) change the initial condtion to –2y(0) and
output y(t). The system, which is initially the forcing function to –2x(t).
relaxed, is excited by a unit step input. The [EC-2013 : 2 Marks]
output y(t) can be represented by the waveform
Q.37 The open-loop transfer function of a dc motor is
y(t) given as,
(s ) 10
(a) =
Va (s ) 1 + 10s
When connected in feedback as shown below,
t
the approximate value of Ka that will reduce the
time constant of the closed-loop system by one
y(t)
hundred times as compared to that of the open-
loop system is
(b)
Va(s) 10
R(s) + Ka
– 1 + 10s (s )
t

y(t)

(a) 1 (b) 5
(c)
(c) 10 (d) 100
[EC-2013 : 2 Marks]
t
Q.38 The natural frequency of an undamped second
y(t) order system is 40 rad/s. If the system is damped
with a damping ratio 0.3, the damped natural
(d) frequency (in radians), is _______ .
[EC-2014 : 1 Mark]
t
Q.39 The steady-state error of the system shown in
[EC-2011 : 1 Mark] the figure for a unit step input is ______ .
Q.36 A system described by a linear, constant R(s ) E(s) C(s )
1
coefficient, ordinary, first order differential + K=4
r (t ) – e(t) s+2 c(t)
equation has an exact solution given by y(t) for
t > 0, when the forcing function is x(t) and the
2
initial condition is y(0). If one wishes to modify
s+4
the system so that the solution becomes –2y(t)
for t > 0, we need to [EC-2014 : 2 Marks]
(a) change the initial condtion to –y(0) and the
Q.40 For the second order closed-loop system shown
forcing function to 2x(t).
in the figure, the natural frequency (in rad/sec),
(b) change the initial condtion to 2y(0) and the
is
forcing function to –x(t).
38 Electronics Engineering Control Systems

The transfer function of the system is


4
U(s) + Y(s )
– s ( s + 4) 2 1
(a) (b)
(s + 2) (s + 3) 2
s + 2s + 1
3 4
(a) 16 (b) 4 (c) 2
(d) 2
s + 2s + 3 s + 2s + 4
(c) 2 (d) 1
[EC-2015 : 2 Marks]
[EC-2014 : 1 Mark]
Q.44 The open-loop transfer function of unity
Q.41 The characteristic equation of a unity negative feedback control system is given by
feedback system is 1 + KG(s) = 0. The open-loop
K
transfer function G(s) has one pole at 0 and two G(s) =
s (s + 2)
poles at –1. The root locus of the system for
varying K is shown in the figure. For the peak overshoot of the closed-loop system
to a unit step input to be 10%, the value of K is
j
_______ .
[EC-2016 : 2 Marks]

= 0.5 Q.45 For the unity feedback control system shown in


A
the figure, the open-loop transfer function G(s)
is given as,
(0, 0)
2
–1 –1/3 Q G(s) =
s (s + 1)
The steady-state error ess due to a unit step input
is
e(t)
x(t ) + G(s ) y(t )

The constant damping ratio line, for = 0.5,


intersects the root locus at point A. The distance
(a) 0 (b) 0.5
from the region to point A is given as 0.5. The
(c) 1.0 (d)
value of K at point A is _____ .
[EC-2016 : 1 Mark]
[EC-2014 : 2 Marks]

Q.42 A unity negative feedback system has an open- s 2


Q.46 The response of the system, G(s) =
loop transfer function, (s + 1) (s + 3)
K
G(s) = to the unit step input u(t) is y(t). The value of
s (s + 10)
dy/dt at t = 0+ is ______ .
The gain K for the system to have a damping
[EC-2016 : 1 Mark]
ratio of 0.25 is ______ .
[EC-2015 : 1 Mark] Q.47 In the feedback system shown below,
1
Q.43 The output of a standard second-order system G(s ) = 2
(s + 2s )
for a unit step input is given as,
The step response of the closed-loop system
2 t should have minimum settling time and have
y (t ) = 1 e cos 3t
3 6 no overshoot.
GATE Previous Years Solved Paper 39

r G(s ) y 1
+

K where, G(s) =
s ( s + 1)

s+1
and C(s) = K
s+3
The required value of gain K to achieve this is
If the steady-state error for a unit ramp input is
______ .
0.1, then the value of K is ______ .
[EC-2016 : 2 Marks]
[EC-2020 : 2 Marks]
Q.48 The open-loop transfer function,
Q.51 Two linear time invariant systems with transfer
( s + 1) function:
G( s ) = p
s (s + 2) (s + 3)
10 10
G1 ( s ) = and G2 (s ) =
where, p is an integer, is connected in unity 2
s +s+1 2
s + s 10 + 10
feedback configuration as shown in the figure.
have unit step responses y 1 (t) and y 2 (t),
+ G(s ) y respectively. Which of the following statements

is/are true?
(a) y1(t) and y2(t) have the same percentage
Given that the steady-state error is zero for unit peak overshoot.
step input and is 6 for unit ramp input, the value (b) y1(t) and y2(t) have the same steady-state
of the parameter p is ______ . value.
[EC-2017 : 1 Mark] (c) y 1(t) and y 2(t) have the same damped
frequency of oscillation.
Q.49 Consider a causal second-order system with the
(d) y1(t) and y2(t) have the same 2% settling
transfer function,
time.
1
G( s ) = [EC-2022]
1 + 2s + s 2
Q.52 The block diagram of a closed-loop control
1
with a unit step R(s ) = as an input. Let C(s) system is shown in the figure. R(s), Y(s), and
s
D(s) are the Laplace transforms of the time
be the corresponding output. The time taken by domain signals r(t), y(t) and d(t), respectively.
the system output c(t) to reach 94% of its steady- Let the error signal be defined as, e(t) = r(t) – y(t).
state value lim c(t ), (rounded off to two Assuming the reference input r(t) = 0 for all t,
t the steady-state error e( ), due to a unit step
decimal places), is disturbance d(t), is _______ (Rounded off to two
(a) 5.25 (b) 4.50 decimal places).

(c) 2.81 (d) 3.89 D(s)

[EC-2019 : 2 Marks]
+
+ + 1
R(s) 10 Y(s )
Q.50 Consider the following closed-loop control s(s + 10)

system,

R(s) + C(s ) G(s) Y(s )


– [EC-2022]
40 Electronics Engineering Control Systems

ELECTRICAL EN GINEERIN G When connected in feedback as shown below,


the approximate value of Ka that will reduce the
(GATE Previous Years Solved Papers)
time constant of the closed-loop system by one
SECTIO N - A hundred times as compared to that of the open-
loop system is
Q.1 The unit impulse response of a unit feedback
Va(s) 10
control system is given by R(s) + Ka (s )
– 1 + 10s
c(t) = –te–1 + 2e–t, (t 0)
The open loop transfer function is equal to
s+1 2s + 1
(a) (b) (a) 1 (b) 5
(s + 2) 2
s2
(c) 10 (d) 100
s+1 s+1
(c) (d) [EE-2013 : 2 Marks]
(s + 1)2 s2
[EE-1996 : 1 Mark] Q.5 The closed-loop transfer function of a system is
4
Q.2 Feedback control systems are T (s ) = 2
(s + 0.4s + 4)
(a) insensitive to both forward and feedback
path parameter changes. The steady-state error due to unit step input is
(b) less sensitive to feedback path parameter ________ .
changes than to forward path parameter [EE-2014 : 1 Mark]
changes.
Q.6 The block diagram of a system is shown in the
(c) less sensitive to forward path parameter
figure.
changes than to feedback path parameter
changes.
(d) equally sensitive to forward and feedback

path parameter changes. + 1/s +– G(s) (s )
R(s ) – C(s )
[EE-2000 : 1 Mark]

Q.3 As shown in the figure, a negative feedback


system has an amplifier of gain 100 with ±10% If the desired transfer function of the system is
tolerance in the forward path, and an attenuator C (s) s
= 2
of value 9/100 in the feedback path. The overall R(s ) s + s + 1
system gain in approximately
then G(s) is
+ 100 ± 10% (a) 1 (b) s

1 s
(c) (d) 3 2
s s +s s 2
9/100
[EE-2014 : 2 Marks]
(a) 10 ± 1% (b) 10 ± 2%
SECTIO N -B
(c) 10 ± 5% (d) 10 ± 10%
[EE-2010 : 1 Mark] Q.1 A first order system and its response to a unit
step input are shown in figure below. The system
Q.4 The open-loop transfer function of a dc motor is
parameters are:
given as,
a = ________
(s) 10
= k = ________
Va ( a) 1 + 10s
GATE Previous Years Solved Paper 41

Q.6 For the system shown in figure, with a damping


c(t )
ratio of 0.7 and an undamped natural
2.0
c(t)
frequency n of 4 rad/sec., the values of ‘K’ and
‘a’ are

K
R(s ) + C(s )
– s ( s + 2)
0 0.2 t(sec)

1 + as
K
r (t ) c(t)
s+a
(a) K = 4, a = 0.35 (b) K = 8, a = 0.455
[EE-1991 : 2 Marks] (c) K = 16, a = 0.225 (d) K = 64, a = 0.9
[EE-1996 : 2 Marks]
Q.2 For what values of ‘a’ does the system shown in
figure have a zero steady-state error Q.7 A unity feedback system has open loop transfer
function G(s). The steady-state error is zero for
[i.e., Lim E(t ) ] for a step input?
t (a) step input and type 1 G(s).
(b) ramp input and type 1 G(s).
s+1
u(t ) + (c) step input and type 0 G(s).
– (s 2 + 5s + a)
(d) ramp input and type 0 G(s).
[EE-2000 : 1 Mark]
1
s+4 Q.8 A unity feedback system has open loop transfer
function,
(a) a = 0 (b) a = 2
(c) a 4 (d) for no value of ‘a’ 25
G(s) =
[EE-1992 : 1 Mark] s(s + 6)
The peak overshoot in the step-input response
Q.3 The steady-state error due to a step input for
of the system is approximately equal to
type 1 system is ______ .
(a) 5% (b) 10%
[EE-1995 : 1 Mark]
(c) 15% (d) 20%
Q.4 Consider the unit step response of a unity [EE-2000 : 2 Marks]
feedback control system whose open loop
transfer function is Q.9 The block diagram shown in figure gives a unity
feedback closed-loop control system. The
1
G(s) = steady-state error in the response of the above
s (s + 1)
system to unit step input is
The maximum overshoot is equal to
(a) 0.143 (b) 0.153 3 15
u(t) + y(t)
– s + 15 s+1
(c) 0.163 (d) 0.173
[EE-1996 : 1 Mark]

Q.5 For a feedback control system of type 2, the (a) 25% (b) 0.75%
steady-state error for a ram input is (c) 6% (d) 33%
(a) infinite (b) constant [EE-2003 : 2 Marks]
(c) zero (d) indeterminate
[EE-1996 : 1 Mark]
42 Electronics Engineering Control Systems

Q.10 The block diagram of a closed-loop control Q.13 If the above step response is to be observed on a
system is given by the figure. The value of K and non-storage CRO, then it would be best have
P such that the system has a damping ratio of the ei as a
0.7 and an undamped natural frequency n of (a) step function
5 rad/sec, are respectively equal to
(b) square wave of 50 Hz

R(s )
K
C(s )
(c) square wave of 300 Hz
+
– s ( s + 2)
(d) square wave of 2.0 kHz
[EE-2007 : 2 Marks]
1 + sP
Q.14 The transfer function of a linear time invariant
(a) 20 and 0.3 (b) 20 and 0.2 system is given as,
(c) 25 and 0.3 (d) 25 and 0.2 1
G(s ) = 2
[EE-2004 : 2 Marks] s + 3s + 2

Q.11 Consider the feedback system shown below The steady-state value of the output of the
which is subjected to a unit step input. The system for a unit impulse input applied at time
system is stable and has following parameters instant t = 1 will be
kp = 4, ki = 10, = 500 and = 0.7. The steady- (a) 0 (b) 0.5
state value of Z is (c) 1 (d) 2
[EE-2008 : 2 Marks]
Ki/s Z(s )
1
Q.15 The transfer function of a system is given as,
+ 2
+
KP 100
+ s2 + 2 s+ 2
2 .
0 – s + 20s + 100
The system is
(a) 1 (b) 0.25
(a) an overdamped system.
(c) 0.1 (d) 0
(b) an underdamped system.
[EE-2007 : 2 Marks]
(c) a critically damped system.
Statement for Common Data Questions (12 and 13): (d) an unstable system.
R-L-C circuit shown in figure.
[EE-2008 : 2 Marks]
R = 10 L=1m
Q.16 The unit step response of a unity feedback
system with open-loop transfer function,
ei e0 C = 10 µF K
G(s) = is shown in the figure. The
(s + 1) (s + 2)

value of K is
Q.12 For a step input ei, the overshoot in the output eo 1
will be 0.75
Response

(a) 0, since the system is not under damped


0.5
(b) 5%
(c) 16%
(d) 48% 0 1 2 3 4
Time
[EE-2007 : 2 Marks]
GATE Previous Years Solved Paper 43

(a) 0.5 (b) 2 Q.20 A open-loop control system results in a response


(c) 4 (d) 6 of e–2t (sin5t + cos5t) for a unit impulse input.
[EE-2009 : 2 Marks] The dc gain of the control system is ______ .
[EE-2015 : 1 Mark]
2
Q.17 For the system , the approximate time Q.21 The unit step response of a system with the
(s + 1)
1 2s
taken for a step response to reach 98% of its transfer function G(s ) = is given by
1+s
final value is
which one of the following waveforms?
(a) 1 s (b) 2 s
y
(c) 4 s (d) 8 s
1
[EE-2010 : 1 Mark]
(a) 0 t
Q.18 The steady-state error of a unity feedback linear 5
system for a unit step input is 0.1. The steady-
state error of the same system, for a pulse input –2
r(t) having a magnitude of 10 and a duration of
y
one second as shown in the figure is
2
r( t)

(b) 1

10
t
t 0 5
1s
(a) 0 (b) 0.1 y

(c) 1 (d) 10 2

[EE-2011 : 1 Mark] 1
(c)
Q.19 A two loop position control system in shown 0 t
5
below.
–0.75
Motor
1
R(s) + + Y(s ) y
– – s (s + 1)
1

ks
(d) 0 t
5
Tacho-generator

The gain of ‘k’ of the tacho-generator influences –2


mainly the
[EE-2015 : 2 Marks]
(a) peak overshoot
(b) natural frequency of oscillation Q.22 A second order real system has the following
(c) phase shift of the closed-loop transfer properties:
function at very low frequencies ( 0) (a) The damping ratio = 0.5 and undamped
(d) phase shift of the closed-loop transfer natural frequency n = 10 rad/sec.
function at very high frequencies ( ) (b) The steady-state value of the output, to a
[EE-2011 : 2 Marks] unit step input, is 1.02.
44 Electronics Engineering Control Systems

The transfer function of the system is Codes:


102 102 (a) P-I, Q-II, R-III (b) P-II, Q-I, R-III
(a) 2 (b) 2
s + 5s + 100 s + 10s + 100 (c) P-III, Q-II, R-I (d) P-III, Q-I, R-II
100 102 [EE-2018 : 1 Mark]
(c) 2 (d) 2
s + 10s + 100 s + 5s + 100 Q.26 Consider a unity feedback system with forward
[EE-2016 : 2 Marks] transfer function given by
Q.23 When a unit ramp input is applied to the unity 1
G(s) =
feedback system having closed-loop transfer (s + 1) (s + 2)
function, The steady-state error in the output of the
C (s) Ks + b system for a unit step input is _____ (upto two
= 2 , ( a > 0, b > 0, K > 0)
R(s) s + as + b decimal places).
The steady-state error will be [EE-2018 : 1 Mark]

a Q.27 The unit step response y(t) of a unity feedback


(a) 0 (b)
b system with open-loop transfer function,
a+K a K K
(c) (d) G( s ) H ( s ) = is shown in the
b b 2
( s + 1) ( s + 2)
[EE-2017 : 1 Mark]
figure.
Q.24 Which of the following system has maximum
The value of ‘K’ is _____ (upto two decimal
peak overshoot due to a unit step input?
places).
100 100
(a) 2 (b) 2
s + 10s + 100 s + 15s + 100 y

100 100 1.4


(c) (d) 1.2
s 2 + 5s + 100 s 2 + 20s + 100
1
[EE-2017 : 2 Marks] 0.8
0.6
Q.25 Match the transfer functions of the second order
0.4
systems with the nature of the system given
0.2
below: 0
0 2 4 6 8 10 12 14 16 18 20 t(sec)
Transfer function Nature of system
15 [EE-2018 : 2 Marks]
P. 2 I. Overdamped
s + 5s + 15
25
Q. 2 II. Critically damped
s + 10s + 25
35
R. 2 III. Underdamped
s + 18s + 35
GATE Previous Years Solved Paper 45

Electronics & Electrical Engineering


GATE Previous Years Solved Paper

A n swe rs & Expl a n a t i o n s

Answers
EC Time Response Analysis

1. (a) 2. (b) 3. (a) 4. (a) 5. (Sol.) 6. (Sol.) 7. (Sol.) 8. (c)

9. (b) 10. (a) 11. (a) 12. (a) 13. (b) 14. (a) 15. (b) 16. (b)

17. (d) 18. (c) 19. (b) 20. (c) 21. (b) 22. (b) 23. (c) 24. (a)

25. (d) 26. (c) 27. (a) 28. (b) 29. (c) 30. (d) 31. (c) 32. (c)

33. (d) 34. (b) 35. (a) 36. (d) 37. (c) 38. (38.16) 39. (0) 40. (c)

41. (0.37) 42. (400) 43. (d) 44. (2.8) 45. (a) 46. (1) 47. (1) 48. (1)

49. (b) 50. (30) 51. (a) 52. (–0.1)

Solutions
EC Time Response Analysis

1. (a) 2 n =8

K p = lim G(s ) H (s) 8 4


s 0 = = = 0.8
2 n 5
K
= lim = = 1 2
s 0 s (s + 10) d n

A A = 5 1 (0.8)2 = 3
ess = = =0
1 + Kp 1 + For 2nd peak n = 3,
n 3
2. (b) tp = = = sec.
d 3
The steady-state error of a stable ‘type 0’ unity
feedback system for a unit step function is 4. (a)
1 s 4(1 + 2 s )
. K V = lim s G(s ) H (s ) = lim
1 + Kp s 0 s 0 s 2 (s + 2)
4(1 + 2s)
= lim =
3. (a) s 0 s (s + 2)

2 = 25 A A
n ess = = =0
KV
n =5
46 Electronics Engineering Control Systems

5. Sol. 11. (a)


The poles of a continuous time oscillator are The final value theorem is used to find the
pure imaginary. steady-state value of the system.

6. Sol. 12. (a)


(a) - 1, (b) - 5, (c) - 2 s = ±j
(a) Overdamped (1) poles on the negative So, poles are pure imaginary, hence final value
real axis > 1, can not be determined.

2 13. (b)
s= n ± n +1
(b) Critically damped (5), multiple poles on Number of poles at origin of open-loop transfer
the negative real axis, function gives the type of the system.

= 1 14. (a)
s= – n, – n K
(c) Oscillatory (2) poles on the imaginary K p = lim G(s) H (s) = lim =
s 0 s 0 s (s + 1)
axis,
A A
= 0 ess = = =0
1 + Kp 1 +
s = ±j n
15. (b)
7. Sol.
1
(a) - 1, (b) - 5, (c) - 4 H(s) =
s
(a) Very low response at very high frequencies
1
Low pass system. R(s) =
s+a
(b) Overshoot Damping ratio
1 1 1 1
(c) Synchro-control transformer output C(s) = R(s ) H (s ) = =
s (s + a ) a s s+a
Phase-sensitive modulation
1 at
C(t) = [1 e ]
8. (c) a
<1
16. (b)
For underdamped system, roots or poles are:
4
2 ts =
s= n±j n 1 n
2 n =4 n=2
10. (b)
4
ts = =2
2 s2 + 2s 2
f = lim sF(s ) = =2
(0 + ) s s 2 + 2s + 5
17. (d)
2
2 s + 2s
f ( ) = lim sF(s ) = 2 =0 an 1 s + an
s s + 2s + 5
sn + a1 s n 1
+ ...an 2 s 2
T(s) =
10. (a) an 1 s + an
1+ n
K p = lim G(s) H (s) s + a1 s n 1 + ...an 2 s 2
s 0
an 1 s + an
1 1 G(s) =
= lim = sn + a1 s n 1
+ ...an 2 s2
s 0 ( s + 6) ( s + 1) 6
GATE Previous Years Solved Paper 47

For type 2, ramp input 1


2 n =
K V = lim sG(s) K
s 0
6 1
KV = 2 × 0.5 × =
K K
1
ess = =0 6 1
KV = 2
K K
18. (c) 1
K=
6
s2 + 2 n + 2
n =0
21. (b)
1
2 = 2, =
n
n
100
G(s) =
(s + 1) (s + 100)
n = 2
Taking dominant pole consideration,
1
= s = –100 pole is not taken
2
100
Q < 1, hence response will be underdamped. G(s) =
s+1
19. (b) Now it is 1st order system
CLTF = ts = 4T = 4 × 1 = 4s
G(s) s+4
= 22. (b)
1 + G(s) H (s ) s 2 + 7 s + 13
C (s) 4
1 + G(s) H (s) s 2 + 7 s + 13 = 2
= R(s ) s + 4s + 4
G(s ) s+4
2 n = 4, n = 2
H(s) = 1 for unity feedback
= 1 (Critical damping)
1 s 2 + 7 s + 13 4 4
= 1 ts = = =2
G(s ) s+4 1× 2
n
2
1 s + 6s + 9
= 23. (c)
G(s ) s+4
1
s+4 H(s) =
G(s) = s+2
s 2 + 6s + 9
r(t) = 10 u(t)
For D.C. : s=0
G(s) = open-loop gain 10
R(s) =
s
4
= 1 10
9 C(s) = H (s ) R(s ) =
s+2 s
20. (c) 10 A B
= +
s+6 s(s + 2) s s+2
s 6 5 5
K s2 + + C(s) =
K K s s+2
c(t) = 5[1 – e–2t] u(t)
Comparing with s 2 + 2 n + 2
n Steady-state value when t = 0 is 5.99% of steady-
6 state value reaches at,
n = 5[1 – e–2t] = 0.99 × 5
K
48 Electronics Engineering Control Systems

1 – e–2t = 0.99 29. (c)


e–2t = 0.01 h(t) = e–t
–2t = ln 0.01 1
t = 2.3 sec. H(s) =
s+1
24. (a) 1
R(s) =
s
d2 y 3 dy 1 1
2
+ + 2 y = X(t) Output = H (s) R(s) =
dt dt (s + 1) s
s2Y(s) + 3sY(s) + 2Y(s) = X(s)
1 1
x(t) = 2u(t) Output = = (1 e t ) u(t )
s s+1
2 When t = at steady-state,
X(s) =
s Output = 1
2
(s2 + 3s + 2) Y(s) = 30. (d)
s
In dominant pole concept, the factor that has to
2
Y(s) = be eliminated should be in time constant for,
s(s + 2) (s + 1)
5 5
2 A B C 2 =
= + + ( s + 5) ( s + s + 1) s
s(s + 2) (s + 1) s s+2 s+1 5 1+ (s2 + s + 1)
5
1 1 2
Y(s) = + 1
s s+2 s+1 = 2
s +s+1
y(t) = [1 + e–2t – 2e–t] u(t)
31. (c)
27. (a)
Peak overshoot depends on as,
1
ess = lim sE(s), R(s) = 2
s 0 s2 Mp = e
/ 1

R(s) where, = cos–1


= lim s
s 0 1 + G(s) where is the angle made by pole from negative
1 real axis.
= lim = finite (given)
s 0 s + sG(s ) To make Mp same, should be the same.
K V = lim sG(s )
s 0 32. (c)
1 1 Transfer function can be written as,
ess = lim = finite = 5% =
s 0 sG(s ) 20 K (s + 1)
H(s) =
K = 20 (s + 2) (s + 4)
KV is finite for type 1 system having ramp input. 1
Input, R(s) =
s
28. (b)
Output, C(s) = R(s) H(s)
1 1 2 lim sC (s ) = 1
C(s) = = Given,
s s + 2 s(s + 2) s 0

1 s K (s + 1)
R(s) = or, lim =1
s s 0 s(s + 2) (s + 4)
C (s) 2 K
H(s) = = or, =1
R(s ) s + 2 8
GATE Previous Years Solved Paper 49

K=8 35. (a)


8(s + 1)
H(s) = d2 y dy
(s + 2) (s + 4) 100 2
20 + y = x(t )
dt dt
4 12 Taking Laplace transform of both sides,
= +
s+2 s+4
X (s)
h(t) = (–4e–2t + 12e–4t) u(t) 100 s 2 Y (s) 20 sY (s) + Y (s) =
s
Which is also the required impulse response of
Y (s ) 1/ s
the system. =
X( s ) 2
(100s 20s + 1)
33. (d)
1
Comparing the given transfer function with =
s (10 s 1)2
2 1 1
n , ,0
2 ,
Poles are at, s=
s2 + 2 ns + n
10 10
At poles are on the right-hand side of s-plane so
25 given system is unstable system.
P= =0
s 2 + 25 Only option (a) represents unstable system.
So, P is undamped,
36. (d)
36
Q=
s 2 + 20s + 36 dy(t )
+ ky(t ) = x(t)
20 dt
= = 1.67 Taking Laplace transform of both sides, we have
2×6
So, Q is overdamped, sY(s) – y(0) + kY(s) = X(s)
Y(s) [s + K] = X(s) + y(0)
36
R= 2 X( s ) y(0)
s + 12 s + 36 Y(s) = +
s+k s+k
12
= =1 Taking inverse Laplace transform, we have
2×6
y(t) = e–kt x(t) + y(0) e–kt
So, R is critically damped,
So, if we want –2y(t) as a solution both x(t) and
49
S= 2
y(0) has to be multiplied by –2; hence change
s + 7 s + 49
x(t) by –2x(t) and y(0) by –2y(0).
7
= = 0.5 37. (c)
2×7
So, S is underdamped. (s ) 10
=
Va (s) 1 + 10s
34. (b)
1 + Ts = 1 + 10s
Steady-state value = –2
Ts = 10
Denominator,
T = 10
2 n = 1.91, n2 = 1.91 T 10 1
T = = =
n 1.4 100 100 10
1.91 1.91 10
= = <1 Ka ×
2 n 2.8 ( s) 1 + 10 s
= 10
...(underdamped) R(s ) 1 + Ka
1 + 10s
50 Electronics Engineering Control Systems

( s) 10 K a 10 K a 41. Sol.
= =
R(s ) 1 + 10 s + 10 K a (1 + 10 K a ) + 10s According the question,
( s) 10 K a K
= G(s) H(s) = ...(i)
R(s ) 10s s(s + 1)2
(1 + 10K a ) 1 +
1 + 10K a
j
10 1
T = =
1 + 10K a 10
1 + 10Ka = 100
10Ka = 99 A jb

Ka = 9.9 10

39. Sol. B O
–1 –1/3
Given, n = 40 rad/sec.
= 0.3
2
d = n 1
= 38.16 rad/sec.

39. Sol.
After converting the system into equivalent The damping ratio is given as = 0.5.
unity feedback system. = cos–1 = 60°
Let the co-ordinates of point ‘A’ is (–a + jb)
4(s + 4)
R(s ) +

C(s ) OB
s (s + 2) where, cos60° = ...(ii)
OA
Also, 0 A = 0.5 (Given)

4(s + 4) BA
Similarly, sin60° = ...(iii)
OLTF = OA
s(s + 2)
BA = jb = 0.433j
Type = 1
Hence, co-ordinates of A = –0.25 + 0.43j
ess for step input = 0
The value of system gain K can be obtained
40. (c) as,
The closed-loop transfer function of the given G(s) H (s) = 1 ...(iv)
system is, From equation (i) and (iv) we get,
C (s) 4 4 s = –0.25 + j4.33
= = 2
R(s ) s (s + 4) + 4 s + 4s + 4 K = 0.37
Comparing with standard second order transfer
42. Sol.
K n2 G(s) K
function 2 , we get, T(s) = =
s + 2 n + n2 1 + G(s) H (s) s 2 + 10s + K
Comparing with standard second order transfer
2 =4
n function,
2
or, n = 4 = 2 rad/sec. n
T(s) =
s2 + 2 ns +
2
n
GATE Previous Years Solved Paper 51
2 2 = (2.3)2 (1 – 2)
we have, 2 =K
n 2
15.16 = (2.3)2
and 2 n = 10 = 0.59
Q = 0.25 (Given)
1
Also, K= = 2.8
10 2
= = 20
n
2 × 0.25
2 45. (a)
and K= n = (20)2 = 400
R(s)
E(s) =
43. (d) 1 + G(s) H (s)
The output of a second order system for unit 1 2
R(s) = , G(s) = , H(s) = 1
step input, s s (s + 1)
e nt
1/ s s+1
y(t) = 1 sin( ot ) ...(i) E(s) = = 2
1 2 2 s +s+2
1+
s (s + 1)
From given data,
2 s (s + 1)
y(t) = 1 e t
cos 3t ess = lim sE(s) = lim 2 =0
s 0 s 0 s +s+2
3 6
= n=1 46. Sol.
and = 3
d dy
L = (sY(s) – y(0))
= 2 2 dt
n + d =2
1 s 2
1 Y(s) = G(s) × =
= s s (s + 1) (s + 3)
2
Therefore, the transfer function. y(0) = lim sY (s )
s
2 (Applying initial value theorem)
n 4
T(s) = =
s2 + 2 ns +
2
n s 2 + 2s + 4 2
1
s 2 s
= lim =
44. Sol. s (s + 1) (s + 3) 1 3
s 1+ 1+
s s
K
G(s) = ; H (s) = 1 y(0) = 0
s (s + 2)
dy s × (s 2) s 2
Characteristic equation = 1 + G(s) H(s) = 0 L = sY (s) = =
dt s (s + 1) (s + 3) (s + 1) (s + 3)
K
1+ =0 dy dy
s (s + 2) = lim sL
dt t = 0 s dt
s2 + 2s + K = 0

n = K 1
2
s × (s 2) s
1 = lim = =1
2 = 2; = s (s + 1) (s + 3) 1 3
n 1+ 1+
K s s

/ 1 2
Mp = e = 0.1 47. Sol.
1
= ln (0.1) G(s) = 2
1 2 s + 2s
52 Electronics Engineering Control Systems

K Since the system is type-1 so far a given unit


Y (s ) s 2 + 2s = K ramp input steady-state,
= K 2
R(s ) s + 2s + K 1
1+ 2 ess =
s + 2s Kv
Minimum settling time and no overshoot K K
where, K v = lim s × =
implies critical damping i.e., s 0 s(s + 3) 3
=1 1 3
So, ess = =
n = K K /3 K
2× n =2 Given that,
n =1 ess = 0.1
K = 1 or K = 1 3
So, 0.1 = K = 30
K
48. Sol.
51. (a)
Steady-state error of type 1 system, for step input
is zero, for ramp input is 1/Kv and for parabolic 10
For system: G1(s) = 2
input is infinity. s +s+1
So, the given system must be of type-1 and p = 1. Characteristic equation,
s2 + s + 1 = 0
49. (b) The standard characteristics equation is,
2 1
G(s) = 2
= s2 + 2 ns +
2
n =0
s + 2s + 1 (s + 1)2
On comparing,
1
R(s) = 1
s n = 1, = = 0.5
2
1
C(s) = G(s ) R(s ) = 1 2
s(s + 1)2 d = n

Using partial fraction expansion, we get, = 1 1 (0.5)2 = 0.866


A B C
C(s) = + + 4
s (s + 1) (s + 1)2 Settling time, ts = = 8 sec
n
1 1 1 1
C(s) = 10 s
s (s + 1) (s + 1)2 s
Steady-state, ess = lim = 10
and c(t) = (1 – e–t – te–t) u(t) s 0 s2 + s + 1

lim c(t ) = 1 10
t For system: G2(s) = 2
s + 10s + 10
In order to reach 94% of its steady-state value,
Characteristic equation,
(1 – e–t – te–t) = 0.94
By trial and error, we get, s2 + 2 ns +
2
n =0
t 4.50 sec. On comparing,
2
50. Sol. n = 10 n = 10

Open-loop transfer function for the system 2 n = 10 = 0.5


K (s + 1) 1 = = 1 2
= C (s) × G(s) = × d n
(s + 3) s(s + 1)
= 10 1 (0.5)2 = 2.739
GATE Previous Years Solved Paper 53

4 4 1
Settling time, ts = = G1(s) = 10, G2 (s) =
n 0.5 10 s(s + 10)
8 E(s) G2 (s)
= = 2.535 =
10 D(s) 1 + G1 (s) G2 (s)
Steady-state error, 1
s(s + 10) 1
1 = = 2
10 s 1 s + 10s + 10
s 1 + 10 ×
ess = lim =1 s(s + 10)
s 0 s 2 + 10 s + 10
ess = lim e(t ) = lim sE(s)
Since ‘ ’ value for both the system is same. So t s 0
percentage peak overshoot for both system is
s(1/ s)( 1)
same. ess = lim 2
s 0 s + 10s + 10

52. (–0.1) 1
= = 0.1
D(s )
10

R(s ) = 0 1
+– 10 ++ Y(s )
s(s + 10)

Answers
EE Time Response Analysis (Section-A)

1. (b) 2. (c) 3. (a) 4. (c) 5. (0) 6. (b)

Solutions
EE Time Response Analysis (Section-A)

1. (b) 2. (c)
Close loop T.F. = L (Impulse response) 1
T .F . =
2s + 1 Q SG
1 + GH
C.L.T.F. =
(s + 1)2
GH
O.L.T.F. STH.F. =
Q C.L.T.F. = 1 + GH
1 + O.L.T.F.
Less sensitive to ‘change in G’ then ‘change
(For unity feedback)
in H’
G(s )
i.e., C.L.T.F. = 3. (a)
1 + G(s ) H ( s ) = 1
G = 100 ± 10%
C.L.T.F.
G(s) = G
1 C.L.T.F. H ( s ) = 1 = 10% or 0.1
G
On putting the value of C.L.T.F.
9
2s + 1 H=
We get, G(s) = 100
s2
54 Electronics Engineering Control Systems

Overall gain, 5. Sol.


G Closed-loop transfer function,
T= ...(i)
1 + GH 4
T(s) = 2
100 (s + 0.4s + 4)
T= = 10
9 G(s)
1 + 100 × 4
100 or, = [H(s) = 1]
1 + G(s) s 2 + 0.4s + 4
Sensitivity w.r.t.,
or, (s2 + 0.4s + 4) = 4 + 4 G(s)
1 4
‘G’ = × 10%
1 + GH or, G(s) =
s (s + 0.4)
1 10 Open loop transfer function (for unity feedback
= × 10% = 1%
1 + 10 × 1 11 system)
Given, Input = u(t) = r(t)
4. (c)
1
(s) 10 10 R(s) =
= = s
Va (s ) 1 + 10s 1 + s Steady-state error,
= 10
sR(s ) sR(s )
Closed loop system, ess = lim = lim
s 0 1 + G(s) H (s ) s 0 1 + G( s )
10 K a
(For H(s) = 1)
(s) 1 + 10s = 10 K a
= 10 K a
R(s ) 1+ 1 + 10 K a + 10s 1

1 + 10s s s (s + 0.4)
= lim = lim 2
s 0 4 s 0 s + 0.4s + 4
1+
10 K a s (s + 0.4)
1 + 10 K a
= s (s + 0.4)
10 = lim =0
1+ s s 0 2
s + 0.4s + 4
1 + 10 K a
Steady-state error for step input, ess = 0.
According to question,
10 1 6. (b)
= × 10
1 + 10 K a 100 For all the given values of G(s) in options, G(s)
1 + 10 Ka = 100 does not satisfy the desired condition. But
Ka = 9.9 option (b) is official answer given by IIT.

Answers
EE Time Response Analysis (Section-B)

1. (5, 10) 2. (a) 3. (0) 4. (c) 5. (c) 6. (c) 7. (a) 8. (b)

9. (a) 10. (d) 11. (a) 12. (c) 13. (c) 14. (a) 15. (c) 16. (d)

17. (c) 18. (a) 19. (a) 20. (0.241) 21. (a) 22. (b) 23. (d) 24. (c)

25. (c) 26. (0.66) 27. (8)


GATE Previous Years Solved Paper 55

Solutions
EE Time Response Analysis (Section-B)

1. Sol. 4. (c)
Q Time constant = 0.2 sec. (from figure) 1
G(s) =
s ( s + 1)
1 K Characteristic equation = 1 + G(s)
= From G(s) =
a s = s2 + s + 1
a 1+
a
On comparing with s 2 + 2 ns +
2
n
1
= 0.2 we have, = 0.5
a
a=5
1 2
Q Final value = lim sC (s) Mp = e = 0.1630
s 0
5. (c)
K 1 K
= slim0 s s + a a = a = 2 1
s
(From figure) lim sE( s ) = lim s2 =0
s 0 s 0 G ( s ) H (s)
K 1+
=2 s
a
K = 2 × 5 = 10 6. (c)
Characteristic equation:
2. (a) 1 + G(s) H(s) = 0
R(s) K
E(s) = = 1+ (1 + as) = 0
1 + G(s) H (s) s (s + 2)
lim E(t ) = lim sE(s) s2 + (Ka + 2) s + K = 0 ...(i)
t s 0
Q Characteristic equation is in the form of

s
1 s2 + 2 ns +
2
n =0
= lim s
( s + 1) 1 For = 0.7 and n = 4
s 0
1+ 2
(s + 5s + a) (s + 4) Characteristic equation will be
s2 + 5.6s + 16 = 0 ...(ii)
1 On comparing equation (i) and (ii), we get,
=0
1 K = 16 and a = 0.225
1+
4a
7. (a)
4a
=0 Steady-state error is zero for step input with
4a + 1
type-1 G(s).
Hence, a=0
8. (b)
3. Sol.
25
1 G(s) =
s s ( s + 6)
lim sE(s) = lim s =0
s 0 s 0 G (s ) H ( s) Characteristic equation,
1+
s 1 + G(s) = 0
56 Electronics Engineering Control Systems

25 So, characteristic equation


1+
s (s + 6) = 0 = s2 + (2 + kP) s + P
s2 + 6s + 25 = 0 Comparing with standard equation
On comparing with, 2 2
= s +2 ns + n

s2 + 2 ns +
2
n =0 k=
2
n = 52 = 25
we get, n = 5 and = 0.6 (where, n = undamped natural frequency)
2 n = 2 + kP
1 2 2 × 0.7 × 5 = 2 + 25 P
Mp = e = 0.0948 10%
(where = damping ratio)
9. (a) P = 0.2

3 15 45 11. (a)
G(s) = × =
s + 15 s+1 (s + 1) (s + 15) 1
Step input R(s ) =
and H(s) = 1 s
Open loop transfer function Ki/s Z(s )
45
= G(s) H (s ) = +
(s + 1) (s + 15) +
KP
2
R(s) 2 2 C (s )
E(s ) + s +2 s+
The system is type-0. –
Steady-state error to unit step input,
1 ki 2
ess = G(s) = kp +
1 + Kp
s s2 + 2 s+ 2

where, and H(s) = 1


kp = Position error constant = lim G(s) H (s) C (s) G(s) G(s)
s 0 =
=
R(s ) 1 + G(s) H (s ) 1 + G(s)
45
k p = lim =3 E(s) = R(s) – C(s)
s 0 (s + 1) (s + 15)

1 1 C (s)
= = R(s ) 1
ess = R(s )
1 + kp 1 + 3
= 0.25 or 25% G(s ) R(s )
= R(s ) 1 =
1 + G(s) 1 + G(s)
10. (d)
ki
k R(s)
ki
G(s) = and H(s) = 1 + sP Z(s) = E(s) = s
s(s + 2) s 1 + G(s)
Closed-loop transfer function, Steady-state value of Z,
k Zss = lim sZ( s)
s 0
G(s) s ( s + 2)
T(s) = = k ki 1
1 + G(s) H (s) 1 + (1 + sP ) s
s ( s + 2) s s
= lim 2
k
s 0 ki
= 1 + kp +
s(s + 2) + k (1 + sP ) s s2 + 2 s+ 2

k ki
T(s) = = 2
=1
s 2 + (2 + kP ) s + k ki 2
GATE Previous Years Solved Paper 57

12. (c) / 1 2
Overshoot = PP = e
R = 10 L = 1 mH
× 0.5 / 1 0.5 2
= e
= 0.163 or 16.3%
ei i C = 10 µF
13. (c)
1
=
n LC
di 1
ei = Ri + L + i dt 1
dt c =
1 × 10 × 10 × 10 6
3
Taking Laplace transform,
= 104 rad/sec.
1
Ei(s) = R + Ls + I (s ) 4 4
Cs Settling time (ts) = = 4
n 10 × 0.5
Ei (s )
I(s) = = 0.8 msec.
1
R + Ls +
Cs
1
Square wave
e0 = i dt
c

1 1 Ei (s )
E0(s) = I (s) =
Cs Cs R + Ls + 1 T
Cs
T/2
Ei (s )
E0(s) = For a square wave T/2 should be greater than ts.
RCs + LCs 2 + 1
E0 (s) 1 For, f1 = 50 Hz
= T1 1
Ei (s) R2 1 = 10 ms >> ts
LC s + s + =
L LC 2 2 × 50
Characteristic equation, For, f2 = 300 Hz

R 1 T2 1
s2 +
s+ = = 1.67 ms > ts
=0 2 2 × 300
L LC
For, f3 = 2 kHz
Comparing with,
T3 1
s2 + 2 2 = = 0.25 ms < ts
ns + n 2 2 × 2 × 10 3
1 Therefore, it would be best to have ei as a square
=
LC wave of 300 Hz.
R 14. (a)
2 n =
L
r(t) = unit impulse applied at t = 1
R 1
= × = (t – 1)
L 2 n
R(s) = 1[r(t)] = e–s
R LC R C C (s) 1
= × = G(s) = =
L 2 2 L R(s) s 2 + 3s + 2
10 10 × 10 6 e s
= = 0.5 C(s) = R(s ) G(s ) =
2 1 × 10 3 s 2 + 3s + 2
58 Electronics Engineering Control Systems

Steady-state value of output, using final value 17. (c)


theorem,
C (s) 2
Css = lim sC (s) =
R( s ) s+1
s 0

s 1
se R(s) = (step-input)
= lim 2 =0 s
s 0 s + 3s + 2
2 2
C(s) = R(s ) =
15. (c) s+1 s (s + 1)
100 1 1
M(s) = 2 = 2
s + 20s + 100 s s+1
Comparing with standard form,
C(t) = L –1 [C(s)] = 2[1 – e–t]
2
n Final value of C(t) = Css = 2
M(s) =
s2 + 2 ns +
2
n 98% of Css = 0.98 × 2 = 1.96
2 = 20 Let, t = T, the response reaches 98% of its final
n
values,
n = 100
=1 1.96 = 2[1 – e–T]
T 4 sec.
n = 10
The system is critically damped.
18. (a)
16. (d) Let the system is represented as,
Steady-state value of response = 0.75 X(s ) + G(s ) Y(s )

Input is unit-step.
So steady-state error,
ess = 1 – 0.75 = 0.25 y( s ) G(s)
=
R(s) x(s) 1 + G(s) H (s)
Error = E(s) =
1 + G(s) H (s) H(s) = 1 (unity feedback)
k x(s)
where, G(s) = Error = E(s) =
(s + 1) (s + 2) 1 + G(s)
1 1
H(s) = 1 and R = Steady-state error for x(s ) = , e = 0.1
s s ss
Steady-state error using find value theorem, sx(s)
ess = lim sE(s) = lim
ess = lim sE(s) s 0 s 0 1 + G( s)
s 0
1
sR(s ) sx
= lim 0.1 = lim s
s 0 1 + G( s ) H ( s )
s 0 1 + G( s )
1 1
s lim
s 1 = 0.1
= lim = s 0 1 + G(s)
s 0 k k
1+ 1+ When input,
(s + 1) (s + 2) 2
1
0.25 = k 10
1+
2
k
1+ =4 k=6 t
2 1s
GATE Previous Years Solved Paper 59

x(t) = 10[u(t) – u(t – 1)] Since mp depnds on which depends on K.


s Hence peak overshoot is influenced by gain (K)
1 e 10
x(s) = 10 = [1 e s ] of the techo-generator.
s s s
sx(s) 20. Sol.
ess = lim sE(s) = lim
s 0 s 0 1 + G( s) 5 (s + 2)
2 2
+
10 (s + 2) + 5 ( s + 2)2 + 5 2
s× [1 e s ]
= lim s Putting s = 0,
s 0 1 + G(s)
5 2
= +
10(1 e s ) 2
2 +5 2
2 + 52
2
ess = lim =0
s 0 1 + G( s )
5 2 7
= + = = 0.241
29 29 29
19. (a)
21. (a)
1
R(s) + + Y(s )
– – s ( s + 1) 1 2s
T.F. =
1+s
ks
1 2s 1 A B
C(s) = = +
1+s s 1+s s
1 3
1 C(s) =
R(s ) + Y(s ) s 1+s
– s (s + 1 + K )
C(t) = (1 – 3e–t) u(t)

22. (b)
1 Damping ratio,
G(s) = and H(s) = 1
s(s + 1 + K ) = 0.5
Characteristic equation, Undamped natural frequency,
1 + G(s) H(s) = 0 n = 10 rad/sec.

1 Steady-state output to a unit step input,


1+ =0
s(s + 1 + K ) Css = 1.02
Hence steady-state error = 1.02 – 1.00
s(s + 1 + K) + 1 = 0
ess = 0.02
s2
+ (K + 1) s + 1 = 0
Characteristic equation is,
Comparing with,
s2 + 2 ns +
2 =0
s2 + 2 ns +
2
n =0 n
s2
+ 2 × 0.5 × 10s + 100 = 0
Natural frequency,
s2 + 10s + 100 = 0
n = 1 From options, if we take option (b).
Remains constant and does not depend on K,
Then, Css = lim sC (s)
2 n = K+1 s 0
K+1
= 1 102
2 = lim s × × 2
s 0 s s + 10s + 100
Damping ratio depends on k,
Css = 1.02
/ 1 2
Peak overshoot = mp = e Hence option (b) is correct answer.
60 Electronics Engineering Control Systems

23. (d) 25
Q= 2
Ks + b s + 10s + 25
Close-loop transfer function = 2
s + as + b =
25 = 5 rad/sec.
n
Open-loop transfer function 2 × 5 = 10
Ks + b =1 (Critically damped)
= G( s ) = 2
s + as + b Ks b Observing all the options, option (c) is correct.
Ks + b Ks + b 26. Sol.
G(s) = 2
=
s + as Ks s(s + a K )
Steady-state error for type-0 and step input,
Steady-state error for ramp input given to type-1 1
system = 1/Kv. ess =
1 + kp
Where, velocity error coefficient,
1 1
Ks + b b k p = lim =
K v = lim s = s 0 (s + 1) (s + 2) 2
s 0 s (s + a K ) a K
1 2
Steady-state error, ess = = = 0.66 units
1 3
a K 1+
ess = 2
b
27. Sol.
24. (c)
Closed-loop transfer function,
For maximum peak over shoot,
K
1 2
Mp C (s) (s + 1) (s + 2)
=
R(s ) K
1+ 2
= 0.25 for option (c), which is least among all (s + 1) (s + 2)
options. Therefore correct option is (c).
C (s) K
= 2
25. (c) R(s ) (s + 1) (s + 2) + K
15 1
P= Given, R(s) =
2 s
s + 5s + 15
K
n = 15 = 3.872 rad/sec. C(s) = 2
s [(s + 1) ( s + 2) + K ]
2 × 3.872 = 5
lim sC (s ) = 0.8 (Given)
5 s 0
= = 0.64
2 × 3.872
K
(Underdamped) = 0.8 K=8
2+K
4 Stability Analysis

ELECTRO NICS EN GINEERIN G Q.6 The number of roots of s3 + 5s2 + 7s + 3 = 0 in the


left half of the s-plane is
(GATE Previous Years Solved Papers)
(a) zero (b) one
Q.1 Consider a characteristic equation given by (c) two (d) three
s4 + 3s3 + 5s2 + 6s + K + 10 [EC-1998 : 1 Mark]
The condition for stability is Q.7 The open-loop transfer function of an unity
(a) K > 5 (b) –10 < K feedback open-loop system is
(c) K > –4 (d) –10 < K < –4
2 s2 + 6s + 5
[EC-1988 : 2 Marks]
(s + 1)2 (s + 2)
Q.2 In order to stabilize the system shown in figure
The characteristic equation of the closed-loop
Ti should satisfy,
system is
1 + sTi 1 (a) 2s2 + 6s + 5 = 0
r( t) +
– s s (1 + sT ) c(t)
(b) (s + 1)2 (s + 2) = 0
(c) 2s2 + 6s + 5 + (s + 1)2 (s + 2) = 0
(d) 2s2 + 6s + 5 – (s + 1)2 (s + 2) = 0
(a) Ti = –T (b) Ti = T
[EC-1998 : 1 Mark]
(c) Ti < T (d) Ti > T
[EC-1989 : 2 Marks] Q.8 An amplifier with resistive negative feedback
has two left half plane poles in its open-loop
Q.3 An electromechanical closed-loop control
transfer function. The amplifier
system has the following characteristic equation:
(a) will always be unstable at high frequency.
s3 + 6Ks2 + (K + 2)s + 8 = 0
(b) will be stable for all frequency.
where K is the forward gain of the system. The
(c) may be unstable, depending on the
condition for closed-loop stability is,
feedback factor.
(a) K = 0.528 (b) K = 2
(d) will oscillate at low frequency.
(c) K = 0 (d) K = –2.258
[EC-2000 : 1 Mark]
[EC-1990 : 2 Marks]
Q.9 A system described by the transfer function,
Q.4 If s3 + 3s2 + 4s + A = 0, then all the roots of this
equation are in the left half plane provided that 1
H (s) = 3 2 is stable.
(a) A > 12 (b) –3 < A < 4 s + s + Ks + 3
(c) 0 < A < 12 (d) 5 < A < 12 The constraints on and K are
[EC-1993 : 2 Marks] (a) > 0, K < 3 (b) > 0, K > 3
Q.5 If G(s) is a stable transfer function, then (c) < 0, K > 3 (d) < 0, K < 3
[EC-2000 : 2 Marks]
1
F(s) = is always a stable transfer function.
G( s) Q.10 The feedback control system in the figure is
stable.
(T/F).
[EC-1994 : 2 Marks]
62 Electronics Engineering Control Systems

s 2
Q.15 For the polynomial
R(s) + K 0 C(s )
– (s + 2)2 P(s) = s5 + s4 + 2s3 + 2s2 + 3s + 15
The number of roots which lie in the right half
of the s-plane is
s–2
(a) 4 (b) 2
(a) for all K 0 (b) only if K 0 (c) 3 (d) 1
(c) only if 0 K < 1 (d) only if 0 K 2 [EC-2004 : 2 Marks]
[EC-2001 : 2 Marks]
Q.16 The positive values of ‘K’ and ‘a’ so that the
Q.11 The system shown in the figure remains stable system shown in the figure below oscillates at a
when frequency of 2 rad/sec respectively are
1 K s–1 1 K (s + 1)
R(s ) Y(s ) R(s ) + C(s )
– ( s + as 2 + 2s + 1)
3

1
(a) 1, 0.75 (b) 2, 0.75
(c) 1, 1 (d) 2, 2
(a) K < –1 (b) –1 < K < 1
[EC-2006 : 2 Marks]
(c) 1 < K < 3 (d) K < –3
[EC-2002 : 2 Marks] Common Data Questions (17 and 18):
Consider a unity gain feedback control system whose
Q.12 The characteristic polynomial of a system is
q(s) = 2s5 + s4 + 4s3 + 2s2 + 2s + 1 as + 1
open-loop transfer function is G(s ) = .
The system is s2
(a) stable (b) marginally stable
Q.17 The value of ‘a’ so that the system has a phase
(c) unstable (d) oscillatory
margin equal to /4 approximately equal to
[EC-2002 : 2 Marks]
(a) 2.40 (b) 1.40
Q.13 The gain margin for the system with open-loop (c) 0.84 (d) 0.74
[EC-2006 : 2 Marks]
2 (1 + s)
transfer function G(s) H (s ) = , is
s2 Q.18 With the value of ‘a’ set for phase margin of /4,
the value of unit impulse response of the open-
(a) (b) 0
loop system at t = 1 second is equal to
(c) 1 (d) –
(a) 3.40 (b) 2.40
[EC-2004 : 1 Mark]
(c) 1.84 (d) 1.74
Q.14 The open-loop transfer function of a unity [EC-2006 : 2 Marks]
feedback system is
Q.19 If the closed-loop transfer function of a control
K
G( s ) = s 5
s (s 2 + s + 2) (s + 3) system is given as, T (s) = .
(s + 2) (s + 3)
The range of K for which the system is stable is Then it is
21 (a) an unstable system.
(a) >K >0 (b) 13 > K > 0
4 (b) an uncontrollable system.
21 (c) a minimum phase system.
(c) <K < (d) –6 < K <
4 (d) a non-minimum phase system.
[EC-2004 : 2 Marks] [EC-2007 : 1 Mark]
GATE Previous Years Solved Paper 63

Q.20 A certain system has transfer function, The value of K which will place both the poles
s+8 of the closed-loop system at the same location,
G( s ) = 2
s + s 4 is ______.
where is a parameter. [EC-2014 : 1 Mark]
Consider the standard negative unity feedback Q.24 Consider a transfer function,
configuration as shown below.
ps2 + 3 ps 2
+ G(s ) Gp ( s ) =
– s 2 + (3 + p ) s + (2 p )
with ‘p’ a positive real parameter. The maximum
Which of the following statements is true? value of ‘p’ until which Gp remains stable is
______ .
(a) The closed-loop system is never stable for
any value of . [EC-2014 : 2 Marks]

(b) For some positive values of , the closed- Q.25 A plant tranfer function is given as,
loop system is stable, but not for all positive
KI 1
values. G(s ) = K P +
s s (s + 2)
(c) For all positive values of , the closed-loop
when the plant operates in a unity feedback
system is stable.
configuration, the condition for the stability of
(d) The closed-loop system is stable for all
the closed-loop system is
values of , both positive and negative
KI
[EC-2008 : 2 Marks] (a) K P > >0 (b) 2KI > KP > 0
2
Q.21 The number of open right half plane poles of (c) 2KI < KP (d) 2KI > KP
10 [EC-2015 : 2 Marks]
G( s ) = 5 4 3 2
is
s + 2 s + 3s + 6 s + 5s + 3 Q.26 The characteristic equation of an LTI system is
(a) 0 (b) 1 given by
(c) 2 (d) 3 F(s) = s5 + 2s4 + 3s3 + 6s2 – 4s – 8 = 0
[EC-2008 : 2 Marks] The number of roots that lie strictly in the left
Q.22 The feedback system shown below oscillates at half s-plane is _____ .
2 rad/sec when, [EC-2015 : 2 Marks]

K (s + 1) Q.27 Match the inference X, Y and Z about a system,


R(s ) + 3 2 Y(s )
– (s + as + 2s + 1) to the corresponding properties of the elements
of first column in Routh’s table of the system
characteristic equation:
(a) K = 2 and a = 0.75 List-I
(b) K = 3 and a = 0.75 X. The system is stable.
(c) K = 4 and a = 0.5 Y. The system is unstable.
(d) K = 2 and a = 0.5 Z. The test breaks down.
[EC-2012 : 2 Marks]
List-II
Q.23 The forward path transfer function of a unity P. ....when all elements are positive.
negative feedback system is given by Q. ....when any one element is zero.
K R. ....when there is a change in sign of
G(s) =
(s + 2) (s 1) coefficients.
64 Electronics Engineering Control Systems

Codes: 1
G(s ) =
(a) X-P, Y-Q, Z-R (b) X-Q, Y-P, Z-R 2
s + 3s + 2
(c) X-R, Y-Q, Z-P (d) X-P, Y-R, Z-Q where K > 0. The positive value of K for which
[EC-2016 : 1 Mark] there are exactly two poles of the unity feedback
Q.28 The transfer function of a linear time invariant system of the j -axis is equal to ______ (rounded
system is given by off to two decimal places).

H(s) = 2s4 – 5s3 + 5s – 2 X(s)


+
K/s G(s ) Y(s )
The number of zeroes in the right half of the –
s-plane is ______ .
[EC-2016 : 2 Marks]
[EC-2019 : 2 Marks]
Q.29 The first two rows in the Routh table for the
characteristic equation of a certain closed-loop Q.33 The loop transfer function of a negative feedback
control system are given as, system is

3 K (s + 11)
s 1 (2K + 3) G(s ) H (s ) =
s (s + 2) (s + 8)
2
s K 4
The value of K, for which the system is
The range of K for which the system is stable is marginally stable, is ______ .
(a) –2.0 < K < 0.5 (b) 0 < K < 0.5 [EC-2020 : 1 Mark]
(c) 0 < K < (d) 0.5 < K < Q.34 The characteristic equation of a system is
[EC-2016 : 2 Marks] s3 + 3s2 + (K + 2)s + 3K = 0
Q.30 Which one of the following options correctly In the root locus plot for the given system, as K
describes the locations of the roots of the varies from 0 to , the break-away or break-in
equation s4 + s2 + 1 = 0 on the complex plane? point(s) lie within
(a) Four left half plane (LHP) roots. (a) (–2, –1) (b) (–1, 0)
(b) One right half plane (RHP) root, one LHP (c) (–3, –2) (d) (– , –3)
root and two roots on the imaginary axis. [EC-2020 : 2 Marks]
(c) Two RHP roots and two LHP roots.
Q.35 Consider an even polynomial p(s) given by:
(d) All four roots are on the imaginary axis.
p(s) = s4 + 5s2 + 4 + K
[EC-2017 : 2 Marks]
where ‘K’ is an unknown real parameter. The
Q.31 A unity feedback control system is characterized complete range of ‘K’ for which p(s) has all its
by the open-loop transfer function, roots on the imaginary axis is ______ .
2(s + 1) 9
G(s) = (a) 4 K
s3 + Ks2 + 2s + 1 4
The value of K for which the system oscillates at 9
(b) 3 K
2 rad/sec is ______ . 2
[EC-2017 : 2 Marks] 5
(c) 6 K
4
Q.32 Consider a unity feedback system, as in the
(d) –5 K 0
figure shown, with an integral compensator K/s
[EC-2022]
and open-loop transfer function,
GATE Previous Years Solved Paper 65

ELECTRICAL EN GINEERIN G that in lie in the right half of s-plane is

(GATE Previous Years Solved Papers) (a) zero (b) one


(c) two (d) three
Q.1 The closed-loop system of figure is stable if the [EE-1998 : 2 Marks]
C (s) Q.8 The roots of the closed-loop characteristic
transfer function T (s) = is stable.
R(s) equation of the system shown above.
(True/False)
3 15
u(t) + y(t)
R(s) G(s ) – s + 15 s+1
+ C(s )

(a) –1 and –15 (b) 6 and 10


H(s )
(c) –4 and –15 (d) –6 and –10
[EE-1994 : 1 Mark] [EE-2003 : 2 Marks]

Q.2 The number of positive real roots of the equation Q.9 The loop gain GH of a closed-loop system is
s3 – 2s + 2 = 0, is ________ .
K
[EE-1994 : 1 Mark] given the following expression .
s (s + 2) (s + 4)
Q.3 Closed-loop stability implies that, [1 + G(s) H(s)] The value of ‘K’ for which the system just
has all the ______ in the left half of the s-plane. becomes unstable is
[EE-1995 : 1 Mark] (a) K = 6 (b) K = 8
Q.4 Determine whether the system given by the block (c) K = 48 (d) K = 96
diagram of figure is stable. [EE-2003 : 2 Marks]

Q.10 For the equation, s3 – 4s2 + s + 6 = 0 the number


1 – 1 1
R(s)
(s + 1)
+

4 +
( s 4)
3 +
– (s + 3)
C(s)
of roots in the left-half of s-plane will be
(a) zero (b) one
4 8
(c) two (d) three
[EE-1997 : 2 Marks]
[EE-2004 : 2 Marks]
Q.5 The system represented by the transfer function
Q.11 A unity feedback system, having an open-loop
s 2 + 10s + s 24
G( s ) = 4 k(1 s)
s + 6s 3 39s 2 + 19s + 84 gain G(s) H (s) = , becomes stable when
(1 + s)
has _____ pole(s) in the right-half of s-plane.
[EE-1997 : 2 Marks] (a) k >1 (b) k > 1
Q.6 None of the poles of a linear control system lie (c) k <1 (d) k < –1
in the right half of s-plane. For a bounded input,
[EE-2005 : 2 Marks]
the output of this system
(a) is always bounded. Q.12 The system shown in the figure is
(b) could be unbounded. + s 1
u1
(c) always tends to zero. s+2

(d) none of the above [EE-1998 : 1 Mark]
+
Q.7 The number of roots on the equation, 1
u2
s 1 +
2s4 + s3 + 3s2 + 5s + 7 = 0
66 Electronics Engineering Control Systems

(a) stable (d) two roots at s = ±j and one root in left half
(b) unstable s-plane.
(c) conditional stable [EE-2009 : 1 Mark]
(d) stable for input u1, but unstable for input u2 Q.16 An open-loop system represented by the transfer
[EE-2007 : 1 Mark] function is
Q.13 If the loop gain ‘k’ of a negative feedback system (s 1)
G(s) =
(s + 2) (s + 3)
k(s + 3)
having a loop transfer function is to be (a) stable and of the minimum phase type
(s + 8)2
(b) stable and of the non-minimum phase type
adjusted to induce a sustained oscillation then (c) unstable and of the minimum phase type
(a) the frequency of this oscillation must be (d) unstable and of the non-minimum phase
4 / 3 rad/sec. type
[EE-2011 : 1 Mark]
(b) the frequency of this oscillation must be
4 rad/sec. Q.17 The feedback system shown below oscillates at
(c) the frequency of this oscillation must be 2 rad/sec when

4 or 4 / 3 rad/sec. k(s + 1)
R(s ) +– Y(s )
s + as 2 + 2s + 1
3
(d) such a ‘k’ does not exist.
[EE-2007 : 2 Marks]

Q.14 Figures shows a feedback system where k > 0. (a) k = 2 and a = 0.75
The range of ‘k’ for which is stable will be (b) k = 3 and a = 0.75
given by (c) k = 4 and a = 0.5

k
(d) k = 2 and a = 0.5
+
s (s + 3) ( s + 10) [EE-2012 : 2 Marks]

Q.18 In the formation of Routh-Hurwitz array for a
polynomial, all the elements of a row have zero
(a) 0 < k < 30 (b) 0 < k < 39 values. This premature termination of the array
(c) 0 < k < 390 (d) k > 390 indicates the presence of
[EE-2008 : 2 Marks] (a) only one root in the origin
Q.15 The first two rows of Routh’s tabulation of a 3rd (b) imaginary roots
order equation are as follows: (c) only positive real roots
(d) only negative real roots
s3 2 2
[EE-2014 : 1 Mark]
s2 4 4
This means three are: Q.19 For the given system, it is desired that the system
(a) two roots at s = ±j and one root in right half be stable. The minimum value of for this
s-plane. condition is _______ .
(b) two roots at s = ±j2 and one root in left half (s + )
R(s ) +– C(s )
s-plane. s 3 + (1 + ) s 2 + ( 1) s + (1 )

(c) two roots at s = ±j2 and one root in right


half s-plane.
[EE-2014 : 2 Marks]
GATE Previous Years Solved Paper 67

Q.20 A system with the open-loop transfer function, The closed-loop system will be stable if,
K 4(K + 1)
G( s ) = (a) 0 < T <
s(s + 2) ( s 2 + 2 s + 2) K 1
is connected in a negative feedback
4(T + 2)
configuration with a feedback gain of unity. For (b) 0 < K <
T 2
the closed-loop system to be marginally stable,
the value of ‘K’ is _______ . T +2
(c) 0 < K <
[EE-2014 : 2 Marks] T 2

Q.21 A single-input single-output feedback system 8( k + 1)


(d) 0 < T <
has forward transfer function G(s) and feedback K 1
transfer function H(s). It is given that, [EE-2016 : 2 Marks]
G(s) H (s) < 1. Q.25 A closed-loop system has the characteristic
Which of the following is true about the stability equation given by,
of the system? s3 + Ks2 + (K + 2) s + 3 = 0
(a) The system is always stable. For this system to be stable, which one of the
(b) The system is stable if all zeros of G(s) H(s) following conditions should be satisfied?
are in left half of the s-plane. (a) 0 < K < 0.5 (b) 0.5 < K < 1
(c) The system is stable if all poles of G(s) H(s) (c) 0 < K < 1 (d) K > 1
are in left half of the s-plane. [EE-2017 : 1 Mark]
(d) It is to possible to say whether or not the Q.26 The range of ‘K’ for which all the roots of the
system is stable from the information given. equation s3 + 3s2 + 2s + K are in the left half of
[EE-2014 : 1 Mark] the complex s-plane is
Q.22 The transfer function of a second order real (a) 0 < K < 6 (b) 0 < K < 16
system with a perfectly flat magnitude response (c) 6 < K < 36 (d) 6 < K < 16
of unity has a pole at (2 – j3). List all the poles [EE-2017 : 2 Marks]
and zeros
Q.27 The number of roots of polynomial,
(a) Poles at (2 ± j3), no zeros
s7 + s6 + 7s5 + 14s4 + 31s3 + 73s2 + 25s + 200,
(b) Poles at (±2 – j3), one zero at origin
in the left half of the complex plane is
(c) Poles at (2 – j3), (–2 + j3), zeros at (–2 – j3),
(a) 3 (b) 4
(2 + j3)
(c) 5 (d) 6
(d) Poles at (2 ± j3), zeros at (–2 ± j3)
[EE-2018 : 2 Marks]
[EE-2015 : 2 Marks]
Q.28 The characteristic equation of a linear time-
Q.23 Given the following polynomial equation,
invariant (LTI) system is given,
s3 + 5.5s2 + 8.5s + 3 = 0
(s) = s4 + 3s3 + 3s2 + s + k = 0
The number of roots of the polynomial, which
The system is BIBO stable is
have real parts strictly less than –1, is _____ .
12
[EE-2016 : 2 Marks] (a) 0 < k < (b) k > 3
9
Q.24 The open-loop transfer function of a unity 8
(c) 0 < k < (d) k > 6
feedback control system is given by 9
K (s + 1) [EE-2019 : 1 Mark]
G(s) = , K > 0, T > 0
s(1 + Ts) (1 + 2s)
68 Electronics Engineering Control Systems

Q.29 Which of the following option is correct for the Q.30 Consider a negative unity feedback system with
system shown below? the forward path transfer function

R(s ) +
1 1
Y(s ) s2 + s + 1
– s+1 s2 , where ‘K’ is a positive real
s3 + 2 s2 + 2s + K

20 number. The value of ‘K’ for which the system


s + 20 will have some of its poles on the imaginary
axis is _________ .
(a) 3rd order and stable
(a) 8 (b) 9
(b) 4th order and unstable
(c) 6 (d) 7
(c) 4th order and stable
[EE-2020 : 2 Marks]
(d) 3rd order and unstable
[EE-2020 : 2 Marks]

Electronics & Electrical Engineering


GATE Previous Years Solved Paper

A n swe rs & Expl a n a t i o n s

Answers
EC Stability Analysis

1. (d) 2. (d) 3. (b) 4. (c) 5. (False) 6. (d) 7. (c) 8. (b)


9. (b) 10. (c) 11. (d) 12. (c) 13. (a) 14. (a) 15. (b) 16. (b)
17. (c) 18. (c) 19. (d) 20. (c) 21. (c) 22. (a) 23. (2.25) 24. (1.99)
25. (a) 26. (0) 27. (d) 28. (3) 29. (d) 30. (c) 31. (0.75) 32. (6)
33. (160) 34. (b) 35. (a)

Solutions
EC Stability Analysis

1. (d) For stable system, all the coefficient of 1st column


Using R-H criterion: should be positive.
12 3K
s4 1 5 K + 10 So, >0
3
s3 3 6 0 –12 – 3 K > 0
2
s 3 K + 10 0 –12 > +3 K
12 3 K –4 > K and K + 10 > 0
s1 0 0
3 K > –10
s0 K + 10 So, the range of K
–10 < K < –4
GATE Previous Years Solved Paper 69

2. (d) 4. (c)
Characteristic equation, s3 + 3s2 + 4s + A = 0
1 + GH = 0 Using R-H criteria:
(1 + sTi ) 1
1+ × ×1 = 0 s3 1 4
s s (1 + sT )
2
s 3 A
s2(1 + sT) + (1 + sTi) = 0
12 A
s3T + s2 + sTi + 1 = 0 s1 0
3
Using R-H method:
s0 A
s3 T Ti
For stable system,
s2 1 1 A>0
s1 Ti T 12 A
0 and >0
s 1 3
12 – A > 0
For stability, 1st column should be positive.
12 > A
So, Ti – T > 0
0 < A < 12
Ti > T
5. Sol.
3. (b)
(False)
s3 + 6Ks2 + (K + 2)s + 8 = 0
Using R-H criteria: ( s + Z1 ) (s + Z2 )
T.F. of G(s) =
(s + P1 ) (s + P2 )
s3 1 K+2
( s + P1 ) (s + P2 )
s 2
6K 8 T.F. of F(s) =
(s + Z1 ) (s + Z2 )
6K 2 + 12 K 8 The condition for stability is that none of the
s1 0
6K pole of G(s) should be on the right half of s-plane,
s0 8 but G(s) may have zeros in the right half of
s-plane. These zeros become pole of F(s).
For stable system, 1st column element should be
Therefore the F(s) need not be stable (False).
positive.

6K 2 + 12K 8 6. (d)
>0 Using R-H criterion:
6K
6K2 + 12K – 8 > 0
s3 1 7
3K2 + 6K – 4 > 0
s2 5 3
6 ± 36 + 48 1
K= s 6.4 0
6
s0 3
K = –2.528, +0.528
K > 0.528 There is no sign change in the first column of
K > –2.528 R-H array. So, there is no roots lie in R.H.S. of
So, K > 0.528 s-plane. So, all the 3 roots will lie in the left half
So, from given option for K = 2 system will be of s-plane.
stable.
70 Electronics Engineering Control Systems

7. (c) For third row,


2s2 + 6s + 5 + (s + 1)2 (s + 2) = 0 K > –1
Characteristic equation, 0 K<1
1 + GH = 0
11. (d)
2
2 s + 6s + 5
1+ =0 Y (s ) K /s K
(s + 1)2 ( s + 2) = =
R(s ) 3 K s (3 + K )
1 +
2s2 + 6s + 5 + (s + 1)2 (s + 2) = 0 s s

8. (b) For system to be stable,


3+K < 0
Second-order system is stable.
K < –3
9. (b)
12. (c)
The characteristic equation is,
Routh table is,
s3 + s2 + Ks + 3 = 0
Routh table is,
s5 2(1) 4(2) 2(1)
3
s 1 K
s4 1 2 1
2
s 3 3
s 0 0 0
1 K 3
s 0 s2

s0 3 0 s1

For system to be stable, s0

K 3
> 0, >0 d 4
(s + 2 s 2 + 1) = 0
ds
K>3
4s3 + 4s = 0
10. (c) s = ±j, s = ±j
K(s 2) d 2
(s + 1) = 0
G1G2 (s + 2) 2 ds
T.F. = = 2s = 0
1 + G1G2 H K (s 2) (s 2)
1+
(s + 2)2 s=0
K (s 2) Double roots on imaginary axis so system is
= unstable.
(s + 2)2 + K (s 2)2
Characteristic equation, 13. (a)
= s2 + 4 + 4s + Ks2 – 4Ks + 4K G(s) H(s) = –180° + tan–1
(1 + K)s2 + K(4 – 4K)s + 4K + 4 = 0 For = –180° + tan–1 = –180°
Routh table is, =0
s2 (1 + K ) (4K + 4) 2
2 1+
1 G(s) H (s) = =
s K (4 4K ) 0 2

s0 4(K + 1) 1
G.M. = =0
For system to be stable,
1–K > 0 In dB G.M. =
1 > K and K 0 (Given in question)
GATE Previous Years Solved Paper 71

14. (a) 16. (b)


K K (s + 1)
G(s) = 2 1 + G( s ) H ( s ) = 1 + =0
s(s + s + 2) (s + 3) s + as 2 + 2 s + 1
3

H(s) = 1
s 3 + as 2 + 2 s + 1 + Ks + K
K =0
1 + G(s) H(s) = 1 + 3 2 2 s 3 + as 2 + 2 s + 1
s(s + 3s + s + 3s + 2s + 6)
s3 + as2 + (2 + K)s + K + 1 = 0
4 3 2
s + 4s + 5s + 6 s + K
= s3 1 2+K
s(s 3 + 4s 2 + 5s + 6)
s2 a K +1
1 + G(s) H(s) = 0
s + 4s3 + 5s2 + 6s + K = 0
4 a (2 + K ) (K + 1)
s
a
s4 1 5 K For oscillation,
3 a (2 + K ) ( K + 1)
s 4 6
=0
7 a
s2 K
2 K +1
7 a=
× 6 4K K+2
s 2 0 as2 + K + 1 = 0
7 /2 s=j ; s2 = – 2 = –4
s0 K –4a + K + 1 = 0
For system to be stable, K > 0 K+1
a=
4
2
(21 4K ) >0 K+1 K +1
7 = K=2
4 K+2
21 21
> K K< a = 0.75
4 4
21 17. (c)
> K>0
4
P.M. =
15. (b) 4
P(s) = s5 + s4 + 2s3 + 2s2 + 3s + 15 180 + tan–1 a – 180° =
4
s5 1 2 3
tan
=a a =1
s4 1 2 15 4
Now for gain crossover frequency,
s3 0( ) 12 0
2 +12 G(s) = 1
s2 15 0

12 (2 +12)
15
1 + a2 2
2 =1
s1
2 +12
2
1+1 = (as a = 1)
0 2
s 15 = 2
= (2)1/4
• Coefficient of s2 tend to + .
1
• Coefficient of s1 tend to –12. a= 1/4
= 0.84
2
Two sign change from s2 to s and s to s0.
Two roots on RHS of s-plane.
72 Electronics Engineering Control Systems

18. (c) 22. (a)


0.84 s + 1 Characteristic equation is,
G(s) = 2 1 + G(s) H(s) = 0
s
H(s) = 1, R(s) = 1 K (s + 1)
1+ =0
C(s) = G(s) R(s) s 3 + as 2 + 2 s + 1
0.84 s + 1 s3 + as2 + (2 + K)s + (1 + K) = 0
C(s) =
s2 Routh array for this is,

1 1 + 0.84 s 1 1 0.84 s3 1 (2 + K )
c(t) = L 2
=L 2
+
s s s 2
s a (1 + K )
c(t) = [t + 0.84] U(t)
a (2 + K ) (1 + K )
At t = 1, c(t) = 1 + 0.84 = 1.84 s1
a
19. (d) s0 (1 + K )
As there is a right half zero, the system is a non- For oscillation,
minimum phase system. a (2 + K ) (1 + K )
=0
20. (c) a

Closed-loop gain is, 1+K


a=
s+8 2+K
G(s)
= 2 Now, as2 + (1 + K) = 0
1 + G(s) s + s 4+s+8
Characteristic equation, –a 2 + (1 + K) = 0
q(s) = s2 + ( + 1)s + 4 Given, = 2 rad/sec
Closed-loop system is stable only for > –1. –4a + (1 + K) = 0
Therefore, for all positive of , the closed-loop (1 + K )
4 + (1 + K ) = 0
system is stable. (2 + K )
–4(1 + K) + (2 + K) (1 + K) = 0
21. (c)
So, K=2
Characteristic equation,
1+K 3
q(s) = s5 + 2s4 + 3s3 + 6s2 + 5s + 3 a= = = 0.75
2+K 4
1
Putting, s= 23. Sol.
z
a (z) = 3z5 + 5z4 + 6z3 + 3z2 + 2z + 1 K
Given that, G(s) =
Routh array is, (s + 2) (s 1)
z5 3 6 2 Using root locus method, the break point can be
obtain as,
z4 5 3 1
21 7 Img
z3
5 5
63 35 = 0.5

z2 5 5 =4 1 Re
21 3 –2 0.5
28 21
z1 15 5 = 35 /15 = 7
4 /3 4 /3 4
z0 1 1 + G(s) = 0
Since sign changes twice in Routh-array,
therefore, there are two poles on right half plane.
GATE Previous Years Solved Paper 73

K For system to be stable,


1+ =0
(s + 2) (s 1) KP > 0
or, K = –(s + 2) (s – 1) 2 Kp KI
and >0
dK 2
= –2s – 1 = 0 or, 2Kp – KI > 0
ds
or, s = –0.5 KI
or, Kp > >0
To have, both the poles at the same directions, 2
G(s) s = 0.5 = 1 26. Sol.
K = 2.25 Using Routh’s tabular form:

24. Sol. s5 1 3 4
4
Given transfer function, s 2 6 8

ps2 + 3 ps 2 s3 0(8) 0(12) 0


Gp(s) = ...(i) 2
s2 + (3 + p) s + (2 p ) s 3 8 0
The characteristic equation is 100
s1 0 0
= s2 + (3 + p) s + (2 – p) ...(ii) 3
Using Routh Hurwitz criterion, s0 8

s2 1 (2 p) Auxiliary equation,
A = 2s4 + 6s2 – 8 = 0
s1 (3 + p) 0
dA
s 0
(2 p) 0 = 8s3 + 12s = 0
ds
For system to be remain stable, As one time row of zero occur in Routh’s table
2–p 0 therefore a pair of imaginary pole exists, also
pmax = 1.99 number of sign change in Routh’s table is one.
Hence, two poles lie strictly in the left half of
25. (a) s-plane.
KI 1 27. (d)
G(s) = Kp +
s s (s + 2)
When all elements are positive, the system is
The closed-loop transfer function for unity stable. When any element is zero, the test breaks
feedback, down. When there is change in sign of
G(s) (K p s + K I ) coefficients the system is unstable.
= 2
1 + G(s) s (s + 2) + (K p s + K I ) 28. Sol.
Kps + KI 2s4 – 5s3 + 5s – 2 = 0
= By Routh array,
s3 + 2s2 + K p s + K I
Using Routh’s tabular form: s4 2 0 2
3
s 5 5
s3 1 Kp
s2 2 2
s2 2 KI
1
2 Kp KI s 0(2)
s1 0 0
2 s 2
0
s Kp Number of sign changes = number of roots
(zeros) in right half of s-plane = 3.
74 Electronics Engineering Control Systems

29. (d) 32. Sol.

s3 1 (2K + 3) Y (s ) K
= 3
X( s ) s + 3s 2 + 2 s + K
s2 2K 4
2 The poles of this system lie on the j -axis when
4K + 6 K 4
s 0 the system is marginally stable,
2K
kmar = 3 × 2 = 6
s0 4
For stability, K > 0 ...(i) 33. Sol.
4K2 + 6K – 4 > 0 Characteristic equation q(s) for the given open-
2K2 + 3K – 2 > 0 loop system will be
(2K – 1) (K + 2) > 0 q(s) = s3 + 10s2 + 16s + Ks + 11K = 0
1 Using R-H criteria:
K= or K < –2 ...(ii)
2
s3 1 16 + K
Hence, 0.5 < K <
2
s 10 11 K
30. (c) 10 (16 + K ) 11 K
s1 0
q(s) = s4 + s2 + 1 = 0 10
s0 11 K 0
s4 1 1 1
For system to be marginally stabled,
s3 4 2 0
10(16 + K ) 11 K
s2 0.5 1 0 dA(s ) =0
= 4s 3 + 2 s1 10
ds
s1 6 0 0 160 + 10K – 11K = 0
s 0
1 0 0 K = 160

There are two sign changes in the first column 34. (b)
of the R-H table and the order of auxiliary Q(s) = 1 + G(s) H(s) = 0
equation is 4. So, 4-poles are symmetric about s3 + 3s2 + 2s + Ks + 3K = 0
origin.
s 3 + 3s 2 + 2s
2 RHP roots and 2 LHP roots. –K =
s+3
j
dK ( s + 3) (3s 2 + 6s + 2) ( s 3 + 3s 2 + 2 s )
= =0
ds (s + 3)2
0 3s3 + 6s2 + 2s + 9s2 + 18s + 6 – s3 – 3s2 – 2s = 0
2s3 + 12s2 + 18s + 6 = 0
31. Sol. s = –0.46, –3.87, –1.65
The given open-loop transfer function is, Img

2(s + 1)
G(s) =
s + Ks 2 + 2s + 1
3

The closed-loop transfer function is,


G(s ) 2 (s + 1) Re
T(s) = = 3 –2 –1 0
1 + G(s) s + Ks 2 + 4s + 3
When system oscillates, i.e., when system is
marginally stable,
(1) (3) = K(4) Break-away point lies between (0, –1),
So, K = 0.75 i.e. (–1, 0).
GATE Previous Years Solved Paper 75

35. (a) 1st column of R-H table must be all positive.


Given, p(s) = s4 + 5s2 + (4 + K) 25 4(4 + K ) 9
Routh’s table: i.e., >0 K<
5 /2 4
s4 1 5 (4 + K ) 4+K > 0 K > –4
s 3
0 0 Range of K :
9
s2 –4 < K <
4
s1
s0
As all row of s3 are zero,
A(s) = s4 + 5s2 + (4 + K)
dA(s )
= 4s3 + 10s + 0
ds

s4 1 5 (4 + K )
s3 4 10 0
10 5
s2 (4 + K )
4 2
25 4(4 + K )
s1
5 /2
s0 4+K

Answers
EE Stability Analysis

6. (d) 7. (c) 8. (d) 9. (c) 10. (b) 11. (c) 12. (d) 13. (b)

14. (c) 15. (d) 16. (b) 17. (a) 18. (b) 19. (0.618) 20. (5) 21. (d)

22. (d) 23. (2) 24. (c) 25. (d) 26. (a) 27. (a) 28. (c) 29. (b)

30. (a)

Solutions
EE Stability Analysis

1. Sol. 3. Sol.
All the close-loop poles must lie in left half of Because zeros of the 1 + G(s) H(s) represents
the s-plane. closed-loop transfer function poles which must
lie in the left half of the s-plane.
2. Sol.
Q s3 – 2s + 2 = 0
s = –1.77, 0.88 ± j0.59
Hence, no positive real root exists.
76 Electronics Engineering Control Systems

4. Sol. 7. (c)
SFG : 2s4 + s3 + 3s2 + 5s + 7 = 0

–1 s4 2 3 7

1 1 1
s3 1 5 0
(s + 1) (s 4) (s + 3) 2
R(s )
4 3 1
C(s )
s 7 7
s1 6
–8 0
s 7
–4
Q Two sign changes in first column.
1 1 1
s+1
× 4×
s 4
× 3×
s+3
× [1] Two poles in the right half of s-plane.
T.F. = 16 8 3 16 8
1 + ×
s 4 s + 3 ( s + 3) ( s 4) s 4 s+3 8. (d)

On solving, 1 + GH = 0 (characteristic equation),

12 45
1+
T.F. =
s 3 + 24s 2 + 158s + 135 (s + 15) (s + 1) = 0
s2 + 16s + 60 = 0
s3 1 158
(s + 10) (s + 6) = 0
s2 24 135 Roots are (–6, –10)
s1 +ve
9. (c)
0
s +ve
Characteristic equation = 1 + GH = 0
Q No sign change in first column K
1+ =0
System is stable. s (s + 2) (s + 4)

5. Sol. Characteristic equation,


s3 + 6s2 + 8s + K = 0
Characteristic equation:
The Routh-array is formed as follows:
s4 + 6s3 – 39s2 + 19s + 84 = 0
Routh Hurwitz criteria:
s3 1 8
4 2
s 1 39 84 s 6 K
s3 6 19 48 K
s1
6
s2 42 84
s0 K
1
s 31
s0 84 According to Routh-Hurwitz criterion, for a
stable system there should be no change of sign
Q Two sign chanegs in first column. in the first column of Routh-array.
Two poles in the right-half of s-plane. 48 K
So, > 0 and K > 0
6. (d) 6
0 < K < 48
If the poles lie on imaginary axis and at the
Hence for K = 48, the system just becomes
origin then output may be bounded or
unstable.
unbounded for a bounded input.
GATE Previous Years Solved Paper 77

10. (b) 12. (d)


– s3 4s2
+s+6=0
s 1 y1(s )
The Routh-array is formed as follows: u1(s ) +
– s+2
s3 1 1
2 1
s 4 6
s 1
4×1 6×1
s1 = 2.5
4
(s 1)
s0 6 (s 1)
s+2 =
(T/F)1 = (s 1) 1
Number of sign changes in first column of 1+ × (s + 3)
Routh-array = 2. (s + 2) (s 1)
Accroding to Routh-Hurwitz criterion, the Poles is in LHS of s-plane, hence stable.
number of changes of sign in the first column
1 y2(s )
gives the number of positive real part roots of u2(s ) +
– s 1
the polynomial. So, number of roots in RHS of
s-plane = 2.
s 1
Total number of roots = 3
s+2
Hence, number of roots in LHS of s-plane =
3 – 2 = 1. 1
( s 1) s+2
11. (c) (T/F)2 = =
1 (s 1) ( s 1) (s + 3)
1+ ×
Characteristic equation, (s 1) (s + 2)
1 + G(s) H(s) = 0
Hence, unstable as it has pole at right hand side
k(1 s ) of s-plane.
1+ =0
1+s
s + 1 + k(1 – s) = 0 13. (b)
s(1 – k) + 1 + k = 0 Open-loop transfer function,
k+1 (s + 3)
s= G(s) H(s) = k
k 1 (s + 8)2
For a stable system pole lies in left hand side of Characteristic equation,
s-plane, it means and must be negative for stable 1 + G(s) H(s) = 0
system, (s + 3)
1+ k =0
s<0 (s + 8)2
k+1 s2 + 16s + 64 + (s + 3) = 0
<0
k 1 s2 + (16 + k) s + (64 + 3k) = 0
Case-I: Routh array,
k + 1 < 0 and k – 1 > 0
k < –1 and k > 1 s2 1 64 + 3 k
which is not possible. s1 16 + k
Case-II: 0
s 64 + 3 k
k + 1 > 0 and k – 1 < 0
For sustained oscillation,
k > –1 and k < 1
16 + k = 0
–1 < k < 1
k = –16
k <1
78 Electronics Engineering Control Systems

Auxiliary equation, The derivative of this auxiliary equation is taken


s2 + 64 + 3k = 0 w.r.t. ‘s’ and the coefficients of the differentiated
s2 + 64 + 3(–16) = 0 equation are taken as the elements of 3rd row,
s2 + 16 = 0 dA(s )
s = ±j4 = 8s
ds
So, frequency of oscillation is 4 rad/sec. Routh-array,

14. (c) s3 2 2
k s2 4 4
G(s) =
s (s + 3) (s + 10) s 1
8
and H(s) = 1 s0 4
Characteristic equation,
There is no root in RHS of s-plane.
1 + G(s) H(s) = 0
Two roots of s = ±j, so one root in LHS of s-plane.
k
1+
s (s + 3) (s + 10) = 0 16. (b)
s(s + 3) (s + 10) + k = 0 s 1
1+ =0
s3 + 13s2 + 30s + k = 0 (s + 2) (s + 3)
Routh-array s2 + 6s + 5 = 0

s3 1 30 s2 1 5
s2 13 k s1 6 0
13 × 30 k 0
s 5 0
s1
13 There is not root in RHS of s-plane.
s0 k
17. (a)
According to Routh-Hurwitz criterion.
The characteristic equation,
For a stable system, signs of first column do not
1 + G(s) H(s) = 0
chane,
s + as2 + (k + 2) s + (k + 1) = 0
3
k>0
Routh array for above equation,
13 × 30 k
and
13
>0 s3 1 ( k + 2)
2
Therefore sytem to be stable 0 < k < 390. s a ( k + 1)
a ( k + 2) ( k + 1)
15. (d) s1 0
a
Routh-array: s0 ( k + 1) 0
3
s 2 2 as2 + (k + 1) = 0
2 For s = j and =2
s 4 4
2× 4 4×2 –a 2 + (k + 1) = 0
s1 =0
4 k = 4a – 1
The third row vanishes. k+1
and a=
An auxiliary equation is formed using elements k+2
of 2nd row. a = 0.75 and k = 2
Auxiliary equation, A(s) = 4s2 + 4 = 0 s = ± j.
GATE Previous Years Solved Paper 79

18. (b) 20. Sol.


When all the elements of a row in Routh-Hurwitz K
Given, G(s) =
array ends abruptly i.e. all elements of that row s(s + 2) ( s 2 + 2 s + 2)
have a zero values then, the system will be either
R(s) + G(s ) Y(s )
unstable or marginally stable. Marginally stable –
means it will have imaginary roots (two equal
and opposite complex roots on imaginary axis 1
of s-plane). The characteristic equation of the given unity-
19. Sol. negative feedback control system is given by
1 + G(s) H(s) = 0
(s + )
Given, G(s) = K
3
s + (a + ) s + ( 2
1) s + (1 ) or, 1+ =0
s(s + 2) ( s 2 + 2 s + 2)
and H(s) = 1 or, s(s + 2) (s2 + 2s + 2) + K = 0
Characteristic equation of given control system
or, s4 + 4s3 + 6s2 + 4s + K = 0
is given by
Forming Routh’s array as shown below,
1 + G(s) H(s) = 0

s+ s4 1 6 K
or, 1 + =0 3
s 3 + (1 + ) s 2 + ( 1) s + (1 ) s 4 4 0
2
or, s3 + (1 + ) s2 + ( – 1) s + (1 – ) + s + =0 s 5 K 0
or, s3 + (1 + ) s2 + s + 1 = 0 20 4 K
s1 0 0
Routh’s array is, 5
s0 K 0 0
3
s 1
s2 (1 + ) 1 For stability of the system, K > 0 and
2
+ 1 20 4 K
s1 0 > 0 or K < 5
1+ 5

s0 1 0 For stable, 0 < K<5


For the given system to be marginally stable,
For the given system to be stable, there should K=5
not be any sign change in the first column of
Routh’s array. 22. (d)
1 + > 0 or > –1 ...(i) Response of transfer function is unit for all .
2 M = 1, P1 = 2 – j3
+ 1
Also, > 0 or 2 + – 1 > 0 Second order system, hence number of poles = 2
1+
Therefore, second pole, P2 = 2 + j3.
or, ( + 1.618) ( – 0.618) > 0
Now for M = 1, and due to x-axis symmetry of
or, < – 1.618 or > 0.618
root locus of transfer function, position of zeroes
As, > –1 [(using equation (i)]
must be Z1 = –2 – j3 and Z2 = –2 + j3.
> 0.618 ...(ii)
Combining conditions (i) and (ii), 23. Sol.
–1 < < 0.618 s3 + 5.5s2 + 8.5s + 3 = 0
Thus, for the system to be stable, minimum value Putting, s = s1 – 1
of = 0.618. (s1 – 1) + 5.5(s1 – 1)2 + (8.5) (s1 – 1) + 3 = 0
3
80 Electronics Engineering Control Systems

or, K2 + 2K – 3 < 0
s13 + 2.5 s12 + 0.5s1 1 = 0
or, (K + 3) (K – 1) > 0
s13 1 0.5 Here, the valid answer will be out of all the
options given.
s12 2.5 1
i.e., K>1
s1 0.9 0
s10 1 26. (a)

As there is one sign change hence, two roots of From the given equation,
s3 + 3s2 + 2s + K = 0
given polynomial will lie to the left of s = –1.
Using Routh’s criterion, we get
24. (c) K < 6 and K > 0 or 0 < K < 6
Open-loop transfer function:
27. (a)
K (s + 1)
G(s) = ; K > 0 and T > 0 Characteristic equation,
s(1 + Ts) (1 + 2 s)
s7 + s6 + 7s5 + 14s4 + 31s3 + 73s2 + 25s + 200 = 0
For closed-loop system stability, characteristic
s7 1 7 31 25
equation is,
6
1 + G(s) H(s) = 0 s 1 14 73 200
5
K (s + 1) s 7 42 175 0
1+ 1 =0
s(1 + Ts ) (1 + 2s) s4 8 48 200 0
s(1 + Ts) (1 + 2s) + K(s + 1) = 0 3
s 32 96 0 0
2
2Ts3 + (2 + T ) s 2 + (1 + K ) s + K = 0 s 24 200 0 0
1
s 170 0 0 0
Using Routh’s criteria,
s0 200 0 0 0
s3 2T (1 + K )
Auxiliary equation,
s2 (2 + T ) K
A(s) = 8s4 + 48s2 + 200
(2 + T ) (1 + K ) 2TK
s1 0 d
(2 + T ) A(s ) = 32s3 + 96s
ds
s0 K Total number of poles = 7
For stability, K > 0 and (2 + T) (1 + K) – 2TK > 0 Two sign change above auxiliary equation =
K(2 + T – 2T) + (2 + T) > 0 two-poles in RHS.
or, –(T – 2) K + 2(2 +T) > 0 Two sign changes below auxiliary equation
(2 + T ) implies out of 4 symmetric roots about origin
T +2
–K > or K < two-poles are in LHS and two-poles are in RHS.
(T 2) (T 2)
Three poles in LHS and 4 poles in RHS.
Hence for stability,
T +2 28. (c)
0 < K<
T 2 Routh array is,

25. (d) s4 1 3 K
Characteristic equation is, s 3
3 1
s3 + Ks2 + (K + 2)s + 3 = 0 2
s 8 /3 K
For this system to be stable, using Routh’s
8
criterion, we can write, 3K
1 3
3 < K(K + 2) s
8 /3
s0 K
GATE Previous Years Solved Paper 81

For BIBO stability, 30. (a)


8 C.E. is, 1 + G(s) H(s) = 0
3K
3 s2 + s + 1
>0
8 1+ 3 =0
s + 2s 2 + 2s + K
3
s3 + 3s2 + 3s + (1 + K) = 0
8 R.H. criteria:
K< and K > 0
9
8 s3 1 3
0 < K< 2
9 s 3 (1 + K )
s1 9 (1 + K ) 0
29. (b)
For marginal stability,
20
1+ 2 =0 9 – (1 + K) = 0
s (s + 1) ( s + 20)
K=8
(s3 + s2) (s + 20) + 20 = 0
s4
+ 20s3 + s3 + 20s2 + 20 = 0
s4 + 21s3 + 20s2 + 20 = 0
s1 coefficeint = 0
Given system is 4th order system and unstable.
5 Root Locus

ELECTRO NICS EN GINEERIN G Im


(GATE Previous Years Solved Papers)

Q.1 Consider a closed-loop system shown in Fig. (a) –1


(b) Re
below. The root locus for it is shown in Fig. (b).
–3
The closed-loop transfer function for the system
is
R(s) + K G (s ) c (t)

Im
Fig. (a)

(c) Re
–3 –1
–1.05
Re
–2 –0.1

Fig. (b) Im

K
(a)
1 + (0.5s + 1) (10 s + 1)
–3
K (d) Re
(b) –1
(s + 2) (s + 0.1)
K
(c)
1 + K (0.5s + 1) (10s + 1)
K [EC-1989 : 2 Marks]
(d)
K + 0.2(0.5s + 1) (10s + 1)
Q.3 The transfer function of a closed-loop system is
[EC-1988 : 2 Marks]
K
T (s) = 2
Q.2 The OLTF of a feedback system is, s + (3 K ) (s + 1)
K (s + 1) (s + 3) where K is the forward path gain. The root locus
G(s) H (s) =
s 2 + 4s + 8 plot of the system is
The root locus for the same is j

Im

(a)
(a)
Re
–3 –1
GATE Previous Years Solved Paper 83

Q.5 Given a unity feedback system with open-loop


j
transfer function,
K=3
K
G(s) =
s (s + 1) (s + 2)
K=1
The root locus plot of the system is of the form.
(b) j
K=0

K=3 (a)

K=

j
(c)
K=0 K=0

K=

(b)

j
(d)

(c)

[EC-1990 : 2 Marks]

Q.4 The characteristic equation of a feedback control


system is given by
s3 + 5s2 + (K + 6) s + K = 0
j
where K > 0 is a scalar variable parameter. In
the root-locus diagram of the system the
asymptotes of the root-loci for large values of K
meet at a point in the s-plane, whose coordinates
are (d)
(a) (–3, 0) (b) (–2, 0)
(c) (–1, 0) (d) (2, 0)
[EC-1991 : 2 Marks]
[EC-1992 : 2 Marks]
84 Electronics Engineering Control Systems

Q.6 If the open-loop transfer function is a ratio of a


(a) s = j 3 (b) s = –1.5
numerator polynomial of degree ‘m’ and a
denominator polynomial of degree ‘n’ then the (c) s = –3 (d) s = –
integer (n-m) represents the number of [EC-2002 : 1 Mark]
(a) breakaway points Q.10 The root locus of the system
(b) unstable poles
K
(c) separate root loci G(s) H (s) =
s (s + 2) (s + 3)
(d) asymptotes
has the break-away point located at
[EC-1994 : 1 Mark]
(a) (–0.5, 0) (b) (–2.548, 0)
Q.7 Consider the points s1 = –3 + j4 and s2 = –3 – j2 (c) (–4, 0) (d) (–0.784, 0)
in the s-plane. Then for a system with the open- [EC-2003 : 2 Marks]
loop transfer function
K K
G(s ) H (s ) = Q.11 Given, G(s) H (s) = the point of
(s + 1) 4 s (s + 1) (s + 3)

(a) s1 is on the root locus, but not s2. intersection of the asymptoes of the root loci
(b) s2 is on the root locus, but not s1. with the real axis is
(c) both s1 and s2 are on the root locus. (a) –4 (b) 1.33
(d) neither s1 nor s2 is on the root locus. (c) –1.33 (d) 4
[EC-1999 : 2 Marks] [EC-2004 : 1 Mark]

Q.8 The root-locus diagram for a closed-loop Q.12 An unity feedback system is given as,
feedback system shown in the figure. The system K (1 s )
G(s ) =
is overdamped. s (s + 3)
j Indicate the correct root locus diagram,

j
K=5
K= K=0 K=0

K –3 –2 –1

K=1 (a)

(a) only if 0 K 1
(b) only if 1 < K < 5
(c) only if K > 5
j
(d) if 0 K < 1 or K > 5
[EC-2001 : 1 Mark]

Q.9 Which of the following points is not on the root


locus of a system with the open-loop transfer (b)
K
function, G(s) H (s) = ?
s (s + 1) (s + 3)
GATE Previous Years Solved Paper 85

j (a) ± 2 and 0° (b) ± 2 and 45°

(c) ± 3 and 0° (d) ± 3 and 45°


[EC-2009 : 2 Marks]
(c) Q.15 The root locus plot for a system is given below.

–3 –2 –1 0

(d)

The open-loop transfer function corresponding


to this plot is given by
[EC-2005 : 2 Marks] s (s + 1)
(a) G(s ) H (s ) = K
(s + 2) (s + 3)
Q.13 A unity feedback control system has an open-
loop transfer function, ( s + 1)
(b) G(s ) H (s ) = K
K s (s + 2) ( s + 3)2
G( s ) =
s (s 2 + 7 s + 12) 1
(c) G(s) H (s) = K
The gain K for which s = –1 + j1 will lie on the s (s 1) (s + 2) (s + 3)
root locus of this system is
(s + 1)
(a) 4 (b) 5.5 (d) G(s) H (s) = K
s (s + 2) (s + 3)
(c) 6.5 (d) 10
[EC-2011 : 1 Mark]
[EC-2007 : 2 Marks]
Q.16 In the root locus plot shown in the figure, the
Q.14 The feedback configuration and the pole-zero
pole/zero marks and the arrows have been
removed. Which one of the following transfer
s2 2s + 2
locations of G(s ) = are shown below. functions has this root locus?
2
s + 2s + 2
The root locus for negative values of K, i.e. for j
– < K < 0, has breakaway/break-in points and
angle of departure at pole P (with respsect to
the positive real axis) equal to 2

Im(s)
1
+ K G(s)

Re(s)
86 Electronics Engineering Control Systems

s+1 Q.21 The forward path transfer function and the


(a)
(s + 2) (s + 4) (s + 7) feedback path transfer function of a single-loop
negative feedback control system are given as,
s+4
(b) K (s + 2)
(s + 1) (s + 2) (s + 7) G(s) = and H(s) = 1
2
s + 2s + 2
s+7 respectively. If the variable parameter K is real
(c)
(s + 1) (s + 2) (s + 4) positive, then the location of the breakaway
(s + 1) (s + 2) point on the root-locus diagram of the system is
(d) ______ . [EC-2016 : 2 Marks]
(s + 7) (s + 4)
[EC-2014 : 2 Marks] Q.22 A linear time invariant (LTI) system with the

Q.17 A unity negative feedback system has the open- K (s 2 + 2 s + 2)


transfer function, G(s ) = is
loop transfer function, (s 2 3s + 2)
K
G(s) = connected in unity feedback configuration as
s (s + 1) s ( +3)
shown in the figure.
The value of the gain K (> 0) at which the root
locus crosses the imaginary axis is _____ . + G(s )

[EC-2015 : 1 Mark]

Q.18 The open-loop transfer function of a unity For the closed-loop system shown, the root-locus
feedback configuration is given as, for 0 < K < intersects the imaginary axis for
K = 1.5. The closed-loop system is stable for
K (s + 4)
G( s ) = (a) K > 1.5
( s + 8) (s 2 9)
(b) 1 < K < 1.5
The value of gain K (> 0) for which –1 + j2 lies
(c) 0 < K < 1.5
on the root locus is ______ .
(d) no positive value of K
[EC-2015 : 2 Marks]
[EC-2017 : 2 Marks]
Q.19 For the sysetm shown in figure, s = –2.75 lies on
Q.23 The root-locus plot of a closed-loop system with
the root locus if K is ______ .
unity negative feedback and transfer function
s+3 KG(s) in the forward path is shown in the figure.
+ K Y(s )
– s+2
Note that, K is varied from 0 to . Select the
transfer function G(s) that results in the root-
10 locus plot of the closed-loop system as shown
in the figure.
[EC-2015 : 2 Marks]
Im (s)

Q.20 The open-loop transfer function of a unity


feedback control system is
72°
K
G( s ) =
s 2 + 5s + 5 72° 36°
36°
The value of K at the breakaway point of the (0, 0) Re (s)
72°
feedback control system’s root-locus plot is __ . 72°

[EC-2016 : 2 Marks]
1.0
GATE Previous Years Solved Paper 87

1 1 Q.2 Which of the following figure(s) represent valid


(a) G(s ) = (b) G(s ) =
(s + 1)5 s5 + 1 root loci in the s-plane for positive K ? Assume
that the system has transfer function with real
s 1 s 1
(c) G(s ) = (d) G(s ) = coefficient.
(s + 1) 6
s6 + 1
[EC-2022]

ELECTRICAL EN GINEERIN G
(a)
(GATE Previous Years Solved Papers)

Q.1 A unity feedback system has an open-loop

K(s + a)
transfer function of the form KG(s ) = ;
s2 (s + b )
b > a which of the loci shown in figure can be
valid root-loci for the system?
(b)
j

(a)
–b –a 0

(c)
j

(b) 0
–b –a

(d)

[EE-1992 : 1 Mark]
–a
(c)
0
–b Q.3 In case of an armature controlled separately
excited dc motor drive with closed-loop speed
control, an inner current loop is useful because it
(a) limits the speed of the motor to a safe value.
j
(b) helps in improving the drive energy
efficiency.
(c) limits the peak current of the motor to be
(d)
–b –a 0 permissible value.
(d) reduces the steady-state speed error.
[EE-2001 : 1 Mark]

[EE-1991 : 1 Mark]
88 Electronics Engineering Control Systems

Q.4 A unity feedback system has an open-loop


Im
K
transfer function, G(s ) = . Its root locus plot
s2 2 3

will be 3

j Re
–3 –2 –1 1 2 3
3

(a) 2 3

K K
(a) 3 (b) 2
s s (s + 1)
j K
K
(c) 2
(d) 2
s(s + 1) s(s 1)
[EE-2005 : 1 Mark]
(b)
Q.6 A closed-loop system has the characteristic
function (s2 – 4) (s + 1) + K(s – 1) = 0. Its root
locus plot against ‘K’ is

j
j

(a)
–2 –1 +1 +2
(c)

(b)
–2 –1 +1 +2

(d)

[EE-2002 : 2 Marks]
(c)
–2 –1 +1 +2
Q.5 Figure shows the root locus plot (location of
poles not given) of a third order system whose
open-loop transfer function is
GATE Previous Years Solved Paper 89

j The closed-loop transfer function of the system


is
C (s) K
(d) (a) =
–2 –1 +1 +2
R(s) ( s + 1) (s + 2)
C(s) K
(b) =
R(s) (s + 1) (s + 2) + K
[EE-2006 : 2 Marks] C (s) K
(c) =
R(s) (s + 1) (s + 2) K
Q.7 The characteristic equation of a closed-loop
system is, C(s) K
(d) =
s(s + 1) (s + 3) + k(s + 2) = 0, k > 0 R(s) (s + 1) (s + 2) + K
Which of the following statements is true? [EE-2014 : 1 Mark]
(a) Its roots are always real.
Q.10 The open-loop poles of a 3 rd order unity
(b) It cannot have a breakaway point in the
feedback system are at 0, –1, –2. Let the frequency
range –1 < Re[s] < 0.
corresponding to the point where the root locus
(c) Two of its roots tend to infinity along the
of the system transits to unstable region be K.
asymptotes Re[s] = –1.
Now suppose we introduce a zero in the open-
(d) It may have complex roots in the right half
loop transfer function at –3, while keeping all
plane.
the earlier open-loop poles intact. Which one of
[EE-2010 : 2 Marks] the following is true about the point where the
Q.8 The open-loop transfer function G(s) of a unity root locus of the modified system transits to
feedback control system is given as, unstable region.

2 (a) It corresponds to a frequency greater than K.


k s+
3 (b) It corresponds to a frequency less than K.
G(s) = 2
s (s + 2) (c) It corresponds to a frequency K.
From the root locus, it can be inferred that when (d) Root locus of modified system never transits
‘k’ tends to positive infinity to unstable region.
(a) three roots with nearly equal real parts exist [EE-2015 : 2 Marks]
on the left half of the s-plane.
Q.11 An open-loop transfer function G(s) of a system
(b) on real root is found on the right half of the is
s-plane.
K
(c) the root loci cross the j -axis for a finite G(s) =
s(s + 1) (s + 2)
value of k : k 0.
(d) three real roots are found on the right half For a unity feedback system, the breakaway
of the s-plane. point of the root loci on the real axis occurs at,
[EE-2011 : 2 Marks] (a) –0.42 (b) –1.58
(c) –0.42 and –1.58 (d) None of the above
Q.9 The root locus of a unity feedback system is
[EE-2015 : 2 Marks]
shown in the figure.
j Q.12 The gain at the breakaway point of the root locus
of a unity feedback system with open-loop
K=0 K=0
Ks
transfer function G(s) = is
–2 –1 (s 1) (s 4)
90 Electronics Engineering Control Systems

(a) 1 (b) 2 where K > 0, enters into the real axis at


(c) 5 (d) 9
(a) s = –1 (b) s = 5
[EE-2016 : 2 Marks]
(c) s = –5 (d) s = 5
Q.13 The root locus of the feedback control system
[EE-2017 : 2 Marks]
having the characteristic equation,
s2 + 6 Ks + 2s + 5 = 0

Electronics & Electrical Engineering


GATE Previous Years Solved Paper

A n swe rs & Expl a n a t i o n s

Answers
EC Root Locus

1. (d) 2. (a) 3. (a) 4. (b) 5. (a) 6. (d) 7. (b) 8. (d)

9. (b) 10. (d) 11. (c) 12. (c) 13. (d) 14. (*) 15. (b) 16. (b)

17. (12) 18. (25.54) 19. (0.3) 20. (1.25) 21. (–3.41) 22. (a) 23. (a)

Solutions
EC Root Locus

1. (d) 4. (b)
1 K (s + 1)
G(s) = O.L.T.F. = G(s) H (s) = 3
(s + 0.1) (s + 2) s + 5s 2 + 6s
K K (s + 1)
K G(s ) (s + 0.1) ( s + 2) =
T.F. = = s(s 2 + 5s + 6)
1 + KG( s) K
1+
(s + 0.1) (s + 2) K (s + 1)
G(s) H(s) =
K s(s + 2) (s + 3)
T.F. =
(s + 0.1) (s + 2) + K Poles: 0, –2, –3
K Zeros: –1
T.F. =
K + 0.2 (1 + 10 s) (1 + 0.5 s) ( 2 3) ( 1)
Centroid =
3 1
2. (a)
5+1
(b) and (d) options are wrong, because root locus = = 2 = ( 2, 0)
2
is symmetrical about real axis.
Option (c) is wrong because root locus directions
are from pole to zeros.
GATE Previous Years Solved Paper 91

5. (a) 10. (d)


P=3 1 + G(s) H(s) = 0
Z=0 K
1+ =0
P–Z = 3–0=3 s(s + 2) (s + 3)
(2 q + 1) 180°
= K
P Z 1+ 2 =0
s (s + 5s + 6)
= 60°, 180°, 300°
So, only option (a) satisfy this. K = –1 (s3 + 5s2 + 6s)
dK
6. (d) = –(3s2 + 10s + 6)
ds
(n – m) For break-away point,
difference between poles and zeros give number dK
of asymptotes. =0
ds
7. (b) 3s2 + 10s + 6 = 0

s1 = –3 + 4j 10 ± 100 72
s=
K K 6
G(s) H(s) = 4
= 10 ± 5.3
( 3 + 4 j + 1) ( 2 + 4 j )4 = = 0.784, 2.55
6
G(s) H(s) = –4[ – tan–1 2]
= –466° odd multiple of Img

So, s1 does not lie on root locus,


s2 = –3 – j2
K K
G(s) H(s) = 4
= Re
( 3 j 2 + 1) ( 2 2 j )4 –3 –2 0

1
G(s) H(s) = 4[tan 1] = 4 × =
4
Odd multiple of
s2 lies on root locus. 2 3 5
Centroid = = = 1.66
3 3
8. (d)
For overdamping roots of characteristic Angle of asymptotes = (2K + 1)
P Z
equation should lie on negative axis and be
unequal. = (2K + 1)
3
9. (b) 5
, ,
3 3
Img
Break-away point is (–0.784, 0) as –2.55 does
not lie on RL.

11. (c)
Re
–3 –1 0
P Z
Centroid =
P Z
RL lies where number of poles and zeros to the
1 3
right of the pole is odd. = = 1.33
3
s = –1.5 does not lie on RL
92 Electronics Engineering Control Systems

12. (c) K
Put, = 0 we have
1 + G(s) H(s) = 0 s
(s2 – 2s + 2) (s + 1) – (s2 + 2s + 2) (s – 1) = 0
s 2 + 3s
K= 2s2 – 4s2 + 4 = 0
1 s
2s2 = +4
For break-away and break in point,
s= ± 2
dK
= (1 – s) (2s + 3) + s2 + 3s = 0 Angle of departure is
ds
= –s2 + 2s + 3 = 0 D =
2 where, = Z–
s – 2s – 3 = 0 P
= 225°
(s – 3) (s + 1) = 0
s = 3, –1 D = ±225°
No option is matching.
–1 is the break-away point and 3 is the break-in
point. 15. (b)

13. (d) From plot we can observe that one pole


terminates at one zero at position –1 and three
G(s)
T.F. = poles terminates to . It means three are 4-poles
1 + G(s) and 1 zero. Pole at –3 goes on both sides. It
As, H(s) = 1 means there are 2-poles at –3.
For the point, s = –1 + j1 to lie on root locus,
1 + G(s) = 0 16. (b)

K j
1+ 2 =0
s(s + 7 s + 12)
s(s2 + 7s + 12) + K = 0
Putting, s = –1 + j
–7 –4 –2 –1
(–1 + j) (1 – 2j – 1 – 7 + 7j + 12) + K = 0
K = +10

14. ( )
Since the root locus always emerges from the
+ K G (s ) break-away points i.e. when two poles are at a

part of same root locus lie. So, = –1 and –2 will
surely be poles. Also we know that locus
emerges from poles and terminates at zero.
Im(s) So, s = –4 is a zero and s = –7 is a pole.
(s + 4)
T(s) =
Re(s) (s + 1) (s + 2) (s + 7)

17. Sol.
K
G(s) =
1 + G(s) H(s) = 0 s (s + 1) (s + 3)
K (s 2 + 2 s + 2) The characteristic equation
1+ =0
2
s + 2s + 2 = 1 + G(s) H(s) = 0
= s(s + 1) (s + 3) + K = 0
s 2 + 2s + 2
K= = s3 + 4s2 + 3s + K = 0
s2 2s + 2
GATE Previous Years Solved Paper 93

Using Routh’s tabular form, K 13


= =1
s3 1 3 53 8 20
2 53 × 8 × 20
s 4 K
K= = 25.54
12 K 13
s1 0
4
19. Sol.
s0 K
The open-loop transfer function of the system,
In order to cross the imaginary axis, system
K (s + 3) × 10
should be marginally stable. G(s) H(s) =
(s + 2)
12 K
=0 In order to lie a point, s = –2.75 the angle
4
condition must be satisfy and therefore the
or, K = 12
magnitude condition is,
18. Sol. G( s ) H ( s ) =1
s = 2.75
K (s + 4) K (s + 4) 0.25 × 10 K
G(s) = 2 = = =1 K = 0.3
(s + 8) (s 9) (s + 8) ( s + 3) ( s 3) 0.75
For the point (–1 + 2j) to lie on the root locus the
20. Sol.
angle condition must be satisfy.
Characteristic equation is,
i.e., G(s) H(s) = ±(2Q + 1) 180°
1 + G(s) H(s) = 0
Taking LHS:
K
G(s) H (s) s = 1+ 2 =0
1+ 2 j s + 5s + 5
K + (s + 4) K = –s2 – 5s – 5
= dK
(s + 8) + (s + 3) + (s 3) For break-away point =0,
ds
K + ( 1 + 2 j + 4)
= ( 1 + 2 j + 8) + ( 1 + 2 j + 3) + ( 1 + 2 j 3) dK
= –2s – 5 = 0
ds
2
0 + tan 1 s = –2.5
3
= 2 1 According to magnitude condition,
tan 1 + tan 1 (1) + 180° tan 1
7 2 G(s) H (s) s = =1
2.5
1 2 1 2 1 1 1
= tan tan tan 1 180° + tan K
3 7 2 G(s) H (s) s = = =1
2.5 2
= 33.69° – 15.945° – 45° – 180° + 26.565° ( 2.5) + 5 × 2.5 + 5
180°
As angle condition is satisfied the value of K = (6.25 + 5 12.5) = 1.25
system gain K can be obtained by using K = ±1.25
magnitude condition.
21. Sol.
i.e., G(s) H (s) s = 1+ 2 j
=1 K (s + 2)
G(s) = 2
; H (s) = 1
2 2
s + 2s + 2
K ( 1 + 4) + 2
= =1 1 + G(s) H(s) = 0
2 2
( 1 + 8) + 2 ( 1 + 3)2 + 2 2 ( 1 3)2 + 2 2
K
(s + 2) = 0
K 9+4 2
(s ) + 2s + 2
= =1
49 + 4 4 + 4 16 + 4
94 Electronics Engineering Control Systems

(s2 + 2s + 2) 22. (a)


K=
(s + 2) K (s 2 + 2 s + 2)
Given that, G(s) =
j s 2 3s + 2
The root locus plot of the given system is as
j
follows:
j
at K = 1.5
–2 –1 0

–j

0 –1 –2
2
dK (2s + 2) (s + 2) (s + 2s + 2)
=
ds (s + 2)2
dK
For break-away points, =0
ds System is stable for K > 1.5.
(2s + 2) (s + 2) – (s2 + 2s + 2) = 0
2s2 + 6s + 4 – s2 – 2s – 2 = 0 23. (a)
s2 + 4s + 2 = 0 There are 5 root-locus branches from the same
point, so there are 5 real multiple poles.
4 ± 16 8
s= So, correct option is (a).
2
4±2 2
= = 2± 2
2
s = –0.58, s = –3.41
But, s = –3.41 lies on root locus
Hence, s = –3.41
GATE Previous Years Solved Paper 95

Answers
EE Root Locus

1. (a) 2. (a) 3. (c) 4. (b) 5. (a) 6. (b) 7. (c) 8. (a)

9. (c) 10. (d) 11. (a) 12. (a) 13. (b)

Solutions
EE Root Locus

1. (a) These are three asymptotes with angle 60°, 180°


P = 3, Z = 1 and 300°.
and two repeating poles at origin, (2 k + 1) × 180°
Angle of asymptotes =
0 b ( a) P Z
Centroid = where, k = 0, 1, 2 upto (P – Z) – 1 as angles are
3 1
(b a) 60°, 180° and 300°.
= <0 It means, P–Z = 3
2
(2 q + 1)180° Intersection of asymptotes on real axis,
Angle of asymptotes =
3 1 poles zeros
x =
= 90° (for q = 0) P Z
= 270° (for q = 1) Since, system does not have zeros,
option (a) is correct.
poles
x =
2. (a) P
The valid root loci in the s-plane can be only As asymptotes intersect at origin, it means all
option (a), because of symmetry and root locus the three poles are at origin.
definition. Hence, option (a) is correct.

3. (c) 6. (b)
Closed-loop system limits the peak value. Characteristic function:
(s2 – 4) (s + 1) + K(s – 1) = 0
4. (b)
K (s 1)
K 1+ 2 = 1 + G(s) H(s)
G(s) H(s) = (s 4) ( s + 1)
2
s
Open-loop transfer function
P–Z = 2
= G(s) H(s)
0
Centroid = =0 K (s 1)
2 = 2
Angle of asymptotes = 90°, 270° (s 4) ( s + 1)
option (a) is correct. Zero of OLTF: s = 1, z = 1
Poles of OLTF: s = –1, –2, + 2, P = 3
5. (a)
Im
k=

k= 60°
Re –2 –1 1 2

s-plane
k=
96 Electronics Engineering Control Systems

The root locus starts from open-loop poles and


Im
terminates either on open-loop zero or infinity.
Root locus exists on a section of real axis it the
k=0 k=
sum of the open-loop poles and zeros to the right Re
of the section is odd. –3 –2 +1

Number of branches terminating on infinity


= P–Z=3–1=2
Angle of asymptotes Breakaway point lies in the range –1 < Re[s] < 0
(2 K + 1) × 180° (2K + 1) × 180° and two branches terminates at infinity along
= =
P Z 2 the asymptotes Re(s) = –1.
= 90° and 270°
8. (a)
Intersection of asymptotes on real axis (centroid)
poles zeros 2
K s+
= 3
P Z G(s) = 2 and H(s) = 1
s (s + 2)
( 1 2 + 2) (1)
= = 1 Characteristic equation,
2
Option (b) is correct on the basic of above 1 + G(s) H(s) = 0
analysis. 2
K s+
3
7. (c) 1+ 2 =0
s (s + 2)
Characteristic equation,
s(s + 1) (s + 3) K(s + 2) = 0 2
s 3 + 2s2 + K s + =0
K (s + 2) 3
1+ =0
s (s + 1) (s + 3) 2K
s 3 + 2 s 2 + Ks + =0
Comparing with 1 + G(s) H(s) = 0 3
G(s) H(s) = Open-loop transfer function (OLTF) Routy array:

K (s + 2) s3 1 K
=
s (s + 1) (s + 2) 2K
s2 2
3
Number of zeros = Z = 1 zero at –2
2K 2K /3 2
Number of poles = P = 3 poles at 0, –1 and –3 s1 = K
2 3
Number of branches terminating at infinity 2K
=P – Z = 3 – 1 = 2 s0
3
Angle of asymptotes
As K > 0, there is no sign change in the 1 st
(2 K + 1) × 180° (2K + 1) × 180° column of Routh array. So the system is stable
= =
P Z 2 and all the three roots lie on LHS of s-plane.
= (2K + 1) × 90° = 90° and 270° For K > 0 (K 0), none of the row of Routh array
poles zeros becomes zero. So root loci does not cross the
Centroid = j -axis.
P Z
Number of zero = Z = 1
0 1 3 ( 2)
= = 1 Number of poles = P = 3
2
Number of branches terminating at infinity
= P–Z=3–1=2
GATE Previous Years Solved Paper 97

(2 K + 1) × 180° Asymptotes angle = 90°, 180°


Angle of asymptotes =
P Z ( 3) ( 3)
Centroid = =0
(2 K + 1) × 180° 2
= As root locus never cross asymtptoes so, will
2
= (2K + 1) × 90° remain in left side of x-axis.
= 90° and 270° 11. (a)
poles zeros K
Centroid = G1(s) =
P Z s (s + 1) (s + 2)
2
0+0 2 K
3 2 1+ =0
= = s (s + 1) (s + 2)
2 3
s(s2 + 3s + 2) + K = 0
s3 + 3s2 + 2s + K = 0
K = –s3 – 3s2 – 2s ...(i)
dK
=0
ds
–2 –2/3 Double pole dK
at origin = –3s3 – 6s – 2 = 0
ds
s = –0.422, –1.577

Since, all the three branches terminates at

2 RL NRL RL
Re(s) = . So, all the three roots have nearly
3 –2 –1 0

equal real part.

9. (c) Only s = –0.422 lie on root locus. Therefore


This is converse root locus having no zero. breakaway point is, s = –0.42.
K
As its, G(s) H(s) = 12. (a)
(s + 1) (s + 2)
Its converse root locus only valid when K < 0 Ks
OLTF G(s) =
(s 1) (s 4)
G(s ) H (s )
So, C.L.T.F. = Now characteristics equation,
1 G(s) H (s)
1 + G(s) H(s) = 0
K
Ks
(s + 1) (s + 2) +1 = 0
= (s 1) (s 4)
K
1
(s + 1) (s + 2) (s2 5s + 4)
K=
K s
=
(s + 1) (s + 2) K 4
= s 5+
s
10. (d)
dK
For breakaway point : =0
K (s + 3) ds
s (s + 1) (s + 2) dK 4
= 1 0 =0
P = 3, Z = 1 ds s2
98 Electronics Engineering Control Systems

we get, s = ±2
Therefore valid breakaway point is
s=2
Now gain at s = 2 is,
Product of distances from all
the poles to breakaway point The point at which root locus enters real axis
K=
Product of distance from all (breakaway point) is given by
the zeros to breakaway point dK
=0
ds
1× 2
Gain, K= =1
2 (s 2 + 2s + 5)
K=
6s
13. (b)
dK
=0 s=± 5
6Ks ds
GH(s) =
s2 + 2s + 5 5
s=
6 Frequency Response Analysis

ELECTRO NICS EN GINEERIN G 2 4 2


(c) n (1 2 )+ 4 4 +2
(GATE Previous Years Solved Papers)
2 4 2
(d) n (1 2 ) 4 4 +2
Q.1 A system has fourteen poles and two zeros. Its
high frequency asymptotes in magnitude plot [EC-1994 : 1 Mark]
having a slope of
Q.5 The open-loop frequency response of a system
(a) –40 dB/decade
at two particular frequencies are given by
(b) –240 dB/decade 1.2 –180° and 1.0 –190°. The closed-loop unity
(c) –280 dB/decade feedback control is then _______ .
(d) –320 dB/decade [EC-1994 : 1 Mark]
[EC-1987 : 2 Marks]
Q.6 Non-minimum phase transfer function is
defined as the transfer function
10
Q.2 The polar plot of G(s ) = intercepts real (a) which has zeros in the right-half s-plane.
s (s + 1)2
(b) which has zeros in the left-half s-plane.
axis at = o. Then, the real part and o are
(c) which has poles in the right-half s-plane.
respectively given by
(d) which has poles in the left-half s-plane.
(a) –2.5, 1 (b) –5, 0.5
[EC-1995 : 1 Mark]
(c) –5, 1 (d) –5, 2
[EC-1987 : 2 Marks] Q.7 In the Bode-plot of a unity feedback control
system, the value of phase of G(j ) at the gain
Q.3 The open-loop transfer function of a feedback
crossover frequency is –125°. The phase margin
control system is
of the system is
1
G(s ) H (s ) = (a) –125° (b) –55°
(s + 1)2
(c) 55° (d) 125°
The gain margin of the system is [EC-1998 : 1 Mark]
(a) 2 (b) 4
Q.8 The Nyquist plot of a loop transfer function
(c) 8 (d) 16
G(j ) H(j ) of a system encloses the (–1, 0) point.
[EC-1991 : 2 Marks]
The gain margin of the system is
Q.4 The 3-dB bandwidth of a typical second-order (a) less than zero
system with the transfer function (b) zero
2 (c) greater than zero
C (s) n
=
R(s ) s 2 + 2 ns +
2
n (d) infinity
is given by [EC-1998 : 1 Mark]

(a) 1 2 2 Q.9 The gain margin (in dB) of a system having the
n

2
(b) (1 2
)+ 4 2
+1 loop transfer function G(s ) H (s ) = is
n s (s + 1)
100 Electronics Engineering Control Systems

(a) 0 (b) 3 (a) 0° (b) 63.4°


(c) 6 (d) (c) 90° (d)
[EC-1999 : 1 Mark] [EC-2002 : 1 Mark]

Q.10 The phase margin (in degrees) of a system Q.14 The gain margin and the phase margin of a
having the loop transfer function
s
feedback system with G(s ) H (s ) = are
2 3 (s + 100)3
G( s ) H ( s ) = , is
s (s + 1) (a) 0 dB, 0° (b) ,
(c) , 0° (d) 88.5 dB,
(a) 45° (b) –30°
[EC-2003 : 2 Marks]
(c) 60° (d) 30°
[EC-1999 : 1 Mark] Q.15 In the figure, the Nyquist plot of the open-loop
transfer function G(s) H(s) of a system is shown.
Q.11 The Nyquist plot for the open-loop transfer
If G(s) H(s) has one right hand pole, the closed-
function G(s) of a unity negative feedback system loop system is
is shown in the figure, if G(s) has no pole in the
Im
right-half of s-plane, the number of roots of the
system characteristic equation in the right-half GH-plane
of s-plane is
=0 Re
(–1, 0)
Im(s)

G(s)-plane positive

(a) always stable.


–1 (b) unstable with one closed-loop right hand
Re(s) pole.
(c) unstable with two closed-loop right hand
poles.
(d) unstable with three closed-loop right and
poles.
(a) 0 (b) 1 [EC-2003 : 1 Mark]
(c) 2 (d) 3
Q.16 The approximate Bode magnitude plot of a
[EC-2001 : 1 Mark]
minimum phase system is shown in the figure.
Q.12 The system with the open-loop transfer function dB
1P
1 160
G( s ) H ( s ) = has a gain margin of
s (s 2 + s + 1) 140
2P

(a) –6 dB (b) 0 dB
(c) 3.5 dB (d) 6 dB 3Z
20
[EC-2002 : 2 Marks]
0.1 10 100
Q.13 The phase margin of a system with the open-
loop transfer function, The transfer function of the system is

(1 s) (s + 0.1)3
G(s) H (s) = (a) 108
(1 + s) (2 + s ) (s + 10)2 ( s + 100)
GATE Previous Years Solved Paper 101

7 (s + 0.1)3 Im
(b) 10
(s + 10) (s + 100)

(s + 0.1)2 (b)
(c) 108 2 =0 =
(s + 10) (s + 100) Re

(s + 0.1)3
(d) 109
(s + 10) (s + 100)2
Im
[EC-2003 : 2 Marks]

Q.17 Consider the Bode magnitude plot shown in the (c)


figure. The transfer function H(s) is =0
Re
dB =

20 log H ( j )
0 Im
–20 dB/decade

–20 (d)
= =0
Re
1 10 100

(s + 10) 10(s + 1) [EC-2005 : 1 Mark]


(a) (b)
(s + 1) (s + 100) (s + 10) (s + 100)
Q.20 The polar diagram of a conditionally stable
2
10 (s + 1) 10 (s + 100)3 system for open-loop gain K = 1 is shown in the
(c) (d) figure. The open-loop transfer function of the
(s + 10) (s + 100) (s + 1) (s + 10)
system is known to be stable. The closed-loop
[EC-2004 : 2 Marks]
system is stable for
Q.18 A system has poles at 0.01 Hz, 1 Hz and 80 Hz,
Im
zeros at 5 Hz, 100 Hz and 200 Hz. The
approximate phase of the system response at
20 Hz is –8 –0.2

(a) –90° (b) 0°


Re
(c) 90° (d) –180°
–2
[EC-2004 : 2 Marks]

Q.19 Which one of the following polar diagrams


corresponds to a lag network? GH-plane

Im

(a) 1 1
(a) K < 5 and <K<
= 2 8
Re
=0 1 1
(b) K < and < K < 5
8 2
102 Electronics Engineering Control Systems

1 Q.25 In the system shown below, x(t) = (sint) u(t). In


(c) K < and 5 < K
8 steady-state, the response y(t) will be

1 1
(d) K < and K < 5 x(t ) y(t )
8 s+1

[EC-2005 : 2 Marks] 1
1
(a) sin t (b) sin t +
Common Data for Questions (21 and 22): 2 4 2 4
The open-loop transfer function of a unity feedback 1 t
(c) e sin t (d) sint – cost
2s
2
3e
system is given by G(s ) = . [EC-2006 : 1 Mark]
s (s + 2)
Q.26 The frequency response of a linear, time-
Q.21 The gain and phase crossover frequencies in invariant system is given by
rad/sec are, respectively 5
H( f ) =
(a) 0.632 and 1.26 (b) 0.632 and 0.485 1 + j 10 f
(c) 0.485 and 0.632 (d) 1.26 and 0.632
The step response of the system is
[EC-2005 : 2 Marks]
(a) 5(1 – e–5t) u(t) (b) 5(1 – e–t/5) u(t)
Q.22 Based on the above results, the gain and phase 1 5t 1
(c) (1 e ) u(t ) (d)
margins of the system will be 5 (s + 5) (s + 1)
(a) –7.09 and 87.5° [EC-2007 : 2 Marks]
(b) 7.09 and 87.5°
Q.27 The asymptotic Bode plot of a transfer function
(c) 7.09 dB and –87.5°
is as shown in the figure. The transfer function
(d) –7.09 dB and –87.5°
G(s) corresponding to this Bode plot is
[EC-2005 : 2 Marks]
G( j ) (dB)
Q.23 The open-loop transfer function of a unity gain
feedback control system is given by 60
–20 dB/decade
K 40
G(s) =
(s + 1) (s + 2) –40 dB/decade
20
The gain margin of the system in dB is given by 20
0
(a) 0 (b) 1 0.1 1 10 100
(c) 20 (d)
–60 dB/decade
[EC-2006 : 1 Mark]
1 1
Q.24 The Nyquist plot of G(j ) H(j ) for a closed-loop (a) (b)
(s + 1) ( s + 20) s (s + 1) (s + 20)
control system, passed through (–1, j0) point in
the GH-plane. The gain margin of the system 100 100
(c) (d)
(in dB) is equal to s (s + 1) (s + 20) s (s + 1) (1 + 0.05s)
(a) infinite [EC-2007 : 2 Marks]
(b) greater than zero
Q.28 The magnitude of frequency response of an
(c) less than zero underdamped second order system is 5 at
(d) zero
10
[EC-2006 : 2 Marks] 0 rad/sec and peaks to at 5 2 rad/sec.
3
GATE Previous Years Solved Paper 103

The transfer function of the system is 10 s + 1 100 s + 1


(a) (b)
500 375 0.1 s + 1 0.1 s + 1
(a) 2
(b) 2
s + 10s + 100 s + 5s + 75 100 s 0.1 s + 1
(c) (d)
720 1125 10 s + 1 10 s + 1
(c) 2 (d) 2
s + 12 s + 144 s + 25s + 225 [EC-2010 : 1 Mark]
[EC-2008 : 2 Marks]
Y (s ) s
Common Data for Questions (29 and 30): Q.32 A system with the transfer function =
X( s ) s + p
The Nyquist plot of a stable transfer function G(s) is
shown in the figure. We are interested in the stability of
the closed-loop system in the feedback configuration has an output y(t ) = cos 2t for the input
3
shown.

Im signal x(t ) = p cos 2t . Then, the system


2

parameter ‘p’ is
+

G(s) 2
–1 –0.5 Re (a) 3 (b)
3
–j
3
(c) 1 (d)
2
[EC-2010 : 1 Mark]
Q.29 Which of the following statement is true?
(a) G(s) is an all-pass filter. Q.33 For the transfer function G(j ) = 5 + j , the
(b) G(s) has a zero in the right-half palne. corresponding Nyquist plot for positive
(c) G(s) is the impedance of a passive network. frequency has the form
(d) G(s) is marginally stable. j
[EC-2009 : 2 Marks]

Q.30 The gain and phase margins of G(s) for closed- (a)
5
loop stability are
(a) 6 dB and 180° (b) 3 dB and 180°
(c) 6 dB and 90° (d) 3 dB and 90°
j
[EC-2009 : 2 Marks]
j5
Q.31 For the asymptotic Bode magnitude plot shown
(b)
below, the system transfer function can be
Magnitude (dB)

40 j

1/5
(c)
0
0.001 0.1 10 1000
104 Electronics Engineering Control Systems

j Q.37 Consider the feedback system shown in the


figure. The Nyquist plot of G(s) is also shown.
Which one of the following conclusions is
(d)
1/5 correct?
R(s ) + K G( s ) C(s )

[EC-2011 : 1 Mark]

Q.34 The gain margin of the system under closed- Im G(j )


loop unity negative feedback is
100
G( s ) H ( s ) =
s (s + 10)2
(a) 0 dB (b) 20 dB –1 +1 Re G(j )
(c) 26 dB (d) 46 dB
[EC-2011 : 2 Marks]

Q.35 A system with transfer function,


(a) G(s) is an all pass filter.
(s 2 + 9) (s + 2) (b) G(s) is strictly proper transfer function.
G(s ) =
(s + 1) (s + 3) ( s + 4) (c) G(s) is a stable and minimum phase transfer
is excited by sin( t). The steady-state output of function.
the system is zero at (d) The closed-loop system is unstable for
(a) = 1 rad/sec. (b) = 2 rad/sec. sufficiently large and positive K.
(c) = 3 rad/sec. (d) = 4 rad/sec. [EC-2014 : 1 Mark]
[EC-2012 : 1 Mark]
Q.38 The phase margin is degrees of
Q.36 The Bode plot of a transfer function G(s) is 10
G(s) =
shown in the figure below. (s + 0.1) (s + 1) (s + 10)
Gain (dB) calculated using the asymptotic Bode plot is
_______ .
40
[EC-2014 : 2 Marks]
32
Q.39 The Bode asymptotic magnitude plot of a
20 minimum phase system is shown in the figure.
G(j ) dB
0 10 100
26.02
1 (rad/s)
–8

6.02
The gain (20 log G( s) ) is 32 dB and –8 dB at 0
0.1 1 2 10 20
1 rad/sec and 10 rad/sec respectively. The –6.02
(rad/s) in log scale
phase is negative for all . Then G(s) is
39.8 39.8 If the system is connected in a unity negative
(a) (b)
s s2 feedback configuration, the steady-state error of
32 32 the closed-loop system, to a unity ramp input,
(c) (d)
s s2 is _______ .
[EC-2013 : 1 Mark] [EC-2014 : 2 Marks]
GATE Previous Years Solved Paper 105

Q.40 In a Bode magnitude plot, which one of the Q.44 The phase margin (in degrees) of the system
following slopes would be exhibited at high
10 is _______ .
frequencies by a 4th order all pole system? G(s) =
s (s + 10)
(a) –80 dB/decade (b) –40 dB/decade
(c) +40 dB/decade (d) +80 dB/decade [EC-2015 : 1 Mark]
[EC-2014 : 1 Mark]
Q.45 A closed-loop control system is stable if the
Q.41 The polar plot of the transfer function, Nyquist plot of the corresponding open-loop
10 ( s + 1) transfer function
G( s ) = for 0 <
s + 10 (a) encircles the s-plane point (–1 + j0) in the
will be in the counterclockwise direction as many times
(a) first quadrant (b) second quadrant as the number of right-half s-plane poles.
(c) third quadrant (d) fourth quadrant (b) encircles the s-plane point (0 – j1) in the
[EC-2015 : 1 Mark] clockwise direction as many times as the
number of right-half s-plane poles.
Q.42 The transer function of a mass-spring-damper
(c) encircles the s-plane point (–1 + j0) in the
system is given by
counterclockwise direction as many times
1
G(s ) = as the number of left-half s-plane poles.
2
Ms + Bs + K
(d) encircles the s-plane point (–1 + j0) in the
The frequency response data for the system are
counterclockwise direction as many times
given in the following table.
as the number of right-half s-plane zeros.
(in rad/s) G( j ) (in dB) arg (G(j )) (in deg)
[EC-2016 : 1 Mark]
0.01 –18.5 –0.2
Q.46 The number and direction of encirclements
0.1 –18.5 –1.3
around the point –1 + j0 in the complex plane
0.2 –18.4 –2.6
1 –16 –16.9 1 s
by the Nyquist plot of G(s ) = is
2 –11.4 –89.4 4 + 2s
3 –21.5 –151 (a) zero
5 –32.8 –167
(b) one, anti-clockwise
10 –45.3 –174.5
(c) one, clockwise
The unit step response of the system (d) two, clockwise
approaches a steady-state value of ______ . [EC-2016 : 1 Mark]
[EC-2015 : 2 Marks]
Q.47 In the feedback system shown below,
Q.43 Consider the Bode plot shown in figure. Assume
1
that all the poles and zeros are real valued. G(s) =
(s + 1) (s + 2) (s + 3)

r + K G(s ) y
40 dB –
–40 dB/decade

40 dB/decade
0 dB The positive value of K for which the gain
fL 300 900 fH f (Hz)
margin of the loop is exactly 0 dB and the phase
The value of fH – fL (in Hz) is _____ . margin of the loop is exactly zero degree is ___.
[EC-2015 : 1 Mark] [EC-2016 : 2 Marks]
106 Electronics Engineering Control Systems

Q.48 The asymptotic Bode phase plot of The Nyquist path and the corresponding
K Nyquist plot of G(s) are shown in the figures
G(s) =
(s + 0.1) (s + 10) (s + p1 ) below.
with K and p1 both positive, is shown below, j

+j s-plane
0.01 0.1 1 10 100

(rad/s) j
–45° s = Re
R

–135° 0

–225°
–270°
–j

The value of p1 is ______ . Nyquist path for G(s)


[EC-2016 : 2 Marks]
jIm G
Q.49 Consider a stable system with transfer function,
+j5.43 K G(s )-plane
s p + b1 s p 1
+ ... + bp
G( s ) =
s q + a1 sq 1
+ ... + aq
–j
where b1, ..., bp and a1, ...., aq are real valued
=0
constants. The slope of the Bode log magnitude –K
Re G
2K
curve of G(s) converges to –60 dB/decade as +j
. A possible pair of values for p and q is
(a) p = 0 and q = 3 (b) p = 1 and q = 7
(c) p = 2 and q = 3 (d) p = 3 and q = 5 –j5.43 K
[EC-2017 : 1 Mark] Nyquist plot for G(s)

Q.50 The Nyquist plot of the transfer function If 0 < K < 1, then the number of poles of the
closed-loop transfer function that lie in the right-
K
G( s ) = does not encircle the half of the s-plane is
2
( s + 2 s + 2) ( s + 2)
(a) 0 (b) 1
point (–1 + j0) for K = 10 but does encircle the
(c) 2 (d) 3
point (–1 + j0) for K = 100. Then the closed-loop
[EC-2017 : 2 Marks]
system (having unity gain feedback) is
(a) stable for K = 10 and stable for K = 100. Q.52 The Nyquist stability criterion and the Routh
(b) stable for K = 10 and unstable for K = 100. criterion both are powerful analysis tools for
(c) unstable for K = 10 and stable for K = 100. determining the stability of feedback controllers.
Identify which of the following statements is
(d) unstable for K = 10 and unstable for K = 100.
false.
[EC-2017 : 2 Marks]
(a) Both the criteria provide information relative
Q.51 A unity feedback control system is characterized to the stable gain range of the system.
by the open-loop transfer function, (b) The generated shape of the Nyquist plot is
10 K( s + 2) readily obtained from the Bode magnitude
G(s) = 3
s + 3s2 + 10 plot for all minimum phase systems.
GATE Previous Years Solved Paper 107

(c) The Routh criterion is not applicable in the Q.55 For an LTI system, the Bode plot for its gain is as
condition of transport lag, which can be illustrated in the figure shown. The number of
readily handled by the Nyquist criterion. system poles Np and the number of system zeros
(d) The closed-loop frequency response for a Nz in the frequency range 1 Hz f 107 Hz is
unity feedback system cannot be obtained Gain (dB)
from the Nyquist plot. –20 dB/dec.
[EC-2018 : 1 Mark] 100
–60 dB/dec.
Q.53 For a unity feedabck control system with the
–40 dB/dec.
K
forward path transfer function G(s) = . 10
4
10
5
10
6 7
10
s (s + 2) 0
10
1
10
2
10
3
f (in Hz)

The peak resonant magnitude Mr of the closed- –40 dB/dec.


loop frequency response is 2. The corresponding
value of the gain K (correct to two decimal –60 dB/dec.
places) is _____ .
[EC-2018 : 2 Marks] (a) Np = 6, Nz = 3 (b) Np = 7, Nz = 4
(c) Np = 5, Nz = 2 (d) Np = 4, Nz = 2
Q.54 The figure below shows the Bode magnitude
[EC-2019 : 1 Mark]
and phase plots of a stable transfer function,
n0 Q.56 The pole-zero map of a rational function G(s) is
G(s ) =
s 3 + d2 s 2 + d1s + d0 shown below. When the closed counter G is
mapped into the G(s) plane, then the mapping
encircles.
G( j )
Im

36 dB s-plane

20 dB
0 dB

Re

–180°
–270°
(a) The point –1 + j0 of the G(s) plane once in
the counter-clockwise direction.
+
G(s)
(b) The origin of the G(s) plane once in the
K
– clockwise direction.
(c) The origin of the G(s) plane once in the
Consider the negative unity feedback counter-clockwise direction.
configuration with gain k in the feed forward (d) The point –1 + j0 of the G(s) plane once in
path. The closed-loop is stable for k > k0. The the clockwise direction.
maximum value of k0 is ______ . [EC-2020 : 1 Mark]
[EC-2018 : 2 Marks]
108 Electronics Engineering Control Systems

Q.57 The complete Nyquist plot of the open-loop ELECTRICAL EN GINEERIN G


transfer function G(s) H(s) of a feedback control
(GATE Previous Years Solved Papers)
system is shown in the figure.
jImGH Q.1 Which of the following is the transfer function
G(s ) H(s) plane of a system having the Nyquist plot shown in
figure below?

Im

0 Re GH
(–1, j0)

+
0 +
Re
– –
0
If G(s) H(s) has one zero in the right half of the
s-plane, the number of poles that the closed-loop
system will have in the right half of the s-plane is
(a) 1 (b) 3
K K
(c) 4 (d) 0 (a) (b)
2
s (s + 2) (s + 5) s 2 (s + 2) (s + 5)
[EC-2021 : 2 Marks]
K (s + 1) K (s + 1) (s + 3)
Q.58 Consider a closed-loop control system with unity (c) 2 (d)
negative feedback and KG(s) in the forward path, s (s + 2) (s + 5) s2 (s + 2) (s + 5)
where the gain K = 2. The complete Nyquist plot [EE-1991 : 1 Mark]
of the transfer function G(s) is shown in the figure.
Note that, Nyquist contour has been chosen to Q.2 The system having the Bode magnitude plot
have the clockwise sense. Assume G(s) has no shown in figure below has the transfer function.
poles on the closed right half of the complex
plane. The number of poles of the closed-loop
transfer function in the closed right half of the 60 –20 dB/dec
complex plane is _________ . –40 dB/dec
–20 dB/dec
Im G(s)

0.01 0.05 0.1 1.0 10

60 ( s + 0.01) ( s + 0.1)
(a)
s 2 (s + 0.05)2
–1 Re G(s )
10(1 + 10s)
(b)
s (1 + 20s)
0.4
3(s + 0.05)
0.8
(c)
s (s + 0.1) (s + 1)
5(s + 0.1)
(a) 0 (b) 1 (d)
(s + 0.05)
(c) 2 (d) 3
[EE-1991 : 1 Mark]
[EC-2022]
GATE Previous Years Solved Paper 109

Q.3 A unity feedback system has the open-loop


transfer function, –40 dB/sec
54 dB
1 –60 dB/sec
G(s) =
(s 1) (s + 2) (s + 3)
–40 dB/sec
The Nyquist plot of G(s) encircle the origin.
(a) Never (b) Once –60 dB/sec
(c) Twice (d) Thrice
[EE-1992 : 1 Mark]
0.1 2 5 25 rad/sec.
Q.4 The closed-loop transfer function of a control
system is given by
20(s + 5) 10(s + 5)
(a) (b)
C (s) 1 s(s + 2) (s + 25) (s + 2)2 (s + 25)
=
R(s ) (1 + s)
20(s + 5) 50(s + 5)
(c) (d)
For the input r(t) = sint, the steady-state value of 2
s (s + 2) (s + 25) 2
s (s + 2) (s + 25)
c(t) is equal to
[EE-2001 : 1 Mark]
1
(a) cos t (b) 1 Q.7 The asymptotic Bode plot of the transfer function
2

1 1 K
(c) sin t (d) sin t is given in figure. The error in phase
2 2 4 s
1+
a
[EE-1996 : 1 Mark]

Q.5 The polar plot of a type-1, 3-pole, open-loop angle and dB gain at a frequency of = 0.5 a are
system is shown in figure below. The closed- respectively
loop system is
GH-plane 20 log K
20 dB/decade
G dB

–1.42 =
a

=0 0.1a 0.5a a

(a) always stable. 45° per decade


Ph°
(b) marginally stable.
(c) unstable with one pole on the right half
s-plane.
(d) unstable with two poles on the right half (a) 4.9°, 0.97 dB (b) 5.7°, 3 dB
s-plane. (c) 4.9°, 3 dB (d) 5.7°, 0.97 dB
[EE-2001 : 1 Mark] [EE-2003 : 2 Marks]

Q.6 The asymptotic approximation of the log- Q.8 The Nyquist plot of loop transfer function
magnitude versus frequency plot of a minimum G(s) H(s) of a closed-loop control system passes
phase system with real poles and one zero is through the point (–1, j0) in the G(s) H(s) plane.
shown in figure. Its transfer function is
110 Electronics Engineering Control Systems

The phase margin of the system is Q.13 In the GH(s) plane, the Nyquist plot of the loop
(a) 0° (b) 45°
e 0.25s
(c) 90° (d) 180° transfer function G(s ) H (s ) = passes
s
[EE-2004 : 1 Mark]
through the negative real axis at the point.
Q.9 In the system shown in figure, the input (a) (–0.25, j0) (b) (–0.5, j0)
x(t) = sint. In the steady-state, the response y(t)
(c) (–1, j0) (d) (–2, j0)
will be
[EE-2005 : 2 Marks]
s
x(t) y (t )
s+1 Q.14 If the compensated system shown in the figure
has a phase margin of 60° at the crossover
1
(a) sin(t 45°) frequency of 1 rad/sec, then value of the gain
2
‘k’ is
1
(b) sin(t + 45°)
2 1
R(s) +– k + 0.366s Y(s )
s( s + 1)
(c) sin(t – 45°)
(d) sin(t + 45°)
[EE-2004 : 2 Marks] (a) 0.366 (b) 0.732
Q.10 The open-loop transfer function of a unity (c) 1.366 (d) 2.738
feedback control system is given as, [EE-2005 : 2 Marks]

as + 1 Q.15 The Bode magnitude plot of


G( s ) = 2
. The value of ‘a’ to give a phase
s 10 4 (1 + j )
H( j ) = is
margin of 45° is equal to (10 + j ) (100 + j )2
(a) 0.141 (b) 0.441
(c) 0.841 (d) 1.141
40
[EE-2004 : 2 Marks]
20
Q.11 A system with zero initial conditions has the
closed-loop transfer function, (a) log( )
–1 0 1 2 3
–20
s2 + 4
T (s) =
(s + 1) (s + 4) –40
The system output is zero at the frequency
(a) 0.5 rad/sec. (b) 1 rad/sec.
(c) 2 rad/sec. (d) 4 rad/sec. H ( j ) dB

[EE-2005 : 1 Mark]
40
Q.12 The gain margin of a unity feedback control 20
system with the open-loop transfer function is
(b) log( )
( s + 1) –1 0 1 2 3
G(s ) = . –20
s2
(a) 0 (b) 1/ 2 –40

(c) 2 (d)
[EE-2005 : 1 Mark]
GATE Previous Years Solved Paper 111

H ( j ) dB Im

40 =
(4) Re
20 –1

(c) log( )
–1 0 1 2 3
–20

–40
(a) (1) only (b) All, except (1)
(c) All, except (3) (d) (1) and (2) only
H ( j ) dB
[EE-2006 : 2 Marks]
40 Q.17 If x = Re G(j ), and y = Im G(j ) then for 0+,
20
1
the Nyquist plot for G(s) = .
(d) log( ) s (s + 1) (s + 2)
–1 0 1 2 3
–20
3
–40 (a) x = 0 (b) x =
4
1 y
[EE-2006 : 2 Marks] (c) x = y (d) x =
6 3
Q.16 Consider the following Nyquist plots of loop
[EE-2007 : 2 Marks]
transfer functions over = 0 to = . Which of
these plots represents a stable closed-loop Q.18 The asymptotic Bode magnitude plot of a
system? minimum phase transfer function is shown in
the figure.
Im

G( j ) dB
=
(1) Re
–40 dB/decade
–1

20
–20 dB/decade

Im 0 0.1 (rad/sec)
(log scale)
–20

0 dB/decade
(2)
=
Re The transfer function has
–1
(a) 3-poles and one zero.
(b) 2-poles and one zero.
Im (c) 2-poles and two zeros.
(d) 1-pole and two zeros.

(3)
= [EE-2008 : 2 Marks]
Re
–1
Q.19 The polar plot of an open-loop stable system is
shown below.
112 Electronics Engineering Control Systems

Im 1
Q.22 The frequency response G(s ) =
[s (s + 1) (s + 2)]
=
Re plotted in the complex G(j ) plane (for 0 < < )
–1.42
is
Im
=0

–3/4
The closed-loop system is (a) Re
(a) always stable
(b) marginally stable
(c) unstable with one pole on the RHS s-plane =0
(d) unstable with two poles on the RHS s-plane
Im
[EE-2009 : 1 Mark] =0

Q.20 The asymptotic approximation of the log-


magnitude vs frequency plot of a system (b)
containing only real poles and zeros is shown. Re
Its transfer function is –3/4

–40 dB/sec Im
80 dB =0
–60 dB/sec

–40 dB/sec (c)

Re
–60 dB/sec –1/6

Im
0.1 2 5 25 rad/sec.
–1/6
Re
10(s + 5) 1000(s + 5)
(a) (b) (d)
s (s + 2) (s + 25) s2 (s + 2) (s + 25)

100(s + 5) 80(s + 5) =0
(c) (d) 2
s (s + 2) (s + 25) s (s + 2) (s + 25)
[EE-2009 : 1 Mark] [EE-2010 : 2 Marks]

Q.21 The open-loop transfer function of a unity Q.23 The frequency response of a linear system G(j )
feedback system is given by is provided in the tabular form below.
G(j ) 1.3 1.2 1.0 0.8 0.5 0.3
0.1s
(e )
G(s ) = G(j ) –130° –140° –150° –160° –180° –200°
s
The gain margin and phase margin of the
The gain margin of this system is
system are
(a) 11.95 dB (b) 17.67 dB
(a) 6 dB and 30° (b) 6 dB and –30°
(c) 21.33 dB (d) 23.9 dB
(c) –6 dB and 30° (d) –6 dB and –30°
[EE-2009 : 2 Marks]
[EE-2011 : 1 Mark]
GATE Previous Years Solved Paper 113

Q.24 A system with transfer function, Q.27 For the transfer function,
(s 2 + 9) (s + 2) 5(s + 4)
G(s ) = G(s) =
(s + 1) (s + 3) ( s + 4) s (s + 0.25) (s2 + 4s + 25)
is excited sin( t). The steady-state output of the The values of the constant gain term and the
system is zero at highest corner frequency of the Bode plot
(a) = 1 rad/sec. (b) = 2 rad/sec. respectively are
(c) = 3 rad/sec. (d) = 4 rad/sec. (a) 3.2 and 5.0 (b) 16.0 and 4.0
[EE-2012 : 1 Mark] (c) 3.2 and 4.0 (d) 16.0 and 5.0
[EE-2014 : 2 Marks]
Q.25 The Bode plot of a transfer function G(s) is
shown in the figure below. Q.28 The magnitude Bode plot of a network is shown
Gain (dB) in the figure.
40
G( j ) dB Slope 20 dB/decade
32

20

0 10
100 (rad/s)
–8
0 1/3 1 log10
The gain (20 log |G(s)|) is 32 dB and –8 dB at
The maximum phase angle m and the
1 rad/sec and 10 rad/sec respectively. The
corresponding gain Gm respectively, are
phase is negative for all . Then G(s) is
(a) –30° and 1.73 dB (b) –30° and 4.77 dB
39.8 39.8
(a) (b) (c) +30° and 4.77 dB (d) +30° and 1.73 dB
s s2
[EE-2014 : 2 Marks]
32 32
(c) (d)
s s2 Q.29 A Bode magnitude plot for the transfer function
[EE-2013 : 1 Mark] G(s) of a plant is shown in the figure. Which one
of the following transfer functions best describes
Q.26 The Bode magnitude plot of the transfer function
the plant?
is shown below.
K (1 + 0.5s ) (1 + as) 20 log G( j 2 f )
G(s ) =
s s
s 1+ (1 + bs ) 1 +
8 36
20
Note that –6 dB/octave = –20 dB/decade. The
value of a/bK is _______ . 0

dB –20
–6 dB/Octave

0 dB/Octave
0 dB/Octave

6 dB/Octave –6 dB/Octave 0.1 1 10 100 1k 10 k 100 k f(Hz)

1000 ( s + 10) 10(s + 10)


(a) (b)
–12 dB/Octave s + 1000 s (s + 1000)
s + 1000 s + 1000
0.01 2 4 8 24 36 (c) (d)
(rad/s) 10s ( s + 10) 10(s + 10)
[EE-2014 : 2 Marks] [EE-2015 : 2 Marks]
114 Electronics Engineering Control Systems

Q.30 Nyquist plots of two functions G1(s) and G2(s)


Im
are shown in figure.
Im

=
(d) Re

Re
0
G1(s)

0
[EE-2015 : 1 Mark]

Im Q.31 The phase cross-over frequency of the transfer

G2(s) 100
function G(s ) = in rad/sec is
=0 (s + 1)3
Re
1
(a) 3 (b)
3

(c) 3 (d) 3 3
Nyquist plot of the product of G1(s) and G2(s) is [EE-2016 : 1 Mark]

Im Q.32 Consider the following asymptotic Bode


magnitude plot ( is in rad/sec). Which one of
=0
the following transfer function is best
(a) represented by the above Bode magnitude plot?
Re
Magnitude (dB)

20 dB/dec

Im 12 dB

–40 dB/dec
0 dB
0.5 8
(b) Re
1
2s
(a)
(1 + 0.5s ) (1 + 0.25s 2 )

4(1 + 0.25s)
(b)
s (1 + 0.25s)
Im
2s
(c)
(1 + 2s) (1 + 4s)

(c) Re 4s
(d)
(1 + 2 s ) (1 + 4s )2
[EE-2016 : 2 Marks]
GATE Previous Years Solved Paper 115

Q.33 Loop transfer function of a feedback system is Q.37 The asymptotic Bode magnitude plot of
minimum phase transfer function G(s) is shown
s+3
G(s ) H (s ) = 2
. Take the Nyquist contour below.
s (s 3)

in the clockwise direction. Then, the Nyquist G( j ) dB

plot of G(s) H(s) encircles –1 + j0


60
(a) once in clockwise direction –20 dB/dec
(b) twice in clockwise direction 40
(c) once in anticlockwise direction –40 dB/dec
20
(d) twice in anticlockwise direction
20
[EE-2016 : 2 Marks] 0 1 10 (log scale)

Q.34 Consider the unity feedback control system


shown. The value of ‘K’ that results in a phase –60 dB/dec
margin of the system to be 30° is _______ .
(Give the answer upto 2 decimal places)
Consider the following statements:
Ke s
U(s ) +– Y(s ) Statement-1: Transfer function G(s) has 3-poles
s
and one zero.
Statement-2: At very high frequency ( ),

[EE-2017 : 1 Mark] 3
the phase angle G( j ) = .
2
Q.35 The transfer function of a system is given by
Which of the following options is correct?
Vo (s ) 1 s
= (a) Statement 2 is true and statement 1 is false.
Vi (s ) 1 + s
(b) Both the statements are false.
Let the output of the system be v o (t) =
(c) Both the statements are true.
Vm sin( t + j) for the input, vi(t) = Vm sin( t).
(d) Statement 1 is true and statement 2 is false.
Then the minimum and maximum values of
[EE-2019 : 2 Marks]
j (in radians) are respectively
Q.38 Consider a negative unity feedback system with
(a) and (b) and 0
2 2 2 forward path transfer function,
K
(c) 0 and (d) – and 0 G(s) =
2 (s + a) ( s b ) (s + c )
where K, a, b, c are positive real numbers. For a
Q.36 The open-loop transfer function of a unity
Nyquist path enclosing the entire imaginary
feedback system is given by
axis and right half of the s-plane in the clockwise
e 0.25s direction, the Nyquist plot of (1 + G(s)), encircles
G( s ) =
s the origin of (1 + G(s)) plane once in the
In G(s) plane, the Nyquist plot of G(s) passes clockwise direction and never passes through
through the negative real axis at the point. this origin for a certain value of K. Then, the
(a) (–1.25, j0) (b) (–0.5, j0) number of poles of G(s)/1 + G(s) lying in the
(c) (–1.5, j0) (d) (–0.75, j0) open right half of the s-plane is ______ .
[EE-2017 : 1 Mark] [EE-2020 : 1 Mark]
116 Electronics Engineering Control Systems

Q.39 A stable real linear time-invariant system with (a) 11.08 (b) 78.13
single pole at p, has a transfer function (c) 8.84 (d) 122.87
[EE-2020 : 2 Marks]
s2 + 100
H(s) = with a dc gain of 5. The smallest
s p

positive frequency, (in rad/sec) at unity gain is


closed to

Electronics & Electrical Engineering


GATE Previous Years Solved Paper

A n swe rs & Expl a n a t i o n s

Answers
EC Frequency Response Analysis

1. (b) 2. (c) 3. (c) 4. (c) 5. (–10) 6. (a, c) 7. (c) 8. (a)


9. (d) 10. (d) 11. (a) 12. (b) 13. (d) 14. (b) 15. (a) 16. (a)
17. (c) 18. (a) 19. (d) 20. (b) 21. (d) 22. (d) 23. (d) 24. (d)
25. (a) 26. (b) 27. (d) 28. (a) 29. (b) 30. (c) 31. (a) 32. (b)
33. (a) 34. (c) 35. (c) 36. (b) 37. (d) 38. (45) 39. (0.5) 40. (a)
41. (a) 42. (0.12) 43. (8970) 44. (84.36) 45. (a) 46. (a) 47. (60) 48. (1)
49. (a) 50. (b) 51. (c) 52. (d) 53. (14.92) 54. (0.10) 55. (a) 56. (b)
57. (b) 58. (c)

Solutions
EC Frequency Response Analysis

1. (b) 2 tan–1( ) = 45°


P = 14, Z = 2 =1
P – Z = 14 – 2 = 12 pc = 1 rad/sec.

Slope = –20(P – Z) = –20(12) G 10 10


= pc = 2
= =5
= –240 dB/decade (1 + ) 1(1 + 1)
At = pc the polar plot crosses the negative
2. (c)
real axis at –5.
10
G(s) = 3. (c)
s(s + 1)2
G = –180° pc
–3 tan–1 ( pc) = –180°
= –90° – 2 tan–1( )
tan–1 ( pc) = 60°
GATE Previous Years Solved Paper 117

= 10. (d)
pc 3 rad/sec.
Gain crossover frequency where gain is 1 is g,
1
G.H. = X =
( )
2 3
2 =1
1+ pc j (1 + j )

1 1 2 3
= = =1
( )
3 8 2
1 + ( 3)2 1+
2
2 3 = 1+
1
G.M. = =8 =3
X
G(j ) H(j ) = –90° – tan–1
4. (c)
= 90° tan 1 3 = 150°
2 4 2 P.M. = 180° – 150° = 30°
n (1 2 )+ 4 4 +2
11. (a)
5. Sol. N=0
At 180° GH = 1.2 (1 encirclement in CW direction and other in
1 1 ACW)
G.M. = 20 log = 20 log P = 0 (no pole in right half)
GH 1.2
So, N = P–Z
= –1.6 dB
Z = P–N=0
At GH = 1 = –190°
No roots on RH of s-plane.
P.M. = 180° +
= 180° – 190° = –10° 12. (b)
Since both G.M. and P.M. are negative. 1
G(s) H(s) =
So, the system is unstable. 2
s(s + s + 1)

6. (a, c) when,
G(s) H(s) = –180°
The non-minimum phase transfer function is
defined as the transfer function which has zeros 1
–180 = 90 tan 2
(or) poles in right side of s-plane. 1
1
–90 = tan 2
7. (c) 1
= –125° 1–
=0 2
P.M. = 180° + pc = 1 rad/sec
= 180° – 125° = 55° Value of gain at pc = 1
1
8. (a) G(s) H (s) =
2 2 2
A system is unstable when Nyquist plot of G(j ) (1 ) + j
H(j ) encloses the point (–1 + j0) and gain G.M. = –20 log1 = 0
margin of unstable system is always negative
or less than zero. Therefore, the gain margin of 13. (d)
given system is less than zero. P.M. = 180° + GH ( j )
pc
9. (d) For gc,
This is a second-order function and so its gain G( s ) H ( s ) = gc =1
margin is infinity.
118 Electronics Engineering Control Systems

1 s 1+ 2 20 log (K / ) = 10 = 140
= =1
(1 + s) (2 + s ) 1+ 2 4+ 2 K
= 107
2 =1 = 10
4+
4+ =1 2 K = 10 8
2 = –3 (Imaginary)
17. (c)
So no gain crossover frequency
20 log K = –20 dB
P.M. =
K = 10–1 = 0.1
14. (b) Zero at = 1 and poles at = 10, 100
s K ( s + 1)
G(s) H(s) = 3 H(s) =
(s + 100) s s
+1 +1
Calculating 10 100
pc,

pc
0.1(s + 1) × 10 × 100
–180° = +90 3 tan
1 =
100 (s + 10) ( s + 100)

pc 100(s + 1)
–270° = 3 tan 1 =
(s + 10) (s + 100)
100
or, = pc
18. (a)
G.M. =
Pole at 0.01 and 1 Hz gives –180° phase.
Hence cannot be determined.
Zero at 5 Hz gives +90° phase.
Calculating gc
at 20 Hz –90° phase shift is provided.
From, G(s) H (s) = 1
= Negative value 19. (d)
gc
G.M. and P.M. of the system cannot be 1
Let be a lag network.
determined. s+1
At = 0, Mag = , =–tan–1 0 = 0
15. (a)
= , Mag = 0, =–tan–1 = –90°
The encirclement of critical point (–1, 0) in
If in the direction of increasing phase shift is
A.C.W. direction is once.
decreasing system is lag network.
N = 1, P = 1 (Given)
Z = P–N=0 20. (b)
No zero in RH of s-plane.
Im
So system is stable.
=0
16. (a)
= 0.1 to 10, +120 dB change
3 zeros at 0.1,
= 10 to 100, –40 dB
Re
[i.e. +60 to +20] change –8
So, two poles at = 10, –2 –0.2
= 100, –20 dB change
One pole at = 100

K (s + 0.1)3 =0
T(s) = 2
(s + 10) ( s + 100)
GATE Previous Years Solved Paper 119

System is stable in region –0.2 to –2 and on the 25. (a)


left side of –8 as number of encirclement there is y(t) = x(t) h(t)
zero. Y(s) = X(s) H(s)
0.2 K < 1 K < 5
1 1
2 K > 1 K > 0.5 H(j ) = = 45°
s+1 2
0.5 < K < 5
1
1 > 8K =
2 4
1 x(t) = sin(t) u(t)
K> (negative sign only shows that it is on
8
1
negative axis). y(t) = sin t
2 4
21. (d)
26. (b)
Gain crossover frequency where gain is 1.
5
G(s) = 1 H(f) =
1 + j 10 f
3
2 =1 5 5 1
( + 4)1/2 H(s) = = =
1 + 5s 1 1
5 s+ s+
9 5 5
2 =1
( 2 + 4) 1 1
Step response =
gc = 1.26 s 1
s+
Phase crossover frequency 5
where, GH = –180° 5 5
Y(s) =
pc = 0.632 s 1
s+
5
22. (d)
y(t) = 5[1 – e–t/5] u(t)
3
G.M. at, = 27. (d)
0.632 (0.632 2 + 4)1/2
a = 2.26 K K /20
G(s) = =
1 s(s + 1) (s + 20) s
G.M. = 20 log s (1 + s) 1 +
a 20
1 Bode plot is in (1 + sT) form
20 log = –7.09
2.26 –20 log + 20 logK = 60 dB = 0.1
Since, G.M. is negative system is unstable.
–20 log + 20 logK = 60 dB
P.M. is also negative,
20 logK = 40 dB
P.M. = –87.5°
K = 100
23. (d) 100
G(s) =
For 2nd order system G.M. = s(s + 1) (1 + 0.5s)

24. (d) 28. (a)


1 5 10
G.M. = 20 log dB Mr = =
a 2 3
2 1
a=1
G.M. = 0 1
or, =
2
120 Electronics Engineering Control Systems

Also, r = n 1 2 z2 = 90° tan


1
p
1
5 2 = n 1 2× 1 2
4 90° tan = 30°
p
or, n = 10 rad/sec.
Only option (a) satisfies the conditions. 2
tan 1 = 60°
p
29. (b)
2
Nyquist plot will encircles the origin as many = tan 60° = 3
p
number of times as the difference between the
number of right side poles and zeros of open- 2
p =
loop transfer function. 3
i.e., N = P–Z
From the plot, 33. (a)
N = –1 G(j ) = 5 + j
and P = 0 (Given) G( j ) = 2
25 +
Z=1 At = 0,
Thus, one open-loop zero in right half of s-plane.
G(0) = 25 + 0 = 5
30. (c) At = ,
1 G( ) =
Gain margin = 20 log
X
= 34. (c)
1 100
20 log = 20 log 2 = 6 dB
0.5 G(s) H(s) =
s(s + 10)2
and phase margin = 90°
1
= 90° 2 tan
31. (a) 10
System transfer function, For phase crossover frequency,
= –180°
s
K 1+
0.1 –180 = 90° 2 tan 1
G(s) H(s) = 10
s
1+
10 = 10 rad/sec.
Here, 20 logK = 0 Put, s=j
K=1 100
G(j ) H(j ) =
10s + 1 j ( j + 100)2
G(s) H(s) =
0.1 s + 1 100
G( j ) H ( j ) =
= 10 ( 2 + 100)
32. (b)
100 1
Phase difference between input and output, = =
10(200) 20
= = = 30° 1
3 2 6 G.M. = = 20
G( j ) H ( j )
and = 2 rad/sec.
G.M. in dB = 20 log20 = 26 dB
From the transfer function,
GATE Previous Years Solved Paper 121

35. (c) 10
=
Transfer function is, s s s
+1 +1 +1
0.1 1 10
(s 2 + 9) (s + 2)
G(s) = Amount of shift = 20 logK = 20 log10 = 20 dB
(s + 1) (s + 3) ( s + 4)
dB
The system has zero at = 3 rad/sec.

36. (b) –20 dB/dec.


20 dB
10 rad/sec to 1 rad/sec is 1 decade.
1 10
32 – (–8) = 40 dB
0 dB 0.1 gc
So, the slope is 40 dB/decade it means there are
two poles at origin, it means either option (b) or
option (d) is correct put = 1 rad/sec in both
the options,

39.8 20 0
20 log = 32 dB –20 dB =
(1)2 log 0.1 log pc

32 or, gc = 1 rad/sec. ...(i)


20 log = 30.1 dB
(1)2 P.M. = 180° + GH ( j ) at ...(ii)
= gc
39.8
So, option (b) is correct option . From the given transfer function,
s2
1 gc 1 gc 1 gc
G(j ) = tan tan tan
37. (d) 0.1 1 10
or, G(j ) = –84.289° – 45° – 5.711°
R(s ) + K G( s ) C(s )
– = –135° ...(iii)
From equation (ii) and (iii) we get,
P.M. = 180° – 135° = 45°
Im G(j )
39. Sol.
dB

26.02 –20 dB/decade

–1 +1 Re G(j )
6.02
10 20
0
0.1 1 2
–6.02 –20 dB/decade

From the Nyquist plot, it can be seen that it does +20 dB/decade
not encircle the critical point (–1, 0), The open-loop transfer function of the system is
N = P – Z, N = 0
s
The closed-loop system is unstable for +1 K
2
sufficiently large and positive K. G(s) H(s) = ...(i)
s
s +1
10
38. Sol.
where,
10
s s s 26.02 = 0.1 = 20 logK – 20 log
0.1 × 10 +1 +1 +1
0.1 1 10
or, K = 1.99 2 ...(ii)
122 Electronics Engineering Control Systems

From equation (i) and (ii), we get 42. Sol.


20(s + 2) C (s) 1
G(s) = ...(iii) = G(s ) =
2s (s + 10) R(s ) 2
Ms + Bs + K
Steady-state error for ramp input is, Steady-state value of response for unit step input,
A 1 1 1
ess = C(s) = lim s × × =
KV 2
s 0 Ms + Bs + K s K
where, KV = lim sG(s)
s 0 1
Also, G( 0) = = 18.5 dB
From equation (iii), K
(as per the given table)
10(s + 2)
K V = lim s × =2 18.5
s 0 s (s + 10) 1
= 10 20 = 0.12
1 K
ess = = 0.5
2 43. Sol.

40. (a) As we know,

A 4th order all-pole system means that the system y2 y1


Slope =
must be having no zero or s-terms in numerator x2 x1
and s4 terms in denominator or 4-poles i.e. 40 0
40 dB/dec =
1 log 300 log f L
H(s)
s4 300 40
or, log = =1
One pole exhibits –20 dB/decade slope, so fL 40
4 pole exhibits a slope of –80 dB/decade.
or, fL = 30 Hz
41. (a) Similarly,
40 0
10( s + 1) –40 dB/dec. =
G(s) = log 900 log f H
(s + 10)
For, =0 M1 = 1 and 1 = 0° 900
log = –1
=0 M2 = 10 fH
and = –tan–1 + tan–1 = 0° or, fH = 9000 Hz
2
Polar plot, fH – fL = 9000 – 30 = 8970 Hz

–270° or +90° 44. Sol.


10
G(s) =
s (s + 10)

P.M. = 180° + G( ) H ( ) = gc
± 180° 0° or 360°
1 10 The gain crossover frequency can be calculated
by
G( ) H ( ) = 1
10
=1
2
–90° or +270° + 100
As the zero is nearer to the imaginary axis hence 4 2
gc + 100 gc 100 = 0
the direction of polar plot is clockwise.
GATE Previous Years Solved Paper 123

or, 2
= 0.99, –100.99 47. Sol.
gc
or, = 0.99 rad/sec. K
gc 1 + G(s) H(s) = 1 + =0
(s + 1) (s + 2) (s + 3)
1 gc
P.M. = 180° + 90° tan (s + 1) (s2 + 5s + 6) + K = 0
10
s3 + 5s2 + 6s + s2 + 5s + 6 + K = 0
0.99 s3 + 6s2 + 11s + 6 + K = 0
= 180° 90° tan 1
10 Gain margin = 0 dB and phase margin = 0 °
84.364° It implies marginal stable system.
By Routh array:
45. (a)
N = P–Z s3 1 11
N = Number of encirclements of (–1 + j0). It is s2 6 (6 + K )
positive if Nyquist plot encircles the point –1 + j0 66 6 K
s1 0
in counterclockwise direction. 6
Z = Number of closed-loop poles lying in the s0 6+K
right half of s-plane.
For marginal stable system,
P = Number of open-loop poles lying in right
60 – K = 0
half of s-plane.
K = 60
For stability, Z = 0 N=P
48. Sol.
46. (a)
K
1 s G(s) =
G(s) = (s + 0.1) (s + 10) (s + p1 )
4 + 2s
1 1 1
1 1 G(s) = tan tan tan
and G(s) = tan tan 0.1 10 p1
2
1 1 1 1 1 1
1 G( s ) =1 = tan tan tan
At s = 0, G(s) = = 0.25 0.1 10 p1
4
= –135°
and G( s ) =0
= 0°
1 1 1 1
tan 10 tan 0.1 tan = –135°
1 p1
At s = , G(s) = = 0.5
2 1 1
84.28° 5.71° tan = 135°
and G(s) = –180° p1
=
Nyquist plot is, 1 1
tan = –135° + 90°
p1
Im
1 1
tan = 45°
p1
1
=1 p1 = 1
Re p1
–1 –0.5 0.25

49. (a)
Final slope = –60 dB/decade
which indicates that, q – p = 3
Hence, number of encirclements of (–1 + j0) = 0 Among the given options, option (a) satisfies
this condition.
124 Electronics Engineering Control Systems

50. (b) 53. Sol.


K Maximum resonant peak,
Given that, G(s) = 2
(s + 2 s + 2) ( s + 2) 1
Mr = =2
2
P = Open-loop poles in RHS of s-plane 2 1
Z = Closed-loop poles in RHS of s-plane
2
1
N = Number of encirclements about the point 2 1 =
2
(–1 + j0)
1
For K = 10; N = P – Z = 0 Z = 0 : stable 2(1 – 2) =
16
For K = 100; N = P – Z = 1 Z 0 : unstable
4 2 1
+ =0
51. (c) 16
Given that, the open-loop transfer function of a
1
unity feedback system is, 1
1 4 = 1± 3
2 = ±
10 K(s + 2) 2 4 2 4
G(s) =
s3 + 3s 2 + 10 1 1
From the given Nyquist plot, for 0 < K < 1, the As Mr = 2 > 1, < and 2 <
2 2
encirclements about the point (–1 + j0) is,
N=0 2
1 3
So, =
N = P–Z 2 4
P = Number of open-loop poles in right half of 2
K n
Given, G(s) = =
s-plane s(s + 2) s (s + 2 n)
Z = Number of closed-loop poles in right half of
So, n = K
s-plane
To determine the value of P : 2 K =2
Applying R-H criteria to G(s), 1
K =
10 K(s + 2)
G(s) =
s + 3s 2 + 10
3
1 1
K= 2
1 3
s3 1 0
2 4
s2 3 10
10 4
s1 0 = = 14.928
3 2 3
s0 10 0
54. Sol.
Two sign changes are there in the first column For G(s),
of the RH table. So, two open-loop poles are there MdB ( pc) = 20 dB
in right half of s-plane. When cascaded with K,
So, P=2 GMdB = –20 dB – 20 log10(K) > 0 dB
N = P–Z 20 dB + 20 log10 (K) < 0
0 = 2–Z 20 log10 (K) < –20
Z=2 K < 10–1 = 0.10
Z = 2 indicates, there are two closed-loop poles So, K 0 = 0.10
in right half of s-plane.
GATE Previous Years Solved Paper 125

55. (a) OL poles = 3[3 OL poles in the RHS]


P = 3
Corner frequency (in Hz) No. of poles (or) zeros
N(–1, 0) = (P – Z)
10 1 pole O = 3–Z
2
10 2 poles Z = 3 (3 CL poles in the RHS plane)
3
10 1 zero
4
58. (c)
10 2 zeros
5 For K = 1,
10 2 poles
6
1 pole 0.4 K
10
0.8 K
Number of poles (Np) = 5
Number of zeros (Nz) = 3

56. (b)
(–1, 0) 0.4
s-plane contour is encircling 2-poles and 3-zeros
in clockwise direction hence the corresponding
G(s) plane contour encirlces origin 2-times in
anticlockwise direction and 3-times in 0.8
clockwise direction. For K = 2, the plot will be
Effectively once in clockwise direction.
–0.8 K

57. (b) –1.6 K

jImGH

G(s ) H(s) plane


(–2, 0)

–1 + j0

0 Re GH N = Number of encirclement about (–1, 0) in


anticlockwise.
P = Total number of open-loop poles, in RHS,
N=0 Z = P–N
N=2 N = –2, P = 0
Z = 0 – (–2) = 2
Open zeros in the RH, Z = 1
Z=2
N(0, 0) = (OL poles – OL zeros)
Two poles lies in right side.
2 = OL poles – 1
126 Electronics Engineering Control Systems

Answers
EE Frequency Response Analysis

1. (b) 2. (b, d) 3. (a) 4. (d) 5. (d) 6. (d) 7. (a) 8. (a)

9. (b) 10. (c) 11. (c) 12. (d) 13. (b) 14. (c) 15. (a) 16. (d)

17. (b) 18. (c) 19. (d) 20. (b) 21. (d) 22. (d) 23. (a) 24. (c)

25. (b) 26. (0.755) 27. (a) 28. (c) 29. (d) 30. (b) 31. (a) 32. (a)

33. (a) 34. (1.047) 35. (d) 36. (b) 37. (a) 38. (2) 39. (c)

Solutions
EE Frequency Response Analysis

1. (b) Applying Routh criteria,


Q Curve starts from –180° type 2 system and
s3 1 1
Q Curve ends at –360°.
Difference between number of poles and s2 4 5
9
360° s1 0
number of zeros = =4 4
90°
s0 5
Q only option (b) is satisfying these two
properties. Hence, number of closed-loop poles at RHS of
Option (b) is correct. s-plane = number of sign changes = one.
Number of encirclements of G(s) about origin =
2. (b, d) Number of encirclements of G(s) H(s) about
Type = 1 (–1, 0) = {Number of open-loop poles at RHS of
Poles at = 0, 0.05 s-plane – Number of closed-loop poles at RHS
Zer at = 0.1 of s-plane}.
Using inital line equation, N = P–Z=1–1=0
dB = 20 log K – 20 log( ) No encirclement of origin of G(s).
At = 0.01,
4. (d)
dB = 60
60 = –20 log(0.01) + 20 log K Q r(t) = sint
On solving, K = 10 =1

10(1 + 10s) 1 1 1
T.F. = Gain = = =
s (1 + 20s) (1 + s) 1 + j 1+ j

5(s + 0.1) 1
= = 45°
s (s + 0.05) 2

1
3. (a) Output = sin t × 45°
2
1
G(s) = 1
(s 1) (s + 2) (s + 3) = sin(t 45°)
2
1 + G(s) H(s) = 0
i.e., s3 + 4s2 + s – 5 = 0
GATE Previous Years Solved Paper 127

5. (d) K
Magnitude of G( j ) = G( j ) =
Nyquist plot will be 2
1+ 2
Im a

Phase of G(j ) G(j ) = tan 1


a
At = 0.5a;

G( j ) K K
= 0.5 a = =
Re (0.5 a)2 1.25
1.42 1+
a2
In dB
= 20 log K – 20 log (1.25)1/2
= 20 log K – 0.97 dB
From the plot,
Number of encirclement of (–1, j0) = –2 and magnitude = 20 log K
number of right sided poles in open-loop error in dB gain = 0.97 dB
system = 0 1 0.5 a
G( j ) = 0.5 a = tan = 26.56°
N = P–Z a
–2 = 0 – 2 Since it is –45°/dec (–ve slope) at = 0.5a
Z=2 phase = –22.5°
closed-loop system is unstable with two
poles on the right half of s-plane. Error in phase of G( j ) = 0.5a

= –22.5 – (–26.56)
6. (d)
= +4.06° +4.9°
Type-2:
Poles: 0, 0, 2, 25 8. (a)
Zeros: 5 Im
Gain = –40 log + 20 log K
Wg
54 = –40 log(0.1) + 20 log K
Re
K=5 (–1, j0)
1
5(1 + 0.2 s)
T.F. = 2
s (1 + 0.5s ) (1 + 0.04s)

50(s + 5)
= 2
s (s + 2) (s + 25) g is gain cross over frequency at which the gain

G( j ) H ( j ) becomes unity.
7. (a)
In this case, phase G(j ) H(j ) is –180° at = g,
K = –180°
Transfer function = G(s ) =
s So, phase margin = 180° + = 0°
1+
a
Put s = j ; 9. (b)

K x(t) = sint = 1 < 0


G( j ) = = 1 rad/sec.
1+ j
a s
T.F. = G(s ) =
s+1
128 Electronics Engineering Control Systems

For sinusoidal input, 1


s = j =j 1 + a2 ×
a2
2 =1
j j 1 1/ a
G( j ) =1 = = = 45°
j +1 j+1 2 1
y(t) = = 2
a2
1 1 1
45° 1 0° = 45° a= = 0.841
2 2 2 1 /4

1
y(t) = sin(t + 45°) 11. (c)
2
s2 + 4
10. (c) T(s) =
(s + 1) (s + 4)
Im For frequency response, put s = j

( j )2 + 4
T(j ) =
( j + 1) ( j + 4)

Re 2
4
–1 + j0 =
(1 + j ) (4 + j )
Magnitude of
Wg
2
4
T(j ) = T ( j ) =
2
(1 + ) (16 + )2

as + 1 2
4
G(s) = 2 T( j ) = =0
s (1 + 2
) (16 + )2
g = gain crossover frequency
= 2 rad/sec.
at
The system output is zero at 2 rad/sec.
which open gain is loop,
phase margin = 180° + G(j ) 12. (d)
45° = 180° + G(j g)
Im
G(j g) = –130°
ja g + 1 (–1 + j0)
G(j g) = 2 Re
(j g)
= –2 × 90° + tan–1 a g
G(j g) = –180° + tan–1 a g = –135°
tan–1 a g = 45° –1 + j0 is not enclosed, system is always stable.
a g= 1 G.M. =
1
g = 13. (b)
a
0.25s
Gain = G( j g ) = 1 e
G(s) H(s) =
s
1 + a2 2g Putting s = j , for frequency response,
2 =1 j 0.25
g e
G(j ) H(j ) =
j
GATE Previous Years Solved Paper 129

phase angle = G(j ) H(j ) 1


where, K= = 0.1
= –90 – 0.25 10
At pc Nyquist plot cuts negative real axis and Corner frequencies,
this frequency G(j pc) H(j pc) is –180°.
1 = 1 rad/sec.

G(j 90° 2 = 10 rad/sec.


pc) H(j pc) =
4 and 3 = 100 rad/sec.

–180° = 90°
4 For frequency less than 1 i.e. < 1
= 360° or 2 Gain of the system is constant as there is no
pole at origin.
j 0.25 × 2
e Gain = 20 log K = 20 log 0.1
G( j pc ) H ( j pc ) = = 0.5
j2 = –20 dB
At = 1 = 1 rad/sec or log 1 = log = 0
14. (c) There is zero, so system gain increases with slope
K + 0.366s +20 dB/decade and system gain becomes 0 dB
G(s) = and H(s) = 1
s(s + 1) at = 10 rad/sec or
K + 0.366s log = 10 = log10 = 1
O.L.T.F. = G(s) H (s) =
s(s + 1)
At = 10 or log = log 10 = 1
Phase margin = 60° at pc 2
2 = 10
(gain crossover frequency) There is pole, so slope is –20 dB/decade.
180° + G(j pc) H(j pc) = 60° Overall slope 2 < < 3
G(j pc) H(j pc) = –120° = 20 dB/decade – 20 dB/decade
0.366 = 0 dB/decade
1 1 gc
90° tan pc + tan = 120° So, gain remains constant between,
K
2 < < 3
gc = 1 rad/sec
or, 1 < log < 2
0.366 × 1
90° tan 1 (1) + tan 1 = 120° At = 3 = 100 rad/sec or
K
log = log 100 = 2
0.366 3 = 100
1
= 90° 45° + tan = 120°
K The double pole are present.
0.366 So, system gain decrease with –40 dB/decade.
1
tan = 15°
K 17. (b)
0.366
K= = 1.366 1
tan 15° G(s) =
s(s + 1) (s + 2)
15. (a) Put s = j ,
1
10 4 (1 + j ) G(j ) =
H(j ) = j (1 + j ) (2 + j )
(10 + j ) (100 + j )2
j(1 j ) (2 j )
(1 + j ) = 2 2
= (1 + ) (4 + )
2
j j
10 × 1 + × 1+ j(2 3 j 2
)
10 100 = 2 2
(1 + ) (4 + )
K (1 + j )
H(j ) = 2 2
j j 3 j(2 )
1+ × 1+ = 2 2
10 100 (1 + ) (4 + )
130 Electronics Engineering Control Systems

2 At = 2 rad/sec, slope changes by –20 dB/dec.


3 j( 2)
= 2 2
+ 2 2 from –40 dB/decade to –60 dB/decade Hence
(1 + ) (4 + ) (1 + ) (4 + )
the corresponding term of the transfer function
= x + iy
At 0, x –3/4, y – 1
is .
s
18. (c) 1+
2
Initial slope is –40 dB/decade, it means there
are double pole or origin. At = 5 rad/sec, slope changes by 20 dB/decade
Slope changes from –40 dB/degree to from –60 dB/decade to –40 dB/decade. Hence
–20 dB/decade. It means there is a zero. the corresponding term of the transfer function
Slope changes from –20 dB/decade to s
0 dB/decade at some other frequency. It means is 1 + .
5
there is one more zero.
Therefore transfer function has two poles and At = 25 the slope changes by –20 dB/decade
two zeros. from –40 dB/decade to –60 dB/decade. Hence
the corresponding term of the transfer function
19. (d)
1
is .
s
Im 1+
– 25
=0

s
K 1+
5
G(s) =
s s
s2 1 + 1+
+ 2 25
–1 =
Re
–1.42 = – At = 0.1 ( 2 rad/sec)
Gain (dB) = 20 log K – 40 log
80 = 20 log K – 40 log 0.1
20 log K = 40
Nyquist-plot
K = 100
+
=0
s
100 1 +
5
G(s) =
Two clockwise encirclement of 1 + j0, s s
s2 1 + 1+
N = –2 2 25
Open-loop system is stable P = 0
1000(s + 5)
N = P–Z = 2
s (s + 2) (s + 25)
–2 = 0 – Z
Z = Number of closed-loop poles in RHS of
21. (d)
s-planes. Hence the system is unstable.
Open-loop transfer function,
20. (b) e 0.1s

Initial slope = –40 dB/decade G(s) =


s
Hence the system is type-2. So the Put s = j ,
corresponding term of the transfer function is j 0.1
e
1/s2. G(j ) =
j
GATE Previous Years Solved Paper 131

At phase crossover frequency ( pc), phase of For zero steady-state output,


OLTF is –180°, 2 = 9

G( j ) = 3 rad/sec
/ pc = –180° = –
25. (b)
0.1 pc = –
2 K
Let, G(s) =
sn
–0.1 =
pc 2 K
G(j ) =
pc =5 ( j )n

e 0.1
is always 1 for any value of , Q Gain = 32 dB at = 1 rad/sec.

1 K
G( j ) 20 log n = 32
pc = 5 = 1
5
1 20 log K = 32
Gain margin = 20 log
G( j K = 39.8
pc )
Now,
= 20 log 5 = 23.9 dB
Q Gain = –8 dB at = 100 rad/sec.
23. (a) 39.8
20 log = –8
At gain across over frequency ( gc), magnitude 10 n
of G(j ) is 1. 10 n = 100
G( j n=2
gc ) =1
Phase of G(j ) = G(j gc) = –150° 26. Sol.
Phase margin = 180° + G(j gc) K(1 + 0.5s) (1 + as)
= 180° – 150° = 30° Given, G(s) =
s s
s 1+ (1 + bs ) 1 +
At phase cross frequency ( pc), phase of G(j ) is 8 36
–180°, G(j pc) = –180°
(1 + as) is addition of zero to the transfer function
M = magnitude of G(j ) at
whose contribution in slope = +20 dB/decade
pc = G( j pc ) = 0.5 or –6 dB/octave.
(1 + bs) is addition of pole to the transfer function
1 1
Gain margin = 20 log = 20 log = 6 dB whose contribution in slope = –20 dB/decade
M 0.5
or –6 dB/octave.
24. (c) Observing the change in the slope at different
For sinusoidal excitation, corner frequencies, we conclude that,
s=j 1
a= rad/sec.
4
( 2 + 9) ( j + 2)
G(j ) = 1
( j + 1) ( j + 3) ( j + 4) and b= rad/sec.
24
For zero steady-state output,
From = 0.01 rad/sec to = 8 rad/sec.
G( j ) = 0 slope = –20 dB/decade
Let the vertical length in dB be y,
2 2
( + 9) +4
= 0 y
2 2 2 –20 =
( + 1) ( + 9) ( + 16 )
log 8 log 0.01
132 Electronics Engineering Control Systems

y 28. (c)
or, –20 =
log 8 + 2 From the given Bode plot, it is clear that corner
or, y = 58 dB frequencies,
Appyling, y = mx + C at = 0.01 rad/sec. 1
we have, 58 = –20 log 0.01 + C c1 and c2 = 1
=
3
or, C = 58 – 40 = 18 Transfer function of given system is given
Now, C = 20 log K by
18 C(s) 1 + 3s 1 + Ts
or, log K = = 0.9 = =
20 T(s) =
R(s ) 1+s 1 + Ts
K = log–1 (0.9) = (10)0.9 = 7.94
1
1 Here, T = 3 and T = 1 or =
3
a 4
= As < 1, therefore the transfer function T(s)
bK 1
× 7.94
24 1
represents a lead compensator having = .
24 3
= = 0.755
4 × 7.94 Maximum phase shift,
a
= 0.755 1
bK 1
1 1 1 3
= tan = tan
m
2 2
27. (a)
3
5(s + 4)
Given, G(s) = 1 2 3 1
s (s + 0.25) (s2 + 4s + 25) = tan × = tan 1 = 30°
3 2 3
Reducing G(s) in time constant form, we have
1+3
s Also, T(j ) =
5× 4 1 + 1+ j
4
G(s) =
s 4 1 1+9 2
s 0.25 1 + 25 1 + s + s2
0.25 25 25 T( j ) =
2
1+
20(1 + 0.25s)
= Gain Gm in dB = 20 log T ( j )
s [0.25(1 + 4s)] [25(1 + 0.16s + 0.04s2 )]
2
or, 1+9 m
= 20 log 10 2
3.2(1 + 0.25s) 1+ m
G(s) = 2
s (1 + 4s) (1 + 0.16s + 0.04s ) 1 1
Now, m = =
The value of constant gain term = 3.2 T 3
Also,
2
1
j 1+ 9×
3.2 1 + 3
4 Gm = 20 log 10 2
G(j ) = 2 1
j 1+
(j ) 1+ 1 + j 0.16 3
0.25 25
Corner frequencies are:
1+ 3
1 = 4 rad/sec = 20 log 10 = 20 log 10 3
1
2 = 0.25 rad/sec 1+
3
and 3 = 5 rad/sec
Highest corner frequency = 3 = 5 rad/sec. or, GM = 4.77 dB
GATE Previous Years Solved Paper 133

29. (d) 32. (a)


From initial line equation, From the given Bode plot, it is evident that there
dB = 20 log10 K ...(i) are 3-poles in the transfer function, out of which
As we know, there are double poles at corner frequency near
20 dB = 20 log10 K but less than = 8 rad/sec and one pole is near
i.e., K = 10 but greater than = 0.5 rad/sec. The initial slope
Now, from the given plot we know type of system is +20 dB/decade. Therefore one zero exist at
is one. s = 0. So from all the given options, option (a)
K (1 + sT1 ) satisfies all the conditions. Therefore option (a)
Hence, G(s) = is correct.
(1 + sT2 )
1 1 33. (a)
T1 = , T2 =
10 1000
s+3
10(1 + 0.001s) Nyquist plot of G(s ) H (s ) = 2 is as
= s (s 3)
(1 + 0.1s)
shown below.
1 (1000 + s)
=
10 (s + 10)

30. (b)
1
G1(s) =
s
=
=0
G2(s) = s
=0
1 =
G1(s) × G2(s) = s=1
s

31. (a)
100
G(s) =
(s + 1)3
From the Nyquist plot G(s) H(s) encircle –1 + j0
100 once in clockwise direction.
G(j ) =
(1 + j )3
34. Sol.
100
= Forward path transfer function,
1 + ( j )3 + 3( j )3 + 3 j
s
Ke
100 G(s) =
= s
(1 3 2 ) + j (3 3
)
Given:
2 2 2 Phase margin = 30°
100[(1 3 ) j (3 ) ]
=
[(1 3 2
)+ 2
(3 2 2
) ] or, Phase margin = 180°
30° = 180° +
For phase crossover frequency,
= –150°
Img[G(j )] = 0
Hence, (3 – 2) = 0 = G( j ) = gc

or, = 0, ± 3 [where, gc is gain crossover frequency]

Therefore, = G(j ) = –90° – 57.3


ph 3 rad/sec.
134 Electronics Engineering Control Systems

At = gc 37. (a)
(gain crossover frequency)
K
G(s) =
G( j ) = 1 s s
s 1+ 1+
1 20
K ×1
or, =1 Transfer function shows two-poles and no
zeros. So statement 1 is wrong.
= gc = K rad/sec.
1 1
G( j ) = gc = –90° – 57.3 K G(j ) = 90 tan tan
20
–90° – 57.3 K° = –150° 3
G( j ) = 270° = radian
– 57.3 K° = –60° 2
60 So, statement 2 is correct.
K= = 1.047
57.3
38. Sol.
35. (d) K
O.L.T.F. G(s) =
Vo (s) 1 s ( s + a ) (s b ) ( s + c )
= = H (s)
Vi (s) 1+s N = P – Z, P=1
For the minimum and maximum values of ‘ ’, –1 = 1 – Z, N = –1
1 j Z=2
H(j ) =
1+ j
39. (c)
H(j ) = –2 tan–1
At = 0; s2 + 100
H(s) = T.F. =
H(j ) = 0° s p
At = , D.C. gain = 5
H(j ) = – 100
= 5 = p – 20
36. (b) p
2
180 pc + 100
G(j ) = 0.25 × 90° H(j ) =
j + 20
= –180° 2
+ 100
H( j ) =
180 pc 2
0.25 × = –90° + 400
2
+ 100
4 =1
pc = =2 2
+ 400
2
0.25s = 8.84 rad/sec.
G( j ) e
= pc = = = 0.5
s 2
= pc

Point is (–0.5, j0).


7 State Space Analysis

ELECTRO NICS EN GINEERIN G Q.4 A certain linear time invariant system has the
state and the output equations given below:
(GATE Previous Years Solved Papers)
x1 1 1 x1 0
= + u
Q.1 Given the following state space description of a x2 0 1 x2 1
system,
x1
y = [1 1] is
x1 2 0 x1 0 x2
= + u
x2 0 4 x2 1 dy
x1(0) = 1, x2(0) = –1, u(0) = 0 then is
x1 dt t = 0
y = [1 0]
x2 (a) 1 (b) –1
Find the state transmission matrix. (c) 0 (d) None of the above
[EC-1988 : 2 Marks] [EC-1997 : 2 Marks]

Q.2 A linear second-order single-input continuous Q.5 The system mode described by the state
time system is described by following set of equations,
differential equations: 0 1 0
x = x+ u
x1 (t ) = –2x1(t) + 4x2(t) 2 3 1
Y = [1 1] x is
x2 (t ) = 2x1(t) – x2(t) + u(t)
(a) controllable and observable
where, x1(t) and x2(t) are the state variables and
(b) controllable, but not observable
u(t) is the control variable. The system is
(c) observable, but not controllable
(a) controllable and stable.
(d) neither controllable nor observable
(b) controllable but unstable.
[EC-1999 : 1 Mark]
(c) uncontrollable and unstable.
(d) uncontrollable but stable. Q.6 For the system described by the state equation,
[EC-1991 : 2 Marks] 0 1 0 0
Q.3 A linear time invariant system is described by x= 0 0 1 x+ 0 u
the state variable model 0.5 1 2 1

x1 1 0 x1 0 If the control signal ‘u’ is given by


= + u
x2 0 2 x2 1 u = [–0.5 – 3 – 5] x + v
then the eigen values of the closed-loop system
x1 will be
Y = [1 2]
x2 (a) 0, –1, –2 (b) 0, –1, –3
(a) The system is completely controllable. (c) –1, –1, –2 (d) 0, –1, –1
(b) The system is not completely controllable. [EC-1999 : 2 Marks]
(c) The system is completely observable. Q.7 The transfer function Y(s)/U(s) of a system
(d) The system is not completely observable. described by the state equations,
[EC-1992 : 2 Marks] x(t ) = 2 x(t ) + 2 u(t ) and y(t) = 0.5x(t) is
136 Electronics Engineering Control Systems

0.5 1 The eigen values of the representations are also


(a) (b) computed as [ ] and [µ]. Which one of the
(s 2) (s 2)
0.5 1 following statements is true?
(c) (d) (a) [ ] = [µ] and X = W
(s + 2) (s + 2)
(b) [ ] = [µ] and X W
[EC-2002 : 1 Mark]
(c) [ ] [µ] and X = W
Q.8 The zero-input response of a system given by
(d) [ ] [µ] and X W [EC-2005 : 1 Mark]
state-space equation,
x1 1 0 x1 Q.12 A linear system is described by the following
= state equation:
x2 1 1 x2
1 0 1
x1 (0) X (t ) = AX(t ) + BU (t ), A =
and = is 1 0
x2 (0) 0
The state-transition matrix of the system is
tet et
(a) (b) cos t sin t cos t sin t
t t (a) (b)
sin t cos t sin t cos t
t t
e
(c) (d) cos t sin t cos t sin t
tet tet (c) sin t cos t (d) cos t sin t
[EC-2003 : 2 Marks] [EC-2006 : 2 Marks]
Q.9 The state variable equations of a system are: Q.13 The state space representation of a separately
1. x1 = 3 x1 x 2 + u excited dc servo-motor dynamics is given as,
2. x 2 = 2 x1 ; y = x1 + u d
The system is dt 1 1 0
= + u
(a) controllable but not observable. dia 1 10 ia 10
(b) observable but not controllable. dt
(c) neither controllable nor observable. where is the speed of the motor, ia is the
(d) controllable and observable. armature current and u is the armature voltage.
[EC-2004 : 2 Marks] The transer function (s)/U(s) of the motor is
10 1
(a) (b)
1 0 s 2 + 11s + 11 2
s + 11s + 11
Q.10 Given, A = , the state transition matrix
0 1 10s + 10 1
(c) (d)
s 2 + 11s + 11 s2 + s + 1
eAT is given by
[EC-2007 : 2 Marks]
0 e t et 0
(a) (b) Statement for Linked Answer Questions (14 and 15):
e t
0 0 et
Consider a linear system whose state space
t t
e 0 0 e representation is x(t ) = Ax (t ). If the initial state vector
(c) (d) t
t e 0
0 e
1
[EC-2004 : 2 Marks] of the system is x(0) = , then the system response
2
Q.11 A linear system is equivalently represented by
two sets of state equations: e 2t

X = AX + BU and W = CW + DU is x(t ) = .
2t
2e
GATE Previous Years Solved Paper 137

If the initial state vector of the system changes to The set of equations that correspond to this
signal flow graph is
1
x(0) = . Then the system response becomes x1 0 x1 1 0
1 u1
d
(a) x2 = 0 x2 + 0 0
dt u2
t x3 0 x3 0 1
e
x (t ) = .
t x1 0 x1 0 0
e
d u1
(b) x2 = 0 x2 + 0 1
dt u2
Q.14 The eigen value and eigen-vector pairs ( i, vi) x3 0 x3 1 0
for the system are:
x1 0 x1 1 0
d u1
1 1 (c) x2 = 0 x2 + 0 1
(a) 1, and 2, dt u2
1 2 x3 0 x3 0 0

1 1 x1 0 x1 0 1
(b) 2, and 1, d u1
1 2 (d) x 2 = 0 x2 + 0 0
dt u2
x3 0 x3 1 0
1 1
(c) 1, and 2, [EC-2008 : 2 Marks]
1 2

1 1 0 1
(d) 2, and 1, Q.17 Consider the matrix P = . The value
1 2 2 3

[EC-2007 : 2 Marks] of eP is

Q.15 The system matrix A is 2 1 1 2


2e 3e e e
(a) 2 1 2 1
0 1 1 1 2e 2e 5e e
(a) 1 1 (b) 1 2
1 2 2 1
e +e 2e e
1 1 0 1 (b) 1
(c) (d) 2e 4e2 3e 1
+ 2e 2
1 2 2 3
2 1 1 2
[EC-2007 : 2 Marks] 5e e 3e e
(c)
2 1 2 1
Q.16 A signal flow graph of a system is given below. 2e 6e 4e +e

– 1 2 1 2
2e e e e
(d)
1 2 1 2
2e + 2e e + 2e
1 x3 1/s
x3 [EC-2008 : 2 Marks]

dx
x2 x2 Q.18 Consider the system = Ax + Bu with
1/s dt

x1 1 0 p
x1 A= and B = where p and q are
1 1/s 0 1 q

arbitrary real numbers. Which of the following


– statements about the controllability of the
system is true?
138 Electronics Engineering Control Systems

(a) The system is completely state controllable


1
for any non-zero values of p and q. V
s+2
(b) Only p = 0 and q = 0 result in controllability.
Y
(c) The system is uncontrollable for all values
2
of p and q.
s+2
(d) We cannot conclude about controllability
from the given data. A state space model of the above system in terms
[EC-2009 : 1 Mark] of the state vector x and the output vector
y = [y1 y2]T is
Common Data Questions (19 and 20):
(a) x = [2]x + [1] u ; y = [1 2] x
The signal flow graph of a system is shown below:
1 1
(b) x = [ 2]x + [1] u ; y= x
2
2 1/s 1/s 0.5
2 0 1
U(s) 1 Y(s ) (c) x = x+ u; y = [1 2] x
0 2 1
–1
2 0 1 1
(d) x = x+ u; y= x
–1 0 2 2 2
Q.19 The state variable representation of the system [EC-2011 : 2 Marks]
can be
Q.22 The state variable description of an LTI system
1 1 0 is given by:
(a) x = x+ u
1 0 2
x1 0 a1 0 x1 0
y = [0 0.5] x x2 = 0 0 a2 x2 + 0 u
1 1 0 x3 a3 0 0 x3 1
(b) x = x+ u
1 0 2 x1
y = [0 0.5] x y = (1 0 0) x2
1 1 0 x3
(c) x = x+ u
1 0 2 where y is the output and u is the input. The
y = [0.5 0.5] x system is controllable for
(a) a1 0, a2 = 0, a3 0
1 1 0
(d) x = x+ u (b) a1 = 0, a2 0, a3 0
1 0 2
(c) a1 = 0, a2 0, a3 = 0
y = [0.5 0.5] x
(d) a1 0, a2 0, a3 = 0
[EC-2010 : 2 Marks]
[EC-2012 : 2 Marks]
Q.20 The transfer function of the system is
Statement for Linked Answer Questions (23 and 24):
s+1 s 1
(a) (b) The state diagram of a system is shown below is
s2 + 1 s2 + 1 described by the state-variable equations:
s+1 s 1
(c) 2
(d) 2 X = AX + Bu; y = CX + Du
s +s+1 s +s+1
1 x1 –1 x1 1 x2 –1 x2 1
[EC-2010 : 2 Marks] u y

Q.21 The block diagram of a system with one input u


and two outputs y1 and y2 is given below. 1/s 1/s
GATE Previous Years Solved Paper 139

Q.23 The state-variable equations of the system in the Q.26 An unforced linear time invariant (LTI) system
figure above are: is represented by
x1 1 0 x1
1 0 1 =
(a) X = X+ u x2 0 2 x2
1 1 1
y = [1 1] X + u If the initial conditons are x 1 (0) = 1 and
x2(0) = –1, the solution of the state equation is
1 0 1 (a) x1(t) = –1, x2(t) = 2
(b) X = X+ u
1 1 1 (b) x1(t) = –e–t, x2(t) = 2e–t
y =[ 1 1] X + u (c) x1(t) = e–t, x2(t) = –e–2t
(d) x1(t) = –e–t, x2(t) = –2e–t
1 0 1
(c) X = X+ u [EC-2014 : 2 Marks]
1 1 1
y =[ 1 1] X u Q.27 Consider the state space system expressed by
the signal flow diagram shown in the figure.
1 1 1
(d) X = X+ u c3
c2
0 1 1
y = [1 1] X u 1 s
–1
s
–1
s
–1
u y
x3 x2 x1 c 1
[EC-2013 : 2 Marks] a3
a2
Q.24 The state transition matrix e4t of the system a1
shown in figure above is
The corresponding system is
t t
e 0 e 0 (a) always controllable.
(a) t t
(b) t t
te e te e (b) always observable.
(c) always stable.
t t t
e 0 e te (d) always unstable. [EC-2014 : 2 Marks]
(c) t t
(d) t
e e 0 e
Q.28 The state equation of a second-order linear
[EC-2013 : 2 Marks] system is given by
Q.25 Consider the state space model of a system, as x(t ) = Ax(t ), x(0) = x0
given below: t
1 e 0
x1 1 1 0 x1 0 For x0 = , x( t ) = and for x0 = ,
1 e t 1
x2 = 0 1 0 x2 + 4 u
t 2t
x3 0 0 2 x3 0 e e
x( t ) =
t 2t
x1 e + 2e
y = [1 1 1] x2 3
When x0 = , x(t ) is
x3 5
The system is 8 e t + 11 e 2 t 11 e t
8e 2t

(a) controllable and observable. (a) (b)


8e t 22 e 2 t 11 e t
+ 16 e 2t
(b) uncontrollable and observable.
t 2t t 2t
(c) uncontrollable and unobservable. 3e 5e 5e +6e
(c) t 2t (d) t 2t
(d) controllable and unobservable. 3e + 10 e 5e +6e
[EC-2014 : 2 Marks] [EC-2014 : 2 Marks]
140 Electronics Engineering Control Systems

Q.29 The state transition matrix (t) of a system (a) controllable but not observable.
(b) observable but not controllable.
x1 0 1 x1
= is (c) both controllable and observable.
x2 0 0 x2
(d) neither controllable nor observable.
t 1 1 0 [EC-2016 : 2 Marks]
(a) (b)
1 0 t 1
Q.33 Consider the state space realization:
0 1 1 t
(c) (d) x 1 (t ) 0 0 x1 (t ) 0
1 t 0 1 = + u(t ) with the
x 2 (t ) 0 9 x 2 (t ) 45
[EC-2014 : 2 Marks]
x1 (0) 0
Q.30 The state variable representation of a system is initial condition = , where u(t)
x2 (0) 0
given as,
denotes the unit step function. The value of
0 1 1
x = x; x(0) =
0 1 0
lim x12 (t ) + x22 (t ) is _______ .
y = [0 1]x t
The response y(t) is [EC-2017 : 1 Mark]
(a) sin(t) (b) 1 – et
Q.34 A second-order LTI system is described by the
(c) 1 – cos(t) (d) 0
following state equations:
[EC-2015 : 2 Marks]
d
x 1 (t ) x 2 ( t ) = 0
Q.31 A network is described by the state model as, dt
x1 = 2x1 – x2 + 3u d
x2 (t ) + 2 x1 (t ) + 3x2 (t ) = r(t)
dt
x2 = –4x2 – u
where x1(t) and x2(t) are the two state variables
y = 3x1 – 2x2 and r(t) denotes the input. The output c(t) = x1(t).
Y (s) The system is
The transfer function, H (s ) = is
U (s) (a) undamped (oscillatory)
(b) underdamped
11 s + 35 11 s 35
(a) (b) (c) critically damped
(s 2) (s + 4) (s 2) (s + 4)
(d) overdamped [EC-2017 : 2 Marks]
11 s + 38 11 s 38
(c) (d) Q.35 The state equation and the output equation of a
(s 2) (s + 4) (s 2) (s + 4)
control system are given below:
[EC-2015 : 2 Marks]
4 1.5 2
x = x+ u
Q.32 A second-order linear time-invariant system is 4 0 0
described by the following state equations: y = [1.5 0.625 x]
d The transfer function representation of the
x1 (t ) + 2 x1 (t ) = 3u(t )
dt system is
d 3s + 5 3s 1.875
x2 (t ) + x2 (t ) = u(t ) (a) (b)
dt
2
s + 4s + 6 s 2 + 4s + 6

where x1(t) and x2(t) are the two state variables 4s + 1.5 6s + 5
(c) 2 (d) 2
and u(t) denotes the input. If the output s + 4s + 6 s + 4s + 6
c(t) = x1(t), then the system is [EC-2018 : 2 Marks]
GATE Previous Years Solved Paper 141

Q.36 Let the state-space representation of an LTI


d v 4 4 v 4 4 i1
system be (c) = +
dt i 2 4 i 4 0 i2
X(t ) = AX(t) + Bu(t)
y(t) = CX(t) + du(t) d v 4 4 v 0 4 i1
(d) = +
dt i 2 4 i 4 4 i2
where A, B, C are matrices, d is a scalar, u(t) is
the input to the system, and y(t) is its output. Let [EC-2020 : 2 Marks]
B = [0 0 1]T and d = 0. Which one of the following
Q.38 The electrical system shown in the figure
options for A and C will ensure that the transfer
converts input source current iS(t) to output
function of this LTI system is
voltage vo(t).
1
H (s) = 3 2
s + 3s + 2 s + 1 iL(t )

0 1 0 1H
vo(t)
(a) A = 0 0 1 and C = [0 0 1] 1
3 2 1
+
is(t) vC(t) 1F 1

0 1 0
(b) A = 0 0 1 and C = [0 0 1]
1 2 3

0 1 0 Current iL(t) in the inductor and voltage vc(t)


(c) A = 0 0 1 and C = [1 0 0] across the capacitor are taken as the state
1 2 3 variables, both assumed to be initially equal to
zero i.e., iL(0) = 0 and vc(0) = 0. The system is
0 1 0 (a) completely state controllable as well as
(d) A = 0 0 1 and C = [1 0 0] completely observable.
3 2 1 (b) neither state controllable nor observable.
(c) completely state controllable but not
[EC-2019 : 2 Marks]
observable.
Q.37 For the given circuit, which one of the following (d) completely observable but not state
is the correct state equation? controllable.
0.5 H [EC-2021 : 2 Marks]
i
ELECTRICAL EN GINEERIN G
+
i1 2 V 0.25 F 1 i2 (GATE Previous Years Solved Papers)

Q.1 The transfer function for the state variable

representation X = AX + BU , Y = CX + DU , is
d v 4 4 v 0 4 i1
(a) = + given as
dt i 2 4 i 4 0 i2
(a) D + C(sI – A)–1 B (b) B(sI – A)–1 C + D
d v 4 4 v 4 0 i1 (c) D(sI – A)–1 B + C (d) C(sI – A)–1 D + B
(b) = +
dt i 2 4 i 0 4 i2 [EE-1993 : 1 Mark]
142 Electronics Engineering Control Systems

Q.2 Consider a second order system whose state Q.7 Given the homogenous state-space equation
space representation is of the form
3 1
X = AX + BU . If x1(t) = x2(t), then system is X= x the steady-state value of
0 2
(a) controllable (b) uncontrollable
(c) observable (d) unstable xss = lim x(t ), given the initial state value of
t
[EE-1993 : 1 Mark]
x(t) = [10 – 10]T, is
Q.3 The matrix of any state space equations for the 0 3
transfer function C(s)/R(s) of the system, shown (a) xss = (b) xss =
0 2
below in figure is
10
(c) xss = (d) xss =
R(s ) 3 +

1/s C(s ) 10
[EE-2001 : 1 Mark]

Q.8 The state transition matrix for the system


1 0 1 0
(a) (b)
0 1 0 1 X = AX with initial state X(0) is

(c) [–1] (d) [3] (a) (sI – A)–1


[EE-1994 : 1 Mark] (b) eAt X(0)
(c) Laplace inverse of [(sI – A)–1]
a 1 (d) Laplace inverse of [(sI – A)–1 X(0)]
Q.4 The eigen-values of the matrix are
a 1 [EE-2008 : 1 Mark]

(a) (a + 1), 0 (b) a, 0 Q.9 For the system


(c) (a – 1), 0 (d) 0, 0 2 0 1
X= X+ u ; y = [4 0] X
[EE-1994 : 1 Mark] 0 4 1

Q.5 A system is described by the state equation with ‘u’ as unit impulse and with zero initial
state, the output ‘y’ becomes
X = AX + BU . The output is given by Y = CX.
(a) 2e2t (b) 4e2t
4 1 1 (c) 2e4t (b) 4e4t
Where, A = ,B= , C = [1 0] .
3 1 1 [EE-2002 : 2 Marks]
Transfer function G(s) of the system is
s 1 2 3 1
(a) (b) Q.10 For the system X = X+ u , which of
2 2 0 5 0
s + 5s + 7 s + 5s + 7
s 1 the following statement is true?
(c) 2 (d) 2
s + 3s + 2 s + 3s + 2 (a) The system is controllable but unstable.
[EE-1995 : 1 Mark]
(b) The system is uncontrollable and unstable.
Q.6 Given the matrix: (c) The system is controllable and stable.
0 1 0 (d) The system is uncontrollable and stable.
A= 0 0 1 [EE-2002 : 2 Marks]
6 11 6 Q.11 A second order system starts with an initial
Its eigen values are ________ .
2
[EE-1995 : 1 Mark] condition of without any external input.
3
GATE Previous Years Solved Paper 143

The state transition matrix for the system is Statement for Common Data Questions (14 and 15):
2t
e 0 0 1 1
given by . The state of the system at A state variable system X (t ) = X (t ) + U (t ),
0 e t 0 3 0

the end of 1 second is given by with the initial condition X(0) = [–1 3]T and the unit
0.271 0.135 step input u(t) has
(a) (b)
1.100 0.368 Q.14 The state transition matrix
0.271 0.135 1 3t
(c) (d) 1 (1 e )
0.736 1.100 (a) 3
3t
[EE-2003 : 1 Mark] 0 e

Q.12 The following equation defines a separately 1 t 3t


1 (e e )
excited dc motor in the form of a differential (b) 3
t
equation, 0 e

d2 Bd K2 K 1 t 3t
2
+ + = Va 1 (e e )
dt J dt LJ LJ (c) 3
3t
The above equation may be organized in the 0 e
state space form as follows:
1 (1 e t )
(d) t
[EE-2005 : 2 Marks]
d2 0 e
d
dt 2 = P dt + QVa
d Q.15 The state transition equation
dt t e t
t e t
(a) X (t ) = (b) X (t ) =
where the ‘P’ matrix is given by e t
3e 3t

B K2 K2 B 3t 3t
t e t e
(a) J LJ (b) LJ J (c) X (t ) = (d) X (t ) =
3t t
1 0 0 1 3e e
[EE-2005 : 2 Marks]
0 1 1 0
K2 Q.16 For a system with the transfer function,
(c) B (d) B K2
LJ J J LJ 3(s 2)
H (s) = 3
[EE-2003 : 2 Marks] s + 4s 2 2 s + 1
The matrix A in the state space form x = Ax + Bu
Q.13 The state variable description of a linear
is equal to
autonomous system is, X = AX, where X is the
two dimensional state vector and A is the system 1 0 0 0 1 0
(a) 0 1 0 (b) 0 0 1
0 2
matrix given by A = . The roots of the 1 2 4 1 2 4
2 0
0 1 0 1 0 0
characteristic equation are
(c) 3 2 1 (d) 0 0 1
(a) –2 and +2 (b) –j2 and +j2
1 2 4 1 2 4
(c) –2 and –2 (d) +2 and +2
[EE-2006 : 1 Mark]
[EE-2004 : 2 Marks]
144 Electronics Engineering Control Systems

Q.17 The state equation for the current I1 shown in (a) 0 (b) 1
the network shown below in terms of the voltage (c) 2 (d)
Vx and the independent source V, is given by [EE-2008 : 2 Marks]
3 0.2 H 5 Statement for Common Data Questions (20 and 21):
+ V – A system is described by the following state and output
x I2
equations:
V I1 –
0.5 H + 0.2Vx
dx1 (t )
= 3x1 (t ) + x2 (t ) + 2 u(t )
dt
dx2 (t )
= 2 x2 (t ) + u(t )
dI1 5 dt
(a) = 1.4 Vx 3.75 I 1 + V
dt 4 y(t) = x1(t)
dI1 5 Q.20 The system transfer function is
(b) = 1.4 Vx 3.75 I1 V
dt 4
s+2 s+3
dI1 5 (a) 2 (b) 2
(c) = 1.4 Vx + 3.75 I 1 + V s + 5s 6 s + 5s + 6
dt 4
2s + 5 2s 5
dI1 5 (c) 2 (d) 2
(d) = 1.4 Vx + 3.75 I 1 V s + 5s + 6 s + 5s 6
dt 4
[EE-2009 : 2 Marks]
[EE-2007 : 2 Marks]
Q.21 The state-transition matrix of the above system
Statement for Linked Answer Questions (18 and 19):
is
The state space equation of a system is described by
x = Ax + Bu, y = Cx where x is state vector, u is input, y e 3t 0
(a)
e 2 t + e 3t e 2t
0 1 0
is output and A = ,B= , C = [1 0] . 3t 2t 3t
0 2 1 e e e
(b)
2t
0 e
Q.18 The transfer function G(s) of this system will be
3t 2t 3t
s s+1 e e +e
(a) (b) (c)
2t
(s + 2) s ( s 2) 0 e

s 1 e 3t e 2t
e 3t
(c) (d) (d)
(s 2) s ( s + 2) 2t
0 e
[EE-2008 : 2 Marks]
[EE-2009 : 2 Marks]
Q.19 A unity feedback is provided to the above system
G(s) to make it a closed-loop system as shown Q.22 The system X = AX + BU and
in figure. 1 2 0
A= , B=
+ 0 2 1 is
r (t ) G(s ) y(t )
– (a) stable and controllable
(b) stable but uncontrollable
(c) unstable but controllable
For a unit step input r(t), the steady state error (d) unstable and uncontrollable
in the input will be
[EE-2010 : 2 Marks]
GATE Previous Years Solved Paper 145

Q.23 The state variable description of an LTI system


et 0 et 0
is given by (a) t t (b) 2 t
e e t e et
x1 0 a1 0 x1 0
x2 = 0 0 a2 x2 + 0 u et 0 et t et
(c) (d)
x3 a3 0 0 x3 1 t et et 0 et
[EE-2014 : 1 Mark]
x1
y = (1 0 0) x2 Q.27 Consider the system described by following
x3 state space equations:
x1 0 1 x1 0
where ‘y’ is the output and ‘u’ is the input. The = + u
x2 1 1 x2 1
system is controllable for
(a) a1 0, a2 = 0, a3 0 x1
y = [1 0]
(b) a1 = 0, a2 0, a3 0 x2
(c) a1 = 0, a2 0, a3 = 0 If ‘u’ is unit step input, then the steady-state
(d) a1 0, a2 0, a3 = 0 error of the system is
[EE-2012 : 2 Marks] (a) 0 (b) 1/2
(c) 2/3 (d) 1
Common Data Questions (24 and 25):
[EE-2014 : 2 Marks]
The state variable formulation of a system is given as,
Q.28 In the signal flow diagram given in the figure,
x1 2 0 x1 1
= + u u1 and u2 are possible inputs whereas y1 and y2
x2 0 1 x2 2 are possible outputs. When would the SISO
x1(0) = (0), x2(0) = 0 system derived from this diagram be controllable
x1 and observable?
and y = [1 0] 5
x2

Q.24 The system is x1


u1 1/s y1
1 1
(a) controllable but not observable.
–2
(b) not controllable but observable.
1 1
(c) both controllable and observable.
(d) both not controllable and not observable. 2

[EE-2013 : 2 Marks] x2
u2 1/s y2
1 –1
Q.25 The response y(t) to a unit step input is
1 1 2t 1 2t 1 t 1
(a) e (b) 1 e e
2 2 2 2 (a) When u1 is the only input and y1 is the only
(c) e–2t – e–t (d) 1 – e–t output.
[EE-2013 : 2 Marks] (b) When u2 is the only input and y1 is the only
output.
Q.26 The state transition matrix for the system (c) When u1 is the only input and y2 is the only
x1 1 0 x1 1 output.
= + u is (d) When u2 is the only input and y2 is the only
x2 1 1 x2 1
output.
[EE-2015 : 2 Marks]
146 Electronics Engineering Control Systems

Q.29 Consider a linear time invariant system x = Ax , Q.32 Consider the system described by the following
state space representation:
with initial conditions x(0) at t = 0. Suppose
and are eigen vectors of (2 × 2) matrix A x1 ( t ) 0 1 x1 ( t ) 0
= 0 + u(t )
corresponding to distinct eigen values 1 and 2 x2 (t ) 2 x2 (t ) 1
respectively. Then the response x(t) of the system
x1 ( t )
due to initial condition x(0) = is y(t) = [1 0]
x2 (t )
(a) e 1t (b) e 2t
x1 ( t ) 1
(c) e 2t (d) e 1t + e 2t If u(t) is a unit step input and = ,
x2 (t ) 0
[EE-2016 : 2 Marks]
value of output y(t) at t = 1 sec. (Rounded off to
Q.30 The second order dynamic system, 3 decimal places), is _______ .
dX [EE-2017 : 2 Marks]
= PX + Qu , y = RX
dt
Q.33 Consider a state-variable model of a system:
has the matrices P, Q and R as follows:
xˆ 1 0 1 x1 0
1 1 0 = + r
P= ,Q = , R = [0 1] xˆ 2 2 x2
0 3 1
x1
The system has the following controllability and y = [1 0]
x2
observability properties:
where ‘y’ is the output, and ‘r’ is the input. The
(a) controllable and observable
damping ratio and the undamped natural
(b) not controllable and observable
frequency n (rad/sec) of the system are given by
(c) controllable but not observable
(d) not controllable and not observable (a) = , n =
[EE-2017 : 2 Marks]

Q.31 The transfer function of the system Y(s)/U(s) (b) = , n =


whose state-space equations are given below is
x1 ( t ) 1 2 x1 ( t ) 1 (c) = , =
= 2 0 + u(t ) n
x2 (t ) x2 (t ) 2

x1 ( t ) (d) = , n =
y(t) = [1 0]
x2 (t ) [EE-2017 : 2 Marks]

(s + 2) (s 2)
(a) 2 (b) 2
(s 2s 2) (s + s 4)

(s 4) (s + 4)
(c) 2 (d) 2
(s + s 4) (s s 4)
[EE-2017 : 2 Marks]
GATE Previous Years Solved Paper 147

Electronics & Electrical Engineering


GATE Previous Years Solved Paper

A n swe rs & Expl a n a t i o n s

Answers
EC State Space Analysis

1. (Sol.) 2. (b) 3. (b, c) 4. (a) 5. (a) 6. (a) 7. (d) 8. (c)

9. (d) 10. (b) 11. (b) 12. (a) 13. (a) 14. (c) 15. (d) 16. (d)

17. (d) 18. (c) 19. (b) 20. (c) 21. (c) 22. (d) 23. (a) 24. (a)

25. (b) 26. (c) 27. (a) 28. (b) 29. (d) 30. (d) 31. (a) 32. (a)

33. (5) 34. (d) 35. (a) 36. (c) 37. (a) 38. (b)

Solutions
EC State Space Analysis

1. Sol. 0
B=
2 0 1
A
0 4
2 4 0 4
AB = =
s+2 0 +2 1 1 1
sI – A =
0 s+4
0 4
[sI – A]–1 = Qc =
1 1
1 s+4 0
( s + 2) ( s + 4) 0 s + 2 Qc = 0 – 4 = 4 0
1 Hence the given system is controllable.
0
s+2 Characteristic equation,
[sI – A]–1 =
1 [sI – A] = 0
0
s+4
s+2 4
State transition matrix, =0
2 s+1
(t) = eAt = L –1{[sI – A]–1}
(s + 2) (s + 1) – 8 = 0
2t
At e 0
(t) = e = 3 ± 9 + 24
4t s=
0 e
2
= –4.37, 1.37
2. (b)
Since 1 pole lie in RHS of s-plane.
x1 2 4 x1 0 So, the given system is unstable.
= + u(t )
x2 +2 1 x2 1
Qc = [B AB]
148 Electronics Engineering Control Systems

3. (b, c) 5. (a)
1 0 Qc = [B AB A2B ... An – 1 B]
A= 0 1
0 2
A=
2 3
0
B= 0
1
B= 1
1 0 0 0
AB = = C = [1 1]
0 2 1 2
Qc = [B AB] +1
AB =
3
0 0
Qc = 0 1
1 2
Qc = 0
1 3 2×2
Qc = 0 – 0 = 2
So, the given system is not completely Order 2, rank 2
controllable, Controllable,
2
1 0 Q0 = [CT AT CT AT CT ... ...]
AT = 0 2 1 0 2
CT = , AT =
C = [1 2] 1 1 3

1 1 2
CT = Q0 = 0
2 1 2
1 0 1 1 rank 2 observable.
AT CT = =
0 2 2 4
6. (a)
Qo = [CT ATCT]
0 1 0 0
1 1
Qo = x = 0 0 1 x + 0 [ 0.5 3 5] x + v
2 4
0.5 1 2 1
Qo = –4 – (–2) = –2 0
0 1 0
So, the given system is completely observable.
x = 0 0 1 x+v
(b) The system is not completely controllable.
0 2 3
(c) The system is completely observable.
Characteristic equation
4. (a) = 3+3 2+2 +0=0
x1 = x1 – x2 = 0, –1, –2

x1 (0) = x1(0) – x2(0) 7. (d)


x1 (0) = 1 – (–1) = 2 x(t ) = –2x(t) + 2u(t) ...(i)
x2 = x 2 y(t) = 0.5x(t) ...(ii)
x2 (0) = x2(0) = –1 From equation (i), taking Laplace transform of
y = x1 + x2 equation (i),
dy sX(s) = –2X(s) + 2U(s)
= x1 + x 2 X(s) [s + 2] = 2U(s)
dt
dy 2 U (s)
= x1 (0) + x2 (0) = 2 1 = 1 X(s) =
dt t = 0 (s + 2)
GATE Previous Years Solved Paper 149

Taking Laplace transform of equation (ii), 3


Y(s) = 0.5 X(s) ATCT =
1
0.5 × 2 U ( s )
Y(s) = 1 3
s+2 Qo = 0
0 1
Y (s) 1
= Observable.
U (s) (s + 2)
10. (b)
8. (c)
s 0 1 0
s 0 1 0 [sI – A] =
(sI – A) = 0 s 0 1
0 s 1 1

s 1 0 s 1 0
= [sI – A] =
1 s 1 0 s 1
eAt = [sI – A]–1
(s 1) 0
+1 (s 1) 1
0
(sI – A)–1 = s 1 et 0
(s 1)2 = =
1 0 et
0
1 s 1
0
s 1
= +1 1 11. (b)
2 s 1
(s 1) Eigen values of A = [ ]
Eigen values of W = [µ]
L –1[sI – A]–1 = e At [ x(0)]
The eigen values of a system are always unique.
et 0 1 et So, [ ] = [µ]
= =
tet et 0 tet But a system can be represented by different state
models having different set of state variables,
9. (d) X=W
x1 3 1 x1 1 X W
= + u
x2 2 0 x2 0 Both are possible conditions.
x1 1 12. (a)
y = [1 0] + u
x2 0 (t) = L –1[sI – A]–1
Qc = [B AB] 1 1
1 s 0 0 1 1 s 1
= L =L
3 0 s 1 0 1 s
AB =
2
s 1
1 3 2 2
0 1 s +1 s +1
Qc =
0 2 = L
1 s
Controllable s2 + 1 s2 + 1
Qo = [CT ATCT]
cos t sin t
1 T 3 2 =
CT = , A = sin t cos t
0 1 0
150 Electronics Engineering Control Systems

13. (a) By solving this we get,

d x1 0 x1 0 1
d u1
dt 1 1 0 x2 = 0 x2 + 0 0
= + u dt u2
dia 1 10 ia 10 x3 0 x3 1 0
dt
d 18. (c)
= – + ia ...(i)
dt 1 0 p
A= ; B=
dia 0 1 q
= – – 10ia + 10 u ...(ii)
dt For controllability condition is,
Taking Laplace transform (i) and (ii), Qc = [B, AB, ... An – 1 B] 0
s (s) = – (s) + Ia(s)
1 0 p p+0 p
(s + 1) (s) = Ia(s) ...(iii) AB = = =
0 1 q 0+q q
sIa(s) = – (s) – 10 Ia(s) + 10 U(s)
(s) = (–10 – s) Ia(s) + 10 U(s) p p
So, Qc = =0
= (–10 – s) (s + 1) (s) + 10 U(s) q q
= –[s2 + 11s + 10] (s) + 10 U(s) So the system is uncontrollable for all values of
2
–[s + 11s + 11] (s) = 10 U(s) p and q.
(s) 10
= 2 20. (c)
U (s) ( s + 11 s + 11)
Forward path gain,
14. (c) 1 1 1
P1 = 2 (0.5) = 2
Sum of the eigen value = Trace of the principle s s s
diagonal matrix, 1 1
P2 = 2 (1) (0.5) =
Sum = –3 s s
Only option (a) satisfies both conditions. =1
1

2 =1
15. (d)
1 1 1 1
Multiplication of the eigen value = determinant = 1 = 1+ + 2
s s 2 s s
of the matrix.
From options it seems determinant should Transfer function of the system,
be ±2. Only option (d) satisfies as det = 2. Y (s) P1 1 + P2 2
=
U (s)
16. (d)
11
– +
s s 2s+1
= =
1 1 s2 + s + 1
1+ + 2
1 x3 1/s s s
u1 x3
– 21. (c)
x2 x2 Y1 ( s) 1
1/s =
U(s) s+2
Y2 (s ) 2
x1 =
u2 x1 U(s ) s+2
1 1/s
Y1 (s) X1 (s) 1
=
X1 ( s ) U ( s ) s+2

GATE Previous Years Solved Paper 151

Y2 (s ) X2 (s) 2 a1 a2
=
X2 (s) U (s) s+2 A2B = 0
X1 ( s ) 1 Y (s) 0
= and 1 =1
U(s) s+2 X1 ( s )
0 0 a1 a2
X2 (s ) 1 Y (s) Qc = 0 a2
= and 2 =2 0
U (s) s+2 U (s) 1 0 0
sX1(s) + 2X1(s) = U(s)
For system to be controllable,
and Y1(s) = X1(s)
sX2(s) + 2X2(s) = U(s) Qc 0
and Y2(s) = 2X2(s) (0 a1 a22 ) 0
x1 (t ) + 2 x1 (t ) = U(t) a1 0
and y1(t) = x1(t) a2 0

x2 (t ) + 2 x2 (t ) = U(t) 23. (a)


and y2(t) = 2x1(t) 1 x1 –1 x1 1 x2 –1 x2 1
From question, u y

T T 1
y = [ y 1 y 2 ] = [1 2] = 1/s 1/s
2
So, x1 = –x1 – u
x1 (t ) = –2x1(t) + u(t)
x2 = ( x 2 + x1 ) = ( x 2 x1 u)
x2 (t ) = –2x2(t) + u(t) x2 = x1 – x2 + u
x1 2 0 x1 1 y = x2
= + u(t )
x2 2 0 x2 1 y = x1 – x2 + u
or, x = [–2] x + [1] u x1 1 0 x1 1
= + u
Only option (b) is satisfied. x2 1 1 x2 1
1 0 1
22. (d) x = X+ u
1 1 1
x1 0 a1 0 x1 0
x2 = 0 0 a2 x2 + 0 u 24. (a)
x3 a3 0 0 x3 1 1 0
A=
1 1
x1
y = [1 0 0] x2 s+1 0
sI – A =
x3 1 s+1

Qc = [B AB A2B] ..(i) 1
0
s+1
0 a1 0 0 0 [sI – A]–1 = 1 1
AB = 0 0 a2 0 = a2 2 s+1
(s + 1)
a3 0 0 1 0
(t) = eAt = L –1{(sI – A)–1}
0 a1 0 0 0 e t
0
A2B = AAB = 0 0 a2 0 = a2 eAt = t t
te e
a3 0 0 1 0
152 Electronics Engineering Control Systems

25. (b) sX2(s) – X2(0) = –2X2(s)


(s + 2) X2(s) = X2(0) X2(0) = –1
x1 1 1 0 x1 0
1
x2 = 0 1 0 x2 + 4 u X2(s) =
s+2
x3 0 0 2 x3 0
x2(t) = –e–2t
x1
27. (a)
y = [1 1 1] x2
The state equation from the given state diagram
x3
is,
1 1 0 0 x1 = x 2
A= 0 1 0 , B = 4 , C = [1 1 1]
x2 = x 3
0 0 2 0
x 3 = a3 x 3 + a2 x 2 + a1 x 1 + u
Check for controllability,
Also, y = c1x1 + c2x2 + c3x3
Qc = [B : AB : A2B]
Thus state matrix,
0 4 8
x1 0 1 0 x1 0
= 4 4 4
x2 = 0 0 1 x2 + 0 u
0 0 0
x3 a1 a2 a3 x3 1
For controllable,
Qc 0 x1
and [ c c
y = 1 2 3c ] x 2
Here, Qc = 4(0) = 0 ( Uncontrollable) x3
Check for observability,
Check for controllabiltiy:
Qo = [CT : ATCT A2T CT]
The system is said to be controllable if, the rank
1 1 1 of controllability matrix Qc is equal to the rank
= 1 0 1 of the state matrix A. However, if the
1 2 4 controllability matrix Qc is a square matrix is
For observable, then the condition for controllability is,

Qo 1 Qc 0
where, Qc = [B : AB : A2B]
Here, Qo = 1 ( Unobservable)
0 0 1
26. (c)
Qc = 0 1 a3
x1 1 0 x1 1 a3 a2 + a32
=
x2 0 2 x2
Q Qc 0 and rank of Qc = Rank of A = 3
x1 = –x 1 ...(i)
The system is controllable.
x2 = –2x 2 ...(ii)
The system is said to be observable if, the rank
Applying Laplace transform on both equations. of observability matrix Qo is equal to the rank of
sX1(s) – x1(0) = –X1(s) the state matrix A. However, if the observability
X1(s) [s + 1] = X1(0) [Given, x1(0) = 1] matrix Qo is a square matrix then the condition
1 for observability is,
X1(s) =
s+1 Qo 0
e–t = x1(t)
GATE Previous Years Solved Paper 153

where, Qo = [CT : ATCT : (AT)2 CT] 29. (d)


c1 c2 c3
Qo = c 3 a1 c 1 + c 2 a2 c 2 + c 2 a3
x1 0 1 x1
Given, =
c1 (c 2 + c3 a3 ) c 3 a1 + (c 2 + c3 a3 ) a2 c 1 + c2 a2 + (c 2 + c 3 a3 ) a3 x2 0 0 x2
Q Qo depends on value of unknown. 0 1
A=
Hence, not observability always. 0 0
[sI – A] =
28. (b)
s 0 0 1 s 1
Given, x(t ) = Ax(t), x(0) = x0 =
0 s 0 0 0 s
Taking the Laplace transform, [sI – A] = s2
sX(s) – x(0) = AX(s) (t) = L –1[sI – A]–1
[sI – A] X(s) = x(0)
1 s 1
X(s) = [sI – A]–1 x(0) =
s2 0 s
x(t) = L –1 [sI – A]–1 x(0) ...(i)
Conditions given are: 1 1
t 1 s s2 = 1 t
1 e = L
For, x0 = ; x(t ) = 1 0 1
1 t 0
e s

t 2t
0 e e 30. (d)
For, x0 = ; x(t ) =
1 e t
+ 2e 2t The response of the system is given by
x(t) = L –1[(sI – A)–1 x(0) + (sI – A)–1 B U(s)]
Using the linearity property in equation (i),
and the complete response,
K1 x1(t) = L –1 [sI – A]–1 x1(0) K1
y(t) = [C] x(t)
K2 x2(t) = L –1 [sI – A]–1 x2(0) K2
From the given state model,
Using the linearity property as,
K1 x1(t) + K2 x2(t) = L –1 [sI – A]–1 0 1
A= ; B = 0; C = [0 1]
= [K1 x1(0) + K2 x2(0)] ...(ii) 0 1
Also, X3(s) = [sI – A]–1 x3(0) 1
and x(0) =
1 0 3 0
So, K1 + K2 =
1 1 5 1
s 1 1 s+1 1
K1 3 (sI – A)–1 = =
0 s+1 s ( s + 1) 0 s
=
K1 + K 2 5
1 s+1 1 1
K1 = 3 (sI – A)–1 x(0) =
K2 = 8 s (s + 1) 0 s 0
So from equation (ii), we get x(t),
1 s+1 1 /s
x(t) = K1x1(t) + K2 x2(t) = =
s (s + 1) 0 0
t t 2t
e e e x(t) = L –1[(sI – A)–1 x(0)]
= 3 +8
t t 2t
e e + 2e 1
= u(t )
0
t 2t
11 e 8e and the complete response,
= t 2t
11 e + 16 e 1
y(t) = [0 1] x(t ) = [0 1] =0
0
154 Electronics Engineering Control Systems

31. (a) Qc 0 Hence system is controllable.


State matrix form:
For observability,
2 1 3
x = x+ u C 1 0
0 4 1 Qo = =
CA 2 0
y = [3 –2] x
The transfer function from state matrix can be Qo = 0 Hence system is not observable.
calculated as,
T(s) = C[sI – A]–1 B + D 33. Sol.
Here, x1 ( t ) 0 0 x1 ( t ) 0
= + u(t )
s 2 1 1 x2 (t ) 0 9 x2 (t ) 45
[sI – A]–1 =
0 s+4 By applying Laplace transform on both sides,
( s + 4) 1 we get,
1
= sX1(s) – x1(0) = 0
( s 2) (s + 4) 0 (s 2)
C[sI – A]–1 B x1 (0)
X1(s) = =0 [Q x1(0) = 0]
s
1 (s + 4) 1 3
= [3 2] So, x1(t) = 0
(s 2) ( s + 4) 0 (s 2) 1
45
1 3s + 12 + 1 and sX2(s) – x2(0) = 9 X2 (s) +
= [3 2] s
( s 2) ( s + 4) s+2
45
1 X2(s) = [Q x2(0) = 0]
= [9s + 39 + 2s 4] s ( s + 9)
(s 2) (s + 4)
11 s + 35 Required value = lim x12 (t ) + x22 (t )
= t
(s 2) (s + 4)

32. (a) = lim x2 (t ) [Q x1(t) = 0]


t

x1 (t ) + 2 x1 (t ) = 3u(t) 45
lim x2 (t ) = lim sX2 (s ) = =5
t s 0 9
x2 (t ) + x2 (t ) = u(t)
So, Required value = 5 = 5
x1 (t ) = –2x1(t) + 3u(t)

x2 (t ) = –x2(t) + u(t) 34. (d)


Given state variable equations are as follows:
x1 2 0 x1 3
= + U d
x2 0 1 x2 1 x 1 (t ) x 2 ( t ) = 0 ...(i)
dt
c(t) = x1(t)
d
x1 x2 (t ) + 2 x1 (t ) + 3x2 (t ) = r(t) ...(ii)
= [1 0] dt
x2
Also given that,
2 0 3 Input = r(t) and input = x1(t)
A= ;B=
0 1 1 By applying Laplace transform to equation (i),
C = [1 0] we get,
For controllability, sX1(s) = X2(s) ...(iii)
3 6 By applying Laplace transform to equation (ii),
Qc = [ B AB] = we get,
1 1
GATE Previous Years Solved Paper 155

sX2(s) + 2X1(s) + 3X2(s) = R(s) Y (s) Y (s ) X 1 (s )


= ×
R(s ) 2 X1 (s ) U (s) X1 ( s ) U ( s )
X2(s) = ...(iv)
s+3 1
By substituting equation (iv) in equation (iii), = 1× 3 2
s + 3s + 2 s + 1
we get,
X1(s) [s3 + 3s2 + 2s + 1] = U(s)
R(s ) 2 X1 (s )
sX1(s) = x 2 = x1 (t )
s+3
(s2 + 3s + 2) X1(s) = R(s) X2(s) = sX1(s)
So, the transfer function of the given system is, x 3 = x2 (t )
X1 ( s ) 1 X3(s) = sX2(s) = s2X1(s)
= 2
R(s ) s + 3s + 2 So, sX3(s) = –X1(s) – 2X2(s) – 3X3(s) + U(s)
2 =2
n x3 (t ) = –x1(t) – 2x2(t) – 3x3(t) + u(t)
2 n =3 y(t) = x1(t)
3 3
= = = 1.06 0 1 0 0
2 n 2 2
x(t ) = 0 0 1 x(t ) + 0 u(t )
As > 1, the given system is overdamped.
1 2 3 1
35. (a) y(t) = [1 0 0] x(t)
Transfer function,
37. (a)
Y (s)
T(s) = = C [sI A] 1 B From source transformation,
U (s)
4 1.5 2 2 i 0.5 H (a ) 1
A= ; B=
4 0 0
C = [1.5 0.625] +
2i1 V 0.25 F i2
s + 4 1.5 Loop 1 –
[sI – A] =
4 s

1 s 1.5
[sI – A]–1 = KVL in loop 1,
(s + 4s + 6) 4 s + 4
2

2s di
1 2i 1 = 2i + 0.5 +V
[sI – A]–1 B = dt
s 2 + 4s + 6 8
di
1 2s = –2V – 4i + 4i1 ...(i)
C[sI – A]–1 B = [1.5 0.625] dt
2 8
s + 4s + 6 KCL at node (a),
3s + 5
T(s) = dV V i2
s 2 + 4s + 6 i = 0.25 +
dt 1
36. (c) dv
= –4V + 4i + 4i2 ...(ii)
X(t) = Ax(t) + Bu(t) dt
y(t) = Cx(t) d v 4 4 v 0 4 i1
= +
0 dt i 2 4 i 4 0 i2
B= 0
1
156 Electronics Engineering Control Systems

38. (b) dVC (t )


= iS(t) – VC(t)
iS(t) = iL(t) + iR(t) dt
V (t ) VC(t) = iS(t) – VC(t) ...(2)
= iL (t ) + R
1 Vo(t) = VC(t) ...(3)
V (t ) iL (t ) 1 0 iL ( t ) 1
iS(t) = iL (t ) + L = + i (t )
1 VC (t ) 0 1 VC (t ) 1 S
But, VL(t) = VR(t) = 1iR(t)
iL ( t )
V (t ) di (t ) di (t ) [Volt] = [0 1]
iR(t) = L = L L = L VC (t )
1 dt dt
Controllability:
diL (t )
iS(t) = iL (t ) + 1 1
dt QC = [ B AB] =
1 1
diL
= iS(t) – iL(t)
dt QC = 0 Not controllable
iL(t) = iS(t) – iL(t) ...(1) Observability :
iS(t) = iC(t) + iR(t) C 0 1
Q0 = =
dV (t ) CA 0 1
iS(t) = C C + iR (t )
dt
Q0 = 0 Not observable
dVC (t ) VR (t )
iS(t) = +
dt 1 Neither controllable nor observable.
dVC (t ) VC (t )
iS(t) = +
dt 1

Answers
EE State Space Analysis

1. (a) 2. (b) 3. (c) 4. (a) 5. (a) 6. (–1, –2, –3) 7. (a) 8. (c)
9. (b) 10. (b) 11. (a) 12. (a) 13. (a) 14. (a) 15. (c) 16. (b)
17. (a) 18. (d) 19. (a) 20. (c) 21. (b) 22. (c) 23. (d) 24. (a)
25. (a) 26. (c) 27. (a) 28. (b) 29. (a) 30. (c) 31. (d) 32. (1.284)
33. (a)

Solutions
EE State Space Analysis

1. (a) e ae + bf
[B] = , [ AB] =
T.F. = C[sI – A]–1 B + D f ce + df

2. (b) Since, x1(t) = x2(t) and x1 (t ) = x2 (t )

x1 a b x1 ( t ) e e = f and a + b = c + d
Let, = + u(t ) e ae + bf
x2 c d x2 (t ) f
QC = B AB = =0
f ce + df
Uncontrollable.
GATE Previous Years Solved Paper 157

3. (c) eAt = L –1 [sI – A]–1


3 e 3t e 2t e 3t
T.F. = =
s+1
0 e 2t
Pole = –1
Eigen value of ‘A’ = –1 x(t) = eAt X(0)
Option (c) is correct. e 3t
e 2t
e 3t
10
=
4. (a) 0 e 2t 10
Characteristic equation:
20 e 3t 10 e 2 t
sI A =0 x(t) =
10 e 2 t
s a 1
=0 0
a s 1
Xss = lim x(t ) =
(s – a) (s – 1) – a = 0 t 0
eigen avlues,
8. (c)
s = 0, (a + 1)
eAT = L –1[sI – A]–1
5. (a)
9. (b)
T.F. = C[sI – A]–1 B + D
1 t
s+4 1 1 AT A( t )
= [1 0] +0 x(t) = e X(0) + e BU ( ) d
3 s+1 1 0

s+1 1 1 Q X(0) = 0 (Given)


[1 0]
3 s+4 1
=
t
e 2(t )
0 1
(s + 1) (s + 4) + 3 x(t) = 0 + (t ) dt
0 e 4( t ) 1
0
1
[s + 1 1]
1 s
1
e 2(t )
e2t
= = = (t ) dt =
s 2 + 5s + 4 + 3 s 2 + 5s + 7 0 e 4(t )
e 4t

6. Sol. Output y(t) = [4 0] x(t)

s 1 0 e 2t
= [4 0]
sI A = 0 s 1 =0 e 4t
6 11 s + 6 Hence, y(t) = 4e2t
s3 + 6s2 + 11s + 6 = 0
10. (b)
eigen values are s = –1, –2, –3.
1 2
7. (a) QC = =0
0 0
1
s+3 1 Uncontrollable
[sI – A]–1 =
0 s+2 Characteristic equation:

1 1 sI A =0
s+3 (s + 2) (s + 3)
= s 2 3
1 =0
0 0 s 5
(s + 2)
s2 – 7s + 13 = 0
158 Electronics Engineering Control Systems

eigen values, s = 3.5 ± j0.866 13. (a)


i.e. roots lies on right side of s-plane.
0 2
unstable. System a matrix = A =
2 0
11. (a) s 0 0 2 s 2
sI – A = =
State transition matrix, 0 s 2 0 2 s

e 2t
0 Characteristic equation sI A =0
(t) = t
0 e s 2
2 s =0
Initial conditions,
2 s2 – 4 = 0
x(0) = Roots of the characteristic equation are
3
–2 and +2.
Zero input response is given by
x(t) = (t) x(0) 14. (a)

e 2t
0 2 2e 2t 0 1 1
= x(t ) = x (t ) + V (t ) ...(i)
t 3 t 0 3 0
0 e 3e
State of the system at t = 1 s, x(t ) = Ax(t) + Bu(t) ...(ii)
Comparing equation (i) and (ii), we get
2e 2 0.271
x(t ) t = 1 = = A = system matrix
3e 1 1.100
0 1 1
= and B =
0 3 0
12. (a)
d s 0 0 1 s 1
Let, x1 = and x2 = [sI – A] = =
dt 0 s 0 3 0 s+3

x2 = x 1 s+3 1
0 s
d2 Bd k2 k [sI – A]–1 =
+ + = Va s(s + 3) 0 x ( 1)
dt 2 J dt LJ LJ
1 1
d2 Bd k2 k s s(s + 3)
= + Va
dt 2 J dt LJ = 1
0
2 s+3
B k k
x1 = x1 x2 + Va
J LJ LJ 1 1 1 1
x2 = x 1 s 3 s s+3
[sI – A]–1 =
1
0
x1 x1 s+3
= P + QVa
x2 x2 State transition matrix
where, = L –1 [(sI – A)–1]

B k2 k
1
1
(1 e 3t
)
P= J LJ and Q = LJ (t) = 3
3t
1 0 0 0 e
GATE Previous Years Solved Paper 159

15. (c) Replacing equation (i),


ZIR (Zero Input Response) = (t) × X(0) x3 + 4 x3 2 x2 + x1 = u(t)

1 3t x3 = x1 + 2 x 2 4x3 + u(t )
1 (1 e ) 1
= 3
3t 3 x1 = x 2
0 e
x2 = x 3
3t 3t
1+1 e e
= = x3 = x1 + 2 x 2 4x3 + u(t )
3t 3t
3e 3e
ZSR (Zero State Response) x1 0 1 0 x1 0
= L –1[(sI – A)–1 BU(s)] x2 = 0 0 1 x2 + 0 u(t )
x3 1 2 4 x3 1
1 1
1 s s(s + 3) 1 1
= L 0 1 0
1 0 s
0 So, A = 0 0 1
s+3
1 2 4
2 t
1 1/ s
= L = 17. (a)
0 0
Any state equation represents the dynamical
State transition equation,
behaviour of the given network. State equations
e 3t t usually follow a sepcific ‘format’ while being
= ZIR + ZSR = +
3 e 3t 0 represented. On the left side of each state
equation, the derivative of only one variable is
t e 3t used. On the right hand side a mathematical
=
3 e 3t function is represented involving any or all the
state variables and the sources.
16. (b)
3 0.2 H I1 + I2 5 I2
Y (s) 3(s 2) + –
H(s) = = 3 + V – I1
U ( s ) s + 4s 2 2 s + 1 x

V –
Y ( s ) X1 ( s ) 1 I 0.5 H II + 0.2Vx
= 3(s 2) 3 2
X1 ( s ) U ( s ) s + 4s 2 s + 1
X1 ( s ) 1
Let, = 3 2
U(s) s + 4s 2 s + 1 Using KVL in Loop-I,
3 2
s x1(s) + 4s x1(s) – 2sx1(s) + x1(s) = u(s) dI 1
V 3( I1 + I 2 ) Vx 0.5 =0 ...(i)
d dt
Replacing ‘s’ by ,
dt Using KVL in Loop-II,
3 2
d x1 d x dx1 dI1
2
+ 4 21 2 + x1 = u(t ) ...(i) 0.5 5 I 2 + 0.2Vx = 0 ...(ii)
dt dt dt dt
dx1 Eliminating I2 from equation (i) and (ii), we get
Let, = x 2 = x1
dt
dI1 5
= 1.4 Vx 3.75 I1 + V
d 2 x1 dt 4
= x2 = x3
dt
160 Electronics Engineering Control Systems

18. (d) x1 ( t ) 3 1 x 1 (t ) 2
= + u(t )
x = Ax + Bu and y = Cx x2 (t ) 0 2 x 2 (t ) 1
where, x = AX + BU
0 1 0 3 1 2
A= ,B= , C = [1 0] and D = 0 So, A= ,B=
0 2 1 0 2 1
s 0 0 1 s 1 y(t) = x1(t)
[sI – A] = =
0 s 0 2 0 s+2 x1 ( t )
y(t) = [1 0]
1 s+2 1 x2 (t )
[sI – A]–1 =
s (s + 2) 0 s y = CX + DU
Transfer function, So, C = [1 0] and D = 0
= C[sI – A]–1 B + D s 0 3 1
[sI – A] =
s+2 1 0 0 s 0 2
0 s 1 s+3 1
= [1 0]
s(s + 2) =
0 s+2
1
s+2 1
s
1
= [1 0] = 0 s+3
s(s + 2) s(s + 2) [sI – A]–1 =
(s + 2) (s + 3)

19. (a) T.F. = C[sI – A]–1 B + D

1 s+2 1
G(s) =
s(s + 2) 0 s+3 2
= [1 0] +0
and H(s) = 1 (s + 2) ( s + 3) 1
r(t) = u(t)
2(s + 2) + 1
1
R(s) = s+3
s = [1 0]
(s + 2) ( s + 3)
R(s) 1/ s
Error = E(s) = = 2s + 5 2s + 5
1 + G(s) H (s) 1 + 1 = =
s(s + 2) (s + 2) (s + 3) s2 + 5s + 6
s+2
E(s) = 21. (b)
s(s + 2) + 1
Steady-state error, using final value theorem, s+2 1
0 s+3
ess = lim sE(s) [sI – A]–1 =
s 0 (s + 2) (s + 3)
s(s + 2)
= lim =0 1 1
s 0 s(s + 2) + 1
s+3 (s + 2) (s + 3)
=
20. (c) 1
0
Selecting x1(t) and x2(t) as state variables, s+2
dx1 (t ) 1 1 1
x1 (t ) = = 3x1 (t ) + x2 (t ) + 2 u(t )
dt s+3 s+2 s+3
dx2 (t ) =
1
x2 = = 2 x2 (t ) + u(t ) 0
dt s+2
GATE Previous Years Solved Paper 161

State transition matrix For system to be controllable, the matrix Qc must


= L –1 [(sI – A)–1] be non-singualr,
1 1 1 Qc = B AB A2 B
1 s + 3 s+2 s+3
= L
AB = [ 0 a2 0]
1 T
0
s+2
0 0 a1 a2
e 3t e 2t e 3t a2 a3 0 0
= A2 =
0 e 2t 0 a1 a3 0

22. (c) A2B = [a1a2 0 0]2


0 0 a1 a2
1 2 0
A= and B = Qc = 0 a2 0
0 2 1
1 0 0
s 0 1 2
[sI – A] =
0 s 0 2 Qc = a1 a22
For Qc to be non-singular,
s+1 2
= 0 s 2 Qc 0
a1 0 and a2 0 and a3 R
s 2 2
0 s+1 24. (a)
[sI – A]–1 = ...(i)
(s + 1) (s 2)
2 0 1
Transfer function = C[sI – A]–1 B A= , B=
0 1 1
So denominator of equation (i) gives poles of
2 0 1 2
the system, AB = =
0 1 1 1
(s + 1) (s – 2) = 0
s = –1 and 2 For controllabilty,
One pole lies in RHS of s-plane. B : AB 0
Hence, the so system is unstable.
1 2
For controllability, Qc is defined as, or, = –1 – (–2) = 1
1 1
Qc = [B : AB]
0
1 2 0 2
AB = = The system is controllable,
0 2 1 2
1
Qc = [B : AB] CT =
0
0 2
= 1 2 2 0 1 2
ATCT = =
0 1 0 0
Qc 0 For observability,
Hence the system is controllabe. CT : AT C T 0
23. (d) 1 2
or, 0 0 =0
0 a1 0 0
A= 0 0 a2 , B = 0 , C = [1 0 0]T The system is not observable.
a3 0 0 1
162 Electronics Engineering Control Systems

25. (a)
et 0
s 0 2 0 L –1[sI – A]–1 = t
[sI – A] = te et
0 s 0 1
et 0
s+2 0 State transition matrix =
= t et et
0 s+1

1 27. (a)
0
s+2
[sI – A]–1 = 0 1 0
1 , B= , C = [1 0]
0 Given, A=
s+1 1 1 1

1 1 Transfer function of the given system is given


0 by
s+2 1 s+2
[sI – A]–1 [B] = =
1 1 1 Y (s)
0
s+1 s+1 = C(sI – A)–1 B
U (s)
1 s 0 0 1
s+2 1 Now, (sI – A) =
C[sI – A]–1 [B] = [ 1 0] = 0 s 1 1
1 s+2
s+1 s 1
=
1 1 s+1
G(s) =
s+2 s+1 1
1
Y (s ) 1 (sI – A)–1 = 2
(s + s + 1) 1 s
=
X( s ) s+2
Transfer function,
1 1 1 1 s+1 1 0
Y(s) = = Y (s) 1
s (s + 2) 2 s s+2 = 2
[1 0]
U (s) s +s+1 1 s 1
1 2t 1 1 2t
y(t) = (1 e ) = e 1 0
2 2 2 = [(s + 1) 1]
2 1
s +s+1
26. (c)
1 1
= ×1 =
x1 1 0 x1 1 2
(s + s + 1) 2
(s + s + 1)
Given, = 1 1 + u
x2 x2 1
Y (s) 1
= 2
or, x(t ) = Ax(t) + Bu U (s) s +s+1
1 0 1 Given, Input = unit step
Here, [A] = , B= 1
1 1 1
U(s) =
State transition matrix is given by, s
Y(s) =
eAt = L –1[sI – A]–1
1 1 1
s 1 0 × =
Now, [sI – A] = s s 2 + s + 1 s(s 2 + s + 1)
1 s 1
Final value = lim {sY (s)}
1 s 1 0 s 0
[sI – A]–1 = 2
(s 1) 1 s 1
1
= lim =1
1 s 0 s2 + s + 1
0
( s 1) Error = Final value – Initial value
= 1 1 ess = 0
(s 1)2 ( s 1)
GATE Previous Years Solved Paper 163

28. (b) 30. (c)


Equations from flow diagram, Given, x(t ) = Px + Qu
x1 = 5x1 – 2x2 + u1, y1 = x1 y = Rx
x2 = 2x1 + x2 + u1 + u2, y2 = x1 – x2 1 1 0
P= ,Q = , R = [0 1]
0 3 1
5 2 1 0 1 0
x3 = 2 x+ u; y = x For controllability,
1 1 1 1 1
A B C Qc = [Q PQ]
Considering the SISO cases: 1 1 0 1
[PQ] = =
1. u1 (Input) and y1 (Output): 0 3 1 3

5 2 1 0 1
A ; B and C = [1 0] Qc =
2 1 1 1 3

Qc = –1 0 ...(Controllable)
C 1 0
O = Also, for observability,
CA 5 2
Q0 = [RT PTRT]
Rank (O) = 2 observable
1 0 0 0
1 3 [PT RT] = =
C [ B AB] = 1 3 1 3
1 3
0 0
Rank (C) 2 non-controllable Q0 =
1 3
2. u2 (Input) and y1 (Output):
Q0 = (0 × –3 – 0 × 1) = 0
5 2 0 ...(Non-observable
A ; B and C [1 1]
2 1 1 Since, Qc 0 and Q0 = 0
System is controllable but not observable.
1 0
O Rank (O) = 2 observable
5 2 31. (d)

0 2 1 2 1
C Given, X = 2 0 X+ 2 u
1 1
and Y = [1 0] X
Rank (C) = 2 non-controllable
Transfer function = C[sI – A]–1 B + D
29. (a) s 2
2 2
x = Ax s s 4 s s 4
[sI – A]–1 =
Eigen values are 2 s 1
1 and 2.
We can write, s2 s 4 s2 s 4

e 1t 0 s+4
(t) = 2
2t
s s 4
0 e (sI – A)–1 × B = s 1
Response due to initial conditions, s2 s 4
x(t) = (t) x(0)
Y (s)
= C[sI – A]–1 × B
e 1t 0 U (s)
x(t) = = e 1t
0 e 2t 0 s+4
= 2
s s 4
164 Electronics Engineering Control Systems

32. Sol. 33. (a)


y(t) = [1 0] x(t) Characteristic equation is,
x(t) = L –1[(sI – A)–1] x(0) sI A =0
+ L –1[(sI – A)–1] BU(s)]
s 1
1 sI A =
L –1[(sI – A)–1] x(0) = s+2
0
= s2 + 2s + =0
2t
0.25 + 0.5t + 0.25 e 2
L –1[(sI – A)–1] BU(s)] = n =
2t
0.5 0.5 e
n =
2t
0.75 + 0.5t + 0.25 e
x(t) = 2 n =2
2t
0.5 0.5 e
=
y(t) = 0.75 + 0.5t + 0.25e–2t
y(1) = 1.284
GATE-2023
Electronics Engineering

Analog Electronics
Chapterwise & Topicwise

Contents
S.No. Topic Page No.

1. Operational Amplifiers ...................................................................................................................... 1-53

2. Diodes Applications ........................................................................................................................ 54-82

3. BJT Analysis ..................................................................................................................................... 83-123

4. FET and MOSFET Analysis .......................................................................................................... 124-141

5. Frequency Response Amplifiers ............................................................................................ 142-145

6. Feedback Amplifiers .................................................................................................................. 146-150

7. Oscillator Circuits ........................................................................................................................ 151-165


1 Operational Amplifiers

ELECTRO NICS EN GINEERIN G R2


(a) For Vi > 0, Vo = Vi
(GATE Previous Years Solved Papers) R1
(b) For Vi > 0, Vo = 0
Q.1 In figure shown below, if the CMRR of the
R2
operational amplifier is 60 dB, then the (c) For Vi < 0, Vo = Vi
R1
magnitude of the output voltage is
(d) For Vi < 0, Vo = 0
100 k
[EC-1989 : 2 Marks]
+ R R 1k
2V – Q.4 The op-amp of figure shown below has a very
– +
+ Vo
poor open-loop voltage gain of 45 but is
1k – otherwise ideal. The gain of the amplifier equals
R R 100 k
8k

2k
[EC-1987 : 2 Marks] –
Vout
+
Q.2 The op-amp shown in figure below is ideal, Vin

R = L /C . The phase angle between Vo and (a) 5 (b) 20


(c) 4 (d) 4.5
Vi, at = 1/ LC . [EC-1990 : 2 Marks]
L R
Q.5 The CMRR of the differential amplifier of the
R C figure shown below is equal to
Vi –
Vo 90 k
+
1k
V1 –
Vout
V2 +
(a) (b)
2 1k
3 100 k
(c) (d) 2
2
[EC-1988 : 2 Marks]
(a) (b) 0
Q.3 Refer to figure shown below:
(c) 1000 (d) 1800
R2
[EC-1990 : 2 Marks]
Vo
Q.6 If the input to the circuit of figure is a since wave
R1
the output will be

+ + –
Vi Input
+ Output

2 Electronics Engineering Analog Electronics

(a) a half wave rectified sine wave. Q.10 For the ideal op-amp circuit of figure. Determine
(b) a full wave rectified sine wave. the output voltage Vo .
(c) a triangular wave.
(d) a square wave. 100 99
[EC-1990 : 2 Marks]

Q.7 An op-amp has an offset voltage of 1 mV and is 10 +
0
ideal in all other respects. If this op-amp is used
4V 0 Vo
in the circuit shown in figure. The output voltage 10
will be (select the nearest value).
1k
[EC-1993 : 2 Marks]
1k
– Q.11 The frequency compensation is used in op-amps
Vo
+ to increase its ______ .
[EC-1994 : 1 Mark]

Q.12 In the given circuit figure, if the voltage inputs


(a) 1 mV (b) 1 V
V– and V+ are to be amplified by the same
(c) ±1 V (d) 0 V
amplification factor, the value of ‘R’ should be
[EC-1992 : 2 Marks]
22 k
Q.8 The circuit of figure uses on ideal op-amp for
small positive values of Vin, the circuit works as 10 k
V– –
Vo
V+ +
Va
R 15 k
Vin – R=? Ideal op-amp
Vo
+

[EC-1995 : 1 Mark]
(a) a half wave rectifier
Q.13 An op-amp is used as a zero-crossing detector.
(b) a differentiator
If maximum output available from the op-amp
(c) a logarithmic amplifier
is ±12 Vp-p and the slew rate of the op-amp is
(d) an exponential amplifier 12 V/µ sec then the maximum frequency of the
[EC-1992 : 2 Marks] input signal that can be applied without causing
Q.9 Assume that the operational amplifier in figure a reduction in the P – P output is
is ideal the current I through the 1 k resistor is [EC-1995 : 1 Mark]
______ . Q.14 The circuit shown in the figure is that of
2k
Vin –
Vo
1k +

+ 1k
2 mA
2k R1
R2

[EC-1992 : 2 Marks]
GATE Previous Years Solved Paper 3

(a) a non-inverting amplifiers (a) –1 V (b) 2 V


(b) an inverting amplifier (c) +1 V (d) +15 V
(c) an oscillator [EC-2000 : 1 Mark]
(d) a schmitt trigger
Q.19 If the op-amp in the figure, is ideal, then Vo is
[EC-1996 : 1 Mark]
C
Q.15 The output voltage Vo of the circuit shown in
V1 sin t C
the figure is

10 k Vo
+
V2 sin t C
5k

Vo
+
2V (a) Zero (b) (V1 – V2) sin t
(c) –(V1 + V2) sin t (d) (V1 + V2) sin t
100 k
[EC-2000 : 1 Mark]
10 k

Q.20 Assume that the op-amp of the figure is ideal. If


Vi is a triangular wave, then Vo will be
(a) –4 V (b) 6 V
R
(c) 5 V (d) –5.5 V
[EC-1997 : 2 Marks] C
Vi –
Q.16 One input terminal of high gain comparator Vo
+
circuit is connected to ground and a sinusoidal
voltage is applied to the other input. The output
of comparator will be (a) square wave (b) triangular wave
(a) a sinusoid (c) parabolic wave (d) sine wave
(b) a full rectified sinusoid
[EC-2000 : 1 Mark]
(c) a half rectified sinusoid
(d) a square wave Q.21 The most commonly used amplifier in sample
and hold circuits is
[EC-1998 : 1 Mark]
(a) a unity gain inverting amplifier.
Q.17 The first dominant pole encountered in the
(b) a unity gain non-inverting amplifier.
frequency response of a compensated op-amp
(c) an inverting amplifier with a gain of 10.
is approximately at
(d) an inverting amplifier with a gain of 100.
(a) 5 Hz (b) 10 kHz
[EC-2000 : 1 Mark]
(c) 1 MHz (d) 100 MHz
[EC-1999 : 1 Mark] Q.22 If the op-amp in the figure has an input offset
voltage of 5 mV and an open-loop voltage gain
Q.18 In the circuit of the figure, Vo is
of 10,000, then Vo will be
+15 V
+15 V

Vo –
+1 V + Vo
R +
–15 V
–15 V

R
4 Electronics Engineering Analog Electronics

(a) 0 V (b) 5 mV Q.26 A 741-type op-amp has a gain bandwidth


(c) +15 V or –15 V (d) +50 V or –50 V product of 1 MHz. A non-inverting amplifier
[EC-2000 : 2 Marks] using this op-amp and having a voltage gain of
20 dB will exhibit a –3 dB bandwidth of
Q.23 The ideal op-amp has the following
(a) 50 kHz (b) 100 kHz
characteristics:
(a) Ri = , A = , R0 = 0 1000 1000
(c) kHz (d) kHz
17 7.07
(b) Ri = 0, A = , R0 = 0
(c) Ri = , A = , R0 = [EC-2002 : 1 Mark]
(a) Ri = 0, A = , R0 = Q.27 An amplifier using an op-amp with a slew-rate
[EC-2001 : 1 Mark] SR = 1 V/µsec has a gain of 40 dB. If this
amplifier has to faithfully amplify sinusoidal
Q.24 The inverting op-amp shown in the figure has
signals from dc to 20 kHz without introducing
an open-loop gain of 100. The closed-loop gain
any slew-rate induced distortion, then the input
Vo/Vs is
signal level must not exceed.
R2 = 10 k (a) 795 mV (b) 395 mV
(c) 79.5 mV (d) 39.5 mV
R1 = 1 k

[EC-2002 : 2 Marks]
Vs Vi +
– + Vo Q.28 If the input to the ideal comparator shown in
the figure is a sinusoidal signal of 8 V (peak to
peak) without any DC component, then the
(a) –8 (b) –9 output of the comparator has a duty cycle of
(c) –10 (d) –11
Vref = 2 V –
[EC-2001 : 2 Marks]
Input + Output
Q.25 In the figure assume the op-amps to be ideal.
The output Vo of the circuit is, 1 1
(a) (b)
10 mH 2 3
10 µF

1 1
10 k (c) (d)
Vs – 100 6 12
1 –
Vs = 10 cos(100t) + 2 3
+
[EC-2003 : 1 Mark]
Vo

Q.29 If the differential voltage gain and the common


mode voltage gain of a differential amplifier are
(a) 10 cos(100t) 48 dB and 2 dB respectively, then its common
t mode rejection ratio is
(b) 10 cos(100 ) d
(a) 23 dB (b) 25 dB
0
(c) 46 dB (d) 50 dB
t
4 [EC-2003 : 1 Mark]
(c) 10 cos(100 ) d
0
Q.30 If the op-amp in the figure is ideal, the output
4 d
(d) 10 cos(100t ) voltage Vout will be equal to
dt
[EC-2001 : 2 Marks]
GATE Previous Years Solved Paper 5

5k R1 R1
Vs
1k
2V –
Vo –
3V + R2
1k +
8k

R2
iL RL

(a) 1 V (b) 6 V
(c) 14 V (d) 17 V
[EC-2003 : 2 Marks] vs vs
(a) (b)
R2 R2
Q.31 Three identical amplifiers with each one having
vs vs
a voltage gain of 50, input resistance of 1 k (c) (d)
RL R1
and output resistance of 250 , are cascaded.
The open-circuit voltage gain of the combined [EC-2004 : 2 Marks]
amplifier is Q.35 The input resistance Ri of the amplifier shown
(a) 49 dB (b) 51 dB in the figure is
(c) 98 dB (d) 102 dB
30 k
[EC-2003 : 2 Marks]

Q.32 An ideal op-amp is an ideal 10 k


(a) voltage controlled current source. –
+
(b) voltage controlled voltage source. Vs
(c) current controlled current source. Ideal operational amplifier

(d) current controlled voltage source. Ri


[EC-2004 : 1 Mark]
30
Q.33 The circuit in the figure is (a) k (b) 10 k
4
(c) 40 k (d) infinite
– [EC-2005 : 1 Mark]
R R Vo
Vi + Q.36 The voltage e0 indicated in the figure has been
measured by an ideal voltmeter. Which of the
following can be calculated?

1M
(a) low-pass filter
(b) high-pass filter
(c) band-pass filter –
eo
+
(d) band-reject filter
[EC-2004 : 1 Mark] 1M

Q.34 In the op-amp circuit given in the figure, the


load current iL is
6 Electronics Engineering Analog Electronics

(a) Bias current of the inverting input only. Q.39 For the op-amp circuit shown in the figure, Vo is
(b) Bias current of the inverting and non- 2k
inverting inputs only.
(c) Input offset current only. 1k

(d) Both the bias currents and the input offset 1V Vo
+
current.
1k
[EC-2005 : 2 Marks] 1k

Q.37 The op-amp circuit shown in the figure is a filter.


The type of filter and its cut-off frequency are
respectively (a) –2 V (b) –1 V
(c) –0.5 V (d) 0.5 V
10 k
[EC-2007 : 2 Marks]

10 k Q.40 In the op-amp circuit shown, assume that the


– diode current follows the equation
Vo
+ I = Is exp(V/VT). For Vi = 2 V, Vo = V01 and for
Vi 1 µF Vi = 4 V, Vo = V02. The relationship between V01
1k
and V02 is

(a) high pass, 1000 rad/sec. 2k


Vi –
(b) low pass, 1000 rad/sec. Vo
+
(c) high pass, 10000 rad/sec.
(d) low pass, 10000 rad/sec.
[EC-2005 : 2 Marks] (a) V02 = 2 V01 (b) V02 = e2 V01

Q.38 For the circuit shown in the following figure, (c) V02 = V01 ln 2 (d) V01 – V02 = VT ln2
the capacitor C is initially uncharged. At t = 0, [EC-2007 : 2 Marks]
the switch S is closed. The voltage VC across the Linked Answer Questions (41 and 42):
capacitor at t = 1 millisecond is
Consider the op-amp circuit shown in the figure.
S R1
C = 1 µF
– +
VC R1
1k –
– Vi Vo
+
+ R C
10 V

Q.41 The transfer function Vo(s)/Vi(s) is


In the figure shown above, the op-amp is
supplied with ±15 V. 1 sRC 1 + sRC
(a) (b)
1 + sRC 1 sRC
(a) 0 Volt (b) 6.3 Volts
1 1
(c) 9.45 Volts (d) 10 Volts (c) (d)
1 sRC 1 + sRC
[EC-2006 : 2 Marks]
[EC-2007 : 2 Marks]
GATE Previous Years Solved Paper 7

Q.42 If Vi = V1 sin( t) and Vo = V2 sin( t + ), then the 10 V


minimum and maximum values of (in radians)
are respectively 5k

(a) and (b) 0 and


2 2 2 V –

and 0
+
(c) – and 0 (d)
2
1.4 k 5V
[EC-2007 : 2 Marks]

Q.43 Consider the following circuit using an ideal


op-amp. The I-V characteristics of the diode is
(a) Positive feedback, V = 10 V
described by the relation I = I o ( e v / vT 1) (b) Positive feedback, V = 0 V
where VT = 25 mV, Io = 1 µA and V is the voltage (c) Negative feedback, V = 5 V
across the diode (taken as positive for foward (d) Negative feedback, V = 2 V
bias). [EC-2009 : 2 Marks]
D 4k
Q.46 Assuming the op-amp to be ideal, the voltage
gain of the amplifier shown below is
100 k
Vi = –1 V –
Vo R1
+ – Vo
+
R2
For an input voltage Vi = –1 V, the output voltage Vi
Vo is
(a) 0 V (b) 0.1 V R3

(c) 0.7 V (d) 1.1 V


[EC-2008 : 2 Marks]
R2 R3
Q.44 The op-amp circuit shown below represents a (a) (b)
R1 R1
C
R2 R3 R2 + R3
(c) (d)
R1 R1
R1 R2
L [EC-2010 : 1 Mark]
Vi –
Vo
+ Q.47 The transfer characteristic for the precision
rectifier circuit shown below is (assume ideal
op-amp and practical diodes).
(a) high-pass filter (b) low-pass filter
+20 V
(c) band-pass filter (d) band-reject filter R
[EC-2008 : 2 Marks]

Q.45 In the circuit shown below, the op-amp is ideal, 4R


D2
the transistor has VBE = 0.6 V and = 150. Decide VI –
R Vo
whether the feedback in the circuit is positive or + D1
negative and determine the voltage V at the
output of the op-amp.
8 Electronics Engineering Analog Electronics

Q.49 The circuit shown is a


Vo

10 R2

(a)
C R1 +5 V +
+ – Output
VI Input
–10 –5 0 + –

–5 V
Vo

(a) low-pass filter with


(b) 5
1
f 3-dB = rad/sec
VI ( R1 + R2 ) C
–10 –5 0
(b) high-pass filter with
Vo 1
f 3-dB = rad/sec
R1C
(c) 5 (c) low-pass filter with
1
+5
VI f 3-dB = rad/sec
0 R1C
(d) high-pass filter with
Vo
1
10
f 3-dB = rad/sec
( R1 + R2 ) C
(d)
[EC-2012 : 2 Marks]
VI
0 +5 Q.50 In the circuit shown below what is the output
voltage (Vout) if a silicon transistor Q and an
[EC-2010 : 2 Marks] ideal op-amp are used?
Q.48 The circuit below implement a filter between the
input current ii and output voltage vo. Assume
that the op-amp is ideal. The filter implemented Q
+15 V
is a 1k

L1 Vo
+
5V
–15 V
I1 R1

+
(a) –15 V (b) –0.7 V
+ vo (c) +0.7 V (d) +15 V

[EC-2013 : 1 Mark]

(a) low-pass filter (b) band-pass filter Q.51 In the circuit shown below the op-amps are
ideal. Then Vout (in Volts) is
(c) band-stop filter (d) high-pass filter
[EC-2011 : 1 Mark]
GATE Previous Years Solved Paper 9

Q.54 In the circuit shown, the op-amp has finite input


1k 1k impedance, infinite voltage gain and zero input
–2 V
+15 V offset voltage. The output voltage Vout is
+15 V
– R2

+ Vo
+ R1 I1
1k
–15 V –
–15 V Vo
+1 V +
I2
1k
1k

(a) –I2(R1 + R2) (b) I2R2


(a) 4 (b) 6 (c) I1R2 (d) –I1(R1 + R2)
(c) 8 (d) 10 [EC-2014 : 2 Marks]
[EC-2013 : 2 Marks] Q.55 In the differential amplifier shown in the figure,
Q.52 In the low-pass filter shown in the figure, for a the magnitudes of the common mode and
cut-off frequency of 5 kHz, the value of R2 (in k ) di fferenti al mode gai ns are A cm and A d ,
is ______ . respectively. If the resistance RE is increased,
then
R2

VCC
C
RC RC
10 nF
1k
Vi – Vo
R1 Vo + –
+
+

Vi
[EC-2014 : 1 Mark]

Q.53 In the voltage regulator circuit shown in the
figure, the op-amp is ideal. The BJT has
RE Io
VBE = 0.7 V and = 100, and the Zener voltage is
4.7 V. For a regulated output of 9 V, the value of
R (in ) is _______ . –VE

Vi = 12 V Vo = 9 V (a) Acm increase


(b) common mode rejection ratio increases
1k 1k (c) Ad increase
+
(d) common mode rejection ratio decreases

[EC-2014 : 1 Mark]

Vz = 4.7 V R Q.56 Assuming that the op-amp in the circuit shown


is ideal, Vo is given by

[EC-2014 : 2 Marks]
10 Electronics Engineering Analog Electronics

Q.59 In the circuit shown, Vo = V0A for switch SW in


3R
position A and Vo = V0B for SW in position B.
Assume that the op-amp is deal. The value of
R V0B/V0A is _______ .
V1 –
Vo
V2 + 1k

2R R 1k
5V –

1k Vo
A
B +
5 5 SW
1k
(a) V1 3V2 (b) 2 V1 V2
2 2 1k
1V
3 7 11
(c) V1 + V2 (d) 3V1 + V2
2 2 2
[EC-2015 : 1 Mark]
[EC-2014 : 2 Marks]
Q.60 In the bistable circuit shown, the ideal op-amp
Q.57 The circuit shown represents
has saturation levels of ±5 V. The value of R1
C2 +12 V
(in k ) that gives a hysteresis width of 500 mV
vi –
vo is ______ .
R2 +
–12 V R2 = 20 k
R1
–2 V
C1 R1
+
Vo
(a) a bandpass filter Vi

(b) a voltage controlled oscillator
(c) an amplitude modulator
(d) a monostable multivibrator [EC-2015 : 1 Mark]
[EC-2014 : 1 Mark]
Q.61 For the voltage regulator circuit shown, the input
Q.58 In the circuit shown, assume that the op-amp is voltage (Vin) is 20 V ± 20% and the regulated
ideal. The bridge output voltage Vo (in mV) for output voltage (Vout) is 10 V. Assume the op-amp
= 0.05 is ______ . to be ideal. For a load RL drawing 200 mA, the
maximum power dissipation in Q1 (in Watts) is
1V
100 + _______ .

250(1 + ) 250(1 – )
Vi Q1 Vo
– +
Vo 4V
Vref + R1 RL
250(1 – ) 250(1 + ) –

100
50
R2 = 10 k

[EC-2015 : 2 Marks]
[EC-2015 : 2 Marks]
GATE Previous Years Solved Paper 11

Q.62 Assuming that the op-amp in the circuit shown The load current Io through RL is
below is deal, the output voltage Vo (in Volts) is + 1 Vref
(a) I o =
______ . R
2k
Vref
(b) I o =
+12 V +1 R
1k
– + 1 Vref
Vo (c) I o =
+ 2R

1V –12 V Vref
(d) I o =
+ 1 2R
[EC-2015 : 2 Marks]
[EC-2016 : 1 Mark]
Q.63 In the circuit shown using an ideal op-amp, the
3-dB cut-off frequency (in Hz) is _____ . Q.66 The following signal Vi of peak voltage 8 V
applied to the non-inverting terminal of an ideal
vi +
10 k 10 k vo op-amp. The transistor has VBE = 0.7 V, = 100,

0.1 µF VLED = 1.5 V, VCC = 10 V and –VCC = –10 V.
10 k 10 k
10 V 10 V

100
[EC-2015 : 1 Mark] 8k

Q.64 In the circuit shown, assume that the op-amp is +VCC


LED
ideal. If the gain (Vo/Vin) is –12, the value of R –

(in k ) is ______ . + 15 k
2k Vi
10 k 10 k –VCC

10 k Vi
Vin –
Vo
+ 6V
4V
2V
[EC-2015 : 2 Marks] t
–2 V
Q.65 Consider the constant current source shown in
–4 V
the figure below. Let represent the current gain
–6 V
of the transistor.
+V CC The number of times the LED glows is _____ .
[EC-2016 : 1 Mark]
R
Vref Q.67 An ideal op-amp has voltage sources, V1, V3,
V5, ..... VN – 1 connected to the non-inverting input

and V2, V4, V6, ....., VN connected to the inverting
+ R2 input as shown in the figure below
(+VCC = 15 Volt, –VCC = –15 Volt). The voltages
R1 RL V1, V2, V3, V4, V5, V6, .... are 1, –1/2, 1/3, –1/4,
1/5, –1/6, .... Volt, respectively. As N approaches
infinity, the output voltage (in Volt) is _____ .
12 Electronics Engineering Analog Electronics

10 k
The time t = t1 (in seconds) at which Vo changes
V2 state is ______ .
10 k 10 k
V4 [EC-2016 : 2 Marks]
10 k +VCC
VN –
Q.70 An op-amp has a finite open-loop voltage gain
1k Vo of 100. Its input offset voltage Vios (= +5 mV) is
V1 +
–VCC modeled as shown in the circuit below. The
1k
V3 amplifier is ideal in all other respects. Vinput is
1k 10 k 25 mV.
VN – 1
1k 15 k

[EC-2016 : 2 Marks]

Q.68 A p-i-n photodiode of responsivity 0.8 A/W is +
connected to the inverting input of an ideal Ao = 100
V ios = 5 mV
op-amp as shown in the figure, +VCC = 15 V,
–VCC = –15 V, Load resistor RL = 10 k . If 10 µW
V input
of power is incident on the photodiode, then
the value of the photocurrent (in µA) through
the load is ______ . The output voltage (in millivolts) is ______ .
[EC-2016 : 2 Marks]
1M
Q.71 For the circuit shown in the figure, R1 = R2 =
R3 = 1 , L = 1 µH and C = 1 µF. If the input
+VCC
Vin = cos(106 t), then the overall voltage gain

Vo (Vout/Vin) of the circuit is ______ .
+
R1
–VCC 10 k R3
1M
L

+VCC –
+ R2 C Vout
[EC-2016 : 2 Marks] +
Vin
Q.69 In the op-amp circuit shown, the Zener diode
Z1 and Z2 clamp the output voltage Vo to +5 V or
–5 V. The switch ‘S’ is initially closed and is [EC-2016 : 2 Marks]
opened at time t = 0.
Q.72 For the operational amplifier circuit shown, the
+10 V output saturation voltages are ±15 V. The upper
and lower threshold voltages for the circuit are,
S
10 µ F respectively
t=0 +10 V
+ –
– 470 Vout
Vo Vin +
+ – 10 k
Z1
–10 V
10 k 5k
4k
1k Z2
3V
–10 V
0V 0V
GATE Previous Years Solved Paper 13

(a) +5 V and –5 V (b) +7 V and –3 V Q.75 In the circuit shown below, the op-amp is ideal
(c) +3 V and –7 V (d) +3 V and –3 V and Zener voltage of the diode is 2.5 Volts. At
[EC-2017 : 1 Mark] the input, unit step voltage is applied i.e.
VIN(t) = u(t) Volts. Also at t = 0, the voltage across
Q.73 The amplifier circuit shown in the figure is each of the capacitors is zero.
implemented using a compensated operational
1 µF
amplifier (op-amp), and has an open-loop
voltage gain, A0 = 105 V/V and an open-loop
1k 1 µF
cut-off frequency, fc = 8 Hz. The voltage gain of 1V –
the amplifier at 15 kHz, (in V/V), is _____ . vIN(t )
0V + vOUT(t)
t=0
R2 = 79 k

R2 = 1 k
The time t, in milliseconds, at which the output
– voltage vOUT crosses –10 V is
Vo
+ (a) 2.5 (b) 5

Vi (c) 7.5 (d) 10


[EC-2018 : 1 Mark]

Q.76 An op-amp based circuit is implemented as


[EC-2017 : 2 Marks]
shown below.
Q.74 In the voltage reference circuit shown in the 31 k
figure, the op-amp is ideal and the transistors
Q1, Q2, ...... Q32 are identical in all respects and +15 V
1k
have infinitely large values of common-emitter –
A
current gain ( ). The collector current (IC) of the + + +
1V
transistors is related to their base-emitter voltage Vo
– –
(VBE) by the relation IC = IS exp(VBE/VT), where –15 V

IS is the saturation current. Assume that the In the above circuit, assume the op-amp to be
voltage VP shown in the figure is 0.7 V and the ideal. The voltage (in volts correct to one decimal
thermal voltage VT = 26 mV. place) at node A, connected to the negative input
of the op-amp as indicated in the figure is ___ .
[EC-2018 : 2 Marks]
20 k
20 k +15 V
– Q.77 In the circuit shown below, all the components
+
V out are ideal. If Vi is +2 V, the current Io sourced by
–15 V the op-amp is ______ mA.
5k
1k
VP
+15 V
1k
Q1 Q2 Q3 Q32 – Io
Vi +
1k
–15 V
The output voltage Vout (in Volts) is _____ .
[EC-2017 : 2 Marks] [EC-2020 : 1 Mark]
14 Electronics Engineering Analog Electronics

Q.78 The components in the circuit shown below are If Vout = 1 Volt for Vin = 0.1 Volt and Vout = 6 Volt
ideal. If the op-amp is in positive feedback and for Vin = 1 Volt, where Vout is measured across
the input voltage Vi is a sine wave of amplitude RL connected at the output of this op-amp, the
1 V, the output voltage Vo is value of RF/Rin is
1k (a) 3.285 (b) 3.825
(c) 5.555 (d) 2.860
1V 1k +5 V [EC-2021 : 1 Mark]
0 Vi –
–1 V Vo
+ Q.81 Consider the circuit with an ideal op-amp
–5 V shown in the figure.

R RF
(a) a square wave of 5 V amplitude.
(b) a inverted sine wave of 1 V amplitude.
R
(c) a non-inverted sine wave of 2 V amplitude.
+VCC
(d) a constant of either +5 or –5 V. Vin
Vref Vout
[EC-2020 : 1 Mark]

Q.79 The components in the circuit given below are –VCC

ideal. If R = 2 k and C = 1 µF, the –3 dB cut-off


frequency of the circuit (in Hz), is Assuming Vin << VCC and Vref << VCC ,
R
the condition at which Vout equals to zero is
C (a) Vin = 2 + Vref (b) Vin = 2Vref
R (c) Vin = Vref (d) Vin = 0.5Vref
Vi( j ) –
Vo( j ) [EC-2021 : 1 Mark]
+
2C 2R Q.82 An ideal op-amp circuit with a sinusoidal input
is shown in the figure. The 3-dB frequency is
the frequency at which the magnitude of the
(a) 34.46 (b) 79.58 voltage gain decreases by 3-dB from the
(c) 59.68 (d) 14.92 maximum value. Which of the options is/are
[EC-2020 : 2 Marks] correct?

Q.80 For the circuit with an ideal op-amp shown in 2k

the figure, Vref is fixed.


1 µF +15 V
1k
Rin RF
+
Vin +
Vo
– –15 V –
Vin
+12 V

R1 (a) The circuit is a low-pass filter.


Vout
(b) The circuit is a high-pass filter.
–12 V
(c) The 3-dB frequency is 1000 rad/sec.
Vref R2 RL = 100
1000
(d) The 3-dB frequency is rad/sec.
3
[EC-2022]
GATE Previous Years Solved Paper 15

Q.83 For the following circuit with an ideal op-amp,


20log10(|Av|)
the difference between the maximum and the
minimum values of the capacitor voltage (VC) is 15

_______ . 12
9
R
6

VC +15 V 3
– +
0
C 1 2 3 4 log10( )

–12 V
(a) R = 3 k , C = 1 µF
R R R R (b) R = 1 k , C = 3 µF
(c) R = 4 k , C = 1 µF
(d) R = 3 k , C = 2 µF
D D
[EC-2022]
ID D
Q.85 Consider the circuit shown with an ideal
op-amp. The output voltage Vo is ______ V.
+ VD –
(Rounded off to two decimal places)
3R
20
ID(mA)

+15 V
10 R R R 2R

0 +
(V) 2R 2R 2R 2R 2R Vo
0 VD –15 V –

(a) 15 V (b) 27 V +1.6 V +1.6 V

(c) 13 V (d) 14 V [EC-2022]


[EC-2022]
ELECTRICAL EN GINEERIN G
Q.84 A circuit with an ideal op-amp is shown. The
(GATE Previous Years Solved Papers)
Bode plot for the magnitude (in dB) of the gain
transfer function (Av(j ) = Vout(j )/Vin(j ) of the Q.1 An ideal op-amp is used to make an inverting
circuit is also provide (here, is the angular amplifier. The two input terminals of the op-amp
frequency in rad/sec). The values of R and C are at the same potential because
are _______ . (a) the two input terminals are directly shorted
internally.
R
(b) the input impedance of the op-amp is
+15 V infinity.
1k
Vo (c) the open-loop gain of the op-amp is infinity.
+ (d) CMRR is infinity.
+
Vin 1k Vout
–12 V [EE-1992 : 1 Mark]
– C

Q.2 The circuit shown in figure is excited by the circuit
waveform shown. Sketch the waveform of the
output. Assume all the components are ideal.
16 Electronics Engineering Analog Electronics

Vi (Volts) Q.6 A non-inverting op-amp amplifier is shown in


figure. The output voltage is

2R
R

0 0.5 1 2 3 t(sec)
–2 +
R
2 + sin100t
– R
Vo
Vi + VA D1 3
C D2 (a) sin 100t (b) 3 sin100t
2
(c) 2 sin100t (d) none of these
[EE-1996 : 1 Mark]
[EE-1992 : 2 Marks]
Q.7 A major advantage of active filters is that they
Q.3 Given figure, shows a non-inverting op-amp
can be realized without using
summer with V1 = 2 V and V2 = –1 V the output
voltage Vo = _______ . (a) op-amps (b) inductors
(c) resistors (d) capacitors
2R [EE-1997 : 1 Mark]
R
V

V
Vo Q.8 Match the following:
2V +
R Circuits:
–1 V VCC
R
R

[EE-1994 : 1 Mark] –
A. Vo
+
Q.4 The common mode voltage of a unity gain Vi
R
(voltage follower) op-amp buffer in terms of its
output voltage V is ______ .
[EE-1995 : 1 Mark]

Q.5 Let the magnitude of the gain in the inverting


op-amp circuit shown be x with switch S1 open.
When the switch is closed, the magnitude of –
B. Vo
gain becomes. +
Vi
R
S1

R R
R
Vi –
Vo
+

Vo
C. +
Vi
(a) x/2 (b) –x
(c) 2x (d) –2x
[EE-1996 : 1 Mark]
GATE Previous Years Solved Paper 17

Functions: Q.12 For the circuit of figure with an ideal operational


P. High-pass filter amplifier, the maximum phase shift of the output
Q. Amplifier Vout with reference to the input Vin is
R. Comparator R1
S. Low-pass filter
[EE-1998 : 2 Marks] R

Vi Vo
Q.9 The feedback factor for the circuit shown in +
figure is R
C
100 k

+ Vo
+
Vs
(a) 0° (b) –90°
– 1k
90 k (c) +90° (d) ±180°
10 k
[EE-2003 : 1 Mark]

Q.13 Assuming the operational amplifier to be ideal,


9 9 the gain Vout/Vin for the circuit shown in figure
(a) (b)
100 10 is
1 1 1k 10 k
(c) (d)
9 10
[EE-2000 : 1 Mark] 1k

Q.10 The circuit shown in figure uses an ideal 1k


op-amp working with +5 V and –5 V power Vi –
Vo
supplies. The output voltage Vo is equal to +

1k

+5 V
(a) –1 (b) –20

Vo (c) –100 (d) –120
a + Va = 0
1 mA [EE-2003-04 : 2 Marks]
–5 V

Q.14 The input resistance RIN (= Vx/Ix) of the circuit


(a) +5 V (b) –5 V in figure is
(c) +1 V (d) –1 V R1 = 10 k R2 = 100 k

[EE-2000 : 1 Mark]

Q.11 An op-amp having a slew rate of 62.8 V/µsec is –


connected in a voltage follower configuration. Vy
+
If the maximum amplitude of the input
sinusoidal is 10 Volts, then the minimum
Vx
frequency at which the slew rate limited Ix R3 = 1 M
distortion would set in the output is
(a) 1 MHz (b) 6.28 MHz (a) +100 k (b) –100 k
(c) +1 M (d) –1 M
(c) 10 MHz (d) 62.8 MHz
[EE-2004 : 2 Marks]
[EE-2001 : 2 Marks]
18 Electronics Engineering Analog Electronics

Q.15 Consider the inverting amplifier, using an ideal Vo


operational amplifier shown in the figure. The
designer wishes to realize the input resistance (d)
t
seen by the small signal source to be as large as
possible, while keeping the voltage gain
between –10 and –25. The upper limit on RF is [EE-2005 : 2 Marks]
1 M . The value of R1 should be Q.17 For a given sinusoidal input voltage, the voltage
waveform at point P of the clamper circuit
R1 RF
shown in figure will be
Vi –
Vo
+
C +12 V

(a) Infinity (b) 1 M –
RL
(c) 100 k (d) 40 k Vin + P
–12 V
[EE-2005 : 2 Marks] +

Q.16 In the given figure, if the input is a sinusoidal


signal, the output will appear as shown in
Vin
Vin

t
t

R +V

+ (a)
+
–V
Vi
RL Vo
R

(b)
Vo

(a)
t

Vo 12 V
(c)
–0.7 V
(b) t

Vo
(d) 0.7 V
(c) –12 V
t

[EE-2006 : 1 Mark]
GATE Previous Years Solved Paper 19

Q.18 A relaxation oscillator is made using op-amp Q.19 The circuit shown in the figure is
as shown in figure. The supply voltages of the
op-amp are ±12 V. The voltage waveform at point R1
P will be +
+ R2 –
V Load
R1 –

R2 r

C
– rV
+
(a) a voltage source with voltage
R1 R2
2k
P r R2
(b) a voltage source with voltage V
R1
10 k 10 k
r R2 V
(c) a current source with current
R1 + R2 r

R2 V
(d) a current source with current
R1 + R2 r
[EE-2007 : 1 Mark]
6
(a) Q.20 The switch ‘S’ in the circuit of the figure is
initially closed, it is opened at time, t = 0. You
–10
may neglect the Zener diode forward voltage
drops. What is the behaviour of Vout for t > 0?

10 +10 V
+10 V
1k
(b)

6 Vo
+
S +10 k 5.0 V
0.01 µF

–10 V

100 k 5.0 V
6
(c) –10 V

(a) It makes a transition from –5 V to +5 V at


–10
t = 12.98 µs.
(b) It makes a transition from –5 V to +5 V at
t = 2.57 µs.
10
(c) It makes a transition from +5 V to –5 V at
(d) t = 12.98 µs.
6 (d) It makes a transition from +5 V to –5 V at
t = 2.57 µs.
[EE-2006 : 2 Marks] [EE-2007 : 2 Marks]
20 Electronics Engineering Analog Electronics

Q.21 The block diagrams of two of half wave rectifiers Statement for Linked Answer Questions (22 and 23):
are shown in the figure with the transfer A general filter circuit is shown in the figure:
characteristics of the rectifiers.
R2
It is desired to make full wave rectifier using
above two half-wave rectifiers. The resultant
circuit will be
R1 C
P Q
Vi –
Vo Vo VA Vo
Vin Vo Vin Vo +
Vin
0 R3
Vin
0 R4

It is desired to make full wave rectifiers using


above two-half-wave rectifiers. The resultants
circuit will be
Q.22 If R1 = R2 = RA and R3 = R4 = RB, the circuit acts
R as a
P R (a) all pass filter (b) band pass filter
(a) Vi –
Vo (c) high pass filter (d) low pass filter
+
Q R [EE-2008 : 2 Marks]

Q.23 The output of the filter is (Q.22) is given to the


R circuit shown in figure.
RA/2
P R
(b) Vi –
Vo
+
Q R Vi C Vo
R

The gain vs frequency characteristic of the


R output (Vo) will be

Gain
Q R
(c) Vi –
Vo
+ (a)
P R
R

R R Gain

– (b)
P R Vo
(d) Vi +

Q R

[EE-2008 : 2 Marks]
GATE Previous Years Solved Paper 21

(a) 10 mA leading by 90°


Gain
(b) 20 mA leading by 90°
(c) 10 mA leading by 90°
(c) (d) 10 mA lagging by 90°
[EE-2009 : 2 Marks]

Q.26 Transformer and emitter follower can both be


used for impedance matching at the output of
Gain
an audio amplifier. The basic relationship
between the input power Pin and output power
Pout in both the cases is
(d)
(a) Pin = Pout for both transformer and emitter
follower.
(b) Pin > Pout for both transformer and emitter
[EE-2008 : 2 Marks] follower.
(c) Pin < Pout for both transformer and Pin = Pout
Q.24 The nature of feedback in the op-amp circuit
for emitter follower.
shown is
(d) Pin = Pout for both transformer and Pin < Pout
for emitter follower. [EE-2009 : 2 Marks]
1k +6 V
2k
– Q.27 An ideal op-amp circuit and its input waveform
V out
+ are shown in the figures. The output waveform
–6 V of this circuit will be
Vin
Vin

3
2
(c) current-current feedback 1
t4 t5 t6
(b) voltage-voltage feedback V0
t1 t2 t3
t
–1
(c) current-voltage feedback
–2
(d) voltage-current feedback –3

[EE-2009 : 1 Mark]
1k 6V
Q.25 The following circuit has R = 10 k , C = 10 µF. Vi –
Vo
The input voltage is a sinusoid at 50 Hz with +
2k
an rms value of 10 V. Under ideal conditions, –3 V
the current is from the source is
1k
R

10 k
is
– Vo
op-amp
6
Vs(rms) = 10 V, +
50 Hz
10 k (a)
t3 t6
R 0 t
C 10 µF
–3
22 Electronics Engineering Analog Electronics

Vo (a) an all-pass filter


(b) an all-stop filter
6
(c) a band-stop (band-reject) filter
(d) a band-pass filter
(b)
t3 t6 [EE-2011 : 1 Mark]
0 t

Q.30 For the circuit shown below.


–3
R

Vo
R +12 V
– +12 V
6 Vi –
+
R –12 V +
(c) R –12 V
Vo
t6
0 t
t2 t4
R R
–3

Vo
Vo
+12 V
6

+6 V
(d) (a) Vi
–6 V
0 t
t2 t4 t6

–3
–12 V

[EE-2009 : 2 Marks]
Vo
Q.28 Given that the op-amp is ideal, the output voltage
Vo is +12 V

2R
+6 V
(b) Vi
–6 V
+10 V
R

Vout –12 V
+

–10 V
+2 V
Vo

+12 V
(a) 4 V (b) 6 V
(c) 7.5 V (d) 12.12 V
[EE-2010 : 1 Mark] (c) Vi
–6 V +6 V
Q.29 A low-pass filter with a cut-off frequency of 30 Hz
is cascaded with a high-pass filter with a cut-off
frequency of 20 Hz. The resultant system of –12 V

filters will functions as


GATE Previous Years Solved Paper 23

Q.33 In the circuit shown below the op-amps are


Vo
ideal. Then Vout (in Volts), is
+12 V
1k 1k

2V
+15 V
– +15 V
(d) Vi –
–6 V +6 V +
+ Vout
–15 V
1k –15 V

–12 V +1 V
1k
1k

[EE-2011 : 1 Mark]

Q.31 The circuit shown is a (a) 4 (b) 6


R2 (c) 8 (d) 10
[EE-2013 : 2 Marks]

C R1 +5 V + Q.34 Given that the op-amps in the figure are ideal,


+ – Output the output voltage Vo, is
Input –
+
– V2 + R
–5 V

R R
1 –
(a) low pass filter with f 3dB = rad/s 2R Vo
( R1 + R2 ) C +
R R
1 – R
(b) high pass filter with f 3dB = rad/s
R1C V1 +

1 (a) (V1 – V2) (b) 2 (V1 – V2)


(c) low pass filter with f 3dB = rad/s
R1C
(V1 V2 )
1 (c) (d) (V1 + V2)
(d) high pass filter with f 3dB = rad/s 2
( R1 + R2 ) C
[EE-2014 : 2 Marks]
[EE-2012 : 2 Marks]
Q.35 In the figure shown, assume the op-amp to be
Q.32 In the circuit shown below what is the output
ideal. Which of the alternatives gives the correct
voltage (Vout) if a silicon transistor Q and an
ideal op-amp are used? Vo ( )
Bode plots for the transfer function ?
Vi ( )

Q
+VCC
1k +15 V 1k
– Vi +
V out Vo
+ –
5V 1 µF
–15 V
–VCC

(a) –15 V (b) –0.7 V Rf


(c) +0.7 V (d) +15 V
[EE-2013 : 1 Mark]
24 Electronics Engineering Analog Electronics

The output of the circuit for a given input vi, is


Vo ( )
20 log
Vi ( ) R2 R2
(a) vi (b) 1+ vi
0 10 10
2
10
3
0 10 10
2 3
10 R1 R1
(a) 1 1
–10 – /4
R2
–20 – /2 (c) 1+ vi (d) +Vsat or –Vsat
R1
–30

[EE-2014 : 1 Mark]

Q.37 The transfer characteristic of the op-amp circuit


20 log
Vo ( )
Vi ( )
/2 shown in figure is
/4
2 3 3 R
(b) 0
1
10 10 10 0
1
10
2
10
10
–10 – /4 R
–20 – /2
R +Vsat
–30 vi – R +Vsat

+
–Vsat + vo
R –Vsat
20 log
Vo ( )
/2 R
Vi ( )
/4
2 3 2 3
(c) 0 10 10 10 0 10 10 10
1 1
–10 – /4 vo
–20 – /2

–30

(a)
–1
vi
Vo ( )
20 log /2
Vi ( )
/4
0 10 10
3
0 10 vo
(d) 1 10
2
1 10
2
10
3

–10 – /4

–20 – /2
(b)
–30
–1
vi
[EE-2014 : 2 Marks]

Q.36 An operational amplifier circuits is shown in vo


the figure.
vi
R (c) 1

R +Vsat
+Vsat –
vi – + vo vo
+ –Vsat
–Vsat vi
(d)
R2 –1
R1

[EE-2014 : 2 Marks]
GATE Previous Years Solved Paper 25

Q.38 Consider the circuit shown in the figure. In this


0.1 µF
circuit R = 1 k , and C = 1 µF. The input voltage
is sinusoidal with a frequency of 50 Hz,
represented as a phasor with magnitude Vi and
1k
phase angle 0 radian as shown in the figure. 1k
The output voltage is represented as a phasor Vi –
Vo
with magnitude Vo and phase angle (in radian) +
relative to the phase angle of the input voltage?

C R
[EE-2015 : 1 Mark]

vi = Vo 0
+ v o = Vo Q.41 The filters F1 and F2 having characteristics as
C shown in Fig. (a) and (b) are connected as shown
R
in Fig. (c).

F1
(a) 0 (b) Vo/Vi

Vi Vo
(c) (d)
2 2
f1
[EE-2015 : 1 Mark] f

Q.39 The op-amp shown in the figure has a finite (a)


gain A = 1000 and an infinite input resistance.
A step voltage Vi = 1 mV is applied at the input F2
Vo/Vi
at time t = 0 as shown. Assuming that the
Vi Vo
operational amplifier is not saturated, the time
constant (in millisecond) of the output voltage, f2
f
Vo, is
(b)
C
R/2
1 µF
R
– +Vsat
1k R
+ + F1 –
1 mV Vi
A = 1000 Vo Vi + Vo
t = 0s – F2 –Vsat
R
(c)

The cut-off frequencies of F1 and F2 are f1 and f2


(a) 1001 (b) 101
respectively. If f1 < f2, the resultant circuit
(c) 11 (d) 1
exhibits the characteristic of a
[EE-2015 : 2 Marks]
(a) Band-pass filter (b) Band-stop filter
Q.40 The operational amplifier shown in the figure (c) All-pass filter (d) High Q-filter
is ideal. The input voltage (in Volt) is [EE-2015 : 1 Mark]
Vi = 2 sin(2 × 2000t). The amplitude of the
output voltage Vo (in Volt), is ______ . Q.42 The saturation voltage of the ideal op-amp
shown below is ±10 V. The output voltage Vo of
the following circuit in the steady-state is
26 Electronics Engineering Analog Electronics

1k 9

+10 V 1 VCC
0.25 µF V3 –
– Vout
Vo 1
+ V2 +
2k –VSS
4
–10 V
V1

2k
(a) 1.8V1 + 7.2V2 – V3
(b) 2V1 + 8V2 – 9V3
(c) 7.2V1 + 1.8V2 – V3
(a) square wave of period 0.55 ms (d) 8V1 + 2V2 – 9V3 [EE-2016 : 2 Marks]
(b) triangular wave of period 0.55 ms
Q.46 The approximate transfer characteristic for the
(c) square wave of period 0.25 ms circuit shown below with an ideal operational
(d) triangular wave of period 0.25 ms amplifier and diode will be
[EE-2015 : 2 Marks] VSS
Vin +
Q.43 Of the four characteristics given below, which
are the major requirements for an –
D
instrumentation amplifier? –V SS
Vo
P. High common mode rejection ratio
R
Q. High input impedance
R. High linearity
S. High output impedance Vo
(a) P, Q and R only (b) P and R only
(c) P, Q and S only (d) Q, R and S only
(a)
[EE-2015 : 1 Mark]
Vin
Q.44 The circuit shown below is an example of a
Vo
R2

(b)
C

Vin
R1 +15 V
Vi + Vo
Vo

–15 V (c)

Vin
(a) low pass filter (b) band pass filter
Vo
(c) high pass filter (d) notch filter
[EE-2016 : 1 Mark]
(d)
Q.45 For the circuit shown below, taking the op-amps
Vin
as ideal, the output voltage Vout in terms of the
input voltages V1, V2 and V3 is [EE-2017 : 2 Marks]
GATE Previous Years Solved Paper 27

Q.47 For the circuit shown below, assume that the R1 R2


op-amp is ideal. Which one of the following is (a) Z (b) Z
R2 R1
true?
R2
R (c) Z (d) Z
R1 + R2
R R [EE-2018 : 1 Mark]

R Q.49 In the circuit below, the operational amplifier is


– ideal. If V1 = 10 mV and V2 = 50 mV, the output
+ vo voltage (Vout), is
2R
100 k
vs
2R
10 k
(a) vo = vs (b) vo = 1.5 vs V1 –
Vout
(c) vo = 2.5 vs (d) vo = 5 vs V2 +
10 k
[EE-2017 : 2 Marks]
100 k
Q.48 The op-amp shown in the figure is ideal. The
input impedance Vin/iin is given by
Z
(a) 400 mV (b) 500 mV
(c) 600 mV (d) 100 mV
iin [EE-2019 : 2 Marks]
+
Vo

Vin

R1
R2
28 Electronics Engineering Analog Electronics

Electronics & Electrical Engineering


GATE Previous Years Solved Paper

A n swe rs & Expl a n a t i o n s

Answers
EC O perational Amplifiers

1. (100) 2. (c) 3. (b, c) 4. (d) 5. (c) 6. (d) 7. ( ) 8. (c)

9. (–4) 10. (0.02) 11. (Sol.) 12. (33) 13. (159) 14. (d) 15. (d) 16. (d)

17. (a) 18. (d) 19. (c) 20. (a) 21. (b) 22. (c) 23. (a) 24. (b)

25. (a) 26. (b) 27. (c) 28. (b) 29. (c) 30. (b) 31. (c) 32. (b)

33. (a) 34. (a) 35. (b) 36. (c) 37. (a) 38. (c) 39. (c) 40. (d)

41. (a) 42. (c) 43. (b) 44. (b) 45. (d) 46. (a) 47. (b) 48. (d)

49. (b) 50. (b) 51. (c) 52. (3.18) 53. (1093.02) 54. (c) 55. (b) 56. (d)

57. (d) 58. (250) 59. (1.5) 60. (1) 61. (2.8056) 62. (12) 63. (159.15) 64. (1)

65. (b) 66. (3) 67. (15) 68. (800) 69. (0.798) 70. (413.79) 71. (–1) 72. (b)

73. (44.4) 74. (1.145) 75. (c) 76. (0.5) 77. (6) 78. (d) 79. (b) 80. (c)

81. (c) 82. (b, c) 83. (c) 84. (a)

Solutions
EC O perational Amplifiers

1. Sol. A+ A
By voltage divider rule: Ad =
2
R 100 ( 100)
V– = 2 =1V = 100
R+R =
2
R Q CMRR = 60 dB = 103
V+ = 2 =1V
R+R
Ad
So, Vd = V + – V– = 1 – 1 = 0 V So, = 10 3
Ac
V+ +V 1+1
Vc = = =1V Ad 1
2 2 Ac = =
3 10
R f 100 k 10
A– = = = 100
R1 k Now, Vo = Ac Vc + Ad vd
Rf 100 1
A+ = 1+ × Vo = Ac Vc = × 1 = 100 mV
R1 100 + 1 10

100 k 100
= 1+ = 100
1 101
GATE Previous Years Solved Paper 29

2. (d) When Vi is negative then Vo is positive.


So, diode D1 is reverse bias and diode D2 is
Vo Zf
= ...(For inverting amplifiers) forward bias,
Vi Z1
R2
Vo ( R + j L) Vo = Vi
= R1
Vi 1
R+
j C 4. (d)
L 1 1 Rf 8k
R= and = = 1+ = 1+
C LC R1 2k
L 1 = 1+4=5
+j L
Vo C LC 1
= == 0.2
Vi L 1 5
+
C j 1 A = 45 × 0.2 = 9
C
LC So, A is not very greater than 1.
L L L A 45
+j (1 + j ) So, Af = = = 4.5
C C C 1+ A 1+9
= =
L 1 L L 1
+ 1+ 5. (c)
C j C C j
Rf 90 k
(1 + j ) A1 = = = 90
Vo = R1 1k
1 j
Rf 100
Vo 3 A2 = 1+ = 90.09
= =2 = R1 100 + 1
Vi 2 2 2
A2 90.09 ( 90)
A1
3. (b, c) Ad = = = 90
2 2
Case-1 : Vi > 0 Ac = A1 + A2 = –90 + 90.09 = 0.09
D1
Ad 90
R1 CMRR = = = 1000
Ac 0.09
Vi – Vo
D2 Vo
+ 6. (d)
The open-loop gain of amplifier is very high, so
it will act as a comparator.
When Vi > 0 then Vo is negative.
So, if sinusoidal signal is applied to the input of
So, diode D1 is forward bias and diode D2 is the high comparator, then the comparator
reverse bias. generates the square wave output.
So, Vo = 0 V
Case-2 : Vi < 0 7. ( )

R2
Vo = Gain (Vin)
Vin = Voffset voltage = 1 mV
D1 Rf 1k
R1 Gain = 1 + = 1+ =2
R1 1k
Vi –
Vo Vo = 2 × 1 × 103 = 2 mV
+ D2
30 Electronics Engineering Analog Electronics

8. (c) 11. Sol.


Logarithmic amplifier, To increase the stability of op-amps, frequency
Vin compensation is used in op-amps.
I=
R
12. Sol.
If = I o e vd / VT
Rf
I = I f = I o e vd / VT A– =
R1
Vin 22 k
= I o e vd / VT
A– = = 2.2
R 10 k
Vin
e vd / VT = A = 2.2
RI o

Vd Vin R
Va = V+
= ln R + 15
VT RI o
Vo = –Vd Rf
A+ = 1 +
R1
Vin
Vo = VT ln Rf
RI o Vo Vo
= 1+ =
Va R1 R
V+
9. Sol. R + 15

2k Vo 22 R
= 1+ ×
V+ 10 R + 15
R
– I A+ = 3.2
Vo R + 15
+ 1k
2 mA 3.2 R
2k
= 2.2
R + 15
3.2 R = 2.2R + 33
Vo = –2 × 10–3 × 2 × 103 R = 33 k
= –4 V
Apply KCL at output node, 13. Sol.

Vo 4 S.R.
I + 2 mA = = = 2 mA fmax =
2k 4k 2 vo
I = –4 mA 12 1
fmax = 6
× = 159 kHz
10 2 × 12
10. Sol.
Rf 14. (d)
99
Vo = Vin = ×4
R1 100 The given figure has positive feedback and an
= –0.99 × 4 = –3.96 input at inverting terminal, so the given figure
is Schmitt trigger.
100
Vx = 4 × =2V
100 + 100 15. (d)
Rf 99 Rf
Vo+ = 1+ Vx = 1 + ×2 Vin
R1 100 Vo =
R1
= 1.99 × 2 = 3.98
10 k
= ×2 V = 4 V
Vo = Vo+ + Vo = 3.98 – 3.96 = 0.02 V 5k
GATE Previous Years Solved Paper 31

10 k Vo C C
Vx = Vo × = Here, Vo = V1 sin t V2 sin t
10 k + 100 k 11 C C
Rf = –(V1 + V2) sin t
Vo+ = 1+ Vx
R1 20. (a)
10 k Vo 3Vo This circuit acts as a differentiator and
= 1+ = differentiation of triangular wave gives square
5k 11 11
wave,
Apply superposition principle,
R
Vo = Vo+ + Vo Vo = Vi = sCRVi
1 / sC
3Vo dVi d
Vo = 4 Vo = RC s=
11 dt dt
3 Vo
Vo = –4
11 21. (b)
Vo = –5.5 V

16. (d) –
Vo
When one input terminal of high gain Vi +

comparator circuit is connected to ground and C


a sinusoidal voltage is applied to the other input.
Then the output of comparator will be a square Control
gate Sample and
wave, hold circuit
va = +ve sinusoidal half Vo = +Vsat
va = –ve sinusoidal half Vo = –Vsat 22. (c)

Vi
Vo = Vio A
= 5 mV × 10,000
= 50 V
t
But, Vo = ± 15 V
Vo can never be greater than ±Vsat.

Vo 24. (b)
If open-loop gain is not infinite then,
t 1 Rf 10 k
= = = 10
R1 1k

18. (d) 1
=
10
In positive feedback op-amp act in saturation
region ±Vsat. Here applied voltage is positive, A 100 100
Af = = =
Vo = +Vsat = +15 V 1+ A 1 11
1 + 100 ×
10
19. (c)
Af –9
Rf
Vo = Vin
R1
32 Electronics Engineering Analog Electronics

25. (a) SR
Vm =
10 mH
A 2 fm
10 µF
1
10 = 6
Vs 100 10 × 100 × 2 × 20 × 10 3

1 – Vm = 79.5 mV
Vs = 10 cos(100t) + 2 3
+ Vo
28. (b)

V
KCL at node 1,
4V
V2 L
=
Vs R 2V
t
3
100 × 10 × 10 1 /6 5 /6 2
= =
10 10
–4 V
Vs
V2 = = cos 100t
10
Vp - p = 8 V
Vo 1/ C So, Vi = 4 sin t
V2 = 100 At, Vi = 2
1 1
= 6
= 10 sin t = t=
100 × 10 × 10 × 100 2 6
Vo = –10 V2 5
Another crossover at, =
= –10(–cos100t) 6 6
Vo = 10 cos100t 4
5
Ton 6 6 6 1
26. (b) Duty cycle = = = =
T 2 2 3
20 log x = 20 dB
x = 10 29. (c)
Gain × B.W. = 1 × 106 CMRR = 20 log Ad – 20 log Ac
1 × 106 = 48 dB – 2 dB = 46 dB
B.W. =
Gain 30. (b)
6
10 Rf Rf 8
= = 100 k = 10 5 Vo = ×2 + 1+ ×3
10 R1 R1 9

27. (c) 8
= 5× 2 + 6× =6V
3
Slew rate = A 2 f Vm
V = AVm sin t 31. (c)
dV 250 V2 250 V3 250
= AVm cos t
dt
+ + +
dV V1 1k – 50V1 1k – 50V2 1k – 50V3 V4
= SR = AVm 2 fm
dt max

20 log x = 40 V4
x = 100 = A Av =
V1
GATE Previous Years Solved Paper 33

V4 V3 V2 35. (b)
Av = × ×
V3 V2 V1 Connect a Vs voltage source across inverting
Voltage across 1 k after 1st stage is, terminal of op-amp.
1000 × 50 V1 30 k
V2 =
1250
V2 Ii 10 k
= 40 –
V1 VA
+
Vs
V3 Ideal operational amplifier
Similarly, = 40
V2
Ri
Av = 40 × 40 × 50 = 8 × 104
By virtual short,
Av in (dB) = 20 log(8 × 104) = 98 dB VA = 0 V
33. (a) Vs VA Vs 0 Vs
Ii = = =
10 k 10 k 10 k
Vo
At = , =0 Vs
Vi = 10 k
So, Ri =
and at = 0, Ii

Vo 36. (c)
=1
Vi
1M
– +
34. (a)
IB2
R1 R1 V2 –
Vs eo
V1 +
IB1

R2 VC 1M
V +

R2
iL RL V1 = –IB1 × 1 M
V2 = V1 = –IB1 × 2 M
(Due to virtual ground)
V V V Vo Drop in feedback resistor,
+ + = 1 M = IB2 × 1 M
R2 RL R2 R2
eo = V2 + IB2 × 1 M
2V V Vo eo = –IB1 × 1 M + IB2 × 1 M
+ = ...(i)
R2 RL R2 eo = (IB2 – IB1) × 1 M
Vs V V Vo where, (IB2 – IB1) is offset current.
=
R1 R1
37. (a)
Vs – 2V = –Vo ...(ii)
High pass filter,
Putting Vo from equation (i),
1
Vs + 2 V 2V V c =
RC
= + +
R2 R2 RL 1
= 3 6
V Vs V 1 × 10 × 1 × 10
iL = = = = iL = 1000 rad/sec.
RL R2 RL
34 Electronics Engineering Analog Electronics

38. (c) = –tan–1 RC – tan–1 RC


Vc = 15 (1 – e–t/RC) = –2 tan–1 RC
= 15 (1 – e–1) = 9.45 V Minimum value of =– (at )
Maximum value of =0 (at = 0)
39. (c)
43. (b)
2k

1k I = I o ( eV /VT 1)
X –
y Vo 0 ( 1)
1V + =
100 k
1k
1k V
6 25 × 10 3 1
10 e 1 =
10 5
X = 1 Volt
y = 0.5 V (using voltage division rule) V = 0.06 V

2k 2k Vo V 1
+ y 1+ =
So, Vo = x 4k 100 k
1k 1k
= 1 × (–2) + 0.5 × (3) = –0.5 V Vo 0.06 1
=
4k 100 k
40. (d) Vo = 0.1 V
VI
Vo = VT ln 44. (b)
R1 I s
Vi Vo
2 4 =
V01 –V02 = VT ln + VT ln R1 + Ls R2
R1 I s R1 I s R2C 2 s + 1
4 Vo R2
= VT ln = VT ln 2 =
2 Vi ( R1 + Ls) ( R2C 2 s + 1)
which is equivalent to standard form of transfer
41. (a)
function of low pass filter, i.e.,
From the figure given the question,
a
R1 1/ sC R H(s) = 2
+ Vi 1+ 1 ps + qs + r
Vo = Vi R1 1 R1
+R
sC 45. (d)
Vo 1
= 1+ ×2 10 V
Vi 1 + RsC
5k I
Vo 2 (1 + RsC )
=
Vi 1 + RsC 5V

Vo 1 RsC V
= +
Vi 1 + RsC
– V BE
+
+
42. (c) 1.4 k V
5V
– –
Vo 1 RsC I
=
Vi 1 + RsC
(1 – sRC) – 1 + sCR
GATE Previous Years Solved Paper 35

10 5 48. (d)
I= = 1 mA
5 At = 0, XL = L = 0
V = 1.4 × 1 = 1.4 Volt Hence circuit can be redrawn as below,
V = V + VBE
R1
= 1.4 + 0.6 = 2 Volts
i1
46. (a) –
+
+ vo
R1 VA = 0
– –
Vo
I1 +
I2 R2
vo = 0
Vi At = , XL =
Hence, circuit can be redrawn as below,
R3
R1

i1
Assuming ideal op-amp, voltage at point A is –
zero. So, the given circuit can be considered as +
+ vo
shown below:

R2
v o = R1 i L
R1 Hence given circuit is a high pass filter.

Vo
+ 49. (b)
Vi R3
(O/P) R2 R2 sC
T(s) = = =
(I/P) 1 R1sC + 1
R1 +
Cs
R2
Av =
R1 R2
s
R1
47. (b) T(s) = 1
s+
At, VI = –10 V R1C
Vo 0 0 20 0 ( 10) It is the transfer function of high pass filter with
= +
R 4R R cut-off frequency.
Vo = –5 + 10 1
= 5V = rad/sec.
R1C
At, VI = –5 V
Vo 0 0 20 0 ( 5) 50. (b)
= +
R 4R R Due to virtual short,
Vo = –5 + 5 VC = VB = 0 V
= 0V (Collector voltage of transistor Q)
For VI > –5 V, both diodes are conducting. and given that, base voltage of transistor Q,
So, Vo = 0 V VB = 0 V
So, VC = 0 V
36 Electronics Engineering Analog Electronics

It means collector to base of transistor Q are short cut-off frequency,


circuited. 1 + R2Cs = 0
If any junction of transistor is short-circuited
or, R2Cs = 1
then the junction acts as reverse bias.
So, the C-B junction is reverse bias.
1 1
or, R2 = =
Cs 2 fC
The given op-amp is an inverting configuration
which have positive voltage as input. So, the = 3184.7 or 3.18 k
output voltage of op-amp will be negative
53. Sol.
voltage,
Given circuit is a op-amp series regulator Vo is
Vout = –ve voltage
given by
Emitter voltage of transistor
VE = –ve voltage R1
Vo = 1+ Vz
Given transistor is npn transistor and R2
VB = 0 V
1k
VE = –ve voltage 9V = 1+ 4.7
R
So, the E-B junction will be forward bias. Thus,
R = 1093.02
the transistor is in active region and will behave
as closed switch. 54. (c)
So, VBE = 0.7 V(for silicon transistor)
R2
VB – VE = 0.7 V
VE = VB – 0.7 I
VE = 0 – 0.7 R1 V2 I1

VE = –0.7 V Ri Vo
I V1 +
I2
51. (c)
Output of first op-amp,
IB = I1 = I2
1 1
Vout 1 = 2 +1 1+
1 1 V2 = ( R1 R2 ) I1 ...(i)
= 2 + 1(2) = 4 V KCL at node ‘2’,
1 I + I = I1
Vout = 4 1 + = 4(2) = 8 V
1 0 V2 Vo V2 V2
+ =
R1 R2 Ri
52. Sol.
Vo R1 + R2
0 Vo 0 Vi = V2 ...(ii)
+ =0 R2 R1 R2
Z2 Z1
Q Ri is very high for op-amp.
where, Z 2 = R2 10 nF By using equations (i) and (ii),
Z 1 = R1 = 1 k Vo = I1R2
1
Vo R2 55. (b)
Z2 Cs
or, = = If the resistance R E is increased then A cm
V1 Z1 R1
decreases.
R2
= Ad
R1 ( R2Cs + 1) Hence, CMRR = increases
Acm
GATE Previous Years Solved Paper 37

56. (d) Hysteresis = VTH – VTL


3R R1 R1
= L + L+
R2 R2
R 1
V1 –
Vo R1 R1
+ 500 mV = ( 5) +5
V2 20 k 20 k
2R R
R1
=
2k
R 1 = 500 × 2 × 103 × 10–3
By applying KCL at node, we get
= 1000 = 1 k
V2 V1 V2 V2 Vo
+ + =0
R 2R 3R 61. Sol.
6V2 6V1 + 3V2 + 2V2 PQ1(max) = VCE(max) × IC max ...(i)
= Vo
2 VCE(max) = Vimax – Vo
11 = (24 – 10 V) = 14 V
Vo = 3V1 + V2
2 IE = IC = 200 mA + 0.4 mA
58. Sol. 4 0
ICmax = 200.4 mA IR2 = mA
10
1
I50 = A = I 100 Put values in equation (i), we get,
50
PQ1(max) = 14 × 200.4 × 10–3 Watt
1
Vo = [250(1 + ) 250(1 )] = 2.8056 Watt
100
1 62. Sol.
= × 250 = 0.25 V = 250 mV
1000 V+ > V–
59. Sol. So, Vo = Vsat
= 12 Volts
1k 1k
VOB = 5 1 = 6V
1k 1k 63. Sol.

1k 1 1k 1
5 + 1+ f3-dB = = 159.15 Hz
VOA = 2 RC
1k 2 1k
1 64. Sol.
= –4 V V+ = V
2
10 k Vx 10 k
VOB
= 1.5
VOA R

10 k
60. Sol. Vin –
Vo
Vo +
L+

Apply nodal analysis at inverting terminal,

VTL VTH
V1 Vin 0 0 Vx
=
10 k 10 k
Vin = –Vx ...(i)
L–
38 Electronics Engineering Analog Electronics

Again apply nodal analysis at node Vx , When V1 exceeds 2 V output of op-amp V01 goes
0 Vx Vx 0 Vx Vo to VCC and drives BJT into saturation shorted
= + ...(ii)
10 k R 10 k LED will glow.
Put the value of V x from equation (i) in In the given problem Vi exceeds 2 V three times
equation (ii), we get and hence output V01 of op-amp goes to VCC
R = 1k thrice so that LED glow three times.

65. (b) 67. Sol.


10 k
+VCC V2
10 k 10 k
V4
R IC VB = VA
+
Vref 10 k +VCC
VB VN –
– VB 1k Vo
– +
V1
+ –VCC
VA R2 1k
V3 VB
R1 IC = IE RL 1k 1k
VN – 1

Node A:
VA = VCC – Vref VA V1 VA V3 V VN 1 VA
VB = VA (since virtual short) + + .... A + =0
1k 1k 1k 1k
VCC VB
IC = N
R VA + 1 = V1 + V3 + ... + VN – 1
2
VCC (VCC Vref ) Vref VB = VA (Q Virtual short)
= =
R R Node B:
IC Vref VA V2 VA V4 V VN VA Vo
Io = I E = = + + .... A + =0
1+ R 10 k 10 k 10 k 10 k
N
66. Sol. Vo = VA +1 (V2 + V4 + V6 + ... + VN )
2
10 V 10 V N (V1 + V3 + ... + VN 1)
= +1
2 N
+1
100 2
8k
–(V2 + V4 + ... + VN)
+VCC
= V1 – V2 + V3 – V4 + ...
VB – V01
1 1 1
Vi + = 1+ + + ...
15 k 2 3 4
2k
–VCC 1
= =
N
Output of op-amp goes to saturation
10 V + 2 k Vo = Vsat = VCC
VB = =2V
8k +2k = 15 V
GATE Previous Years Solved Paper 39

68. Sol. Let at t = T1,


1M VB exceeds VA(–1 V) so that V01 changes from
–10 V to 10 V.
+VCC
Vo charges from –5 V to 5 V

0V Vo VB = Vt + (Vi – Vf ) e–t/
+
= –10 + [10 – (10)] e–t/RC
–VCC 10 k At t = T1, VB = –1
1M
–1 V = –10 + 20 e–T1/RC
VCC
20
T1 = RC ln
Generated photo current 9
Responsivity =
Incident light power = 10 × 10 × 100 × 10–6 × 0.798
3

Io = 0.798 sec.
=
Pi
70. Sol.
Io Overall input = Vios + Vinput
0.8 A/W =
10 × 10 6 = 5 mV + 25 mV = 30 mV
Io = 8 µA
Rf
Vo = Io × 1 M 1+
R1
= 8 × 10–6 × 1 × 106 = 8 V Vo = × Overall input
1 Rf
The photo current through load RL = 10 k is 1+ 1+
AOL R1
given by
Vo 8 15 k
IL = = 1+
RL 10 × 10 3 1k
× 30 × 10 3
=
1 15 k
= 800 µA (in upward direction) 1+ 1+
100 1k
69. Sol.
= 413.79 mV
Initially switch is closed and VB = 10 V
V01 = –10 V 71. Sol.
V0 = –V2 = –5 V V01 R
H1(s) = = 1+ 1
Vo Vi sL
VA = ×1 k = –1 V
4 k +1k 1 1
= 1+ = 1+ 6
10 V sL / R1 s × 10
V0 R3 sR3C
H2(s) = = =
10 µF V01 R2Cs + 1 1 + sR2C
+10 V Cs
– 470
VB Vo Cs 1
+ V01 = =
10 k 1 + Cs 1 + 1
VA –10 V Cs
–10 V 4k H(s) = H1(s) H2(s)
1k
6
1 s × 1 × 10
= 1+ 6 6
s × 10 1 + s × 10
At t = 0;
= –1
The switch is opened and as t , VB
approaches –10 V.
40 Electronics Engineering Analog Electronics

72. (b) 74. Sol.


+15 V 20 k
Vin –
V out 5k
+
Vp –
–15 V 31I a Vo
Vx +
V1
I1
10 k
5k
20 k
3V KCL at a,
3 × 10 + Vo × 5 6 + Vo Vo Vx Vx0.7
V1 = = =
15 3 20 5
Vo – Vx = 4Vx – 2.8
6 + 15
VUT = =7 V Vo = 5Vx – 2.8 ...(i)
3
Now, I1 = 31 I
6 15
VLT = = 3V Is eVx /VT = 31 Is eVP /VT
3
Vx V
73. Sol. = ln 31 + P
VT VT
• In the given circuit: Vx VP
= ln 31
Feedback factor, VT
R1 1 Vx = 0.789 V
= =
R1 + R2 80 From equation (i),
Vo = 5 × 0.789 – 2.8 = 1.145 V
Gain
Without feedback 75. (c)
5
A0 : 10 For t > 0,
With feedback 1 µF
+ – +
V1 +
I
Aof : 80 2.5 V 1 µF V2
1k
1V – – –
0V
Af + vOUT(t )

f
fc fc 15 k
1V
I= = 1 mA
Ao 1k
• Aof = 80
1 + Ao The capacitor charges with constant current I
and both V1 and V2 will increase till V2 reaches
10 5
• f c = f c (1 + Ao ) = 8 1 + Hz = 10008 Hz 2.5 V. Thereafter, V2 = 2.5 V and V1 increases
80
with time.
• Gain at f = 15 kHz = 15000 Hz is When, vout(t) = –10 V
Aof 80 V1 = 7.5 V
Af = = 44.4 t
2
f 15000 2 1
1+ 1+ So, (1 mA) dt = 7.5 V
fc 10008 1 µF
0
103t = 7.5
t = 7.5 msec
GATE Previous Years Solved Paper 41

76. Sol. 78. (d)


Applying the concept of virtual ground, we get 1k
R2
Vo = Vin
R1 1k +5 V
Vi –
[Q Non-inverting amplifier] Vo
+
31 k
Vo = ×1 V –5 V
1k
Vo = –31 V < –15 V Given circuit is a Schmitt trigger of non-inverting
which is not possible. type,
Hence, the output voltage of the op-amp is equal Vo = ± 5 V
to –15 V. Vo + 1 + Vi × 1 Vo + V
V+ = +
1+1 2
31 k
Let, Vo = –5 V
+15 V
1k 5 + Vi

V+ =
A
2
+ + +
1V Vo = –15 V Vo can change from –5 V to +5 V if V + > 0 i.e.
– –
–15 V 5 + Vi
>0 Vi > 5 V.
2
Now applying KCL of node ‘A’, we get,
Similarly Vo can change from +5 V to –5 V if
VA ( 15) VA 1
+ =0 Vi < –5 V.
31 k 1k
But given input has peak value 1 V. Hence
VA V 15 1 output cannot change from +5 V to –5 V or –5 V
+ A = +
31 k 1k 31 k 1k to +5 V.
1 1 15 Output remain constant at +5 V or –5 V.
VA + = +1 Correct answer is option (d).
31 1 31
VA = 0.5 V 79. (b)

77. Sol. Op-amp active filter (LPF) inverting type 3-dB


cut-off frequency,
1k
1 1
fc = =
2 RC 2 × 2 × 10 3 × 10 6
1k
– Io 500
Vo = = 79.58 Hz
2V + 2
1k
80. (c)

Vo = (1 + 1) × 2 = 4 V Rin
RF

2 4 0 4
+ Io + =0 (KCL at node Vo) V01
1k 1k V1

–2 + Io – 4 = 0 R1
Io = 6 mA Vref R2
42 Electronics Engineering Analog Electronics

Rf Rf Step-1 : KVCL at V2,


Vref R2
V01 = 1+ Vx = 1 + Iin + IF = Iref ...(1)
Rin Rin R1 + R2
Vin V2 Vout V2 V2 + Vref
+ = ...(2)
R Rf R
RF
Rin Step-2 :
Vin
Q V1 = 0 V2 = 0
V 02
Vin Vout Vref
R1 + =
R Rf R
R2
Vout Vref Vin Vref Vin
= =
Rf R R R

Rf Rf
V02 = Vin Vout = (Vref Vin ) ...(3)
Rin R
Step-3 :
Total, V0 = V01 + V02
For, Vout = 0
Rf R2 Rf
V0 = 1+ Vref + Vin Rf
Rin R1 + R2 Rin (Vref Vin ) = 0
R
Given,
Vref –Vin = 0
Rf R2 Rf Vin = Vref ...(4)
1 = 1+ Vref (0.1)
Rin R1 + R2 Rin
82. (b, c)
...(1)
Given circuit is a high-pass filter.
Rf R2 Rf
6 = 1+ Vref (1) R2 = 2 k
Rin R1 + R2 Rin
...(2) +15 V
1 µF R1 = 1 k
Rf
(1 0.1) +
(2) – (1) 6–1 = Vin +
Rin Vo
– –15 V –
Rf 5
= = 5.555
Rin 0.9
1 1
Rf = =
c R1C1 10 3 × 10 6
= 5.555
Rin
= 1000 rad/sec
Hence, option (b) and (c) is correct.
81. (c)
83. (c)
Given: Vin << VCC ; Vref << VCC

R V2 RF
15 V
Iin IF VCmax
R Iref
+VCC t
Vin VCmin
Vref V1 Vout
–12 V
–VCC
GATE Previous Years Solved Paper 43

If, V0 = +15 V
R
D1 is ON.
R2
D2 is OFF. 1k

R 1k Vo
Vi
+15 V R3
C

C Vo

–12 V Maximum gain = 12 dB


R R R R 20 × log Amax = 12
Amax = 4
R2
1+ =4 R2 = 3R1
Capacitor charge upto VUT, R1
15 × R R = 3×1=3k
VCmax = VUT = =5V
R + 2R log10 c = 3
If, V0 = –12 V c = 1000 rad/sec
D1 is OFF. 1 1
D2 is ON. c = C=
R3C R3 × c
Capacitor discharge upto VLT,
1
R C= = 1 µF
1000 × 1000
+15 V 85. (–0.5)

Vo Analog output,
C
Vo = –Resolution × Gain ×
–12 V
Decimal equivalent of
R R R R binary data
vr 1.6
Resolution = n
= = 0.4
2 24
12 × 2 R Decimal equivalent = 5
VC min = VLT = = 8V
2R + R Gain = 1
VCmax – VCmin = 5 – (–8) = 13 V vo = –(0.1) (5) (1)
= –0.5 V
84. (a)

20 log A

12
9

3 log10( )
44 Electronics Engineering Analog Electronics

Answers
EE O perational Amplifiers

1. (c, d) 3. (1) 5. (a) 6. (a) 7. (b) 9. (d) 10. (d) 11. (a)

12. (d) 13. (d) 14. (b) 15. (d) 16. (c) 17. (d) 18. (a) 19. (d)

20. (c) 21. (b) 22. (c) 23. (d) 24. (b) 25. (d) 26. (d) 27. (d)

28. (b) 29. (d) 30. (d) 31. (b) 32. (b) 33. (c) 34. (b) 35. (a)

36. (d) 37. (c) 38. (d) 39. (d) 40. (1.245) 41. (b) 42. (a) 43. (a)

44. (a) 45. (d) 46. (a) 47. (c) 48. (b) 49. (a)

Solutions
EE O perational Amplifiers

2. Sol. Vi(volts)

– D1
Vo t(sec)
Vi + VA 0.5 1 2
C D2 Vout

1V

For 0 < t < 0.5 s


t(sec)
Capacitor get charged upto 1 Volt. D1 is forward
bias and D2 is reversed bias.
3. Sol.
2R

Vi + Vo = 1 V R V
+ –
1V R Vo
– V +
2V
R
–1 V
There is no discharging path providing for this R
capacitor. It will remain charged with 1 V.

2 V 1 V V
– + =
Vo R R R
Vi +
+ 1
1V V = Volts
– 3
0 V V Vo
=
R 2R
Vo = 3 V = 1 Volt
GATE Previous Years Solved Paper 45

4. Sol. 9. (d)

100 k
a –
– Vo
Vo + +
Vi + Vs
b
– 1k
Here, Vi = Vo 90 k
10 k
Vcm = Va + Vb
1
= (Vi + Vo ) = Vi = Vo
2 I2 = 0, I1 = I
V+ = 10 k × I
5. (a)
V0 = (90 + 10) k × I
2R = 100 k × I
x = = 2
R V+ 10 k × I 1
= = =
R V0 100 k × I 10
x = = 1
R
10. (d)
x 1 1
= x = x
x 2 2 1k

6. (a)

V0
2R +
1 mA

R

V+ + V0
–2 V0 = –1 mA × 1 k
R = –1 V
2 + sin100t
R
11. (a)
2 V+ 2 + sin 100t V+ S.R. = Vm
+ =0
R R 65.8 V/µs = 2 f × 10
1 62.8 × 10 6
V+ = sin 100t f=
2 2 × 10
0 V+ V+ V0 = 1 MHz
=
R 2R
12. (d)
3
V0 = 3 V+ = sin 100t 1
2 Vin ×
jC Vin
8. Sol. V(+) = =
1 1 + jRC
R+
(A) (R) : There is no feedback. jC
(B) (S) : Input impedance is resistive and where, V(–) = V(+) (For ideal op-amp)
feedback is capacitive in nature.
Vin V( ) V( ) V0
(C) (P) : Input impedance is capacitive and =
feedback is resistive in nature. R1 R1
V0 = 2V(–) – Vin
= 2V(+) – Vin (V(–) = V(+))
46 Electronics Engineering Analog Electronics

2 100 k
= 1 Vin But, Vy = 1+ Vx = 11Vx
1 + j RC 10 k

1 j RC 10 Vx Vx
= Vin Ix = = 100 k
1 + j RC 1M Ix

V0 15. (d)
= 2 tan–1 RC
Vin V0 Rf
=
For –90° 90°, maximum phase shift occurs Vin R1
(±180°). If, gain = –25
13. (d) 1000 k
then, R1 = = 40 k
Using KCL at node 1 we have, 25
If, gain = –10
10 k 1 10 k 1000 k
then, R1 = = 100 k
1k
10
1k
Vi So, if we keep R1 to be 100 k then we never get
the gain –25 for any RF so we can keep R1 to be
– 40 k .
Vo
+
16. (c)
Output will be at its saturation values and it is
having a phase difference of (–180°).
V1 0 V1 V1 Vout
+ + =0 For +ve half of the input 1st diode will be on
10 1 10
making –ve terminal of op-amp to 0.7 V larger
12V1 = Vout ...(i)
than the voltage at +ve input so output will be
Also, using KCL at inverting node, we get
–Vsat.
Vin 0 0 V1 For –ve half of input reverse of above will
=
1 10 happen and output will go +Vsat.
V1 = –Vin × 10 ...(ii)
From equation (i) and (ii) we get, 17. (d)
Vout For –ve half of the input +ve terminal of op-amp
= –120 is at higher potential than –ve terminal and
Vin
output goes to +V sat but due to this high
14. (b) potential diode gets on and restricts the output
to 0.7 V only. And for +ve half of the input +ve
100 k
terminal of op-amp is at lower potential than
–ve terminal’s potential and output goes to –Vsat
10 k and remains at –Vsat.

Vy
+ 18. (a)
0A Output will be either at +Vsat or –Vsat. When
Vx output will be at +Vsat diode connected to 10 k
Ix Ix R3 = 1 M resistance will be on making voltage at point P
equal to 6 V.
Vx Vy
Here, Ix = When output is at –Vsat diode connected to 2 k
R3 resistance will be on making voltage at point P
equal to –10 V.
GATE Previous Years Solved Paper 47

19. (d) R
It behaves as current source because the output
V01 R
current (I0) depends upon (Vin) and resistance Vi P –
only. Vo
+
Q
V × R2 V02 R
R1 + R2 R2 V R
Where, I0 = =
r R1 + R2 r
Vo will be positive due to non-inverting action.
20. (c)
So, output is always rectified.
Let the voltage at the non-inverting terminal of
the op-amp be Va volts and the voltage at 22. (c)
inverting terminal be Vb volts.
R2
At t = 0+ switch is open V0 = 5 V and Vb = –10 V
V0 × 10 k 50
Va = = Volts
100 k + 10 k 11 R1 C
Vi –
At t , VA Vo
+
Vb 10 V
R3
Vb = Vf + (Vi – Vf ) e–t/RC ...(i) R4

50
Putting, Vb = at t = T1
11
Vi = –10 V R4
VA = × Vi
and Vf = 10 V in equation (i), R4 + R3
we get, T1 = 12.98 µ-sec RB V
VA = × Vi = i
At, t = 12.98 µ-sec RB + RB 2
V0 changes from 5 V to –5 V Let, Vi only on inverting terminal,
21. (b) Rf
V01 = Vi
For (Vin > 0), R1
P V01 = Negative 1 R2
Q V02 = 0 Here, R f = R2 =
sC 1 + sCR2
R
Let, Vi only on non-inverting terminal,

V01 R Rf Vi
Vi P – V02 = 1+
Vo R1 2
+
Q V0 = V01 + V02
V02 R
R Rf Vi
= 1
R1 2
Putting the value of Rf , we get,
V0 will be positive due to inverting action.
For (Vin < 0), R2 Vi
V0 = 1
P V01 = 0 R1 (1 + sCR2 ) 2
Q V02 = Positive Here, (R1 = R2 = RA)
48 Electronics Engineering Analog Electronics

1 Vi 26. (d)
V0 = ×
1 2 For emitter follower:
1+
sCR A Av 1; AI is high
So, it is a high pass filter. Ap = Av AI is high Pout > Pin
For transformer,
23. (d) Pin = Pout
(sCR A ) V
Vin = × i 27. (d)
1 + (sCR A ) 2
1k +6 V
1/ sC 1 Vi –
V0 = 1 R Vin = Vin Vo
C
+ A 1 + sRA V1 +
sC 2 2 –3 V 2k

sRAC
1
V0 = V
C (1 + sR AC ) in
1 + sRA 1k
2
Which is similar to equation of a band pass
filter. when Vi < V1 (upto t2);
V0 = +ve
24. (b) when Vi > V1 (t2 t t4);
Voltage-series feedback arrangement or voltage- V0 = –ve
voltage feedback.
1 1
V1 = V0 = V0
25. (d) 1+ 2 3

Is R = 10 k 28. (b)

2R
V0 = 1 + V = (1 + 2) (2) = 6 V
Vs +
R
Vs Vo
– 29. (d)

Vs
High pass filter
R = 10 k Los pass filter
C = 10 µ F

Vs V0
Is = ...(i)
R
1/ j C 20 Hz 30 Hz
Vs = V0 ...(ii)
1
R+ Pass band
j C
Is = –Vs j C It is a band pass filter.
= –j(10 × 2 × 50 × 10 × 10–6)
30. (d)
= –j10 .... mA
First section is differential amplifier having
= 10 mA lagging by 90°
gain –1.
GATE Previous Years Solved Paper 49

31. (b)
Vo

R2
+12 V

C R1 Vo
12 V Vi –
Vi
–12 V
+

–12 V

Vin 0 0 V0
Rf =
1 R2
R1 +
SC
R1
V1 –
Vo V0 R2
V2 +
=
R2 Vi 1
R1 +
Rg sC
It is a high pass filter with
1
f3-dB = rad/sec.
Output is, R1C
R f + R1 Rg Rf
Vo = V2 V1 32. (b)
R g + R2 R1 R1

Vo = V2 – V1 = –Vi
Q
Second stage-schmitt trigger,
1k +15 V
– –
V V out
Vo +
+ 5V
–15 V

Vt
Rf
Rg Using the concept of virtual ground, V = 0

Rg V0
Vt = ± V0 = =±6V Q
R f + Rg 2

Vo
Vout

+12 V Vout = –0.7 V

33. (c)
Vi Output of first op-amp,
–6 V +6 V
1 1
Vout 1 = 2 +1 1+ = 2 + 1(2) = 4 V
1 1
–12 V
1
Vout = 4 1 + = 4(2) = 8 V
1
50 Electronics Engineering Analog Electronics

34. (b)
Op-amp ‘3’ circuit is a differential amplifier so,
V0 = V01 – V02 ...(i) Corner
frequency
Now, apply KCL at node ‘2’,
V2 V1 V2 V02 0 10 10
2
10
3
0 10 10
2 3
10
+ =0 ...(ii) 1 1
2R R
–10 – /4
and apply KCL at node ‘1’,
–20 – /2
V1 V2 V1 V01
+ =0 ...(iii) –30
–20 dB/dec
2R R
From equation (i), (ii) and (iii), we get,
V0 = 2(V1 – V2)
36. (d)
35. (a) The circuit of op-amp ‘1’ is a Schmitt trigger,
Rf therefore,
V01 = ±Vsat
and the circuit of op-amp ‘2’ is a non-inverting

R=1k Vo amplifier.
Vi +
V0 R
1 µF = 1+
V01 R
V0 = 2V01
where, V01 = ±Vsat
1k
– Therefore, the answer is V0 = ±Vsat.
Vo
Vi +
37. (c)
1 µF
Case-I : Vi > 0, the circuit will look like.
Buffer

R
R +Vsat
Low pass vi – R

+
This filter is considered as low-pass filter (LPF). +
–Vsat vo
The transfer function for LPF, R
R
V0 1/ sC 1
= =
Vi 1 1 + sCR
R+
SC
Hence, V0 = 0
1 1 Case-II : Vi < 0, the circuit will look like.
= =
1 + j (10 3 ) (10 6
) 1 + j (10 3
) R

1000 1
= = R
1000 + j s
1+ +Vsat
1000 R
vi – R +Vsat
c (corner frequency) = 103 rad/sec. –
+ V01
The gain at low frequencies, + vo
–Vsat
1 1 R –Vsat
Av = = =1 R
1 + sCR 1
GATE Previous Years Solved Paper 51

V01 R 39. (d)


= ...(i)
Vi R The circuit is,
V01 = –Vi ...(ii) C = 1 µF

V0 R
and = R = 1000
V01 R

V0 = –V01 ...(iii) VA Vo
Vi +
From equation (ii) and (iii), A = 1000

V0 = Vi

38. (d) Now, the gain of op-amp is 1000.


The circuit is, So, V0 = 1000(V+ – V–)
Since, V+ = 0 (grounded)
I R
V– = VA
The above circuit can be redrawn as,
C
– C = 1 µF
B
Vi Vo
A 1000
+
‘A ’ Vo
I C VA

I R Vi –1000 VA

So, V0 = –1000 VA
Since, circuit has negative feedback, so with help
Now, KCL at node ‘A’,
of virtual short VA = VB.
VA Vi VA V0
So, KVL in the loop from A to B given, =
1000 1 / SC
I I
I Vi + I =0 VA Vi VA ( 1000 VA )
sC sC =
1000 1/SC
Vi (sC )
So, I= VA – Vi = (1001 VA) sC
2
VA – Vi = (1001) × 1000 VA sC
So, VA = –IR
VA – Vi = s (1001000 × 10–6) VA
sCRVi VA – s1.001 VA = Vi
VA =
2
Vi
and VA = VB VA =
1 s(1.001)
sCRVi
so, VB = Since, V0 = –1000 VA
2
1000
VB V0 = Vi
Now, =I 1 s(1.001)
R
So, V0 = VB – IR Since, pole is at (1/1.001), so time constant is
approx ‘1’.
Vi (sCR ) Vi (sCR )
V0 =
2 2 40. Sol.
= –Vi (sCR) Vi = 2 sin(2 × 2000t)
So, V0 = –j RCVi The transfer function of the system is,
So, V0 lag Vi by 90° or phase or V0 w.r.t. Vi is –90°.
1000
H(s) =
(1000 × 0.1 × 10 6 s + 1)1000
52 Electronics Engineering Analog Electronics

1 44. (a)
H(s) = 4
(10 s + 1) R2
The input is 2 sin(2 × 2000t)
= 4000
V0 = 2 H ( j ) = 4000 × sin(4000 t ) R1
C
Vi –
Output = 1.245 sin(4000 t – 51.46°) Vo
So, amplitude of output is 1.245. +

41. (b)
To check the type of system:
1 1
We apply a delta function at input Vi = (t) R2 R2 +
Vout j C j C
R/2 =
Vin R1
R
F1 –
Vo Vout R2
Vi + =
A Vin R1 ( R2 j C + 1)
F2
R So the system is a low pass filter.
So, voltage at A will be same as voltage at output
45. (d)
V0. VA will be equal to voltage due to F1 + voltage
due to F2. Since, f1 < f2, so VA/Vi will be 9

VA /Vi 1 VB
V3 –
1 Vout
VA
V1 +

f 4
f1 f2

V0/Vi will be V2

Vo/Vi 4 1
VA = V1 + V2
5 5
Vout = –9 V3 + 10 VA
= –9 V3 + 8 V1 + 2 V2
f 46. (a)
f1 f2

So, the system work as a band stop filter. +V ss


Vin +
42. (a) VA

D
The given circuit in a astable multivibrators so –Vss
Vo
output will be a periodic square wave and from
the circuit = 0.5. R

1+
So, time period will be 2 ln ,
1
Vin > 0 VA = +VSS, D on, V0 = Vin
1 + 0.5 Vin < 0 VA = –VSS, D off, V0 = 0
T = 2 × RC ln = 0.55 ms
1 0.5
GATE Previous Years Solved Paper 53

Vo 1 1 V0
Vin + =
R2 R1 R1
V in
=
Vo
R1 + R2
Vo = 0
V in
Vin × R1 = V0
R1 R2

R1 + R2
V0 = Vin × ...(ii)
R2
Equation (ii) in equation (i),
47. (c) Vin V0
= iin
I3 R VB R I1 Z
R1 + R2
I2 R Vin Vin
R2
I3 R = iin
– Z
VA Vo
+ I1 Vin R1 + R2
2R
VA 1 = Z
iin R2
Vs Vin R2 R1 + R2
2R
iin R2 = Z
Vs
VA = Vin R2
2 = Z
iin R1
VA Vs
I3 = = ...(i)
R 2R 49. (a)
VB – VA = I 3 R
100 k
V V
VB = VA + I 3 R = s + s = Vs
2 2
10 k
VB Vs V3 = 10 mV
I2 = = ...(ii) –
R R Vo
I1 = I2 + I3 V2 = 50 mV +
10 k
Vs Vs Vs
= + = [1.5] R2 = 100 k
R 2R R
V0 – VB = I 1 R
V
V0 = VB + s [1.5] R R2
R V0 = (V2 V1 )
R1
= Vs + 1.5 Vs = 2.5 Vs
100 k
48. (b) = 10 k (50 mV 10 mV)
According to virtual ground,
= 10 (40 mV) = 400 mV
VA = VB = Vin
At node A,
Vin V0
= iin ...(i)
Z
2 Diodes Applications

ELECTRO NICS EN GINEERIN G Vo

(GATE Previous Years Solved Papers)


(d) 5.9
Q.1 The 6 V Zener diode shown below has zero 0
Zener resistance and a knee current of 5 mA.
The minimum value of R. So that the voltage [EC-1993 : 1 Mark]
across it does not fall below 6 V is Q.3 The Ebers Moll model is applicable to
50 I (a) Bipolar junction transistors
(b) NMOS transistors
10 V 6V R 6V (c) Unipolar junction transistors
(d) Junction field effect
[EC-1995 : 1 Mark]
(a) 1.2 k (b) 50
Q.4 A Zener diode in the circuit shown in below
(c) 80 (d) 0
figure has a knee current of 5 mA, and a maximum
[EC-1992 : 2 Marks]
allowed power dissipation of 300 mW. What are
Q.2 The wave shape of Vo in figure is the minimum and maximum load currents that
4.1 V 4.1 V can be drawn safely from the circuit, keeping the
output voltage Vo constant at 6 V?
50
10 sin314t 10 k Vo +
+ +
9V Load


Vo –

5.9
(a) 0 mA, 180 mA (b) 5 mA, 110 mA
(a)
0 (c) 10 mA, 55 mA (d) 60 mA, 180 mA
–5.9 [EC-1996 : 2 Marks]

Vo Q.5 For small signal ac operation, a practical


forward biased diode can be modeled as
4.1
(a) a resistance and a capacitance.
(b)
0 (b) an ideal diode and resistance in parallel.
–4.1 (c) a resistance and an ideal diode in series.
(d) a resistance.
Vo
[EC-1998 : 1 Mark]
4.1
Q.6 For full wave rectification, a four diode bridge
(c)
0 rectifier is claimed to have the following
advantages over a two diode circuit:
–4.1
GATE Previous Years Solved Paper 55

1. Less expensive transformer R


2. Smaller size transformer and +
IZ IL = 10 mA
3. Suitability for higher voltage application of
these Vin Vo
DZ
(a) only (1) and (2) are true.

(b) only (1) and (3) are true.
(c) only (2) and (3) are true. (a) R 1800
(d) (1), (2) as well as (3) are true. (b) 2000 R 2200
[EC-1998 : 1 Mark] (c) 3700 R 4000
(d) R > 4000
Q.7 A dc power supply has a no-load voltage of 30 V,
and a full load voltage of 25 V at a full load [EC-2002 : 2 Marks]
current of 1 A. Its output resistance and load Q.10 The circuit shown in the figure is best described
regulation, respectively are as a
(a) 5 and 20% (b) 25 and 20%
(c) 5 and 16.7% (d) 25 and 16.7%
[EC-1999 : 2 Marks]
Output
Q.8 The transistor shunt regulator shown in the
figure has a regulated output voltage of 10 V,
when the input varies from 20 V to 30 V. The
(a) bridge rectifier
relevant parameters for the Zener diode and the
transistor are: VZ = 9.5, VBE = 0.5 V, = 99. (b) ring modulator
Neglect the current through R B. Then the (c) frequency discriminatory
maximum power dissipated in the Zener diode (d) voltage doubler
(PZ) and the transistor (PT) are: [EC-2003 : 1 Mark]
20 Q.11 The output voltage of the regulated power
IZ IC supply shown in the figure is

VZ
Vin = 20-30 V Vo = 10 V +
+
VBE – 1k
RB
+
15 DC
Vz = 3 V
unregulated –
(a) PZ = 75 mW, PT = 7.9 W power source 40 k
(b) PZ = 85 mW, PT = 8.9 W Regulated
20 k
(c) PZ = 95 mW, PT = 9.9 W DC output

(d) PZ = 115 mW, PT = 11.9 W
[EC-2001 : 2 Marks] (a) 3 V (b) 6 V
Q.9 A Zener diode regulator in the figure is to be (c) 9 V (d) 12 V
designated to meet the specifications: IL = 10 mA, [EC-2003 : 2 Marks]
Vo = 10 V and Vin varies from 30 V to 50 V. The
Q.12 In the voltage regulator shown in the figure, the
Zener diode has Vz = 10 V and Izk (knee current)
load current can vary from 100 mA to 500 mA.
= 1 mA. For satisfactory operation.
56 Electronics Engineering Analog Electronics

Assuming that the Zener diode is ideal (i.e., the Q.15 For the circuit shown below, assume that the
Zener knee current is negligibly small and Zener Zener diode is ideal with a breakdown voltage
resistance is zero in the breakdown region), the of 6 Volts. The waveform observed across R is
value of R is 6V
R
+

+ 12 sin t R VR
Variable load
12 V 5V 100 to 500 mA
– –

6V
(a) 7 (b) 70 (a)
70
(c) (d) 14 6V
3
[EC-2004 : 2 Marks] (b)
Q.13 In a full wave rectifier using two ideal diodes,
Vdc and Vm are the dc and peak values of the –12 V

voltage respectively across a resistive load. If


12 V
PIV is the peak inverse voltage of the diode, then
the appropriate relationship for this rectifier are (c)
Vm
(a) Vdc = , PIV = 2 Vm
–6 V
V
(b) Vdc = 2 m , PIV = 2 Vm (d)
–6 V
Vm [EC-2006 : 2 Marks]
(c) Vdc = 2 , PIV = Vm

V Common Data for Questions (16 and 17):


(d) Vdc = m , PIV = Vm
A regulated power supply, shown in figure below, has
[EC-2004 : 2 Marks] an unregulated input (UR) of 15 Volts and generates a
regulated output Vout. Use the component values
Q.14 The Zener diode in the regulator circuit shown shown in the figure.
in the figure has a Zener voltage of 5.8 Volts
15 V(UR)
and a Zener knee current of 0.5 mA. The Q1
maximum load current drawn from this circuit
ensuring proper functioning over the input 12 k 10
1k
voltage range between 20 and 30 Volts, is
1k +

Vi V1 = 5.8 V Load
20-30 6V 24 k

(a) 23.7 mA (b) 14.2 mA


(c) 13.7 mA (d) 24.2 mA Q.16 The power dissipation across the transistor
[EC-2005 : 2 Marks] shown in the figure is
GATE Previous Years Solved Paper 57

(a) 4.8 Watts (b) 5.0 Watts Q.19 For the Zener diode shown in the figure, the
(c) 5.4 Watts (d) 6.0 Watts Zener voltage at knee is 7 V, the knee current is
[EC-2006 : 2 Marks] negligible and the Zener dynamic resistance is
10 . If the input voltage (Vi) range is from 10 to
Q.17 If the unregulated voltage increases by 20%, the 16 V, the output voltage (Vo) ranges from
power dissipation across the transistor Q1
200
(a) increases by 20%
+
(b) increases by 50%
(c) remains unchanged Vi Vo
(d) decreases by 20%

[EC-2006 : 2 Marks]

Q.18 The correct full wave rectifier circuit is (a) 7.00 to 7.29 V (b) 7.14 to 7.29 V
(c) 7.14 to 7.43 V (d) 7.29 to 7.43 V
[EC-2007 : 2 Marks]

Q.20 In the following limiter circuit, an input voltage


(a) Input Vi = 10 sin100 t is applied. Assume that the
Output diode drop is 0.7 V when it is forward biased.
The Zener breakdown voltage is 6.8 V.
1k

D1

Vi D2 Vo
Z 6.8 V
(b) Input

Output
The maximum and minimum values of the
output voltage respectively are
(a) 6.1 V, –0.7 V (b) 0.7 V, –7.5 V
(c) 7.5 V, –0.7 V (d) 7.5 V, –7.5 V
[EC-2008 : 1 Mark]

Input Q.21 In the circuit below, the diode is ideal. The


(c)
voltage V is given by
Output
1 1
+ V –

Vi 1A

(d) Input

Output
(a) min (Vi , 1) (b) max (Vi , 1)
(c) min (–Vi , 1) (d) max (–Vi , 1)
[EC-2009 : 2 Marks]
[EC-2007 : 1 Mark]
58 Electronics Engineering Analog Electronics

Statement for Linked Answer Questions (22 and 23):


In the circuit shown below, assume that the voltage
drop across a forward biased diode is 0.7 V. The thermal 100
voltage Vt = kT/q = 25 mV. The small signal input ILoad
10 V
vi = Vp cos( t) where Vp = 100 mV.
9900 VZ = 5 V RL

12.7 V

IDC + IAC VDC + VAC


(a) 125 and 125 (b) 125 and 250
Vi (c) 250 and 125 (d) 250 and 250
[EC-2013 : 2 Marks]

Q.26 A voltage 1000 sin t volts is applied across YZ.


Assuming ideal diodes, the voltage measured
across WX (in Volts), is
Q.22 The bias current IDC through the diodes is
(a) 1 mA (b) 1.28 mA
(c) 1.5 mA (d) 2 mA 1k
[EC-2011 : 2 Marks] W Y X

Q.23 The ac output voltage vac is Z


(a) 0.25 cos( t) mV (b) 1 cos( t) mV
(c) 2 cos( t) mV (d) 22 cos( t) mV
1k
[EC-2011 : 2 Marks] + –

Q.24 The diodes and capacitors in the circuit shown


are ideal. The voltage v(t) across the diode D1 is (sin t + sin t )
(a) sin t (b)
2
v (t )
(sin t sin t )
C1 D2 (c) (d) 0 for all t
2
+
cos( t) [EC-2013 : 2 Marks]
D1 C2
– Q.27 In the figure, assume that the forward voltage
drops of the PN diode D1 and schottky diode D2
are 0.7 V and 0.3 V, respectively. If ON denotes
conducting state of the diode and OFF denotes
(a) cos( t) – 1 (b) sin( t) non-conducting state of the diode, then in the
(c) 1 – cos( t) (d) 1 – sin( t) circuit.
[EC-2012 : 1 Mark]
1k 20
Q.25 In the circuit shown below, the knee current of
the ideal Zener diode is 10 mA. To maintain 5 V
10 V D1 D2
across RL, the minimum value of RL in and
the minimum power rating of the Zener diode
(in mW) respectively, are
GATE Previous Years Solved Paper 59

(a) Both D1 and D2 are ON. (a) –0.3 V < Vi < 1.3 V
(b) D1 is ON and D2 is OFF. (b) –0.3 V < Vi < 2 V
(c) Both D1 and D2 are OFF. (c) –1.0 V < Vi < 2.0 V
(d) D1 is OFF and D2 is ON. (d) –1.7 V < Vi < 2.7 V
[EC-2014 : 1 Mark] [EC-2014 : 1 Mark]

Q.28 The diode in the circuit shown has Von = 0.7 V Q.31 For the circuit with ideal diodes shown in the
but is ideal otherwise. If Vi = 5 sin( t) Volts, the figure, the shape of the output (vout) for the given
minimum and maximum values of Vo (in Volts) sine wave input (vin) will be
are, respectively
R1
Vi Vo T
0 0.5 T
1k
R2 1k

2V
+
Vin Vout

(a) –5 and 2.7 (b) 2.7 and 5 –

(c) –5 and 3.85 (d) 1.3 and 5 –


[EC-2014 : 2 Marks]

Q.29 The figure shows a half-wave rectifier. The


diode D is ideal. The average steady-state
(a) T
current (in Amperes) through the diode is 0 0.5 T
approximately ______ .

D
0.5 T
10 sin t 100 R C 4 mF (b) 0 T
f = 50 Hz

[EC-2014 : 1 Mark] 0.5 T


0 T
(c)
Q.30 Two silicon diodes, with a forward voltage drop
of 0.7 V, are used in the circuit shown in the
figure. The range of input voltage Vi for which
the output voltage Vo = Vi, is (d)
0 T
0.5 T
R
[EC-2015 : 1 Mark]
+ +
D1 D2
Q.32 In the circuit shown below, the Zener diode is
Vi Vo ideal and the Zener voltage is 6 V. The output
–1 V 2V voltage, Vo (in Volts) is ______ .
– –
60 Electronics Engineering Analog Electronics

Q.36 In the circuit shown, assume that the diodes D1


1k and D2 are ideal. The average value of voltage
+
10 V 1k Vo
Vab (in Volts), across terminals ‘a’ and ‘b’ is
– ________ .

[EC-2015 : 1 Mark] D1 D2
10 k
6 sin( t) + –
Q.33 If the circuit shown has to function as a a b
clamping circuit, then which one of the Vab
10 k 20 k
following conditions should be satisfied for the
sinusoidal signal of period T ?
[EC-2015 : 2 Marks]
+ –
C
Q.37 The figure shows a half-wave rectifier with a
V R 475 µF filter capacitor. The load draws a
constant current Io = 1 A from the rectifier. The
figure also shows the input voltage Vi, the output
(a) RC << T (b) RC = 0.35 T voltage VC and the peak-to-peak voltage ripple
(c) RC T (d) RC >> T u on VC. The input voltage Vi is triangle wave
with an amplitude of 10 V and a period of 1 ms.
[EC-2015 : 1 Mark]

Q.34 The diode in the circuit given below has


VON = 0.7 V but is ideal otherwise. The current
+
(in mA) in the 4 k resistor is ______ . Vi 475 µF VC Io = 1 A

2k 3k

D 1k +10 V Vi
1 mA
0 t

4k 6k

–10 V
VC
[EC-2015 : 2 Marks] u

Q.35 In the circuit shown, assume that diodes D1 and 0 t


D2 are ideal. In the steady-state condition the The value of the ripple u (in Volts) is ______ .
average voltage Vab (in Volts) across the 0.5 µF [EC-2016 : 2 Marks]
capacitor is ______ .
Q.38 Assume that the diode in the figure has
1 µF
Von = 0.7 V, but is otherwise ideal.

D1 D2
50 sin( t) R1

2k
0.5 µF i2

2V
b –V + a R2 6k
ab

[EC-2015 : 1 Mark]
GATE Previous Years Solved Paper 61

The magnitude of the current i2 (in mA) is equal (a) D1 only (b) D2 only
to _______ . (c) Both D1 and D2 (d) None of D1 and D2
[EC-2016 : 1 Mark] [EC-2016 : 2 Marks]
Q.39 The diodes D1 and D2 in the figure are ideal and Q.41 The output Vo of the diode circuit shown in the
the capacitors are identical. The product RC is figure is connected to an averaging DC
very large compared to the time period of the ac voltmeter. The reading on the DC voltmeter in
voltage. Assuming that the diodes do not volts, neglecting the voltage drop across the
breakdown in the reverse bias, the output diode, is ________ .
voltage Vo (in Volt) at the steady-state is _____ .
+

+ 10 sin t
D1 1k Vo
f = 50 Hz
C
10 sin t –
R Vo
[EC-2017 : 1 Mark]
AC
C
Q.42 In the figure, D1 is a real silicon pn-junction
D2
– diode with a drop of 0.7 V under forward bias
condition and D 2 is a Zener diode with
[EC-2016 : 1 Mark] breakdown voltage of –6.8 V. The input Vin(t) is
a periodic square wave of period T, whose one
Q.40 The I-V characteristics of the Zener diodes D1
and D2 are shown in Fig. (i). These diodes are period is shown in the figure.
used in the circuit given in Fig. (ii). If the supply Vi(t)
voltage is varied from 0 to 100 V, then breakdown +14 V
occurs in
T
I 0
t(sec)
–80 V –70 V
V
–14 V

10 µF

D1 D2 D1
10 V o (t )

D2
Fig. (i)

Assuming 10 << T, where is the time constant


D1
of the circuit, the maximum and minimum
0 - 100 V values of the output waveform are respectively.
(a) 7.5 V and –20.5 V
D2 (b) 6.1 V and –21.9 V
(c) 7.5 V and –21.2 V
Fig. (ii) (d) 6.1 V and –22.6 V [EC-2017 : 1 Mark]
62 Electronics Engineering Analog Electronics

Q.43 A DC current of 26 µA flows through the circuit Q.45 In the circuit shown, Vs is a square wave of
shown. The diode in the circuit is forward period T with maximum and minimum values
biased and it has an ideality factor of one. At of 8 V and –10 V, respectively. Assume that the
the quiescent point, the diode has a junction diodes is ideal and R1 = R2 = 50 . The average
capacitance of 0.5 nF. Its neutral region value of VL is _____ Volts. (Rounded off to one
resistances can be neglected. Assume that the decimal place).
room temperature thermal equivalent voltage is
26 mV.
+8 V

5 sin( t) mV 0
T/2 T
100

V
–10 V

R1
For = 2 × 106 rad/sec, the amplitude of the
small signal component of diode current (in µA,
correct to one decimal place) is _____ .
+
[EC-2018 : 2 Marks] +
Vs R2 VL
Q.44 The circuit shown in the figure is used to provide

regulated voltage (5 V) across the 1 k resistor. –
Assume that the Zener diode has a constant
reverse breakdown voltage for a current range, [EC-2019 : 1 Mark]
starting from a minimum required Zener
Q.46 In the circuit shown, Vs is a 10 V square wave of
current, IZmin = 2 mA to its maximum allowable
period. T = 4 ms with R = 500 and C = 10 µF.
current. The input voltage V1 may vary by 5%
The capacitor is initially uncharged at t = 0, and
from its nominal value of 6 V. The resistance of
the diode is assumed to be ideal. The voltage
the diode in the breakdown region is negligible.
across the capacitor (VC) at 3 ms is equal to _____
R Volts (rounded off to one decimal place).

VI 1k +10 V
5V
0
T/2 T

–10 V
The value of R and the minimum required
t=0
power dissipation rating of the diode,
respectively, are R

(a) 186 and 10 mW +

(b) 100 and 40 mW


Vs C VC
(c) 100 and 10 mW
(d) 186 W and 40 mW –
[EC-2018 : 2 Marks]
[EC-2019 : 2 Marks]
GATE Previous Years Solved Paper 63

Q.47 In the circuit shown, the breakdown voltage and TOFF = 1 µs is applied to the circuit shown in the
the maximum current of the Zener diode are 20 V figure. The diode D1 is ideal.
and 60 mA, respectively. The values of R1 and 20 nF
RL are 200 and 1 k , respectively. What is the +
range of Vi that will maintain the Zener diode
in the ‘ON’ state? Vin 500 k D1 Vo

R1

Vi RL
The difference between the maximum voltage
and minimum voltage of the output waveform
V0 (in integer) is _______ V.
(a) 20 V to 28 V (b) 24 V to 36 V
[EC-2021 : 2 Marks]
(c) 18 V to 24 V (d) 22 V to 34 V
[EC-2019 : 2 Marks] Q.51 A circuit and the characteristics of the diode (D)
in it are shown. The ratio of the minimum to the
Q.48 In the circuit shown below, all the components
are ideal and the input voltage is sinusoidal. Vout
maximum small signal voltage gain is
The magnitude of the steady-state output Vo Vin
(rounded off to two decimal places) is _____ V. _____ (Rounded off to two decimal places).
C1 = 0.1 µF D2
2k 2k
+
+

230 V(rms) D1 C2 = 0.1 µF Vo D


V in
+
– V out 2k
– –
[EC-2020 : 1 Mark]
ID D
Q.49 A circuit with an ideal Op-amp is shown in the
figure. A pulse VIN of 20 ms duration is applied + VD –
to the input. The capacitors are initially
uncharged. 20
ID(mA)

5V
10
10 k
0
0V ( V)
0 0.7 VD
t = 0 t = 20 ms 1 µF
+12 V
[EC-2022]
VIN –
VOUT
+
1 µF ELECTRICAL EN GINEERIN G
–12 V

(GATE Previous Years Solved Papers)


The output voltage VOUT of this circuit at t = 0+
(in integer) is _____ V. Q.1 Figure shows an electronic voltage regulator.
[EC-2021 : 2 Marks] The zener diode may be assumed to require a
minimum current of 25 mA for satisfactory
Q.50 An asymmetrical periodic pulse train Vin of 10 V operations. The value of R required for
amplitude with on time TON = 1 ms and off time satisfactory voltage regulation of the circuit is,
64 Electronics Engineering Analog Electronics

D I
R

20 V 10 V 100
Vi = Vm sin t Vi 45
= 314 rad/sec

[EE-1991 : 2 Marks]
Vm Vm
Q.2 The depletion region (or) space change region (a) (b)
50 50 2
(or) transition region in a semiconductor p-n
junction diode has Vm 2Vm
(c) (d)
(a) electronics and holes 100 2 50 2
(b) positive ions and electrons [EE-2002 : 1 Mark]
(c) positive and negative ions
Q.7 The cut-in voltage of both zener diode Dz and D
(d) negative ions and holes shown in figure is 0.7 V, while breakdown
[EE-1996 : 1 Mark] voltage of the zener is 3.3 and reverse
Q.3 As temperature is increased, the voltage across breakdown of D is 5 V. The other parameters
a diode carrying a constant current can be assumed to be the same as those of an
ideal diode. The values of the peak output
(a) increases
voltage (Vo) are
(b) decreases
(c) remains constant 1k

(d) may increase or decrease depending upon


the doping levels in the junction
[EE-1999 : 1 Mark] = 314 rad/sec 10 sin t 1k Vo

Q.4 The mobility of an electron in a conductor


expressed in terms of
(a) cm2/V-sec (b) cm/V-sec
(a) 3.3 V in the positive half cycle and 1.4 V in
(c) cm2/V (d) cm2/sec
the negative half cycle.
[EE-1999 : 1 Mark]
(b) 4 V in the positive half cycle and 5 V in the
Q.5 A diode whose terminal characteristics are negative half cycle.
related as iD = Is eV/VT, where Is is the reverse (c) 3.3 V in the both positive and negative half
saturation current and VT is thermal voltage cycle.
(= 25 mV), is biased at iD = 2 mA. Its dynamic
(d) 4 V in the both positive and negative half
resistance is
cycle.
(a) 25 (b) 12.5
[EE-2002 : 1 Mark]
(c) 50 (d) 100
[EE-2000 : 2 Marks] Q.8 In the single-phase diode bridge rectifier shown
in figure, the load resistor is R = 50 . The source
Q.6 The forward resistance of the diode shown in
voltage is V = 200 sin t, where = 2 × 50 rad/s.
figure is 5 and the remaining parameters are
The power dissipated in the load resistor R is
same as those of ideal diode. The dc components
of the source current is
GATE Previous Years Solved Paper 65

Q.11 Assuming that the diodes are ideal in figure,


the current in diode D1 is
1k 1k

A
V
B R D2
5V D1

8V

3200 (a) 8 mA (b) 5 mA


(a) W (b) 400 W
(c) 0 mA (d) –3 mA
[EE-2004 : 2 Marks]
400
(c) W (d) 800 W
Q.12 Assume that D1 and D2 in figure are ideal diodes.
[EE-2002 : 2 Marks] The value of current I is,

Q.9 A voltage signal 10 sin t is applied to the circuit


with ideal diodes, as shown in figure. The D1 2k
maximum and minimum values of the output
waveform Vout of the circuit are respectively. 1 mA
(DC)
I
10 k
2k
+ D2

Vin Vout
(a) 0 mA (b) 0.5 mA
4V 4V
(c) 1 mA (d) 2 mA
10 k [EE-2005 : 1 Mark]

Q.13 What are the states of the three ideal diodes of
(a) +10 V and –10 V (b) +4 V and –4 V the circuit shown in figure?
(c) +7 V and –4 V (d) +4 V and –7 V 1
[EE-2003 : 2 Marks]
1

Q.10 The current through the Zener diode in figure is D2

2.2 k
10 V D1 1 D3 5A
RZ = 0.1 k
IZ

+ +
10 V R1 3.5 V
VZ = 3.3 V (a) D1 ON, D2 OFF, D3 OFF

– (b) D1 OFF, D2 ON, D3 OFF
(c) D1 ON, D2 OFF, D3 ON
(a) 33 mA (b) 3.3 mA (d) D1 OFF, D2 ON, D3 ON
(c) 2 mA (d) 0 mA [EE-2006 : 1 Mark]
[EE-2004 : 1 Mark]
66 Electronics Engineering Analog Electronics

Q.14 Assuming the diodes D1 and D2 of the circuit


+10 V
shown in figure to be ideal ones, the transfer
characteristics of the circuit will be 1k

Vin RL = 10
D1 6.6 V
D2 Zener diode
2
Vi Vo RL =
0

10 V 5V
(a) 0.6 W (b) 2.4 W
(c) 4.2 W (d) 5.4 W
Vo
[EE-2007 : 1 Mark]

10 Q.16 The equivalent circuits of a diode, during


(a)
forward biased and reverse biased conditions,
Vi are shown in the figure.
10

+ – + –
Vo
– + + –
(a)

(b) 5 10 k

Vi
5
10 sin t 5V Vo 1k

Vo

10
(b)

(c) If such a diode is used in clipper circuit of figure


5
given above, the output voltage (Vo) of the circuit
Vi will be
5 10

+5 V
Vo

(a) t
10 0 2
(d)
–5 V
Vi
10

10 V
[EE-2006 : 2 Marks]

Q.15 The three-terminal linear voltage regulator is (b) t


0 2
connected to a 10 load resistor as shown in
–5 V
the figure. If Vin is 10 V, what is the power
dissipated in the transistor?
GATE Previous Years Solved Paper 67

(a) 4 V (b) 5 V
(c) 7.5 V (d) 12.12 V
+5.7 V
[EE-2010 : 1 Mark]
(c) t
0 2 Q.19 The transistor used in the circuit shown below
has a of 30 and ICBO is negligible.
–10 V

2.2 k
15 k
+5.7 V
1k D
VBE = 0.7 V
(d) t
0 2 VCE(sat) = 0.2 V

–5 V Vz = 5 V

[EE-2008 : 1 Mark]
–12 V
Q.17 In the voltage doubler circuit shown in the
If the forward voltage drop of diode is 0.7 V,
figure, the switch ‘S’ is closed at t = 0. Assuming
then the current through collector will be
diodes D1 and D2 to be ideal, load resistance to
(a) 168 mA (b) 108 mA
be infinite and initial capacitor voltages to be
zero. The steady-state voltage across capacitors (c) 20.54 mA (d) 5.36 mA
C1 and C2 will be [EE-2011 : 2 Marks]

V C1 Q.20 A clipper circuit is shown below.


S
+ –
1k
C1 D2
t=0 +
+ D
Vz = 10 V

5 sin t D1 C2 VC2 RL
Vi Vo

5V

(a) VC1 = 10 V, VC2 = 5 V Assuming forward voltage drops of the diodes


(b) VC1 = 10 V, VC2 = –5 V to be 0.7 V, the input-output transfer
(c) VC1 = 5 V, VC2 = 10 V characteristics of the circuit is
(d) VC1 = 5 V, VC2 = –10 V Vo
[EE-2008 : 2 Marks]

Q.18 Assuming that the diodes in the given circuit (a) 4.3
are ideal, the voltage Vo is Vi
4.3
10 k

Vo
10 k 10

10 V Vo 15 V
(b)
10 k 4.3
Vi
4.3 10
68 Electronics Engineering Analog Electronics

Vo (a) 125 and 125 (b) 125 and 250


(c) 250 and 125 (d) 250 and 250
–0.7 [EE-2013 : 2 Marks]
(c) Vi
–0.7 Q.23 A voltage 1000 sin t Volts is applied across YZ.
Assuming ideal diodes, the voltage measured
across WX (in Volts), is
Vo

10
1k

(d) W Y X
Z
–5.7
Vi
10
–5.7
+ –
1k
[EE-2011 : 2 Marks]
(sin t + sin t )
(a) sin t (b)
Q.21 The i-v characteristics of the diode in the circuit 2
given below are: (sin t sin t )
(c) (d) 0 for all t
v 0.7 2
A, v 0.7 V
i = 500 [EE-2013 : 2 Marks]
0 A, v < 0.7 V
Q.24 The sinusoidal ac source in the figure has an
The current in the circuit is
rms value of 20 / 2 V. Considering all
1k
possible values of RL, the minimum value of Rs
i (in ) to avoid burn out of the Zener diode is
+
10 V +
______ .

D v

Rs

(a) 10 mA (b) 9.3 mA


(c) 6.67 mA (d) 6.2 mA 20 5V RL
V 1/4 W
[EE-2012 : 1 Mark] 2

Q.22 In the circuit shown below, the knee current of


the ideal Zener diode is 10 mA. To maintain 5 V
across RL, the minimum value of RL in and
the minimum power rating of the Zener diode [EE-2014 : 1 Mark]
(in mW), respectively are
Q.25 Assuming the diodes to be ideal in the figure,
for the output to be clipped, the input voltage vi
100 must be outside the range.

Iload 10 k
10 V

Vz = 5 V RL
Vi 10 k Vo

1V 2V
GATE Previous Years Solved Paper 69

(a) –1 V to –2 V (b) –2 V to –4 V is biased so that voltage, V > 0 and current, I < 0.


(c) +1 V to –2 V (d) +2 V to –4 V If you had a design this circuit to transfer
[EE-2014 : 2 Marks] maximum power from the current source (I1) to
a resistive load (not shown) at the output, what
Q.26 A non-ideal diode is biased with a voltage of values R1 and R2 would you choose?
–0.03 V, and a diode current of I1 is measured.
R1 I
The thermal voltage is 26 mV and the ideality
factor for the diode is 15/13. The voltage, (in V), +

at which the measured current increases to 1.5I1 I1 D R2 V


is closest to
(a) –4.50 (b) –0.09 I –
(c) –0.02 (d) –1.50
(a) Small R1 and small R2
[EE-2020 : 2 Marks]
(b) Large R1 and large R2
Q.27 Consider the diode circuit shown below. The (c) Small R1 and large R2
diode, D, obeys the current-voltage characteristic, (d) Large R1 and small R2
VD [EE-2020 : 2 Marks]
ID = I s exp 1
nVT
where n > 1, VT > 0, VD is the voltage across the
diode and ID is the current through it. The circuit

Electronics & Electrical Engineering


GATE Previous Years Solved Paper

A n swe rs & Expl a n a t i o n s

Answers
EC Diodes Applications

1. (c) 2. (a) 3. (a) 4. (c) 5. (d) 6. (d) 7. (b) 8. (c)


9. (a) 10. (d) 11. (c) 12. (d) 13. (b) 14. (c) 15. (b) 16. (d)
17. (b) 18. (c) 19. (c) 20. (c) 21. (a) 22. (a) 23. (b) 24. (a)
25. (b) 26. (d) 27. (d) 28. (c) 29. (0.10) 30. (d) 31. (c) 32. (5)
33. (d) 34. (0.6) 35. (100) 36. (5) 37. (2.105) 38. (0.25) 39. (0) 40. (a)
41. (3.183) 42. (a) 43. (6.40) 44. (b) 45. (–3) 46. (3.3) 47. (b) 48. (650.40)
49. (–12) 50. (10) 51. (0.75)
70 Electronics Engineering Analog Electronics

Solutions
EC Diodes Applications

1. (c) Vo
10 6 4
I= = = 80 mA
50 50
I= IZ + IL
5.9 V
= IZmin + ILmax
IZmin = 5 mA
80 = 5 + ILmax 3. (a)
ILmax = 75 mA Ebers Moll model is a composite model and is
VL used to predict the operation of BJT all of its
ILmax =
Rmin possible modes.
VL 6 4. (c)
Rmin = = 3
I L max 75 × 10
Current through 50 resistance = I
= 80
9 6 3
I= = = 60 mA
2. (a) 50 50
Given that,
Vo
IZmin = 5 mA
5.9 PZmax = IZmaxVz = 300 mW
0 3
300 × 10
–5.9
IZmax =
Vz
3
DA DB 300 × 10
= = 50 mA
6
4.1 V 4.1 V I= IZmin + ILmax
10 sin314t 10 k Vo
= IZmax + ILmin
60 = 50 + ILmin
ILmin = 10 mA
60 = 5 + ILmax
Case-1: During +ve half cycle
ILmax = 55 mA
Diode DA is forward bias, so DA is short-circuit.
Diode DB is reverse bias, so DB is in conducting 5. (d)
state when Vi > 4.1 V. A resistance: For small signal ac operation, a
Vo practical forward biased diode can be modeled
5.9 V as a resistance.

6. (d)
A four diode bridge rectifier uses the smaller
size of transformer, which is less expensive
Case-2: During –ve half cycle
transformer and these rectifiers are suitable for
Diode DB is forward bias, so DB is short-circuit.
higher voltage applications, because of low PIV
Diode DA is reverse bias, so DA is in conducting
rating required of each diode.
state when Vi > 4.1 V.
GATE Previous Years Solved Paper 71

7. (b) Vin Vo
IZ + IL
VNL VFL R
Regulation = when, Vin = 30 V
VFL
30 10
30 25 1 (10 + 1) mA
= = = 20% R
25 5
20
VDCNL VDCFL 11 mA
Ro = =5 R
IDC
R 1818
8. (c) 11. (c)
I1 20 As voltage at non-inverting terminal is 3 V due
to zener diode, voltage at inverting terminal will
IZ IC
be 3 V because of virtual ground.
IB 3 3
Vin = 20-30 V
+
Vo = 10 V So, current in 20 k is = mA
20 k 20
RB VBE –
IE 3
Vo = × 60 k =9V
20 k
Vinmax Vo
I1max = 12. (d)
20
I = IZ + IL
30 10 = IZmin + ILmax
I1 = = 1 A (i.e. when, IZ = 0)
20
= 0 + 500 mA
IE = IC + IZ ...(i)
Vin Vz 12 5
IB = IZ (as no current flows in RB) R= =
I 500 mA
I I
= C = C = 14
I B IZ
IC = IZ 14. (c)
From equation (i), I1 1k I2
IE = IZ + IZ = (99 + 1) IZ
IE = 100 IZ IZ

I1 = IE = 100 IZ 20-30 5.8 V Load


I1 1
IZ = = = 0.01 A
100 100
PZ = VZ IZ = 9.5 × 0.01 = 95 mW
VL = 5.8 V
IC = 99IZ = 99 × 0.01 = 0.99 A Maximum load current will be drawn, when
PC = VC IC = 10 × 0.99 = 9.9 W Vi = Vmin = 20 V
9. (a) 20 5.8
= = IL + IZ
1k
R
14.2 mA = IL + IZ
+
IL
IZ IL = 14.2 mA – 0.5 mA
Vin Vo RL = 13.7 mA
DZ


72 Electronics Engineering Analog Electronics

15. (b) 20. (c)


Zener diode works as normal diode is FB. For the positive half of Vi ,
So, when Vin < 0, VR = Vin D1 is forward biased and Zener diode is in
when 0 < Vin < 6, Diode is OFF and VR = 0 breakdown stage,
when Vin > 6, Diode conducts and VR = Vin Vo = 0.7 + 6.8 = 7.5 V
For the negative half of Vi , D2 is forward biased,
16. (d)
Vo = –0.7 V
15 V
Q1 22. (a)
+
For d.c. biasing a.c. will be short-circuited,
1k 12 k 12 k
Vout 9900
+ –
+

(6 V)

6V 24 k VD = 4 × 2.7
12.7 V
= 2.8

Volt across 24 = 6 V due to virtual ground


concept. So volt across 12 is 13 V, –

Vout = V12 + V24


= 3+6=9V 12.7 2.8
IDC = = 1 mA
VCE = 15 – Vout 9900
= 15 – 9 = 6 V
23. (b)
9V 9V
IC = + =1A For a.c. analysis diode will be replaced by its
12 36
dynamic resistance,
P = VCE IC = 6 V × 1 A
VT 1 × 25 mV
P = 6 Watts rd = =
I 1 mA
17. (b) = 25
New unregulated voltage = 18 V vi
(vo)ac = × 100
VCE = 18 – 9 = 9 V 9900 + 100
IC = 1 A vp
vi
P = 9 × 1 = 9 Watts = = cos t
100 100
9 6 (vo)ac = 1 cos( t) mV
% increase = × 100 = 50%
6
24. (a)
19. (c)
It is voltage doubler circuit in which C1 will be
V Vz charged to maximum value of input that is 1 V.
Vo = in rz + Vz
R + rz So, v(t) = (cos t – 1) according to KVL.
If Vin = 10 V,
25. (b)
10 7
Vo = × 10 + 7 = 7.14 V
200 + 10 Vin Vz 10 5
I= = = 50 mA
If Vin = 15 V, R 100
16 7 Vz Vz
Vo = × 10 + 7 = 7.43 V RLmin = = = 125
200 + 10 I L max I IZ min
GATE Previous Years Solved Paper 73

Minimum power rating of Zener diode, Vin 0.7 2


Vo = × 1 k + 0.7 + 2
Pz = Vz IZmax 1k +1k
IZmax = 50 mA
5 2.7
Pz = 5 × 50 = 250 mW = + 2.7 = 3.85 V
2
26. (d)
29. Sol.
Case-1: When VYZ is positive.
The average current through the diode is
Then all the four diodes are reverse bias.
approximate,
So, Vw = Vx = 0
Vp 10
Vwx = 0 V iD(avg) = = 0.10 A
R 100
Case-2: When VYZ is negative.
So, all the diodes will be short-circuit, 30. (d)
Vw = Vx For case-(i): Vi < –1.7
Vwx = Vw – Vx D1 ON
= Vw – Vw = 0 D2 OFF
Vwx = 0
So, Vwx = 0 for all t R
+ +
27. (d)
Consider D1 OFF and D2 ON then, Vi Vo
–1 V
1k I 1k
– –

+ Vo = –1.7 V
10 V VD1 0.3 V For case-(ii):

–1.7 < Vi < 2.7
D1 OFF
Apply KVL, D2 OFF
10 = 1000I + 20I + 0.3 R
9.7 + +
I= = 9.5 mA
1020
Vi Vo
Now, we calculate VD of
1

10 = 9.5 + VD1 – –

VD1 = 0.5 V Vo = Vi

31. (c)
Since, VD1 < 0.7 V, D1 is in OFF state i.e. our
The circuit can be redrawn as:
assumption is correct and hence (d) is the correct
D1
option. a d
vin +
vout

28. (c) Vo
R t
t
If, Vin = –5 V Diode is off D2
+

Vo = Vin = –5 V b c

If, Vin = 5 V Diode is on


74 Electronics Engineering Analog Electronics

• During positive pulse, both diodes are 37. Sol.


forward biased. 3
IDC T 1 × 1 × 10
• During negative pulse, both diodes are Vripple = = 6
= 2.105 Volts
C 475 × 10
reverse biased.
So, Vo = 0 V 38. Sol.
id
32. Sol.
Zener is not in breakdown region.
i1
1 VA
Hence, Vo = 10 × =5V 2k
2 i2

2V
34. Sol. 6k
The bridge is balanced bridge, so diode will be
OFF.
Let diode: ON
Current through 4 k resistor can be obtained
VA = 2 – 0.7 = 1.3 V
by applying current division rule.
1.3
i2 =
6k
2k 3k 0.7
i1 =
2k
1 mA i d = i2 – i1
i
1.3 0.7
4k 6k = mA mA = ve
6 2
Diode is OFF:
Current through 4 k resistor is 0.6 mA.

35. Sol.
2k
The circuits work as a voltage doubler. i2
Vab = 2 × 50 = 100 2V
6k
36. Sol.
During positive cycles,
2V
V i2 = = 0.25 mA
V1 = in 2 k +6 k
2
During negative cycles, 39. Sol.
Vin During first positive quarter cycle both diodes
Vo =
3 D1 and D2 conduct and at t = T/4 both will be
Vo OFF.

Vm/2 +
Vm/3
10 V
t 10 sin t – +
R Vo = 0
Vm 1 Vm 1 6 6 – –
VDC = + = + =5V 10 V
2 3 2 3 +
GATE Previous Years Solved Paper 75

Both the capacitors will charge to 10 V as shown


1 (1000) ( j1000)
is above figure and Vo = 0 in steady-state. rd =
j Cj 1000 j 1000
41. Sol.
j(1 + j )
The given circuit is a halfwave rectifier. = k
2
Voltmeter reads the average value of Vo.
1
Vm = (1 j ) k
Average value of Vo = 2
= (500 – j500)
Vm = peak value of the applied sine wave = 10 V Z = 600 – j500
10
So, reading of meter = V = 3.183 V Z = 100 36 + 25

= 100 61
42. (a)
Its a negative clamper with Vref = 6.8 V Vm 5 mV
Im = =
In steady-state, Z 100 61
Vcap = 14 – 0.7 – 6.8 = 6.5 V 50
= µA = 6.40 µA
Vomax = Vin max – Vcap 61
= 14 – 6.5 V
Vo min = Vin in – Vcap 44. (b)
= –14 – 6.5 = –20.5 V VI = 6 V ± 5%
= 6 V ± 0.3 V
43. Sol.
= 5.7 V to 6.3 V
The small-signal equivalent model of the given
5V
circuit can be drawn as shown below: IL = = 5 mA
1k
Is(min) = IL + IZ(min)
Cj
= 5 mA + 2 mA = 7 mA
i(t)
rd VI Vz
Is =
5 sin( t) mV 100 R
VI (min) Vz
Is(min) = = 7 mA
R
Given that, = 2 × 106 rad/sec 5.7 5
So, R= k
Cj = 0.5 nF 7
IDC = 26 µA 700
= = 100
VT = 26 mV 7
= 1 When R = 100 ,
VT 26 mV 6.3 5
So, rd = = =1k A = 13 mA
I DC 26 µA Is(max) =
100
1 1 Iz(max) = Is(max) – IL
= 6 9
Cj 2 × 10 × 0.5 × 10 = 13 mA – 5 mA
= 1k = 8 mA
So, total impedance of the circuit will be Pz(min) = Vz Iz(max)
= (5 × 8) mW
1
Z= rd + 100 = 40 mW
j Cj
76 Electronics Engineering Analog Electronics

45. Sol. For 2 ms < t < 4 ms, diode is OFF and capacitor
When, Vs = 8 V Diode is in reverse bias, has no path to discharge.
50
Hence, at t = 3 ms, Vcap = 3.3 V

47. (b)
+
Vs 50 VL R1 = 200
– +
I1 IZ IL

Vi RL = 1 k Vo
8 × 50
VL = =4V
50 + 50

If Vs = –10 V, diode is in forward bias,
Vz = 20 V
IZmax = 60 mA
+
Vs 50 VL
Set zener diode be OFF,
– Vi × 1 Vi
Vo = =
0.2 + 1 1.2
Average value of VL Zener diode can become ON i.e. it goes into
breakdown, when
Area
= Vi
Time period > 20 V
1.2
4 × 0.5T + ( 10) × 0.5T
= = –3 V Vi > 24 V
T
When Zener diode is in breakdown region,
46. Sol. Vi 20 Vi 20
I1 = = mA
vi 0.2 k 0.2

+10 V Vo 20
IL = = = 20 mA
RL 1 k
0 t
2 ms 4 ms
Vi 20
–10 V Iz = I1 IL = 20
0.2
vcap For safe operation,
10 V
Iz Izmax
Vi 20
20 60
0 t 0.2
2 ms 4 ms
Vi 36 V
= RC = 500 × 10 × 10–6 Hence, 24 < Vi < 36 V
= 5 ms
t 48. Sol.
0<t< ; Vcap = Vf + (Vi – Vf ) e–t/
2 Voltage doubles,
= 10 + (0 – 10) e–t/RC Vo = 2 Vm
At t = 2 msec,
= 2 × 230 2
2 × 10 3 650.4 V
5 × 10 3
Vcap = 10 10 e
= 3.296 V 3.3 V
GATE Previous Years Solved Paper 77

49. (–12) 20 nF
+ –
+ +
5V
10 k
10 V 500 k D1 Vo = 0

0 20 m 1 µF
+12 V – –
VIN –
VOUT
+ and VC (Voltage across capacitor) = 10 V ...(1)
1 µF
–12 V
Case (ii) : If Vin = 0 V D1 is OFF.
20 nF

+ + – +
5V VC
10 k
Vi 500 k D1 Vo
iC
0 20 m
C = 2 µF +12 V – –
VIN –
0V VOUT
iC + The duration of input pulse with amplitude 0 V
–12 V is, TOFF = 1 µsec
and discharge time for capacitor,
Tdischarge = 20 nF × 500 k
0 Vo dV = 10 msec
iC = = C in
R dt i.e., Tdischarge >>TOFF
d Vo = Vi – VC
Vo = RC Vin
dt = 0 V – 10 V = –10 V ...(2)
= –RC [Slope of input Vin] Note : The difference between maximum voltage
= –RC [ ] and minimum voltage of the output waveform
= –Vsat = 0 – (–10 V)
= –12 V = 10 V ...(3)
vi
50. (10)
10 V TOFF
Given: Vin = 10 V TON = 1 ms
Asymmetrical periodic pulse train, t
0
TON = 1 m-sec vo
and TOFF = 1 µsec 1 µs
t
Diode is ideal, 0
–10 V
20 nF
+
51. (0.75)
Vin 500 k D1 Vo When diode is OFF :
I 2k 2k

Vin
Case (i) : If Vin = 10 V D1 is ON. Vout
2k
78 Electronics Engineering Analog Electronics

Vo = 4I; Vi = 6I (Vo 0.7)


Vi = 0.7 + (4 k )
Vo 4 2 (2 k )
Av = = =
Vi 6 3 = 0.7 + 2Vo – 1.4
When diode is ON, voltage drop across diode is Vi + 0.7
0.7 V. Vo =
2
2k 2k Vo 1
I Av = =
Vi 2
I
+
Maximum voltage gain,
Vin 0.7 V 2k 2
Vout Av =
3
– Minimum voltage gain,
1
Vo = 0.7 + 2 (k ) I Av =
2
Vi = 0.7 + (4 k ) I
( Vo / Vi )min Av 1 3 3
V 0.7 = = × = = 0.75
I= o ( Vo / Vi )max Av 2 2 4
2k

Answers
EE Diodes Applications

1. (80) 2. (c) 3. (b) 4. (a) 5. (b) 6. (a) 7. (b) 8. (b)

9. (d) 10. (c) 11. (c) 12. (a) 13. (a) 14. (a) 15. (b) 16. (a)

17. (d) 18. (b) 19. (d) 20. (c) 21. (d) 22. (b) 23. (d) 24. (300)

25. (b) 26. (b) 27. (c)

Solutions
EE Diodes Applications

1. Sol. 2. (c)
Current in 100 resistance, Charge recombination occurs in depletion
10 V region.
I100 = = 100 mA
100
3. (b)
I2 = 25 mA
dVD
Current in resistance, = 2.5 mV/°C
dT
R = 100 mA + 25 mA
= 125 mA 4. (a)
Also, 20 – 10 = 125 × 10–3 × R
Drift velocity
10 × 1000 µ=
R= = 80 Electric field
125
GATE Previous Years Solved Paper 79

5. (b) 9. (d)

V 25 mV 10 k
rD = T = = 12.5
ID 2 mA
D1 D2
6. (a)
Vin I 4V 4V
Vm
Vav = 10 k

Vav Vav
Iav = = For +ve cycle, when Vin < –4 V
R (5 + 45)
(Vin > 4 V) D2 ON, D1 OFF, Vout = 4 V
Vav Vm where Vin < 4 V, then Vout = Vin because D2 also
= =
50 50 become (OFF).
For –ve cycle, when Vin < –4
7. (b)
D1 ON, D2 OFF:
For positive half cycle, at peak input voltage,
Vin + 4
I=
1k 4 mA 20 k
But, Vin = –ve
+ 3.3 V Vin + 4 3
I= = mA
10 V 1k Vo 20 10
– (Q for minimum output, Vin = –10 V)
0.7 V
3
Vout = × 10 k 4 = 7 V
10
So, Vo = 4 V
For negative half cycle, at peak input voltage, 10. (c)
Vz 3.5 3.3
1k 2 mA Rz = =
Iz Iz

– 0.2
Iz = = 2 mA
Vo
0.1 k
10 V 1k
+ O.C. 11. (c)
From the circuit, if ‘D1’ is ‘ON’ Kirchoff’s law is
not satisfied.
1
So, Vo = 10 × =5V D1 is OFF and D2 is ON.
1+1 I = 0 mA

8. (b) 12. (a)


Vm Current will pass through its simplest path (for)
Vrms =
2 low resistance path. D1 become forward biased
2
and D2 is reverse biased, I = 0 mA.
Vrms V2
P= = m
R 2R 13. (a)
Analyzing the circuit, we can see that D1 and
200 2
= = 400 W D2 are in forward bias and D3 is in reverse bias.
2 × 50
But no current flow through D2, because current
gets shortest path through D1.
80 Electronics Engineering Analog Electronics

14. (a) 17. (d)


Vi < 10 V D1 OFF, D2 OFF VC1
S
Vo = 10 V + –

Vi > 10 V D1 ON, D2 OFF t=0


C1 D2
Vo = Vin +
5 sin t D1 VC2 C2 RL
15. (b) –
IC1 IC2
+10 V + VCE – Vo
IR 1k IL First capacitor charge through diode (D1) upto
IB
Vmax (5 V) with shown polarities.
RL = 10
VC1 = 5 V
IZ 6.6 V Now diode D1 will be reversed biased and D2
will be forward biased and capacitor C2 will
charge in reverse direction through diode D2
V0 = Vz – VBE upto 2 Vmax.
= 6.6 V – 0.7 V = 5.9 V VC2 = –10 V

V0 5.9 18. (b)


IL = = = 0.59 A = IC
RL 10 Diode D1 is on and diode D2 is off.
VCE = Vi – V0 10
= 10 – 5.9 = 4.1 V So, V0 = 10 × =5V
10 + 10
PQ = VCE × IC
= 4.1 × 0.59 = 2.4 W 19. (d)

16. (a) 0 ( 12) 0.2


IC(sat) =
RC
10 k P
11.8
= = 5.3636 mA
2.2 k
10 sin t 10 k This is the maximum current possible in any
case.
5V

20. (c)
10 For positive voltage the waveform clips at +5.7 V.
VP = × 10 sin t
10 + 10 For negative voltage at –0.7 V, the zener diode
= 5 sin t conducts and clips out.
Since maximum voltage across at point P may Vo
be 5 V, hence voltage across the diode always
will be less than or equal to zero. So it will be 5.7 V

reversed always. –0.7


Vi
10 5.7 V
V0 = × 10 sin t –0.7
10 + 10
= 5 sin t
GATE Previous Years Solved Paper 81

21. (d) For negative half cycle,


By applying KVL in loop,
1k 1k

i W Y X
+
10 V + Z

v

10 = 103 i + v 1k

3 v 0.7 Short circuit condition,


10 = 10 +v
500 VWX = 0
10 = 2v – 1.4 + v
24. Sol.
3v = 11.4
Is Iz + IL
v = 3.8
1 20 5
i = (10 – 3.8) × 10–3 (Putting IL = 0)
20 Rs
= 6.2 mA
R s 300
22. (b) Hence, Rsmin = 300
Is 100 Iload
25. (b)
Case-I : Vi –4 V
D2 conducts
10 V Vz = 5 V RL
D1 OFF
So, Vo = –2 V
Case-II : –4 V Vi –2 V
Both the diodes will be OFF,
10 5
Is = = 50 mA Vo = V1
100
Iload = 50 mA – 10 mA = 40 mA Case-III : Vi –2 V
5V D1 conducts
RL = = 125 D2 OFF
40 mA
Vo = –1 V
Pmin = 50 mA × 5 V = 250 mW
26. (b)
23. (d)
0.03
For positive half cycle,
15/13×26 mV
I1 = I0 e 1

1k
As, VD = –ve
W Y X ‘1’ can not be neglected in diode current
Z equation,

I1 = I0 [ e 30 mV /30 mV 1]

1k
= I0 [e–1 – 1]
= –0.64 I0 ...(i)
VWX = 0
82 Electronics Engineering Analog Electronics

27. (c)
1.5 I1 = I0 [ eVD /30 mV 1]
R1, low, R2 high,
–1.5 × 0.64I0 = I0 [ eVD /30 mV 1] R2
VD = V ×
R1 + R2
V /30 mV
–0.96 = e D 2 1
If R2 is large Vd (high).
V /30 mV
1 – 0.96 = e D R1 is less VD = V.
30 mV ln(0.04) = VD = –0.09 V So for maximum power to deliver to load R1 is
small and R2 is large.
3 BJT Analysis

ELECTRO NICS EN GINEERIN G All capacitances are large. gm of the MOSFET =


2 mA/V, and hfa of the bipolar = 99. The overall
(GATE Previous Years Solved Papers)
transconductance gm of the composite
Q.1 The configuration of cascade amplifier is transistor is
(a) CE-CE (b) CE-CB (a) 198 mA/V (b) 9.9 mA/V
(c) CC-CB (d) CC-CC (c) 4.95 mA/V (d) 1.98 mA/V
[EC-1987 : 2 Marks] [EC-1988 : 2 Marks]
Q.2 The quiescent collector current IC of a transistor Q.5 The transistor in the amplifier shown below has
is increased by changing resistance. As a result following parameters:
(a) gm will not be affected.
hfe = 100, hje = 2 k , hre = 0, hoe = 0.05
(b) gm will decrease.
C is very large. The output impedance is
(c) gm will increase.
VCC
(d) gm will increase or decrease depending
upon bias stability. 5k
58 k
[EC-1988 : 2 Marks]
Vo
C
Q.3 Each transistor in the Darlington pair 100 nF
(see figure) below) has hFE = 100. The overall hFE
of the composite transistor neglecting the 10 k
leakage currents is 1k C

(a) 20 k (b) 16 k
B (c) 5 k (d) 4 k
[EC-1988 : 2 Marks]

Q.6 Of the four biasing circuits shown in figure. For


E
a BJT, indicate the one which can have
(a) 10000 (b) 1001 maximum bias stability?
(c) 10100 (d) 10200
V CC VCC V CC
[EC-1988 : 2 Marks]

Q.4 The amplifier circuit shown below uses a VCC

composite transistor of a MOSFET and bipolar


in cascade.

C (a) (b) (c) (d)


C
Vin [EC-1989 : 2 Marks]
84 Electronics Engineering Analog Electronics

Q.7 For good stabilized biasing of the transistor of Q.10 If the transistor in figure has high value of
the CE amplifier of figure, we should have and VBE of 0.65 the current I flowing through
the 2 k resistance will be ______ .
+VCC
RC I
R2

6.5 k 2k
+
+ R1 R2 = RB
Vo
Vin R1 +
RE
10 V
– – 1.85 k –

RE RE
(a) << 1 (b) >> 1
RB RB
1.65 k
1k
RE RE
(c) << hFE (d) >> hFE
RB RB
[EC-1990 : 2 Marks]
[EC-1992 : 2 Marks]
Q.8 Which of the following statements are correct
for basic transistor amplifier configurations? Q.11 The bandwidth of an n-stage tuned amplifier,
with each stage having a bandwidth of B, is
(a) CB amplifiers has low input impedance and
given by
low current gain.
(b) CC amplifiers has low output impedance B B
(a) (b)
and high current gain. n n
(c) CE amplifiers has very poor voltage gain B
(c) B 2 1/ n 1 (d)
but very high input impedance. 2 1 /n
1
(d) The current gain of CB amplifier is higher [EC-1993 : 1 Mark]
than the current gain of C amplifiers.
Q.12 For the amplifier circuit of figure. The transistor
[EC-1990 : 2 Marks]
has a of 800. The mid-band voltage gain Vo/Vi
Q.9 In figure all transistors are identical and have a of the circuit will be
high value of beta. The voltage VDC is equal to
_______ . +15 V

10 V
470
200 k
5 mA 1k
4.74 µF 6.4 µF +
V DC = ?
+
Vo
Vin 100 k

– –

(a) 0 (b) < 1


(c) 1 (d) 800
[EC-1993 : 1 Mark]
[EC-1991 : 2 Marks]
GATE Previous Years Solved Paper 85

Q.13 cut-off frequency of a bipolar junction (a) gm1 (b) 0.5 gm1
transistor (c) gm2 (d) 0.5 gm2
(a) increase with the increase in base width. [EC-1996 : 2 Marks]
(b) increase with the increase in emitter width.
Q.18 Match the following:
(c) increase with the increase in collector
(a) Cascade amplifier
width.
(b) Differential amplifier
(d) increase with decrease in base width.
(c) Darlington pair common-collector amplifier
[EC-1993 : 2 Marks]
1. does not provide current gain
Q.14 In order to reduce the harmonic distortion in an 2. is a wide band amplifier
amplifier its dynamic range has to be _____ . 3. has very low input impedance emitter
[EC-1994 : 1 Mark] amplifier
4. has very high input impedance and very
Q.15 A common emitter transistor amplifier has a
high current gain
collector current of 1.0 mA when its a base
5. provides high common mode voltage
current is 25 µA at the room temperature. It’s
rejection
input resistance is approximately equal to ___ .
[EC-1996 : 2 Marks]
[EC-1994 : 1 Mark]
Q.19 In the BJT amplifier shown in the figure is the
Q.16 A transistor having = 0.99 and VBE = 0.7 V, is
transistor is biased in the forward active region
used in the circuit of the figure is the value of
putting a capacitor across RE will
the collector current will be
+VCC
+12 V

1k RL
Rbias
+
1k
10 k +
Vout
Vin
RE
– –
1k
(a) decrease the voltage gain and decrease the
input impedance.
[EC-1995 : 1 Mark] (b) increase the voltage gain and decrease the
input impedance.
Q.17 A Darlington stage is shown in the figure is, if
(c) decrease the voltage gain and increase the
the transconductance of Q1 is gm1 and Q2 is gm2,
input impedance.
then the overall transconductance is given by
gm is (d) increase the voltage gain and increase the
input impedance.
VCC
[EC-1997 : 1 Mark]

Q1 ic Q.20 A cascade amplifier stage is equivalent to


(a) a common emitter stage followed by a
Q2
common base stage.
Vbe (b) a common base stage followed by an emitter
follower.
86 Electronics Engineering Analog Electronics

(c) an emitter follower stage followed by a Q.24 The emitter coupled pair of BJT’s gives a linear
common base stage. transfer relation between the differential output
(d) a common base stage followed by a common voltage and the differential input voltage Vid
emitter stage. only. When the magnitude of Vid is less ‘ ’ times
[EC-1997 : 1 Mark] the thermal voltage, when ‘ ’ is
(a) 4 (b) 3
Q.21 The circuit of the figure is an example of
(c) 2 (d) 1
feedback of the following type:
[EC-1998 : 1 Mark]
+VCC
Q.25 A multistage amplifier has a low pass response
Vo
with three real poles at s = – 1, – 2 and 3. The
approximate overall bandwidth B of the
amplifier will be given by
Vi
(a) B = 1 + 2 + 3

1 1 1 1
(a) current series (b) current shunt (b) = + +
B 1 2 3
(c) voltage series (d) voltage shunt 1/3
(c) B = ( 1 + 2+ 3)
[EC-1998 : 1 Mark]
(d) B = 2 2 3
1 + 2 + 3 [EC-1998 : 1 Mark]
Q.22 From measurement of the rise time of the output
pulse of an amplifier whose input is a small
Q.26 In a series regulated power supply circuit the
amplitude square wave, one can estimate the
voltage gain Av of the “pass”, transistor satisfies
following parameter of the amplifier
the condition
(a) gain-bandwidth product
(a) Av (b) 1 << Av <<
(b) slew rate
(c) Av » 1 (d) Av << 1
(c) upper 3-dB frequency
[EC-1998 : 1 Mark]
(d) lower 3-dB frequency
[EC-1998 : 1 Mark] Q.27 In the cascade amplifier shown in the figure, if
the common-emitter stage (Q 1 ) has a
Q.23 A distorted sinusoid has the amplitude, A1, A2, transconductance gm1, and the common base
A3,...... of the fundamental, second harmonic, stage (Q2) has a transconductance gm2, then the
third harmonic, ..... respectively. The total overall transconductance g(= i o/v i) of the
harmonic distortion is cascade amplifier is
A 2 + A3 + .....
(a) Q2 Vo
A1 io

A22 + A32 + .....


(b) RL
A1 Vi Q1

A22 + A32 + .....


(c)
A12 + A22 + A32 + .....
(a) gm1 (b) gm2
( A22 + A32 + ......)
(d) gm1 gm 2
A1 (c) (d)
2 2
[EC-1998 : 1 Mark] [EC-1999 : 1 Mark]
GATE Previous Years Solved Paper 87

Q.28 In the differential amplifier of the figure, if the (a) indeterminate since Rc is not given
source resistance of the current source IEE is (b) 1 mA
infinite, then the common mode gain is (c) 5 mA
(d) 10 mA
VCC
[EC-2000 : 2 Marks]

R R
Q.31 The current gain of a bipolar transistor drops at
high frequencies because of
(a) transistor capacitances
Vin 1 V in2 (b) high current effects in the base
(c) parasitic inductive elements
IEE (d) the early effect

–VEE
[EC-2000 : 1 Mark]

Q.32 The current gain of a BJT is


(a) zero (b) infinite gm
(a) gm ro (b)
Vin1 + Vin 2 ro
(c) indeterminate (d)
2VT gm
(c) gm r (d)
[EC-2000 : 1 Mark] r
[EC-2001 : 1 Mark]
Q.29 Introducing a resistor in the emitter of a common
emitter amplifier stabilizes the dc operating Q.33 Three identical RC coupled transistor amplifiers
point against variation in are cascaded. If each of the amplifiers has a
(a) only the temperature frequency response as shown in the figure, the
(b) only the of the transistor overall frequency response is as given in
(c) both temperature and A v dB
(d) none of the above
0
[EC-2000 : 1 Mark] –3

Q.30 In the circuit of the figure, assume that the


transistor is in the active region. It has a large f
20 Hz 1 kHz
and its base-emitter voltage is 0.7 V. The value
of Ic is Av dB

0
15 V
–3
(a)
RC
10 k f
IC 20 Hz 1 kHz

Av dB

0
5k
430 –3
(b)

f
40 Hz 0.5 kHz
88 Electronics Engineering Analog Electronics

Av dB
(a) CB-LO, CC-MO, CE-HI
(b) CB-LO, CC-HI, CE-MO
0
(c) CB-MO, CC-HI, CE-LO
–3
(c) (d) CB-HI, CC-LO, CE-MO
[EC-2003 : 1 Mark]
f
40 Hz 1 kHz Q.36 Generally, the gain of a transistor amplifier falls
at high frequencies due to the
(a) internal capacitances of the device
Av dB
(b) coupling capacitor at the input
0 (c) skin effect
–3
(d) (d) coupling capacitor at the output
[EC-2003 : 1 Mark]
f
40 Hz 2 kHz Q.37 In the amplifier circuit shown in the figure, the
values of R1 and R2 are such that the transistor
is operating at VCE = 3 V and IC = 1.5 mA when
[EC-2002 : 1 Mark]
its is 150. For a transistor with of 200, the
Q.34 If the transistor in the figure is the saturation, operating point (VCE, IC) is
then VCC = 6 V
C R2
R1
IC

dc denotes the
B
IB dc current gain

E
(a) (2 V, 2 mA) (b) (3 V, 2 mA)
(a) Ic is always equal to dcIB. (c) (4 V, 2 mA) (d) (4 V, 1 mA)
(b) Ic is always equal to – dcIB. [EC-2003 : 2 Marks]
(c) Ic is greater than or equal to dcIB. Q.38 Assuming V CEsat = 0.2 V and = 50, the
(d) Ic is less than or equal to dcIB. minimum base current (IB) required to drive the
[EC-2002 : 1 Mark] transistor in the figure to saturation is
Q.35 Choose the correct match for input resistance of 3V
various amplifier configurations shown below: IC
Configuration: 1k
CB : Common Base
IB
CC : Common Collector
CE : Common Emitter
Input resistance:
LO : Low
(a) 56 µA (b) 140 µA
MO : Moderate
(c) 60 µA (d) 3 µA
HI : High
[EC-2004 : 1 Mark]
GATE Previous Years Solved Paper 89

Q.39 Assuming that the of the transistor is extremely


VCC
large and VBE = 0.7 V, IC and VCE in the circuit
shown in the figure are RC RC

5V

IC
V1 V2
2.2 k
4k
RE
+
VCE –VEE


1k (a) increases both the differential and common
300 mode gains.
(b) increases the common mode gain only.
(c) decreases the differential mode gain only.
(a) IC = 1 mA, VCE = 4.7 V (d) decreases the common mode gain only.
(b) IC = 0.5 mA, VCE = 3.75 V [EC-2005 : 2 Marks]
(c) IC = 1 mA, VCE = 2.5 V
Q.43 The circuit using a BJT with = 50 and
(d) IC = 0.5 mA, VCE = 3.9 V VBE = 0.7 V is shown in the figure. The base
[EC-2004 : 2 Marks] current I B and collector voltage V C are
Q.40 The cascade amplifier is a multistage respectively
configuration of
20 V
(a) CC-CB (b) CE-CB
2k Vo
(c) CB-CC (d) CE-CC 430 k
[EC-2005 : 1 Mark]

Q.41 For an npn transistor connected a shown in the


10 µF
figure, VBE = 0.7 Volts. Given that reverse
saturation current of the junction at room 1k 40 µF
temperature 300°K is 10–13 A, the emitter current
is

(a) 43 µA and 11.4 volts


IC
(b) 40 µA and 16 volts
(c) 45 µA and 11 volts
(d) 50 µA and 10 volts
VBE
[EC-2005 : 2 Marks]

(a) 30 mA (b) 39 mA Common Data for Questions (44 to 46):


(c) 49 mA (d) 20 mA In the transistor amplifier circuit shown in the figure
[EC-2005 : 2 Marks] below, the transistor has the following parameters:
dc = 60, VBE = 0.7 V, hje , hfe
Q.42 In an ideal differential amplifier shown in the
The capacitance CC can be assumed to be infinite.
figure, a large value of (RE)
90 Electronics Engineering Analog Electronics

(a) cut-off (b) saturation


12 V
(c) normal active (d) reverse active
1k
[EC-2007 : 2 Marks]
53 k
+ Q.48 The DC current gain ( ) of a BJT is 50. Assuming
5.3 k that the emitter injection efficiency is 0.995, the
VC
CC base transport factor is
Vs – (a) 0.980 (b) 0.985
(c) 0.990 (d) 0.995
[EC-2007 : 2 Marks]
In the figure above, the ground has been shown by the Statement for Linked Answer Questions (49 and 50):
symbol. In the following transistor circuit, V BE = 0.7 V,
Q.44 Under the DC conditions, the collector-to-emitter re = 25 mV/IE, and and all the capacitances are very
voltage drop is large.
(a) 4.8 Volts (b) 5.3 Volts
(c) 6.0 Volts (d) 6.6 Volts VCC = 9 V
[EC-2006 : 2 Marks] 3k
20 k
Q.45 If dc is increased by 10%, the collector-to-emitter CC 2
voltage drop
CC 1
(a) increases by less than or equal to 10%.
3k
IE
(b) decreases by less than or equal to 10%. 10 k
CE
(c) increases by more than 10%. 2.3 k
(d) decreases by more than 10%.
[EC-2006 : 2 Marks]

Q.46 The small signal gain of the amplifier Vc/Vs is


(a) –10 (b) –5.3 Q.49 The value of dc current IE is

(c) 5.3 (d) 10 (a) 1 mA (b) 2 mA

[EC-2006 : 2 Marks] (c) 5 mA (d) 10 mA


[EC-2008 : 2 Marks]
Q.47 For the BJT circuit shown, assume that the of
the transistor is very large and VBE = 0.7 V. The Q.50 The mid-band voltage gain of the amplifier is
mode of operation of the BJT is approximately
(a) –180 (b) –120
10 k
(c) –90 (d) –60
[EC-2008 : 2 Marks]

Q.51 A small signal source vi(t) = A cos20t + B sin106t


10 V
is applied to a transistor amplifier as shown
2V
1k below. The transistor has = 150 and hie = 3 k .
Which expression best approximates vo(t)?
GATE Previous Years Solved Paper 91

12 V
Q.53 In the silicon BJT circuit shown below, assume
that the emitter area of transistor Q1 is half that
of transistor Q2.
3k
100 k
V o(t )
10 nF R = 9.3 k Io
Vi(t)
10 nF
Q1 Q2
20 k
900 k 10 µF
( 1 = 700) ( 2 = 715)

–10 V
(a) vo(t) = –1500 (A cos20t + B sin106t) The value of current Io is approximately
(b) vo(t) = –150 (A cos20t + B sin106t)
(a) 0.5 mA (b) 2 mA
(c) vo(t) = –1500 B sin106t
(c) 9.3 mA (d) 15 mA
(d) vo(t) = –150 B sin106t
[EC-2010 : 1 Mark]
[EC-2009 : 2 Marks]
Common Data for Questions (54 and 55):
Q.52 The amplifier circuit shown below uses a silicon Consider the common emitter amplifier shown below
transistor. The capacitors CC and CE can be with the following circuit parameters:
assumed to be short at signal frequency and the
= 100, gm = 0.3861 A/V, ro = , r = 259 ,
effect of output resistance ro can be ignored. If
Rs = 1 k , RB = 93 k , RC = 250 , RL = 1 k ,
CE is disconnected from the circuit, which one
C1 = and C2 = 4.7 µF
of the following statements is true?
V CC = 9 V +10 V

RC
RB
RC = 2.7 k C1
RB = 800 k C2
Vo
CC +
= 100 Rs
Vi Vo RL
CC
+ –
Vs Vs
RE = 0.3 k CE –
RI Ro

Q.54 The resistance seen by the source Vs is


(a) The input resistance Ri increases and the
(a) 250 (b) 1258
magnitude of voltage gain Av decreases.
(c) 93 k (d)
(b) The input resistance Ri decreases and the
[EC-2010 : 2 Marks]
magnitude of voltage gain Av decreases.
(c) Both input resistance Ri and the magnitude Q.55 The lower cut-off frequency due to C2 is
of voltage gain Av decreases. (a) 33.9 Hz (b) 27.1 Hz
(d) Both input resistance Ri and the magnitude (c) 13.6 Hz (d) 16.9 Hz
of voltage gain Av increases. [EC-2010 : 2 Marks]
[EC-2010 : 1 Mark]
92 Electronics Engineering Analog Electronics

Q.56 In the circuit shown below, capacitors C1 and


ib B C
C 2 are very large and shorts at the input
frequency. v i is a small input. The gain
r ro
magnitude vo / vi at 10 M rad/sec is

5V
E

10 µH 2k 1 nF
(a) 250 (b) 27.5
(c) 25 (d) 22.5
C2 [EC-2012 : 1 Mark]
Q1
+ Q.59 The voltage gain A v of the circuit shown
2.7 V
vo 2k below is

2k C1 13.7 Volts
vi

12 k

C
(a) maximum (b) minimum vo
10 k
C
(c) unity (d) zero = 100
[EC-2011 : 1 Mark] 10 k
vi
Q.57 For the BJT Q1 in the circuit shown below, = ,
VBEon = 0.7 V, VCEsat = 0.7 V. The switch is
initially closed. At time t = 0, the switch is
opened. The time t at which Q1 leaves the active (a) Av 200 (b) Av 100
region is (c) Av 20 (d) Av 10
5V [EC-2012 : 2 Marks]

0.5 mA Q.60 In the circuit shown below, the silicon npn


transistor Q has a very high value of . The
required value of R2 in k to produce IC = 1 mA is
Q1 5 µF
–5 V VCC = 3 V
t=0

4.3 k IC
R1 = 60 k
–10 V
Q
(a) 10 ms (b) 25 ms
(c) 50 ms (d) 100 ms R2
RE = 500
[EC-2011 : 2 Marks]

Q.58 The current ib through the base of a silicon npn


(a) 20 (b) 30
transistor is 1 + 0.1 cos(10000 t) mA. At 300 K,
the r in the small signal model of the transistor (c) 40 (d) 50
is [EC-2013 : 2 Marks]
GATE Previous Years Solved Paper 93

Q.61 A good current buffer has Q.64 In the circuit shown, the pnp transistor has
(a) low input impedance and low output VBE = 0.7 V and = 50. Assume that
impedance.
RB = 100 k . For Vo to be 5 V, the value of RC
(b) low input impedance and high output
(in k ) ________ .
impedance.
(c) high input impedance and low output
impedance. RC
(d) high input impedance and high output
impedance. Vo

[EC-2014 : 1 Mark]

Q.62 For the amplifier shown in the figure, the BJT


RB
parameters are VBE = 0.7 V, = 200 and thermal VEE = 10 V
voltage, VT = 25 mV. The voltage gain (vo/vi) of
the amplifier is _______ .

12 V
[EC-2014 : 1 Mark]

Q.65 In the circuit shown, the silicon BJT has = 50.


RC 5k
33 k R1 Assume VBE = 0.7 V and VCE(sat) = 0.2 V. Which
vo one of the following statements is correct?
1 µF
1 µF
vi 10 V

11 k R2 Rs 10 RC

50 k
1k RE C 1 mF 5V
RB

[EC-2014 : 2 Marks]
(a) For RC = 1 k , the BJT operates in the
Q.63 A cascade connection of two voltage amplifiers
saturation region.
A1 and A3 is shown in the figure. The open-loop
(b) For RC = 3 k , the BJT operates in the
gain A vo , input resistance R in and output
saturation region.
resistance Ro for A1 and A2 are as follows:
(c) For RC = 20 k , the BJT operates in the
A1 : Avo = 10, Rin = 10 k , Ro = 1 k
cut-off region.
A2 : Avo = 5, Rin = 5 k , Ro = 200 k
(d) For RC = 20 k , the BJT operates in the linear
The approximate overall voltage gain vout/vin
region.
is
[EC-2014 : 2 Marks]

+ + Q.66 If the emitter resistance in a common-emitter


voltage amplifier is not by passed, it will
Vin A1 A2 1k RL Vo
(a) reduce both the voltage gain and the input
– – impedance.
(b) reduce the voltage gain and the input
[EC-2014 : 1 Mark] impedance.
94 Electronics Engineering Analog Electronics

(c) increase the voltage gain and reduce the Q.69 An increase in the base recombination of a BJT
input impedance. increase
(d) increase both the voltage gain and the input (a) the common-emitter dc current gain .
impedance. (b) the breakdown voltage BVCEO.
[EC-2014 : 1 Mark] (c) the unity gain cut-off frequency fT.
Q.67 Consider the common-collector amplifier in the (d) the transconductance gm.
figure (bias circuitry ensures that the transistor [EC-2014 : 1 Mark]
operates in forward active region, but has been Q.70 A BJT in a common base configuration is used
omitted for simplicity). Let IC be the collector to amplify a signal received by a 50 antenna.
current, VBE be the base-emitter voltage and VT Assume kT/q = 25 mV. The value of the collector
be the thermal voltage. Also, gm and ro are the bias current (in mA) required to match the input
small signal transconductance and output impedance of the amplifier to the impedance of
resistance of the transistor, respectively. Which the antenna is ______ .
one of the following conditions ensures a nearly
[EC-2014 : 2 Marks]
constant small signal voltage gain for a wide
range of values of RE ? Q.71 In the circuit shown, I1 = 80 mA and I2 = 4 mA.
Transistors T1 and T2 are identical. Assume that
the thermal voltage VT is 26 mV at 27°C. At 50°C,
the value of voltage V12 = V1 – V2 (in mV) is
Vin
Vs
Vo

RE I2 I1

– +
V2 V 12 V1

(a) gm RE << 1 (b) IC RE >> VT


(c) gm ro >> 1 (d) VBE >> VT T2 T1
[EC-2014 : 2 Marks]

Q.68 For the common collector amplifier shown in


the figure, the BJT has high , negligible VCE(sat),
[EC-2015 : 2 Marks]
and VBE = 0.7 V. The maximum undistorted
peak-to-peak output voltage vo (in Volts) is Q.72 In the ac equivalent circuit shown, the two BJTs
_________ . are biased in active region and have identical
V CC = +12 V
parameters with >> 1. The open circuit small
signal voltage gain is approximately _______ .

R1 5k

1 µF
vi
Vo
1 µF
vo
R2 10 k
Vi
RE 1k

[EC-2014 : 2 Marks] [EC-2015 : 2 Marks]


GATE Previous Years Solved Paper 95

Q.73 In the circuit shown in the figure, the BJT has a


VCC = 2.5 V
current gain ( ) of 50. For an emitter base voltage
VEB = 600 mV, the emitter collector voltage VEC
(in Volts) is ______ .
1 = 100
3V
Q1 Q2
10 k
2 = 50

60 k V C2
500 1V
1k

[EC-2015 : 1 Mark]

Q.74 The Ebers-Moll model of a BJT is valid


[EC-2016 : 1 Mark]
(a) only in a active mode.
(b) only in active and saturation modes. Q.77 In the figure shown, the npn transistor acts as a
switch.
(c) only in active and cut-off modes.
(d) in active, saturation and cut-off modes. 5V
Vin(t)
[EC-2016 : 1 Mark] 4.8 k
2V
Q.75 Resistor R 1 in the circuit below has been
12 k
adjusted so that I 1 = 1 mA. The bipolar
+
transistors Q1 and Q2 are perfectly matched and
Vin(t )
have very high current gain, so their base T –
currents are negligible. The supply voltage VCC 0 t(sec)
is 6 V. The thermal voltage kT/q is 26 mV.
For the input Vin(t) as shown in the figure, the
VCC
transistor switches between the cut-off and
saturation regions of operation, when T is large.
R1 I2 Assume collector-to-emitter voltage at
saturation VCE(sat) = 0.2 V and base-to-emitter
voltage VBE = 0.7 V. The minimum value of the
Q1 Q2
common-base current gain ( ) of the transistor
for the switching should be ______ .
I1 R2
[EC-2017 : 2 Marks]

Q.78 For the dc analysis of the common-emitter


amplifier shown, neglect the base current and
The value of R2 (in ) for which I2 = 100 µA is assume that the emitter and collector currents
_______ . are equal. Given that, VT = 25 mV, VBE = 0.7 V,
[EC-2016 : 1 Mark] and the BJT output resistance ro is practically
infinite. Under these conditions, the mid-band
Q.76 Consider the circuit shown in the figure.
Assuming VBE1 = VEB2 = 0.7 volt , the value of voltage gain magnitude, Av = vo / vi V/V, is
the dc voltage VC2 (in Volt) is ______ . _________ .
96 Electronics Engineering Analog Electronics

VCC = 12 V 5V

RC 2k
1k
73 k R1 C2
vo
C1 +
10 µ F
Q1
10 µF
RL 8k Vo vs
Vi 47 k R2
RE 2k CE
100 µF –
Q2

RB2
[EC-2017 : 2 Marks]

Q.79 Consider the circuit shown in the figure. –5 V


Assume base-to-emitter voltage VBE = 0.8 V and
[EC-2017 : 2 Marks]
common-base current gain ( ) of the transistor
is unity. Q.81 For the BJT in the amplifier shown below.
VBE = 0.7 V, kT/q = 26 mV. Assume the BJT
+18 V output resistance (ro) is very high and the base
current is negligible. The capacitors are also
assumed to be short circuited at signal
4k frequencies gain vo/vi of the amplifier is
44 k
VCC = 10 V

RC 10 k
16 k 2k C1
vo
1 µF
RL 10 k
vi
The value of the collector-to-emitter voltage VCE
(in Volt) is ______ .
20 k RE CE 100 µF
[EC-2017 : 1 Mark]
VEE = –10 V
Q.80 In the circuit shown, transistor Q1 and Q2 are
biased at a collector current of 2.6 mA. (a) –178.85 (b) –256.42
Assuming, that transistor current gains are (c) –128.21 (d) –89.42
sufficiently large to assume collector current [EC-2020 : 2 Marks]
equal to emitter current and thermal voltage of
26 mV, the magnitude of voltage gain Vo/Vs in Q.82 In the voltage regulator shown below, Vt is the
the mid-band frequency range is ______ (upto unregulated at 15 V. Assume, VBE = 0.7 V and
second decimal place). the base current is negligible for both the BJTs. If
the regulated output Vo is 9 V, the value of R2 is
_______ .
GATE Previous Years Solved Paper 97

(a) Collector emitter terminals shorted


V1 = 15 V V0 = 9 V
(b) Emitter to ground connection open
(c) 10 k resistor open
R3 = 1 k
R1 = 1 k (d) Collector base terminals shorted
[EE-1994 : 1 Mark]

Q.3 One of the applications of current mirror is


(a) output current limiting
R2 (b) obtaining a very high current gain
Vz = 3.3 V
(c) current feedback
(d) temperature stabilized biasing
[EC-2020 : 2 Marks] [EE-1998 : 1 Mark]

Q.4 A NPN Si transistor is meant for low-current


ELECTRICAL EN GINEERIN G
audio amplification. Match its following
(GATE Previous Years Solved Papers) characteristics against their values:
Characteristics Values
SECTIO N - A
(a) VEBmax (P) 0.7 V
Q.1 Figure shown below, shows a common emitter (b) VCBmax (Q) 0.2 V
amplifier. The quiescent collector voltage of the (c) VCEmax (R) 6 V
circuit is approximately (S) 50 V

20 V
[EE-1998 : 2 Marks]
10 k
10 k Q.5 The enhancement type n-channel MOSFET is
represented by symbol
= 100
(a) (b)
5k
10 k

(b) (d)
20
(a) V (b) 10 V [EE-1999 : 1 Mark]
3
(c) 14 V (d) 20 V Q.6 In the circuit of figure, the value of the base
[EE-1991 : 1 Mark] current IB will be

Q.2 In the transistor circuit shown in figure. 5V


Collector to ground voltage is +20 V. Which of
5k
the following is the probable cause of error?
20 V
= 80
10 k +
IB 0.7 V
47 k 3k
– RE
+10 V
10 V
98 Electronics Engineering Analog Electronics

(a) 0 µA (b) 18.2 µA


3.3 3.3
(c) 26.7 µA (d) 40 µA (a) mA (b) mA
3.3 (3.3 + 0.33)
[EE-2000 : 2 Marks]
3.3 3.3
Q.7 An n-channel JFET having a pinch-off voltage (c) mA (d) mA
33 (33 + 3.3)
(Vp) of –5 V shows a transconductance (gm) of
1 mA/V when the applied Gate to source voltage [EE-2003 : 1 Mark]
(VGS) is –3 V. Its maximum transconductance
Q.10 For the n-channel enhancement MOSFET shown
(in mA/V), is in figure, the threshold voltage Vth = 2 V. The
(a) 1.5 (b) 2.0 radian current ID of the MOSFET is 4 mA when
(c) 2.5 (d) 3.0 the drain resistance RD is 1 k . If the value of RD
[EE-2001 : 2 Marks] is increased to 4 k , drain current ID will become

Q.8 The variation of drain current with Gate-to- 10 V


source voltage (ID – VGS characteristic) of a
MOSFET is shown in figure. The MOSFET is ID RD

ID

VGS
0
(a) 2.8 mA (b) 2.0 mA
(a) an n-channel depletion mode device (c) 1.4 mA (d) 1.0 mA
(b) an n-channel enhancement mode device [EE-2003 : 2 Marks]
(c) an p-channel depletion mode device Q.11 Two perfectly matched silicon transistors are
(d) an p-channel enhancement mode device connected as shown in figure. The value of the
[EE-2003 : 1 Mark] current I is

Q.9 In the circuit of figure, assume that the transistor +3 V

has hFE = 99 and VBE = 0.7 V. The value of 1k I


collector current I C of the transistor is
approximately
= 1000 = 1000
3.3 k
+0.7 V

IC

33 k
12 V –5 V

4V (a) 0 mA (b) 2.3 mA


3.3 k
(c) 4.3 mA (d) 7.3 mA
[EE-2004 : 1 Mark]
GATE Previous Years Solved Paper 99

Q.12 The transconductance g m of the transistor Q.15 The common emitter amplifier shown in the
shown in figure is 10 mS. The value of the input figure is biased using a 1 mA ideal current
resistance RIN is source. The approximate base current value is
V CC = 5 V
V CC
RC = 1 k
RC
10 k C Vo
Vo
C = 100
Vs = 50
+
Vin
1 mA
10 k –
1k C

(a) 0 µA (b) 10 µA
(a) 10.0 k (b) 8.3 k (c) 100 µA (d) 1000 µA
(c) 5.0 k (d) 2.5 k [EE-2005 : 2 Marks]
[EE-2004 : 2 Marks] Statement for Common Data Questions (16 and 17):
Q.13 The value of R for which the PMOS transistor in Assume that the threshold voltage of the n-channel
figure will be biased in linear region is MOSFET shown in figure is +0.75 V. The output
+4 V characteristics of the MOSFET are also shown.

VDD = 25 V
VT = –1 V

R = 10 k
R 1 mA

Vout

(a) 220 (b) 470 +


(c) 680 (d) 1200 Vin = 2 mV
[EE-2004 : 2 Marks] –

Q.14 Assume that the n-channel MOSFET shown in 2V


the figure is ideal, and that its threshold voltage
is +1.0 V the voltage Vab between nodes a and b is
1k 1k IDS = 4 (mA)
a
VGS = 4 V
4

D VGS = 3 V
G 3
10 V 2k Vab
S
VGS = 2 V
2V 2
VGS = 1 V
b 1

(a) 5 V (b) 2 V
(c) 1 V (d) 0 V 0 VDS (V)
[EE-2005 : 1 Mark]
100 Electronics Engineering Analog Electronics

Q.16 The transconductance of the MOSFET is Q.20 Two perfectly matched silicon transistor are
(a) 0.75 mS (b) 1 mS connected as shown in the figure. Assuming
(c) 2 mS (d) 10 mS the of the transistors to be very high and the
forward voltage drop in diodes to be 0.7 V, the
[EE-2005 : 2 Marks]
value of current I is
Q.17 The voltage gain of the amplifier is +5 V
(a) +5 (b) –7.5 1k
I
(c) +10 (d) –10
[EE-2005 : 2 Marks]
Q1 Q2
Q.18 Consider the circuit shown in figure. If the of
the transistor is 30 and ICBO is 20 nA and the
–5 V
input voltage is +5 V, the transistor would be
(a) 0 mA (b) 3.6 mA
operating in
(c) 4.3 mA (d) 5.7 mA
+12 V
[EE-2008 : 2 Marks]
2.2 k
Q.21 The transistor circuit shown uses a silicon
15 k transistor with VBE = 0.7 V, IC IE and a dc
Vi Q
current gain of 100. The value of Vo is
100 k +10 V

–12 V
50 k
(a) saturation region 10 k
(b) active region
(c) breakdown region
(d) cut-off region
Vo
[EE-2006 : 2 Marks]
100
Q.19 The common emitter forward current gain of the
transistor shown is F = 100.
+10 V
(a) 4.65 V (b) 5 V
1k (c) 6.3 V (d) 7.23 V
[EE-2010 : 2 Marks]

Q.22 The voltage gain Av of the circuit shown below


is
270 k
13.7 V
1k
10 V
12 k

The transistor is operating in C


10 k Vo
(a) saturation region
C
(b) cut-off region = 100
10 k
(c) reverse active region Vi
(d) forward active region
[EE-2007 : 1 Mark]
GATE Previous Years Solved Paper 101

(a) Av 200 (b) Av 100 Q.26 When a bipolar junction transistor is operating
in the saturation mode, which one of the
(c) Av 20 (d) Av 10
following statements is true about the state of
[EE-2012 : 2 Marks]
its collector-base (CB) and the base-emitter (BE)
Q.23 The transistor in the given circuit should always junctions?
be in active region. (a) The CB junctions is forward biased and the
(Take, VCE(sat) = 0.2 V, VBE = 0.7 V) BE junction is reverse biased.
The maximum value of Rc (in ) which can be (b) The CB junctions is reverse biased and the
used, is _______ . BE junction is forward biased.
(c) Both the CB and BE junctions are forward
RC biased.
(d) Both the CB and BE junctions are reverse
Rs = 2 k +
= 100 5V biased.

+ [EE-2015 : 1 Mark]
5V

Q.27 A transistor circuit is given below. The Zener
diode breakdown voltage is 5.3 V as shown
[EE-2014 : 2 Marks]
below. Take base to emitter voltage drop to be
Q.24 In the given circuit, the silicon transistor has 0.6 V. The value of the current gain is _____ .
= 75 and a collector voltage VC = 9 V. Then the
ratio of RB and RC is 10 V

15 V
220
RC 4.7 k
RB
Vc
0.5 mA

5.3 V
470

[EE-2015 : 1 Mark]

Q.25 In the following circuit, the transistor is in active [EE-2016 : 1 Mark]


mode and VC = 2 V. To get VC = 4 V, we replace
Q.28 The circuit shown in the figure uses matched
RC with RC . Then the ratio RC / RC is _____ .
transistors with a thermal voltage VT = 25 mV.
+10 V The base currents of the transistors are
negligible. The value of the resistance R in k
RC that is required to provide 1 mA bias current for
RB
Vc the differential amplifier block shown is ____ .
(Give the answer up to one decimal place).

[EE-2015 : 1 Mark]
102 Electronics Engineering Analog Electronics

10 k
+12 V a

+
Differential 10 k
15 V 1k
amplifier
– + RTh
10.7 V
1 µA
1 mA –
b

[EE-2018 : 2 Marks]

Q.31 The enhancement type MOSFET in the circuit


R below operates according to the square law.
µnCox = 100 µA/V2, the threshold voltage (VT) is
500 mV. Ignore channel length modulation. The
–12 V output voltage Vout is
VDD = 2 V
[EE-2017 : 2 Marks]
ID = 5 µA
Q.29 For the circuit shown in the figure below, it is

V Vo
given that, VCE = CC . The transistor has = 29
2 W 10 µm
=
and VBE = 0.7 V when the B-E junction is forward L 1 µm

biased.

VCC = 10 V
(a) 2 V (b) 500 mV
(c) 100 mV (d) 600 mV
4R
RB [EE-2019 : 2 Marks]

Q.32 Given, Vgs is the gate-source voltage, Vds is the


C
B drain source voltage and VTh is the threshold
= 29
voltage of an enhancement type NMOS
E transistor, the conditions for transistor to be
R biased in saturation are
(a) Vgs > Vth; Vds Vgs – Vth
(b) Vgs < Vth; Vds Vgs – Vth
(c) Vgs > Vth; Vds Vgs – Vth
For this circuit, the value of RB/R is
(d) Vgs < Vth; Vds Vgs – Vth
(a) 43 (b) 92
[EE-2019 : 1 Mark]
(c) 121 (d) 129
[EE-2017 : 2 Marks] Q.33 A common source amplifier with a drain
resistance, RD = 4.7 k , is powered using a 10 V
Q.30 In the circuit shown in the figure, the bipolar power supply. Assuming that the trans-
junction transistor (BJT) has a current gain conductance, gm is 520 µA/V, the voltage gain
= 100. The base-emitter voltage drop is a of the amplifier is closest to
constant, VBE = 0.7 V. The value of the Thevenin (a) 1.22 (b) –1.22
equivalent resistance RTh (in ) as shown in (c) 2.44 (d) –2.44
the figure is ______ (upto 2 decimal places). [EE-2020 : 1 Mark]
GATE Previous Years Solved Paper 103

SECTIO N -B VCC

Q.1 In the transistor amplifier shown in figure, the RC


R1
ratio of small signal voltage gain, when the Vo
CC
emitter resistor is by passed by the capacitor
‘Ce’ to when it is not by passed. + C1
(Assuming of simplified approximate R2
Vi
h-parameter model for transistor) is CE
RE

VCC

Rc
R1 (a) increases (b) decreases
Vo
Cc (c) is unaffected (d) drops to zero
Vi [EE-2001 : 1 Mark]
C1
Q.3 The magnitude of the mid-band voltage gain of
R2
the circuit shown in figure is (assuming hfe of
Re Ce the transistor to be 100)
+VCC

1k
(a) 1 (b) h fe C
Vo
(1 + h fe ) Re (1 + h fe ) Re 10 k
(c) (d) 1 + hfe = 100
hie hie C

[EE-1996 : 1 Mark]
Vi
1k C
Q.2 In the single stage transistor amplifier circuit
shown in figure, the capacitor CE is removed
then the ac small signal mid-band voltage gain
of the amplifier (a) 1 (b) 10
(c) 20 (d) 100
[EE-2014 : 1 Mark]
104 Electronics Engineering Analog Electronics

Electronics & Electrical Engineering


GATE Previous Years Solved Paper

A n swe rs & Expl a n a t i o n s

Answers
EC BJT Analysis

1. (b) 2. (c) 3. (d) 4. (d) 5. (d) 6. (a) 7. (b) 8. (a, b)

9. (5) 10. (1) 11. (c) 12. (c) 13. (d) 14. (Large) 15. (1) 16. (3.75)

17. (c) 18. (a-2, b-5, c-4) 19. (b) 20. (a) 21. (d) 22. (c) 23. (b)

24. (d) 25. (b) 26. (c) 27. (a) 28. (a) 29. (c) 30. (d) 31. (a)

32. (c) 33. (b) 34. (d) 35. (b) 36. (a) 37. (a) 38. (a) 39. (c)

40. (b) 41. (c) 42. (d) 43. (b) 44. (c) 45. (b) 46. (a) 47. (b)

48. (b) 49. (a) 50. (d) 51. (d) 52. (a) 53. (b) 54. (b) 55. (b)

56. (a) 57. (c) 58. (c) 59. (d) 60. (c) 61. (b) 62. (–233.6) 63. (34.72)

64. (1.07) 65. (b) 66. (b) 67. (b) 68. (9.4) 69. (b) 70. (0.50) 71. (83.15)

72. (–1) 73. (2.04) 74. (d) 75. (598.68) 76. (0.5) 77. (0.902) 78. (128) 79. (6)

80. (50) 81. (d) 82. (800)

Solutions
EC BJT Analysis

1. (b) 3. (d)
Cascade amplifier is the common-emitter C
followed by common base configuration. IC

2. (c)
B
Ic IB
gm =
VT
So, if Ic then gm , gm Ic
E
So, if the quiescent collector current Ic increases
then the transconductance gm also increases. So, overall of the composite transistor,
IC
= = 1+ 2+ 1 2
IB
= 10200
GATE Previous Years Solved Paper 105

4. (d) 6. (a)
In the BJT self bias circuit or potential divider
circuit provides the maximum bias stability.

7. (b)
RE
>> 1 ,
Vin = V gs RB

VCC
IC
gm = RC
Vgs
RTh
IC = IE = IE
1+ +
Vbe –
VTh
IE Re
1+ IE
gm = =
Vgs (1 + ) Vgs
ID = IE
I Simplified self bias circuit using Thevenin
gm = × D = gm theorem.
(1 + ) Vgs 1 +
Thevenin open-circuit voltage,
99
= × 2 mA/V VCC R1
1 + 99 VTh =
R1 + R2
gm = 1.98 mA/V
Thevenin internal resistance,
5. (d) R1 R2
RTh = R1 R2 = = RB
R1 + R2
Apply KVL to input mesh,
10 k 58 k 1/hoe 5k VTh = IBR + Vbe + IERE
Put, IE = IB + IC
VTh = IBRB + Vbe + (IB + IC) RE
Differentiate w.r.t. IC, keeping and Vbe constant,
Z01 = 20 k Zo
IB
Output admittance, 0 = ( RB + RE ) + RE + 0
IC
h fe hre
Y 0 = hoe IB RE
hie + Rs =
IC RB + RE
3 100 × 0
= 0.05 × 10 1+
2 × 10 3 + 10 × 10 3 S= Ib
Y 0 = 0.05 × 10–3 1
Ic
1 1
Z 01 = = = 20 k 1+
Y0 0.05 × 10 3 S=
Output impedance, RE
1+
RB + RE
Z 0 = Z01 5k
>> 1
20 × 5 RE
= 20 k 5k = =4k >> 1
20 + 5 RB + RE
106 Electronics Engineering Analog Electronics

Given that is very large.


S= So, Ib = 0
RE
RB + RE 1.65
So, V1.65 = 10 ×
1.65 × 1.85 + 6.5
R
S = 1+ B = 1.65 V
RE
Apply KVL at input mesh,
For better stability,
V1.65 = Vbe + IeRe
S 1
= 0.65 × Ie + 1 k
RB RE = 1.65
So, << 1; >> 1
RE RB Ie = 1 mA
Q is very large.
8. (a, b)
So, Ic = Ie
In common base (CB) amplifiers input
I = Ic
impedance (Zf ) is low and current gain ( ) is
= 1 mA
also low,
Ic 11. (c)
=
Ie The overall bandwidth of an n-stage turned
In common collector (CC) amplifiers output amplifier is
impedance (Zo ) is low and current gain ( ) is BW = B 2 1/ n 1
high,
Ie 12. (c)
= = 1+
Ib IB
+
9. Sol. Vi
– 100 k 200 k
Given that, b is very large. hie hfeIB

So, IC = IE
So, the current through 1 k resistance, +
I = 5 mA Vo
470 –
VDC = 10 – IR
= 10 – 5 × 103 × 1 × 103
VDC = 5 V
Vo I B (1 + h fe ) RE
AV = = 1
10. Sol. Vi I B [ hie + (1 + h fe )] RE

I 13. (d)
6.5 k 2k
DB
f =
WB2

10 V Thus, if WB will decrease f will increase.


1.85 k
14. Sol.
+
+ In order to reduce the harmonic distortion in an
Vbe
1.65 k V1.65 – amplifier is dynamic range has to be large.
1k Ie

GATE Previous Years Solved Paper 107

15. Sol. 19. (b)


For common-emitter configuration: Increase the voltage gain and decrease the input
impedance.
VT
Input resistance = Ri = The by-pass capacitor C across RE will act as
IB
short-circuit for ac signal. So, the by-pass
25 × 10 3 capacitor C will increase the voltage gain and
= =1k
25 × 10 6 decrease the input impedance.
16. Sol. 20. (a)
+12 V A common-emitter stage followed by common
1k base stage.

22. (c)
Loop-1
1k Upper 3-dB frequency,
10 k
0.35
B.W. = f H =
tr

23. (b)
1k
Loop-2
A22 + A32 + ....
IC cannot be 5.32 mA because IC = 5.32 mA will A1
make VCE negative which implies transistor is The total harmonic distortion is,
in saturation.
A22 + A32 + ....
Through KVL, T.H.D. =
A1
12IB + 2IC = 11.2 (Loop-1)
10IB – IC = 0.6 (Loop-2) 25. (b)
Upon solving, IC 3.75 mA
1 1 1 1
= + +
17. (c) B 1 2 3

ic ic2 Cascading of amplifier results in decrease of


gm = = higher cut-off frequency and increase in lower
Vbe Vbe1 + Vbe2
cut-off frequency.
ic2
= gm2 So, fH and fL
Vbe2 B.W. = fH – fL
So, B.W.
18. Sol.
a-2, b-5, c-4 26. (c)
(a) Cascade amplifier: The pass transistor in a series voltage regulator
(2) It provides a wide band amplifier in common collector configuration, the voltage
(b) Differential amplifier: gain is almost equal to 1.

(5) Provides high common mode voltage 27. (a)


rejection
io
(c) Darlington pair common collector amplifier: gm = , io I E2 = IC1
vi
(4) Has very high input impedance and very
high current gain IC1 = I B1 , IE2 = IC1
108 Electronics Engineering Analog Electronics

io I B1 , Vi = I B1 r 32. (c)

I B1 IC h fe = gm r
io
= = gm1 = 1 (as IC = I B )
vi I B1 r vi 1 1 33. (b)
fL = 20 Hz
28. (a)
fH = 1 kHz for single stage
Common mode gain, For cascaded stage,
VC = ACVi (Vi1 = Vi2 = Vi ) fL 20
fL = =
If Re is infinite then because of symmetry of 1/ n 1 /3
2 1 2 1
figure, VC becomes zero, = 39.2 Hz
ie1 = ie2 = 0 1/ n
fL = f H 2 1 = 0.5 k
ib2 << ic2
37. (a)
So, ic2 ie2
VCE = VCC – ICR2
3 = 6 – 1.5 mA × R2
30. (d)
1.5 mA × R2 = 3
15 V R2 = 2k
RC IC 1.5 mA
IB = = = 0.01 mA
150
RTh
When, = 200
10/3 IC = IB
5V VTh (as R1 is same IB remains same)
430 = 0.01 mA × 200
IC = 2 mA
VCE = VCC – ICR2
R2VCC 5
VTh = = × 15 = 5 V = 6 – 2 mA × 2 k
R1 + R2 15
VCE = 2 V
Since, is large,
38. (a)
IB 0, RTh = 5 10
VCE = VCC – ICRC
V 0.7 0.2 = 3 – IC × 1 k
IC = Th
RE IC = 2.8 mA
VTh 0.7 5 0.7 IC 2.8 mA
= = = , IB = C
=
RE 0.430 k IB 50
4.3 = 0.056 mA = 56 µA
= = 10 mA
0.430 k
39. (c)
31. (a) 5V

gm 2.2 k
Ai =
gb e + j (C e + Cc ) 4/5 k
where, Ce and Cc are the transistor capacitances
so, at high frequencies the current gain of 1V
300
bipolar transistor drops due to the transistor
capacitors.
GATE Previous Years Solved Paper 109

R2 1 44. (c)
VTh = × VCC = × 5 = 1 V
R1 + R2 5 Drawing DC equivalent circuit, the capacitor is
4 treated as open-circuit.
RTh = R1 R2 = k
5 12 V
1 0.7 RC
IC = 1 mA 1k
0.3 k
Rf IE
(Q is large IB 0)
VCE = VCC – ICRC – IERE 53 k +
(IE = IC as IB 0)
IC VCE
VCE = 5 – 2.2 × 1 – 0.3 × 1 IB
VCE = 2.5 V –

41. (c)
When two terminals of a transistor are shorted Applying KVL in base-emitter loop.
it acts as diode, Applying KVL in base-emitter loop,
12 – IERC – IB Rf – 0.7 = 0
VD
VT IE
I = Io e 1 12 – 0.7 = IE I k + 53 k
( + 1)
11.3 11.3
0.7 IE = =
3 53 1.87 k
= 10 13
e 1 × 26 × 10 1 1+ k
61
= 6.046 mA = 6 mA
= 49 mA VCE = VCC – IERC
= 12 – 6 mA × 1 k
42. (d) = 6V
Only common-mode gain depends on RE and
differential mode gain is independent of RE. 45. (b)
When increases by 10%, new = 66
43. (b)
11.3
IE = IC + IB IE = = 6.31 mA
53
= IB + IB = ( + 1) IB 1+ k
67
KVL in input loop gives,
VCE = VCC – IE × RC
VCC – VBE = IBRB + IERE
= 12 – 6.31 mA × 1 k
= IBRB + ( + 1) IBRE
= 5.7
VCC VBE % change in,
IB =
RB + ( + 1) RE
6 5.7
VCE = × 100 = 5%
20 0.7 6
=
430 k + 51 × 1 k
46. (a)
IB = 40 µA
Given circuit is a voltage shunt feedback
IC = IB = 50 × 40 µA
amplifier.
= 2000 µA = 2 mA
So, the approximate voltage gain is,
VC = VCC – ICRC
= 20 – 2 mA × 2 k Rf 53 k
Avf = = = 10
VC = 16 V Rs 5.3 k
110 Electronics Engineering Analog Electronics

47. (b) 49. (a)


Given, is large. So, IB = 0 and IE = IC The given circuit can be redesigned as shown
Assuming BJT is in active. below.
Applying KVL in base emitter loop, VCC = 9 V
2 – 0.7 = 1 k × IE
3k
IE = 1.3 mA
10 0.2 RTh
Now, ICsat =
1 k + 10 k

9.8 VTh
= 0.9 mA IE 2.3
11 k

As, ICactive > ICsat


So, BJT is in saturation. 10
VTh = ×9 = 3 V
10 + 20
48. (b)
10 × 20
RTh = = 6.67 k
IPC1 10 + 20
IPE Since is large, IB can be ignored,
3 (0.7)
P n P IE = = 1 mA
2.3 k
InE
50. (d)
Mid-band voltage gain,
RC RL
AVm =
re
+ – + –
RL (3 3)
VEB VCB AVm = = = 60
re 0.026
I PC1
Transport factor = = 51. (d)
I PE
Current in emitter is both due to holes and The best approx answer for output voltage Vo
electrons. is,
Neglect current due to electrons, Vo = Av vi
h fe Rc
I PC1
I × vi
= = PE = × hie
I PE IE
–150 (A cos20t + B sin106t)
I PE Since the coupling capacitor is large,
= = emitter efficiency
IE Vo –150 B sin106t
50 52. (a)
= = = 0.9853
51 × 0.995
By disconnecting emitter bypass capacitor CE:
50 (i) Input impedance increases by a factor of
= =
1+ 51 (1 + RE).
(ii) Magnitude of voltage gain Av decreases by
the same factor.
GATE Previous Years Solved Paper 111

53. (b) 55. (b)


The given circuit is a current mirror circuit in Lower cut-off frequency due to C2,
which the output current is a mirror image of 1
fL =
the input current if both the transistors are 2 ( RC + RL ) C 2
identical. 1
=
2 (250 + 1000) × 4.7 × 10 6
Ii R = 9.3 k = 27.1 Hz
Io

56. (a)
Q1 Q2
Whenever we use a bypass capacitor in parallel
( 1 = 700) ( 2 = 715) RE in a BJT biased with “common emitter with
RE” then it always increases voltage gain. As
compare to “CE with RE without capacitor”.
–10 V Hence only answer (a) is possible,

To calculate Ii, 1 1
and =
=
9.3Ii + 0.7 = 0 – (10) = 10 LC 10 × 10 6 × 10 9
Ii = 1 mA = 10 M rad/sec
Since the emitter area of transistor Q1 is half and gain is maximum at resonance frequency.
that of transistor Q2.
57. (c)
I
So, Ii = o In active region,
2
–5 – 0.7 – 4.3IE = –10
Therefore, Io = 2 mA
10 5.7 4.3
IE = = = 1 mA
54. (b) 4.3 4.3
Equivalent model of the given circuit is shown IC = IE = I + 0.5 mA = 1 mA
below: I = 0.5 mA
C2 5V
B Ib C
0.5 mA

Rs I
VC
RB hie +
Ib RC RL
+ Q1 0.7 Vsat
+ 5 µF
Vs –5 V t=0

– 0.7 V –
active
E

–10 V
100 In saturation region:
h ie = = = 259
gm 0.3861 VC – 0.7 – 4.3 × 1 = –10
The resistance seen by the source Vs, VC = –5 V
Rin = Rs + ( RB hie ) q = CVC
= –5 × 10–6 × 5 V
93000 × 259 = –25 × 10–6
= 1000 +
(93000 + 259) and q = it, I (0 – t) = –25 × 10–6
Rin = 1000 + 258 = 1258 25 × 10 6
t= = 50 msec
0.5 × 10 3
112 Electronics Engineering Analog Electronics

58. (c) 100


Rm = = 0.245
We know that, 1 + 407.1
r = ( + 1) re
Ri = r Rm
V
r = ( + 1) T
Ie = 2.63 0.245 = 0.224
VT Vo Vo Ri
r = ( + 1) = ×
( + 1) I b Vi Vi Rs + Ri
VT 0.224
r = = 407.14 ×
Ib 10 + 0.224
Where Ib is dc current through base so Ib = 1 mA. = –8.92
VT = 25 mV at room temperature.
Av 10
25 × 10 3
So, r = = 25
1 × 10 3 60. (c)
Given that, is very large.
59. (d)
So, IC = IE = 1 mA
13.7 Volts VE = IERE
12 k = 1 × 10–3 × 500 = 0.5 V
VBE = 0.7 V
C
100 k
vo VR2 = VBE + VE
C
= 100 = 0.7 + 0.5 = 1.2 V
10 k
It is a self bias circuit.
vi V1 R2
So, VR2 = VCC ×
R1 + R2

KVL : R2
1.2 = 3 ×
–13.7 + 12 × 101IB × 100IB + VBE = 0 60 + R2
1312IB = 1 72 + 1.2 R2 = 3 R 2
IB 0.01 mA 1.8 R2 = 72
IC = IB = 1 mA R 2 = 40 k
IC
gm = = 38 m 62. Sol.
VT
100 12 V
r = = = 2.63 k 12 V
gm 38 m
Apply Miller’s theorem to 100 k resistor, 5k 5k
33 k
100
Rm = 8.25 k
1 Av
100
Rn = 100 k 11 k 3V
1 1.01 1.01
1
Av
V0
= gm RL = 38 × (12 100) By applying KVL in input loop, we get,
V1
3 = 8.25 k IB + 0.7 + 1.10 k IE
= –407.14
GATE Previous Years Solved Paper 113

8.25 k 64. Sol.


2.3 = IE + 1.01 k
201 Applying KVL in input loop, we get,
= IE × 1.051 k 10 = 0.7 + IBRB
IE = 2.2 mA 9.3
IB = = 0.093 mA
AC equivalent model of amplifier is given as, 100 k
ib Hence, IC = IB = 4.65 mA
+ + Vo 0 5
where, IC = =
Vi re = 2.27 k ib Vo RC RC
– – 5
R1 R2 5k
RC = = 1.07 k
4.65
0.01 k
65. (b)

10 V
where, re is given by
VT 25 mV
re = = = 11.36 RL
I E 2.2 mA
V BE = 0.7 V
Vo = –5 ib ...(i) 50 k
5V V CEsat = 0.2 V
Vi = 2.27 k ib + 0.01 k (ib + ib) RB = 50
S
...(ii)
From equations (i) and (ii), we get
Vo 5 k × 200 Assume the transistor is in active region,
= = 233.6
Vi 4.28 k

63. Sol.
50 k
R 01 R 02 5V
+
+ IBactive
+ 0.7 V –
+
Vin R in1 + AV V2 Rin2 + AV V RL Vo
– 01 – 02 in



– 5 0.7
IBactive = = 86 µA
50 k
Vo RL
= ...(i) ICactive = IBactive
AVo2 × V2 RL + Ro2
= 50 × 86 µA = 4.3 mA
AVo1 × Vin Rin 2 Assume, Re = 1 k
where, V2 = ...(ii)
Rin 2 + Ro1 10 V

From equations (i) and (ii), we get,


1k 4.3 mA
Vo RL Rin 2
= AVo2 × AVo1 × ×
Vi RL + Ro2 Rin 2 + Ro1 OFF
VC = 5.7 V
Vo 1k 5k 50 k
= 5 × 10 × × 5V
Vi 1 k + 0.2 k 5k +1k
VB = 0.7 V
Vo
= 34.72
Vi
It is in saturation.
114 Electronics Engineering Analog Electronics

Assume, Rc = 3 k 68. Sol.


10 V V CC = +12 V

3k 4.3 mA

ON R1 5k
VC = –2.9 V
50 k 1 µF
5V vi
1 µF
VB = 0.7 V vo
R2 10 k
RE 1k
It is in active.

66. (b)
DC analysis,
For unbypassed RE,
VCC × R2 12 × 10 k
R i = re + (1 + ) RE (VB)q = = =8V
R1 + R2 15 k
AI RL
and Av = (VE)Q = VB – 0.7 = 8 – 0.7 = 7.3 V
Ri
(VCE)Q = (VC)Q – (VE)Q
67. (b) = 12 – 7.3 = 4.7 V
AC equivalent circuit for given common
collector amplifier is, 4.7 V

(VCE )Q
+ VCE = Vo
re ib 4.7 V

Vin
Vo Vo(p – p) = 2 × 4.7 = 9.4 V

RE 70. Sol.
– The input impedance of CB-amplifier is,
Z i = re
Vin = reib + (1 + ) ibRE ...(i) So, according to given condition,
Vo = (1 + ) RE ib re = 50
Vo (1 + ) RE VT
Then, = = 50
Vin re + (1 + ) RE
IC
25 mV
Vo RE RE IC = = 0.50 mA
= = 50
Vin re + RE re + RE
The condition for small signal voltage gain to 71. Sol.
be nearly constant is, VBE2
R E >> re
I2 = I s e VT
V
R E >> T VBE2 = V2
IC
VBE1
ICRE >> VT
I1 = I s e VT
GATE Previous Years Solved Paper 115

VBE1 = V1 76. Sol.

V1 V2
2.5 V
I1 VT
= e
I2
Since, VT at 27° is 26 mV then VT at 50° is +
Loop-1
VCE 1
27.99 mV. +
0.7 V –
Thus, V12 = V1 – V2 – + 0.7 V
= 83.15 mV – IB2

10 k
72. Sol.
V C2
1V
Both the transistors are biased at the same 1k IC2
collector current thus their transconductance
will be same, the small signal equivalent circuit
is,
Vout Q1 : VCB = 0
+
We know,

V gm1Vi 1/gm2
VCB1 + VBE1 = VCE1
VCE1 = VBE1 = 0.7 V

KVL to loop 1,
–2.5 + VCE + 0.7 + IB × 10 k +1V=0
1 2
gm1 = gm2 2.5 1.4 1
IB = = 0.01 mA
Thus, Vo = –Vi
2 10 k
So, gain equal to –1. IC = 2 I B2 = 0.5 mA
2

73. Sol. VC 2 = IC2 × 1 k


VEB = 0.7 V
= 0.5 × 10–3 × 1 × 103
IB = 0.0383 mA
= 0.5 V
IC = 1.913 mA
VEC = 3 – ICRC 77. Sol.
= 3 – (1.916 × 0.5) +5 V
= 2.04 V
4.8 k
74. (d)
Ebers-Moll model is valid for all the region of 12 k
2V
operation.

75. Sol.

VT I 2 0.7
R2 = ln 1 IB = = 0.10833 mA
I2 I2 12
3 3 5 0.2
26 × 10 1 × 10 IC(sat) = = 1 mA
= 6
ln 6 4.8
100 × 10 100 × 10
IC (sat)
= 598.67 IB IB(min) =
116 Electronics Engineering Analog Electronics

1 mA +18 V
IB
RC 4k
1
0.10833 IC
and min = 9.23 RTh +
VTh VCE
min IB +
= = 0.902 VBE – –
min 1 + min
RE 2k
78. Sol.
IE
12 V

RC 2k 16
VTh = × 18 V = 4.8 V
16 + 44
IC
IERE = VTh – VBE – IBRTh
RTh
IB = 0 A (Q = 1))
VTh
IB = 0 + So, IERE = 4.8 – 0.8 = 4 V
VBE –
4
RE 2k IE = mA = 2 mA
2
IC = IE = 2 mA (Q = 1)
VCE = VCC – ICRC – IERE
By taking the Thevenin’s equivalent between = 18 – (2 × 4) – (2 × 2)
base and ground nodes, the given circuit can be = 6V
reduced as follows for DC analysis:
80. Sol.
12 × 47
VTh = = 4.7 V In AC equivalent circuit, Q2 becomes diode-
73 + 47
connected transistor because collector and base
Neglecting base current (IB = 0),
get shorted.
V VBE 4.7 0.7
IE = IC = Th = E
RE 2
IC = 2 mA
Q2 1/gm
IC 2
gm = = = 80 m
VT 25
Voltage gain,
Computer AC equivalent circuit is as shown
Av = gm ( RC RL )
below.
= 80(2 8) = 128

Av = 128 1k
Q1
79. Sol.
By taking the Thevenin’s equivalent between Vs
1/gm
base and ground nodes, the given circuit can be
reduced as follows:
GATE Previous Years Solved Paper 117

It is a CE amplifier with unbypassed RE,


I EQ 0.465
gm RL gm = = A/V
Av = VT 26
1 + gm RE
IC 2.6 Vout
where, gm = = = 100 m = gm ( Re RL )
VT 26 Vin

RL = 11 k
0.465
= × 5000 = 89.423
1 26
RE =
gm
82. Sol.
100 × 1
Av = = 50
1+1 R2
9× =4
Av = 50 R2 + 1 k
9R 2 = 4R2 + 4 k
81. (d) 5R 2 = 4 k
10 0.7 4000
IEQ = = 0.465 mA R2 = = 800
20 5

Answers
EE BJT Analysis (Section-A)

1. (c) 2. (b) 3. (d) 5. (a) 6. (b) 7. (c) 8. (c) 9. (b)

10. (c) 11. (c) 12. (d) 13. (d) 14. (d) 15. (b) 16. (b) 17. (d)

18. (b) 19. (d) 20. (b) 21. (a) 22. (d) 23. (22.32) 24. (105.13) 25. (0.75)

26. (c) 27. (19) 28. (172.7) 29. (d) 30. (90.09) 31. (d) 32. (c) 33. (d)

Solutions
EE BJT Analysis (Section-A)

1. (c) 5
VTh = × 20 V = 6.67 V
20 V 15

10 × 5
10 k RTh = = 3.33
10 + 5
Vc
VTh – Ib RTh – 0.7 – ( + 1) Ib × 10 k = 0
RTh
5.97 = Ib (3.33 + 101 × 10) k
Ib = 5.891 µA
VTh Ic = Ib = 0.589 mA
10 k
Vc = 20 – 10 × Ic
= 14.11 V 14 V
118 Electronics Engineering Analog Electronics

2. (b) 4 – 33 × Ib – 0.7 – 3.3 × 100 Ib = 0 (Ic = Ib)


20 – 10 k × IC = 20 4 0.7
Ib =
IC = 0 which gives, (33 + 330)
IE = IC = IB = 0
3.3 × 100 3.3
That is only possible when emitter connection Ic = = mA
33 + 330 (0.33 + 3.3)
is open. In other cases there will be base current.
10. (c)
4. Sol.
ID = K(VGS – Vth)2
(a) (P)
4 = K(6 – 2)2
(b) (R)
1
(c) (Q) K= mA/V 2
4
6. (b) VGS = 10 – 4 × 1 = 6 V
0 – 0.7 – Ib ( + 1) × 6.3 = –10 V when RD is increased to 4 k ,
10 0.7 VGS = 10 – 4ID
Ib = = 18.452 µA
81 × 6.3 1
ID = (10 4 I D 2)2
4
7. (c)
2
4ID = 16 ID + 64 64 ID
2 IDSS VGS
gm = 1 2
Vp Vp 16 ID + 68 I D + 64 =0
ID = 2.84 mA, 1.4 mA
2 IDSS 3
1 mA/V = 1 For MOSFET to be on, VGS must be greater than
5 5
Vth and this is possible only if,
2 3 ID = 1.4 mA
= × 1 IDSS
5 5 If, ID = 2.84 mA
2 then VGS become –ve and less than V th so
5
IDSS = mA
2 transistor will be off for this value which is not
2 I DSS 2 × 25 possible since VDG = 0 – Vth.
(gm)max = =
VP 4 × ( 5) 11. (c)
= 2.5 mA/V Both transistor are perfectly matched, hence
8. (c) VBE2 = VBE1

This is the characteristics of a p-channel Ic 1


= exp [(VBE1 – VBE2)/VT] = 1
depletion mode device. Ic 2
Also, ‘ ’ is same,
9. (b)
3V
Using KVL in base-emitter loop, IR
1k I = IC2
3.3 k

2IB

33 k = 1000 = 1000
12 V

4V
3.3 k
–5 V
GATE Previous Years Solved Paper 119

+5 0.7 16. (b)


IR = = 4.3 mA
1k IDS (4 3) mA
gm = = = 1 mS
writing KCL at node B, VGS (4 3) V
Ic + 2Ib – IR = 0
Ic = Ib 17. (d)

+
Ic = IR IR
+2
Vin
(Because is very large) gmVGS rd R Vo
Hence, Ic1 = Ic2 = IR = 4.3 mA

12. (d) –
h fe
Input resistance = = Since, rd >> R
gm gm All current will pass through R,
50 Vout = –gmVGS R
= =5k
10 × 10 3 = –1 × 10–3 × 2 × 10–3 × 10 × 10–3
But overall input resistance seen from source is = –20 mV
RIN = 10 10 5 = 2.5 k Vout 20 mV
Voltage gain = = = 10
Vin 2 mV
13. (d)
18. (b)
Here, VS = +4 V
VG = 0V Assume BJT is in active region and we neglect
VT = –1 V ICBO,
Therefore, VSG = 4V 12 V
VSD = VS – VD 2.2 k
= 4 – I aR
Now for linear region of operation, 13 k
VSD < (VSG + VT)
4 – Ia R < (4 – 1) 2.78 V
Ia R > 1
–3
10 × R > 1
R > 1000 12 × 15 5 × 100
Vth = + = 2.78 V
115 5
14. (d)
Rth = (15 100)
MOSFET is N-channel. Gate through source is
so connected that MOSFET will be in enhance 15 × 100
= = 13 k
mode and so conductivity of the channel will 115
be increased very much and effectively ‘ ’ Ic = Ib
terminal act as short circuited. So Vab = 0 V. 2.78 0.7
But, Ib = = 0.16 mA
13
15. (b)
Ic = 4.8 mA
Ic 12 0.2
= But, Ic(sat) = = 5.36 mA
Ib 2.2 k
3
1 × 10 As, Ic(sat) > Ic(active)
Ib = = 10 µA
100 BJT is in active region.
120 Electronics Engineering Analog Electronics

19. (d) 21. (a)


We assumed BJT is in active region, applying KVL :
KVL in base-emitter circuit, 10 = 10 kIb + 0.7 + (100) (Ib) 100
10 – 0.7 = 1 k × Ic + 270 × Ib 9.3
Ib = = 0.465 mA
(Assuming IC = IE) 20 k
= Ib (270 + 100 k ) V0 = 100 × 0.465 × 100
9.3 = 4.65 V
Ib = mA
370
22. (d)
93
Ic = mA
37 13.7 V
10 0.2
Ic(sat) = = 4.9 mA
2k 12 k

Ic(sat) > Ic(active)


C
BJT is in active region. Vo
10 k
C
20. (b) = 100
Since both transistor are perfectly matched. 10 k
Vi
So, VBE1 = VBE2

1k IR +5 V
P
IC Equivalent A.C. model will be taking,
IC h ie = 1 k
h fe = = 100
i 10 k ib

Q1 Q2
IB IB
Vi 100
k Vin hie hiei b 12 k
1 Av
–5 V

100
100 k as Av
1
1
Av
Ic 1 V VBE2
= exp BE1 = e0 = 1
Ic 2 VT h fe ib × (100 12) k
Vo
AV = =
Since for both are same, therefore, Vin hie × ib
Ib1 = Ib2 = Ib
100 × 10.71 k
Applying KVL to loop as shown, AV = = 1071.42
1k
0 0.7 0.7 ( 5)
IR = = 3.6 mA Vi = hie ib + 104 i
1k
= 103 ib + ib[104 + (1072.42) × 100]
By KCL, IR = Ic + 2Ib
Ic Vo h fe ib × 10.7142 × 103
= Ic + 2 AVS = =
Vi ib [10 3 + 10 4 + 10 2 × 1072.42]
= –9.06
Ic = × IR
+2
AVS = 9.06 10
IR (Because is very large)
GATE Previous Years Solved Paper 121

23. Sol. 25. Sol.


In input loop: Case-I: VC = 2 V
5 0.7 4.3 10 2
Ib = = = 2.15 mA iC = ...(i)
2k 2k RC
So, Ic = × Ib 10 – iBRB – 0.7 = 0
= 100 × 2.15 mA = 0.215 A 10 0.7
iB = ...(ii)
Now KVL in output loop, RB
VCE = 5 – 0.215 RC +10 V
For active region,
VCE > 0.2 V
RC
0.215 RC < 5 – 0.2 RB
4.8 Vc
RCmax = 22.32
0.215

24. Sol.
Consider the circuit shown in figure,
15 V
Case-II :
When, VC = 4 V
IE = IB + IC
RC RC
RC
10 4
RB IB iC = ...(iii)
RC
VC(9 V) From equation (iii) and (i),
IC
10 4 10 2
=
RC RC
+
0.7 V RC 6 3
– = = = 0.75
RC 8 4

VC = 9 V 27. Sol.
15 9 10 V
so, = IE
RC
6 220
= IE I1 4.7 k
RC
IB
9 0.7 VB
and = IB
RB
VE
8.3
so, = IB 5.3 V I2 = 0.5 mA
RB IE 470

IE 6 × RB
so, =
IB RC × 8.3
RB × 6 VB = 5.3 V
( + 1) = VE = VB – 0.6 = 4.7 V
RC × 8.3
VE
RB IE = = 10 mA
= 105.13 470
RC
122 Electronics Engineering Analog Electronics

10 5.3 5 = 30IB × 4R + 30 × IB × R
I1 = = 1 mA
4.7 k 5 = 150IB × IB × R ...(ii)
IB = I1 – I2 = 0.5 mA Using equation (i) and (ii),
IBRB = 9.3 – 5 = 4.2
IE
= + 1 = 20 and simultaneously putting value of IBR from
IB
equation (ii) in equation (i),
= 19
R
9.3 = I B R 150 + B
28. Sol. R
5 R
+12 V 9.3 = 150 + B
150 R
RB
279 = 150 +
Ra R
I0 = 1 µ A
RB
IR = 1 mA
= 129
R

Q1 Q2 30. Sol.

VBE1 VBE2 +15 V


R I0
10 k

–12 V

VBE1 = VBE2 + I0R 10 k

I0R = VBE1 – VBE2 1k


10.7 V
IR I0
= Vi ln VT ln
I3 I3 Rth
where, Is Reverse saturation current
To calculate Rth d.c. voltage should be short-
IR 1 mA circuited,
VT ln 0.025 ln
I0 1 µA
R= =
I0 1 µA
10 k
3
0.025 ln (10 )
= = 172.7 k
1 µA

29. (d)
In input loop, 10 k
1k
10 = (1 + ) Ib × 4R + IB × RB + 0.7
+ (1 + ) IB × R
Rth
10 = 30IB × 4R + IB × RB + 0.7
+ 30 × IB × R 10 k
9.3 = 150 × IB × R + IB × RB ...(i) Rth = 1 k
1+
Output loop,
= 1k 99.0099
10 = (1 + ) × IB × 4R + 5 V
+ (1 + ) × IB × R Rth = 90.09
GATE Previous Years Solved Paper 123

31. (d) VGS = 0.6 V


V0 = 600 mV
2V
32. (c)
ID = 5 µA
For NMOS transistor to be in saturation the
Vo condition will be
Vgs > Vth
+
VDS
and Vds Vgs – Vth
+
VGS –
– 33. (d)
Given data:
As, VDS = VGS RD = 4.7 k , gm = 520 µA/V
MOSFET is in saturation, Voltage gain of CS amplifier
= –gm RD
1 W
ID = µnC ox (VGS VT )2 = –520 µAV × 4.7 k
2 L
= –2.44
1 6
5 × 10–6 = × 100 × 10 × 10(VGS 0.5)2
2

Answers
EE BJT Analysis (Section-B)

1. (d) 2. (b) 3. (d)

Solutions
EE BJT Analysis (Section-B)

1. (d) 3. (d)
When Ce is unbypassed, AC model,
h fe Rc Vo
AV 1 = 10 k
hie
Vi hfeIB 10 k
When Ce is bypassed,
h fe Rc
AV2 =
hie + (1 + h fe ) Re Zi = 10 k

Z i = 10 k
AV 1 hie + (1 + h fe ) Re 1 + h fe
= = 1+ Re Mid-band voltage gain,
AV 2 hie hie
A1 RL
Av =
2. (b) Zi
h fe RL
AV 1 1 + h fe Av =
= 1+ Re Zi
AV 2 hie
100 × 10 k
AV 1 = = 100
>1 10 k
AV 2
AV2 < AV1 Av = 100
41 FET and MOSFET Analysis

ELECTRO NICS EN GINEERIN G Which of the following is correct?


(a) Only statement 1 is true.
(GATE Previous Years Solved Papers)
(b) Only statement 2 is true.
Q.1 Two identical FETs, each characterized by the
(c) Both the statements are true.
parameters gm and rd are connected in parallel.
(d) Both the statements are false.
The composite FET is then characterized by the
parameters. [EC-2002 : 2 Marks]

gm gm r Q.4 For an n-channel MOSFET and its transfer


(a) and 2rd (b) and d
2 2 2 curve shown in the figure, the threshold voltage
r is
(c) 2 gm and d (d) 2gm and 2rd
2 VD = 5 V
[EC-1998 : 1 Mark] D
ID
Q.2 In the MOSFET amplifier of the figure is the
characteristics

VG = 3 V
signal output V1 and V2 obey the relationship
Transfer

RD
VGS S
1V VS = 1 V
+

V1
(a) 1 V and the device is in active region.
+
+ (b) –1 V and the device is in saturation region.
V2 –
Vi –
RD/2 (c) 1 V and the device is in saturation region.

(d) –1 V and the device is in active region.
[EC-2005 : 2 Marks]
V V2
(a) V1 = 2 (b) V1 =
2 2 Q.5 In the CMOS inverter circuit shown, if the
(c) V1 = 2V2 (d) V1 = –2V2 transconductance parameters of the NMOS and
[EC-1998 : 1 Mark] PMOS transistors are:

Q.3 Consider the following statements in Wn W


Kn = K p = µnC ox = µ pC ox P = 40 µA/V 2
connection with the CMOS inverter in the figure, Ln LP
where both the MOSFETs are of enhancement and their threshold voltages are
type and both have a threshold voltage of 2 V. VTH n = VTH p = 1 V, the current I is
Statement-1 : T1 conducts when Vi 2 V.
5V
Statement-2 : T1 is always in saturation when
Vo = 0 V.
PMOS
+5V

T2 I
2.5 V
Vi Vo
NMOS
T1
GATE Previous Years Solved Paper 125

(a) 0 A (b) 25 µA The current Ix is related to Ibias as


(c) 45 µA (d) 90 µA (a) Ix = Ibias + Is
[EC-2007 : 2 Marks] (b) Ix = Ibias
(c) Ix = Ibias – Is
Q.6 Two identical NMOS transistors M1 and M2 are
connected as shown below. Vbias is chosen so Vout
(d) I x = I bias VDD
that both transistors are in saturation. The RE

Iout [EC-2008 : 2 Marks]


equivalent gm of the pair is defined to be
Vi
Statement for Linked Answer Question (8 and 9):
at constant Vout. Consider the CMOS circuit shown, where the gate
Iout voltage VG of the n-MOSFET is increased from zero,
Vout while the gate voltage of the p-MOSFET is kept constant
at 3 V. Assume that, for both transistors, the magnitude
Vbias M2
of the threshold voltage is 1 V and the product of the
transconductance parameter and the (W/L) ratio, i.e.
the quantity µCox(W/L), is 1 mA, V–2.
Vi M1 5V

3V
The equivalent gm of the pair is
Vo
(a) the sum of individual gm’s of the transistors.
(b) the product of individual g m ’s of the VG
transistors.
(c) nearly equal to the gm of M1.
(d) nearly equal to the gm/go of M2.
[EC-2008 : 2 Marks] Q.8 For small increase in VG beyond 1 V, which of
the following gives the correct description of the
Q.7 For the circuit shown in the following figure, region of operation of each MOSFET?
transistors M1 and M2 are identical NMOS (a) Both the MOSFETs are in saturation region.
transistors. Assume that M2 is in saturation and
(b) Both the MOSFETs are in triode region.
the output is unloaded.
(c) n-MOSFET is in triode and p-MOSFET is in
VDD saturation region.
(d) n-MOSFET is in saturation and p-MOSFET
RE is in triode region.
Ibias
V out [EC-2009 : 2 Marks]

Va Ix Q.9 Estimate the output voltage Vo for VG = 1.5 V.


1 1
M1 M2 (a) 4 V (b) 4 + V
2 2
Is
3 3
(c) 4 V (d) 4 + V
2 2
[EC-2009 : 2 Marks]
126 Electronics Engineering Analog Electronics

Q.10 In the circuit shown below, for the MOS VB


transistors, µnC ox = 100 mA/V 2 and the ID
threshold voltage VT = 1 V. The voltage Vx at the
source of the upper transistor is
M
6V

5V W/L = 4 (a) 12.5 (b) 25

Vx (c) 50 (d) 100


[EC-2013 : 2 Marks]
W/L = 1
Q.13 In a MOSFET operating in the saturation region,
the channel length modulation effect causes
(a) an increase in the gate-source capacitance.
(a) 1 V (b) 2 V (b) a decrease in the transconductance.
(c) 3 V (d) 3.67 V (c) a decrease in the unity-gain cut-off
[EC-2011 : 2 Marks] frequency.
(d) a decrease in the output resistance.
Q.11 In the CMOS circuit shown, electron and hole
[EC-2013 : 2 Marks]
mobilities are equal, and M1 and M2 are equally
sized. The device M1 is in the linear region if Q.14 For the n-channel MOSFET transistor shown in
figure the threshold voltage VTh is 0.8 V. Neglect
5V
channel length modulation effects. When the
M1 drain voltage VD = 1.6 V, the drain current ID
VTp = 1 V was found to be 0.5 mA. If VD is adjusted to be
2 V by changing the values of R and VDD, the
Vin
new value of ID (in mA) is
VTn = 1 V VDD
M2 R

(a) Vin < 1.875 V G


(b) 1.875 V < Vin < 3.125 V S
(c) Vin > 3.125 V
(d) 0 < Vin < 5 V [EC-2012 : 2 Marks] (a) 0.625 (b) 0.75
Q.12 The small signal resistance (i.e., dVB/dID) in kW (c) 1.125 (d) 1.5
offered by the n-channel MOSFET ‘M’ shown in [EC-2014 : 2 Marks]
the figure below, at a bias point of VB = 2 V is
Q.15 For the MOSFET shown in the figure, the
(device data for M, device transconductance
threshold voltage Vt = 2 V and
parameter kN = µnCOX ( W / L ) = 40 μA /V 2 ,
threshold voltage VTN = 1 V, and neglect body 1 W
K= µC = 0.1 mA/V 2 . The value of ID
effect and channel length modulation effects) 2 L
(in mA) is _______ .
GATE Previous Years Solved Paper 127

VDD = +12 V
Q.18 What is the voltage Vout in the following circuit?

VDD
R1 10 k

R2 10 k ID
10 k

Vout

V SS = –5 V

[EC-2014 : 2 Marks]

Q.16 For the MOSFET M 1 shown in the figure, (a) 0 V


assume W/L = 2, VDD = 2.0 V, µnCox = 100 mA/V2 ( VT of PMOS + VT of NMOS)
and VTh = 0.5 V. The transistor M1 switches from (b)
2
saturation region to linear region when Vin
(in Volts) is _______ . (c) Switching threshold of inverter
(d) VDD
VDD
[EC-2016 : 1 Mark]
R = 10 k

Vout Q.19 In the circuit shown in the figure, transistor M1


is in saturation and has transconductance
Vin M1
gm = 0.01 Siemens. Ignoring internal parasitic
capacitances and assuming the channel length
modulation to be zero, the small signal input
[EC-2014 : 2 Marks]
pole frequency (in kHz) is _______ .
Q.17 In the circuit shown, both the enhancement
VDD
mode NMOS transistor have the following
characteristics: kn = µnCox(W/L) = 1 mA/V2, 1k
VTN = 1 V. Assume that the channel length
Vo
modulation parameter is zero and body is
shorted to source. The minimum supply voltage 50 pF
VDD (in Volts) needed to ensure that transistor Vin M1
M1 operates in saturation mode of operation is 5k
________ .
VDD

[EC-2016 : 2 Marks]
M2
Q.20 In the circuit shown in the figure, the channel
length modulation of all transistor is non-zero
( 0). Also, all transistor operate in saturation
2V M1 and have negligible body effect. The ac small
signal voltage gain (Vo/Vin) of the circuit is

[EC-2015 : 2 Marks]
128 Electronics Engineering Analog Electronics

Q.22 An n-channel enhancement mode MOSFET is


VDD
biased at VGS > VTh and VDS > (VGS – VTh), where
VGS is the gate-to-source voltage. VDS is the
drain-to-source voltage and VTh is the threshold
M3 M2
voltage. Considering channel length
VG modulation effect to be significant, the MOSFET
behaves as a
Vo
(a) voltage source with zero output impedance.
Vin M1 (b) voltage source with non-zero output
impedance.
(c) current source with finite output
impedance.
(a) gm1 ( r01 r02 r03 ) (d) current source with infinite output
impedance.
1 [EC-2017 : 1 Mark]
(b) gm1 r01 r03
gm 3
Q.23 Assuming that transistors M 1 and M 2 are
identical and have a threshold voltage to 1 V,
1 the state of transistors M 1 and M 2 are
(c) gm1 r01 r02 r03
gm2 respectively
3V
1
(d) gm1 r01 r03 r02
gm 3 2.5 V M2

[EC-2016 : 2 Marks]

Q.21 For the circuit shown, assume that the NMOS 2V M1


transistor is in saturation. Its threshold voltage
Vtn = 1 V and its transconductance parameter

W (a) saturation, saturation


µnC ox = 1 mA/V 2 . Neglect channel
L (b) linear, linear
(c) linear, saturation
length modulation and body bias effects. Under
these conditions, the drain current ID (in mA), is (d) saturation, linear
_________ . [EC-2017 : 2 Marks]

VDD = 8 V Q.24 Two identical NMOS transistors M1 and M2 are


RD 1k connected as shown below. The circuit is used
R1 3M as an amplifier with the input connected
ID
between G and S terminals and the output taken
between D and S terminals, Vbias and VD are so
adjusted that both transistors are in saturation.
R2 5M The transconductance of this combination is
RS 1k
iD
defined as gm = while the output
vGS
[EC-2017 : 2 Marks]
GATE Previous Years Solved Paper 129

The voltage (in volts, accurate to two identical


v
resistance is ro = DS , where iD is the current places at Vx is) _______ .
iD
[EC-2018 : 2 Marks]
flowing into the drain of M2. Let gm1, gm2 be the
transconductances and r01, r02 be the output Q.26 A CMOS inverter, designed to have a mid-point
resistance of transistors M1 and M2, respectively. voltage V1 equal to half of Vdd, as shown in the
figure has the following parameters:
VD
D Vdd = 3 V
µnCox = 100 mA/V2; Vtn = 0.7 V for nMOS
Vbias M2
µpCox = 40 mA/V2, Vtp = 0.9 V for pMOS

The ratio of (W/L)n to (W/L)p is equal to ____


G M1 (rounded off to three decimal places).

Vout
S

Which of the following statements about Vdd


estimates for gm and r0 is correct?
(a) gm gm1 gm2 r02 and r0 r01 + r02
Vdd/2
(b) gm gm1 + gm2 and r0 r01 + r02
(c) gm gm1 and r0 r01 gm2 r02
(d) gm gm1 and r0 r02
Vin
[EC-2018 : 1 Mark] V Vdd
V1 = dd
2
Q.25 In the circuit shown below, the (W/L) value for
M 2 is twice that for M 1 . The two nMOS [EC-2019 : 2 Marks]
transistors are otherwise identical. The
threshold voltage VT for both transistors is 1.0 V. Q.27 In the circuit shown, the threshold voltages of
Note that VGS for M2 must be > 1.0 V.
the pMOS Vtp and nMOS (Vtn) transistors are
3.3 V both equal to 1 V. All the transistors have the
2.0 V M2 same output resistance rds of 6 M . The other
parameters are listed below:
Vx
W
µnCox = 60 µA/V2; =5
M1 L nMOS

W
µpCox = 30 µA/V2; = 10
L pMOS
Current through the nMOS transistors can be
µn and µp are the carrier mobilities, and Cox is
modeled as:
the oxide capacitance per unit area. Ignoring
W 1 2 the effect of the channel length modulation and
IDS = µC ox (VGS VT )VDS VDS
L 2 body bias, the gain of the circuit is _____
for VDS VGS – VT (rounded off to 1 decimal place).

W (VGS VT )2
I DS = µC ox for VDS VGS – VT
L 2
130 Electronics Engineering Analog Electronics

Vdd = 4 V (a) 10 V and –13 V (b) 13 V and –7 V


(c) 10 V and –10 V (d) 3 V and –3 V
[EC-2020 : 2 Marks]

Q.30 Using the incremental low frequency small-


signal model of the MOS device, the Norton
Vout equivalent resistance of the following circuit is
V DD

Vin R

gm, rds

[EC-2019 : 2 Marks]

Q.28 In the circuit shown, V1 = 0 and V2 = Vdd. The


1
other relevant parameters are mentioned in the (a) rds + R + gm rds R (b) rds + +R
figure. Ignoring the effect of channel length gm
modulation and the body effect, the value of Iout rds + R
(c) rds + R (d)
is _______ mA (rounded off to 1 decimal place). 1 + gm rds
Vdd [EC-2020 : 2 Marks]

Q.31 For the transistor M1 in the circuit shown in the


W/L = 10 W/L = 10 W/L = 40 figure, µnCox = 100 µA/V2 and (W/L) = 10, where
µn is the mobility of electron Cox is the oxide
Iout
Vdd capacitance per unit area. W is the width and L
V1 W/L = 5 W/L = 5 V2 is the length.
1 mA VDD = 3 V

RD = 20 k
W /L = 2 W /L = 3
Vout

M1
VGS

[EC-2019 : 2 Marks]

Q.29 An enhancement MOSFET of threshold voltage The channel length modulation coefficient is
3 V is being used in the sample and hold circuit ignored. If the gate-to-source voltage VGS is 1 V
given below. Assume that the substrate of the to keep the transistor at the edge of saturation,
MOS device is connected to –10 V. If the input then the threshold voltage of the transistor
voltage Vi lies between ± 10 V. The minimum (Rounded off to one decimal place) is _____ V.
and the maximum values of VG required for
[EC-2021 : 2 Marks]
proper sampling and holding respectively, are
Q.32 In the circuit shown in the figure, the transistors
Vi Vo
M1 and M2 are operating in saturation. The
channel length modulation coefficients of both
transistors are non-zero. The transconductance
VG of the MOSFETs M1 and M2 are r01 and r02,
respectively.
GATE Previous Years Solved Paper 131

(a) equal to 0 V (b) more than 2 V


VDD
(c) less than 2 V (d) equal to 2 V
[EC-2022]
Vin M2
Q.34 The ideal long channel nMOSFET and
Vout
pMOSFET devices shown in the circuits have
M1 threshold voltages of 1 V and –1 V, respectively.
The MOSFET substrates are connected to their
respective sources. Ignore leakage currents and
Ignoring the body effect, the ac small signal assume that the capacitors are initially
discharged. For the applied voltages as shown,
voltage gain ( Vout / Vin ) of the circuit is
the steady-state voltages are _______ .

1 5V
(a) gm1 r01 r02
gm 2

(b) –gm2 (r01 r02)


5V V1
1 nMOSFET
(c) gm 1 r01 r02 1 µF
gm 1

1
(d) gm 2 r02
gm1 –5 V

[EC-2021 : 1 Mark]

Q.33 Consider the CMOS circuit shown in the figure 5V V2


(substrates are connected to their respective pMOSFET 1 µF
sources). The gate width (W) to gate length (L)
ratios (W/L) of the transistors are as shown.
Both the transistors have the same gate oxide
(a) V1 = 5 V, V2 = 5 V
capacitance per unit area. For the pMOSFET,
(b) V1 = 5 V, V2 = 4 V
the threshold voltage is –1 V and the mobility
of holes is 40 cm2/V-s. For the nMOSFET, the (c) V1 = 4 V, V2 = 5 V
threshold voltage is 1 V and the mobility of (d) V1 = 4 V, V2 = –5 V [EC-2022]
electrons is 300 cm2/V-s. The steady-state Q.35 Select the correct statement(s) regarding CMOS
output voltage Vo is ______ . implementation of NOT gates:
(a) Noise Margin High (NMH) is always equal
4V
to the Nose Margin Low (NML), irrespective
of the sizing of transistors.
pMOSFET W (b) Dynamic power consumption during
=5
L
switching is zero.
Vo (c) For a logical high input under steady-state,
the nMOSFET is in the linear regime of
W operation.
=1
nMOSFET L
(d) Mobility of electrons never influences the
switching speed of the NOT gate.
[EC-2022]
132 Electronics Engineering Analog Electronics

Q.36 Consider an ideal long channel nMOSFET acts as a linear amplifier. vi is the small signal
(enhancement mode) with gate length 10 µm ac input voltage. vA and vB represent the small
and width 100 µm. The product of electron signal voltages at the nodes ‘A’ and ‘B’,
mobility (µn) and oxide capacitance per unit area respectively. The value of v A/v B is ______
(Cox) is µnCox = 1 mA/V2. The threshold voltage (Rounded off to one decimal place).
of the transistor is 1 V. For a gate-to-source VDD
voltage, VGS = [2 – sin(2t)] V and drain-to-source
voltage VDS = 1 V (substrate connected to the 4k
source), the maximum value of the drain-to-
A
source current is _______ .
(a) 40 mA (b) 20 mA
(c) 15 mA (d) 5 mA Vi B
[EC-2022]
2k
VGG
Q.37 Consider the circuit shown with an ideal long
channel nMOSFET (enhancement mode,
substrate is connected to the source). The [EC-2022]
transistor is appropriately biased in the
saturation region with VGG and VDD such that it

Electronics & Electrical Engineering


GATE Previous Years Solved Paper

A n swe rs & Expl a n a t i o n s

Answers
EC FET and MOSFET Analysis

1. (c) 2. (c) 3. (a) 4. (c) 5. (c) 6. (c) 7. (b) 8. (d)

9. (d) 10. (c) 11. (a) 12. (b) 13. (d) 14. (c) 15. (0.9) 16. (1.5)

17. (3) 18. (c) 19. (57.87) 20. (c) 21. (2) 22. (c) 23. (c) 24. (c)

25. (0.422) 26. (0.225) 27. (–900) 28. (6) 29. (b) 30. (d) 31. (0.5) 32. (c)
33. (c) 34. (c) 35. (c) 36. (c) 37. (–2)
GATE Previous Years Solved Paper 133

Solutions
EC FET and MOSFET Analysis

1. (c) 6. (c)
r I D1
2gm and d gm1 =
2 Vi
ID1 = ID2
rd I D2 I D1
µV gs I out
gm = = =
Vi Vi Vi
rd gm = gm1
µV gs

7. (b)
rd/2 µV gs It is a current mirror circuit,

r IG1 = IG2 = 0
So, rd = d
2 Ix = Is
µ = µ = gm rd
Ibias = Is + IG1 + IG2
rd
= gm = gm rd
2 = Is + 0 + 0
gm = 2g m Ix = Ibias

2. (c) 8. (d)
V1 = 2V2 When VG is little higher than 1 V.
V1 = IDRD • For n-MOSFET:
IS = ID VGSn = VG = 1+ V (1+ Little higher than 1)
R I R V VGSn – VTn = 0+ V (0+ Little higher than 0)
V2 = Is D = D D = 1
2 2 2 • For p-MOSFET:
So, V1 = 2V2
VSGP = VDD – 3 V = 2 V
3. (a) VSGP VTP = 1 V
Statement-2 is false because VDS will be less than
VGS = –VT, if Vo = 0. (VGSn – VTn) < (VSGP VTP )
So, n-MOSFET will be in saturation region
4. (c)
and p-MOSFET will be in triode region.
From the graph its clear that,
VTh = 1 V 9. (d)
VGS = 3 – 1 = 2 V When VG = 1.5 V:
VDS = 5 – 1 = 4 V
• For n-MOSFET:
Since, VDS VGS – VT
VGSn = VG = 1.5 V
So, MOSFET is saturation region.
VGSn – VTn = 0.5 V
5. (c) • For p-MOSFET:
VGS for each MOS is 2.5 V. VSGP = VDD – 3 V = 2 V
VT = 1 volt, device parameter K = 40 mA/V2
VSGP VTP = 2 V 1 V = 1 V
K
So, ID = (VGS VT )2
2 (VGSn VTn ) < (VSGP VTP )
= 20(2.5 – 1)2 = 45 µA
134 Electronics Engineering Analog Electronics

So, n-MOSFET will be in saturation region and Hence both MOS will be in saturation,
p-MOSFET will be in triode region. W
ID1 = µnC ox (VGS VT )2
To determine Vo : L
IDSn = ISDP
ID1 = µnCox(4) (4 – Vx)2
2
Kn(VGSn – VTn)2 = K p [2 (VSGP VTP ) VSDP VSDP ]
Similarly, ID2 = µnCox(1) (Vx – 1)2
Given that, Kn = K p
So, (0.5) = 2(1 V) (VDD – Vo) – (VDD – Vo)2
2
But, ID1 = ID2
0.25 = 2(5 – Vo) – (5 – Vo)2
µnCox(4) (4 – Vx)2 = µnCox(1) (Vx – 1)2
0.25 = 10 2 Vo 25 + 10 Vo Vo2 On solving, Vx = 3 Volt

Vo2 8Vo + 15.25 = 0 11. (a)


For PMOS, VSG = VS – VG = 5 – Vin
8 ± 64 4(15.25)
Vo = For PMOS to be ON,
2
VSG > VTP
3
= 4+ = 4.866 V or 3.134 V 5 – Vin > 1
2
Vin < 4
Check for valid Vo :
So, Vin must be less than 4 V for MOS to be in
• We know that, n-MOSFET is in saturation
linear regions so option (c) and (d) are rejected.
region and p-MOSFET is in triode region.
Now we know that for small Vin output is high
So, Vo VGSn – VTn
and PMOS is in linear region and NMOS is in
Both the possibilities of Vo satisfies this
cut-off region. Similarly for high Vin PMOS is in
and [VSDP = (5 – Vo)] > [VGSP VTP = 1 V] cut-off and NMOS is in linear region and for Vin
in between both are in saturation.
• For Vo = 4.866 V
So, PMOS will be in linear region for
[VSDP = 0.314 V] < [VSGP VTP = 1 V] Vin < 1.875 V.
• For Vo = 3.134 V
12. (b)
[VSDP = 0.866 V] > [VSGP VTP = 1 V] ID
gm =
VGS
3
So, the valid value of Vo is 4.866 V or 4 + V. VB = VG
2
VS = 0
10. (c) VGS = VG – VS
For upper MOS, = VG – 0 = VG = VB
VDS = 6 – Vx VB 1
=
VGS – VT = 5 – Vx – 1 ID gm
= 4 – Vx VD = VG
Upper MOS will be in saturation because, VD – VS = VG – VS
VDS > VGS – VT VDS = VGS
For lower MOS, VDS > VGS – VT
VDS = Vx So, the given MOSFET is in saturation region,
VGS – VT = Vx – 1
W
So, VDS > VGS – VT gm = µnCOX (VGS VT )
L
GATE Previous Years Solved Paper 135

gm = 40 × 10–6 × (2 – 1) So, for minimum VDD,


= 4 × 10–5 S VDS1 = VGS1 VTN
VB 1 1
= = VDS1 = 2 – 1 = 1 V
ID gm 4 × 10 5
VD1 = 1 V
100 × 10 3
= = 25 k
4 and ID1 = K (VGS VTN )2
1

14. (d) 1 mA
ID1 = × (2 1)2 = 1 mA
We know that, V2
ID = Kn(VGS – VT)2 [Q VDS > VGS – VT] Now, transistor M2,
0.5 mA VDS = 0 V
So, Kn = = 0.78 mA/V So, it will work into saturation region and same
(1.6 0.8)2
current will flow,
Therefore, for VD = 2 V
ID = 0.78(2 – 0.8)2 = 1.123 mA ID2 = ID = K (VGS VTN )2
1 2

15. Sol. 1 mA = 1 mA/V2 × (VDD – 1 – 1)2


( VS2 = VD1 )
ID1 = ID2 = Kn (VGS Vt )2 VDD = 3 V

1 W 19. Sol.
= µnC ox (VGS Vt )2
2 L With respect to A.C.
= 0.1(5 – 2)2 = 0.9 mA

16. Sol.
50 pF 1k
For saturation, Vo
ID = Kn(VGS – Vt)2 C
µnC ox W 5k
ID = (VGS Vt )2 Vin
2 L
1
ID = × 100 × 10 6 × 2 (Vo )2
2 Taking Miller’s equivalent and assume, ro =
6
ID = 100 × 10 Vo2 ...(i) 5k

Now, KVL in outer loop we get, R +


VDD = ID × 10 k + Vo ...(ii) Vin C(1 – Av) VSS C 1
1
1k
gmVgs Av
From equations (i) and (ii), we get

6 3
2 = 100 × 10 × 10 × 10 Vo2 + Vo

Vo2 + Vo 2 =0
Av = –gmRD
Vo = 1 V
= –0.01 × 103 = –10
VDS = VGS – Vt
Small signal input pole frequency,
Vin = 1.5 V
1
= 3 12
17. Sol. 2 × 5 × 10 × 50 × 10 (1 + 10)
Lower transistor (M1) to work in saturation, = 57.87 kHz
VDS1 = VGS1 VTN
136 Electronics Engineering Analog Electronics

20. (c) 22. (c)


VDD The small signal equivalent circuit of MOSFET
in saturation is as given below.
G D
+
M3 M2
Vgs gm3Vgs3 r0

Vo –
Vin M1
S
So, when the channel length modulation effect
is significant, the MOSFET can be modelled as
S3
Vgs 3 = 0 G3 a current source with finite output impedance.
G3
+ 23. (c)
D2
gm3Vgs3 r03 Vgs2 gm2Vgs2 r02
– 3V

D3
G1 D1 +
V01
Vin 2.5 V M2 V DS2
+
+ –
Vgs1 gm1Vgs1 r01 VGS2 –

+
2V M1 V DS1 = Vx
+ –
VGS1 –
Node equation at P1 :
Vo Vo Vo
gm1Vgs1 + + + gm2 Vgs2 = 0 VGS1 = 2 V
r01 r03 r02

1 1 1 VGS2 = 2.5 – Vx
Vo + + gm2 + = gm1Vin
r01 r02 r03 VDS1 = Vx

Vo 1 VDS2 = 3 – Vx
Av = = gm1 r01 r02 r03
Vin gm2 Assume both MOSFETs in saturation and equate
their currents,
21. Sol.
I DS1 = I DS2
8× 5
VGS = 1 × ID = 5 ID
8 kn k
(2 1)2 = n (2.5 Vx 1)2
(Here ID is numerically in mA) 2 2
ID = 5 – VGS ...(i) After solving, Vx = 0.5 V, 2.5 V
µnC ox W Vx cannot be 2.5 V.
ID = (VGS VT )2
2 L Because this will make M2 OFF.
1 2 Hence Vx may be 0.5 V.
5 – VGS = (VGS 1)
2 M1 is in linear region.
10 – 2VGS = 2
VGS + 1 2 VGS M2 is in saturation region.
2 = 9 VGS = 3 V To verify further,
VGS
ID = 5 – 3 = 2 mA
I DS1 = I DS2
GATE Previous Years Solved Paper 137

Vx2 k 25. Sol.


kn (2 1)Vx = n (2.5 Vx 1)2
2 2 µ nC ox W
Let, Kn =
2 L
VGS2 = 2.5 V Vx must be greater than VTh = 1 V
Given that,
After solving, Vx = 0.588 V, 1.91 V W W
Hence correct value of Vx = 0.588 V = 2
L 2 L 1
This verifies above conclusion i.e.
So, Kn2 = 2Kn1
M1 Linear region
M2 Saturation region For M1 :
VGS1 VT = 2 – 1 = 1 V
24. (c)
For M2 :
iD
D VGS2 VT = 2 – Vx – 1
= 1 V – Vx < 1 V
M2
VDS2 = (3.3 – Vx) > (VGS VT )
2
iD1 = iD So, M1 will be in linear region and M2 will be in
saturation region,
G M1
+
Vgs
ID1 = ID2

S 2
Kn1 [2(VGS1 VT )VDS1 VDS ] = Kn2 (VGS2 VT )2
1

Kn1 [2 (2 1)Vx Vx2 ] = 2Kn1 (2 Vx 1)2


ID i iD
gm = = D = 1 = gm1
Vin v gs v gs 2Vx Vx2 = 2 (1 + Vx2 2 Vx )
To calculate ro : 2
= 2Vx 4Vx + 2

+ + Ix 3Vx2 6Vx + 2 = 0
V gm1V 2 r02
2
2

Vx2 2Vx + =0
Ix 3
0A
Vx Vx 8
+
4
+
Vx = 1 ± 3 = 1± 1 V
V 1=0V gm2V ro1 Ixr01
1 4 3

– – VGS2 = (2 – Vx) VT
(1 – Vx) 0

V = –Ix r01 1
2 So, valid value, Vx = 1 = 0.4226 V
3
(Vx I x r01 )
Ix = gm2 v 2 + 26. Sol.
r02
Vx IDn = IDp and both will be in saturation.
r0 = = r01 + r02 + r01 r02 gm2 VDD
Ix If, Vin = = 1.5 V = VGSN = VSGP
2
r01 r02 gm2
1 W
(µnC ox ) [VGSN VTN ]2
2 L n
138 Electronics Engineering Analog Electronics

1 W
= (µ pC ox ) [VGSP + VTP ]2 1
2 L p R3 = rds
gm
6 W
100 × 10 [1.5 0.7]2 M3 M2
L n

W 1
= 40 × 10 6 [1.5 0.9]2 R4 =
gm
rds
L p

W M4
2
L n 40 (0.6)
= × = 0.225
W 100 (0.8)2
L p
G2 D2
27. Sol. +
M3 and M4 are identical PMOS transistor and
they have equal current. R4 R3 Vgs rds2

Hence their VSG should be equal.



Vdd = 4 V
Vgs = 0 S2

M1 is common source amplifier.


Vout
M1 M2 = Av = gm1 × ( rds2 rds1 )
Vin
= –300 M × 3 M
Vout
= –900

M4 M1 Vin 28. Sol.


V dd

M7 I5 M5 M6 I6
V
VSG3 = VSG4 = DD = 2 V
2 I3
Vdd
µ pC ox W
ISD = (VGS VT )2 V1 = 0
M3 I4 M4 V2 = VDD
2 L p I1 = 1 mA

30 I2
=× 10(2 1)2 = 150 µA
2 W/L = 2 M1 M2 W /L = 3
Now, by using current mirror property all
transistor should have equal current,
IDSN = ISDP = 150 µA
For M1 : I2 ( W / L )2 3
= =
I1 ( W / L )1 2
W
gm1 = 2µnC ox × IDS 3
L × I1 = 1.5 mA
I2 =
2
= 2 × 60 × 5 × 150 = 300 µ M3 is OFF because, V1 = 0 I3 = 0
M2, M3 and M4 from active load for M1. This active
M4 is ON because, V2 = VDD
load in equivalent to resistance rds2 i.e. 6 M . I5 = I4 = I2 = 1.5 mA
GATE Previous Years Solved Paper 139

I6 ( W / L )6 40 VDD = 3 V
= = =4
I5 ( W / L )5 10
ID RD = 20 k
I6 = 4I5 = 4 × 1.5 = 6 mA
Iout = I6 = 6 mA Vout
+
29. (b) M1 VDS
VGS –

Vi Vo
10 V
Edge of saturation,
VDS = VGS – VT
VG VDS = 1 – VT
–10 V
1 W
ID = µnC ox (VGS VT )2
For holding MOSFET should be OFF, 2 L
Vi min –10 V Applying KVL, we can write,
VG – Vi min < 3 VDS = VDD – IDRD
VG < 3 – 10 V –7 V 1
(1 – VT) = 3 × 100 µ × 10(1 VT )2 (20 k)
For sampling, 2
VG – Vimax > 3 1 – VT = 3 – 10(1 – VT)2
VG > 3 + Vimax
= 3 10 10 VT2 + 20 VT
VG > 13
10 VT2 21VT + 8 = 0
30. (d)

G D VT2 2.1VT + 0.8 = 0


+ (VT – 1.6) (VT – 0.5) = 0
V gm v rds VT = 1.6 V and VT = 0.5 V
Ix
To turn-on the transistor VGS > VT

Ix So, VT = 0.5 V
S R

32. (c)
Vx
Step-1 : AC model of the given circuit
(All DC sources = 0)

D2
V = –Vx V out
Vin M2 M1
Vx = (Ix – gmVx) rds + IxR
Vx (1 + gm rds) = (rds + R) Ix
S2 Req
V R + rds
RN = x =
VDD = 0
I x 1 + gm rds
Consider the circuit w.r.t M1,
31. (0.5)
µnCox = 100 µA/V2
M1
W
= 10
L Req
= 0
VGS = 1V 1
Req = r01 ...(1)
VT = ? gm1
140 Electronics Engineering Analog Electronics

Step-2: Small signal model of the given circuit, µnC ox W


× (VGSN VTN )2
2 L N
G2 D2
+ + µ pC ox W
= × (VSGP VTP )2
2 L P
Vin Vgs2 gm2Vgs2 r02 Req Vout 300 × 1 (Vo – 1)2 = 40 × 5(4 – Vo – 1)2
2
– – 3(Vo2 + 1 2Vo ) = 2(9 + Vo 6Vo )
S2 S2
Vo2 + 6Vo 15 = 0

6 ± 36 + 4 × 15 6 ± 96
Vo = =
G2 D2 Io 2 2
+ + Vo = 1.898 V, –7.89 V
Vo cannot be negative because Vo should lie
Vin Vgs2 gm2Vgs2 RL Vout
between 0 and 4 V.
Vo = 1.898 V
– –
S2 S2 34. (c)
5V

where, RL = r02 Req ...(2)

Vout = IoRL = –gm2 Vgm2 RL ...(3)


5V V1
Vout nMOSFET
= –gm2 RL ...(4) 1 µF
Vgs 2
But, Vgs2 = Vin
Vout nMOSFET will provide
= Av = gm 2 [ r02 Req ] V1 = 5 – VT = 4 V
Vin
–5 V
1
= gm2 r02 r01
gm1

5V V2
33. (c)
pMOSFET
Both MOSFETs are in saturation because drain 1 µF
is shorted to gate,
4V
pMOSFET will provide
V2 = 5 V
W
=5
L
35. (c)
Vo (a) NML = VIL – VOL
NMH = VOH – VIH
W
=1
L when, VTN = VTP
and Kn = K p
VDD
IDSN = ISDP VIT =
2
GATE Previous Years Solved Paper 141

and NML = NMH


Kp ID = µnC ox
W
L {
VGS Vt )VDS
1 2
2
VDS }
when, > 1, VIL , VIH
Kn W 2 1
IDmax = µnCox L (VGS max Vt )VDS 2 VDS
NMH and NML
Kp 2 1
when, > 1, VIL , VIH = 1 mA/V × 10 (3 1)1 ×1
2
Kn
= 10[1.5] = 15 mA
NMH and NML
i.e., NML and NMH depends on transistor 37. (–2)
sizing and they are equal for certain
Consider Va as an output then the small signal
condition only.
model is,
(b) Dynamic power consumption during
switching is non-zero due to capacitive
loading of next stage. 4k RD

(c) For : Va
VDD VTP Vin VDD PMOS cut-off
(Logic high input) NMOS Linear
(d) Switching speed depends on charging and Vi
discharging of load capacitor for pull up 2k RS
and pull down of output voltage
respectively. (CS in bypass amplifier)
Fast charging and discharging of capacitor
depends on mobility of charge carrier, Va gm RD
Voltage gain, = ...(1)
charging = RPMOS CL
Vi 1 + gm Rs
(to bring output at logic high) Consider Vb as an output, then the small signal
discharging = RNMOS CL model,
(to bring output at logic low)
and also propagation delay,
4k RD
PLH + PHL
p =
2
CL VDD
where, TPLH =
W
µ pC ox (VGS VTP )2 Vi Vb
L
CL VDD 2k RS
TPHL =
W
µ pC ox (VGS VTN )2
L (CD amplifier)
Using average charge model.
Vb gm Rs
Hence, (c) is only correct statement. Voltage gain, = ...(2)
Vi 1 + gm Rs
36. (c) On dividing equation (1) and (2),
VGS = [2 – sin2t] Volt 1V
Va /Vi RD
VGS min = 2–1=1V =
Vb /Vi Rs
VGS max = 2 – (–1) = 3 V
Va 4k
VDS = 1V = = 2
–1 V Vb 2k
VDS < VGS – Vt (Linear)
5 Frequency Response
Amplifiers
ELECTRO NICS EN GINEERIN G The input impedance of the feedback amplifier
with the feedback impedance Z connected as
(GATE Previous Years Solved Papers)
shown will be
Q.1 In a multi-stage RC-coupled amplifier the Z
coupling capacitor
(a) limits the low frequency response.
+ +
(b) limits the high frequency response.
Vi Vo
(c) does not effect the frequency response. N
– –
(d) blocks the d.c. components without effecting
the frequency response.
[EC-1993 : 1 Mark]
1
Q.2 An RC-coupled amplifier is assumed to have a (a) Z 1 (b) Z(1 – K)
K
single-pole frequency transfer function. The
maximum lower cut-off frequency allowed for Z Z
(c) (d)
the amplifier to pass 50 Hz. Square wave with K 1 1 K
no more than 10% tilt is ______ . [EC-1996 : 2 Marks]
[EC-1995 : 1 Mark]
Q.6 The fT of a BJT is related to its gm, C and Cµ as
Q.3 An amplifier has an open-loop gain of 100 and follows:
its lower and upper cut-off frequency of 100 Hz C + Cµ 2 (C + Cµ )
and 100 kHz respectively, a feedback network (a) fT = (b) fT =
gm gm
with a feedback factor of 0.99 is connected to
the amplifier. The new lower and upper cut-off gm gm
(c) fT = (d) fT =
frequencies are at ______ and ______ . C + Cµ 2 (C + Cµ )
[EC-1995 : 1 Mark] [EC-1998 : 1 Mark]

Q.4 An npn transistor has a beta cut-off frequency Q.7 An npn transistor (with C = 0.36 pF) has a unity
f of 1 MHz and common emitter short-circuit gain cut-off frequency fT of 400 MHz at a dc bias
low frequency current gain 0 of 200 at unity current IC = 1 mA. The value of its Cµ (in pF) is
gain frequency fT and the alpha cut-off frequency approximately (VT = 26 mV)
f respectively are (a) 15 (b) 30
(a) 200 MHz, 201 MHz (c) 50 (d) 96
(b) 200 MHz, 1999 MHz [EC-1999 : 2 Marks]
(c) 199 MHz, 200 MHz
Q.8 An amplifier is assumed to have a single-pole
(d) 201 MHz, 200 MHz
high-frequency transfer function. The rise time
[EC-1996 : 1 Mark] of its output response to a step function input is
Q.5 In the circuit shown in figure is a finite gain 35 nsec. The upper –3 dB frequency (in MHz)
amplifier with a gain of K, a very large input for the amplifier to a sinusoidal input is
impedance, and a very low output impedance. approximately at
GATE Previous Years Solved Paper 143

(a) 4.55 (b) 10 (a) 8 (b) 32


(c) 20 (d) 28.6 (c) 50 (d) 200
[EC-1999 : 2 Marks] [EC-2013 : 2 Marks]

Q.9 An npn BJT has gm = 38 mA/V, Cµ = 10–14 F, Q.12 Which one of the following statements is correct
C = 4 × 10–13 F, and DC current gain 0 = 90. For about an ac coupled common-emitter amplifier
this transistor fT and f are: operating in the mid-band region?
(a) fT = 1.64 × 108 Hz and f = 1.47 × 1010 Hz (a) The device parasitic capacitances behave
(b) fT = 1.47 × 1010 Hz and f = 1.64 × 108 Hz like open-circuits, whereas coupling and
(c) fT = 1.33 × 1012 Hz and f = 1.47 × 1010 Hz bypass capacitances behave like short-
(d) fT = 1.47 × 1010 Hz and f = 1.33 × 1012 Hz circuits.

[EC-2001 : 2 Marks] (b) The device parasitic capacitances, coupling


capacitances and bypass capacitances
Q.10 A bipolar transistor is operating in the active behave like open-circuits.
region with a collector current of 1 mA. (c) The device parasitic capacitances, coupling
Assuming that the of the transistor is 100 and capacitances and bypass capacitances
the thermal voltage (V T ) is 25 mV, the behave like short-circuits.
transconductance (gm) and the input resistance
(d) The device parasitic capacitances behave
(r ) of the transistor in the common emitter
like short-circuits, whereas coupling and
configuration, are
bypass capacitances behave like open-
(a) gm = 25 mA/V and r = 15.625 k circuits.
(b) gm = 40 mA/V and r = 4.0 k [EC-2016 : 1 Mark]
(c) gm = 25 mA/V and r = 2.5 k
Q.13 The Miller effect in the context of a common-
(d) gm = 40 mA/V and r = 2.5 k
emitter amplifier explains
[EC-2004 : 2 Marks]
(a) an increase in the low frequency cut-off
Q.11 The ac schematic of an NMOS common-source frequency.
stage is shown in the figure below, where part (b) an increase in the high frequency cut-off
of the biasing circuits has been omitted for frequency.
simplicity. For the n-channel MOSFET ‘M’, the (c) a decrease in the low frequency cut-off
transconductance gm = 1 mA/V, and body effect frequency.
and channel length modulation effect are to be
(d) a decrease in the high frequency cut-off
neglected. The lower cut-off frequency (in Hz)
frequency.
of the circuit is approximately at
[EC-2017 : 1 Mark]

RD = 10 k

C = 1 µF
Vi M
RL = 10 k
144 Electronics Engineering Analog Electronics

Electronics & Electrical Engineering


GATE Previous Years Solved Paper

A n swe rs & Expl a n a t i o n s

Answers
EC Frequency Response Amplifiers

1. (a) 2. (1.59) 3. (10) 4. (a) 5. (d) 6. (d) 7. (a) 8. (b)

9. (b) 10. (d) 11. (a) 12. (a) 13. (d)

Solutions
EC Frequency Response Amplifiers

1. (a) 5. (d)
The low frequency of operation of a multi-stage Miller’s Theorem:
RC coupled amplifier is limited by the coupling Miller’s theorem state that if a series impedance
capacitor. Z is connected between the input and output
terminal of a network, then it can be replaced
2. Sol. with a shunt impedance Z1 in the input side
fL and by a shunt impedance Z2 in the output
%Tilt = × 100%
f section of the network.
f × % Tilt Z
fL =
× 100
I1 I2
50 × 10 + +
= = 1.59 Hz
× 100 Vi Vo
N
3. Sol. – –

1 + A = 1 + 100 × 0.99
= 1 + 99 = 100
+ +
fL 100
fL = = = 1 Hz Vi Z1 Z2 Vo
1+ A 100 N
– –
f H = fH (1 + A )
= 100 × 103 × (100) = 10 MHz
If Vi > V2 then I1 flows:
4. (a)
V1 V2 V1 [1 V2 /V1 ] V1 [1 Av ]
fT = f I1 = = =
Z Z Z
f T = 200 × 1 = 200 MHz V1 Z
f = Z1 =
I1 1 Av
f = = (1 + ) f
1 Z
= (1 + 200) 1 = 201 MHz Z1 =
1 K
GATE Previous Years Solved Paper 145

6. (d) 38 × 10 3

Unity gain bandwidth product of the C.E. = 13


2 × 10 (0.1 + 4)
transistor,
= 1.47 × 1010 Hz
gm gm
fT = = fT 1.47 × 1010
2 (C + Cµ ) 2 (C e + Cc ) f = =
o 90
7. (a) = 1.64 × 108 Hz ( o = hfe)

1 10. (d)
fT =
2 RC
IC 1 mA
1 IC gm = =
= gm = VT 25 mV
R VT
= 0.04 = 40 mA/V
IC h fe = gm r , hfe =
fT =
2 VT × Cµ
100
r = = 3
= 2.5 k
1 mA gm 40 × 10
Cµ =
2 × 26 mV × 400 × 10 6
11. (a)
= 15 pF
1
8. (b) fL =
2 RC
tr × B.W. = 0.35 1
=
0.35 2 × 3.14 × (10 k + 10 k ) × 1 µF
B.W. = 9
= 10
35 × 10 = 8 Hz

9. (b) 13. (d)


gm Miller effect increases input capacitances and
fT =
2 (Cµ + C ) thereby decreases the higher cut-off frequency.
3
38 × 10
= 14 13
2 × (10 + 4 × 10 )
6 Feedback Amplifiers

ELECTRO NICS EN GINEERIN G (b) improves the signal to noise ratio at the
output.
(GATE Previous Years Solved Papers)
(c) does not effect the signal to noise ratio at
Q.1 The feedback amplifier shown in figure has the output.

VCC
(d) reduces distortion.
[EC-1993 : 1 Mark]

Q.4 To obtain very high input and output


impedances in a feedback amplifier, the mostly
used is
(a) Voltage-series (b) Current-series
(c) Voltage-shunt (d) Current-shunt
[EC-1995 : 1 Mark]

Q.5 Negative feedback in


1. voltage series configuration
2. current shunt configuration
(a) increase input impedance
(a) current-series feedback with large input
(b) decrease input impedance
impedance and large output impedance.
(c) increase closed loop gain
(b) voltage-series feedback with large input
(d) leads to oscillation
impedance and low output impedance.
[EC-1997 : 2 Marks]
(c) voltage-series feedback with low input
impedance and low output impedance. Q.6 In a shunt-shunt negative feedback amplifier,
(d) current-shunt feedback with low input as compared to the basic amplifier,
impedance and output impedance. (a) both input and output impedance
[EC-1989 : 2 Marks] decreases.
(b) input impedance decreases but output
Q.2 Two non-inverting amplifiers, one having a
impedance increases.
unity gain and the other having a gain of twenty,
are made using identical operational amplifiers. (c) input impedance increases but output
As compared to the unity gain amplifier, the impedance decreases.
amplifier with gain twenty has (d) both input and output impedance increases.
(a) less negative feedback [EC-1998 : 1 Mark]
(b) greater input impedance Q.7 Negative feedback in an amplifier
(c) less bandwidth (a) reduces gain
(d) none of the above [EC-1991 : 2 Marks] (b) increases frequency and phase distortions
Q.3 Negative feedback in amplifiers (c) reduces bandwidth
(a) improves the signal to noise ratio at the (d) increases noise
input. [EC-1999 : 1 Mark]
GATE Previous Years Solved Paper 147

Q.8 An amplifier has an open-loop gain of 100, an Q.12 The effect of current shunt feedback in an
input impedance of 1 k , and an output amplifier is to
impedance of 100 . A feedback network with (a) increase the input resistance and decrease
a feedback factor of 0.99 is connected to the the output resistance.
amplifier in a voltage series feedback mode. The (b) increase both input and output resistances.
new input and output impedances, (c) decrease both input and output resistances.
respectively, are
(d) decrease the input resistance and increase
(a) 10 and 1 (b) 10 and 10 the output resistance.
(c) 100 k and 1 (d) 100 k and 1 k [EC-2005 : 1 Mark]
[EC-1999 : 2 Marks]
Q.13 The input impedance (Z i ) and the output
Q.9 In a negative feedback amplifier using voltage impedance (Zo) of an ideal transconductance
series (i.e. voltage-sampling, series mixing) (voltage controlled current source) amplifier are
feedback (a) Zi = 0, Zo = 0 (b) Zi = 0, Zo =
(a) Ri decreases and Ro decreases (c) Zi = , Zo = 0 (d) Zi = , Zo =
(b) Ri decreases and Ro increases [EC-2006 : 1 Mark]
(c) Ri increases and Ro decreases
Q.14 In a transconductance amplifier, it is desirable
(d) Ri increases and Ro increases
to have
(Ri and Ro denote the input and output resistance
(a) a large input resistance and a large output
respectively.
resistance.
[EC-2002 : 1 Mark]
(b) a large input resistance and a small output
Q.10 An amplifier without feedback has a voltage resistance.
gain of 50, input resistance of 1 k and output (c) a small input resistance and a large output
resistance of 2.5 k . The input resistance of the resistance.
current-shunt negative feedback amplifier using (d) a small input resistance and a small output
the above amplifier with a feedback factor of resistance.
0.2 is
[EC-2007 : 1 Mark]
1 1
(a) k (b) k Q.15 In a voltage-voltage feedback as shown below,
11 5
which one of the following statements is true if
(c) 5 k (d) 11 k
the gain k is increased?
[EC-2003 : 2 Marks]
+ + +
Q.11 Voltage series feedback (also called series-shunt Vin V1 Ao Vout
– –
feedback) results in –

(a) increase in both input and output


impedances.
(b) decrease in both input and output
impedances. + +
Vf = kVout k
(c) increase in input impedance and decrease – –

in output impedances.
(a) The input impedance increases and output
(d) decrease in input impedance and increase
impedance decreases.
in output impedances.
(b) The input impedance increases and output
[EC-2004 : 1 Mark]
impedance also increases.
148 Electronics Engineering Analog Electronics

(c) The input impedance decreases and output Q.18 The feedback topology in the amplifier circuit
impedance also decreases. (the base bias circuit is not shown for simplicity)
(d) The input impedance decreases and output in the figure is
impedance increases. VCC
[EC-2013 : 1 Mark]
RC Io
Q.16 In the ac equivalent circuit shown in the figure,
if iin is the input current and RF is very large, the Vo
type of feedback is

RS

RE
RD Vs
RD
Vout

(a) voltage shunt feedback


M2
(b) current series feedback
M1 (c) current shunt feedback
RF (d) voltage series feedback
[EC-2014 : 1 Mark]
small signal iin
input Q.19 A good transconductance amplifier should
have
(a) high input resistance and low output
(a) voltage-voltage feedback
resistance.
(b) voltage-current feedback (b) low input resistance and high output
(c) current-voltage feedback resistance.
(d) current-current feedback (c) high input and output resistances.
[EC-2014 : 1 Mark] (d) low input and output resistances.
Q.17 The desirable characteristics of a [EC-2017 : 1 Mark]
transconductance amplifier are
Q.20 A good trans-impedance amplifier has
(a) high input resistance and high output
(a) low input impedance and high output
resistance. impedance.
(b) high input resistance and low output (b) high input impedance and high output
resistance.
impedance.
(c) low input resistance and high output
(c) high input impedance and low output
resistance. impedance.
(d) low input resistance and low output
(d) low input impedance and low output
resistance.
impedance.
[EC-2014 : 1 Mark] [EC-2018 : 1 Mark]
GATE Previous Years Solved Paper 149

Electronics & Electrical Engineering


GATE Previous Years Solved Paper

A n swe rs & Expl a n a t i o n s

Answers
EC Feedback Amplifiers

1. (c) 2. (a, c) 3. (b, c) 4. (b) 5. (1-a, 2-b) 6. (a) 7. (a) 8. (c)

9. (c) 10. (a) 11. (c) 12. (d) 13. (d) 14. (a) 15. (a) 16. (b)

17. (a) 18. (b) 19. (c) 20. (d) 21. (800)

Solutions
EC Feedback Amplifiers

1. (c) 5. Sol.
Emitter is output node, it is voltage sampler 1-a, 2-b
voltage shunt feedback. Voltage-series configuration increases the input
impedance,
2. (a, c)
R if = Ri(1 + A ) = Ri(1 + Av )
For identical operational amplifiers, gain- Current-shunt configuration decreases the input
bandwidth product is constant, impedance,
G.B.W. = Constant
Ri Rf
A1 × BW1 = A2 × BW2 R if = =
1+ A 1 + AI
A1 × BW1
BW2 =
A2 6. (a)
1 × BW1 BW2 Shunt-shunt feedback amplifiers has very low
= =
20 20 input and very low output impedance,
So, as compare to the unity gain amplifier with
Ri Ri
gain twenty has less bandwidth. R if = =
1+ A 1 + Rm
3. (b, d) Ro Ro
Rof = =
Negative feedback in amplifiers 1+ A 1 + Rm
(b) improves the signal to noise ratio at the
output. 7. (a)
(d) reduces distortion. A
Af =
1+ A
4. (b)
Current-series feedback amplifiers has very 8. (c)
high input and very high output impedances, A = 100, = 0.99, 1 + A = 100
R if = Ri (1 + A ) = Ri (1 + Gm ) For voltage series, Ri and Ro by 1 + A
Rof = Ro(1 + A ) = Ro (1 + Gm )
150 Electronics Engineering Analog Electronics

We have, R if = Ri (1 + A ) and, the output impedance decreases,


Ro Ro
and Ro = Rof =
1+ A 1 + Ao K
R if = 100 × 1 k = 100 k
16. (b)
100
Rof = =1 Sampling Current
100
Mixing Voltage
9. (c)
18. (b)
Ri increase by factor of 1 + A and Ro decreases
by 1 + A . The feedback topology in the amplifier circuit is
current series because,
10. (a) Sampling Current
In current shunt negative feedback amplifier, Mixing Series
Ri So, current series feedback.
R if =
1+ A
19. (c)
3 3
1 × 10 10 A good transconductance amplifier should
= =
1 + 50 × 0.2 1 + 10 have very high input resistance and very high
1 output resistance.
R if = k
11
20. (d)
15. (a) A good trans-impedance amplifier should have
The given configuration is a voltage-series low input impedance and low output
feedback configuration. impedance.
So, the input impedance increases,
R if = Ri (1 + AoK)
7 Oscillator Circuits

ELECTRO NICS EN GINEERIN G (a) Precision integrator


(b) Hartley oscillator
(GATE Previous Years Solved Papers)
(c) Butteworth high pass filter
SECTIO N - A (d) Wien-bridge oscillator

Q.1 Match the following: [EC-2000 : 1 Mark]


List-I List-II Q.4 The oscillator circuit shown in the figure is
A. Hartley 1. Low frequency oscillator –VCC
B. Wein-bridge 2. High frequency oscillator
LC
C. Crystal 3. Stable frequency oscillator
4. Relaxation oscillator
5. Negative resistance oscillator L = 10 µH
CC R1
[EC-1994 : 2 Marks] Vo

Q.2 Value of R in the oscillator shown in the given


figure. So chosen that it just oscillates at an C1 = 2 pF C2 = 2 pF
angular frequencies of ‘ ’. The value of ‘ ’ and R2
Re Ce
the required value of R will respectively be
100 k

5k
– (a) Hartley oscillator with foscillation = 79.6 MHz
Vo
+ (b) Colpitts oscillator with foscillation = 50.3 MHz
R (c) Hartley oscillator with foscillation = 159.2 MHz
(d) Colpitts oscillator with foscillation = 159.2 MHz
[EC-2000 : 2 Marks]
0.01 µF 10 mH 1k

Q.5 The circuit in the figure employs positive


feedback and is intended to generate sinusoidal
(a) 105 rad/sec, 2 × 104 oscillation.
(b) 2 × 104 rad/sec, 2 × 104
Vf ( f ) 1
(c) 2 × 104 rad/sec, 105 If at a frequency fo, B( f ) = = 0° then
Vo ( f ) 6
(d) 105 rad/sec, 105 [EC-1996 : 2 Marks]

Q.3 The configuration of the figure is a to sustain oscillation at this frequency.


R1 R2
R1 R2


Vo + +
+
R
Vo(f )
C + Network
Vf (f ) (f )

R C –
152 Electronics Engineering Analog Electronics

(a) R2 = 5R1 (b) R2 = 6R1 Q.8 The circuit shown in the figure has an ideal
R R op-amp. The oscillation frequency and the
(c) R2 = 1 (d) R2 = 1 condition to sustain the oscillations,
6 5
respectively are
[EC-2002 : 2 Marks]
R1
Q.6 The oscillator circuit shown in the figure has
an ideal inverting amplifier. Its frequency of R2
oscillation (in Hz) is –
Vo
+

C 2R
2C R

C C C

R R R 1
(a) and R1 = R2
CR
1
1 1 (b) and R1 = 4R2
(a) (b) CR
(2 6 RC ) (2 RC )
1
(c) and R1 = R2
1 1 2CR
(c) (d)
( 6 RC ) ( 6 (2 RC )
1
[EC-2003 : 2 Marks] (d) and R1 = 4R2
2CR
Q.7 The value of C required for sinusoidal [EC-2015 : 2 Marks]
oscillations of frequency 1 kHz in the circuit of
Q.9 Consider the oscillator circuit shown in the
the figure is
figure. The function of the network (shown in
1k 2.1 k dotted lines) consisting of the 100 k resistor in
series with the two diodes connected back-to-
back is to

Vout 1 nF 158 k
+
C

1k 1 nF 158 k
+VCC

1k C +
–VCC
22.1 k

1 100 k
(a) µF (b) 2 µF D1
2
1 10 k D2
(c) µF (d) 2 6 µF
2 6
[EC-2004 : 2 Marks]
GATE Previous Years Solved Paper 153

(a) introduce amplitude stabilization by


+10 V
preventing the op-amp from saturating and Vi –
Vo
thus producing sinusoidal oscillations of +
fixed amplitude. –10 V

(b) introduce amplitude stabilization by


2k
forcing the op-amp to swing between
positive and negative saturation and thus
0.5 k
producing square wave oscillations of fixed
2k
amplitude.
(c) introduce frequency stabilization by forcing
the circuit to oscillate at a single frequency.
Vo
(d) enable the loop gain to take on a value that
+10 V
produces square wave oscillations.
[EC-2016 : 1 Mark] (a) –8 V –5 V
Vi

SECTIO N -B

Q.1 Consider the following two statements: –10 V


Statement-1 : Astable multivibrator can be used
Vo
for generating square wave.
Statement-1 : Bistable multivibrator can be used +10 V
for storing binary information.
–5 V +8 V
(a) Only statement 1 is correct. (b) Vi

(b) Only statement 2 is correct.


(c) Both the statements 1 and 2 are correct.
–10 V
(d) Both the statements 1 and 2 are incorrect.
[EC-2001 : 1 Mark] Vo
+5 V
Q.2 An ideal sawtooth voltage waveform of
frequency 500 Hz and amplitude 3 V is
–5 V +5 V
generated by charging a capacitor of 2 µF in (c) Vi

every cycle. The charging requires


(a) constant voltage source of 3 V for 1 ms.
(b) constant voltage source of 3 V for 2 ms. –10 V

(c) constant current source of 3 mA for 1 ms. Vo


+10 V
(d) constant current source of 3 mA for 2 ms.
[EC-2003 : 2 Marks]
(d) –5 V +5 V
Q.3 Given the ideal operational amplifier circuit Vi
shown in the figure indicate the correct transfer
characteristics assuming ideal diodes with zero
cut-in voltage. –5 V

[EC-2005 : 2 Marks]
154 Electronics Engineering Analog Electronics

Q.4 Consider the Schmidt trigger circuit shown Q.6 In the following astable multivibrating circuit,
below: which properties of vo(t) depend on R2?

+15 V R1
10 k

– –
Vi vo(t)
Vo +
+ R3
C

10 k R2 R4
10 k
–15 V

A triangular wave which goes from –12 V to (a) Only the frequency
12 V is applied to the inverting input of the
(b) Only the amplitude
op-amp. Assume that the output of the op-amp
(c) Both the amplitude and the frequency
swings from +15 V to –15 V. The voltage at the
(d) Neither the amplitude nor the frequency
non-inverting input switches between
[EC-2009 : 2 Marks]
(a) –12 V and +12 V (b) –7.5 and +7.5 V
(c) –5 V and +5 V (d) 0 and 5 V Q.7 In the astable multivibrator circuit shown in the
[EC-2008 : 2 Marks] figure, the frequency of oscillation (in kHz) at
the output pin 3 is _______ .
Q.5 An stable multivibrator circuit using IC 555
timer is shown below. Assume that the circuit
is oscillating steadily. VCC

9k 8 4
RA = 2.2 k VCC Res
3k 7
4 8 Disch
(Reset) (Supply)
6 (Threshold) RB = 4.7 k 555 Timer
6 3
10 k (Output) 3 Thresh Out

2 (Trigger) (Gnd)
(Discharge) 1 2
7 Trig

12 k C = 0.022 µF Gnd
1
VC 0.01 µF

[EC-2016 : 1 Mark]
The voltage VC across the capacitor varies
between
(a) 3 V to 5 V (b) 3 V to 6 V
(c) 3.6 V to 6 V (d) 3.6 V to 5 V
[EC-2008 : 2 Marks]
GATE Previous Years Solved Paper 155

ELECTRICAL EN GINEERIN G List-I

(GATE Previous Years Solved Papers) A. Capacitor C1 is open


B. Capacitor C3 is open
SECTIO N - A C. Capacitor C3 is open
Q.1 In a common emitter amplifier, the unbypassed D. RC2 is shorted
emitter resistance provides List-II
(a) voltage-shunt feedback P. All dc voltages normal, V o increase
(b) current-series feedback marginally.
(c) negative-voltage feedback Q. Collector of TR2 at VCC , Vo = 0
(d) positive-current feedback R. All dc voltages normal, gain of 2nd stage
[EE-1992 : 1 Mark] decrease, Vo decrease
S. All dc voltage normal, Vo = 0
Q.2 A Wein bridge oscillator is shown in figure.
T. All dc voltage normal, overall gain of the
Which of the following statements are true, if ‘f ’
amplifier increases, Vo increase
is the frequency of oscillation?
U. No change
R2 [EE-1994 : 1 Mark]
R
R1 Q.4 A practical RC sinusoidal oscillator is built
C –
using a positive feedback amplifier with a
+ Vo
closed-loop gain slightly less than unity.
(True/False)
R C
[EE-1994 : 1 Mark]

Q.5 An op-amp has an open-loop gain of 105 and


1 an open-loop upper cut-off frequency of 10 Hz.
(a) For R = 1 k , C = µF, f = 1 kHz
2 If this op-amp is connected as an amplifier with
1 a closed-loop gain of 100, then the new upper
(b) For R = 3 k , C = µF, f = 3 kHz
18 cut-off frequency is
(c) The gain of the op-amp stage should be less
(a) 10 Hz (b) 100 Hz
than two for proper operation.
(c) 10 kHz (d) 100 kHz
(d) The gain of op-amp should be three for
proper operation. [EE-2001 : 1 Mark]
[EE-1993 : 1 Mark] Q.6 For the oscillator circuit shown in figure, the
Q.3 Given figure shows a two stage small signal expression for the time period of oscillation can
transistor feedback amplifier. Match the be given by (where = RC).
defective component (listed on the left hand side R
below) with its probable effect on the circuit
(listed on the right hand side). C
VC –
Rc1 Rc2 Vo
R1 R1
+
C5 R
Rs C1 C2
TR1 TR2 Vo

Vs R2 R2 R
Re1 Re2 C3 C4

RF
156 Electronics Engineering Analog Electronics

(a) ln 3 (b) 2 ln 3 Q.9 The typical frequency response of a two-stage


(c) ln 2 (d) 2 ln 2 direct coupled voltage amplifier is as shown in
[EE-2001 : 2 Marks]

Q.7 The output voltage (Vo) of the schmitt trigger |Gain|


(a)
shown in figure swings between +15 V and
–15 V. Assume that the operational amplifier is Frequency
ideal. The output will change from +15 V to
–15 V when the instantaneous value of the
input since wave is, |Gain|
(b)
100 k
– Frequency
Vi = 10 sin t Vo
+
10 k
|Gain|
(c)
3k
Frequency
2V

(a) 5 V in the positive slope only.


|Gain|
(b) 5 V in the negative slope only. (d)
(c) 5 V in the positive and negative slopes.
Frequency
(d) 3 V in the positive and negative slopes.
[EE-2005 : 2 Marks]
[EE-2002 : 2 Marks]
Q.10 In the feedback network shown below, if the
Q.8 The feedback used in the circuit shown in figure
feedback factor ‘k’ is increased, then
can be classified as,
+ + +
VCC
Vin V1 A0 Vout
– – –
RC =
RF

C C

+ +
Vf = kVout k
RL
– –
RS RB
RE C
(a) the input impedance increases and output
impedance decreases.
(b) the input impedance increases and output
impedance also decreases.
(a) shunt-series feedback
(c) the input impedance decreases and output
(b) shunt-shunt feedback
impedance decreases.
(c) series-shunt feedback
(d) the input impedance decreases and output
(d) series-series feedback
impedance increases.
[EE-2004 : 1 Mark]
[EE-2013 : 1 Mark]
GATE Previous Years Solved Paper 157

Q.11 In the Wein bridge oscillator circuit shown in Q.13 A hysteresis type TTL inverter is used to realize
figure, the bridge is balanced when an oscillator in the circuit shown in the figure.
10 k
C1
+5 V
R1
+VCC vo
+
0.1 µF

–VCC R3

If the lower and upper trigger level voltages are


C1 R2 R4 0.9 V and 1.7 V, the period (in ms), for which
output is LOW, is _______ .
[EE-2014 : 2 Marks]

R3 R1 1 Q.14 A Current Controlled Current Source (CCCS)


(a) = , =
R4 R2 R1C1 R2C 2 has an input impedance of 10 and output
impedance of 100 k . When this CCCS is used
R2 C 2 1
(b) = , = in a negative feedback closed-loop with a loop
R1 C1 R1C1 R2C 2
gain of 9, the closed-loop output impedance is
R3 R1 C 2 1 (a) 10 (b) 100 k
(c) = + , =
R4 R2 C1 R1C1 R2C 2
(c) 1000 k (d) 100
R3 R1 C 2 1 [EE-2019 : 1 Mark]
(d) + = , =
R4 R2 C1 R1C1 R2C2
[EE-2014 : 1 Mark] SECTIO N -B

Q.12 An oscillator circuit using ideal op-amp and Q.1 The circuit of figure shows a 555 Timer IC
diodes is shown in the figure. connected as an astable multivibrator. The
value of the capacitor C is 10 nF. The values of
R
the resistors RA and RB for a frequency of 10 kHz
and a duty cycle of 0.75 for the output voltage
+5 V waveform are

Vo VCC
C +
–5 V RA
Th
3k
RB 555
Tr
R1 Timer
1k Vout
C IC
1k

The time duration for +ve part of the cycle is t1


(a) RA = 3.62 k , RB = 3.62 k
t1 t2
(b) RA = 3.62 k , RB = 7.25 k
and for –ve parts at t2. The value of e RC
(c) RA = 7.25 k , RB = 3.62 k
will be _______ . (d) RA = 7.25 k , RB = 7.25 k
[EE-2014 : 2 Marks] [EE-2003 : 2 Marks]
158 Electronics Engineering Analog Electronics

Q.2 In the Schmitt trigger circuit shown in figure, if Q.4 IC 555 in the figure is configured as an astable
VCE(sat) = 0.1 V, the output logic low level (VOL), multivibrator. It is enabled to oscillate at t = 0 by
is applying a high input to pin 4. The pin
description is 1 and 8-supply, 2-trigger, 4-resel,
+5 V
6-threshold, 7-discharge. The waveform
appearing across the capacitor starting from
t = 0, as observed on a storage CRO is
200
+
Vo

Vi = 0 10 K
8
7

10 K IC 555 3

2, 6
1k I = 1.25 mA

4 1

(a) 1.25 V (b) 1.35 V


(c) 2.50 V (d) 5.00 V
(a) (b)
[EE-2004 : 2 Marks]

Q.3 The input signal Vin shown in the figure is a


1 kHz square wave voltage that alternates
between +7 V and –7 V with a 50% duty cycle.
Both transistors have the same current gain, (c) (d)
which is large. The circuit delivers power to the
load resistor RL. What is the efficiency of this
circuit for the given cicuit? Choose the closest [EE-2007 : 2 Marks]
answer.
Q.5 The following circuit has a source voltage Vs as
+10 V
shown in the graph. The current through the
circuit is also shown.
a b

+
Vs R 10 k
Vin –

RL = 10 15
10
5
V s (volts)

–10 V 0
–5
(a) 46% (b) 55%
–10
(c) 63% (d) 92% –15
[EE-2007 : 2 Marks] 0 100 200 300 400
Time (ms)
GATE Previous Years Solved Paper 159

15 vo

1
Current (mA)

T1 T1 T1 T1
0.5 (b)
0 t
–0.5
–1 vo
–1.5
0 100 200 300 400
Time (ms) T2 T1 T2 T1 T2
(c)
t
The element connected between a and b could be
(a) a b
vo
(b)
(c) a T2 T1 T2 T1 T2 T2
b (d)
t

[EE-2014 : 2 Marks]
(d)
Q.7 The cross-section of a metal-oxide
[EE-2009 : 1 Mark] semiconductor structure is shown
schematically. Starting from an uncharged
Q.6 Two monoshot multivibrators, one positive edge condition, a bias of +3 V is applied to the gate
triggered (M 1) and another negative edge contact with respect to the body contact. The
triggered (M2), are connected as shown in figure. charge inside the silicon dioxide layer is then
measured to be +Q. The total charge contained
+5 V
within the dashed box shown, upon application
M1 M2 of bias, expressed as multiple of Q (absolute
10 k
Q1 Q2 vo value in Coulombs, rounded off to the nearest
integer), is ______ .
10 µF Q1 Q2
GATE
Silicon Dioxide

The monoshots M1 and M2 when triggered Si


produce pulses of width T1 and T2 respectively,
BODY
where T1 > T2. The steady-state output voltage
Vo of the circuit is DASHED BOX

vo [EE-2020 : 1 Mark]

T1 T2 T1 T2 T1
(a)
t
160 Electronics Engineering Analog Electronics

Electronics & Electrical Engineering


GATE Previous Years Solved Paper

A n swe rs & Expl a n a t i o n s

Answers
EC Oscillator Circuits (Section-A)

2. (a) 3. (d) 4. (b) 5. (a) 6. (a) 7. (a) 8. (d) 9. (a)

Solutions
EC Oscillator Circuits (Section-A)

2. (a) 1 1 1/ R 1/ R
= = = =
1 A 21 1 1 R +1k
+
= R k R×1 k
LC
1 R+1k = 21 × 1 k
= R = 20 k
10 × 10 3 × 0.01 × 10 6
= 2 × 104
= 105 rad/sec
R Vf 4. (b)
+ Figure shown is Colpitts oscillator,

Vo 1k 10 mH 0.01 µF
1
Vf f=
2 LC eq

C1C 2 2×2
Ceq = = = 1 pF
Apply nodal analysis, C1 + C 2 4
Vf Vo Vf Vf
+ + + V f Cs = 0 1
R 1k Ls =
2 10 × 10 6 × 10 12
1 1 1 Vo 1 × 10 9
Vf + + Vf j C = = 50.3 MHz
R 1k L R =
2 10
1
For oscillation C =0 5. (a)
L
R2
1 1 Vo
Vf + =
R 1k R R1

Vf 1/R 1 Vo
= = +
Vo 1 1 Vo
+
R 1k
Rf 100 k
A = 1+ = 1+ = 21
R1 5k
GATE Previous Years Solved Paper 161

KCL at node-1, V0 j C 1 2 jR
= + + R2 +1
Vo 0 V Vo V1 R 2 2
C C
+ o =0
R1 R2 For oscillation imaginary part is zero.
1 1 1 C 1 C
+ = i.e., j × 2 2+ × R2 = 0
R1 R2 R2 R C R
R1 + R2 1
=1 + CR = 0
R1 CR
2C2R2 – 1 = 0
R2
R1 1 +
R1 1 2 1
= =6 = 2 2
R1 R C

R2 1 1
C= =
=5 R 2 × 10 3 × 10 3
R1
R 2 = 5R 1 1
= µF
2
6. (a)
Frequency of oscillation for RC phase shift 8. (d)
Frequency of Wein bridge oscillation is
1
oscillator is .
2 6RC 1
o = , but in the question time constant is
RC
7. (a) doubled so, frequency becomes half,
2.1 k 1
o =
2RC
1k 1
– Z1 = 2 R + = 2 (R jR )
+
Vout j C
C
1
1k R×
2j C R2 / j
Z2 = =
V1 1 R jR
R+
2j C
1k C
Z2 1
= =
Z1 + Z2 5
R = 1k
R1
V0 V1 V V 1+ =5 R1 = 4R2
= 1 + 1 R2
XC + R XC R
V0 V1 9. (a)
XC + R
= V1 The given circuit is Wein-bridge oscillator
XC + R XC R
which produced sinusoidal oscillations and the
V0 ( XC + R)2 XC =
j
amplitude of output wave is decided by
= +1
V1 XC R C
feedback through inverting input terminal of
1 2 jR op-amp.
+ R2
V0 2 2
C C
= +1
V1 R
j C
162 Electronics Engineering Analog Electronics

Answers
EC Oscillator Circuits (Section-B)

1. (c) 2. (d) 3. (b) 4. (c) 5. (a) 6. (a) 7. (5.681)

Solutions
EC Oscillator Circuits (Section-B)

2. (d) Vo

1 1 10 V
T= = = 2 ms
f 500 Hz
CdV 6 3
I= = 2 × 10 × 3
Vi
dt 2 × 10 –5 V

= 3 × 10–3
I = 3 mA –10 V

3. (b)
4. (c)
The given circuit is of a Schmitt trigger. The
upper and lower transition voltages can be Let the voltage at the non-inverting input be V1.
obtained as, Applying KCL at non-inverting input end,
Vut = uVsat 15 V1 V0 V1 V1 ( 15)
+ =
When lower diode is ON, 10 10 10
15 – V1 + V0 – V1 = V1 + 15
2k
u = Vo
2.5 k V1 =
3
2 Since, Vo swings from –15 V to +15 V.
Vut = × 10 = 8 V
2.5 Therefore, V1 switches between –5 V and +5 V.
Vlt = – lVsat
5. (a)
when upper diode is ON,
VC varies between 3 V to 5 V.
2k 1
l = = 7. Sol.
4k 2
1
1 f =
= × 10 = 5 V 0.69( RA + 2 RB ) C
2
1
Vut and Vlt are upper and lower transition =
0.69 (2.2 × 10 3 + 2 × 4.7 × 10 3 ) × 0.022 × 10 6
voltage. The transfer characteristics of the circuit
= 5.6818 kHz
is given as below.
GATE Previous Years Solved Paper 163

Answers
EE Oscillator Circuits (Section-A)

1. (b) 2. (a, b, d) 4. (False) 5. (c) 6. (b) 7. (a) 8. (b) 9. (b)

10. (a) 11. (c) 12. (0.8) 13. (0.63) 14. (c)

Solutions
EE Oscillator Circuits (Section-A)

3. Sol. 10. (a)


A S, B R, C T, D Q The given configuration is a voltage-series
feedback configuration.
4. Sol.
So, the input impedance increases,
False R if = Ri (1 + Aok)
5. (c) and the output impedance decreases,

AOL = 10 5 Ro
Rof =
f = 10 Hz 1 + Ao k
AOL
ACL = 11. (c)
1 + AOL
R3 R C
10 5 = 1+ 2
(1 + AOL) = = 10 3 R4 R2 C1
10 2
1
f 2 = f2(1 + AOL) =
R1 R2C1C 2
= 10 × 103 Hz
= 10 kHz 12. Sol.

6. (b) R

T = 2RC ln 3 = 2 ln 3

8. (b) Vo
C +
Equivalent circuit can be drawn with input
voltage comparison and current feedback. It is 3k D1

shunt-shunt feedback.

9. (b)
1k D2
1k
|Gain|

This circuit is astable multivibrator (or) free


For RC coupled amplifier
running oscillator.
When, V0 = Vsat
|Gain| Vsat
1k
VUTP = Vsat × =
1k +3k 4
For direct coupled amplifier
164 Electronics Engineering Analog Electronics

When, V0 = –Vsat From equation (i) and (ii),

Vsat ×
1k Vsat e t1 / RC
VLTP =
1k +1k
= = e( t1 t2 )/ RC
2 e t2 / RC
Vc = Vfinal + (Vinitial – Vfinal) e–t/RC 2 4
= = = 0.8
+V sat 5 /2 5
Charging
VUTP 13. Sol.
Discharging

V0
+5
1.7
VLTP
t1 t2 –V sat

–0.9 V
In time t = t1,
t
Vc = VUTP, Vinitial = VLTP, Vfinal = +Vsat 0V Vc(t)

VUTP = Vsat + (VLTP Vsat ) e t1 / RC


Discharging curve,
Vc(t) = 0 – (0 – 1.7) e–t/RC
Vsat Vsat t1 / RC
= Vsat + Vsat e At t = T2,
4 2
Vc(t) = 0.9 V
1 1 t1 / RC 0.9 = 1.7 e–t/RC
Vsat 1+ = Vsat +1 e
4 2 0.63 = T2
1 1 T2 = 0.63 ms
1 = + 1 e t1 / RC
4 2
14. (c)
3 3 t / RC
= e 1 “CCCS” [Current Controlled Current Source]
4 2
amplifier.
t1 / RC =2 ...(i)
e Given, Z 0 = 100 k
In time, t = t2 Loop gain, A =9
Vc = VLTP, Vinitial = VUTP, Vfinal = –Vsat Z0F = Z0[1 + A ]
t2 / RC [High impedance CS]
VLTP = Vsat + (VUTP + Vsat ) e
= 100 k [1 + 9]
Vsat V t2 / RC = 100 k × 10
= Vsat + sat + Vsat e
2 4 = 1000 k
1 1
1 Vsat = Vsat 1 + e t2 / RC
2 4
1 5 t2 / RC
= e
2 4
5
t2 / RC = ...(ii)
e 2
GATE Previous Years Solved Paper 165

Answers
EE Oscillator Circuits (Section-B)

1. (c) 2. (b) 3. (d) 4. (a) 5. (a) 6. (c) 7. (0)

Solutions
EE Oscillator Circuits (Section-B)

1. (c) But in this case initial voltage at capacitor is


zero so it starts from zero also charging time
T
Duty cycle = = ON will be larger (normally) than discharging time
T
but it is made equal by using a diode.
TON = T
= 0.75 × 10–4 = 75 µ-sec 5. (a)
Diode acts as a switch. When forward biased it
75 × 10 6
= RA + RB is short-circuited. But when suddenly reverse
0.7 × 10 8
biased, current does not becomes zero instantly,
RA + RB = 10.714 k
initially the same current flow in opposite
TOFF = 0.7 CRB
direction and after some time (turn off time) it
25 × 10 6 will become zero.
RB =
0.7 × 10 8
7. Sol.
R B = 3.57 k , RA = 7.14 k

2. (b)
Vi = 0, then first transistor will be cut-off and GATE + q
current through left resistor will drive the second
SiO2 + q
transistor into saturation. 3V
Then, Vo = VCE,sat + 1.25 + 10–3 × 103 Si – q

= 1.35 V Body – q

3. (d)
It is class-D amplifier, so should be high of
Overall charge in side the box q + q – q – q = 0
class-D amplifier is 90% to 100%.
charge.
4. (a)
An astable multi-vibrator is providing pulses
as given below.
GATE-2023
Electronics Engineering

Electromagnetic Theory
Chapterwise & Topicwise

Contents
S.No. Topic Page No.

1. Basics of Electromagnetics .............................................................................................................. 1-40

2. Uniform Plane Waves ..................................................................................................................... 41-60

3. Transmission Lines ........................................................................................................................... 61-80

4. Waveguides ....................................................................................................................................... 81-90

5. Antennas .......................................................................................................................................... 91-105


1 Basics of Electromagnetics

ELECTRO NICS EN GINEERIN G (a) remains unchanged in its magnitude and


direction.
(GATE Previous Years Solved Papers)
(b) remains unchanged in its magnitude but
Q.1 An electrostatic field is said to be conservative reverse in direction.
when:
(c) would be that due to dipole formed by the
(a) The divergence of the field is equal to zero. charge, +q, at O and –q induced.
(b) The curl of the field is equal to zero. (d) would be zero.
E [EC-1989 : 2 Marks]
(c) The curl of the field is equal to .
t2
Q.5 Which of the following field equations indicate
(d) The Laplacian of the field is equal to
that the free magnetic charge do not exist?
2
E
µ 2
. 1 Idl × R
t (a) H = ×A (b) H =
[EC-1987 : 2 Marks]
µ 4 R2

Q.2 On either side of a charge free interface between (c) H =0 (d) ×H = J


two media [EC-1990 : 2 Marks]
(a) the normal components of the electric field
are equal. Q.6 Given, V = x cos 2 y iˆ + x 2 e z ˆj + z sin 2 y kˆ and S
(b) the tangential components of the electric
the surface of unit cube with one corner at the
field are equal.
origin and edges parallel to the coordinate axis,
(c) the normal components of the electric flux
density are equal. the value of the integral V nˆ dS is ______ .
C
(d) the tangential components of the electric
flux density are equal. [EC-1993 : 2 Marks]
[EC-1988 : 2 Marks] Q.7 For a uniformly charged sphere of radius R and
Q.3 Vector potential is a vector charge density , the ratio of magnitude of
(a) whose curl is equal to the magnetic flux electric fields at distances R/2 and 2R from the
density. E(r = R /2)
(b) whose curl is equal to the electric field center, i.e., is ________ .
E(r = 2 R )
intensity.
(c) whose divergence is equal to the electric [EC-1993 : 2 Marks]
potential.
Q.8 Match List-I with List-II and select the correct
(d) which is equal to the vector product E × H.
answer using the code given below the lists:
[EC-1988 : 2 Marks]
List-I List-II
Q.4 The electric field strength at a far-off point P
due to a point charge, +q located at the origin, O A. ×H = J 1. Continuity equation
is 100 millivolts/meter. The point charge is now
B
enclosed by a perfectly conducting hollow metal B. E dI = ds 2. Faraday’s law
t
sphere with its center at the origin, O. The electric c s

field strength at the point P


2 Electronics Engineering Electromagnetic Theory

(a) a monopole
C. J= 3. Ampere’s law
t (b) a dipole
4. Gauss’s law (c) both a monopole and a dipole
5. Biot-Savart law (d) a quadru pole
Codes: [EC-1999 : 1 Mark]
A B C
Q.13 The electric field on the surface of a perfect
(a) 3 2 1 conductor is 2 V/m. The conductor is immersed
(b) 2 1 3 in water with = 80 0. The surface charge
(c) 4 3 1 density on the conductor is
(d) 1 2 3 (a) 0 C/m2
[EC-1994 : 2 Marks] (b) 2 C/m2
Q.9 The electric field strength at distance point P (c) 1.8 × 10–11 C/m2
due to a point charge, +q, located at the origin, (d) 1.41 × 10–9 C/m2
is 100 µV/m. If the point charge is now enclosed [EC-2002 : 2 Marks]
by a perfectly conducting metal sheet sphere
whose center is at the origin, then the electric Q.14 The unit of × H is
field strength at the point P, outside the sphere, (a) Ampere (b) Ampere/meter
becomes
(c) Ampere/meter2 (d) Ampere-meter
(a) zero (b) 100 µV/m
[EC-2003 : 1 Mark]
(c) –100 µV/m (d) 50 µV/m
[EC-1995 : 1 Mark] Q.15 If the electric field intensity is given by

Q.10 In the infinite plane, y = 6 m, three exists a E = ( x uˆ x + y uˆ x + z uˆ z ) Volt/m


uniform surface charge density of The potential difference between X(2, 0, 0) and
(1/6000 ) µC/m2. The associated electric field Y(1, 2, 3) is
strength is
(a) +1 Volt (b) –1 Volt
(a) 30 iˆ V/m (b) 30 ˆj V/m (c) +5 Volt (d) +6 Volt

(c) 30 kˆ V/m (d) 60 ˆj V/m [EC-2003 : 2 Marks]

[EC-1995 : 1 Mark] Q.16 Medium 1 has the electrical permittivity


1 = 1.5 0 Farad/m and occupies the region to
Q.11 A metal sphere with 1 m radius and a surface
the left of x = 0 plane. Medium 2 has the electrical
charge density of 10 C/m2 is enclosed in a cube
permittivity 2 = 2.5 0 Farad/m and occupies
of 10 m side. The total outward electric
the region to the right of x = 0 plane. If E1 in
displacement flux normal to the surface of the
medium 1 is E1 (2ux – 3uy + 1 uz) Volt/m, then E2
cube is
in medium 2 is
(a) 40 Coulombs (b) 10 Coulombs
(a) (2.0 ux – 7.5 uy + 2.5 uz) Volt/m
(c) 5 Coulombs (d) None of the above
(b) (2.0 ux – 2.0 uy + 0.6 uz) Volt/m
[EC-1996 : 1 Mark]
(c) (1.2 ux – 3.0 uy + 1.0 uz) Volt/m
Q.12 An electric field on a plane is described by its (d) (1.2 ux – 2.0 uy + 0.6 uz) Volt/m
potential V = 20 (r–1 + r–2) where ‘r’ is the [EC-2003 : 2 Marks]
distance from the source. The field is due to
GATE Previous Years Solved Paper 3

Q.17 A medium is divided into regions I and II about (a) E=0 (b) E=0
x = 0 plane, as shown in the figure below. An
×B = 0 B=0
electromagnetic wave with electric field
(c) ×E = 0 (d) ×E = 0
E1 = 4 aˆ x + 3aˆ y + 5aˆ z is incident normally on the
×B = 0 B=0
interface from region-I. The electric field E2 in [EC-2008 : 1 Mark]
region-II at the interface is Q.20 Two infinitely long wires carrying current are
Region-I Region-II as shown in the figure below. One wire is in the
= 0, µ1 = µ0, = 0, µ2 = µ0, y-z plane and parallel to the y-axis. The other
1 2

r1 = 3 r2 = 4
wire is in the x-y plane and parallel to the x-axis.
Which components of the resulting magnetic
field are non-zero at the origin?
E1 E2
z
1A
x<0 x=0 x>0

(a) E2 = E1

(b) 4 aˆ x + 0.75 aˆ y 1.25 aˆ z


y
(c) 3 aˆx + 3 aˆ y + 5 aˆ z
1A
(d) 3 aˆ x + 3 aˆ y + 5 aˆ z
[EC-2006 : 2 Marks] x

Q.18 If C is a closed curve enclosing a surface S, then (a) x, y, z components


the magnetic field intensity H , the current (b) x, y components
(c) y, z components
density J and the electric flux density D are (d) x, z components [EC-2009 : 1 Mark]
related by
Q.21 If a vector field V is related to another vector
D
(a) H ds = J+ dl
t field A through V = × A, which of the
s c
following is true?
D
(b) H dl = J+ ds Note: C and SC refer to any closed contour and
t
c s any surface whose boundary is C.
D (a) V dl = A ds
(c) H ds = J+ dl c
t SC
s c

D (b) A dl = V ds
(d) H dl = J+ ds c
SC
t
c s
[EC-2007 : 1 Mark] (c) × V dl = × A ds
C
SC
Q.19 For static electric and magnetic fields in an
(d) × A dl = V ds
inhomogeneous source free medium, which of C
SC
the following represents the correct from of two
of Maxwell’s equations? [EC-2009 : 2 Marks]
4 Electronics Engineering Electromagnetic Theory

Q.22 A magnetic field in air is measured to be Statement for Linked Answer Questions (24 and 25):
An infinitely long uniform solid wire of radius a carries
x y
B = B0 yˆ xˆ
x2 + y2 x2 + y2 a uniform dc current of density j .

What current distribution leads to this field? Q.24 The magnetic field at a distance ‘r’ from the
[Hint: the algebra is trivial in cylindrical center of the wire is proportional to
coordinates]. (a) r for r < a and 1/r2 for r > a
B0 zˆ 1 (b) 0 for r < a and 1/r for r > a
(a) J = , r 0
µ0 x 2 + y 2 (c) r for r < a and 1/r for r > a
(d) 0 for r < a and 1/r2 for r > a
B0 zˆ 2 [EC-2012 : 2 Marks]
(b) J = , r 0
µ0 x 2 + y 2
Q.25 A hole of radius b(b < a) is now drilled along the
(c) J = 0, r 0 length of the wire at a distance ‘d’ from the center
of the wire as shown below.
B zˆ 1
(d) J = 0 , r 0
µ0 x 2 + y 2
b
d
[EC-2009 : 2 Marks]

Q.23 A current sheet J = 10 uˆ y A/m lies on the a

dielectric interface x = 0 between two dielectric


The magnetic field inside the hole is
media with r1 = 1, µr1 = 1 in Region-1 (x < 0)
(a) uniform and depends only on d
and r2 = 2, µr2 = 2 in Region-2 (x > 0). If the (b) uniform and depends only on b
(c) uniform and depends on both b and d
magnetic field in Region-1 at x = 0 – is
(d) non uniform
H1 = 3 uˆ x + 30 uˆ y A/m, the magnetic field in [EC-2012 : 2 Marks]
Region-2 at x = 0+ is Q.26 The force on a point charge +q kept at a distance
x ‘d’ from the surface of an infinite grounded metal
x > 0 (Region-2) : r1 = 2, µr2 = 1
plate in a medium of permittivity is
J
x=0 y (a) 0

x < 0 (Region-1) : r1 = 1, µr1 = 1 q2


(b) away from the plate
16 d2
(a) H 2 = 1.5 uˆ x + 30 uˆ y 10 uˆ z A/m
q2
(b) H 2 = 3 uˆ x + 30 uˆ y 10 uˆ z A/m (c) towards the plate
16 d2
(c) H 2 = 1.5 uˆ x + 40 uˆ y A/m
q2
(d) towards the plate
(d) H 2 = 3 uˆ x + 30 uˆ y + 10 uˆ z A/m 4 d2

[EC-2011 : 2 Marks] [EC-2014 : 1 Mark]


GATE Previous Years Solved Paper 5

Q.27 A region shown below contains a perfect Q.30 Consider a straight, infinitely long, current
conducting half-space and air. The surface carrying conductor lying on the z-axis. Which
one of the following plots (in linear scale)
current K s on the surface of the perfect
qualitatively represents the dependence of H
conductor is Ks = xˆ 2 amperes per meter. The on r, where H is the magnitude of the azimuthal
component of magnetic field outside the
tangential H field in the air just above the conductor and r is the radial distance from the
perfect conductor is conductor?
y H

Ks Air (a)
Perfect conductor

r
H

(b)
(a) ( xˆ + zˆ) amperes per meter
(b) xˆ 2 amperes per meter
r
(c) zˆ 2 amperes per meter
H
(d) zˆ 2 amperes per meter
[EC-2014 : 2 Marks] (c)
Q.28 The electric field (assumed to be one-
dimensional) between two points A and B is
r
shown. Let A and B be the electrostatic
potential at A and B, respectively. The value of H

A – B (in Volts) is ______ .


(d)

r
40 kV/cm

[EC-2015 : 1 Mark]

20 kV/cm Q.31 In a source free region in vacuum, if the


electrostatic potential = 2x2 + y2 + cz2, the value
0 kV/cm of constant c must be _____ .
A 5 µm B
[EC-2015 : 1 Mark]
[EC-2014 : 2 Marks]
Q.32 A vector field D = 2 2 aˆ + z aˆ z exists inside a
Q.29 If E = (2 y 3 3 yz 2 ) xˆ (6 xy 2 3xz 2 ) yˆ + (6 xyz) zˆ
cylindrical region enclosed by the surfaces = 1,
is the electric field in a source free region, a valid z = 0 and z = 5. Let S be the surface bounding
expression for the electrostatic potential is this cylindrical region. The surface integral of
(a) xy3 – yz2 (b) 2xy3 – xyz2
this field on s D ds is ______ .
(c) y3 + xyz2 (d) 2xy3 – 3xyz2 s

[EC-2014 : 2 Marks] [EC-2015 : 2 Marks]


6 Electronics Engineering Electromagnetic Theory

Q.33 Concentric spherical shells of radii 2 m, 4 m


and 8 m carry uniform surface charge densities
of 20 nC/m2, –4 nC/m2 and s, respectively.
The value of s (nC/m2) required to ensure that

at x = +d
+q

at x = –d
the electric flux density D = 0 at radius 10 m is x=0
________ .
[EC-2016 : 1 Mark]

Q.34 The current density in a medium is given by

400 sin The charge is at rest at t = 0, when a voltage +V


J= aˆ r Am 2
2
2 ( r + 4) is applied to the plate at –d and voltage –V is
applied to the plate at x = +d. Assume that the
The total current and the average current quantity of the charge q is small enough that it
density flowing through the portion of a
does not perturb the field set up by the metal
plates. The time that the charge q takes to reach
spherical surface, r = 0.8 m, ,
12 4 the right plate is proportional to
0 2 are given, respectively by d d
(a) 15.09 A, 12.86 Am–2 (a) (b)
V V
(b) 18.73 A, 13.65 Am–2
d d
(c) 12.86 A, 9.23 Am–2 (c) (d)
V V
(d) 10.28 A, 7.56 Am–2
[EC-2016 : 2 Marks]
[EC-2016 : 2 Marks]
Q.37 The parallel-plate capacitor shown in the figure
Q.35 A uniform and constant magnetic field B = zB
ˆ has movable plates. The capacitor is charged so
exists in the ẑ direction in vacuum. A particle that the energy stored in it is E when the plate
of mass m with a small charge q is introduced separation is d. The capacitor is then isolated
into this region with an initial velocity electrically and the plates are moved such that
the plate separation becomes 2d.
v = xˆ vx + zˆ vz . Given that: B, m, q, vx and vz are
all non-zero, which one of the following d

describes the eventual trajectory of the particle?


(a) Helical motion in the ẑ direction.
(b) Circular motion in the xy-plane.
(c) Linear motion in the ẑ direction.

(d) Linear motion in the x̂ direciton.


[EC-2016 : 1 Mark]
At this new plate separation, what is the energy
Q.36 A positive charge q is placed at x = 0 between stored in the capacitor, neglecting fringing
two infinite metal plates placed at x = –d and at effect?
x = +d respectively. The metal plates lie in the
yz-plane. (a) 2 E (b) 2E
(c) E (d) E/2
[EC-2016 : 2 Marks]
GATE Previous Years Solved Paper 7

Q.38 Consider the charge profile shown in the figure. Q.39 Two conducting spheres S1 and S2 of radii a
The resultant potential distribution is best and b (b > a) respectively, are placed far apart
described by and connected by a long, thin conducting wire,
(x ) as shown in the figure.
S2
1
S1
b 0 Wire
a x
Radius a
2 Radius b

For some charge placed on this structure, the


V(x) potential and surface electric field on S1 are Va
and Ea , and that on S2 are Vb and Eb respectively.
Then, which of the following is correct?
b 0 (a) Va = Vb and Ea < Eb
(a) x
a (b) Va > Vb and Ea > Eb
(c) Va = Vb and Ea > Eb
(d) Va > Vb and Ea = Eb
[EC-2017 : 1 Mark]
V(x)
Q.40 An electron (q1) is moving in free space with
velocity 105 m/s towards a stationary electron
b 0
(q2) far away. The closest distance that this
(b) a
x
moving electron gets to the stationary electron
before the repulsive force diverts its path is
______ × 10–8 m.
[Given, mass of electrons m = 9.11 × 10–31 kg,
V(x) charge of electron e = –1.6 × 10 –19 C, and
permittivity 0 = (1/36 ) × 10–9 F/m]
[EC-2017 : 2 Marks]

(c) b 0 x Q.41 In the table shown, List-I and List-II respectively,


a
contain terms appearing on the left-hand side
and the right-hand side of Maxwell’s equations
(in their standard form). Match the left-hand
side withe the corresponding right-hand side.
V(x) List-I List-II

1. D P. 0

b 0 2. ×E Q.
(d) x
a
B
3. B R.
t

D
4. ×H S. J +
[EC-2016 : 2 Marks] t
8 Electronics Engineering Electromagnetic Theory

Codes:
Q.44 The vector function, F(r ) = xiˆ + yjˆ is defined
(a) 1 - R, 2 - Q, 3 - S, 4 - P
over a circular arc C shown in the figure.
(b) 1 - Q, 2 - S, 3 - P, 4 - R
(c) 1 - P, 2 - R, 3 - Q, 4 - S
j
(d) 1 - Q, 2 - R, 3 - P, 4 - S
C
[EC-2019 : 1 Mark]
45°
Q.42 What is the electric flux ( E d aˆ ) through a i 1

quarter-cylinder of height H (as shown in the The line integral of F(r ) dr is


C
figure) due to an infinitely long line charge along
the axis of the cylinder with a charge 1 1
(a) (b)
density of Q? 2 3
1 1
0 (c) (d)
4 6
[EC-2021 : 1 Mark]

Q Q.45 For a vector field, D = cos2 a + z 2 sin 2 a


H

in a cylindrical coordinate system ( , , z) with

unit vectors a , a and az, the next flux of D

4H H 0 leaving the closed surface of the cylinder


(a) (b) ( = 3, 0 z 2) (Rounded off to two decimal
Q 0 4Q
places) is ________ . [EC-2021 : 2 Marks]
HQ HQ
(c) (d)
0 4 0 Q.46 Consider the vector field:
[EC-2019 : 1 Mark] F = ax(4y – c1z) + ay(4x + 2z) + az(2y + z)
in a rectangular coordinate system (x, y, z) with
Q.43 Two identical copper wires W1 and W2 placed
unit vectors ax, ay and az. If the field ‘F’ is
in parallel as shown in the figure, carry currents
irrotational (conservative), then the constant c1
I and 2I, respectively, in opposite directions. If
(in integer) is ______ .
the two wires are separated by a distance of 4r,
[EC-2021 : 1 Mark]
then the magnitude of the magnetic field B
between the wires at a distance r from W1 is Q.47 Consider the two dimensional vector field
W1 F( x , y ) = x i + y j , where i and j denote the
r
unit vectors along the x-axis and the y-axis,
respectively. A counter ‘C’ in the x-y plane, as
W2 shown in the figure, is composed of two
horizontal lines connected at the two ends by
µ02 I 2 6µ 0 I two semicircular arcs of units radius. The
(a) 2
(b)
2 r 5 r contour is traversed in the counter-clockwise
µ0 I 5µ0 I sense. The value of the closed path integral,
(c) (d)
6 r 6 r
F( x , y ) ( dx i + dy j ) is ________ .
[EC-2019 : 2 Marks]
C
GATE Previous Years Solved Paper 9

y ELECTRICAL EN GINEERIN G
(GATE Previous Years Solved Papers)
(0, 2)
1 r=
r= 1
SECTIO N - A
c
Q.1 Given a vector field F , the divergence theorem
x states that,
(0, 0) (4, 0)
(a) F ds = F dV
(a) 0 (b) 1 s v

(c) 8 + 2 (d) –1
(b) F ds = × F dV
[EC-2022] s v

Q.48 The value of the integral,


(c) F×ds = F dV
2 2
3( x + y ) dx dy s v
D
(d) F×ds = × F dV
where D is the shaded triangular region shown
s v
in the diagram, is ______ (Rounded off to the
nearest integer). [EE-2002 : 1 Mark]
y
Q.2 If E is the electric field intensity, ( × E ) is
(4, 4)
(0, 4)
equal to

(a) E (b) E

x (c) Null vector (d) Zero


(0, 0)
[EE-2005 : 1 Mark]

Q.3 Divergence of the vector field,


(0, –4)
(4, –4)
V(x, y, z) = ( x cos xy + x ) iˆ + ( y cos xy ) ˆj

[EC-2022] + (sin z2 + x 2 + y 2 ) kˆ , is

Q.49 In an electrostatic field, the electric displacement (a) 2z cos z2 (b) sin xy + 2z cos z2
(c) x sinxy – cos z (d) none of these
density vector, D, is given by
[EE-2007 : 1 Mark]
3 3 2 2
D( x , y , z) = ( x i + y j + xy k ) C/m Q.4 The direction of vector A is radially outward
where, i , j , k are the unit vectors along x-axis, from the origin, with A = kr n where
y-axis and z-axis respectively. Consider a r2 = x2 + y2 + z2 and ‘k’ is a constant. The value
cubical region R centered at the origin with each of ‘n’ for which . A = 0 is,
side of length 1 m, and vertices at (± 0.5 m, (a) –2 (b) 2
± 0.5 m, ± 0.5 m). The electric charge enclosed
(c) 1 (d) 0
within R is _____ C (Rounded off to two
[EE-2012 : 2 Marks]
decimal places).
[EC-2022]
10 Electronics Engineering Electromagnetic Theory

Q.8 The line integral of the vector field,


1
Q.5 Consider a function f = rˆ , where ‘r’ is the
r2 F = 5xz iˆ + (3x 2 + 2 y ) i + x 2 zkˆ
distance from the origin and r̂ is the unit vector along a path from (0, 0, 0) to (1, 1, 1)
in the radial direction. The divergence of this parameterized by (t, t2, t) is _______ .
function over a sphere of radius R, which [EE-2016 : 2 Marks]
includes the origin, is
Q.9 The figures shows diagrammatic
(a) 0 (b) 2
(c) 4 (d) R representations of vector fields X , Y and Z,
[EE-2015 : 1 Mark] respectively. Which one of the following choices
Q.6 Match List-I with List-II the following: is true?

List-I X Y Z
A. Stoke’s theorem
B. Gauss’s theorem
C. Divergence theorem
D. Cauchy’s integral theorem
List-II
(a) X = 0, ×Y 0, ×Z = 0
1. D ds = Q
(b) X 0, × Y = 0, ×Z 0
2. f ( z ) dz = 0
(c) X 0, ×Y 0, ×Z 0

3. ( A) dV = A ds (d) X = 0, × Y = 0, ×Z = 0
[EE-2017 : 1 Mark]
4. ( × A ) ds = A dl
Q.10 Let ar, a and az be unit vectors along r, and z
Codes:
directions, respectively in the cylindrical
A B C D
coordinate system. For the electric flux density
(a) 2 1 4 3 given by
(b) 4 1 3 2 D = (ar 15 + a 2r – az 3rz) Coulomb/m2
(c) 4 3 1 2 The total electric flux (in Coulomb), emanating
(d) 3 4 2 1 from the volume enclosed by a solid cylinder of
[EE-2015 : 1 Mark] radius 3 m and height 5 m oriented along the
z-axis with its base at the origin is
Q.7 In cylindrical coordinate system, the potential
(a) 108 (b) 54
produced by a uniform ring charge is given by
(c) 90 (d) 180
f = f(r, z), where ‘f ’ is a continuous function of
[EE-2020 : 2 Marks]
r and z. Let E be the resulting electric field. Then
SECTIO N -B
the magnitude of ×E
Q.1 An electrostatic potential is given by
(a) increases with r (b) is 0
(c) is 3 (d) decreases with z = 2 x y V in the rectangular co-ordinate
[EE-2016 : 1 Mark] system. The magnitude of the electric field at
x = 1 m, y = 1 m is _______ V/m.
[EE-1992 : 1 Mark]
GATE Previous Years Solved Paper 11

Q.2 Which of the following equations represents the Q.7 The capacitance of an isolated sphere of radius
Gauss’ law in a homogenous isotropic medium? 10 cm in air is equal to ______ pF.
[EE-1997 : 2 Marks]
(a) D ds = dV
Q.8 Given the potential function in free space to be
(b) ×H = D V(x) = (50x2 + 50y2 + 50z2) Volts, the magnitude
(c) J+ =0 (in Volts/meter) and the direction of the electric
field at a point (1, –1, 1), where the dimensions
(d) E= are in meters, are

(a) 100 ; (iˆ + ˆj + kˆ )


[EE-1992 : 1 Mark]
100 ˆ ˆj + kˆ )
(b) ; (i
Q.3 In electrostatic field, × E = 0 (True/False). 3

[EE-1994 : 1 Mark] (iˆ + ˆj kˆ )


(c) 100 3 ;
3
Q.4 If V, W, q stand for voltage, energy and charge,
then V can be expressed as, ( iˆ + ˆj kˆ )
(d) 100 3 ;
dq dW 3
(a) V = (b) V =
dW dq [EE-2001 : 2 Marks]
dW dq
(c) dV = (d) dV =
dq dW Q.9 The electric field E (in Volts/meter) at the point
[EE-1996 : 1 Mark] (1, 1, 0) due to a point charge of +1 µC located at
(–1, 1, 1) (co-ordinates in meters) is
Q.5 In a uniform electric field, field lines at 6
10 6 10
equipotential surfaces (a) (2 i k ) (b) (2i k )
20 5 0 20 0
(a) are parallel to one another
6 6
(b) intersect at 45° 10 10
(c) (2 i k ) (d) (2i k )
(c) intersect at 30° 20 5 0 20 0

(d) are orthogonal [EE-2001 : 2 Marks]


[EE-1997 : 1 Mark]
Q.10 A point charge of +1 nC is placed in a space
Q.6 The capacitance of the arrangement shown in with permittivity of 8.85 × 10–12 F/m as shown
figure is ______ pF. in figure. The potential difference VPG between
two points P and Q at distance of 40 mm and
Square conducting plate
20 cm 20 mm respectively from the point charge is

Q
10 cm r1 =1
m
m
20
20 cm

r3 =4 1 nC
=2 + P
10 cm r2 40 mm

(a) 0.22 kV (b) –225 V


20 cm
10 cm (c) –2.24 kV (d) 15 V
Conducting plate
[EE-2003 : 2 Marks]
[EE-1997 : 2 Marks]
12 Electronics Engineering Electromagnetic Theory

Q.11 A parallel plate capacitor has an electrode area 1 Qr 3 Qr


(a) (b)
of 100 mm2, with spacing of 0.1 mm between 4 0 R 3 4 0 R3
the electrodes. The dielectric between the plates
1 Q 1 QR
is air with a permittivity of 8.85 × 10–12 F/m. (c) (d)
4 0 r 2 4 0 r3
The voltage on the capacitor is 100 V. The stored
[EE-2007 : 2 Marks]
energy in the capacitor is
(a) 8.85 pJ (b) 440 pJ Q.15 Two point charges, Q1 = 10 µC and Q2 = 20 µC
(c) 22.1 nJ (d) 44.3 nJ are placed at co-ordinates (1, 1, 0) and (–1, –1, 0)
[EE-2003 : 2 Marks] respectively. The total electric flux passing
through a plane z = 20 will be
Q.12 A composite parallel plate capacitor is made
(a) 7.5 µC (b) 13.5 µC
up of two different dielectric materials with
(c) 15.0 µC (d) 22.5 µC
different thickness (t1 and t2) as shown in figure.
[EE-2008 : 2 Marks]
The two different dielectric materials are
separated by a conducting foil F. The voltage of Q.16 A capacitor consists of two metal plates each
the conducting foil is 500 × 500 mm2 and spaced 6 mm apart. The
space between the metal plates is filled with a
glass plate of 4 mm, thickness and a layer of
r1 = 3, t1 = 0.5 mm
100 V paper of 2 mm thickness. The relative
= 4, t2 = 1 mm
r2
primitivities of the glass and paper are 8 and 2
respectively. Neglecting the fringing effect, the
(a) 52 V (b) 60 V capacitance will be

(c) 57 V (d) 33 V (Given that, 0 = 8.85 × 10–12 F/m)

[EE-2003 : 2 Marks] (a) 983.3 pF (b) 1475 pF


(c) 637.5 pF (d) 9956.25 pF
Q.13 A parallel plate capacitor is shown in figure. It
[EE-2008 : 2 Marks]
is made of two square metal plates of 400 mm
side. The 1.4 mm space between the plates is Q.17 A capacitor is made with a polymetric dielectric
filled with two layers of dielectrics of r = 4, 6 mm having an r of 2.26 and a dielectric breakdown
thick and r = 2, 8 mm thick. Neglecting fringing strength of 50 kV/cm. The permittivity of free
of fields at the edges the capacitance is space is 8.85 pF/m.
If the rectangular plates of the capacitor have
= 4, d = 6 mm width of 20 cm and length of 40 cm, then the
r
maximum electric charge in the capacitor is
r = 2, d = 8 mm
(a) 2 µC (b) 4 µC
(c) 8 µC (d) 10 µC
(a) 1298 pF (b) 944 pF
[EE-2011 : 2 Marks]
(c) 354 pF (d) 257 pF
[EE-2004 : 1 Mark] Q.18 A dielectric slab with 500 mm × 500 mm cross-
section is 0.4 m long. The slab is subjected to a
Q.14 A solid sphere made of insulating material has uniform electric field of E = 6ax + 8ay kV/mm.
a radius R and has a total charge Q distributed The relative permittivity of the dielectric
uniformly in its volume. What is the magnitude material is equal to 2. The value of constant 0 is
of the electric field intensity, E, at a distance 8.85 × 10–12 F/m. The energy stored in the
r(0 < r < R) inside the sphere? dielectric (in Joules), is
GATE Previous Years Solved Paper 13

(a) 8.85 × 10–11 (b) 8.85 × 10–5 Q.22 A perfectly conducting metal plate is placed in
(c) 88.5 (d) 885 x-y plane in right handed coordinate system. A
[EE-2013 : 2 Marks] charge of +32 2 Coulomb is placed at
0
Q.19 C0 is capacitance of a parallel plate capacitor coordinate (0, 0, 2). 0 is the permittivity of free
with air as dielectric [as in Fig. (a)]. If, half of the
space. Assume iˆ , ˆj , kˆ to be unit vectors along x,
entire gap as shown in Fig. (b) is filled with a
dielectric of permittivity r, the expression for y and z axes respectively. At the coordinate
the modified capacitance is ( 2 , 2 , 0), the electric field vector E
(Newtons/Coulomb) will be

z
(a) (b) 32 0 2 Columbs
y Perfectly conducting
C0
(a) (1 + r ) (b) (C0 + r)
(0, 0, 2) metal place
2
C0 ( 2 , 2 , 0)
(c) r (d) C0(1 + r)
2
(0, 0, 0)
[EE-2014 : 1 Mark] x

Q.20 A parallel plate capacitor consisting two


dielectric material is shown in the figure. The
middle dielectric slab is placed symmetrically
with respect to the plates.
10 V (a) 2 2 kˆ (b) 2 kˆ

(c) 2 kˆ (d) 2 2 kˆ

1 1
[EE-2014 : 2 Marks]

Q.23 A parallel plate capacitor is partially filled with


2
glass of dielectric constant 4.0 as shown below.
d/2
d Air, = 1.0 5 mm
r
10 mm

If the potential difference between one of the


plates and the nearest surface of dielectric Glass, = 4.0
r
interface is 2 Volts, then the ratio 1 : 2 is
(a) 1 : 4 (b) 2 : 3 The dielectric strengths of air and glass are
(c) 3 : 2 (d) 4 : 1 30 kV/cm and 300 kV/cm, respectively. The
[EE-2014 : 1 Mark] maximum voltage (in kV), which can be applied
across the capacitor without any breakdown, is
Q.21 A hollow metallic sphere of radius ‘r’ is kept at
________ .
potential of 1 Volt. The total electric flux coming
[EE-2015 : 2 Marks]
out of the concentric spherical surface of radius
R(> r), is Q.24 Two semi-infinite dielectric regions are
(a) 4 0r (b) 4 0 r2 separated by a plane boundary at y = 0. The
(c) 4 0R (d) 4 0R
2 dielectric constants of regions 1(y < 0) and
[EE-2014 : 1 Mark] region 2(y > 0) are 2 and 5, respectively.
14 Electronics Engineering Electromagnetic Theory

Region 1 has uniform electric field

E = 3 aˆx + 4 aˆ y + 2 aˆ z , where aˆ x , aˆ y and aˆ z are A B


2 cm
unit vectors along the x, y and z axes, r=1 r=4

respectively. The electric field in region 2 is


(a) 3 aˆ x + 1.6 aˆ y + 2 aˆ z (a) 1 (b) 2
(b) 1.2 aˆx + 4 aˆ y + 2 aˆ z (c) 4 (d) 16
[EE-2016 : 1 Mark]
(c) 1.2 aˆ x + 4 aˆ y + 0.8 aˆ z
Q.28 Two electrodes, whose cross-sectional view is
(d) 3 aˆx + 10 aˆ y + 0.8 aˆ z shown in the figure below, are at the same
[EE-2015 : 1 Mark] potential. The maximum electric field will be at
the point
Q.25 Two semi-infinite conducting sheets are placed
at right angles to each other as shown in the
A D C
figure. A point charge of +Q is placed as a
distance of d from both sheets. The net force on B
(a) A (b) B
Q2 K
the charge is , where K is given by (c) C (d) D
4 0 d2
[EE-2016 : 1 Mark]
y
Q.29 Consider an electron, a neutron and a proton
d
+Q initially at rest and placed along a straight line
such that the neutron is exactly at the center of
d the line joining the electron and proton. At t = 0,
the particles are released but are constrained to
x move along the same straight line. Which of
these will collide first?
(a) The particles will never collide.
1ˆ 1 ˆ (b) All will collide together.
(a) 0 (b) i j
4 4
(c) Proton and neutron.
1ˆ 1ˆ 1 2 2 ˆ 1 2 2 ˆ (d) Electron and neutron. [EE-2017 : 1 Mark]
(c) i j (d) i+ j
8 8 8 2 8 2
Q.30 Consider a solid sphere of radius 5 cm made of
[EE-2015 : 2 Marks] a perfect electric conductor. If one million
electrons are added to this sphere, these
Q.26 Two electric charges Q and –2Q are placed at
electrons will be distributed
(0, 0) and (6, 0) on the x-y plane. The equation of
the zero equipotential curve in the x-y plane is (a) uniformly over the entire volume of the
sphere.
(a) x = –2 (b) y = 2
(b) uniformly over the outer surface of the
(c) x2 + y2 =2 (d) (x + 2)2 + y2 = 16
sphere.
[EE-2016 : 2 Marks]
(c) concentrated around the center of the
Q.27 A parallel plate capacitor filled with two sphere.
dielectrics is shown in the figure below. If the (d) along a straight line passing through the
electric field in the region ‘A’ is 4 kV/cm, the center of the sphere.
electric field in the region ‘B’, in kV/cm, is [EE-2017 : 1 Mark]
GATE Previous Years Solved Paper 15

Q.31 A thin soap bubble of radius R = 1 cm, and


thickness a = 3.3 µm (a << R), is at a potential of r2
R R
1 V with respect to a reference point at infinity.
r r
The bubble bursts and becomes a single
spherical drop of soap (assuming all the soap
r1 =2 r1 =2
is contained in the drop) of radius r. The volume
Fig. (i) Fig. (ii)
of the soap in the thin bubble is 4 R2a and that
[EE-2019 : 1 Mark]
4 3
of the drop is r . The potential in volts, of
3 Q.35 The static electric field inside a dielectric
medium with relative permittivity. r = 2.25,
the resulting single spherical drop with respect
expressed in cylindrical coordinate system is
to the same reference point at infinity is _____ .
given by the following expression,
(Give the answer up to two decimal places).
3
E = aˆr 2r + a + aˆ z 6
a T
R Bursts where aˆr , aˆ , aˆ z are unit vectors along r, and z
Soap drop directions, respectively. If the above expression
Soap bubble of radius ‘r ’
represents a valid electrostatic field inside the
[EE-2017 : 2 Marks] medium, then the volume charge density
associated with this field in terms of free space
Q.32 A positive charge of 1 nC is placed at (0, 0, 0.2) permittivity, 0, in SI units is given by
where all dimensions are in meters. Consider
(a) 4 0 (b) 5 0
the x-y plane to be a conducting ground plane.
(c) 3 0 (d) 9 0
Take 0 = 8.85 × 10–12 F/m. The Z component of
[EE-2020 : 2 Marks]
the E field at (0, 0, 0.1) is closed to
(a) 899.18 V/m (b) –899.18 V/m SECTIO N -C
(c) 999.09 V/m (d) –999.09 V/m
Q.1 The line integral of the vector potential A around
[EE-2018 : 1 Mark]
the boundary of a surface S represents
Q.33 The capacitance of an air-filled parallel plate (a) flux through the surface S
capacitor is 60 pF. When a dielectric slab whose (b) flux density in the surface S
thickness is half the distance between the plates, (c) magnetic density
is placed on one of the plates covering it entirely, (d) current density
the capacitance becomes 86 pF. Neglecting the [EE-1993 : 1 Mark1]
fringing effects, the relative permittivity of the
dielectric is _____ (upto 2 decimal places). Q.2 Static magnetic fields induce currents in closed
conducting loops. (True/False)
[EE-2018 : 2 Marks]
[EE-1994 : 1 Mark1]
Q.34 A co-axial cylindrical capacitor shown in Fig. (i)
has dielectric with relative permittivity, r1 = 2. Q.3 The energy stored in the magnetic field or
When one-fourth portion of the dielectric is solenoid 30 cm long and 3 cm diameter would
replaced with another dielectric of relative with 1000 turns of wire carrying of 10 A is
permittivity r2 as shown in Fig. (ii), the (a) 0.015 J (b) 0.15 J
capacitance is doubled. The value of r2 is (c) 0.5 J (d) 1.15 J
__________ . [EE-1996 : 1 Mark1]
16 Electronics Engineering Electromagnetic Theory

Q.4 Two parallel wires separated by a distance ‘d’


Z
are carrying a current ‘I’ in the same direction. +I
The magnetic field along a line running parallel
x
to these wires and midway between them X
–I
(a) depends upon ‘I’
w
(b) is zero
Y
(c) depends upon ‘d’
(a) 2 L H/m (b) L/4 H/m
(d) depends upon the permeability of medium
between the wires (c) L/2 H/m (d) 4 L H/m
[EE-1999 : 1 Mark1] [EE-2003 : 1 Mark1]

Q.8 The inductance of a long solenoid of length


Q.5 A electron with velocity, u is placed in an
1000 mm wound uniformly with 3000 turns on
electric field, E and magnetic field, B. The force a cylindrical paper tube of 60 mm diameter is
experienced by the electron is given by (a) 3.2 µH (b) 3.2 mH
(c) 32.0 mH (d) 3.2 H
(a) eE (b) eu×B
[EE-2004 : 1 Mark1]
(c) e ( u × E + B) (d) e ( E + u × B)
Q.9 Which of the following statements holds for the
[EE-2000 : 1 Mark1] divergence of electric and magnetic flux
densities?
Q.6 Two conductors are carrying forward and
(a) Both are zero.
return current of +I and –I as shown in figure.
(b) These are zero for static densities but non-
Z zero for time varying densities.
+I –I (c) It is zero for the electric flux density.
+ P – (d) It is zero for the magnetic flux density.
X d d
[EE-2006 : 1 Mark1]

Y
Statements for Linked Answer Questions (10 and 11):
An inductor designed with 400 turns coil wound on
The magnetic field intensity H at point P is an iron core 16 cm2 cross-sectional are and with a cut
I I of an air gap length of 1 mm. The coil is connected to a
(a) y (b) x
d d 230 V, 50 Hz ac supply. Neglect coil resistance, core
I I loss, iron reluctance and leakage inductance,
(c) y (d) x
2 d 2 d (µ0 = 4 × 10–7 H/m).

[EE-2003 : 1 Mark1] Q.10 The current in the inductor is

Q.7 Two infinite strips of width w m in X-direction (a) 18.08 A (b) 9.04 A
as shown in figure, are carrying forward and (c) 4.56 A (d) 2.28 A
return currents of +I and –I in the Z-direction. [EE-2007 : 2 Marks]
The strips are separated by a direction of x m. Q.11 The average force on the core to reduce the air
The inductance per unit length of the gap will be
configuration is measured to be L H/m. If the
(a) 832.29 N (b) 1666.22 N
distance of separation between the strips is now
(c) 3332.47 N (d) 6664.84 N
reduced to x/2 m, the inductance per unit length
of the configuration is [EE-2007 : 2 Marks]
GATE Previous Years Solved Paper 17

Q.12 A coil of 300 turns is wound on a non-magnetic 3


(a) k Weber /m 2
core having a mean circumference of 300 mm 2
and a cross-sectional area of 300 mm2. The 4
inductance of the coil corresponding to a (b) i A/m
3
magnetizing current of 3 A will be
3
(Given that, µ0 = 4 × 10–7 H/m) (c) k A/m
2
(a) 37.68 µH (b) 113.04 µH
(d) 0 A/m [EE-2014 : 2 Marks]
(c) 3.768 µH (d) 1.1304 µH
Q.15 The following four vector fields are given in
[EE-2008 : 2 Marks]
Cartesian coordinate system. The vector field
Q.13 The flux density at a point in space is given by: which does not satisfy the property of magnetic
B = 4xax + 2kyay + 8az Wb/m2 flux density is
The value of constant ‘k’ must be equal to
(a) y 2 aˆ x + z 2 aˆ y + x 2 aˆ z
(a) –2 (b) –0.5
(c) +0.5 (d) +2 (b) z2 aˆx + x 2 aˆ y + y 2 aˆ z
[EE-2013 : 2 Marks]
(c) x 2 aˆ x + y 2 aˆ y + z 2 aˆ z

Q.14 The magnitude of magnetic flux density ( B) at


(d) y 2 z2 aˆ x + z 2 z 2 aˆ y + x 2 y 2 aˆ z
a point having normal distance ‘d’ meters from
[EE-2014 : 2 Marks]
an infinitely extended wire carrying current of
‘I’ A is µoI/2 d (in SI units). Q.16 A steady current I is flowing in the –x-direction
An infinitely extended wire is laid along the through each of two infinitely long wires at
x-axis and is carrying current of 4 A in the +ve L
y=± as shown in the figure. The
x-direction. Another infinitely extended wire is 2
laid along the y-axis and is carrying 2 A current
in the +ve y-direction. µ0 is permeability of free permeability of the medium is µ0. The B field at
(0, L, 0) is
space. Assume iˆ , ˆj , kˆ to be unit vectors along
z
x, y and z axes respectively.

y
y = –L/2 y = L/2
y
2A (2, 1, 0) 0
1 Current = I Current = I
Iamps
µ0 I
B= x
2 d
d 4A 4 µ0 I 4 µ0 I
x (a) zˆ (b) + zˆ
1 2 3 L 3 L
z
3 µ0 I
(c) 0 (d) zˆ
4 L
Assuming right handed coordinate system, [EE-2015 : 1 Mark]
magnetic field intensity, H at coordinate
Q.17 A soft-iron toroid is concentric with a long
(2, 1, 0) will be straight conductor carrying a direct current I. If
the relative permeability µr of soft-iron is 100,
18 Electronics Engineering Electromagnetic Theory

the ratio of the magnetic flux densities at two SECTIO N - D


adjacent points located just inside and just
outside the toroid, is ______ . Q.1 The laws of electromagnetic induction
[EE-2016 : 1 Mark] (Faraday’s and Lenz’s law) are summarized in
the following equation
Q.18 A solid iron cylinder is placed in a region di
containing a uniform magnetic field such that (a) e = iR (b) e = L
dt
the cylinder axis is parallel to the magnetic field d
direction. The magnetic field lines inside the (c) e = (d) none of these
dt
cylinder will [EE-1998 : 1 Mark]
(a) bend closer to the cylinder axis.
Q.2 An electromagnetic field is radiated from
(b) bend farther away from the axis.
(a) a stationary point charge
(c) remain uniform as before.
(b) a capacitor with dc voltage
(d) cease to exist inside the cylinder.
(c) a conductor carrying a dc current
[EE-2017 : 1 Mark]
(d) an oscillating dipole
Q.19 The magnitude of magnetic flux density (B) in [EE-1999 : 1 Mark]
micro Teslas (µT), at the center of a loop of wire
Q.3 A circular turn of radius 1 m revolves at 60 rpm
wound as a regular hexagon of side length 1 m
about its diameter aligned with the x-axis as
carrying a current (I = 1 A) and placed in vacuum
shown in the figure. The value of µ0 is 4 × 10–7
as shown in the figure is ______ .
in SI unit. If a uniform magnetic field intensity
(Give the answer up to two decimal places).
H = 107 zˆ A/m is applied, them the peak value
if the induced voltage, Vturn (in Volts), is _____ .
Z
H

i Vturn
X

[EE-2017 : 2 Marks]

Q.20 A conducting square loop of side length 1 m is


[EE-2015 : 1 Mark]
placed at a distance of 1 m from a long straight
wire carrying a current l = 2 A as shown below. Q.4 Consider a one-turn rectangular loop of wire
The mutual inductance, in nH (rounded off to placed in a uniform magnetic field as shown in
two decimal places), between the conducting the figure. The plane of the loop is perpendicular
loop and the long wire is ______ . to the field lines. The resistance of the loop is
Z
0.4 , and its inductance is negligible. The
magnetic flux density (in Tesla) is a function of
I=2A
time, and is given by B(t) = 0.25 sin t, where
a=1m
= 2 × 50 radian/second. The power absorbed
d=1m (in Watt) by the loop from the magnetic field is
_______ .
a=1m

[EE-2020 : 2 Marks]
GATE Previous Years Solved Paper 19

10 cm Z

B
5 cm

1m
[EE-2015 : 1 Mark]

Q.5 A rotating conductor of 1 m length is placed in


a radially outward (about the z-axis) magnetic 1m

flux density (B) of 1 Tesla as shown in figure


below. Conductor is parallel to and at 1 m
distance from the z-axis. The speed of the [EE-2016 : 2 Marks]
conductor in rpm required to induce a voltage
of 1 V across it, should be ______ .

Electronics & Electrical Engineering


GATE Previous Years Solved Paper

A n swe rs & Expl a n a t i o n s

Answers
EC Basics of Electromagnetics

1. (b) 2. (b, c) 3. (a) 4. (a) 5. (c) 6. (1) 7. (2) 8. (a)

9. (b) 10. (b) 11. (a) 12. (c) 13. (d) 14. (c) 15. (c) 16. (c)

17. (c) 18. (d) 19. (d) 20. (d) 21. (b) 22. (c) 23. (a) 24. (c)

25. (a) 26. (c) 27. (d) 28. (15) 29. (d) 30. (c) 31. (–3) 32. (78.54)

33. (–0.25) 34. (*) 35. (a) 36. (c) 37. (a) 38. (d) 39. (c) 40. (5.06)

41. (d) 42. (d) 43. (d) 44. (a) 45. (56.54) 46. (0) 47. (a) 48. (512)

49. (0.5)
20 Electronics Engineering Electromagnetic Theory

Solutions
EC Basics of Electromagnetics

1. (b) 5. (c)
An electrostatic field is said to be conservative Magnetic field lines are closed around the
when the closed line integral of the field is zero, current and have no sources or sink points,

E dl = 0 B ds = Bdv

Stoke’s theorem, B=0


In any closed surface,
E dl = ( × E) ds
Entering flux = Leaving flux
c s
Dipole is the cause of magnetic fields.
So, ×E = 0 Magnetic monopoles or free charges do not exist
is justified by B = 0.
2. (b, c)
According to the Gauss’s law for magnetic
Boundary conditions, fields,
Et1 = Et2
B ds = 0
DN1 = DN 2 s

B =0
3. (a)
µH = 0
Vector potential magnetic fields is the measure
µ H =0
of work done to move a current element I dl ,
H =0
W
A =
I dl 6. Sol.

It relates to B (which is force experienced by a V ds = ( V ) dV

current element) as × A = B, V = cos2 y + sin2 y = 1

× A = B = µH ( V ) dV = 1

4. (a) 7. Sol.
The radially outward field is normal to vr
D= ; r<R
conductor and no induction effects E is same, 3
3
vR
D= ; r>R
2
+ +
+
3r
– – – Applying Gauss law with spherical Gaussian
+

surface concentric with the charge,


+

+q vR


+

E (at r = R/2) =
6


+

vR


+ + E(2R) =
+ 12
E(at r = R /2)
So, =2
E(at r = 2 R )
GATE Previous Years Solved Paper 21

8. (a) V
(A) – 3, (B) – (2), (C) – 1
×H = J Ampere’s law
B
E dl = ds Faraday’s law
c
t
s
+Q –Q
J = Continuity equation d
dt

9. (b) 13. (d)


The radially outward field is normal to The surface change density is equally to normal
conductor and no induction effects E is same. flux density,
Dnormal = s
+ + + D = s = E = 80 2
– – –
+

14. (c)
+

+q


+

D
×H = +J


+

t
+



+ +
+
1 Amere A
× =
m m m2
10. (b)
15. (c)
s
E = aˆn
2 V= E dl
where, aˆn = aˆ y 2 0 0
1 = x dx + y dy + z dz
= × 10 6 C/m 2 1 2 3
s 6000
2 0 0
10 6 × x2 y2 z2
E = aˆ y = + +
2 × 8.854 × 10 12 × 6000 2
1
2
2
2
3

E = 3 aˆ y = 3 j V/m 1 2
= [2 12 + 02 2 2 + 02 32 ]
11. (a)
2
1
Net flux leaving = Charged enclosed = × 10 = 5 V
2
Cube encloses the sphere completely,
16. (c)
D ds = Qenclosed
E1 = 2ux – 3uy + 1uz
Qenclosed = s A = s (4 r2)
= 10 (4 × 12) = 40 E1t = –3uy + uz = E2t (x = 0 plane)

12. (c) E1n = 2ux


Q
V= (monopole) D1n = D2n = E
4 or
1 E1n = 2 E2n
Qd cos
V= 2
(dipole)
4 or = 1.5 o 2 ux = 2.5 o E2 n
22 Electronics Engineering Electromagnetic Theory

3 For wire in the y-z plane,


E2n = ux = 1.2 ux
2.5 Hdirection = aˆ y × ( aˆ z ) = aˆ x
E2 = E2t + E2 n For wire in the x-y plane,

E2 = –3uy + uz + 1.2ux Hdirection = aˆx × ( aˆ y ) = aˆ z


Therefore, resultant magnetic field has x and z
17. (c)
components at the origin.
Et1 = 3aˆ y + 5aˆ z = Et2
22. (c)
Dn1 = Dn2 Applying,
H=J
2 En2 = 1 En1

3 × 4 aˆx
En2 = = 3 aˆ x aˆ x aˆ y aˆ z
4
E2 = Et2 + En2 ×B = =0
x y z
= 3 aˆ y + 5 aˆ z + 3 aˆx y x
2 2
0
x +y x + y2
2
19. (d)
Intensity – Curl – Line integral J=0
Density – Divergence – Surface integral
23. (a)
This is order of understanding Maxwell’s
For magnetic field boundary relations are,
equations.
For any material, Bn1 = Bn2
×E = 0; B=0
and Ht Ht2 = J s × an
×H = J; D= v
1

For source free like charge or current free Bx1 = Bx2 (x is normal component)
conditions,
×E = 0; B=0 µ1 H x1 = µ2 H x2
×H = 0; D=0
1 × 3 = 2 × H x2
If D = E and B = µH and or µ are constant
with distance or direction then, H x2 = 1.5
B
( E) = 0 ; × =0 Ht1 Ht 2 = 10 uˆ y × uˆ x = +10 uˆ z
µ
E = 0; ×B=0
H t2 = 30 uˆ y 10 uˆ z
These are called as homogeneous/isotropic
medium.
H 2 = 1.5 uˆ x + 30 uˆ y 10 uˆ z A/m
For non-homogeneous material. However they
are not valid.
24. (c)
20. (d) We know that magnetic flux density at a
Hdirection due to any current wire = I flow distance ‘r’ from the wire is,
direction × Radial vector from current to point
of consideration.
GATE Previous Years Solved Paper 23

µ0 I 28. Sol.
B =
2 r y
For r < a, I=J r2
µ0 Jr
So, B =

40 kV/cm
2
B r for r < a
For r > a, I = J × a2 20 kV/cm
2
µ0 J a
So, B =
2 r 0 kV/cm x
A 5 µm B
1
B for r > a
r E is linear from A to B and E = mx + c.
We can simply find area of E – x plot to find
25. (a)
answer = 15 V.
+J
29. (d)
–J
r1 V V V
E= aˆ x aˆ y aˆ z
a r2 x y z
b

V
Hole = 2y3 – 3yz2
x
Cylindrical
V = 2xy3 – 3xyz2 + K1
The H field due to entire cylinder without hole, V
= 6xy2 – 3xz2
Jr1 y
H(r1) =
2 V = 2xy3 – 3xyz2 + K2
The H field due to hole only, V
= –6xyz
Jr z
H(r2) = 2
2 V = –3xyz2 + K3
Current inside the hole is zero means, Comparing all three for uniqueness solution of
Jr1 Jr2 Jd potential,
H(r) =
2 2 2 K 1 = K2 = 0, K3 = 2xy3
27. (d) 3 2 2 2
E = (2 y 3 yz ) xˆ (6 xy 3xz ) yˆ + (6 xyz) zˆ
Since, we know from the property of magnetic As we know that,
boundary conditions, E=– V

Ht2 Ht1 = Kˆ s × aN
E= xˆ + yˆ + zˆ V ...(i)
x y z
In the given question the tangential field H in
Clearly option (d) satisfies the given relation.
the air just above the perfect conductor is asked
with H = 0 inside the conductor, 30. (c)
Ht1 = 0 I
H = aˆ
Ht2 = xˆ 2 × yˆ 2 r
1
Ht2 = zˆ 2 A/m H
r
Option (c) is satisfied.
24 Electronics Engineering Electromagnetic Theory

31. Sol. /4
400 r 2 1 cos 2
= 2x2 + y2 + cz2 = 2
d
(r + 4) /12 2
In source free region,
D =0
/4
400 r 2 4 12 sin 2
( E) = 0 =
(r 2 + 4) 2 4 /12
E =0
where, E = V= 400 r 2 1 1/2
= 2
aˆ x + aˆ y + aˆ z (r + 4) 12 4 r = 0.8
Now, V=
x y z
400 × 0.8 × 0.8
E = 4x aˆ x + 2 y aˆ y + 2cz aˆ z = × 0.13 = 7.17 A
4.64
Again, E =0 Total area
Ex Ey E = ds = r 2 sin d d
+ + t = 4 + 2 + 2c = 0
x y z /4
2
c = –3 = r sin d 2
= /12
32. Sol.
2
= r 2 0.259
2ˆ r = 0.8
D = 2 a + zaˆ z
1
Surface, = 1, z = 0 and z = 5 = 0.82 × 0.5 × 2 × = 1.041 m 2
4
D ds = D ds + D ds + D ds 7.17
Top Bottom Curved Average current = = 6.88 A/m 2
s surface surface surface 1.041
= 5 + 0 + 20 = 25 = 78.54 Note : GATE key mentioned (MTA-marks to all)

33. Sol. 35. (a)


Baz magnetic field,
D ds = Q (Charge enclosed)
vx ax + vz az velocity,
Q1 + Q2 + Q3 = 0 F = Q (v × B) By Lorentz’s
For D= 0 law
4 2 2+ 4 4 2+ 4 82 = Q = 0
s1 s2 s3 = Q (vx ax + vz az) Baz
20 4 – 4.42 + s3 82 = 0 Fy = Qvx B(–ay)
80 – 64 + s3 82 = 0 This results in circular path in the XY plane
16 with vz az component causing a linear path.
s3 = = 0.25 nC/m 2
64 Both resulting helical path along Z axis.

34. (*) 36. (c)


I= J ds Velocity being free velocity,
1
/4 2 mv 2 = qV
400 sin 2
= 2
r 2 sin d d
= /12 = 0 2 ( r + 4) d 2qV
v = =
2 /4 t m
400
= r2 sin 2 d d
2 (r 2 + 4) 0 /12
t
V
GATE Previous Years Solved Paper 25

37. (a) • Va = Vb
Let, E = E1 Q Q
So, =
Q12 4 0 a 4 0b
Energy, E1 =
2C1 Qb b
=
Electrically isolated Q2 = Q1 Qa a
d 2 = 2d 1 • Now, surface electric fields,
C1
C2 = Ea Qa /4 0 a2
2 =
Eb Qb /4 0 b2
Q22 Q12 Q12
E2 = = 2
2C 2 2C 1 2C 1 Qa × b 2 b
= = >1
2 Qb × a 2 a
= 2E1 = 2E
So, Ea > Eb
38. (d)
40. Sol.
Applying Poisson’s equations,
q1 5
v = 10 m/s q2
2
V v
2V = = =K moving r stationary
x2
Constant charge density, ‘r’ is the distance at which kinetic energy of q1
V becomes zero [(because kinetic energy (KE) is
= –Kx + K converted into potential energy (PE)].
x
When, q1 reaches ‘r’, it starts diverting.
Kx 2
V= +K x+K 1
2 Kinetic energy, KE = mv 2 and work done in
Towards positive x or negative side. 2
It is a second order parabolic increase. moving q1 charge to distance ‘r’ is
Due to symmetry of + and – charges K = 0 is q2
expected with V = 0 at center and graph passing q 1v 2 = q1
4 0r
through origin.
(q1 = q2 = –1.6 × 10–19 C)
Beyond x > a or x < b, E = 0 due to capacitive
1 q q
nature of + and – charges, Now, mv 2 = 1 2
2 4 0r
V= 0 dl = constant 19 19
(2 × 1.6 × 10 ) × ( 1.6 × 10 )
r= 9
This constant is same at x = a or x = b. 10 31
4 × × 9.11 × 10 × (10 5 )2
36
39. (c)
5.06 × 10–8 m
S2
S1 41. (d)
Wire
D = v

Radius a B
Radius b ×E =
t
• When charge is placed on this structure,
equilibrium is established such that both B =0
spheres are at same potential i.e., D
×H = J +
Va = Vb t
26 Electronics Engineering Electromagnetic Theory

42. (d)
Then, I = C ( x dx + y dy )
Electric field intensity (E) at ‘ ’ distance due to
infinite long line having line charge density Q
j
is
C
Q
E = aˆ
2 0
45°
Q
E da = aˆ d dz aˆ i 1
2 0
H But, x = r cos , Y = r sin
Q
= d dz dx = –r sin d , dy = r cos d
2 0 < /2 > z=0
Here, : 0 to
Q HQ 4
= H=
2 0 2 4 0 Now,
/4

43. (d) I= [( r cos ) ( r sin ) d + (r sin ) (r cos ) d ]


=0
W1 I
/4
r I= [2r 2 cos sin ] d
=0
4r
/4
W2 2I I= sin(2 ) d ( r = 1)
=0
Magnetic flux density ( B) at ‘r’ distance due to /4
cos(2 ) 1 1
I= = (1 0) =
µ I 2 0 2 2
infinite line carrying current ‘I’ is B = 0 .
2 r
45. (56.54)
• B at ‘r’ distance due to W1 wire
Given: D = cos2 aˆ + z2 sin 2 aˆ
µ0 I
= B1 = ...(i) Z
2 r
top
• B at 3r distance due to W2 wire
Z=2 =3
µ0 (2 I )
= B2 = ...(ii)
2 (3r )

From right-hand thumb rule, B due to both lines side

add in between conductors.

So, B = B1 + B2
Z=0
µ0 I 2µ0 I 5µ0 I
B = + = bottom
2 r 6 r 6 r
The net flux leaving the closed surface of the
44. (a)
cylinder is,
Let, I = C f (r ) dr = C ( f 1 dx + f 2 dy ) net = bottom + top + side
GATE Previous Years Solved Paper 27

48. (512)
net = D dS + D dS + D dS
( z = 0) ( z = 2) ( = 3)
S 4 x
I= (3x 2 + 3y 2 ) dy dx
As dS = d d ( ± az ) for z = constant and hence 0 x

4 x
D dS = 0 for both z = 0 and z = 2 for =3: 3y 3
= 3x 2 y + dx
2 2
3
D dS = ( cos aˆ + z sin aˆ ) d dz aˆ 0 x

4
2 2 = (3x 2 (2 x ) + 2 x 3 ) dx
2
net = cos2 d dz 0
( = 3)
z=0 =0
4 4

2 1 = (6x 3 + 2 x 3 ) dx = 8x 3 dx
= (3) × (2) × (2 ) 0 0
2
= 18 (or) 56.54 8 4 4
net = ( x )0 = 2 × 4 4 = 512
4
46. (0)
Given that: 49. (0.5)

F = F1 aˆx + F2 aˆ y + F3 aˆ z is an irrotational Qenclosed = v dv = ( D) dv


v
× F = 0 (or) Curl F = 0
D= ( x3 ) + (y3 ) + ( xy 2 )
aˆ x aˆ y aˆ z x y z
= (3x2 + 3y2) = 3(x2 + y2)
=0
x y z dv = dx dy dz
(4 y c1 z) (4 x 2 z) (2 y z)
Qenclosed = 3( x 2 + y 2 ) dx dy dz
v
(2 2) aˆ x [0 (0 c1 )] aˆ y + (4 + 4) aˆ z = 0
0.5 0.5 0.5
(0) aˆx (c1 ) aˆ y + (0) aˆ z = (0) aˆx + (0) aˆ y + (0) aˆ z = 3 ( x 2 + y 2 ) dx dy dz
x = 0.5 y = 0.5 z = 0.5
–c1 = 0
c1 = 0 x3
0.5
y3
0.5

= 3 { y } 0.5 0.5 0.5


0.5 { z} 0.5 + { x} 0.5 { z} 0.5
0.5
3 3
0.5 0.5
47. (a)
= (((0.5)3 – (0.5)3) (0.5 + 0.5)
F( x , y ) ( dxi + dyj ) = ( xi + yj ) ( dxi + dyj ) (0.5 + 0.5) × 2
C C Qenclosed = 0.5C
= xdx + ydy
C
By Green’s theorem

= (0 0) dx dy
R
=0
28 Electronics Engineering Electromagnetic Theory

Answers
EE Basics of Electromagnetics (Section-A)

1. (a) 2. (d) 3. (a) 4. (a) 5. (c) 6. (b) 7. (b) 8. (4.41)

9. (c) 10. (180)

Solutions
EE Basics of Electromagnetics (Section-A)

2. (d) 6. (b)
Divergence of a curl field is always zero. Stokes theorem,
i.e., ( × E) = 0 A dl = ( × A ) nˆ ds
Gauss theorem,
3. (a)
V(x, y, z) = –x(cos xy + y) i + (y cos xy) j D ds = Q
× [sin(z2) + (x2) + (y2)] k Divergence theorem,
Divergence = V
A nˆ ds = A dV
Vx Vy V
= + + z Cauchys integral theorem,
x y z
= –cos xy + x(sinxy)y + cos xy f ( z) dz = 0
–y sin(xy)x + 2z cosz2 c
= 2z cosz2
7. (b)
4. (a) V is given as static field in time invariant.
A = kr iˆr n Hence, ×E = 0
1
A = (r 2 kr n ) 8. Sol.
r2 r
1 E = 5x zi + (3x 2 + 2 y ) j + x 2 zk
= ( kr n + 2 )
r2 r = F dr
n+1 C
r
= ( n + 2) k 2
= ( n + 2) kr n 1
r 5x z dx + (3x 2 + 2 y ) dy + x 2 z dz
=
For A = 0 C
n+2 = 0 x = t, y = t2, z = t, t = 0 to 1
n = –2 dx = dt
dy = 2t dt, dz = dt
5. (c)
1
= 5t 2 dt + (3t 2 + 2t 2 ) 2t dt + t 3 dt
1 0
r̂ f =
r2 1
= (5t 2 + 11t 3 ) dt
From divergence theorem as we know, 0

1
( f ) dV = f dS 5t 3 11t 4 5 11 53
vol. s + = + = = 4.41
=
3 4 3 4 12
1 0
f dS = 2
rˆ × r 2 sin d d rˆ
s s r

= sin d d =4
s
GATE Previous Years Solved Paper 29

9. (c) ( D) dV
X is going away so X 0. 15
= 3 d d dz
Y is moving circulator direction so ×Y 0.
2
= 15 d d dz 3 d d dz
Z has circular rotation so ×Z 0.
3 2 5 3 2 5
2
10. Sol. = 15 d d 3 d d dz
=0 =0 z=0 =0 =0 z=0
/crossing closed surface

= D ds = ( D) dV ...(i) 33
= 15(3 0) (2 ) (5) 3 × (2 ) (5)
3
1 1 D Dz
D = ( D )+ + = 45(10 ) – 27(10 ) = 180 C
z
1 1
= ( 15) + (2 ) + ( 3 z)
z
1
= 15 3

Answers
EE Basics of Electromagnetics (Section-B)

1. (2.24) 2. (a) 3. (True) 4. (b) 5. (d) 6. (4.72) 7. (11.11) 8. (d)

9. (a) 10. (b) 11. (d) 12. (b) 13. (d) 14. (a) 15. (c) 16. (b)

17. (c) 18. (c) 19. (a) 20. (c) 21. (a) 22. (b) 23. (18.75) 24. (a)

25. (d) 26. (d) 27. (c) 28. (c) 29. (d) 30. (b) 31. (10.03) 32. (d)

33. (2.53) 34. (10) 35. (d)

Solutions
EE Basics of Electromagnetics (Section-B)

1. Sol. Magnitude of E ,
Electrostatic potential,
E = 2 2 + 12
= 2x y
= 5 = 2.24 V/m
Q E = –grad ( ) = –

E = aˆx (2 x y ) + aˆ y (2 x y ) 2. (a)
x y
Gauss Law: The electric flux passing through
x any closed surface is equal to the total charge in
E = aˆ x 2 y aˆ y
y the volume enclosed by the surface i.e.,
Now, at x = 1 m and y = 1 m
D ds = v dV
E = 2 aˆ x aˆ y
30 Electronics Engineering Electromagnetic Theory

3. Sol. On putting the given values,


(True) 8.85 × 10 12 × 400 × 10 4 1 × 2 4
C= +
By stroke’s theorem, 20 × 10 2 1+2 2
( × E) d s = E dl C = 4.72 × 10–12 F
s c = 4.72 pF
which represents the potential difference
7. Sol.
around a closed path, C which is zero in
electrostatic field. C=4 0R

10
Hence, × E = 0 i.e. electromagnetic fields are = 4 × 8.85 × 10 12 × (in air)
100
conservative in nature.
= 11.11 pF
4. (b)
8. (d)
By the fundamental definition of voltage,
V(x, y, z) = 50x2 + 50y2 + 50z2
dW
V= E =– V
dq
= [100x iˆ + 100 y ˆj + 100 z kˆ ] V/m
5. (d)
E (1, 1, 1) = [100 iˆ 100 ˆj + 100 kˆ ] V/m
Field liens are always from higher potential to
lower potential point, so at equipotential E = 100 2 + 100 2 + 100 2
surfaces field lines are always normal or
= 100 3
orthogonal.
E 1
6. Sol. aˆE = = (i j + k)
E 3

C1 9. (a)
C3
(1, 1, 0)
C C2

r
d = 20 cm
A = 20 × 20 = 400 cm2
(–1, 1, 1)
C1C2
C= + C3
C1 + C 2
r = [1 ( 1)] iˆ + (1 1) ˆj + (0 1) kˆ
A A
0 r1
2
0 r2
2 = 2iˆ kˆ
d d A
0 r3
Distance between (1, 1, 0) and (–1, 1, 1) = r
= 2 2 + 2
A A d = (1 ( 1)2 + (1 1)2 + (0 1)2 = 5
0 r1 0 r2
2 + 2
d d
Q 10 6
2 2 E = 3
(r ) = 3
(2 iˆ kˆ )
4 0r 4 0 ×( 5 )
A r1 r2 0 r3 A
= 0 +
d r1 + r2 2d 10 6
= (2 iˆ kˆ )
0 A r1 r2 r3
20 5 0
= +
d r1 + r2 2
GATE Previous Years Solved Paper 31

10. (b) 14. (a)


q q By Gauss’s theorem,
VP – VQ =
4 0 (OP ) 4 0 (OQ ) Qenclosed
E × ds =
9 9 0
10 10 s
= 9 × 109 3 3 where, Qenclosed in radius r,
40 × 10 20 × 10
Q 4 1
= –225 Volts E ds = 4 × r3 ×
s R3 3 0
11. (d) 3
1 1 Qr 3 1
E= QV = CV 2 E(4 r2) = ×
2 2 3
R 0
1 0A
= × ×V 2 Qr
2 d E=
4 0R
1 8.85 × 10 12 × 100 × 10 6 × 10 4
= ×
2 10 4 15. (c)
= 44.3 nJ Net electrical flux passing through plane Z = 20,
Q1 + Q2 10 + 20
12. (b) Q= = = 15 µC
2 2
r1 0 A
C1 = 12. (b)
t1
A1 = A2 = 500 × 500 mm2 = 25 × 10–2 m2
r2 0 A
C2 =
t2 r1 = 8, r2 = 2, d1 = 4 mm, d2 = 2 mm

Let voltage on the conductor coil is V, then


C1(100 – V) = C2 V r1 =8 d1

[Charges on both capacitors are equal]


r2 r2 =2 d2
r1
(100 V ) =
t1 t2
6(100 – V) = 4 V
A r1 0 A r2 0
600 ×
V= = 60 Volts C1 × C 2 d1 d2
10 Ceq = =
C1 + C 2 A r 1 0 + A r 2 0
13. (d) d1 d2
When two capacitors formed by two layers of A 0 r1 r2
dielectrics are connected in series, then Ceq =
r 1 d2 + r 2 d1
equivalent capacitance,
6 12
C1 × C2 500 × 500 × 10 × 8.85 × 10 ×8× 2
Ceq = = 3
C1 + C 2 (8 × 2 + 2 × 4) × 10

A 0 r2 A = 1475 pF
r1 0 ×
d1 d2 17. (c)
=
r1 0 A 0 r2 A
+ A
d1 d2 Q = CV = 0 r × V
d
12 3 2
(8.85 × 10 ) (400 × 10 ) × 4×2
= V
4 × 8 × 10 3
+ 6 × 2 × 10 3 = 0 r A× = 0 r A× E
d
= 257 pF
32 Electronics Engineering Electromagnetic Theory

= 8.85 × 10–12 × 2.26 × 20 × 10–2


× 40 × 10–2 × 5000 × 103
V
= 8 mC V 0 r
C1 C2

18. (c)
1 2 Equivalent capacitance is,
Energy density =E
2 Ceq = C1 + C2
Energy = Energy density × Volume 0A 0 rA
= +
1
× 2
+ 82 ) × 1012 × 0.5 × 0.5 × 0.4
2d 2d
Energy = 0 × r (6
2
0A C0
1 Ceq = (1 + r)=
(1 + r )
12
= × 8.85 × 10 × 2 × 100 × 1012 × 0.1 2d 2
2 [Using equation (i)]
= 88.5 J
Therefore, modified capacitance,
19. (a) C0
Ceq = (1 + r )
Let A be the area of the parallel plate capacitor 2
and ‘d’ be the distance between the plates. 20. (c)
With air dielectric:
Let, A = Area of plates
Let, C1 = C3 be the capacitance formed with
=1
dielectric having dielectric constant 1.
r d
Ceq be the equivalent capacitance.
C2 be the capacitance formed with dielectric
having dielectric constant r2.
A
Capacitance, C0 = 0 ...(i) 4
d 0 1A
Then, C1 = C3 =
With the new arrangement: d
0 1A 2 0 2A
and C2 = =
d /2 d
0 r d Also, equivalent capacitance = Ceq
1 1 1 1 2 1
= + + = +
C eq C1 C 2 C 3 C1 C 2
Let C1 be the capacitance of half portion with (Q C1 = C3) (C1, C2, C3 all in series)
air as dielectric medium and C2 be capacitance 2d d
1
with a dielectric of permittivity, r. or, = +
C eq 4 0 1d 2 0 2 A
A
0 2 1 2 0A
2 or, Ceq =
Then, C1 = (As area becomes half) d( 1 + 2 )
d
Given, Veq = Total voltage = 10 Volt
A V1 = V3 = 2 Volt
0 r
2 1
and C2 = (As area becomes half)
d We know that, C
V
Now, these two capacitance will be in parallel
C eq V1 C eq 1
if a voltage is applied between the plates as same = or =
C1 Veq C1 5
potential difference will be there between both
the capacitances. 2 0 1 2A d 1
or, × =
d( 1 + 2 ) 4 0 1 A 5
GATE Previous Years Solved Paper 33

2 1 24. (a)
or, =
2( 1 + 2) 5 Given that:
or, 5 2 = 2 1+2 2
At the interface (y = 0) there is no surface charge.
or, 2 1 =3 2 Region 1 (y < 0) has r = 2
or, 1: 2 = 3:2 Region 2 (y > 0) has r = 5
Electric field in region 1 is 3ax + 4ay + 2az
22. (b) Normal component of electric field is 4a y
Due to charge at (0, 0, 2) and conductor plane tangential component of electric field is 3ax + 2ay
there is an image at (0, 0, –2). Now, E1 = E1t + E1n
Total field is E1 + E2 The tangential component of E1 = tangential
component of E2,
32 0 2 E1t = E2t = 3ax + 2az
E1 = 3 /2
( 2 aˆx + 2 aˆ y 2 aˆ z )
4 0 (2 + 2 + 4) and 1E1n = 2E2n
32 2
so, E2n =
0
E2 = ( 2 aˆx + 2 aˆ y + 2 aˆ z )
4 0 (2 + 2 + 4)
3 /2 1 × 4 ay 2 × 4 ay
= = 1.6 ay
2 5
E = 2 aˆ z = 2 kˆ
so, E2 = E2t + E2n
= 3ax + 1.6ay + 2az
23. Sol.
A capacitor is made as shown in figure. 25. (d)
A Due to image of the charge, three new charges
will be created as shown in figure.
5 mm C1 =1
r y
5 mm Air r=1

5 mm r =4
3 4
5 mm C2 =4
r –Q d d Q

d
B 2d
Breakdown of dielectric air is 30 kV/cm and x
breakdown of dielectric glass is 300 kV/cm. d
We need to find maximum value of VAB so that 2d

no C1 or C2 breaks down. +Q –Q
Since, area and width of C1 and C2 is same but 2 1

r1 = 1 and r2 = 4.
Total force (4) due to (1), (2), (3) will be
So, C 2 = 4C 1
Ftotal = F41 + F42 + F43
So, if VAB is applied,
F41 = Force as (4) due to (1)
4
VAB and VC 2 = 1 VAB
VC1 =
5 5 Q2 1 ˆ
= j
For C1 not breakdown, we need, 4 0 (4 d 2 )
4 VAB F42 = Force as (4) due to (2)
< 30 kV/cm
5 5 mm
Q2 1 ˆj iˆ
30 × 25 × 1000 = + +
VAB < V 4 0 8d 2 2 2
4
VA < 18.75 kV
34 Electronics Engineering Electromagnetic Theory

F43 = Force as (4) due to (1) 29. (d)


Q 2
1 ˆ Given that electron, neutron and proton are in a
= i straight line.
4 0 4d 2
– +
Q2 1 2 2 ˆ 1 2 2 ˆ
Ftotal = i+ j Electron Neutron Proton
4 0 8 2 8 2
The electron will more towards proton and
26. (d) proton will more towards electron and force will
Charge, Q is located at (0, 0) and –2 is located q1 q 2
be same F = . But acceleration of
at (6, 0). 4 2
0R
To find V at any point (x, y),
electron will be more than proton as mass of
Q
VQ = electron < mass of proton. Since neutron are
4 x2 + y 2 neutral they will not move. Thus electron will
2Q hit neutron first.

( )
V–2Q =
4 ( x 6)2 + y 2 30. (b)
Added charge (one million electrons) to be solid
Vtotal = 0
spherical conductor is uniformly distributed
Q 2Q
+
( ) ( )
= over the outer surface of the sphere.
4 x2 + y 2 4 ( x 6)2 + y 2
31. Sol.
2 2 = 2( x + y ) 2 2
( x 6) + y 4 3
After burst, 4 R2a = r
3
x2 + 36 – 12x + y2 = 4x2 + 4y2
Radius of soap drop
3x2 + 3y2 + 12x = 36
= (3R2a)1/3 = 0.099665 cm
x2 + y2 + 4x = 12
Initial voltage was 1 V and C = 4 0R and initial
(x + 2)2 + y2 = 16
charge,
27. (c) Q = (4 0R × 1)
As voltage is same across both the regions and since after bursting Q remain same and C = 4 0r
distance between two plates in also same, then New potential on soap drop,
electric field remains same throughout the both Q 4 0R 1
V= = = = 10.03 V
region. As we know, C 4 0 rR 0.099665
V
E= = constant for both regions 32. (d)
d
Z
28. (c)
(0, 0, 0.2) Q = 1 nC

P (0, 0, 0.1)
A C
Z=0
B

At the point C , E (electric field intensity) is


(0, 0, –0.2) Q = –1 nC
maximum being closest to the other plate.
Image change
GATE Previous Years Solved Paper 35

Net electric field at point P due to charge Q is, 34. Sol.


QR12 Co-axial cylindrical capacitor-1,
E12 = 3
4 0 R12 R

1 × 10 9 ( 0.1 aˆ z ) r
Total, E12 = 12
4 (8.854 × 10 ) (0.1)3
9
( 1 × 10 ) ((0.3) aˆ z ) =2
+ 12 3
r1

4 (8.854 × 10 ) (0.3)
2 h 2 (2) h
0
10 5 105 C1 = =
= aˆ z b R
4 (8.854) 4 (8.854)9 ln ln
a r
= ( 898.774 99.863) aˆ z 4 0h
C1 = R ...(i)
999.09 aˆ z V/m ln
r
33. Sol. Co-axial cylindrical capacitor-2,
0A
Given, C= = 60 pF
d
R
In second case:
C1 C2
3 /2 r /2
r2

r1 =2

3
0 (2) h 0 r2 h
2
C2 = +2
R R
ln ln
d/2 r r
Capacitance, h 0 r2
C2 = 3+ ...(ii)
0A 2 A R 2
C1 = = 0 ln
d /2 d r
= 2 × (60 pF) = 120 pF Given, C 2 = 2C 1
2 0 rA Put equation (i), (ii) in equation (iii),
and C2 = = (2 × 60) r pF
d
= 120 r pF h 4 h
0 r2 0
120 × 120 r 3+ = 2
C1C 2 R 2 R
Now, Ceq = = pF ln ln
C1 + C 2 (120 + 120 r ) r r
= 80 pF (Given)
120 × 120 r 3 + r2 = 8
or, 86 = 2
120(1 + r )
r2
86 r 86 =5 = 10
= or = 2.53 2 r2
120 r= 34
1+ r
36 Electronics Engineering Electromagnetic Theory

35. (d) 1 1D D
v = (rDr ) + + z
E= r r r z
D = 0 rE
1 1 6.75 0
3 = r r (r 4.5 0 r ) + r + (13.5 0)
0 2.25 2r aˆr + aˆ + 6 aˆ z r z
v
D = r
1 2
6.75 = (4.5 0r )+0 +0
0 r r
D = 4.5 0 r aˆr + aˆ + 13.5 0 aˆ z
r
1
Volume charge density, = (4.5 0 ) (2 r ) = 9 0
r
v = D

Answers
EE Basics of Electromagnetics (Section-C)

1. (a) 2. (False) 3. (b) 4. (b) 5. (d) 6. (a) 7. (c) 8. (c)

9. (d) 10. (d) 11. (a) 12. (b) 13. (a) 14. (c) 15. (c) 16. (a)

17. (100) 18. (a) 19. (0.69) 20. (138.63)

Solutions
EE Basics of Electromagnetics (Section-C)

1. (a) 3. (b)
Using Stoke’s theorem, µo N 2 A
L=
A dL = ( × A) d s l
s 2
7 3
4 × 10 × (1000)2 × × × 10 2
Q A is vector magnetic potential = 2
2
× A = curl ( A) 30 × 10
= 2.96 mH
= Magnetic flux density B
1 2 1
E= LI = × 2.96 × 10 3 × 10 2
( × A) d s = B ds 2 2
s s = 0.148 J 0.15 J
= flux through the surface S
4. (b)
2. Sol. Use right hand thumb rule.
(False)
5. (d)
According to Faraday’s law: induced voltage
F = Felect + Fmag = qE + qu + B
d
i.e. only time varying fields induce voltage
dt = q (E + u + B) ...(Lorentz force)
and hence responsible for production of For an electron q = –e
induced currents in closed loops. F= e ( E + u × B)
GATE Previous Years Solved Paper 37

6. (a) XL = 2 fL

H = H1 + H2 V V
I= =
XL 2 fL
I I I
= yˆ + yˆ = yˆ
2 d 2 d d 230
= = 2.28 A
2 × 50 × 0.3215

11. (a)
1
+I + P – –I Energy stored = ×L× I2
2
1
× 0.3215 × 2.282
=
2
= 0.8356 J
7. (c) Force to reduce 1 mm air gap = E/d
Inductance/m is proportional to separation 0.8356
= 3
= 835.6 N
between the strips so when separation is 1 × 10
reduced to x/2 then inductance will become
12. (b)
L/2 H/m.
n = 300
8. (c) (I) circumference = 2 r = 300 mm
Let, B = µ0 nI A = 30 mm2
then, = B.S = µ0 n SI = 300 × 10–6 m2
where ‘S’ is the cross-sectional area of solenoid. Inductance of coil,
Flux linkage,
µ0 N 2 A
a = n = µ0 n2 SI L=
I
Hence inductance/length = µ0 n2 S
For, l = 1000 mm = 1 m 4 × 10 7 × 300 × 300 × 300 × 10 6
=
L = 4 × 10–7 × 30002 × × (30 × 10–3)2 300 × 10 3
= 32 mH = 113.04 µH

9. (d) 13. (a)


The divergence of magnetic field is always zero B=0
because magnetic flux makes always a closed
ax + ay + az (4 xax + 2 kyay + 8az ) = 0
path. x y z
So, B = 0 (Maxwell’s equation) or, 4+2k = 0
while divergence of electric field, k = –2
v
E = 15. (c)
The property of a solid magnetic field is,
10. (d) B=0
µ0 N 2 A (i.e. solenoidal property)
L= 2 2 2
l when, B = x aˆ x + y aˆ y + z aˆ z
7
4 × 10 × (400)2 × 16 × 10 4
B = 2x + 2y + 2z 0
=
(1 × 10 3 ) Hence option (c).
= 0.3215
38 Electronics Engineering Electromagnetic Theory

16. (a) Here, B at point P is,

µ0 I µ0 I µ0 I
( zˆ ) + ( zˆ ) B= (sin 1 + sin 2)
B = L 3L 4 d
2 2
2 2 We know for each segment of hexagon,

µ0 I ( zˆ ) 2 4 µ0 I P
= 2+ = zˆ
2 L 3 3 L

17. Sol. d
Toroid has field,
B µ 1 2
x
As, µ = 100 (inside field) I
Magnetic field density B at any point at a
Magnetic field intensity due to elemental length
distance at r is,
dx,
µI
B= I dx
2 r dH = i × ur
4 r2
µ 0 µr I
Now, B = (Just inside toroid) where,
at r 2 r
i × ur = (sin ) kˆ
µ0 I ...(Perpendicular to plane of paper
and B = (Just outside toroid)
at r + 2 r+
I sin
H= dH = ( dx ) kˆ
B 4 r2
at r
= µr = 100 and B = µ0 H
B
at r +
µ0 I ˆ
So, Bp = (cos 1 + cos 2) k
4 d
18. (a)
Iron being a ferromagnetic material, magnetic 3
d = l (where, l = 1 m)
lines of force bend closer to cylindrical axis. 2
and 1 = 2 = 60°
19. Sol.
i = 1A µ0 I
Bp = (cos 60° + cos 60°)
3
4
2
6
l For all six segments,

4 × 10 7 × 6 × 1
1 d 5 6 × Bp = (1)
3
4 ×
3 2
d=
2
12 7 1.20 6
= × 10 = × 10 T
2 4 3 3
3 B = 0.69 µT

i
GATE Previous Years Solved Paper 39

20. Sol. µ0 I
= (ln )2 = 1 ( z)1z = 0
I 2
= MI µ0 I
= (ln 2)
µ0 I 2
B = aˆ ( B due to infinite long line)
2
µ0 (ln 2) 4 × 10 7 (ln 2)
Magnetic flux crossing square loop is, M= = =
I 2 2
= B ds = 1.386 × 10–7 Henry
µ0 I 138.63 nH
= aˆ ( d dz) aˆ
2
2 1
µ0 I d
= dz
2
=1 z=0

Answers
EE Basics of Electromagnetics (Section-D)

1. (c) 2. (d) 3. (248.05) 4. (0.193) 5. (9.55)

Solutions
EE Basics of Electromagnetics (Section-D)

1. (c) The turn rotate at 60 rpm


d So, o = 2
According to Faraday’s law e . So, the flux flowing through the circular turn
dt
Even by considering Lenz’s law we have, will be
= (µ0 Hz × Area of turn × cos ot)
d
e= = 4 × 10–7 × 107 A/m × × 12 × cos ot
dt
d
2. (d) Maximum voltage induced is .
dt max
An oscillating dipole is a dipole (or antenna) d
through which when ac current is fed, it radiates So, =( o×4 × sin ot)max
dt max
EM wave. Vmax = (4 2×2 ) = 248.08 Volts
3. Sol. 4. Sol.
Z
A = 10 cm × 5 cm
Hz
B(t) = 0.25 sin t, R = 0.4
P = i2R
X
d d
Q e= = ( B × A)
dt dt
d
The circular turn rotate with 60 rpm, let the angle = [0.25 × 50 × 10 4 sin t ]
dt
made by ring w.r.t. x-axis e = –0.25 × 50 × 10–4 cos t
and = ot = –1.25 × 10–3 cos t
40 Electronics Engineering Electromagnetic Theory

2
erms So, Em = 1 V/m
Pavg. = As we know, Em = V × B
R
T
(12.5 × 100 × 10 4 )2
where, V = (radius of path) × (angular velocity)
P= × cos 2 t d( t )
0.4 1V
0 = (V × 1 Tesla)
m
2 2 1
= (12.5 × × 10 ) × v = 1 m/sec
2 × 0.4
v = r × = 1 m/sec
= 0.193 Watt
Since, r = 1m
5. Sol. So, = 1 rad/sec
Now from this we get,
1
Voltage induced = Em dl N
= 2× × = 1rad/sec.
0 60
(where Em is induced electric field) 30
N= = 9.55 revolutions/minute
= Em × Volts
Since,
Voltage induced = 1 V
2 Uniform Plane Waves

ELECTRO NICS EN GINEERIN G Q.4 The incoming solar radiation at a place on the
surface of the earth is 1.2 kW/m2. The amplitude
(GATE Previous Years Solved Papers)
of the electric field corresponding to this
Q.1 For an electromagnetic wave incident from one incident power is nearly equal to
medium to a second medium, total reflection (a) 80 mV/m (b) 2.5 V/m
takes place when (c) 30 V/m (d) 950 V/m
(a) the angle of incidence is equal to the [EC-1990 : 2 Marks]
Brewster angle with E field perpendicular
to the plane of incidence. Q.5 The electric field component of a uniform plane
(b) the angle of incidence is equal to the electromagnetic wave propagation in the
Brewster angle with E field parallel to the Y-direction in a lossless medium will satisfy the
plane of incidence. equation
(c) the angle of incidence is equal to the critical 2
Ey 2
Ey
angle with the wave moving from the denser (a) =µ
2
y t2
medium to a rarer medium.
(d) the angle of incidence is equal to the critical 2 2
Ey Ey
angle with the wave moving from a rarer (b) =µ
x2 t2
medium to a rarer medium.
2 2
[EC-1987 : 2 Marks] Ex Ex
(c) =µ
y2 t2
Q.2 In a good conductor the phase relation between
the tangential components of electric field Et and Ex2 + Ez2 µ
the magnetic field Ht is as follows: (d) =
H x2 + H z2
(a) Et and Ht are in phase.
(b) Et and Ht are out of phase. [EC-1991 : 2 Marks]
(c) Ht leads Et by 90°. Q.6 A material is described by the following
(d) Et leads Ht by 45°. electrical parameters at a frequency of 10 GHz,
[EC-1988 : 2 Marks] = 106 /m, µ = µ0 and / 0 = 10. The material
at this frequency is considered to be
Q.3 The skin depth of copper at a frequency of 3 GHz
is 1 micro (10–6 meter). At 12 GHz, for a non 1
0 = × 10 9 F/m .
magnetic conductor whose conductivity is 1/9 36
times that of copper, the skin depth would be
(a) a good conductor
9 (b) a good dielectric
(a) 9 × 4 microns (b) microns
4 (c) neither a good conductor, nor a good
dielectric
4 1
(c) microns (d) microns (d) a good magnetic material
9 9× 4
[EC-1993 : 2 Marks]
[EC-1989 : 2 Marks]
42 Electronics Engineering Electromagnetic Theory

Q.7 A plane wave is incident normally on a perfect (c) conductor or dielectric depending on the
frequency
conductor as shown in figure. Here Exi , H yi and
(d) conductor or dielectric depending on the
P are electric field, magnetic field and pointing
i electric current density
vector, respectively, for the incident wave. The [EC-1995 : 1 Mark]
reflected waver should have Q.11 A uniform plane wave in air is normally
X incident on an infinitely thick slab. If the
refractive index of the glass slab is 1.5, then the
percentage of the incident power that is reflected
j from the air glass interface is
Ex
(a) 0% (b) 4%
Z (c) 20% (d) 100%
Pi
i [EC-1996 : 2 Marks]
Hy
Q.12 Some unknown material has a conductivity of
106 /m and a permeability of 4 × 10–7 H/m.
The skin depth for the material at 1 GHz is
(a) 15.9 µm (b) 20.9 µm
(a) Exr = Exi (b) H yr = H yi
(c) 25.9 µm (d) 30.9 µm
(c) P r = P i (d) Exr = Exi [EC-1996 : 2 Marks]
[EC-1993 : 2 Marks]
Q.13 The intrinsic impedance of copper at high
Q.8 A plane electromagnetic wave travelling along frequency is
the +z-direction, has its electric field given by (a) purely resistive
Ex = 2 cos( t) and Ey = 2 cos( t + 90)° (b) purely inductive
The wave is (c) complex with a capacitive component
(a) linearly polarized (d) complex with an inductive component
(b) right circularly polarized [EC-1998 : 1 Mark]
(c) left circularly polarized
Q.14 The waveform of a wave with propagation
(d) elliptically polarized constant (0.1 + j0.2 ) m–1 is
[EC-1994 : 1 Mark]
2
(a) m (b) 10 m
Q.9 The intrinsic impedance of a lossy dielectric 0.05
medium is given by
(c) 20 m (d) 30 m
j µ j
[EC-1998 : 1 Mark]
(a) (b)
µ
Q.15 The depth of penetration of a wave in a lossy
j µ µ
(c) (d) dielectric increases with increasing
( +j )
(a) conductivity (b) permeability
[EC-1995 : 1 Mark]
(c) wavelength (d) permittivity
Q.10 Copper behaves as a [EC-1998 : 1 Mark]
(a) conductor always
Q.16 The polarization of a wave with electric field
(b) conductor or dielectric depending on the
applied electric field strength vector E = Eo e j( t z)
( ax + ay ) is
GATE Previous Years Solved Paper 43

(a) linear The wave impedance, (in ) is


(b) elliptical (a) 377 (b) 198.5 180°
(c) left hand circular (c) 182.9 14° (d) 188.5
(d) right hand circular [EC-1999 : 2 Marks]
[EC-1998 : 1 Mark] Q.22 A TEM wave is incident normally upon a perfect
Q.17 The time averaged pointing vector, (in W/m2), conductor. The E and H fields at the boundary
will be respectively
for a wave with E = 24 e j( t + z ) ay V/m in free (a) minimum and minimum
space is (b) maximum and maximum
(c) minimum and maximum
2.4 2.4
(a) az (b) az (d) maximum and minimum
[EC-2000 : 1 Mark]
4.8 4.8
(c) az (d) az
Q.23 A uniform plane wave in air impinges at 45°
[EC-1998 : 1 Mark] angle on a lossless dielectric material with
dielectric constant r. The transmitted wave
Q.18 A loop is rotating about the y-axis i a magnetic propagates in a 30° direction with respect to the
field B = B0 cos( t + ) aˆ xT . The voltage in the normal. The value of r is
(a) 1.5 (b) 1.5
loop is
(c) 2 (d) 2
(a) zero
[EC-2000 : 2 Marks]
(b) due to rotation only
(c) due to transformer action only Q.24 Two coaxial cables 1 and 2 are filled with
(d) due to both rotation and transformer action different dielectric constants and
r1 r2
[EC-1998 : 1 Mark]
respectively. The ratio of wavelengths in the two
cables, ( 1/ 2) is
D
Q.19 The Maxwell’s equation, ×H = J + is
t r1 r2
(a) (b)
r2 r1
based on
(a) Ampere’s law (b) Gauss’s law r1 r2
(c) (d)
(c) Faraday’s law (d) Coulomb’s law r2 r1

[EC-1998 : 1 Mark] [EC-2000 : 2 Marks]

Q.20 Identify which one of the following will not Q.25 If a plane electromagnetic wave satisfies the
satisfy the wave equation?
2 2
Ex
1 Ex
(a) 50 ej( t – 3z) (b) sin[ (10z + 5t)] equation = 2 , the wave
z2 c t2
(c) cos(y2 + 5t) (d) sin(x) cos(t)
[EC-1999 : 1 Mark] propagates in the
(a) x-direction
Q.21 A plane wave propagation through a medium (b) z-direction
[ r = 8, µr = 2 and = 0] has its electric field (c) y-direction
given by (d) xy-plane at an angle of 45° between the x
E = 0.5sin (10 8 t z) V/m and z-directions
[EC-2001 : 1 Mark]
44 Electronics Engineering Electromagnetic Theory

Q.26 A material has conductivity of 10–2 /m and a The velocity of the travelling wave is
relative permittivity of 4. The frequency at which (a) 3.00 × 108 m/sec
the conduction current in the medium is equal (b) 2.00 × 108 m/sec
to the displacement current is (c) 6.28 × 107 m/sec
(a) 45 MHz (b) 90 MHz (d) 2.00 × 107 m/sec
(c) 450 MHz (d) 900 MHz [EC-2003 : 2 Marks]
[EC-2001 : 2 Marks]
Q.32 A parallel plate air-filled capacitor has plate area
Q.27 A plane wave is characterized by of 10–4 m2 and plate separation of 10–3 m. It is
connected to a 0.5 V, 3.6 GHz source. The
E = (0.5 xˆ + yˆ e j /2
) ej t jkz
magnitude of the displacement current is
The wave is ( 0 = 1/36 × 10–9 F/m)
(a) linearly polarized (a) 10 mA (b) 100 mA
(b) circularly polarized (c) 10 A (d) 1.59 mA
(c) elliptically polarized [EC-2004 : 2 Marks]
(d) unpolarized
jkz j t
[EC-2002 : 2 Marks] Q.33 If, E = ( aˆx + jaˆ y ) e

Q.28 Distilled water at 25°C is characterized by k


and H = ( aˆ y + jaˆ x ) e jkz j t
= 1.7 × 10–4 /m and = 78 0 µ
at a frequency of 3 GHz. Its loss tangent tan is the time averaged pointing vector is
(a) 1.3 × 10–5 (b) 1.3 × 10–3 k
(a) null vector (b) aˆ z
(c) 1.7 × 10–4/78 (d) 1.7 × 10–4/(78 0) µ
[EC-2002 : 2 Marks]
2k k
(c) aˆ z (d) aˆ z
Q.29 The depth of penetration of electromagnetic µ 2 µ
wave in a medium having conductivity at a [EC-2004 : 2 Marks]
frequency of 1 MHz is 25 cm. The depth of
penetration at a frequency of 4 MHz will be Q.34 The magnetic field intensity vector of a plane
(a) 6.25 cm (b) 12.50 cm wave is given by
(c) 50.00 cm (d) 100.00 cm H ( x , t ) = 10 sin(50000t + 0.004 x + 30) aˆ y
[EC-2003 : 1 Mark]
where, aˆ y denotes the unit vector in y-direction.
Q.30 A uniform plane wave travelling in air is
incident on the plane boundary between air and The wave is propagating with a phase velocity
another dielectric medium with r = 4. The (a) 5 × 104 m/s (b) 3 × 108 m/s
reflection coefficient for electric field at normal (c) 1.25 × 107 m/s (d) 3 × 106 m/s
incidence, is [EC-2005 : 1 Mark]
(a) zero (b) 0.5 180°
Q.35 Refractive index of glass is 1.5. Find the
(c) 0.333 0° (d) 0.333 180°
wavelength of a beam of light with frequency of
[EC-2003 : 2 Marks]
1014 Hz in glass. Assume velocity of light is
Q.31 If the electric field intensity associated with a 3 × 108 m/s in vacuum.
uniform plane electromagnetic wave travelling (a) 3 µm (b) 3 mm
in a perfect dielectric medium is given by (c) 2 µm (d) 1 mm
E(z, t) = 10 cos(2 × 107t – 0.1 z) Volt/m [EC-2005 : 1 Mark]
GATE Previous Years Solved Paper 45

Q.36 The electric field on an electromagnetic wave 100


0
propagating in the positive z-direction is given (a) (b)
100 0
by
50
(c) 50 02 (d)
E = aˆ x sin ( t z) + aˆ y sin t z+ 0
2
[EC-2007 : 2 Marks]
The wave is
(a) linearly polarized in the z-direction. Q.40 A Right Circularly Polarized (RCP) plane wave
is incident at an angle of 60° to the normal, on
(b) elliptically polarized.
an air-dielectric interface. If the reflected wave
(c) left-hand circularly polarized.
is linearly polarized, the relative dielectric
(d) right-hand circularly polarized.
[EC-2006 : 1 Mark] constant r2 is

Q.37 A medium of relative permittivity r2 = 2 forms Linearly


polarized
an interface with free-space. A point source of
electromagnetic energy is located in the medium RCP

at a depth of 1 meter from the interface. Due to 60° 60°

Air =1
the total internal reflection, the transmitted beam r1

has a circular cross-section over the interface. Dielectric r2


The area of the beam cross-section at the
t
interface is given by
(a) 2 m2 (b) 2 m2

(c) m2 (d) m2
2 (a) 2 (b) 3
[EC-2006 : 2 Marks] (c) 2 (d) 3
Q.38 When a plane wave travelling in free-space is [EC-2007 : 2 Marks]
incident normally on a medium having r = 4.0,
Q.41 A plane wave of wavelength is travelling in a
the fraction of power transmitted into the
direction making an angle 30° with positive
medium is given by
x-axis and 90° with positive y-axis. The E field
8 1
(a) (b) of the plane wave can be represented as
9 2
(E0 is a constant)
1 5 3
(c) (d) j t x z
3 6 (a) E = yˆ E0 e
[EC-2006 : 2 Marks]
3
j t x z
(b) E = yˆ E0 e
Q.39 The H field (in A/m) of a plane wave
propagating in free space is given by 3
j t+ x+ z
(c) E = yˆ E0 e
5 3 5
H = xˆ cos( t z) + yˆ t z+
0 0 2 j t x+
3
z
The time average power flow density in watts is (d) E = yˆ E0 e
[EC-2007 : 1 Mark]
46 Electronics Engineering Electromagnetic Theory

Q.42 A uniform plane wave in the free space is Assuming the speed of light in free space to be
normally incident on an infinitely thick 3 × 108 m/s, intrinsic impedance of free space
dielectric slab (dielectric constant r = 9). The to be 120 , the relative permittivity r of the
magnitude of the reflection coefficient is medium and the electric field amplitude Ep are
(a) 0 (b) 0.3 (a) r = 3, Ep = 120 (b) r = 3, Ep = 360
(c) 0.5 (d) 0.8 (c) r = 9, Ep = 360 (d) r = 9, Ep = 120
[EC-2008 : 2 Marks] [EC-2011 : 2 Marks]

Q.43 The electric field component of a time harmonic Q.46 The electric field a uniform plane
plane EM wave travelling in a non-magnetic electromagnetic wave in free space, along the
lossless dielectric medium has an amplitude of positive x-direction, is given by
1 V/m. If the relative permittivity of the medium j 25x
E = 10( aˆ y + jaˆ z ) e
is 4, the magnitude of the time average power
density vector (in W/m2) is The frequency and polarization of the wave,
respectively are
1 1
(a) (b) (a) 1.2 GHz and left circular
30 60
(b) 4 GHz and left circular
1 1
(c) (d) (c) 1.2 GHz and right circular
120 240
(d) 4 Hz and right circular
[EC-2010 : 1 Mark]
[EC-2012 : 1 Mark]
Q.44 A plane wave having the electric field
Q.47 A plane wave propagating in air with
component, Ei = 24 cos(3 × 10 8 t y ) aˆ z V/m
t + 3x 4 y )
E = (8 aˆ x + 6 aˆ y + 5 aˆ z ) e j( V/m is
and travelling in free space is incident normally
on a lossless medium with µ = µ0 and = 9 0 incident on a perfectly conducting slab
which occupies the region y 0. The reflected positioned at x 0. The E field of the reflected
magnetic field component is given by
waves is
1
(a) cos(3 × 108 t + y ) aˆ x A/m (a) ( 8 aˆ x 6 aˆ y 5 aˆ z ) e j( t + 3x + 4 y )
V/m
10
t + 3x + 4 y )
1 (b) ( 8 aˆx + 6 aˆ y 5aˆ z ) e j( V/m
(b) cos(3 × 108 t + y ) aˆx A/m
20
(c) (8 aˆ x 6 aˆ y 5aˆ z ) e j( t 3x 4 y )
V/m
1
(c) cos(3 × 10 8 t + y ) aˆ x A/m
20 (d) (8 aˆ x + 6 aˆ y 5aˆ z ) e j( t 3x 4 y )
V/m
1
(d) cos(3 × 108 t + y ) aˆx A/m [EC-2012 : 1 Mark]
10
[EC-2010 : 2 Marks] Statement for Linked Answer Questions (48 and 49):
A monochromatic plane wave of wavelength = 600 µm
Q.45 The electric and magnetic fields for a TEM wave
is propagating in the direction as shown in the figure
of frequency 14 GHz in a homogeneous medium
of relative permittivity r and relative below. Ei , Er and Et denote incident, reflected, and
permeability µr = 1 are given by
transmitted electric field vectors associated with the
j( t 280 y ) wave.
E = Ep e uˆ z V/m

H = 3 e j( t 280 y )
uˆ x V/m
GATE Previous Years Solved Paper 47

Q.50 Which one of the following field patterns


Ei Er represents a TEM wave travelling in the positive
kr x-direction?
Hr
Hr (a) E = + 8 yˆ , H = 4 zˆ
ki

(b) E = 2 yˆ , H = 3zˆ
i r

r = 1.0
0
x (c) E = + 2 zˆ , H = + 2 yˆ
r = 4.5

(d) E = 3 yˆ , H = + 4 yˆ
19.2° Et
Ht [EC-2014 : 1 Mark]

Q.51 If the electric field of a plane wave is


kt
z E( z , t ) = xˆ 3 cos( t kz + 30°)

Q.48 The angle of incidence i and the expression for yˆ 4 sin( t kz + 45°) (mV/m)
Ei are The polarization state of the plane wave is
(a) left elliptical (b) left circular
× 10 4 ( x + z )
Eo j
(a) 60° and ( aˆx aˆ z ) e 3 2 V/m (c) right elliptical (d) right circular
2 [EC-2014 : 2 Marks]
× 10 4 z
Eo j Q.52 Assume that a plane wave in air with an electric
(b) 45° and ( aˆ x + aˆ z ) e 3 V/m
2
field, E = 10 cos( t 3x 3z) aˆ y V/m is
× 10 4 ( x + z )
Eo j
(c) 45° and ( aˆ x aˆ z ) e 3 2 V/m incident on a non-magnetic dielectric slab or
2 relative permittivity 3 which covers the region
× 10 4 z z > 0. The angle of transmission in the dielectric
Eo j
(d) 60° and ( aˆ x aˆ z ) e 3 V/m slab is _______ degrees.
2
[EC-2014 : 2 Marks]
[EC-2013 : 2 Marks]
Q.53 The electric field component of a plane wave
Q.49 The expression for Er is travelling in lossless dielectric medium is given
by
× 10 4 ( x z )
E j
(a) 0.23 o ( aˆ x + aˆ z ) e 3 2 V/m E( z , t ) = aˆ y 2 cos 108 t
z
V/m
2 2
× 10 4 z The wavelength (in m) for the wave is ______ .
Eo j
(b) ( aˆx + aˆ z ) e 3 V/m
2 [EC-2015 : 1 Mark]

× 10 4 ( x z ) Q.54 Consider a uniform plane wave with amplitude


Eo j
(c) 0.44 ( aˆ x + aˆ z ) e 3 V/m (E0) of 10 V/m and 1.1 GHz frequency travelling
2
an air, and incident normally on a dielectric
× 10 4 ( x + z ) medium with complex relative permittivity ( r)
Eo j
(d) ( aˆx + aˆ z ) e 3 V/m and permeability (µr) as shown in figure.
2
[EC-2013 : 2 Marks]
48 Electronics Engineering Electromagnetic Theory

(a) right-hand circular


Air Dielectric (b) right-hand elliptical
= 120 µr = 1 – j2 (c) right-hand elliptical
r = 1 – j2
(d) linear
[EC-2015 : 2 Marks]
E =?
10 cm Q.58 Let the electric field vector of a plane
electromagnetic wave propagating in a
homogenous medium be expressed as,
E0 = 10 V/m
j( t z)
Frequency = 1.1 GHz E = xˆ Ex e
where the propagation constant is function of
The magnitude of the transmitted electric field the angular frequency . Assume that, ( ) and
component (in V/m) after it has travelled a Ex are known and are real. From the information
distance of 10 cm inside the dielectric region is available, which one of the following cannot be
________ . determined?
[EC-2015 : 2 Marks] (a) The type of polarization of the wave.
Q.55 The electric field intensity of a plane wave (b) The group velocity of the wave.
travelling in free space is given by the following (c) The phase velocity of the wave.
expression: (d) The power flux through the z = 0 plane.

E( x , t ) = aˆ y 24 cos( t ko x ) V/m [EC-2016 : 1 Mark]

In this field, consider a square area 10 cm × 10 cm Q.59 If a right-handed circularly polarized wave is
on a plane x + y = 1. The total time averaged incident normally on a plane perfect conductor,
power (in MW) passing through the square area then the reflected wave will be
is _______ . (a) right-handed circularly polarized
[EC-2015 : 2 Marks] (b) left-handed circularly polarized
(c) elliptically polarized with a tilt angle of 45°
Q.56 The electric field of a uniform plane
(d) horizontally polarized
electromagnetic wave is
[EC-2016 : 1 Mark]
E = ( aˆx + j 2 aˆ y ) exp[ j(2 × 107 t 0.2 z)]
Q.60 Faraday’s law of electromagnetic induction is
The polarization of the wave is
mathematically described by which one of the
(a) right-handed circular following equations?
(b) right-handed elliptical
(a) B=0 (b) D= v
(c) left-handed circular
(d) left-handed elliptical B D
(c) ×E = (d) ×H = E+
t t
[EC-2015 : 1 Mark]
[EC-2016 : 1 Mark]
Q.57 The electric field of a plane wave propagating
Q.61 The expression for an electric field in free space
in a lossless non-magnetic medium is given by
is,
the following expression:
j( t kx + ky )
9 E = E0 ( xˆ + yˆ + j 2 zˆ ) e
E( z , t ) = aˆx 5 cos(2 × 10 t + z)
where, x, y, z represent the spatial coordinates,
9 t represents time, and , k are constants. The
+ aˆ y 3 cos 2 × 10 t + z
2 electric field
GATE Previous Years Solved Paper 49

(a) does not represent a plane wave. Q.64 The distance (in meters) a wave has to propagate
(b) represents a circularly polarized plane in a medium having a skin depth of 0.1 m so
wave propagating normal to the z-axis. that the amplitude of the wave attenuates by
(c) represents an elliptically polarized plane 20 dB, is
wave propagating along the x-y plane. (a) 0.12 (b) 0.23
(d) represents a linearly polarized plane wave. (c) 0.46 (d) 2.3
[EC-2017 : 2 Marks] [EC-2018 : 2 Marks]
Q.62 The permittivity of water at optical frequencies Q.65 The magnitude field of a uniform plane wave in
is 1.75 0. It is found that an isotropic light source vacuum is given by
at a distance ‘d’ under water forms an
illuminated circular area of radius 5 m as shown H ( x , y , z , t ) = ( aˆx + 2 aˆ y + baˆ z ) cos( t + 3x y z)
in the figure. The critical angle is c. The value of b is _______ .
[EC-2020 : 2 Marks]

5m Q.66 Consider a rectangular coordinate system


Air (x, y, z) with unit vectors ax, ay and az. A plane
wave traveling in the region z 0 with electric
field vector E = 10 cos(2 × 108t + z) ay is incident
normally on the plane at z = 0, where is the
Water c d phase constant. The region z 0 is in free space
and the region z < 0 is filled with a lossless
Light source medium (permittivity, = 0, permeability,
The value of ‘d’ (in meter) is _______ . µ = 4µ0, where 0 = 8.85 × 10 –12 F/m and
µ0 = 4 × 10–7 H/m). The value of the reflection
[EC-2017 : 2 Marks]
coefficient is,
Q.63 A uniform plane wave travelling in free space
3 2
and having the electric field, (a) (b)
5 3
E( 2 aˆ x aˆ z ) cos[6 3 × 108 t 2 ( x + 2 z)] V/m 2 1
(c) (d)
is incident on a dielectric medium (relative 5 3
permittivity > 1, relative permeability = 1) as [EC-2021 : 1 Mark]
shown in the figure and there is no reflected
Q.67 In a circuit, there is a series connection of an
wave.
ideal resistor and an ideal capacitor. The
z
conduction current (in Amperes) through the

Free space resistor is 2 sin t + . The displacement


Dielectric 2
medium ( r > 1)
x current (in Amperes) through the capacitor is
________ .
(a) 2 sin(t) (b) 2 sin(t + )

(c) 2 sin t + (d) 0


The relative permittivity (correct to two decimal 2
places) of the dielectric medium is ______ . [EC-2022]
[EC-2018 : 2 Marks]
50 Electronics Engineering Electromagnetic Theory

Electronics & Electrical Engineering


GATE Previous Years Solved Paper

A n swe rs & Expl a n a t i o n s

Answers
EC Uniform Plane Waves

1. (c) 2. (d) 3. (b) 4. (d) 5. (c, d) 6. (a) 7. (a, c) 8. (c)

9. (d) 10. (a) 11. (b) 12. (a) 13. (d) 14. (b) 15. (d) 16. (a)

17. (a) 18. (d) 19. (a) 20. (c) 21. (d) 22. (c) 23. (c) 24. (b)

25. (b) 26. (a) 27. (c) 28. (a) 29. (b) 30. (d) 31. (b) 32. (a)

33. (a) 34. (c) 35. (b) 36. (c) 37. (d) 38. (a) 39. (d) 40. (d)

41. (a) 42. (c) 43. (c) 44. (a) 45. (d) 46. (a) 47. (c) 48. (c)

49. (a) 50. (b) 51. (a) 52. (30) 53. (8.88) 54. (0.1) 55. (53.29) 56. (b)

57. (b) 58. (d) 59. (b) 60. (c) 61. (c) 62. (4.33) 63. (2) 64. (b)

65. (1) 66. (d) 67. (c)

Solutions
EC Uniform Plane Waves

1. (c) For a good conductor,


>>
1 n2 r2
= sin = sin 1 Et j µ µ
ej /4
c n1 r1 = =
Ht
For total internal reflection, So, Et leads Ht by an angle of 45°.
i c
3. (b)
1 r2
i
sin with r1 > r2 We know that, for a good conductor,
r1
1
For total internal reflection to take place, the skin depth = =

wave should move from a denser medium to a
1
rarer medium and the angle of incidence should
be greater than or equal to the critical angle. f

2 f1 1
2. (d) =
1 f2 2
= intrinsic impedance
Given that,
Et j µ
= = 1 = 1 micron
Ht +j
GATE Previous Years Solved Paper 51

f 1 = 3 GHz j µ
As, = 0 with being large
2 1
f 2 = 12 GHz =
1 9 when E reflection completely out of phase, H
reflects in phase,
2 3 9 9
= × =
1 12 1 4 Exr = Exi
9 H xr = H xi
2 = microns
4
So, Pr = Pi
4. (d)
8. (c)
Power density,
Two equal E field components out of phase by
E2 90° gives circular polarization,
P=
2 Ex = 2 cos t
E= 2 P Ey = 2 cos( t + 90°) = –2 sin t
= 0 = 120 Tracing with time for sense of rotation as left or
P = 1.2 kW/m2 = 1200 W/m2 right,
t = 0, Ex = 2, Ey = 0
E= 2 × (120 ) × (1200)
T
950 V/m t= , Ex = 0, Ey = –2
4
5. (c, d) Y
When E is a function of y and directed in x and z
possibly orthogonal to propagation,
E y = Hy = 0
2 2 t = T/2 t=0
Ex Ex X
2 = µ 2
Z
y t

E Ex2 + H z2 µ
= = t = T/4
H H x2 + H z2
Left circular for +z propagation.
6. (a)
Loss tangent, 10. (a)
6 A material behave as conductor if,
10
= 9 12
2 (10 × 10 ) (8.854 × 10 × 10)
>>
= 1.798 × 105 >> 1 >>
So, the given material at f = 10 GHz is For copper, conductivity,
considered as a good conductor. = 5.8 × 107 /m
= 8.856 × 10–12 F/m
7. (a, c) So, even at very large frequencies,
Reflection coefficient for electric fields, >>
So, copper always behave as conductor for
j µ
120 practical frequencies.
E = = 1 for conductors
j µ
+ 120
52 Electronics Engineering Electromagnetic Theory

11. (b) Comparing the real and imaginary part,


Reflection coefficient: 2
= 0.2 =
2
Given, = 1.5 2
1 = = 10 m
0.2
1.5 1 0.5 1
= = =
1.5 + 1 2.5 5 15. (d)
1 In a lossy dielectric, attenuation constant,
=
5 µ
=
Pr 1 2
1 2
= =
Pi 5 25 Skin depth,

Pi 1 2
Pr = × 100% = 4% of Pi = =
25 µ
So, the depth of penetration of a wave i a lossy
12. (a)
dielectric increases with increasing permittivity.

= 18 × 106 >> 1 16. (a)


Hence material is a good conductor. E = Eo e( j z)
( ax + ay )
1
Skin depth, = E = Eo e j( t z)
ax + Eo e j( t z)
ay

1 E = Ex ax + Ey ay
=
× 109 × 4 × 10 7 × 10 6 So, Ex = Eo ej( t – z)

4
10 Ey = Eo ej( t – z)
= = 15.9 µm
2
Ex = at any time t
13. (d) So, the polarization of the wave is linear.
We know that,
17. (a)
j µ
Intrinsic impedance = =
+j E = 24 e j( t + z) aˆ y V/m

For copper, Wave is travelling in –z-direction,


>> E = 24
j µ µ
= = e j 45° Direction of power = z = az

= +45° In free space,


So, the intrinsic impedance of copper at high = 0 = 120
frequency is complex with an inductive 1 2
Pavg = E ( aˆ z )
component. 2 0

14. (b) (24)2


Pavg = ( az )
Propagating constant, 2 (120 )
= + j = 0.1 + j0.2 2.4
Pavg = az W/m 2
GATE Previous Years Solved Paper 53

18. (d) 22. (c)


Reflection coefficient for electric fields,
y
j µ
120
E =
j µ
+ 120

x
j µ
As, = 0 with s being large
B ( t ) ax
when E reflection completely out of phase, H
reflects in phase,
Er – Ei = 0, Etang should be zero
d m d
V= = ( BA cos ) Hr + Hi = 2Ht, Htang should be maximum
dt dt
A = Area of the loop 23. (c)
with = angle between B and area of loop. 1
sin t
This changes due to rotation, =
sin i r
B(t) = cos t Harmonic
sin 30° 1
d cos dB =
V= AB A cos sin 45° r
dt dt
Rotation Harmonics
motional emf transfer action 1/2 1
=
1/ 2 r
19. (a)
r =2
According to Ampere’s law,
24. (b)
D
×H = J + 1
t v1 =
µ1 1
20. (c)
1
cos(y2 + 5t) does not satisfy the wave equation v2 =
µ2 2
due to non-linearity of phase with distance.
v1 2
21. (d) =
v2 1

j µ v =f
= ; =0 f = frequency of operation
+j
v
µ
= 0° 1 2
=
0 r2
=
r2
=
2 1 0 r1 r1

µ o µr µo µr
= = 25. (b)
o r o r
The E field is a function of z and directed in X.
2 377 i.e., harmonic function of z or travelling in z
= 377 = = 188.5
8 2 direction.
54 Electronics Engineering Electromagnetic Theory

26. (a) 31. (b)

E = j E = 2 × 107
= 0.1
=2 f o r
Comparing with A cos( t – z)
9 × 109 × 2 × 10 2
f = = 2 × 107
2 × o r 4 v = = = 2 × 108 m/s
0.1
f = 45 × 106 = 45 MHz
32. (a)
27. (c)
Displacement current,
E x and E y out of phase by 90°, unequal
Id = AJd
amplitudes. This is elliptical polarization.
D E
= A =A
28. (a) t t
V
Loss tangent = Id = A E =A
d
4 After putting values, we get,
1.7 × 10
= Id = 10 mA
2 × 3 × 10 9 × 78 o
33. (a)
1.7 × 10 4 × 9 × 109
= 1
3 × 109 × 39 Pavg = Re[E × H ]
= 0.13 × 10–4 = 1.3 × 10–5 2
1 k
= Re ( aˆ x + jaˆ y ) e jkz j t × ( aˆ y jaˆ x ) e jkz j t
29. (b) 2 µ

For conductors, k
Pavg = [0] = 0

2
=
µ 34. (c)
1 = 50,000; = 0.004
f
5 × 10 4
vp = =
2 f1 4 × 10 3
=
1 f2 = 1.25 × 107 m/s

1 MHz 35. (b)


2 = × 25 cm = 12.5 cm
4 MHz
2
Refractive index =
1
30. (d)
For glass with respective air = R
2 1
= 8 8
2 + 1 3 × 10 3 × 10
Vp = = = 2 µm
R×f 1.5 × 10 14
µo µo 1
1
=
o r o
= r
, =4 36. (c)
1 r
µo µo +1 Equal amplitudes and out of phase by 90°, wave
+
o r o r
is circularly polarized. Tracing the time
1 /2 1 advancing E on the z = 0 plane at t = 0, t = T/4,
= = 0.333 180°
3 /2 3 t = T/2 gives left/right notations.
GATE Previous Years Solved Paper 55

37. (d) 40. (d)


The reflected wave is linearly polarized when
circularly polarized wave is incident under
Air
Water
Brewster’s angle of incidence,
1m
r2
tan = = tan 60°
B
r1
(Total internal reflection beyond critical angle )
r2 =3
Circular cross-section beam for rays incident
below critical angle. 41. (a)
Calculating critical angle,
Wave
X propagation
o
sin c = z direction
2 o P1 P2
x

c = 45°
30°
1 m depth; 1 m radius 30°
Area = R2 =
Z
38. (a)
Pt = (1 – 2) Pi
µo µo Wave is travelling in the ZX-plane (90° to -axis)
2 1 o r o and 30° to X-axis,
= =
2 + 1 µo µo 2 3 3
+ = cos 30° = =
x
o r o 2
1
1 = sin 30° =
2 1 z
= 1 =
+1 3 The two solutions possible are shown as P1 and
2 P2 paths in the diagrams, options (a) is P2.
2 1 8
= 1 =1 = 42. (c)
9 9
Pt = Pi Dielectric constant of free space,
Pt 8 r1 =1
=
Pi 9 Dielectric constant of dielectric slab,

39. (d) r2 =9

For free space, Magnitude of reflection coefficient,

E2 r1 r2 1 3 2
P= ; E= 0H = = = = 0.5
2 0 r1 + r2 1+ 3 4
2 2 2
0H 0H
P= = 43. (c)
2 0 2
Magnitude of the time-average power density
1 10
H= (5 3)2 + (5)2 = vector,
0 0
100 50 E2
P=
0
× = Watts Pavg =
2 2 2
0 0
56 Electronics Engineering Electromagnetic Theory

µ 1
where, = v = f = 14 × 10 9 × m/sec
140
v = 1 × 108 m/sec
µo 120
= = 0 = C
4 2 2 But, v =
r µr
12 1 2
Pavg = = W/m
120 120 3 × 108
2 1 × 108 =
2 1× r

r =9
44. (a)
EP
=
Ei = 24 cos(3 × 108 t – y) aˆ z V/m HP
The wave is travelling in +y-direction, µ µ0 × 1 1 µ0
= = =
1 0 ×9 3 0
Hi = ( aˆ y × Ei )
EP 1
24 cos = × 120
3 3
= (3 × 108 t y ) aˆ x
120 EP = 120
1
= cos(3 × 108 t y ) aˆx 46. (a)
5
Out of phase by 90° and equal amplitude the
µ0 µ0 wave is circularly polarized.
Hr 1 2 1 2 Check the trace of E with the time which gives
= =
Hi 1+ 2 µ0 µ0 left circularly polarized,
+
1 2 a y = sin t
az = cos t
2 1 9 1
= = 47. (c)
2 + 1 9+ 1
3 1 2 1 Y Free space
= = =
3+1 4 2 X > 0 Reflected
X<0
Therefore, reflected magnetic field component, conductor
1
Hr = cos(3 × 108 t + y ) aˆ x 3a x + 4 a y
10
Due to change in propagation direction to –y. x

45. (d) i

j( t 280 y ) ˆ –3ax + 4ay


Given, E = Ep e U z V/m

H = 3 e j( t 280 y )
Uˆ x V/m Incidence

v = 3 × 108 m/s
From given expression we conclude that, Incident wave propagation 3aˆ x + 4 aˆ y
2
= 280 = Reflected wave propagation 3aˆ x + 4 aˆ y

1 Ei incident direction 8 aˆ x + 6 aˆ y + 5 aˆ z
= meter
140
GATE Previous Years Solved Paper 57

Ei tangential 6 aˆ y + 5 aˆ z The plane wave is travelling in + z-direction,


Etang = 0
Er tangential 6 aˆ y 5 aˆ z Eˆ x = 3 cos( t – kz + 30°)

Ei normal = 8 aˆ x Eˆ y = –4 sin( t – kz + 45°)


E = Emax
Er normal = 8 aˆx normal At z = 0, tracing E locus for various times.
Ei reflected direction = 8 aˆ x 6 aˆ y 5 aˆ z 1 3
At t = 0, Ex = 3 cos30° = 3 = = 1.5
2 2
48. (c) Ey = 4 sin 45° = 2 2 = 2.83
Ei propagates in ZX-plane.
T
Ei directed in ZX-plane. At t = ,
6
It is a p-plane of incidence polarized wave.
Using Snell’s law, 2 T
Ex = 3 cos × + 30° = 0
T 6
sin i 4.5 o
= ( i = 45°) Ey = –4 sin(60° + 45°) = –3.86
sin 19.2° o
Y

x = z = cos 45° =
2
(For propagation direction)
aˆ aˆ z X
For field direction = x (45°) Z
2 at t = 0 (Ex = 1.5, Ey = –2.83)

49. (a)
at t = T/6 (Ex = 0, Ey = –3.86)
For reflected wave:
Er propagation direction changes without This is left elliptical polarized plane-wave.
change in angle 45°.
aˆx aˆ z 52. Sol.
Er propagation direction =
2 X
reflection coefficient,
= –0.23 Air Z < 0
Z>0 R = 3
Er field direction suitably changes satisfying,
E P=0 Pr

50. (b) z
t
Z
Since the TEM wave or plane wave is travelling i

in positive x-direction, then conditions x

should be
Pi
P = E× H
Wave
ˆax = aˆ y × aˆ z incidence

or, aˆx = ( aˆ y ) × ( aˆ z )
So, the option (b) is satisfies the condition. x = 3, z = 3 from Ei of the wave

i = 60°
51. (a)
sin 60°
= 3
E( z , t ) = 3xˆ cos( t kz + 30°) sin t
4 yˆ sin ( t kz + 45°) t = 30°
58 Electronics Engineering Electromagnetic Theory

53. Sol.
Pavg = P ds
8 z
E( z , t ) = aˆ y 2 cos 10 t V/m 1 (24 )2 aˆx + aˆ y 2
2 = aˆ x × 10
2 120 2
General form of
24
E( z , t ) = aˆ y Eo cos( t z) V/m Pavg = = 53.29 mW
2
Comparing both, we get,
1 57. (b)
=
2 9
E ( z , t ) = aˆx 5 cos(2 × 10 t + z)
2
Since, = + aˆ y 3 cos 2 × 109 t + z
2
2 2
= = =2 2 m • Wave is travelling in aˆ z direction.
1/ 2
= 8.88 m • Wave has orthogonal components with
unequal amplitudes and checking the time
54. Sol. trace.
Material has µR and R complex, Wave is left hand elliptically polarized.
of material =
j µ µoµR 58. (d)
= = 120
+j o R
( ) is known so Vg can be calculated.
where, = 0 (dielectric) vp = / can be calculated.
No reflections at the boundary, Polarization can be identified.
µr and r cannot be found, due to which power
= Propagation constant
flux cannot be calculated as power flux,
= j µ oµ R (0 + j o R) 2
1 E
P=
= j µo R (1 j 2)2 2

µo +2 µo µr
= j o o where, = 120 ×
r
= 2 µo o = 46.07 Np/m
59. (b)
Magnitude of transmitted electric field
Left circularly polarized, due to direction change
component,
after the reflection from conductor.
E = Eo e– z with Z = 0.1 m
E = 0.1 V/m 60. (c)
Rate of change of magnetic field results in
55. Sol. induced voltage,

E( x , t ) = aˆ y 24 cos( t K o x ) V/m B
×E =
t
Power density,
2 2 61. (c)
1 E 1 (24 )
P = 2 = aˆ x
2 120 E field direction ( aˆ x + aˆ y + j 2 aˆ z )
Time-averaged power passing through square Propagation direction k aˆ x k aˆ y
area,
GATE Previous Years Solved Paper 59

E is perpendicular to propagation, Eo
20 log 10 = 20 dB
E P =0 Ex

Component in aˆ z has magnitude of 2. Eo Eo


= 10 Ex =
Ex 10
Component in X-Y plane has magnitude of 2.
x 10 x Eo
These two components are out of phase by 90° Ex = Eo e = Eo e =
10
and have unequal amplitudes. So, it is
1
elliptically polarized wave. e–10x =
10
62. Sol. 1
x = ln(0) = 0.23 m
Critical angle, 10

1 2 1 0 65. Sol.
c = sin = sin = 49.1°
1 1.75 0 For uniform plane wave,
5m aˆ H aˆ = 0

aˆ H is unit vector in magnetic field direction â


is unit vector in power flow direction,

d c 1 aˆx + 2 aˆ y + b aˆ z
aˆ H =
12 + 2 2 + b 2
3 aˆ x + aˆ y + aˆ z
5 â =
tan 49.1 = 1.1547 =
d 3 2 + 1 2 + 12
5 aˆ H aˆ = 0
d = = 4.33 m
1.1547
( aˆ x + 2 aˆ y + baˆ z ) ( 3 aˆx + aˆ y + aˆ z ) = 0
63. Sol. –3 + 2 + b = 0
The wave has Ei direction and propagation b=1
direction both in same plane (ZX).
66. (d)
The wave is plane of incidence (P) polarized,
Reflecting
x = 1, z = 2
surface
z
tan i = = 2 Medium 2 Medium 1
x lossless medium free space
No reflection in P polarized wave under 2 = 0, µ 2 = 4µ0 1 = 2, µ 1 = µ0, 1 = 0 = 120

Brewster’s angle of incidence,


i = B Normal incidence
tan B = 2= r

r =2

64. (b) Z < 0, Z = 0, Z > 0


Attenuation constant,
µr 2 4
1 2 = 120 = 120 = 2 × 120
= = 10 Np/m r2 1
skin depth
60 Electronics Engineering Electromagnetic Theory

Reflection coefficient, is given by 67. (c)


2 1 ic R
=
2+ 1
id
2 × 120 120 C
=
2 × 120 + 120
(2 1)120
= In series connection, current through each
(2 + 1)120
elements remain same.
1
= Hence, ic = i d
3
So, i d = 2 sin t +
2
3 Transmission Lines

ELECTRO NICS EN GINEERIN G Q.5 A transmission line whose characteristic


impedance is a real
(GATE Previous Years Solved Papers)
(a) must be a lossless line.
Q.1 A transmission line of real characteristic (b) must be a distortionless line.
impedance is terminated with an unknown (c) may not be a lossless line.
load. The measured value of VSWR on the line (d) may not be a distortionless line.
is equal to 2 and a voltage minimum point is [EC-1992 : 2 Marks]
found to be at the load. The load impedance is
Q.6 Consider a transmission line of characteristic
then
impedance 50 . Let it be terminated at one end
(a) complex
by (+j50) . The VSWR produced by it in the
(b) purely capacitive transmission line will be
(c) purely resistive (a) +1 (b) 0
(d) purely inductive (c) (d) +j
[EC-1987 : 2 Marks] [EC-1993 : 2 Marks]
Q.2 A two-wire transmission line of characteristic Q.7 A load impedance, (200 + j0) is to be matched
impedance Z 0 is connected to a load of to a 50 lossless transmission line by using a
impedance ZL(ZL Z0). Impedance matching quarter wave line transformer (QWT). The
cannot be achieved with characteristic impedance of the QWT required
(a) a quarter-wavelength transformer is _______ .
(b) a half-wavelength transformer [EC-1994 : 1 Mark]
(c) an open-circuited parallel stub Q.8 If a pure resistance load, when connected to a
(d) a short-circuited parallel stub lossless 75 line, produces a VSWR of 3 on the
[EC-1988 : 2 Marks] line, then the load impedance can only be 25 .
True/False (Given reason)
Q.3 A 50 lossless transmission line has a pure
[EC-1994 : 2 Marks]
reactance of (j100) as its load. The VSWR in
the line is Q.9 A lossless transmission line having 50
(a) 1/2 (Half) (b) 2 (Two) characteristic impedance and length /4 is
(c) 4 (Four) (d) (Infinity) short-circuited at one end and connected to an
ideal voltage source of 1 V at the other end. The
[EC-1989 : 2 Marks]
current drawn from the voltage sources is
Q.4 The input impedance of a short-circuited (a) 0 (b) 0.02 A
lossless transmission line quarter wave long is (c) (d) None of these
(a) purely reactive [EC-1996 : 1 Mark]
(b) purely resistive
Q.10 The capacitance per unit length and the
(c) infinite
characteristic impedance of a lossless
(d) dependent on the characteristic impedance transmission line are C and Z0 respectively. The
of the line velocity of a travelling wave on the transmission
[EC-1991 : 2 Marks] line is
62 Electronics Engineering Electromagnetic Theory

1 Q.15 The magnitude of the open-circuit and short-


(a) Z0 C (b) circuit input impedance of a transmission line
(Z0C )
C are 100 and 25 respectively. The
Z0
(c) (d) characteristic impedance of the line is,
C Z0
(a) 25 (b) 50
[EC-1996 : 1 Mark]
(c) 75 (d) 100
Q.11 A transmission line of 50 characteristic [EC-2000 : 1 Mark]
impedance is terminated with a 100
resistance. The minimum impedance measured Q.16 A transmission line is distortionless if
on the line is equal to 1
(a) RL = (b) RL = GC
(a) 0 (b) 25 GC
(c) 50 (d) 100 (c) LG = RC (d) RG = LC
[EC-1997 : 1 Mark] [EC-2001 : 1 Mark]
Q.12 All transmission line section in figure, have a Q.17 A uniform plane electromagnetic wave incident
characteristic impedance R0 + j0. The input normally on a plane surface of a dielectric
impedance Zin equals, material is reflected with a VSWR of 3. What is
/8 /2 the percentage of incident power that is
reflected?
Zin 2R0 (a) 10% (b) 25%
(c) 50% (d) 75%
[EC-2001 : 2 Marks]
R0/2 Q.18 The VSWR can have any value between
(a) 0 and 1 (b) –1 and +1
/4
(c) 0 and (d) 1 and
2 [EC-2002 : 1 Mark]
(a) R0 (b) R 0
3
3 Q.19 In an impedance Smith chart, a clockwise
(c) R0 (d) 2R 0 movement along a constant resistance circle
2
[EC-1998 : 1 Mark] gives rise to
(a) a decrease in the value of reactance.
Q.13 In a twin-wire transmission line in air, the
adjacent voltage maxima are at 12.5 cm and (b) an increase in the value of reactance.
27.5 cm. The operating frequency is (c) no change in the reactance value.
(a) 300 MHz (b) 1 GHz (d) no change in the impedance value.
(c) 2 GHz (d) 6.28 GHz [EC-2002 : 1 Mark]
[EC-1999 : 2 Marks]
Q.20 A short-circuited stub is shunt connected to a
Q.14 In air, a lossless transmission line of length transmission line as shown in the figure. If
50 cm with L = 10 µH/m, C = 40 pF/m is operated Z0 = 50 , the admittance Y seen at the junction
at 25 MHz. Its electrical path length is of the stub and the transmission line is
(a) 0.5 meters (b) meters
(c) /2 meters (d) 180 degrees
[EC-1999 : 2 Marks]
GATE Previous Years Solved Paper 63

(a) adding an inductance in series with Z


(b) adding an capacitance in series with Z
/8 (c) adding an inductance in shunt across Z
(d) adding a capacitance in shunt across Z
[EC-2004 : 2 Marks]

Z0 Q.23 A plane electromagnetic wave propagating in


ZL = 100 free space is incident normally on a large slab
Z0 Z0 of lossless, non-magnetic, dielectric material
with > 0. Maxima and minima are observed
/2
when the electric field is measured in front of
(a) (0.01 – j0.02) (b) (0.02 – j0.01) the slab. The maximum electric field is found to
(c) (0.04 – j0.02) (d) (0.02 + j0) be 5 times the minimum field. The intrinsic
impedance of the medium should be
[EC-2003 : 2 Marks]
(a) 120 (b) 60
Q.21 Consider a 300 , quarter wave long (at 1 GHz) (c) 600 (d) 24
transmission line as shown in figure. It is [EC-2004 : 2 Marks]
connected to a 10 V, 50 source at one end and
is left open-circuited at the other end. The Q.24 A lossless transmission line is terminated in a
magnitude of the voltage at the open-circuit end load which reflects a part of the incident power.
The measured VSWR is 2. The percentage of the
of the line is,
power that is reflected back is
(a) 57.73 (b) 33.33
10 V, 50 (c) 0.11 (d) 11.11
Z0 = 300
source [EC-2004 : 2 Marks]

Q.25 Characteristic impedance of a transmission line


/4 is 50 . Input impedance of the open-circuited
(a) 10 V (b) 5 V line is Z OC = 100 + j150 . When the
transmission line is short-circuited, then value
(c) 60 V (d) 60/7 V
of the input impedance will be
[EC-2004 : 2 Marks]
(a) 50 (b) 100 + j150
Q.22 Consider an impedance Z = R + jX marked with (c) 7.69 + j11.54 (d) 7.69 – j11.54
point P in an impedance Smith chart as shown [EC-2005 : 2 Marks]
in the figure. The movement from point P along
a constant resistance circle in the clockwise Common Data for Questions (26 and 27):
Voltage standing wave pattern in a lossless
direction by an angle 45° is equivalent to
transmission line with characteristic impedance 50
r= and a resistive load is shown in the figure.
0.5
V (z )

x=0
4

1
P
x = –0.5 z /2
x = –1
Load
64 Electronics Engineering Electromagnetic Theory

Q.26 The value of the load resistance is Q.31 The parallel branches of a 2-wire transmission
(a) 50 (b) 200 line are terminated in 100 and 200 resistors
(c) 12.5 (d) 0 as shown in the figure. The characteristic
impedance of the line is Z0 = 50 and each
[EC-2005 : 2 Marks]
section has a length of /4. The voltage
Q.27 The reflection coefficient is given by reflection coefficient at the input is
(a) –0.6 (b) –1
(c) 0.6 (d) 0 /4
[EC-2005 : 2 Marks] /4 0
=5
Q.28 Many circles are drawn in a Smith chart used Z0 100
for transmission line calculations. The circles
shown in the figure represent
Z0 = 50

Z 200
0 =5
0
/4

7 5
(a) j (b)
(a) unit circles 5 7
(b) constant resistance circles 5 5
(c) j (d)
(c) constant reactance circles 7 7
[EC-2007 : 2 Marks]
(d) constant reflection coefficient circles
[EC-2005 : 2 Marks] Q.32 A load of 50 is connected in shunt in a 2-wire
transmission line of Z0 = 50 as shown in the
Common Data for Questions (29 and 30):
figure. The 2-port scattering parameter matrix
A 30 Volts a battery with zero source resistance is
(S-matrix) of the shunt element is
connected to a co-axial line of characteristic impedance
of 50 at t = 0 seconds and terminated in an unknown
resistive load. The line length is such that it takes 400 µs
for an electromagnetic wave to travel from source end Z0 = 50 50 Z0 = 50

to load end and vice-versa. At t = 400 µs, the voltage at


the load end is found to be 40 Volts.

Q.29 The load resistance is 1 1


(a) 25 (b) 50 2 2 0 1
(a) (b)
(c) 75 (d) 100 1 1 1 0
2 2
[EC-2006 : 2 Marks]

Q.30 The steady-state current through the load 1 2 1 3


resistance is 3 3 4 4
(c) (d)
2 1 3 1
(a) 1.2 Amps. (b) 0.3 Amps.
3 3 4 4
(c) 0.6 Amps. (d) 0.4 Amps.
[EC-2007 : 2 Marks]
[EC-2006 : 2 Marks]
GATE Previous Years Solved Paper 65

Q.33 In the design of a single mode step index optical Q.37 In the circuit shown, all the transmission line
fiber close to upper cut-off, the single-mode sections are lossless. The Voltage Standing
operation is not preserved if Wave Ratio (VSWR) on the 60 line is
(a) radius as well as operating wavelength are Short
halved. Z0 = 30
/8
(b) radius as well as operating wavelength are
doubled.
(c) radius is halved and operating wavelength Z0 = 60 Z0 = 30 2 ZL = 30
is doubled.
(d) radius is doubled and operating /4
wavelength is halved.
(a) 1.00 (b) 1.64
[EC-2008 : 2 Marks]
(c) 2.50 (d) 3.00
Q.34 One end of a lossless transmission line having [EC-2010 : 2 Marks]
the characteristic impedance of 75 and length
of 1 cm is short-circuited. At 3 GHz, the input Q.38 If the scattering matrix [S] of a two-port network
impedance at the other end of the transmission is
line is 0.2 0° 0.9 90°
[S ] =
(a) 0 (b) resistive 0.9 90° 0.1 90°
(c) capacitive (d) inductive then the network is
[EC-2008 : 2 Marks] (a) lossless and reciprocal
Q.35 A transmission line terminates in two branches, (b) lossless but not reciprocal
each of length /4, as shown. The branches are (c) not lossless but reciprocal
terminated by 50 loads. The lines are lossless (d) neither lossless nor reciprocal
and have the characteristic impedances shown.
[EC-2010 : 1 Mark]
Determine the impedance Zi as seen by the
source. Q.39 A transmission line of characteristic impedance
50 is terminated by a 50 load. When excited
/4 Z L = 50 by a sinusoidal voltage source at 10 GHz, the
/4 00
=1 phase difference between two points spaced
Z0
2 mm apart on the line is found to be /4 radians.
Z0 = 50
The phase velocity of the wave along the line is
Z
0 =1
00 (a) 0.8 × 108 m/s (b) 1.2 × 108 m/s
Zi
/4 Z L = 50 (c) 1.6 × 108 m/s (d) 3 × 108 m/s
[EC-2011 : 1 Mark]
(a) 200 (b) 100
(c) 50 (d) 25 Q.40 A transmission line of characteristic impedance
[EC-2009 : 2 Marks] 50 is terminated in a load impedance ZL. The
VSWR of the line is measured as 5 and the first of
Q.36 A transmission line has a characteristic
the voltage maxima in the line is observed at a
impedance of 50 and a resistance of 0.1 /m.
If the line is distortionless, the attenuation distance of /4 from the load. The value of ZL is
constant (in Np/m) is (a) 10
(a) 500 (b) 5 (b) 250
(c) 0.014 (d) 0.002 (c) (19.23 + j46.15)
[EC-2010 : 1 Mark] (d) (19.23 – j46.15) [EC-2011 : 2 Marks]
66 Electronics Engineering Electromagnetic Theory

Q.41 A coaxial cable with an inner diameter of 1 mm Q.46 To maximize power transfer, a lossless
and outer diameter of 2.4 mm is filled with a transmission line is to be matched to a resistive
dielectric of relative permittivity 10.89. Given, load impedance via a /4 transformer as shown.

10 9 Lossless transmission line


µ0 = 4 × 10 –7 H/m, 0 = F/m, the /4 transformer
36
characteristic impedance of the cable as, Zin = 50 ZL = 100
(a) 330 (b) 100
(c) 143.3 (d) 43.4
[EC-2012 : 1 Mark]
The characteristic impedance (in ) of the /4
Q.42 A transmission line with a characteristic transformer is ______ .
impedance of 100 is used to match a 50 [EC-2014 : 1 Mark]
section to a 200 section. If the matching is to
Q.47 In the transmission line shown, the impedance
be done both at 429 MHz and 1 GHz, the length
Zin (in ) between node A and the ground is
of the transmission line can be approximately
Z0 = 50 , L = 0.5
(a) 82.5 cm (b) 1.05 m A
(c) 1.58 m (d) 1.75 m
[EC-2012 : 2 Marks] 100 50
Zin = ?
Q.43 The return loss of a device is found to be 20 dB.
The voltage standing was ration(VSWR) and
magnitude of reflection coefficient are [EC-2014 : 2 Marks]
respectively Q.48 In the following figure, the transmitter Tx sends
(a) 1.22 and 0.1 (b) 0.81 and 0.1 a wideband modulated RF signal via a coaxial
(c) –1.22 and 0.1 (d) 2.44 and 0.2 cable to the receiver Rx. The output impedance
[EC-2013 : 1 Mark] ZT of Tx, the characteristic impedance Z0 of the
cable and the input impedance ZR of Rx are all
Q.44 For a parallel plate transmission line, let v be real.
the speed of propagation and z be the
Transmitter Characteristic impedance = Z0 Receiver
characteristic impedance. Neglecting fringe
effects, a reduction of the spacing between the
plates by a factor of two results in ZT ZR

(a) having of v and no change in z. TX RX


(b) no changes in v and having of z.
(c) no change in both v and z. Which one of the following statements is true
(d) halving of both v and z. about the distortion of the received signal due
[EC-2014 : 2 Marks] to impedance mismatch?
(a) The signal gets distorted if Z R Z0
Q.45 The input impedance of /8 section of a lossless
irrespective of the value of ZT.
transmission line of characteristic impedance
50 is found to be real when the other end is (b) The signal gets distorted if Z T Z0,
terminated by a load ZL = (R + jX) . If X is 30 , irrespective of the value of ZR.
the value of R (in ) is ______ . (c) Signal distortion implies impedance
[EC-2014 : 2 Marks] mismatch at both ends, ZT Z0 and ZR Z0.
GATE Previous Years Solved Paper 67

(d) Impedance mismatches do not result in L z/2 L z/2


signal distortion but reduce power transfer
efficiency.
[EC-2014 : 1 Mark] (c) G z C z

Q.49 A two-port network has scattering parameters


given by
z
s11 s12
[S ] = R z L z
s21 s22
If the port-2 of the two-port is short circuited,
the s11 parameter for the resultant one-port
(d) G z C z
network is
s11 s11s22 + s12 s21
(a)
1 + s22 z

s11 + s11s22 s12 s21 [EC-2015 : 1 Mark]


(b)
1 + s22
Q.51 Consider the 3 m long lossless air-filled
s11 + s11s22 + s12 s21 transmission line shown in the figure. It has a
(c)
1 s22 characteristic impedance of 120 , is
terminated by a short-circuit, and is excited with
s11 s11s22 + s12 s21
(d) a frequency of 37.5 MHz. What is the nature of
1 s22
the input impedance (Zin) ?
[EC-2014 : 1 Mark]

Q.50 A co-axial cable is made of two brass conductors.


The spacing between the conductors is filled ZL = 0
with Teflon ( r = 2.1, tan = 0). Which of the Zin
following circuits can represent the lumped
3m
element model of a small piece of this cable
having length z? (a) Open (b) Short

R z/2 L z/2 R z/2 L z/2


(c) Inductive (d) Capacitive
[EC-2015 : 2 Marks]

Q.52 A coaxial capacitor of inner radius 1 mm and


(a) G z C z outer radius 5 mm has a capacitance a per unit
length of 172 pF/m. If the ratio of outer radius
to inner is doubled, the capacitance per unit
z length (in pF/m) is ______ .
R z/2 L z/2 R z/2 L z/2 [EC-2015 : 2 Marks]

Q.53 A 200 m long transmission line having


parameters shown in the figure is terminated
(b) C z into a load RL. The line is connected to a 400 V
source having source Rs through a switch which
is closed at t = 0. The transient response of the
z circuit at the input of the line (z = 0) is also drawn
in the figure. The value of RL (in ) is _____ .
68 Electronics Engineering Electromagnetic Theory

Which one of the following inequalities is


always satisfied?
R0 = 50
Rs = 150 V r,eff = 2.25 Lt Lt
RL (a) Zo > (b) Zo <
o rw o rw
Vs = 400 V
Lw Lw
200 m
(c) Zo > (d) Zo <
Z=0 Z=L o rt o rt

V(0, t) [EC-2016 : 2 Marks]


100 V
Q.56 A microwave circuit consisting of lossless
transmission lines T1 and T2 is shown in the
62.5 V figure. The plot shows the magnitude of the
input reflection coefficient as a function f. The
phase velocity of the signal in the transmission
t(µs) lines is 2 × 108 m/s.
2.0
[EC-2015 : 2 Marks] T1 Length = 1 m T2 Length = L
Input
Z0 = 50 Z0 = 50
Q.54 The propagation constant of a lossy
Open
transmission line is (2 + j5) m –1 and its 50
characteristic impedance is (50 + j0) at
6 –1
= 10 rad s . The value of the line constant
L, C, R, G are respectively
(a) L = 200 µH/m, C = 0.1 µF/m, R = 50 /m,
G = 0.02 S/m 1
(reflection coefficient)

(b) L = 250 µH/m, C = 0.1 µF/m, 0.8


R = 100 /m, G = 0.04 S/m
0.6
(c) L = 200 µH/m, C = 0.2 µF/m,
0.4
R = 100 /m, G = 0.02 S/m
(d) L = 250 µH/m, C = 0.2 µF/m, R = 50 /m, 0.2

G = 0.04 S/m
0 0.5 1 1.5 2 2.5 f (in GHz)
[EC-2016 : 1 Mark]
The length L (in meters) of T2 is ______ .
Q.55 A lossless microstrip transmission line consists
of a trace of width w. It is drawn over a practically [EC-2016 : 2 Marks]
infinite ground plane and is separated by a Q.57 Light from free space is incident at an angle i
dielectric slab of thickness t and relative to the normal of the face of a step-index large
permittivity r > 1. The inductance per unit core optical fibre. The core and cladding
length and the characteristic impedance of this refractive indices are n1 = 1.5 and n2 = 1.4,
line are L and Z0, respectively. respectively.
W
= 0
n2 (cladding)
Free space
= 0 r, r >1 t n1 (core)
i

Light
GATE Previous Years Solved Paper 69

The maximum value of i (in degrees) for which (a) P : Open-circuit, Q : Short-circuit,
the incident light will be guided in the core of R : Matched load
the fibre is ______ . (b) P : Open-circuit, Q : Matched load,
[EC-2016 : 1 Mark] R : Short-circuit
(c) P : Short circuit, Q : Matched load,
Q.58 The voltage of an electromagnetic wave
R : Open-circuit
propagating in a coaxial cable with uniform
characteristic impedance is V(l) = e– l + j t Volts, (d) P : Short circuit, Q : Open-circuit,
where l is the distance along the length of the R : Matched load
cable in meters, = (0.1 + j40) m–1 is the complex [EC-2018 : 1 Mark]
propagation constant, and = 2 × 106 rad/s is
Q.62 A lossy transmission line has resistance per unit
the angular frequency. The absolute value of the
length R = 0.05 /m. The line is distortionless
attenuation in the cable (in dB/meter) is ____ .
and has characteristic impedance of 50 . The
[EC-2017 : 1 Mark]
attenuation constant (in Np/m correct to three
Q.59 A two-wire transmission line terminates in a decimal places) of the line is ______ .
television set. The VSWR measured on the line [EC-2018 : 1 Mark]
is 5.8. The percentage of power that is reflected
from the television set is _______ . Q.63 A transmission line of length 3 /4 and having
[EC-2017 : 1 Mark] a characteristic impedance of 50 is terminated
with a load of 400 . The impedance (rounded
Q.60 An optical fibre is kept along the zˆ -direction. off to two decimal places) seen at the input end
The refractive indices for the electric fields along of the transmission line is ______ .

x̂ and yˆ -directions in the fibre are nx = 1.5000 [EC-2020 : 1 Mark]

and ny = 1.5001, respectively (nx ny due to the Q.64 The impedances Z = jX for all X in the range
imperfection in the fibre cross-section). The free (– , ), map to the Smith chart as,
space wavelength of a light wave propagating (a) a circle of radius 0.5 with center at (0.5, 0).
in the fibre is 1.5 µm. If the light wave is (b) a point at the center of the chart.
circularly polarized at the input of the fibre, the (c) a line passing through the center of the
minimum propagation distance after which is chart.
becomes linearly polarized (in centimeters), is
(d) a circle of radius 1 with center at (0, 0).
________ .
[EC-2020 : 1 Mark]
[EC-2017 : 2 Marks]
Q.65 The impedance matching network shown in the
Q.61 The points P, Q and R shows on the Smith chart
figure is to match a lossless line having
(normalized impedance chart) in the following
characteristic impedance Z0 = 50 with a load
figure represent:
impedance ZL. A quarter wave line having a
characteristic impedance Z1 = 75 is connected
to Z L . Two stubs having characteristic
impedance of 75 each are connected to this
P R quarter wave line. One is a short-circuited (S.C.)
Q
stub of length of 0.25 connected across PQ and
the other one is an open-circuit (O.C.) stub of
length 0.5 connected across RS.
70 Electronics Engineering Electromagnetic Theory

The impedance matching is achieved when the


0.2
P R real part of ZL is
0.25 (a) 50.0 (b) 33.3
Z1 = 75
Z0 = 50 ZL (c) 75.0 (d) 112.5
[EC-2021 : 2 Marks]

ub
O.C 75
S.C = 75
ub

. st
1 =
. st
1
Z

0.5

Electronics & Electrical Engineering


GATE Previous Years Solved Paper

A n swe rs & Expl a n a t i o n s

Answers
EC Transmission Lines

1. (c) 2. (b) 3. (d) 4. (c) 5. (b, c) 6. (c) 7. (100) 8. (False)

9. (a) 10. (b) 11. (b) 12. (b) 13. (b) 14. (c) 15. (b) 16. (c)

17. (b) 18. (d) 19. (b) 20. (a) 21. (c) 22. (a) 23. () 24. (d)

25. (d) 26. (c) 27. (a) 28. (b) 29. (d) 30. (b) 31. (d) 32. (b)

33. (d) 34. (d) 35. (d) 36. (d) 37. (b) 38. (c) 39. (c) 40. (a)

41. (*) 42. (c) 43. (a) 44. (b) 45. (40) 46. (70.72) 47. (33.3) 48. (c)

49. (b) 50. (b) 51. (d) 52. (120.22) 53. (30) 54. (b) 55. (b) 56. (0.1)

57. (Sol.) 58. (0.868) 59. (49.83) 60. (Sol.) 61. (c) 62. (0.001) 63. (6.25) 64. (d)

65. (d)
GATE Previous Years Solved Paper 71

Solutions
EC Transmission Lines

1. (c) For a lossless line,


Given, voltage minimum (Vmin) point is at load. R = 0, G=0
If Vmin or Vmax occurs at the load for a lossless =0
transmission line then load impedance ZL is = LC
purely resistive. L
Zo = Ro = (real)
If ZL (resistive) > Zo, voltage maxima occurs at c
the load. Zo is real means either lossless or distortionless
If ZL (resistive) < Zo, voltage minima occurs at and lossless lines also satisfies LG = RC which
the load. means they are distortionless.

2. (b) 6. (c)
/2 length transmission line has Zin = ZL and ZL Zo
hence cannot be a impedance matching device. Reflection coefficient = =
ZL + Zo
3. (d) j 50 50 50 + j 50
= =
ZL Z0 j 50 + 50 50 + j 50
Reflection coefficient = =
ZL + Z0
(50)2 + (50)2
j 100 50 = =1
= (50)2 + (50)2
j 100 + 50
1+ 1+1 2
2 2 VSWR = = = =
(100) + (50) 1 1 1 0
= =1
(100)2 + (50)2
7. Sol.
1+ 1+1 2 For quarter wave line transformer,
VSWR = = = =
1 1 1 0
Z02 = Zin ZL
4. (c) Z02 = (50) (200)
For a quarter wave transformer ( /4), Z 0 = 100
Z02
Zin = 8. Sol.
ZL
(False)
ZL = 0
For a pure resistive load of a lossless line,
Z02 RL
Zin = = VSWR = if RL > R0
0 R0

5. (b, c) R0
VSWR = if R0 > RL
For a distortionless line,
RL
RC = LG Given that, VSWR = 3
So, RL = VSWR × R0
= RG
= 3 × 75 = 225
= LC R0 75
or, RL = = = 25
L VSWR 3
Zo = Ro = (real)
c Hence, RL = 25 or 75
72 Electronics Engineering Electromagnetic Theory

9. (a) R0 + jR0
Zin = R0 = R0
For quarter wave transformer ( /4), R0 + jR0
Z02
Zin = 13. (b)
ZL
Adjacent volt maximum are at /2 distance.
ZL = 0
27.5 – 12.5 =
Z02 2
Zin = = = open-circuit
Z0 = 30 cm = 0.3 m

Vs Vs c 3 × 108
Is = = =0 f= = = 1 GHz
Zin 0.3

14. (c)
10. (b)
Electrical path length = l deg or rad,
L 1 1
Z0 = =
C v =
LC 10 × 10 × 40 × 10 12
6
L = 0.5 × 108
Z02 =
C
v 0.5 × 108
L= Z02C = =
f 25 × 106
1 1 1 2 2 × 0.5 × 25 × 10 6
v = = = ×l =
LC ( Z02C ) (C ) Z0C l=
0.5 × 10 8

11. (b) = rad.


2
RL = 100 ; R0 = 50
100 15. (b)
SWR = =2
50 Zo = ZOC ZSC
Zo 50
Zmin = = = 25 = 100 × 25
SWR 2
Zo = 10 × 5 = 50
12. (b)
17. (b)
For /4 line,
1+
2
Z01 R02 s=
= = 2 R0 1
Zin1 =
ZL1 R0 /2
1+
For /2 line, 3 =
1
Zin 2 = ZL2 = 2 R0 = 0.5
For /8 line, Pr 2
= = 0.25
ZL = (Zin 1 ) ( Zin 2 ) Pi
25% of incident power is reflected.
ZL = (2 R0 ) (2 R0 )
ZL = R 0 19. (b)
Moving along clockwise direction is an
ZL + jZ0
For /8 line, Zin = Z0 inductive increase in the load impedance.
Z0 + jZL
GATE Previous Years Solved Paper 73

20. (a) 22. (a)


Movement on constant resistance circle by an
45° in clockwise direction.
/8 R is same and reactance increases i.e. addition
of an inductance in series with Z.

23. (d)

Z0
Emax
SWR =
ZL = 100 Emin
Z0 Z0 5 Emin
s= =5
/2 Emin
1+ 2
2 5 = ; =
= 1 3
As, 2 < 1
2
For Yd , d= × = 2 120
2 2
= 2 = 24
Zo [ZL + jZo tan d ]
3 1 + 120
Zd =
Zo + jZL tan d 24. (d)
50[100 + j 50 tan ] 1+ 1+
Zd = = 100 SWR = 2=
(50 j 100 tan ) 1 1
1 1 Voltage reflection coefficient,
Yd = = = 0.01
Zd 100 1
=
2 3
For Ys , d= × = , ZL 0 Power reflection coefficient,
8 4
Pref 2 1
= =
Zo ZL + jZo tan Pinc 9
4
Zs = = jZo
11.11% of Pi is reflected.
Zo + jZL tan
4
25. (d)
1 1
Ys = = = 0.02 j
Zs jZo Z02 = ZOC ZSC
Y = Yd + Ys = 0.01 – 0.02j 50 × 50 50 50(2 3 j )
ZSC = = =
21. (c) 100 + j 50 2 + 3 j 13
ZSC = 7.69 – 11.54j
The /4 line has Zin = 0 as ZL =
V 26. (c)
Current Iin into the line = = 0.2 A
50 Vmax 4
S= = =4
Using current equation of the line, Vmin 1
jVL Zmax = Zo S
I(x) = Iin = IL cos x + sin x
Zo Zo
Zmin =
S
with x = , I L = 0 as ZL =
4 The diagram shows that minima is at load.
Open-circuit end voltage, 50
ZL = Zmin = = 12.5
VL = 60 V 4
74 Electronics Engineering Electromagnetic Theory

27. (a) (50)2


Zs = = 300
ZL Zo 12.5 50 25 /3
= = = 0.6
ZL + Zo 12.5 + 50 Zs Zo 300 50 5
= = =
Zs + Zo 300 + 50 7
29. (d)
32. (b)
The line is terminated with 50 at the center
30 V Zo = 50 ZL and so, matched on both the sides.

V = 40 V 34. (d)
t = 400 µs
t = 0s It is a short-circuit line with l = 1 cm.
10 V reflects to add to 30 V in phase and gives
3 × 108
40 V, As, = = 0.3 = 30 cm
3 × 10 9
Vr 10 1
= = =
Vi 30 3 <
4
ZL Zo The line has inductive nature.
=
ZL + Zo As, ZSC = jZo tan l
1 ZL 50
= 35. (d)
3 ZL + 50
ZL = 100 Zo2 (100)2
Z1 = = = 200
ZL1 50
30. (b)
In steady-state, Zo2 (100)2
Z2 = = = 200
VL = 30 V Z2 50
30 ZL = Z1 Z2 = 100
IL = = 0.3 A
100
Zo2 (50)2
31. (d) Zi = = = 25
ZL 100
/4
36. (d)
For distortionless transmission line,
Zo = 50 R1 R2 LR = RC
L R
=
C G
Zo2 Characteristic impedance,
Zin = ; if l =
ZL 4 L R
Zo = =
C G
502
R1 due to 100 = = 25 Attenuation constant,
100
502 25 R R 0.1
= = RG = R = =
R2 due to 200 = Zo Zo 50
200 3
25 25 = 0.002 Np/m
R1 R2 = 25 =
2 3
GATE Previous Years Solved Paper 75

37. (b) 38. (c)

ZL + jZo tan l For reciprocal networks,


Zin = Zo S12 = S21
Zo + jZL tan l
For symmetrical networks,
Short
S11 = S22
Zo = 30 /8 For antimetrical networks,
S11 = –S22
Z1
For lossless reciprocal networks,
Zo = 60 Z2 Zo = 30 2 ZL = 30
S11 = S22
2 2
and S11 + S12 =1
/4

Input impedance for /8 line, 39. (c)

2 2
0 + j 30 tan Phase difference = (Path difference)
8
Z 1 = 30
30 + 0 2
= (2 × 10 3 )
4
Z 1 = j30 = 8 × 2 × 10–3
Input impedance for /4 line, = 16 × 10–3 m
2 f = 10 GHz = 10 × 109 Hz
30 + j 30 2 tan
4 Hence the phase velocity of wave along the line
Z 2 = 30 2
2 is,
30 2 + j 30 tan
4 v =f
= 10 × 109 × 16 × 10–3 m/sec
30
+ j 30 2 v = 1.6 × 108 m/s
tan /2
= 30 2 = 60
30 2 40. (a)
+ j 30
tan /2
1+
VSWR = 5 =
(30 2 )2 1
or, Z2 = = 60
30
2 2
Load impedance, ==±
3 3
ZL = Z1 + Z2 = j30 + 60 If maximum is at /4 from load then minimum
Magnitude of reflection coefficient, will be at load itself.
ZL Zo 60 + j 30 60 For the phase of ,
= =
ZL + Zo 60 + j 30 + 60 2 xmax = 2n +
j 30 j1 where xmax is distance of Vmax from load,
= =
120 + j 30 4+ j 2
2 =
1 1 4
= =
16 + 1 17 2
=
VSWR on 60 line, 3
1 2 Z Zo ZL 50
1+ Now, = L =
1+ 17 = 1.64 3 ZL + Zo ZL + 50
VSWR = =
1 1 ZL = 10
1
17
76 Electronics Engineering Electromagnetic Theory

41. (*) 45. Sol.


Characteristics impedance of the co-axial cable ZL + jZo tan l
is given by Zin = Zo
Zo + jZL tan l
138 D
Zo = log
r d At, =
8
138 2.4 2
= log = 15.89 l= × =
10.89 1 8 4

42. (c) tan =1


4
The 100 device used to match 50 and 200
ZL + j 50
devices is a quarter wave transformer. Zin = Zo
Zo + jZL
Length of matching device
Given, ZL = R + jX
= (2n + 1) = odd multiple of = R + j30 and Zo = 50
4 4
30 R = 80 (50 + R)
where n-depends on frequency of . R + j 30 + j 50
4 Zin = 50
50 + j( R + j 30)
1 of frequency 429 MHz = 0.7 m

2 of frequency 1 GHz = 0.3 m R + j 80


= 50
Common multiple = 2.1 m (50 30) + jR
Dividing by 4 = 0.2525 m (not available in
R + j 80
options) = 50
20 + jR
Another odd multiple of 0.525 × 3 = 1.58 m
( R + j 80) (20 jR )
43. (a) = 50 2 2
20 + R
Return loss = 20 log Since, only real part of Zin exists, so imaginary
20 log = 20 part of Zin = 0.

= 10–1 = 0.1 20 R + 1600 j jR 2 + 50 R


Zin = 50
1+ 1 + 0.1 20 2 + R 2
VSWR = = = 1.22
1 1 0.1 1600 R 2
Zimaginary = 0 =
20 2 + R 2
44. (b)
R 2 = 1600
L R = 40
Zo = for parallel plate line
C
46. Sol.
C W L µd
= and = Input impedance for quarter wave transformer
l d l W
where, W = Width of the line is given as,
d = Separation between the line Zo2
Zin =
Zo d (Zo changes by half) ZL
1 1 Given, Zin = 50 , ZL = 100
Velocity = =
LC µo o
Zo2 = 50 × 100
= 3 × 108 m/s
(No change in V) Zo = 50 × 100 = 70.72
GATE Previous Years Solved Paper 77

47. Sol. 50. (b)


A The teflon dielectric between the lines has tan
Zo = 50 , L = 0.5 = loss tangent = 0
100 50
For the medium between the lines,
G=0
Zin = ? Zin1
51. (d)
ZL + jZo tan l l = 3m
Zin1 = Zo
Zo + jZL tan l f = 37.5 MHz
2 3 × 108
l= × = = =8m
2 37.5 × 10 6
ZL + j 0 Zin (SC) = jZo
Zin1 = Zo
Zo + j0 2
tan l = j 120 tan 3
8
Zin1 = ZL = 50
= –j120
Zin = ( Zin 1 100 )
52. Sol.
100 × 50
= = 33.3 2
150 C=
b
ln
48. (c) a
The signal wave gets distorted if there is any 2
172 × 10–12 =
impedance mismatch between the characteristic ln 5
impedance Zo and the transmitting impedance
172 × 10 12 × ln 5
ZT and Zo and receiving impedance ZR. =
2
i.e., Z T Zo and ZR Zo
If ratio of b/a is doubled then,
then only distortion occurs.
As standing waveforms the signal has multi- 172 × 10 12 × ln 5
2 ×
harmonics and hence distortion occurs. 2 2
C= =
ln 10 ln 10
49. (b)
= 120.22 pF
b1 = s11 a1 + s12 a2 ...(i)
b2 = s21 a1 + s22 a2 ...(ii) 53. Sol.
When output is short, At t = 0,
a 2 = –b 2 R0 50
–a 2 = s21 a1 + s22 a2 Vin = 400 × = 400 × = 100 V
R0 + Rs 200
s21 a1
a2 = At t = T0 L 100 volts reflects,
1 + s22
At t = 2T0,
Substituting in first equation,
Vin = 100 + (1 + in) L 100 = 62.5
s21 a1
b1 = s11 a1 + s12 1 RL 50
1 + s22 L = =
4 RL + 50
s11 a1 + s11 s22 a1 s12 s21 a1
= RL = 30
1 + s22
b1 s11 + s11 s22 s12 s21
= s11 =
a1 1 + s22
78 Electronics Engineering Electromagnetic Theory

54. (b) 58. Sol.

= ( R + j L ) (G + j C ) V(l) = Vo e– l e–j l e j t

Input Vo (0)
R+ j L Attenuation = =
Zo = Output Vo (l )
G+ j C
y Zo = R+j L Vo
Attenuation per meter = =e
= (2 + j5) (50 – j0) Vo (1 m)
= 100 + j250 Attenuation in dB/m = (20 log e ) dB/m
R= 100 /m = 20(0.1) log e = 0.868 dB/m
250 250
L= = = 250 µH/m 59. Sol.
10 6
Given that,
2 + j5
= = G + j C = 0.04 + j0.1 VSWR (or) s = 5.8
Zo 50
1+
G = 0.04 S/m s= = 5.8
1
0.1 0.1
C= = = 0.1 µF/m = Reflection coefficient
10 6
of television set
56. Sol. s 1 4.8
= = = 0.70588
At frequency of 1 GHz the line is matched and s + 1 6.8
Preflected
= 0 as seen in the graph. = 2 = 0.4983 or 49.83%
Pincident
T1 T2
60. Sol.

Open-circuit
Initially the wave is circularly polarized. So, the
50 50
initial phase difference between field
components in aˆx direction and aˆ y direction is
The line T1 is having 50 load parallel to T2 /2.
input impedance. To become linearly polarized, the wave must
This load is 50 when T2 input impedance is travel a minimum distance, such that, the phase
infinite, difference at that point between the field
Zin 2 = = ZL2 (open-circuit) components in aˆx direction and aˆ y direction is
T2 has to be a /2 line,
(i.e., the travel of this minimum distance
f = 1 GHz
should provide an additional phase difference
2 × 1010 of /2 between aˆx and aˆ y field components).
= cm
1 × 109

= L = 10 cm = 0.1 m So, zmin kx ~ zmin ky =


2 2

57. Sol. zmin ~ =


v px v py 2
sin max = n12 n22 = 1.5 2
1.4 2
f f
= sin–1 (0.5385) = 32.58° 2 zmin rx ~ rx =
max c c 2
GATE Previous Years Solved Paper 79

4 zmin 63. Sol.


[nx ~ ny ] = 1
0 l = 3 /4

0
zmin =
4(nx ~ ny )
Zo = 50 ZL = 400
1.5
= µm
4(1.5 ~ 1.5001)
1.5 1.5
= µm = cm Zin
4(0.0001) 4
zmin = 0.375 cm Zo2 502 25
Zin for ( = /4) = = = = 6.25
ZL 400 4
61. (c)
For short-circuit, 64. (d)
r = x=0 Point “P” For given impedance normalized impedance is
For open-circuit,
Z jX
r = x= Point “R” =
Zo Zo
For matched load,
Z = jX
r = 1 and x = 0 Point “Q”
Z = 0 + jX
P : Short-circuit
Normalized resistance = 0 r = 0
Q : Matched load
X = – to
R : Open-circuit
r = 0 and X from – is a unit circle (radius 1)
62. Sol. and center (0, 0) on a complex reflection
For a distortionless transmission line, coefficient plane:
L C i
=
R G r=0
Propagation constant,
= + j = ( R + j L ) (G + j C )
r

L
= RG 1 + j
R
Attenuation constant,
= RG 65. (d)
Characteristic impedance,
0.25
P R
( R + j L) R
Zo = = 0.25
(G + j C ) G Z1 = 75
Z0 = 50 ZL
R
G =
Zo Q
ub
O.C 75
S.C = 75
ub

. st

R R
1 =
. st

So, = R =
1

Z
Z

Zo Zo 0.5

0.05 0.01
= = = 0.001 Np/m Inverter
50 10

Mirror
80 Electronics Engineering Electromagnetic Theory

For the matching to take place there should not


0.25
P R be any reflection.
i.e., L = 0
Z0 = 50 Z1 = 75 ZL ZPQ Z0
=0
ZPQ + Z0
Q S ZPQ = Z 0
P 5625
= Z0
RL
Z0 = 50 ZPQ 5625
RL = = 112.5
50
Q

752 5625
ZPQ = = (ZL = RL)
ZL RL
4 Waveguides

ELECTRO NICS EN GINEERIN G 20 20


Q.4 The interior of a cm × cm rectangular
(GATE Previous Years Solved Papers) 3 4
waveguide is completely filled with a dielectric
Q.1 The cut-off frequency of a waveguide depends
of r = 4. Waves of free space wave lengths
upon
shorter than _____ can be propagated in the
(a) the dimensions of waveguide. TE11 mode.
(b) the dielectric property of the medium in the [EC-1994 : 1 Mark]
waveguide.
Q.5 A rectangular air filled waveguide has a cross-
(c) the characteristic impedance of the
section of 4 cm × 10 cm. The minimum frequency
waveguide.
which can propagate in the waveguide is
(d) the transverse and axial components of the
(a) 1.5 GHz (b) 2.0 GHz
fields.
(c) 2.5 GHz (d) 3.0 GHz
[EC-1987 : 2 Marks]
[EC-1997 : 1 Mark]
Q.2 For a normal mode EM wave propagating in a
Q.6 Assuming perfect conductors of a transmission
hollow rectangular wave guide
line, pure TEM propagation is not possible in
(a) the phase velocity is greater than the group
(a) coaxial cable
velocity.
(b) air filled cylindrical waveguide
(b) the phase velocity is greater than velocity
(c) parallel twin-wire line in air
of light in free space.
(d) semi-infinite parallel to plate waveguide
(c) the phase velocity is less than the velocity
[EC-1999 : 1 Mark]
of light in free space.
(d) the phase velocity may be either greater Q.7 A rectangular waveguide has dimensions
than or less than the group velocity. 1 cm × 0.5 cm. Its cut-off frequency is
[EC-1988 : 2 Marks] (a) 5 GHz (b) 10 GHz
(c) 15 GHz (d) 12 GHz
Q.3 Choose the correct statements:
[EC-2000 : 2 Marks]
For a wave propagating in an air filled
rectangular waveguide Q.8 The phase velocity of waves propagating in a
(a) guided wavelength is never less than the hollow metal waveguide is
free space wavelength. (a) greater than the velocity of light in free
space.
(b) wave impedance is never less than the free
space impedance. (b) less than the velocity of light in free space.
(c) equal to the velocity of light in free space.
(c) phase velocity is never less than the free
space velocity. (d) equal to the group velocity.

(d) TEM mode is possible if the dimensions of [EC-2001 : 1 Mark]


the wave guide are properly chosen. Q.9 The dominant mode in a rectangular waveguide
[EC-1990 : 2 Marks] is TE10, because this mode has
82 Electronics Engineering Electromagnetic Theory

(a) no attenuation y
(b) no cut-off
(c) no magnetic field component (c)
(d) the highest cut-off wavelength
[EC-2001 : 1 Mark] x

y
Q.10 The phase velocity for the TE10 mode in an air
filled rectangular waveguide is
(d)
(a) less than c (b) equal to c
(c) greater than c (d) none of the above x
Note: (c) is the velocity of plane waves in free [EC-2005 : 2 Marks]
space.
[EC-2002 : 1 Mark] Q.14 A rectangular waveguide having TE10 mode as
dominant mode is having a cut-off frequency of
Q.11 A rectangular metal wave guide filled with a 18 GHz for the TE30 mode. The inner broad wall
dielectric material of relative permittivity r = 4 dimension of the rectangular waveguide is
has the inside dimensions 3.0 cm × 1.2 cm. The (a) 5/3 cm (b) 5 cm
cut-off frequency for the dominant mode is
(c) 5/2 cm (d) 10 cm
(a) 2.5 GHz (b) 5.0 GHz
[EC-2006 : 2 Marks]
(c) 10.0 GHz (d) 12.5 GHz
[EC-2003 : 2 Marks] Q.15 An air filled rectangular waveguide has inner
dimensions of 3 cm × 2 cm. The wave impedance
Q.12 The phase velocity of an electromagnetic wave of the TE 20 mode of propagation in the
propagating in a hollow metallic rectangular waveguide at a frequency of 30 GHz is (free
waveguide in the TE10 mode is space impedance 0 = 377 )
(a) equal to its group velocity. (a) 308 (b) 355
(b) less than the velocity of light in free space. (c) 400 (d) 461
(c) equal to the velocity of light in free space. [EC-2007 : 2 Marks]
(d) greater than the velocity of light in free
space. Q.16 The E field in a rectangular waveguide of inner
[EC-2004 : 1 Mark] dimensions a × b is given by

Q.13 Which one of the following does represent the µ 2 x


E= 2 H 0 sin sin ( t z) yˆ
electric field lines for the TE02 mode in the cross- h a a
section of a hollow rectangular metallic where H0 is a constant, and a and b are the
waveguide? dimensions along the x-axis and the y-axis
y respectively. The mode of propagation in the
waveguide is
(a) (a) TE20 (b) TE11
(c) TM20 (d) TE10
x
[EC-2007 : 2 Marks]
y
Q.17 A rectangular waveguide of internal
(b) dimensions (a = 4 cm and b = 3 cm) is to be
operated in TE11 mode. The minimum operating
x frequency is
GATE Previous Years Solved Paper 83

(a) 6.25 GHz (b) 6.0 GHz The phase velocity vp of the wave inside the
(c) 5.0 GHz (d) 3.75 GHz waveguide satisfies
[EC-2008 : 2 Marks] (a) vp > c (b) vp = c
(c) 0 < vp < c (d) vp = 0
Q.18 Which of the following statements is true
[EC-2012 : 2 Marks]
regarding the fundamental mode of the metallic
waveguides shown? Q.21 For a rectangular waveguide of internal
dimensions a × b (a > b), the cut-off frequency for
the TE11 mode is the arithmetic mean of the
P : Coaxial Q : Cylindrical cut-off frequencies for TE10 mode and TE20
mode. If a = 5 cm, the value of b (in cm) is _____.
[EC-2014 : 2 Marks]
R : Rectangular

(a) Only P has no cut-off frequency. Q.22 Consider an air filled rectangular waveguide
with a cross-section of 5 cm × 3 cm. For this
(b) Only Q has no cut-off frequency.
waveguide, the cut-off frequency (in MHz) of
(c) Only R has no cut-off frequency.
TE21 mode is _______ .
(d) All three have cut-off frequencies.
[EC-2014 : 2 Marks]
[EC-2009 : 1 Mark]
Q.23 The longitudinal component of the magnetic
Q.19 The modes in a rectangular waveguide are
field inside an air filled rectangular waveguide
denoted by TEmn/TMmn where ‘m’ and ‘n’ are
made of a perfect electric conductor is given by
the eigen numbers along the larger and smaller
the following expression:
dimensions of the waveguide respectively.
H z (x, y, z, t) = 0.1 cos(25 x) cos(30.3 y)
Which one of the following statements is true?
cos(12 × 109t – z) A/m
(a) The TM10 mode of the waveguide does not
The cross-sectional dimensions of the
exist.
waveguide are given as a = 0.08 m and
(b) The TE10 mode of the waveguide does not
b = 0.033 m. The mode of propagation inside the
exist.
waveguide is
(c) The TM10 and the TE10 modes both exist
and have the same cut-off frequencies. (a) TM12 (b) TM21
(d) The TM10 and the TE01 modes both exist (c) TE21 (d) TE12
and have the same cut-off frequencies. [EC-2015 : 2 Marks]
[EC-2011 : 1 Mark] Q.24 An air filled rectangular waveguide of interval
Q.20 The magnetic field along the propagation dimensions a cm × b cm (a > b) has a cut-off
direction inside a rectangular waveguide with frequency of 6 GHz for the dominant TE10 mode.
the cross-section shown in figure is For the same waveguide, if the cut-off frequency
of the TM 11 mode is 15 GHz, the cut-off
y
frequency of the TE01 mode (in GHz) is _____ .
[EC-2015 : 2 Marks]

Q.25 Consider an air filled rectangular waveguide


1.2 cm
x
with dimensions, a = 2.286 cm and b = 1.016 cm.
3 cm At 10 GHz operating frequency, the value of the
Hz = 3 cos(2.094 × 102 x) cos(2.618 × 10 2 y) propagation constant (per meter) of the
cos(6.283 × 1010t – z) corresponding propagating mode is ______ .
[EC-2016 : 2 Marks]
84 Electronics Engineering Electromagnetic Theory

Q.26 Consider an air filled rectangular waveguide 1 2 2


with dimensions, a = 2.286 and b = 1.016 cm. k( ) = o
c
The increasing order of the cut-off frequencies
where the speed of light c = 3 × 108 m/s, and wo
for different modes is
is a constant. If the group velocity is 2 × 108 m/s,
(a) TE01 < TE10 < TE11 < TE20 then the phase velocity is
(b) TE20 < TE11 < TE10 < TE01 (a) 2 × 108 m/s (b) 3 × 108 m/s
(c) TE10 < TE20 < TE01 < TE11 (c) 1.5 × 108 m/s (d) 4.5 × 108 m/s
(d) TE10 < TE11 < TE20 < TE01 [EC-2019 : 2 Marks]
[EC-2016 : 2 Marks]
Q.31 A standard air-filled rectangular waveguide
Q.27 Standard air filled rectangular waveguides of with dimensions, a = 8 cm, b = 4 cm, operates at
dimensions a = 2.29 cm and b = 1.02 cm are 3.4 GHz. For the dominant mode of wave
designed for radar applications. It is desired that propagation, the phase velocity of the signal is
these waveguides operate only in the dominant vp. The value (Rounded off to two decimal
TE10 mode with the operating frequency at least places) of vp/c, where ‘c’ denotes the velocity of
25% above the cut-off frequency of the TE10 light, is _______ .
mode but not higher than 95% of the next higher [EC-2021 : 2 Marks]
cut-off frequency. The range of the allowable
operating frequency, f is ______ . Q.32 Consider the following wave equation:
(a) 8.19 GHz f 13.1 GHz 2
f (x , t ) 2
f (x , t)
= 10000
(b) 8.19 GHz f 12.45 GHz t 2
x2
(c) 6.55 GHz f 13.1 GHz Which of the given options is/are solution(s) to
(d) 1.64 GHz f 10.24 GHz the given wave equation?
[EC-2017 : 2 Marks]
( x 100t )2 ( x + 100t )2
(a) f (x , t ) = e +e
Q.28 The cut-off frequency of TE01 mode of an air filled
( x 100t ) ( x + 1000t )
rectangular waveguide having inner (b) f (x , t ) = e + 0.5e
dimensions a cm × b cm (a > b) is twice that of ( x 100t )
(c) f (x , t ) = e + sin( x + 100t )
the dominant TE10 mode. When the waveguide
is operated at a frequency which is 25% higher (d) f ( x , t ) = e 100 ( 100 x + t ) + e j 100 (100 x + t )
than the cut-off frequency of the dominant mode, [EC-2022]
the guide wavelength is found to be 4 cm. The
value of b (in cm) correct to two decimal places, Q.33 A waveguide consists of two infinite parallel
is ______ . plates (perfect conductors) at a separation of
[EC-2018 : 2 Marks] 10–4 cm, with air as the dielectric. Assume the
speed of light in air to be 3 × 108 m/s. The
Q.29 A rectangular waveguide of width ‘w’ and frequency/frequencies of TM waves which can
height ‘h’ has cut-off frequencies for TE10 and propagate in this waveguide is/are ______ .
TE11 modes in the ratio 1 : 2. The aspect ratio (a) 6 × 1015 Hz (b) 0.5 × 1012 Hz
w/h, rounded off to two decimal places, is _____.
(c) 8 × 1014 Hz (d) 1 × 1013 Hz
[EC-2019 : 2 Marks]
[EC-2022]
Q.30 The dispersion equation of a waveguide, which
relates the wave number ‘k’ to the frequency ,
is
GATE Previous Years Solved Paper 85

Electronics & Electrical Engineering


GATE Previous Years Solved Paper

A n swe rs & Expl a n a t i o n s

Answers
EC Waveguides

1. (a, b) 2. (a, b) 3. (a, c) 4. (8) 5. (a) 6. (b) 7. (c) 8. (a)

9. (d) 10. (c) 11. (a) 12. (d) 13. (d) 14. (c) 15. (c) 16. (a)

17. (a) 18. (a) 19. (a) 20. (a) 21. (2) 22. (7810) 23. (c) 24. (13.75)

25. (158) 26. (c) 27. (b) 28. (0.75) 29. (1.732) 30. (d) 31. (1.19 or 1.20)
32. (a, c) 33. (a, c)

Solutions
EC Waveguides

1. (a, b) 2
fc
2 2 vg = c 1
vo m n f
fc = +
2 a b
3. (a, c)
c
where, vo =
µr r =
g 2
fc
2 2 1
c m n f
fc = +
µr r ×2 a b
g >
So, fc depends on ‘a’ and ‘b’ (dimensions of
0
waveguide), TE =
2
fc
1 1
fc f
µr r
TE > 0
So, fc depends upon the dielectric property of
2
the medium in the waveguide. fc
TM = 0 1
f
2. (a, b)
v p > c > vg TM < 0
1 c
c vp = = = 2
vp = µ fc
2
fc µr r 1
1 r
f
vp > c
86 Electronics Engineering Electromagnetic Theory

4. Sol. 8. (a)
1 1 c c
v = = Vp = =
µ µ0 0 r cos 2
fc
1 1 1
= × f
µ0 0 r
with f > fc, Vp > c always.
8
3 × 10
v = = 1.5 × 108 m/sec 9. (d)
4
m = n=1 Dominant mode in waveguide has lowest cut-
off frequency and hence the highest cut-off
2 2
v m n wavelength.
fc = +
2 a b
10. (c)
8 2 2
1.5 × 10 3 4 c c
= + × 102 Vp = =
2 20 20 cos 2
fc
1
7.5 × 109 f
fc = × 5 = 1.875 × 10 9 Hz
20 with f > fc, Vp > c always.
8
v 1.5 × 10 11. (a)
= = = 8 cm
c
f c 1.875 × 109
c 3 × 108
So, waves of free space wavelength shorter than v = = = 1.5 × 108
r 2
8 cm can be propagated,
c = 8 cm v 1.5 × 108
fc = = 2
= 2.5 GHz
c 6 × 10
5. (a)
a = 10 cm v 1.5 × 10 8
or, fc = = = 2.5 GHz
2 a 6 × 10 2
vo
fc =
2a 13. (d)
vo = 3 × 1010 cm/sec Two maxima in the field along y-direction
10
3 × 10 identifies TE02.
fc = = 1.5 × 109
2 × 10
14. (c)
= 1.5 GHz
2a 2a
TE30 : = =
6. (b) c
m 3
A pure TEM mode is not possible in single c 3 × 108 1
conductor waveguide such as rectangular or c = = 9
=
f 18 × 10 60
cylindrical waveguide.
1 2a
=
7. (c) 60 3
= 2 a (For TE10 dominant mode) 1 5
c a= = 2.5 cm = cm
= 2 × 1 cm = 2 × 10–2 m 40 2

c 3 × 108 15. (c)


fc = = 2
= 15 GHz
c 2 × 10 For TE20 :
c = a = 3 × 10–2 m
GATE Previous Years Solved Paper 87

c 3 × 108 2 2 3 × 1010
fc = = = 1010 Hz f c = ( (2.094) + (2.618) ) ×
3 × 10 2 2 × 3.14
c
= 16 GHz
377
= = 6.283 × 1010
TE 2 2
fc 1010 f= = 10 GHz
1 1 2 × 3.14
f 3 × 1010 Hence, f < fc ; = + j0
= 400
= 0, Vp = =
16. (a)
The wave is E(x, z, t) with no propagation in 21. Sol.
y-direction. Cut-off frequency of a rectangular waveguide is
This is possible only in TE waves when n = 0
2 2
c m n
n fc = +
cos y = 1 2 a b
b
2
17. (a) c 1 c
fc for : TE10 = =
2 a 2a
Minimum operating frequency,
2
c m 2
n 2 c 2 c
f0 = + fc for : TE20 = =
2 a b 2 a 2a
For TE11 mode, 2 2
c 1 1
2 2 fc for : TE11 = +
3 × 108 1 1 2 a b
f0 = +
2 0.04 0.03 fc TE + fcTE
10 20
8 Given, fcTE =
3 × 10 0.05 11 2
= ×
2 0.04 × 0.03 c c
= 6.25 GHz 2 2 +
c 1 1 2a a
+ =
2 a b 2
18. (a)
2 2 1 1 3
P is co-axial line and support TEM wave. 1 1 + =
+ =
P has no cut-off frequency, a b 2a a 2a
Q and R are waveguides and cut-off frequency 2 2
1 1 9
of each depends upon their dimensions. + =
a b 4 a2
19. (a) 2
1 5
In case of rectangular wave guide TEmn exists =
b 4 a2
for all values of m and n except m = 0 and n = 0.
For TMmn to exist both values of m and n must be 2a 2 × 5
b= =
non-zero. 5 5

20. (a) (Given, a = 5 cm)


= 2 cm
From the given data,
2 2
22. Sol.
m n
c = + c Given, a = 5 cm = 5 × 10–2 m
a b
b = 3 cm = 3 × 10–2 m
88 Electronics Engineering Electromagnetic Theory

2 2 25. Sol.
c m n
fc = + Operation frequency = 10 GHz
2 a b
Cut-off frequency for TE10,
2 2
c 2 1 c 3 × 10 8
fcTE = + =
fc =
21 2 5 cm 3 cm 2 a 2 × 2.286 × 10 2
= 6.57 GHz
2 2
3 × 108 2 × 100 100 Next mode TE20 has fc = 13 GHz
= +
2 5 3 10 GHz operates only in dominant mode,
f c = 7810 MHz 2
m 2
= µ
23. (c) a
Hz (x, y, z, t) = 0.1 cos(25 x) cos(30.3 y) 2
(2 f )2
cos(12 × 109t – z) A/m =
a (3 × 108 )2
a = 0.08 m, b = 0.033 m
Axial component is H the propagating mode 2 2
1 2 × 10 × 109
is TEmn, m, n can be found by =
2.286 × 10 2 3 × 108
m
x = 25 x = j158 (m–1)
a
m 26. (c)
= 25
0.08
a = 2.286 cm
m=2
b = 1.016 cm
n
y = 30.3 y c
b fc TE =
10 2a
n
= 30.3 c
0.033 fcTE =
01 2b
n=1
c
The mode of propagation is TE21. fc TE =
20 a
24. Sol. 2 2
c 1 1
Given, fc TE10 mode is 6 GHz, fcTE = +
11 2 a b
c 3 × 10 10 As, a > 2b f cTE < f cTE < f cTE < f cTE
= = 6 × 109 10 20 01 11
2a 2× a
a = 2.5 cm 27. (b)
Given, fc for TM11 mode is 15 GHz. a = 2.29 cm
c 1 1 b = 1.02 cm
2
+ 2 = 15 × 109
2 a b Waveguide is operating in (TE10) dominant
mode:
3 × 1010 1 1
+ = 15 × 109
2 6.25 b 2 c 3 × 1010
• fc (TE10) = = = 6.55 GHz
2.5 2 a 2 × 2.29
b= 25% above cut-off frequency
5.25
Now fc for TE01 mode is, = 1.25 × 6.55 GHz

c 3 × 1010 = 8.1875 GHz


fc = = 5.25 = 13.75 GHz
2b 2×5
GATE Previous Years Solved Paper 89

• Next higher order mode is TE20 (Q a > 2b) and for TE11 mode,

c 3 × 1010 c 1 2
1 2
fc (TE20) = = = 13.1 GHz f c 11 = +
a 2.29 2 w h
95% of fc (TE20) = 0.95 × 13.1
2
= 12.445 GHz c w
= 1+ ...(ii)
• Range of allowable operating frequency f is 2w h
8.19 GHz f 12.45 GHz f c 10 1
Given, = ...(iii)
f c 11 2
28. Sol.
Put, (i), (ii) in equation (iii),
2c c
fc (01) = 2 f c (10) = = c /2 w 1
2a a =
c w
2 2
c c 1+
= 2w h
2b a
a 2
a = 2b b= w
2 1+ =2
h
Operating frequency,
On solving above equation, we get,
f = 1.25fc (10)
fc (10) < 1.25fc(10) < [fc(01) = 2fc(10)] w
= 3 = 1.732
So, for the given frequency, the waveguide will h
work in TE10 mode. 30. (d)

So, =
0 By definition,
g 2
fc (10)
1 vp = =
f k
1 2 2
c/ f c/ f where, k( ) = o (given)
= = c
1 2 0.6
1 c
1.25 vp =
2
c 1 o
= g = 4 cm
(1.25) fc(10) (0.6)

c d d
By definition, v g = =
= 3 × 10–2 = 2a d dk
f c (10)
a = 1.5 cm dk 1 1
= ×2
d c2 2 2
a o
b = = 0.75 cm
2
2
o
29. Sol. or, vg = c 1

c m 2
n 2 Q v p v g = c2
fcmn = +
2 a b c 2 (3 × 108 )2
vp = =
For TE10 mode, vg 2 × 108
c = 4.5 × 108 m/sec.
f c 10 = ...(i)
2w
90 Electronics Engineering Electromagnetic Theory

31. (1.19 or 1.20) 32. (a, c)


Given: An air-filled rectangular waveguide, Given wave equation,
a = 8 cm, b = 4 cm 2
f (x , t ) 2 d2 f
f = 3.4 GHz = c ; c = 100
t2 dt 2
Operating in dominant mode (TE10)
Solution is given as,
Phase velocity, vp is given by
F = f(x ± ct)
c Hence, (a) and (c) satisfies the above solution.
vp =
2
fc
1 33. (a, c)
f
The TM mode that propagate at the lowest
vp 1 cut-off frequency is TM1 mode. Hence,
=
c 2 mc
fc fc TM1 = ; m=1
1 2a
f
3 × 1010
where, fc = 4
= 1.5 × 1014
2 × 10
c 3 × 1010
fC(TE10) = = = 1.875 GHz
2a 2×8
vp 1
= = 1.198
c 1.875 2
1
3.4
The factor vp/c is 1.198 (or) 1.20.
5 Antennas

ELECTRO NICS EN GINEERIN G Q.5 The beamwidth between first nulls of a uniform
linear array of N equally spaced (element
(GATE Previous Years Solved Papers)
spacing = d), equal excited antennas, is
Q.1 The electric field E and the magnetic field H of a determine by
short dipole antenna satisfy the condition (a) N alone and not by d
(a) the r component of E is equal to zero. (b) d alone and not by N
(b) both r and components of H are equal to (c) the ratio, (N/d)
zero. (d) the product, (Nd)
(c) the component of E dominates the r [EC-1992 : 2 Marks]
component in the far field region.
(d) the and components of H are of the same Q.6 For a half-wave dipole antenna, which of the
order of magnitude in the near field region. following statements are right?
[EC-1988 : 2 Marks] (a) The radiation intensity is maximum along
the normal to the dipole axis.
Q.2 Two isotropic antennas are separated by a
(b) The current distribution along its length is
distance of two wavelengths. If both the
uniform irrespective of the length.
antennas are fed with currents of equal phase
and magnitude, the number of lobes in the (c) The effective length equals its physical
radiation pattern in the horizontal plane are length.
(a) 2 (b) 4 (d) The input impedance is independent of the
(c) 6 (d) 8 location of the feed point.

[EC-1990 : 2 Marks] [EC-1994 : 1 Mark]

Q.3 In a broad side array of 20 isotropic radiators, Q.7 An antenna, when radiating has a highly
equally spaced at distance of /2, the beam directional radiation pattern. When the antenna
width between first nulls is is receiving, its radiation pattern
(a) 51.3 degrees (b) 11.46 degrees (a) is more directive
(c) 22.9 degrees (d) 102.6 degrees (b) is less directive
[EC-1991 : 2 Marks] (c) is the same
(d) exhibits no directivity at all
Q.4 Two dissimilar antennas having their
maximum directivities equal, which of the [EC-1995 : 1 Mark]
following statements are right? Q.8 A transverse electromagnetic wave with circular
(a) must have their beam widths also equal. polarization is received by a dipole antenna.
(b) cannot have their beam widths equal Due to polarization mismatch, the power
because they are dissimilar antenna. transfer efficiency from the wave to the antenna
(c) may not necessarily have their maximum is reduced to about
power gains equal. (a) 50% (b) 35.3%
(d) must have their effective aperture areas (c) 25% (d) 0%
(capture areas) also equal. [EC-1996 : 2 Marks]
[EC-1992 : 2 Marks]
92 Electronics Engineering Electromagnetic Theory

Q.9 At 1 km long microwave link use two antennas Q.15 A transmitting antenna radiates 251 W
each having 30 dB gain. If the power transmitted isotropically. A receiving antenna, located
by one antenna is 1 W at 3 GHz, the power 100 m away from the transmitting antenna, has
received by the other antenna is approximately an effective aperture of 500 cm2. The total power
(a) 98.6 µW (b) 76.8 µW received by the antenna is
(c) 63.4 µW (d) 55.2 µW (a) 10 µW (b) 1 µW
[EC-1996 : 2 Marks] (c) 20 µW (d) 100 µW
Q.10 A parabolic dish antenna has conical beam 2° [EC-1999 : 2 Marks]
wide. The directivity of the antenna is Q.16 If the diameter of a /2 dipole antenna is
approximately
(a) 20 dB (b) 30 dB increased from to , then its
100 50
(c) 40 dB (d) 50 dB
[EC-1997 : 2 Marks] (a) bandwidth increases
(b) bandwidth decreases
Q.11 The vector H in the far field on an antenna (c) gain increases
satisfies (d) gain decreases
(a) H = 0 and ×H = 0 [EC-2000 : 1 Mark]
(b) H 0 and ×H 0 Q.17 For an 8 feet (2.4 m) parabolic dish antenna
(c) H = 0 and ×H 0 operating at 4 GHz, the minimum distance
required for far field measurement is closest to
(d) H 0 and ×H = 0
(a) 7.5 cm (b) 15 cm
[EC-1998 : 1 Mark]
(c) 15 m (d) 150 m
Q.12 The radiation resistance of a circular loop of [EC-2000 : 2 Marks]
one turn is 0.01 . The radiation resistance of
Q.18 A medium wave radio transmitter operating at
five turns of such a loop will be
a wavelength of 492 m has a tower antenna of
(a) 0.002 (b) 0.01
height 124 m. What is the radiation resistance
(c) 0.05 (d) 0.25
of the antenna?
[EC-1998 : 1 Mark]
(a) 25 (b) 36.5
Q.13 An antenna in free space receives 2 µW of power (c) 50 (d) 73
when the incident electric field is 20 mV/m rms. [EC-2001 : 2 Marks]
The effective aperture of the antenna is
Q.19 In a uniform linear array, four isotropic radiating
(a) 0.005 m2 (b) 0.05 m2
elements are spaced /4 apart. The progressive
(c) 1.885 m2 (d) 3.77 m2
phase shift between the elements required for
[EC-1998 : 1 Mark] forming the main beam at 60° off the end fire is
Q.14 The far field on an antenna varies with distance (a) – radians (b) – /2 radians
‘r’ as, (c) – /4 radians (d) – /8 radians
1 1 [EC-2001 : 2 Marks]
(a) (b) 2
r r
Q.20 The line of sight communication requires the
1 1
(c) (d) transmit and receive antennas to face each other.
r3 r
If the transmit antenna is vertically polarized,
[EC-1998 : 1 Mark] for best reception the receiver antenna should be
GATE Previous Years Solved Paper 93

(a) horizontally polarized Q.24 Two identical and parallel dipole antennas are
(b) vertically polarized kept apart by a distance of l/4 in the H-plane.
(c) at 45° with respect to horizontal polarized They are fed with equal currents but the right
(d) at 45° with respect to vertical polarized most antenna has a phase shift of +90°. The
radiation pattern is given as,
[EC-2002 : 1 Mark]

Q.21 A person with a receiver is 5 km away from the


transmitter. What is the distance that this person
must move further to detect a 3 dB decrease in (a) (b)
signal strength?
(a) 942 m (b) 2070 m
(c) 4978 m (d) 5320 m
[EC-2002 : 2 Marks]

Q.22 Two identical antennas are placed in the = /2


plane as shown in the figure. The elements have (c) (d)
equal amplitude excitation with 180° polarity
difference, operating at wavelength . The
correct values of the magnitude of the far zone
[EC-2005 : 2 Marks]
resultant electric field strength normalized with
that of a single element, both computed for Q.25 A transmission line is feeding 1 W of power to a
= 0, is horn antenna having gain of 10 dB. The antenna
is matched to the transmission line. The total
power radiated
(a) 10 Watts (b) 1 Watt
(c) 0.1 Watt (d) 0.01 Watt
s
[EC-2006 : 1 Mark]

s Q.26 A mast antenna consisting of a 50 meter long


vertical conductor operates over a perfectly
conducting ground plane. It is base fed at a
frequency of 600 kHz. The radiation resistance
2 s 2 s
(a) 2 cos (b) 2 sin of the antenna (in ) is
2 2
2
s s (a) (b)
(c) 2 cos (d) 2 sin 5 5
2
[EC-2003 : 2 Marks] 4 2
(c) (d) 20
5
Q.23 Consider a lossless antenna with a directive gain
[EC-2006 : 2 Marks]
of +6 dB. If 1 mW of power is fed to it the total
power radiated by the antenna will be Q.27 A /2 dipole is kept horizontally at a height of
(a) 4 mW (b) 1 mW 0/2 above a perfectly conducting infinite

1 ground plane. The radiation pattern in the plane


(c) 7 mW (d) mW
4 of the dipole ( E plane) looks approximately as,
[EC-2004 : 1 Mark]
94 Electronics Engineering Electromagnetic Theory

y
(a) Re ( P ) remains constant at any radial

(a) distance from the source.

(b) Re ( P ) increases with increasing radial


z
distance from the source.
y
(c) Re( P) nˆ dS remains constant at any
(b) s

radial distance from the source.


z
y (d) Re( P) nˆ dS decreases with increasing
s

(c) radial distance from the source.


[EC-2011 : 1 Mark]
z
y Q.31 The radiation pattern of an antenna in spherical
co-ordinates is given by
(d) F( ) = cos 4 , 0
2
z The directivity of the antenna is
[EC-2007 : 2 Marks] (a) 10 dB (b) 12.6 dB
(c) 11.5 dB (d) 18 dB
Q.28 For a Hertz dipole antenna, the Half Power
Beam Width (HPBW) in the E-plane is [EC-2012 : 1 Mark]

(a) 360° (b) 180°


Q.32 In spherical coordinates, let aˆ , aˆ denote unit
(c) 90° (d) 45°
vectors along the , -directions.
[EC-2008 : 1 Mark]
100
Q.29 At 20 GHz, the gain of a parabolic dish antenna E = sin cos( t r ) aˆ V/m
r
of 1 meter diameter and 70% efficiency is 0.265
and H = sin cos( t r ) aˆ V/m
(a) 15 dB (b) 25 dB r
(c) 35 dB (d) 45 dB represent the electric and magnetic field
[EC-2008 : 2 Marks] components of the EM wave at large distances r
from a dipole antenna, in free space. The average
Q.30 Consider the following statements regarding the
power (W) crossing the hemispherical shell
complex pointing vector P for the power located at r = 1 km, 0 /2 is ______ .
radiated by a point source in an infinite [EC-2014 : 2 Marks]
homogenous and lossless medium. Re ( P ) Q.33 For an antenna radiating in free space, the
electric field at a distance of 1 km is found to be
denotes the real part of P , S denotes a spherical
12 mV/m. Given the intrinsic impedance of the
surface whose center is at the point source, and free space is 120 , the magnitude of average
n̂ denotes the unit surface normal on S. Which power density due to this antenna at a distance
of the following statements is true? of 2 km from the antenna (in nW/m2) is _____ .
[EC-2014 : 1 Mark]
GATE Previous Years Solved Paper 95

Q.34 Match Column-A with Column-B: Q.37 Two lossless X-band horn antennas are
Column-A separated by a distance of 200 . The amplitude
1. Point electromagnetic source reflection coefficients at the terminals of the
2. Dish antenna transmitting and receiving antennas are 0.15
3. Yagi-Uda antenna and 0.18, respectively. The maximum
directivities of the transmitting and receiving
Column-B
antennas (over the isotropic antenna) are 18 dB
P. Highly directional
and 22 dB, respectively. Assuming that the input
Q. End fire
power in the lossless transmission line
R. Isotropic connected to the antenna is 2 W, and that the
Codes: antennas are perfectly aligned and polarization
P Q R matched, the power (in mW) delivered to the
(a) 1 2 3 load at the receiver is _______ .
(b) 2 3 1 [EC-2016 : 2 Marks]
(c) 2 1 3
Q.38 The electric field of a uniform plane wave
(d) 3 2 1
travelling along the negative z-direction is given
[EC-2014 : 1 Mark]
by the following equation:
Q.35 Two half-wave dipole antennas placed as i
Ew = ( aˆ x + jaˆ y ) E0 e jkz
shown in the figure are excited with
sinusoidally varying currents of frequency This wave is incident upon a receiving antenna
3 MHz and phase shift of /2 between them placed at the origin and whose radiated electric
(the element at the origin leads in phase). If the field towards the incident wave is given by the
maximum radiated E field at the point P in the following equation:
x-y plane occurs at an azimuthal angle of 60°, 1 jkr
Ea = ( aˆ x + 2 aˆ y ) EI e
the distance d (in meters) between the antenna r
is ________ . The polarization of the incident wave, the
z polarization of the antenna and losses due to
polarization mismatch are, respectively,
(a) linear, circular (clockwise), –5 dB
0 (b) circular (clockwise), linear, –5 dB
d y
(c) circular (clockwise), linear, –3 dB
60° OP >> d
(b) circular (anticlockwise), linear, –3 dB
x P
[EC-2016 : 2 Marks]
[EC-2015 : 2 Marks]
Q.39 The far zone power density radiated by a helical
Q.36 The directivity of an antenna array can be
antenna is approximated as,
increased by adding more antenna elements, as
1
a larger number of elements Wrad = Waverage aˆ r C 0 cos 4
r2
(a) improves the radiation efficiency
The radiated power density is symmetrical with
(b) increases the effective area of the antenna
respect to and exists only in the upper
(c) results in a better impedance matching
(d) allows more power to be transmitted by the hemisphere 0 ,0 2 , C 0 is a
antenna 2
[EC-2015 : 1 Mark] constant.
96 Electronics Engineering Electromagnetic Theory

The power radiated by the antenna (in Watts) Q.43 Radiation resistance of a small dipole current
and the maximum directivity of antenna, element of length l at a frequency of 3 GHz is
respectively, are 3 . If the length is changed by 1%, then the
(a) 15 C0, 10 dB (b) 1.256 C0, 10 dB percentage change in the radiation resistance,
(c) 1.256 C0, 12 dB (d) 1.5 C0, 12 dB rounded off to two decimal places, is ______ %.
[EC-2016 : 2 Marks] [EC-2019 : 1 Mark]

Q.40 A radar operating at 5 GHz uses a common Q.44 For an infinitesimally small dipole in free space,
antenna for transmission and reception. The the electric field E in the far field is proportional
antenna has a gain of 150 and a aligned for to (e–jkr/r) sin , where k = 2 / . A vertical
maximum directional radiation and reception infinitesimally small electric dipole ( I << ) is
to a target 1 km away having radar cross-section placed at a distance h(h > 0) above an infinite
of 3 m2. If it transmits 100 kW, then the received ideal conducting plane, as shown in the figure.
power (in µW) is _______ . The minimum value of h, for which one of the
[EC-2016 : 2 Marks] maxima in the far field radiation pattern occurs
at = 60°, is

z
Q.41 Consider a wireless communication link
between a transmitter and a receiver located in
free space, with finite and strictly positive Idl

capacity. If the effective areas of the transmitter


and the receiver antennas, and the distance
h
between them are all doubled, and everything
else remains unchanged, the maximum capacity
y
of the wireless link 0
(a) increased by a factor of 2
(a) 0.75 (b)
(b) decreased by a factor of 2
(c) 0.25 (d) 0.5
(c) remains unchanged
[EC-2020 : 2 Marks]
(d) decreases by a factor of 2
Q.45 An antenna with a directive gain of 6 dB is
[EC-2017 : 1 Mark]
radiating a total power of 16 kW. The amplitude
Q.42 A half wavelength dipole is kept in the x-y plane of the electric field in free space at a distance of
and oriented along 45° from the x-axis. 8 kM from the antenna in the direction of 6 dB
Determine the direction of null in the radiation gain (Rounded off to 3 decimal places) is
pattern for 0 . Here the angle (0 ) is _________ V/m.
measured from the z-axis, and the angle [EC-2021 : 2 Marks]
(0 2 ) is measured from the x-axis in the
x-y plane.
(a) = 90°, = 45° (b) = 45°, = 90°
(c) = 90°, = 135° (d) = 45°, = 135°
[EC-2017 : 2 Marks]
GATE Previous Years Solved Paper 97

Electronics & Electrical Engineering


GATE Previous Years Solved Paper

A n swe rs & Expl a n a t i o n s

Answers
EC Antennas

1. (b, c) 2. (d) 3. (b) 4. (a, c) 5. (d) 6. (a) 7. (b) 8. (d)

9. (c) 10. (c) 11. (c) 12. (d) 13. (c) 14. (a) 15. (d) 16. (b)

17. (d) 18. (b) 19. (c) 20. (b) 21. (b) 22. (d) 23. (b) 24. (c)

25. (b) 26. (a) 27. (b) 28. (c) 29. (d) 30. (c) 31. (a) 32. (55.5)

33. (47.7) 34. (b) 35. (50) 36. (b) 37. (3) 38. (c, d) 39. (b) 40. (Sol.)

41. (c) 42. (a) 43. (2.01) 44. (b) 45. (0.244)

Solutions
EC Antennas

1. (b, c) When = 0, 2 , –2 , 4 , –4 maxima occurs in


Every dipole antenna has field in directions of that direction.
r, , with Er, E , H . 2 4 3 5
Maximum at = q , ,, , ,
, ,
These fields depend on 1/r, 1/r2 and 1/r3 with 3 2 3 3 2 3
distance. So, number of lobes in the radiation pattern in
1/r terms are called as radiation fields and exist the horizontal plane = 8.
even at far away distance.
3. (b)
E and H have 1 terms each depending on 1/r.
Electric field strength of N element array,
1/r2 and 1/r3 terms are called as induction fields
and exist at closer distances with zero values sin ( N /2)
ET = NEo
far away. sin ( /2)
Er has only 1/r2 and 1/r3 terms.
Option (a) wrong Er exists. For null points sin N =0
2
Option (b) right E only exists.
Option (c) right E has 1/r terms which Er does with sin 0 (Denominator term)
2
not have.
Option (d) wrong H does not exist. N = 2 n for null points
2
2. (d)
4n
Given that, d = 2 and = 0 =
N
= + d cos
4n
2 + d cos =
= 0+ 2 cos = 4 cos N
98 Electronics Engineering Electromagnetic Theory

With = 0 for broadside array, • Current distribution is uniform only for


2 4n very short lengths like hertzian dipole.
cos =
2 N 2l
• leff = for /2 dipole
4n
cos =
N
st
• Z in for any wire antenna depends on
with n = 1 for 1 null points
loading effects from either sides of the feed
4 points or length on either sides.
cos =
20
NP1

= 78.46 7. (b)
NP1
with n = 2 for 2nd null points An antenna, when radiating, has a highly
8 directional radiation pattern. When the antenna
cos =
20NP2 is receiving, its radiation pattern is the same.
NP2 = 66.42
Antenna is reciprocal device, whose
Beam which between first nulls = 12.04° characteristics are the same when it is
transmitting or receiving.
4. (a, c)
4 4 8. (d)
Directivity = 2
Ae =
A 2
Polarization loss = cos
A = Beam solid angle
when is angle between received and
Ae = Capture area
transmission E fields of antennas,
Power gain depends on efficiency and losses,
so they may not be equal is right. 2
ET ER
Polarization loss =
5. (d) ET ER

Beam widths depend on the array factor


N
. ET = ( aˆ x + jaˆ y ) circular polarization
2
N ER = ( aˆ x or aˆ y ) linear polarization
When = 2 n for minima condition,
2 2
( aˆ x + jaˆ y )
4n ˆ
a
+ d cos = = Polarization loss = x
N 2
4n 1
4n N = = 50% = 3 dB
N = 2
cos =
d Nd
= 0 for equally excited (in phase) 9. (c)
1 R = 1000 m
cos Gt = Gr = 30 dB = 1000
Nd
BWFN in both arrays is determined by the f = 3 GHz
product (Nd). Pt = 1W

6. (a) C 3 × 1010
= = = 10 cm
f 3 × 109
• Dipole antenna has radiation depending
Power density Pd at 1 km
cos cos Pt Gt 1 × 1000 1
as 2 and maximum at 90° to = 2
= 3 2
=
4 R 4 (10 ) 4000
sin
array axis.
GATE Previous Years Solved Paper 99

Gr 2
1 15. (d)
Pr = ( Ae )r Pd = Intensity of EM wave at 100 m away
4 4000
251
1000(0.1)2 1 = 2
W/m 2
= × = 63.3 µW 4 × 100
4 4000
Effective aperture = 500 cm2 = 500 × 10–4 m2
10. (c) 251 4
Power = × 500 × 10 W
4 × 100 × 100
4
Directivity = (Steradian) = 100 µW
A

3-dB = 2° 16. (b)


41, 253° Diameter of the conductor of the dipole effects
Directivity = only losses and loss resistance,
3-dB 3-dB

41, 253° 1
10, 000 Rl
= A
2.2
Losses reduce when area increases and gain
D = 40 dB
increases. Bandwidth is unaffected as the gain
11. (c) increase is due to loss aspects but not beam angle
For field free space conditions have JC = 0, aspects.
Unlike an amplifier we cannot relate gain ×
× H = JC + JD
bandwidth relationship in antennas.
× H = JC
17. (d)
H =0
To measure the antennas performance in any
So, ×H 0 and H =0 aspect the near field or 1/r2 and 1/r3 have to be
neglected. The radiation field 1/r term should
12. (d)
be strong enough ignoring 1/r2 and 1/r3.
2
dl This distance is called as Fraunhofer zone
R r = 80 (for short loops)
where measurements are carried,

dl 2 2D2
= 20 (for longer loops) r>

Where D is the biggest dimension of the


Rr l 2
antenna,
When turns are increased l increases as Nl
Rr N 2 3 × 108
= = 7.5
R r = 0.25 4 × 109

13. (c) 2 × 2.4 × 2.4


r=
7.5
Power density,
= 1.536 m
E2 (20 × 10 3 )2
Pd = = 18. (b)
120
= 1.061 × 10–6 W/m2 492
= = 123 L
4 4
Pr 2 × 10 6 It is a quarter wave monopole antenna.
Ae = =
Pd 1.06 × 10 6 So, Ra = 36.5
= 1.885 m2
100 Electronics Engineering Electromagnetic Theory

19. (c) Y

max = 60° from end fire means 60° from array


axis.
Condition for maximum, 0
s 2s
sin ( N /2) = 45°
ET = Eo
sin( /2)
s X
ET = NEo = Emax
= + d cos60° = 0
Normalized array factor = 2 cos
2 1 2
+ =0
4 2 = d cos +

= d = 2s, = 45°, = 180°


4
d cos +
2 cos = 2 cos
20. (b) 2 2
Both transmitting and receiving antennas
2 180°
should be oriented correctly for maximum = 2 cos 2 s cos 45° +
2 2
induction in the receiver.
Transmitting antenna is vertical produced E s s
= 2 cos + 90° = 2 sin
field is vertical inducting E field is vertical so
receiving antenna has to be vertical.
23. (b)
21. (b) When an antenna is lossless then the antenna
P efficiency is 100%. In such case whole of the
Signal length = power fed to antenna is radiated in the form of
4 R2
radiation. Therefore, if power fed to antenna is
P P
P1 = and P2 = 1 mW then total radiated power is also 1 mW.
2
4 (5000) 4 ( x )2
24. (c)
P2 1
Given, = (3-dB decrease) H-plane means pattern.
P1 2
Both dipoles are horizontally isotropic in the
2
1 H-plane,
5000
=
x 2
E = 2 Eo cos
2
x = 5000 2
= + d cos
Required distance
2
= + cos
= 5000 2 5000 = 2071 m 2 4

22. (d) + cos


=
2 2
= 90° means XY plane means Z = 1 plane far =0 cos = –1 max = 180°
zone calculations of field at = 0 means point = cos = 1 NP = 0°
on X-axis.
90°
Angle made by the array axis to the point of = cos = 0 HPP =
2 270°
consideration, if taken as .
Option (c) when is considered as shown in
above calculations.
GATE Previous Years Solved Paper 101

Dipole pattern,

D1 D2 Array factor between D1 and D2,


= + d cos
25. (b) d = , = 180° (Image is out of phase)
Gain of antenna is directive gain i.e., different = + 2 cos
from that of amplifier. It radiates same power. Array pattern,
So 1 W. 120° 60°

26. (a)

A mast antenna is a tower antenna or monopole
vertically erected on the ground for low
Resultant pattern can be obtained by
frequency ground wave communication,
multiplication of patterns.
l = 50 m, = 500 m
28. (c)
l= electrically short monopole
10 E-plane mean -plane or vertical plane
2 E sin in Hertzian dipole
2 dl
R r = 20 ×2
Emax when = 90°
This 2 in the multiplication is due to image in Emin when = 0°
conducting earth, E has half power of maximum when = 45°
2 and 135° HPBW in side = 135° – 45° = 90°
2
Rr = 4
5 29. (d)

27. (b) 4 4
Gain = 2
Ae = 2
R2
/2 dipole
Along with illumination or efficiency,
/2
2
2 (2 R)2 D
Gain = 2
=7
Earth
2
A horizontally placed dipole has an image in 1 × 20 × 109
= 0.7 2
the earth and interference with the image 3 × 108
formed gives the final pattern of radiation, = 30705.4359
G (dB) = 10 log10 G
D1
/2
= 10 log10 (30705.4359)
= 45 dB

30. (c)
Earth
Power density of any point source decreases
/2
D2 with distance i.e. the density decreases and area
of crossover increases with the product being
constant.
102 Electronics Engineering Electromagnetic Theory

31. (a) /2
26.5
= ×2 sin 3 d
F( ) = cos4 , 0 2
=0
2
We know that directivity D is, 26.5 2
= ×2 = 55.5 W
4 Umax 2 3
D= ...(i)
Wrad 33. Sol.
F( ) is nothing but radiation intensity u( , ) and Given electric field, E at a distance of
Wrad is radiated power, 1 km = 12 mV/m
2 /2
Wrad = cos 4 sin d d ...(ii) 1
Also we know that E , where r is the
0 0 r
above equation is written from the formula, distance where electric field is to be measured.
Wrad = u( , ) d So, electric field, E at a distance of 2 km
12 mV/m
= = 6 mV/m
where d is solid angle and d = sin d d . 2
So from equation (ii), Also, power density due to the antenna is given
2 /2
as,
Wrad = cos 4 sin d d 1 E2 1 6 × 6 × 10 6
0 0 Pavg = = ×
2 2 120
/2
= 47.7 nW/m2
Wrad = 2 cos 4 sin d
0 34. (b)
/2 • Point electromagnetic source radiates in all
cos 5 2
Wrad = 2 = direction.
5 5
0
• Dish antenna radiates any electromagnetic
4 umax energy in any particular direction with
So, D= = 10 umax = 10
2 narrow beam width and high directivity.
5
• Yagi-uda antenna is high gain antenna
In dB directivity = 10 log10 D = 10 dB used for TV reception. Its radiation is along
the axis of the antenna.
32. Sol.
100
E = Re sin e j r e j t aˆ V/m
r
0.265 j r
H = Re sin e ej t
aˆ V/m
r 35. Sol.
Poynting vector, Both dipole antenna have isotropic pattern in
1 2 2 26.5 view of H-plane view.
P = 2 Re[E × H ] = 2 sin aˆr W/m
2r z
The average power crossing the given
hemispherical shell is,
/2 2 0 y
2 d
Power = P r sin d d aˆr
60° OP >> d
=0 =0
x P
GATE Previous Years Solved Paper 103

= d cos + • Wave contains two orthogonal components


where, = 90° with unequal amplitudes and both are in
2 2 2 phase.
= = =
3 × 10 8 100 • Linear polarization:
6 Polarization loss factor,
3 × 10
Since maximum radiation ( = 0) is at = 60°. 2
PLF = EˆT Eˆ R
2
0 = d cos 60° 90°
100 aˆ x + jaˆ y
where, ET =
360 1 2
90 = d d = 50 m
100 2
aˆ x + 2 aˆ y
ER =
36. (b) 5
Directivity (D) is increased by adding more 2
1 + j2 5 1
antenna elements in an antenna array. Effective PLF = = =
10 10 2
area (Ae) and directivity (D) are related by
2 1
PLF (dB) = 10 log = 3 dB
Ae = D 2
4
D Ae 39. (b)
Power radiated
37. Sol.
= Prad ds
d = 200
TX RX /2 2
1
t = 0.15 r = 0.18 = C0 2 cos 4 r 2 sin d d
18 dB 22 dB =0 =0 r
Pt = 2 W Pr = ?
/2
Gt = 101.8, Gr = 102.2 C0
= 2 2
r cos 4 d ( cos )
2 2 r =0
(1 t ) (1 t ) Gt Gr
Pr = 2
Pt
4 d /2
cos5
= 2 C0
5
0
(1 0.15 2 ) (1 0.18 2 ) 101.8 10 2.2
= 2
×2 2
4 200 = C0 = 1.256 C0
5
4 U( , )
= 2.99476 × 10–3 W Directivity =
= 2.995 mW 3 mW U( , ) d

38. (c, d) 4 C0 cos 4


= /2 2
jkz
ETi = ( aˆ x + jaˆ y ) Eo e C0 cos 4 sin d d
=0 =0
• Wave contains two orthogonal components
and Y component leads X component leads 2 cos 5
by 90° and also wave is travelling in = = 10 cos 5
1/5
negative z-direction, max value = 10
1 jkr max value (in dB) = 10 log10 10 = 10 dB
ER = ( aˆ x + 2 aˆ y ) EI e
r
104 Electronics Engineering Electromagnetic Theory

40. Sol. R2 1.01 l 2


= = 1.0201
f = 5 GHz, G = 150, R = 1 km R1 l
Pt = 100 kW, = 3 m2 Percentage change in radiation resistance
8 2 R2 R1
3 × 10 × 100
150 × 3 =
(G )2 5 × 10 9 R1
Pr = 3 4
× Pt = × 10 5
(4 ) R (4 )3 1012 = 0.0201 × 100 = 2.01%

81 × 3 × 10 5 8 44. (b)
= = 1.224 × 10
(4 )3 × 1012
z
= 0.012 µW

41. (c) Idl


Idl
As per Friis free space propagation equation,
Image
Wt Aer Aet h theory
Wr = h
( d )2
y
when Aer and Aet are doubled and d also doubled,
Infinite conduction plate
Wr is same. Hence capacity is also same. h

42. (a)
Idl
XY plane means Z = 0 with Z = r cos (spherical
coordinates) = 90°.
With null points along the axis of the dipole it Total E = (Esingle element ) (A.F.)
is = 45° and 225°.
y
sin N sin 2
2 2
z (A.F.) = =
null
sin sin
2 2

2 sin cos
2 2
= 45° = = 2 cos
x 2
/2 sin
Dipole 2
+

2 cos
A.F 2
A.FN = = = cos
null
A.Fmax 2 2

where, 2
d cos = (2 h ) cos
43. Sol.
Radiation resistance of a small dipole current 2
A.FN = 60° = cos h cos 60°
element of length ‘l’ is

l 2 h
R rad = 80 2 R l2 = cos

2
cos is maximum, whenever = n , n = 0, 1, 2....
R2 l2
= h
R1 l1 =n h = nl
If length is changed by 1% then percentage For n = 1, hmin =
change in the radiation resistance,
GATE Previous Years Solved Paper 105

45. (0.244) Prad × D × 2


E=
Given: Directive gain, 4 r2
10 log D = 6
D = 100.6 = 3.98 16 × 10 3 × 3.98 × 2 × 120
=
Prad = 16 kW 4 × (8 × 10 3 )2
r = 8 km
= 2
Directivity (or) Directive gain is given by 5.97 × 10 = 0.244 V/m
Amplitude of electric field intensity
Umax r 2 Wrad
D= 4 =4 E = 0.244 V/m
Prad Prad

E2
4 r2 ×
2
=
Prad
GATE-2023
Electronics Engineering

Digital Electronics
Chapterwise & Topicwise

Contents
S.No. Topic Page No.

1. Number Systems ................................................................................................................................... 1-4

2. Boolean Algebra .................................................................................................................................. 5-14

3. Logic Gates ......................................................................................................................................... 15-27

4. Combinational Circuits ................................................................................................................... 28-44

5. Sequential Circuits ........................................................................................................................... 45-69

6. Finite State Machine and Miscellaneous .................................................................................. 70-75

7. Logic Families .................................................................................................................................... 76-88

8. Memories ............................................................................................................................................ 89-92

9. ADC and DAC .................................................................................................................................... 93-98


1 Number Systems

ELECTRO NICS EN GINEERIN G (a) 110100 (b) 001101


(c) 110111 (d) 111101
(GATE Previous Years Solved Papers)
[EC-1998 : 1 Mark]
Q.1 The subtraction of a binary number Y from
Q.5 The 2’s complement representation of –17 is
another binary number X, done by adding the
2’s complement of Y to X, results in a binary (a) 101110 (b) 101111
number without overflow. This implies that the (c) 111110 (d) 110001
result is [EC-2001 : 1 Mark]
(a) negative and is in normal form. Q.6 4-bit 2’s complement representation of a decimal
(b) negative and is in 2’s complement form. number is 1000. The number is
(c) positive and is in normal form. (a) +8 (b) 0
(d) positive and is in 2’s complement form. (c) –7 (d) –8
[EC-1987 : 1 Mark] [EC-2002 : 1 Mark]
Q.2 2’s complement representation of a 16-bit Q.7 The range of signed decimal numbers that can
number (one sign bit and 15 magnitude bits) if be represented by 6-bit 1’s complement number
FFFF. Its magnitude in decimal representation is
is
(a) –31 to +31 (b) –63 to +63
(a) 0 (b) 1
(c) –64 to +63 (d) –32 to +31
(c) 32, 767 (d) 65, 535
[EC-2004 : 1 Mark]
[EC-1993 : 1 Mark]
Q.8 11001, 1001 and 111001 correspond to the 2’s
Q.3 A signed integer has been stored in a byte using
complement representation of which one of the
the 2’s complement format. We wish to store the
following sets of number?
same integer in a 16-bit word. We should
(a) 25, 9 and 57 respectively
(a) copy the original byte to the less significant
(b) –6, –6 and –6 respectively
byte of the word and fill the more significant
byte with zeros. (c) –7, –7 and –7 respectively

(b) copy the original byte to the more significant (d) –25, –9 and –57 respectively
byte of the word and fill the less significant [EC-2004 : 1 Mark]
byte with zeros. Q.9 Decimal 43 in Hexadecimal and BCD number
(c) copy the original byte to the less significant system is respectively
byte of the word and make each bit of the (a) B2, 0100 0011 (b) 2B, 0100 0011
more significant byte equal to the most
(c) 2B, 0011 0100 (d) B2, 0100, 0100
significant bit of the original byte.
[EC-2005 : 1 Mark]
(d) copy the original byte to the less significant
bytes well as the more significant byte of Q.10 A new Binary Coded Pentary (BCP) number
the word. system is proposed in which every digit of a
[EC-1997 : 1 Mark] base-5 number is represented by its
corresponding 3-bit binary code. For example,
Q.4 An equivalent 2’s complement representation
the base-5 number 24 will be represented by its
of the 2’s complement number 1101 is
2 Electronics Engineering Digital Electronics

BCP code 010100. In this numbering system, the Q.14 P, Q and R are the decimal integers
BCP code 100010011001 corresponding to the corresponding to the 4-bit binary number 1100
following number in base-5 system. considered in signed magnitude, 1’s
(a) 423 (b) 1324 complement, and 2’s complement
(c) 2201 (d) 4231 representations, respectively. The 6-bit 2’s
[EC-2006 : 2 Marks] complement representation of (P + Q + R) is
(a) 111101 (b) 110101
Q.11 X = 01110 and Y = 11001 are two 5-bit binary
(c) 110010 (d) 111001
numbers represented in two’s complement
[EC-2020 : 2 Marks]
format. The sum of X and Y represented in 2’s
complement format using 6-bits is Q.15 If (1235)x = (3033)y, where ‘x’ and ‘y’ indicate
(a) 100111 (b) 001000 the bases of the corresponding numbers, then
(c) 000111 (d) 101001 (a) x = 9 and y = 7 (b) x = 8 and y = 6
[EC-2007 : 1 Mark] (c) x = 6 and y = 4 (d) x = 7 and y = 5

Q.12 The two numbers represented in signed 2’s [EC-2021 : 1 Mark]


complement form are: Q.16 Select the Boolean function(s) equivalent to
P = 11101101 and Q = 11100110 x + yz, where x, y and z are Boolean variables,
If Q is subtracted from P, the value obtained in and +denotes logical OR operation.
signed 2’s complement form is
(a) x + z + xy (b) (x + y) (x + z)
(a) 100000111 (b) 00000111
(c) x + xy + yz (d) x + xz + xy
(c) 111111001 (d) 11111001
[EC-2022]
[EC-2008 : 2 Marks]

Q.13 The number of bytes required to represent the


decimal number 1856357 in packed BCD
(Binary Coded Decimal) form is ______ .
[EC-2014 : 1 Mark]
GATE Previous Years Solved Paper 3

Electronics & Electrical Engineering


GATE Previous Years Solved Paper

A n swe rs & Expl a n a t i o n s

Answers
EC Number Systems

1. (b, c) 2. (b) 3. (c) 4. (d) 5. (b) 6. (d) 7. (a) 8. (c)

9. (b) 10. (d) 11. (c) 12. (b) 13. (4) 14. (b) 15. (b) 16. (b, c)

Solutions
EC Number Systems

1. (b, c) Take 2’s complement for magnitude


Negative and is in 2’s complement form. 0111
Positive and is in normal form. 1
+ =8
1000
2. (b)
1
7. (a)
FFFF
1111 1111 1111 1111 Range = – (2n – 1 – 1) to + (2n – 1 – 1)
2’s complement = – (26 – 1 – 1) to + (26 – 1 – 1)
0000 0000 0000 0001 = – 31 to +31
=0001
=1 8. (c)
11001 00111(+7)
3. (c) 1001 0111(+7)
The MSB of the integer in 8-bit formal should be 111001 000111(+7)
repeated to expand the representation of 2’s Number given in question in 2’s
complement form of 16-bit. complement correspond to –7.
4. (b)
9. (b)
According to sign bit extension the sign bit can
be extended towards left. 16 43
(43)10 16 2 B
5. (b) 0 2
17 = 010001
–17 = 101111 (2’s complement) (2B)H (43)10 = (01000011)BCD

6. (d) 10. (d)


100 100010011001 4231
MSB is 1 so, –ve number
4 Electronics Engineering Digital Electronics

11. (c) 14. (b)

X = 01110 Given, binary number 1100


1’s complement of 1100 = –3
Y = 11001
Sign magnitude of 1100 = –4
X+Y =100111
2’s complement of 1100 = –4
Carry is discarded in the addition of numbers
P + Q + R = –4 – 3 – 4
represented in 2’s complement form. X + Y in
= –11
6-bits is 000111.
The 6 digit 2’s complement of (–11) = 110101
12. (b)
415. (b)
Q Signed 2’s complement of
(1235)x = (3033)y
P = 11101101
Convert LHS, RHS into decimal,
No. P = 00010011
1 × x3 + 2 × x2 + 3 × x1 + 5 × x0 = 3 × y3 + 0 + 3 ×
Q Signed 2’s complement of
y1 + 3 × y0
Q = 11100110
(x3 + 2x2 + 3x + 5)10 = (3y3 + 3y + 3)10
P – Q = P + (2’s complement of Q)
Substituting the given options, we find x = 8,
= 00010011 y=6
11100110 i.e. 83 + 2(8)2 + 3(8) + 5 = 3(63) + 3(6) + 3
+
11111001 LHS: 512 + 128 + 24 + 5 = 669 10
2’s complement of RHS: 3(216) + 18 + 3 = 669 10
(P – Q) = 00000111 16. (b, c)
13. Sol. Given, f = x + yz
To represent decimal number into BCD number (a) x + xy + xz = x(1 + y) + xz = x + xz = x
each decimal number is represented in 4-bits (b) (x + y) (x + z) = x + yz
while converting in BCD numbers, as (c) x + xy + yz = x(1 + y) + yz = x + yz
1 0001 6 0110 (d) xy + yz = y(x + z)
8 1000 3 0011 So, option (b, c) are correct.
5 0101 5 0101
7 0111
(1 8 5 6 3 5 7) 0000 0001 1000 0101
1 byte 1 byte

0110 0011 0101 0111


1 byte 1 byte
So, total bytes are required.
2 Boolean Algebra

ELECTRO NICS EN GINEERIN G Q.5 The minimized form of the logical expression

(GATE Previous Years Solved Papers) ( ABC + ABC + ABC + ABC ) is


Q.1 The number of Boolean functions that can be (a) AC + BC + AB
generated by n-variables is equal to
(b) AC + BC + AB
2n 1 2n
(a) 2 (b) 2 (c) AC + BC + AB
(c) 2n – 1 (d) 2n
(d) AC + BC + AB
[EC-1990 : 1 Mark]
[EC-1999 : 2 Marks]
Q.2 Two 2’s complement numbers having sign bits
Q.6 The number of distinct Boolean expressions of
x and y are added and the sign bit of the result
4 variables is
is z. Then, the occurrence of overflow is indicated
by the Boolean function. (a) 16 (b) 256
(c) 1024 (d) 65536
(a) xyz (b) x y z
[EC-2003 : 1 Mark]
(c) x y z + x y z (d) xy + yz + zx
Q.7 If the functions W, X, Y and Z are as follows:
[EC-1998 : 1 Mark]
W = R + PQ + RS
Q.3 The K-map for a Boolean function is shown in
X = PQRS + PQRS + PQRS
figure. The number of essential prime implicants
for this function is Y = RS + PR + PQ + PQ

AB Z = R + S + PQ + PQR + PQS
CD 00 01 11 10
Then,
00 1 1 0 1
(a) W = Z , X = Z (b) W = Z, X = Y
01 0 0 0 1
(c) W = Y (d) W = Y = Z
11 1 0 0 0 [EC-2003 : 2 Marks]
10 1 0 0 1
Q.8 The Boolean expression AC + BC is equivalent
to
(a) 4 (b) 5
(c) 6 (d) 8 (a) AC + BC + AC
[EC-1998 : 1 Mark] (b) BC + AC + BC + ACB
(c) AC + BC + BC + ABC
Q.4 The logical expression y = A + AB is
(d) ABC + ABC + ABC + ABC
equivalent to
[EC-2004 : 2 Marks]
(a) y = AB (b) y = AB
Q.9 The Boolean expression for the truth table
(c) y = A + B (d) y = A + B
shown is
[EC-1998 : 1 Mark]
6 Electronics Engineering Digital Electronics

A B C f Q.14 The Boolean expression:


0 0 0 0
( X + Y ) ( X + Y ) + ( XY ) + X simplifies to
0 0 1 0
0 1 0 0 (a) X (b) Y
0 1 1 1 (c) XY (d) X + Y
1 0 0 0 [EC-2014 : 1 Mark]
1 0 1 0
1 1 0 1 Q.15 Consider the Boolean function:
1 1 1 0 F( w , x , y , z) = wy + xy + wxyz + w xy + xz + x y z

(a) B( A + C ) ( A + C ) (b) B( A + C ) ( A + C ) Which of the following is the complete set of


essential prime implicants?
(c) B( A + C ) ( A + C ) (d) B( A + C ) ( A + C )
(a) w, y , xz, x z (b) w, y, xz
[EC-2005 : 2 Marks]
(c) y , x y z (d) y , xz , x z
Q.10 The Boolean function Y = AB + CD to be realized
using only 2-input NAND gates. The minimum [EC-2014 : 2 Marks]
number of gates required is Q.16 For an n-variables Boolean function, the
(a) 2 (b) 3 maximum number of prime implicants is
(c) 4 (d) 5
n
[EC-2007 : 1 Mark] (a) 2(n – 1) (b)
2
Q.11 The Boolean expression: (c) 2n (d) 2(n – 1)
Y = ABCD + ABCD + ABCD + ABCD [EC-2014 : 1 Mark]
can be minimized to Q.17 The Boolean expression:
(a) Y = ABCD + ABC + ACD
F( X , Y , Z) = X Y Z + X Y Z + XY Z + XYZ
(b) Y = ABCD + BCD + ABCD
converted into Canonical Product of Sum (POS)
(c) Y = ABCD + BCD + ABCD form is
(d) Y = ABCD + BCD + ABCD
(a) (X + Y + Z) (X + Y + Z) ( X + Y + Z) ( X + Y + Z)
[EC-2007 : 2 Marks]
(b) (X + Y + Z) (X + Y + Z) ( X + Y + Z) ( X + Y + Z)
Q.12 If X = 1 in the logic equation,
[ X + Z {Y + (Z + XY }] { X + Z( X + Y )} = 1, then (c) (X + Y + Z) (X + Y + Z) ( X + Y + Z) ( X + Y + Z)

(a) Y = Z (b) Y = Z (d) (X + Y + Z) (X + Y + Z) ( X + Y + Z) ( X + Y + Z)


(c) Z = 1 (d) Z = 0 [EC-2015 : 2 Marks]
[EC-2009 : 2 Marks]
Q.18 A function of Boolean variables, X, Y and Z is
Q.13 In the sum of products function f (X, Y, Z) = expressed in terms of the min-terms as:
(2, 3 , 4, 5), the prime implicants are F(X, Y, Z) = (1, 2, 5, 6, 7)
(a) XY , XY Which one of the product of sums given below
is equal to the function F(X, Y, Z)?
(b) XY , XYZ, XYZ
(a) ( X + Y + Z ) ( X + Y + Z) ( X + Y + Z)
(c) XYZ, XYZ , XY
(d) XYZ , XYZ, XYZ, XYZ (b) ( X + Y + Z ) ( X + Y + Z) ( X + Y + Z)
[EC-2012 : 1 Mark]
GATE Previous Years Solved Paper 7

(c) ( X + Y + Z ) ( X + Y + Z) ( X + Y + Z) (a) A B + A BC + A BC

(X + Y + Z) (X + Y + Z) (b) AC + A B + A BC
(c) AC + A B + A BC
(d) ( X + Y + Z ) ( X + Y + Z ) ( X + Y + Z )
(d) A BC + AC + A BC
(X + Y + Z) (X + Y + Z)
[EC-2017 : 2 Marks]
[EC-2015 : 2 Marks]
Q.21 A function F(A, B, C) defined by three Boolean
Q.19 Following is the K-map of a Boolean function of variables A, B and C when expressed as sum of
5 variables P, Q, R, S and X. The minimum sum products is given by
of product (SOP) expression for the function is
F= A B C+A B C+A B C
PQ
RS 00 01 11 10 where, AB and C are the complements of the
00 0 0 0 0 respective variables. The product of sums (POS)
form of the function F is
01 1 0 0 1
(a) F = ( A + B + C ) ( A + B + C ) ( A + B + C )
11 1 0 0 1
(b) F = ( A + B + C ) ( A + B + C ) ( A + B + C )
10 1 0 0 1 (c) F = ( A + B + C ) ( A + B + C ) ( A + B + C )
X=0
( A + B + C) (A + B + C)

PQ (d) F = ( A + B + C ) ( A + B + C ) ( A + B + C )
RS 00 01 11 10
(A + B + C) (A + B + C)
00 0 1 1 0
[EC-2018 : 1 Mark]
01 0 0 0 0
ELECTRICAL EN GINEERIN G
11 0 0 0 0
(GATE Previous Years Solved Papers)
10 0 1 1 0

X=1 Q.1 The logic function, f = ( x y ) + ( x y ) is the


same as,
(a) PQ S X + PQS X + Q RS X + Q RS X
(a) f = (x + y ) (x + y )
(b) Q S X + QS X (b) f = (x + y ) ( x + y )
(c) Q S X + QS X (c) f = ( x y ) (x y )
(d) Q S + QS (d) none of the above

[EC-2016 : 2 Marks] [EE-1992 : 2 Marks]

Q.20 Which one of the following gives the simplified Q.2 The minimal product of sums function
sum of products expression for the Boolean described by the K-map given in figure, is
AB
function, F = m0 + m2 + m3 + m5, where m0, m2, m3 00 01 11 10
C
and m5 are minterms corresponding to the
0 1 1 0 0
inputs A, B and C with A as the MSB and C as
the LSB? 1 0 0 0 0
8 Electronics Engineering Digital Electronics

(a) A C (b) A + C
(c) X
(c) A + C (d) AC
F
[EE-2000 : 2 Marks] Y
Z
Q.3 The Boolean expression:
X Y Z + X Y Z + X Y Z + X Y Z + XYZ (d) X
F
can be simplified to Y
Z
(a) XZ + XZ + YZ (b) XY + YZ + YZ

(c) XY + YZ + XZ (d) XY + YZ + XZ [EE-2010 : 2 Marks]

[EE-2003 : 2 Marks] Q.7 In the sum of products function:


f(X, Y, Z) = (2, 3, 4, 5)
Q.4 The simplified form of the Boolean expression
The prime implicants are
Y = ( ABC + D) ( AD + BC ) can be written as,
(a) X Y + X Y
(a) AD + BCD (b) AD + BCD (b) X Y + X Y Z + X Y Z

(c) ( A + D) ( BC + D) (d) AD + BCD (c) X Y Z + X Y Z + X Y


[EE-2004 : 2 Marks] (d) X Y Z + X Y Z + X Y Z + X Y Z
Statement for Linked Answer Questions (5 and 6): [EE-2012 : 1 Mark]
The following Karnaugh map represents a function F. Q.8 Which of the following is an invalid state in
F YZ 8-4-2-1 binary code decimal counter?
00 01 11 10
(a) 1 0 0 0 (b) 1 0 0 1
0 1 1 1 0 (c) 0 0 1 1 (d) 1 1 0 0
X
[EE-2014 : 1 Mark]
1 0 0 1 0
Q.9 The SOP (Sum of Products) form of a Boolean
function is (0, 1, 3, 7, 11), where inputs are A,
Q.5 A minimized form of the function F is
B, C, D(A is MSB and D is LSB). The equivalent
(a) F = X Y + YZ (b) F = X Y + YZ minimized expression of the function is

(c) F = X Y + Y Z (d) F = X Y + Y Z (a) ( B + C ) ( A + C ) ( A + B) (C + D)


[EE-2010 : 2 Marks] (b) ( B + C ) ( A + C ) ( A + C ) (C + D)
Q.6 Which of the following circuits is a realization
(c) ( B + C ) ( A + C ) ( A + C ) (C + D)
of the above function F ?
(d) ( B + C ) ( A + B) ( A + B) (C + D)
(a) X
[EE-2014 : 2 Marks]
F
Y
Z Q.10 f(A, B, C, D) = M(0, 1, 3, 4, 5, 7, 9, 11, 12, 13, 14, 15)
is a maxterm representation of Boolean function
(b) X
f(A, B, C, D) where A is the MSB and D is the
F LSB. The equivalent minimized representation
Y
Z of this function is
GATE Previous Years Solved Paper 9

Q.15 The output expression for the Karnaugh map


(a) ( A + C + D) ( A + B + D)
shown below is
(b) AC D + A BD CD
AB 00 01 11 10
(c) ACD + ABCD + ABCD
00 0 0 0 0
(d) ( B + C + D) ( A + B + C + D) ( A + B + C + D)
01 1 0 0 1
[EE-2015 : 2 Marks]
11 1 0 1 1
Q.11 Consider the following sum of products
expression, F : 10 0 0 0 0

F = ABC + A BC + A BC + A BC + A BC
(a) BD + BCD (b) BD + AB
The equivalent product of sums expression is
(c) BD + ABC (d) BD + ABC
(a) F = ( A + B + C ) ( A + B + C ) ( A + B + C )
[EE-2017 : 2 Marks]
(b) F = ( A + B + C ) ( A + B + C ) ( A + B + C )
Q.16 Digital input signals A, B, C with A as the MSB
(c) F = ( A + B + C ) ( A + B + C ) ( A + B + C )
and C as the LSB are used to realize the Boolean
(d) F = ( A + B + C ) ( A + B + C ) ( A + B + C ) function F = m0 + m2 + m3 + m5 + m7, where mi
[EE-2015 : 1 Mark] denotes the Ith minterm. In addition, F has a
don’t care for m1. The simplified expression for
Q.12 The output expression for the Karnaugh map
F is given by
shown below is
BC (a) AC + BC + AC (b) A + C
A 00 01 11 10
(c) C + A (d) AC + BC + AC
0 1 0 0 1
[EE-2018 : 2 Marks]
1 1 1 1 1
Q.17 The output expression for the Karnaugh map
shown below is
(a) A + B (b) A + C
PQ
(c) A + C (d) A + C RS 00 01 11 10
[EE-2016 : 1 Mark] 00 0 1 1 0

Q.13 The Boolean expression: 01 1 1 1 1


( a + b + c + d ) + (b + c )
11 1 1 1 1
simplifies is
10 0 0 0 0
(a) 1 (b) a b
(c) a, b (d) 0
(a) QR + S (b) QR + S
[EE-2016 : 2 Marks]
(c) QR + S (d) QR + S
Q.14 The Boolean expression, AB + AC + BC
simplifies to [EE-2019 : 2 Marks]

(a) BC + AC (b) AB + AC + B
(c) AB + AC (d) AB + BC
[EE-2017 : 1 Mark]
10 Electronics Engineering Digital Electronics

Electronics & Electrical Engineering


GATE Previous Years Solved Paper

A n swe rs & Expl a n a t i o n s

Answers
EC Boolean Algebra

1. (b) 2. (c) 3. (a) 4. (d) 5. (a) 6. (d) 7. (a) 8. (d)

9. (a) 10. (b) 11. (d) 12. (d) 13. (a) 14. (a) 15. (d) 16. (d)

17. (a) 18. (b) 19. (b) 20. (b) 21. (c)

Solutions
EC Boolean Algebra

1. (b) 5. (a)
n Y = A BC + A BC + A BC + A BC
Boolean functions possible are 2 2 .
= AC ( B + B) + A BC + A BC
2. (c)
The condition for overflow to occur is = A (C + BC ) + A BC

x y z + x y z. = A (C + C ) (C + B) + A BC

= AC + A B + A BC
3. (a)
= AC + B ( A + AC )
AB
00 01 11 10 = AC + B ( A + A) ( A + C )
CD
00 1 1 1 = AC + A B + BC

01 1 6. (d)
n 4
11 1 22 = 2 2 = 2 16 = 65536
10 1 1 7. (a)

RS
Number of essential prime implicants = 4. 1. W = PQ 00 01 10 11

00 1 1 1
4. (d)
01 1 1 1 1
y = A + AB
11 1 1 1
= ( A + A) ( A + B) 10 1 1 1
= A+B
W = R + PQ + RS
GATE Previous Years Solved Paper 11

9. (a)
2. X= RS
PQ 00 01 11 10
f = A BC + A BC
00 1
= B ( AC + AC )
01 = B( A + C) ( A + C)
11 1
10. (b)
10 1 y = AB + CD = A B + C D
A
We can see that, X = B
AB + CD
RS
3. Y= 00 01 11 10
PQ
C
00 1 D

01 1 1 1 1 11. (d)
11 1 1 1 K-map corresponding to given Boolean
expression,
10 1
CD
AB 00 01 11 10
RS 00 1
4. Z= 00 01 11 10
PQ
00 1 1 1 01 1

11 1
01 1 1 1 1
10 1
11 1 1 1

10 1 1 1 Simplified expression from the K-map,


Y = A BC D + A BC D + BC D
K-map(1) = K-map(4)
W=Z 12. (d)
From map (2) and (4),
[ X + Z {Y + (Z + XY )}] [ X + Z( X + Y )] = 1
X= Z
Since, X = 1 so we get
8. (d) Z (1 + Y ) = 1

AC + BC Z =1
Writing the SOP form of the above expression Z=0
and filling the K-map, we get, 13. (a)
BC
00 01 11 10 f (X, Y, Z) = (2, 3, 4, 5)
A
YZ YZ YZ YZ
0 1
X 1 1
1 1 1 1
X 1 1
Now, comparing it which the expression given
in the question, we find option (d) satisfies the f (X, Y, Z) = X Y + X Y
given condition. So, prime implicants are X Y and X Y .
12 Electronics Engineering Digital Electronics

14. (a) 19. (b)

Let, F = (X + Y ) (X + Y ) + (X Y + X ) PQ
RS
F = X + X Y + XY + ( X Y X )
= X + (X + Y ) X 1 1
= X + XY
1 1
=X

15. (d)
X=0
f ( w , x , y , z ) = wy + xy + w xyz + w x y + xz + x y z QSX

PQ
f (wxyz)
RS
yz yz yz yz
1 1
wx 1 1 1 1

wx 1 1 1 1

wx 1 1 1
1 1

wx 1 1 1 X=1
QSX

minimum sum of product expression of the


y , xz , x z
function is
From the above K-map, it can be seen that, the
set of essential prime-implicants is = Q S X + QS X

y , xz , x z 20. (b)

16. (d) Given Boolean function is,


F = m0 + m2 + m3 + m5
For a variable Boolean function the maximum
It can be minimized by using K-map as given
number of prime implicants is 2(n – 1).
below.
17. (a) BC
A BC BC BC BC
F(X, Y, Z) = X Y Z + X YZ + XY Z + XYZ 0 1 3 2
A 1 1 1 AB
YZ AC
00 01 11 10 4 5 7 6
X A 1
0 0 0 0 1
ABC
1 1 1 0 1
F = AC + A B + A BC
F= ( X + Y + Z ) (X + Y + Z) ( X + Y + Z) (X + Y + Z)
21. (c)
18. (b)
F(A, B, C, D) = A BC + A BC + A BC
F(X, Y, Z) = (1, 2, 5, 6, 7)
= m(0, 2, 4) = M(1, 3, 5, 6, 7)
= [0, 3, 4]
= (A + B + C) (A + B + C) (A + B + C)
= ( X + Y + Z ) ( X + Y + Z) ( X + Y + Z)
( A + B + C) (A + B + C)
GATE Previous Years Solved Paper 13

Answers
EE Boolean Algebra

1. (b) 2. (a) 3. (b) 4. (a) 5. (b) 6. (d) 7. (a) 8. (d)

9. (a) 10. (c) 11. (a) 12. (b) 13. (d) 14. (a) 15. (d) 16. (b)

17. (b)

Solutions
EE Boolean Algebra

1. (b) 5. (b)
Given, XY
f = ( x y ) + (x y ) = (x y ) (x y ) YZ
X 00 01 11 10
= ( x + y ) ( x + y ) = xy + x y
0 1 1 1 0
Option (b):
( x + y ) (x + y ) = ( x + y ) + (x + y ) 1 0 0 1 0

= xy + x y = f YZ

2. (a) F = XY +Y Z
AB
C 00 01 11 10 6. (d)
A From the figure it is clear that, two NAND gates
0 1 1 0 0
generate the X and Y and now two AND gates
1 0 0 0 0
with inputs X and Y and inputs Y and Z is
C used to generate two terms of SOP form and now
OR gate is used to sum them and generate the F.
3. (b)
By K-map: 7. (a)

XY f(X, Y, Z) = (2, 3, 4, 5)
Z 00 01 11 10
YZ
X 00 01 11 10
0 1 1
0 1 1 XY
1 1 1 1
1 1 1

The simplified form is XY + Y Z + Y Z .


XY
4. (a)
f(X, Y, Z) = X Y + X Y
Y = ( A BC + D) ( A D + BC )

= A BCD + AD + BC D 8. (d)
Binary coded decimal counter counts from 0 to 9.
= ( AD + BC D)
So, 1100 is an invalid state i.e. 12.
14 Electronics Engineering Digital Electronics

9. (a) = ( a + b + c + d ) (b + c )
The 4 variable Boolean function is given in
= a b c d b c
canonical sum of products form as, F=0
f (A, B, C, D) = (0, 1, 3, 7, 11)
As the options are given in the simplified 14. (a)
product of sums form, we first convert the given BC BC BC BC
function in canonical product of sums form, as
A 1
under: 0 1 3 2

f (A, B, C, D) = (2, 4, 5, 6, 8, 9, 10, 12, 13, 14, 15) A 1 1 1


4 5 7 6
Now by plotting the above function on a 4
variable K-map (maxterms map), we obtain the AC + BC
simplified expression of the function,
BC + AC
CD
AB C+D C+D C+D C+D 15. (d)
0 1 3 2
A+B 0 C+D CD CD CD CD
4 5 7 6
AB
A+B 0 0 0
B+C
12 13 15 14 AB 1 1
A+B 0 0 0 0 A+B
AB 1 1 1
8 9 11 10
A+C
A+B 0 0 0
AB

f = ( B + C ) ( A + C ) ( A + B) (C + D) ABC + BD

11. (a) 16. (b)


The SOP form of F is (shown in K-map), Given, f = m0 + m2 + m3 + m5 + m7
BC and m1 = don’t care
A 00 01 11 10
BC BC BC BC
0 1 1 1 0
A 1 X 1 1
1 0 1 1 0 0 1 3 2

A 1 1
So, POS form can be formed using ‘0’ from the 4 5 7 6

K-map. f = A +C
POS = ( A + B + C ) ( A + B + C ) ( A + B + C )
17. (b)
12. (b) PQ
RS 00 01 11 10
BC
A 00 01 11 10 00 0 1 1 0

0 1 0 0 1 01 1 1 1 1

1 1 1 1 1 11 1 1 1 1

10 0 0 0 0
F = A +C

13. (d) Output = QR + S


Option (b) is correct.
F = ( a + b + c + d ) + (b + c )
3 Logic G ates

ELECTRO NICS EN GINEERIN G Q.5 For the logic circuit shown in figure, the output
is equal to
(GATE Previous Years Solved Papers)

Q.1 For the circuit shown below the output F is given


by A

X
B Y
F

(a) F = 1 (b) F = 0 (a) ABC (b) A + B + C


(c) F = X (d) F = X
(c) AB + BC + A + C (d) AB + BC
[EC-1988 : 1 Mark]
[EC-1993 : 1 Mark]
Q.2 Minimum number of 2-input NAND gates
Q.6 The output of a logic gate is ‘1’ when all its a
required to implement the function,
inputs are at logic ‘0’. The gate is either
F = ( X + Y ) (Z + W ) is (a) a NAND or an EX-OR gate
(a) 3 (b) 4 (b) a NOR or an EX-NOR gate
(c) 5 (d) 6 (c) an OR or an EX-NOR gate
[EC-1988 : 1 Mark] (d) an AND or an EX-OR gate
[EC-1994 : 1 Mark]
Q.3 Indicate which of the following logic gates can
be used to realized all possible combinational Q.7 A ring oscillator consisting of 5 inverters is
logic functions: running at a frequency of 1.0 MHz. The
(a) OR gates only propagation delay per gate is ______ n-sec.
(b) NAND gates only [EC-1994 : 1 Mark]
(c) EX-OR gates only
Q.8 The output of the circuit shown in figure is equal
(d) NOR gates only to
[EC-1989 : 1 Mark]
A
Q.4 Boolean expression for the output of XNOR B
(equivalence) logic gate with inputs A and B is
(a) A B + A B
A
(b) A B + A B B

(c) ( A + B) ( A + B) (a) 0 (b) 1


(d) ( A + B) ( A + B) (c) A B + A B (d) ( A B) (A B)
[EC-1993 : 1 Mark] [EC-1995 : 1 Mark]
16 Electronics Engineering Digital Electronics

Q.9 The minimum number of NAND gates required


A
to implement the Boolean function B
A + A B + A BC is equal to
(a) zero (b) 1 X
C
(c) 4 (d) 7
[EC-1995 : 1 Mark]
(a) 1, 0, 1 (b) 0, 0, 1
Q.10 The output of the logic gate in figure is (c) 1, 1, 1 (d) 0, 1, 1
A [EC-2000 : 1 Mark]
F
Q.15 For the logic circuit shown in the figure, the
simplified Boolean expression for theoutput Y is
(a) 0 (b) 1
A
(c) A (d) A B

[EC-1997 : 1 Mark]
Y
Q.11 The Boolean function A + BC is a reduced form
of C

(a) AB + BC (b) (A + B) (A + C) (a) A + B + C (b) A


(c) A B + A BC (d) (A + C) B (c) B (d) C

[EC-1997 : 1 Mark] [EC-2000 : 2 Marks]

Q.12 The minimum number of 2-input NAND gates Q.16 In the figure, the LED
required to implement the Boolean function
VCC = 5 V
Z = ABC , assuming that A, B and C are
available, is 1k 1k 1k LED
(a) two (b) three
(c) five (d) six 1k

[EC-1998 : 1 Mark]
S1 S2
Q.13 For the identity AB + AC + BC = AB + AC , the
dual form is
(a) ( A + B) ( A + C ) ( B + C ) = ( A + B) ( A + C )
(a) emits light when both S1 and S2 are closed.
(b) ( A + B) ( A + C ) ( B + C ) = ( A + B) ( A + C ) (b) emits light when both S1 and S2 are open.
(c) emits light when only of S1 and S2 is closed.
(c) ( A + B) ( A + C ) ( B + C ) = ( A + B) ( A + C )
(d) does not emit light, irrespective of the switch
(d) A B + AC + BC = A B + AC positions.
[EC-1998 : 1 Mark] [EC-2001 : 2 Marks]

Q.14 For the logic circuit shown in the figure, the Q.17 If the input to the digital circuit (in the figure)
required input condition (A, B, C) to make the consisting of a cascade of 20 XOR-gates is X,
output (X) = 1 is then the output Y is equal to
GATE Previous Years Solved Paper 17

1 Q.20 The number of product terms in the minimized


sum-of-product expression obtained through
Y
the following K-map is (where, “d” denotes
X don’t care states)
(a) 0 (b) 1
1 0 0 1
(c) X (d) X
0 d 0 0
[EC-2002 : 1 Mark]
0 0 d 1
Q.18 The gates G 1 and G 2 in the figure have
propagation delays of 10 n-sec and 20 n-sec 1 0 0 1
respectively. If the input Vi makes an abrupt
(a) 2 (b) 3
change from logic 0 to 1 at time t = t0, then the
output waveform Vo is (c) 4 (d) 5
[EC-2005 : 2 Marks]
G1
0 G2
Vo
Q.21 Which of the following Boolean expressions
1
1
correctly represents the relation between P, Q, R
vi vi
and M1?
0 Vi
t0
P
X
1 Q
Z
(a)
0 M1
t0 t1 t2 t3 Y

1 R
(b)
0 (a) M1 = (P OR Q) XOR R
t0 t1 t2 t3
(b) M2 = (P AND Q) XOR R
1 (c) M1 = (P NOR Q) XOR R
(c)
0 (d) M1 = (P XOR Q) XOR R
t0 t1 t2 t3
[EC-2008 : 2 Marks]
1
(d) Q.22 Match the logic gates in Column-A with their
0
t0 t1 t2 t3 equivalent in Column-B:
Column-A Column-B
[EC-2002 : 2 Marks]
P. 1.
Q.19 A Boolean function ‘f ’ of two variables x and y
is defined as follows:
Q. 2.
f(0, 0) = f(0, 1) = f(1, 1) = 1; f(1, 0) = 0
Assuming complements of x and y are not
R. 3.
available, a minimum cost solution of realizing
‘f ’ using only 2-input NOR gates and 2-input
S. 4.
OR gates (each having unit cost) would have a
total cost of (a) P-2, Q-4, R-1, S-3
(a) 1 unit (b) 4 unit (b) P-4, Q-2, R-1, S-3
(c) 3 unit (d) 2 unit (c) P-2, Q-4, R-3, S-1
[EC-2004 : 2 Marks] (d) P-4, Q-2, R-3, S-1 [EC-2010 : 1 Mark]
18 Electronics Engineering Digital Electronics

Q.23 For the output ‘F ’ to be 1 is the logic circuit XOR


shown, the input combination should be X
Y
A AND
B F

F Z
XNOR
C
(a) F = X Y Z + X Y Z
(a) A = 1, B = 1, C = 0
(b) A = 1, B = 0, C = 0 (b) F = X Y Z + X Y Z

(c) A = 0, B = 1, C = 0 (c) F = X Y Z + X Y Z
(d) A = 0, B = 0, C = 1
(d) F = X Y Z + X Y Z
[EC-2010 : 1 Mark]
[EC-2014 : 2 Marks]
Q.24 The output Y in the circuit below is always ‘1’
when Q.27 In the circuit shown in the figure, if C = 0, the
expression for Y is
P
C
A
Q
Y B

Y
R
A
(a) two or more of the inputs P, Q, R are ‘0’. B

(b) two or more of the inputs P, Q, R are ‘1’.


(a) Y = AB + AB (b) Y = A + B
(c) any odd number of the inputs P, Q, R is ‘0’.
(c) Y = A + B (d) Y = AB
(d) any odd number of the inputs P, Q, R is ‘1’.
[EC-2014 : 1 Mark]
[EC-2011 : 1 Mark]
Q.28 A 3-input majority gate is defined by the logic
Q.25 A bulb in a stair case has two switches, one
function M(a, b, c) = ab + bc + ca. Which one of
switch being at the ground floor and the other
the following gate is represented by the function,
one at the first floor. The bulb can be turned-ON
and also can be turned-OFF by any one of the M ( M ( a, b , c ), M( a , b , c ), c ) ?
switches irrespective of the state of the other (a) 3-input NAND gate
switch. The logic of switching of the bulb
(b) 3-input XOR gate
resembles
(c) 3-input NOR gate
(a) an AND gate
(d) 3-input XNOR gate
(b) an OR gate
[EC-2015 : 2 Marks]
(c) an XOR gate
(d) a NAND gate Q.29 All the logic gates shown in the figure have a
[EC-2013 : 1 Mark] propagation delay of 20 ns. Let A = C = 0 and
B = 1 unit time t = 0. At t = 0, all the inputs flip
Q.26 The output F in the digital logic circuit shown (i.e. A = C = 1 and B = 0) and remain in that state.
in the figure is For t > 0, output Z = 1 for a duration (in ns) of
GATE Previous Years Solved Paper 19

A Q.33 The minimum number of 2-input NAND gates


B Z required to implement a 2-input XOR gate is
C (a) 4 (b) 5
[EC-2015 : 2 Marks] (c) 6 (d) 7
[EC-2016 : 1 Mark]
Q.30 In the figure shown, the output Y is required to
Q.34 Consider a Boolean gate (D) where the output Y
be Y = AB + C D. The gates G1 and G2 must be
is related to the inputs A and B as, Y = A + B ,
A where + denotes logical OR operation. The
G1
B Boolean inputs ‘0’ and ‘1’ are also available
G2 Y
C
separately. Using instances of only D gates and
D inputs ‘0’ and ‘1’, ______ (select the correct
(a) NOR, OR (b) OR, NAND option(s)).
(c) NAND, OR (d) AND, NAND (a) NAND logic can be implemented
[EC-2015 : 1 Mark] (b) OR logic cannot be implemented
(c) NOR logic can be implemented
Q.31 A universal logic gate can implement any
(d) AND logic cannot be implemented
Boolean function by connecting sufficient
[EC-2022]
number of them appropriately. Three gates are
shown.
ELECTRICAL EN GINEERIN G
F1 = X + Y X F2 = X Y
X (GATE Previous Years Solved Papers)
Y Y
Gate 1 Gate 2
Q.1 The Boolean expression of the output of the
F3 = X + Y logic circuit shown in figure is
X
Y A
Gate 3
B
Which one of the following statements is true?
(a) Gate 1 is a universal gate. Y
(b) Gate 2 is a universal gate. C

(c) Gate 3 is a universal gate. (a) Y = A B + AB + C


(d) None of the gates shown is a universal gate.
(b) Y = A B + AB + C
[EC-2015 : 2 Marks]
(c) Y = A B + A B + C
Q.32 The output of the combinational circuit given
below is (d) Y = A B + A B + C

A [EE-1996 : 1 Mark]

Q.2 The output of a logic gate is “1” when all its


B
C inputs are at logic “0”. The gate is either
Y
(a) a NAND or an EX-OR gate
(b) a NOR or an EX-OR gate
(a) A + B + C (b) A(B + C) (c) an AND or an EX-NOR gate
(c) B(C + A) (d) C(A + B) (d) a NOR or an EX-NOR gate
[EC-2016 : 1 Mark] [EE-2001 : 1 Mark]
20 Electronics Engineering Digital Electronics

Q.3 For the circuit shown in figure the Boolean Q.6 The output Y of the logic circuit given below is
expression for the output Y in terms of inputs P,
Y
Q, R and S.

P (a) 1 (b) 0
(c) x (d) x
Q [EE-2011 : 1 Mark]
Y Q.7 A bulb in a staircase has two switches, one
R
switch being at the ground floor and the other
one at the first floor. The bulb can be turned ON
S
and also can be turned OFF by one of the
switches irrespective of the state of the other
(a) P + Q + R + S (b) P + Q + R + S switch. The logic of switching of the bulb
resembles
(c) ( P + Q) + ( R + S ) (d) (P + Q) + (R + S) (a) and AND gate (b) an OR gate

[EE-2002 : 2 Marks] (c) an XOR gate (d) a NAND gate


[EE-2013 : 1 Mark]
Q.4 A, B, C and D are input, and Y is the output bit
in the XOR gate circuit of the figure below. Which Q.8 Which of the following logic circuits is a
of the following statements about the sum S of realization of the function F whose Karnaugh
A, B, C, D and Y is correct? map is shown in figure?
AB
A C 00 01 11 10
XOR
B 0 1 1

XOR Y
1 1 1
C
XOR
D
A
(a) S is always either zero or odd.
(a)
(b) S is always either zero or even.
B
(c) S = 1 only if the sum of A, B, C and D is C
even.
(d) S = 1 only if the sum of A, B, C and D is odd. A
[EE-2007 : 1 Mark]
(b)
Q.5 The complete set of only those logic gates B
designated as universal gates is C
(a) NOT, OR and AND gates
(b) XNOR, NOR and AND gates A
(c) NOR and NAND gates
(d) XOR, NOR and NAND gates (c) C

[EE-2009 : 1 Mark]
B
GATE Previous Years Solved Paper 21

(a) Y = ABCD
A
C (b) Y = (A + B) (C + D)

(d) (c) Y = A + B + C + D
(d) Y = AB + CD

B
[EE-2018 : 1 Mark]

[EE-2014 : 2 Marks] Q.11 In the circuit shown below, X and Y are digital
inputs, and Z is a digital output. The equivalent
Q.9 For a 3 input logic circuit shown below, the
circuit is
output Z can be expressed as
P X

Y
Q Z
Z

(a) Q + R (b) PQ + R

(c) Q + R (d) P + Q + R
(a) XNOR gate (b) NOR gate
[EE-2017 : 1 Mark] (c) NAND gate (d) XOR gate
Q.10 In the logic circuit shown in the figure, Y is [EE-2019 : 2 Marks]
given by
A
B
Y
C
D

Electronics & Electrical Engineering


GATE Previous Years Solved Paper

A n swe rs & Expl a n a t i o n s

Answers
EC Logic G ates

1. (b) 2. (b) 3. (b, d) 4. (b, c) 5. (b) 6. (b) 7. (100) 8. (b)

9. (a) 10. (c) 11. (b) 12. (c) 13. (a) 14. (d) 15. ( ) 16. (d)

17. (b) 18. (b) 19. (d) 20. (a) 21. (d) 22. (d) 23. (d) 24. (b)

25. (c) 26. (a) 27. (a) 28. (b) 29. (40) 30. (a) 31. (c) 32. (c)

33. (a) 34. (a, c)


22 Electronics Engineering Digital Electronics

Solutions
EC Logic G ates

1. (b) 7. Sol.
Output of 1st EX-OR gate, 1
F1 = X X = 0 fclk =
2 Nt pdFF
Output of 2nd EX-OR gate,
F2 = X 0 = X 1 1
tpdFF = =
Output of 3rd EX-OR gate, 2 Ntclk 2 × 5 × 106
F = X X=0 = 100 nsec

2. (b) 8. (b)

W XY W Y = (A B) ( A B)
F = XYZ + XYW
= (X + Y) (Z + W) Y= A B A B
X XY
Y Y = (A A) ( B B)
XY Z
Y=0 0=1
Z
9. (a)
3. (b, d)
NAND and NOR gates can be used to realize F = A + A B + A BC
all possible combinational logic functions. = A (1 + B + BC ) = A
4. (b, c) So to implement A, zero NAND gates are
required.
A B + AB and ( A + B) ( A + B)
10. (c)
5. (b) F=A 0

A = A 0 + A 0 = A 1+0

F= A
A AB
11. (b)
AB + BC
B Y (A + B) (A + C) = A A + A C + A B + BC
= A + AC + AB + BC
C BC = A (1 + C + B) + BC
= A + BC

12. (c)
Y = A + AB + BC + C
A
Y = A + ( A + B) + ( B + C ) + C AC AC
C
Y = A+ B +C AC B ABC

6. (b)
B
B
NOR gate F = A + B = AB

EX-NOR gate F=A B = A B + AB


GATE Previous Years Solved Paper 23

13. (a) 18. (b)


AND OR
OR AND
t0 t1 t2 t3 t4 t5
14. (d)
For X = 1
Output
A B=1 i.e. A = 0, B = 1
or NOR t0 t1 t2 t3
B C=1 i.e. B = 1, C = 1
Vo
15. ( )
t2 t3
A P
B
Z 19. (d)

X x y f y
x 0 1
Y 0 0 1
0 1 1 0 1 1
C D =x+y
1 0 0
1 1
Y = X+D 1 1 1

Y = X+D Since complements are not available.


y
D = (Z + C ) = ZC
f
X = (Z B ) = Z + B x
x

Y = Z + B + ZC 2 units.

Y = Z+B 20. (a)


Z = P B= P+B
Y = ( P + B) + B 1 0 0 1

P = AB 0 d 0 0
Y = P B+ B = B+ P
0 0 d 1
= B + AB = B + A + B = 1
Y=0 1 0 0 1

16. (d)
For LED to be ON, output of NAND gate = 0 21. (d)
No condition of S1 and S2 gives output of NAND M1 = [ PQ ( P + Q] R
gate zero. So LED will never glow.
= [( P + Q) ( P + Q] R
17. (b)
= (P Q) R
Output of 1st XOR gate = X
22. (d)
Output of 2nd XOR gate = X X
NOR gate is equivalent to INVERT – AND gate.
= (X ) X + X X = X + X = 1 NOR gate is equivalent to INVERT – OR gate.
Output of 20 XOR gates is 1.
24 Electronics Engineering Digital Electronics

X 26. (a)
R: Y F F = XY + XY
XOR
X X
3: F F = XY + XY Y
Y
A AND
F = XY + XY F
X
S: Y F F = XY + X Y
B
Z
X
1: Y F F = XY + XY XNOR

A=X Y
F = XY + X Y
B=A Z = A Z + AZ
23. (d) = Z (X Y ) + (X Y) Z
A X F = AB
B
= (X Y )[Z ( X Y ) + Z (X Y )]
Y
F = Z [( X Y ) (X Y ) + A (X Y ) (X Y )]

As we know,
C
A A=A
If, A = 0, B = 0 then
A A =0
X = AB+ AB = 0
= Z [(X Y) + 0]
Y = AB + A B = 1
= X YZ + X Y Z
F will be ‘1’ if even number of inputs to XNOR
gate is ‘1’, hence option (d) is the correct answer. 27. (a)
24. (b)
1
P PQ
C

PQ RQ PQ RQ A
PQ RQ PR 2
B
Q Y
RQ 3 6 Y

R PR A 5
4
B

Y = PQ + PR + RQ
Output of gate 1 : C
25. (c)
Output of gate 2 : ( A + B)
Truth table of XOR gate:
Output of gate 3 : (A + B + C)
A B Y
0 0 0 Output of gate 4 : AB
0 1 1 Output of gate 5 : ( A + B + C ) + AB
1 0 1
1 1 0 Output of gate 6 is output Y i.e.,

So, from the XOR gate truth table it is clear that Y = C ( A + B + C ) + AB


the bulb can be turned-ON and also can be
= C + ( A + B + C + AB)
turned-OFF by any one of the switches
irrespective of the state of the other switch.
GATE Previous Years Solved Paper 25

Using Demorgon’s theorem


Y = AB + C D
= C + ( A + B) C ( AB)
G1 = NOR Gate A NOR B = AB
= C + ( A + B) C ( AB) G2 = OR Gate AB + C D
Given in question C = 0.
31. (c)
So, Y = 0 + ( A + B) 0 ( A + B)
Universal gate is a gate by which every other
= AB+ AB
gate can be realized.
28. (b) Gate 1 and Gate 2 are basic gates and can not be
used as universal gates.
M(a, b, c) = ab + bc + ca
M ( a , b , c ) = ab + bc + ca 32. (c)
y = ABC AB BC
= ab bc ca
= [ ABC AB + ABC AB] BC
= ( a + b ) (b + c ) ( c + a )
M ( a , b , c ) = ab + b c + c a = [( A + B + C ) AB + ABC ( A + B)] BC

= ( A BC ) ( BC )
M ( M ( a , b , c ), M( a, b , c ) c ) =
( ab bc ca ) ( ab + bc + ca ) + ( ab + bc + ca ) (c ) + ( ab bc ca ) c = ABC BC + ABC BC

F = ab c + b c a + c a b + abc = ( A + B + C ) BC + ABC ( B + C )
F=A B C = A BC + BC + ABC

29. Sol. = BC ( A + 1) + ABC = BC + ABC


= B (C + AC ) = B (C + A)
A
B Z
33. (a)
C

A
t=0
B x
f
y

B
20 ns

Y 34. (a, c)
40 ns
F(A, B) = A + B
C As ‘0’ and ‘1’ are available,
Z
20 ns 60 ns F(0, B) = A + B = 0 + B
= B (NOT)
Z is ‘1’ for 40 n-sec.
F( A + B) = A + B = A + B
30. (a)
F( A + B ) = A + B (OR)
A with the combination of OR and NOT, NOR gate
A
G1 can be implemented. Since NOR gate is
B
B
G2 Y universal logic gate, so all the functions can be
C CD implemented. So, correct option is (a, c).
D
26 Electronics Engineering Digital Electronics

Answers
EE Logic G ates

1. (a) 2. (d) 3. (b) 4. (b) 5. (c) 6. (a) 7. (c) 8. (c)

9. (c) 10. (d) 11. (d)

Solutions
EE Logic G ates

1. (a) 6. (a)

Y = (A B) C = A B+C x x y

= A B + C = AB + A B + C 1 0 1

2. (d) 0 1 1

A B Y
B
Y = x x+x x =1
0 0 1 A 0 1

0 1 X 0 1 1 7. (c)
1 0 X 1 1 Truth table XOR gate is,
1 1 X A B Y
0 0 0
Y = AB = A+ B NOR GATE 0 1 1
1 0 1
Y = AB + A B EX-NOR GATE
1 1 0

3. (b) So, from the XOR gate truth table it is clear that
the bulb can be turned ON and also can be
Y = (P Q) ( R S )
turned OFF by any one of the switches
Q ( A B ) = (A + B) irrespective of the state of the other switch.

Y = P Q+R S 8. (c)
AB
C 00 01 11 10
4. (b)
0 1 1
Y = A B C D from the given diagram. We
know that sum of any number of bits is XOR of
1 1 1
all bits.
So, S=A B C D Y
F = AC + BC
S=Y Y
So, option (c) is correct.
S = either zero even because LSB is zero (always)
9. (c)
5. (c)
P
NOR and NAND are designated as universal
logic gates because using any one of them we Q Z
can implement all the logic gates.
R
GATE Previous Years Solved Paper 27

11. (d)
Z = PQ Q Q R = P Q + Q + Q R
The Boolean expression for the output of the
= PQ + Q + QR = Q ( P + 1) + QR digital circuit is shown below.
= Q + QR X

= (Q + Q) (Q + R) = Q + R Y

10. (d) Z = XY + XY

A AB
B
Y
The above expression is a XOR gate.
C
D CD

AB
Y = AB + CD
CD
4 Combinational Circuits

ELECTRO NICS EN GINEERIN G Q.5 Without any additional circuitry, an 8 : 1 MUX


can be used to obtain
(GATE Previous Years Solved Papers)
(a) some but not all Boolean functions of
Q.1 The minimal function that can detect a 3 variables.
‘divisible by 3’ 8421 BCD code digit (b) all functions of 3 variables but none of
(representation is D7 D4 D2 D1) is given by 4 variables.
(a) D8 D1 + D4 D2 + D8 D2 D1 (c) all functions of 3 variables and some but
(b) D8 D1 + D4 D2 D1 + D4 D2 D1 + D8 D4 D2 D1 not all of 4 variables.
(c) D8 D1 + D4 D2 + D8 D4 D2 D1 (d) all functions of 4 variables.
(d) D4 D2 D1 + D4 D2 D1 + D8 D4 D2 D1 [EC-2003 : 1 Mark]

[EC-1990 : 1 Mark] Q.6 The circuit shown in the figure converts.


Inputs
Q.2 The logic realized by the circuit shown in figure
MBS
is

C I0
I1
4×1
I2 MUX F

C I3
S1 S 2

A B MBS
(a) F = A C (b) F = A C Output

(c) F = B C (d) F = B C (a) BCD to Binary code


[EC-1992 : 1 Mark] (b) Binary to Excess-3 code
Q.3 A 2-bit binary multiplier can be implemented (c) Excess-3 to Gray code
using (d) Gray to Binary code
(a) 2-input ANDs only [EC-2003 : 2 Marks]
(b) 2-input X-ORs and 4-input AND gates only
Q.7 The circuit shown in the figure has 4 boxes each
(c) Two (2) input NORs and one XNOR gate
described by inputs P, Q, R and outputs Y, Z
(d) XOR gates and shift registers
[EC-1997 : 1 Mark] with Y = P Q R, Z = R Q + P R + Q P .
Q
Q.4 For a binary half-subtractor having two inputs A
and B, the correct set of logical expressions for
the outputs D (= A minus B) and X(= borrow) are
P
(a) D = AB + A B, X = A B
P Q P Q P Q P Q
(b) D = A B + A B + A B , X = A B Z R Z R Z R Z R
Y Y Y Y
(c) D = A B + A B, X = A B
(d) D = A B + A B, X = A B Output
[EC-1999 : 2 Marks]
GATE Previous Years Solved Paper 29

The circuit acts as a


P I3
(a) 4-bit adder giving P + Q.
P
(b) 4-bit adder giving P – Q. I2
Q 4-to-1
(c) 4-bit adder giving Q – P. Z
P I1 MUX
(d) 4-bit adder giving P + Q + R. P
I0
[EC-2003 : 2 Marks] Q

Q.8 The minimum number of 2-to-1 multiplexers


R S
required to realize a 4-to-1 multiplexer is
(a) 1 (b) 2 The output Z can be represented by
(c) 3 (d) 4 (a) PQ + PQ S + Q R S
[EC-2004 : 2 Marks] (b) PQ + PQ R + PQ S
Q.9 The Boolean function ‘f ’ implemented in the (c) PQ R + PQ R + PQRS + Q R S
figure using two input multiplexers is
(d) PQ R + PQ R S + PQ R S + Q R S
C 0 0 0
[EC-2008 : 2 Marks]
f
Statement for Linked Answer Questions (12 and 13):
C 1 A 1
Two products are sold from a vending machine, which
has two push buttons P1 and P2. When a button is
B E
pressed, the price of the corresponding product is
(a) A BC + A BC (b) ABC + A BC displayed in a 7-segment display.
If no buttons are pressed, ‘0’ is displayed,
(c) A BC + A BC (d) ABC + ABC
signifying ‘Rs. 0’.
[EC-2005 : 1 Mark] If only P1 is pressed, ‘2’ is displayed, signifying
Q.10 In the following circuit, X is given by ‘Rs. 2’.
If only P2 is pressed, ‘5’ is displayed, signifying
0 I0 0 I0 ‘Rs. 5’.
1 I1 1 I1 If both P1 and P2 are pressed, ‘E’ is displayed,
4-to-1 4-to-1
1 I2 MUX 1 I2 MUX X signifying ‘error’.
0 I3 0 I3 The names of the segments in the 7-segment display,
S1 S0 S1 S0 and the glow of the display for ‘0’, ‘2’, ‘5’, and ‘E’, are
shown below.
A B C
a 0 2 5 E
f g b
(a) X = A BC + A BC + A BC + ABC
e c
(b) X = A BC + A BC + A BC + A BC d

(c) X = AB + BC + AC Consider :

(d) X = A B + BC + AC (i) push button pressed/not pressed in


equivalent to logic 1/0 respectively.
[EC-2007 : 2 Marks]
(ii) a segment glowing/not glowing in the
Q.11 For the circuit shown in the following figure, display is equivalent to logic 1/0
I0 – I3 are inputs to the 4 : 1 multiplexer. R(MSB) respectively.
and S are control bits.
30 Electronics Engineering Digital Electronics

Q.12 If segments a to ‘g’ are considered as functions


I0
of P1 and P2, then which of the following is
I1
correct? 4×1
I2 MUX Y F
(a) g = P1 + P2 , d = c + e
I3
(b) g = P1 + P2, d = c + e S1 S0

(c) g = P1 + P2 , e = b + c P Q
(d) g = P1 + P2, e = b + c (a) F = AND (P, Q) (b) F = OR (P, Q)
[EC-2009 : 2 Marks] (c) F = XNOR (P, Q) (d) F = XOR (P, Q)
Q.13 What are the minimum numbers of NOT gates [EC-2011 : 1 Mark]
and 2-input OR gates required to design the Q.17 The output Y of a 2-bit comparator is logic 1
logic of the driver for this 7-segment display? whenever the 2-bit input A is greater than the
(a) 3 NOT and 4 OR (b) 2 NOT and 4 OR 2-bit input B. The number of combination for
(c) 1 NOT and 3 OR (d) 2 NOT and 3 OR which the output is logic 1, is
[EC-2009 : 2 Marks] (a) 4 (b) 6
(c) 8 (d) 10
Q.14 What are the minimum number of 2-to-1 [EC-2012 : 1 Mark]
multiplexers required to generate a 2-input AND
gate and a 2-input EX-OR gate? Q.18 In a half-subtractor circuit with X and Y as
inputs, the Borrow (M) and Difference (N = X – Y)
(a) 1 and 2 (b) 1 and 3
are given by
(c) 1 and 1 (d) 2 and 2
(a) M = X Y, N = XY
[EC-2009 : 2 Marks]
(b) M = XY, N = X Y
Q.15 The Boolean function realized by the logic (c) M = X Y , N = X Y
circuit shown is
(d) M = X Y , N = X Y

C I0 [EC-2014 : 1 Mark]

D
I1
Q.19 Consider the multiplexer based logic circuit
4×1
F(A, B, C, D) shown in the figure.
I2 MUX
W 0
I3
S1 S0 MUX 0

1 MUX F
A B
1
S1
(a) F = m(0, 1, 3, 5, 9, 10, 14)
(b) F = m(2, 3, 5, 7, 8, 12, 13) S2

(c) F = m(1, 2, 4, 5, 11, 14, 15) Which one of the following Boolean functions
(d) F = m(2, 3, 5, 7, 8, 9, 12) is realized by the circuit?
[EC-2010 : 2 Marks] (a) F = W S1 S2
(b) F = WS1 + WS2 + S1S2
Q.16 The logic function implemented by the circuit
below is (ground implies a logic ‘0’) (c) F = W + S1 + S2
(d) F = W S1 S2 [EC-2014 : 1 Mark]
GATE Previous Years Solved Paper 31

Q.20 If X and Y are inputs and the Difference


(d) X I0
(D = X – Y) and the Borrow (B) are the outputs,
2:1
which one of the following diagrams B
MUX
implements a half subtractor?
I1

(a) Y I0 S
Y
2:1 S
D
MUX X I0

I1 2:1
D
S MUX
X I1
S
Y I0
[EC-2014 : 2 Marks]
2:1
B
MUX Q.21 In the circuit shown, W and Y are MSBs of the
I1 control inputs. The output MSBs is given by
4 : 1 MUX 4 : 1 MUX
I0 I0

(b) X I0 I1 I1
2:1 V CC Q Q F
D I2 I2
MUX

I1 I3 I3
S
Y W X Y Z
S
(a) F = W X + W X + Y Z
X I0

2:1
(b) F = W X + W X + Y Z
B
MUX
(c) F = W X Y + W X Y
I1
(d) F = ( W + X ) Y Z [EC-2014 : 2 Marks]

Q.22 An 8-to-1 multiplexer is used to implement a


(c) Y I0
logical function Y as shown in the figure. The
output
2:1
B
MUX 0 I0

I1 D I1

S 0 I2
X D I3
S 0 I4 Y
Y I0 0 I5

2:1 1 I6
D
MUX 0 I7
S2 S1 S0
I1

A B C
32 Electronics Engineering Digital Electronics

(a) Y = A BC + AC D Q.25 Identify the circuit below,

(b) Y = A BC + A B D O P0 IP0

X2 OP1 IP1 Y2
(c) Y = A BC + AC D
OP2 IP2

(d) Y = A B D + A BC X1
3:8 OP3 IP3 8:3
Y1
Decoder OP4 IP4 Decoder
[EC-2014 : 2 Marks] OP5 IP5
X0 OP6 IP6 Y0
Q.23 A 16-bit ripple carry adder is realized using 16
OP7 IP7
identical full adders (FA) as shown in the figure.
The carry-propagation delay of each FA is 12 ns (a) Binary to Gray code converter
and the sum-propagation delay of each FA is
(b) Binary to XS3 converter
15 ns. The worst case delay (in ns) of this 16-bit
(c) Gray to Binary converter
adder will be _______ .
(d) XS3 to Binary converter
A0 B0 A 1 B1 A14 B14 A 15 B15
[EC-2016 : 2 Marks]
C0 C1 C14 C15
FA0 FA1 FA14 FA 15 Q.26 A 4 : 1 multiplexer is to be used for generating
the output carry of a full adder. A and B are the
S0 S1 S14 S15 bits to be added while Cin is the input carry and
[EC-2014 : 2 Marks] Cout is the output carry. A and B are to be used
as the select bits with A being the more
Q.24 A 1-to-8 demultiplexer with data input Din,
significant select bit.
address inputs S0, S1, S2 (with S0 as the LSB)
I0
and Y0 to Y7 as the eight demultiplexed output,
I1
is to be designed using two 2-to-4 decoders (with 4:1
I2 Cout
MUX
enable input E and address inputs A0 and A1)
I3
as shown in the figure Din, S0, S1 and S2 are to be
S1 S0
connected to P, Q, R and S, but not necessarily
in this order. The respective input connections
A B
to P, Q, R and S terminals should be
Which one of the following statements correctly
P
1 Y0 Y0
describes the choice of signals to be connected
1E
Q to the inputs I0, I1, I2 and I3 so that the output is
2-to-4 1 Y
1 Y1
Decoder Cout?
R 1 A0 1 Y 2 Y2
(a) I0 = 0, I1 = Cin, I2 = Cin and I3 = 1
S 1 A1 1 Y3 Y3
(b) I0 = 1, I1 = Cin, I2 = Cin and I3 = 1
(c) I0 = Cin, I1 = 0, I2 = 1 and I3 = Cin
2E 2 Y0 Y4
2-to-4
(d) I0 = 0, I1 = Cin, I2 = 1 and I3 = Cin
2 Y1 Y5
Decoder [EC-2016 : 1 Mark]
2 A0 2 Y2 Y6

2 A1 2 Y3 Y7 Q.27 For the circuit shown in the figure, the delays of


NOR gates, multiplexers and inverters are 2 ns,
(a) S2, Din, S0, S1 (b) S1, Din, S0, S2 1.5 ns and 1 ns, respectively. If all the inputs P,
(c) Din, S0, S1, S2 (d) Din, S2, S0, S1 Q, R, S and T are applied at the same time instant,
the maximum propagation delay (in ns) of the
[EC-2015 : 2 Marks]
circuit is _______ .
GATE Previous Years Solved Paper 33

P The Boolean function ‘F’ implemented is


(a) PQ R + PQ R + PQ R
Q
0 0
(b) ( P + Q + R ) ( P + Q + R ) ( P + Q + R )
R
MUX MUX Y (c) PQ R + PQ R + PQ R

S 1 1 (d) ( P + Q + R ) ( P + Q + R ) ( P + Q + R )
S0 S0
[EC-2017 : 2 Marks]
T
Q.30 Figure I shows a 4-bit ripple carry adder realized
[EC-2016 : 2 Marks]
using full adders and Figure II shows the circuit
Q.28 Consider the circuit shown in the figure. of a full-adder (FA). The propagation delay of
the XOR, AND and OR gates in Figure II are
Y 0 20 ns, 15 ns and 10 ns, respectively. Assume all
the inputs to the 4-bit adder are initially
MUX 0
reset to 0.
0 1 MUX F Y3 X3 Y2 X 2 Y1 X1 Y 0 X0

1
Z3 Z2 Z1 Z0
X Z4 FA FA FA FA

Z
S3 S2 S1 S0
The Boolean expression F implemented by the Fig. (I)

circuit is At t = 0, the inputs to the 4-bit adder are changed


(a) X Y Z + X Y + Y Z to X3 X2 X1 X0 = 1100, Y3 Y2 Y1 Y0 = 0100 and
Z0 = 1. The output of the ripple carry adder will
(b) X Y Z + X Z + Y Z
be stable at t (in ns) = _______ .
(c) X Y Z + X Y + Y Z Xn

(d) X Y Z + X Z + Y Z Yn Sn

You might also like